EEVblog Electronics Community Forum

General => General Technical Chat => Topic started by: EEVblog on November 05, 2018, 11:58:51 pm

Title: Does Kirchhoff's Law Hold? Disagreeing with a Master
Post by: EEVblog on November 05, 2018, 11:58:51 pm
I'm putting this here instead of in the other blog section, as it's an important technical discussion.
Mehdi claims Walter Lewin is wrong about his infamous KVL violation video, and I've always felt the same but have never investigated myself.

https://www.youtube.com/watch?v=0TTEFF0D8SA (https://www.youtube.com/watch?v=0TTEFF0D8SA)
Title: Re: Does Kirchhoff's Law Hold? Disagreeing with a Master
Post by: ataradov on November 06, 2018, 12:27:06 am
Wow. I was not even aware that there is such a controversy over Kirchhoff's law.

I though Kirchhoff's laws were derivable from Faraday's law and Maxwell equations in general. But quick search shows that it is not very easy.
Title: Re: Does Kirchhoff's Law Hold? Disagreeing with a Master
Post by: bson on November 06, 2018, 01:47:49 am
KVL only applies in static fields.  KCL still applies in fields under flux.

A meter used to measure across points in an induction loop in field flux becomes part of the loop, and the voltage will depend on where the meter and leads are placed relative to the loop.  Move it from one side to the other and the voltage reverses.

The current induced in a loop is fixed.  The voltage depends on the impedance - the higher the impedance, the higher the voltage between any two points in the loop.  This is actually normal inductor behavior if you think about it.
Title: Re: Does Kirchhoff's Law Hold? Disagreeing with a Master
Post by: beenosam on November 06, 2018, 03:44:33 am
The worst part of this is the way Lewin is acting on YouTube on his own video. He's being pretty childish and refuses to discuss it at all.
Title: Re: Does Kirchhoff's Law Hold? Disagreeing with a Master
Post by: chris_leyson on November 06, 2018, 05:00:25 am
When I first watched Walter Lewins video the first thing that struck me was the experimental setup and that the test leads were also part of the experiment. Mehdi's got it right an you can't fault his experimental setup and explanation. KVL still holds true in an varying magnetic field if you draw the circuit correctly by including the test leads. I think Walter Lewin knows this and is just being controversial to get students to think about the problem. Mehdi's experimental setup is nicely done and very easy to replicate and hopefully it will teach EEs a little about good wiring practice when there are stray magnetic fields.
Title: Re: Does Kirchhoff's Law Hold? Disagreeing with a Master
Post by: chris_leyson on November 06, 2018, 05:24:48 am
Mehdi Sadaghdar's lecture and experiment is also a good way of showing how you should attach twisted pair into a circuit to make a measurement, think twice before you measure.
As an aside, I wonder how the electrostatic "dual" of the experiment would be arranged ?
Title: Re: Does Kirchhoff's Law Hold? Disagreeing with a Master
Post by: HackedFridgeMagnet on November 06, 2018, 05:42:37 am
The nub of the discussion seems to be do the sum of the voltages around a circuit always add to zero.

You have to admit Maxwells-Faradays law seem to be on Walter Lewins side.

Also
From Wikipedia
Quote
KVL is based on the assumption that there is no fluctuating magnetic field linking the closed loop. This is not a safe assumption for high-frequency (short-wavelength) AC circuits.[2] In the presence of a changing magnetic field the electric field is not a conservative vector field. Therefore, the electric field cannot be the gradient of any potential. That is to say, the line integral of the electric field around the loop is not zero, directly contradicting KVL.

It is often possible to improve the applicability of KVL by considering "parasitic inductances" (including mutual inductances) distributed along the conductors.[2] These are treated as imaginary circuit elements that produce a voltage drop equal to the rate-of-change of the flux.
But then Electrobooms arguments are convincing too.

Maybe the demonstration by Walter Lewin doesn't really show what he is trying to show? the loop seems to include the path to the scope.
How would you measure EMF in a closed loop anyway?

IDK but I really would like to understand this better.
Title: Re: Does Kirchhoff's Law Hold? Disagreeing with a Master
Post by: ajb on November 06, 2018, 06:11:34 am
From Wikipedia
Quote
It is often possible to improve the applicability of KVL by considering "parasitic inductances" (including mutual inductances) distributed along the conductors.

Et voila.

The crux of the matter is that Lewin didn't account for the fact that KCL require the EE equivalent of spherical cows.  In the real world wires have resistance and inductance and mutual capacitance, and are not the simple indications of equivalence that they are assumed to be in KCL.  So for the math to be accurate you have to consider them as lots of little spherical cows all stuck together in different ways.

If you have a whole lot of little spherical cows you get to call that finite element analysis.
Title: Re: Does Kirchhoff's Law Hold? Disagreeing with a Master
Post by: chris_leyson on November 06, 2018, 06:36:41 am
Thanks HackedFridgeMagnet, "KVL is based on the assumption that there is no fluctuating magnetic field linking the closed loop" which is true and KVL only holds up for a static field. However, if you take into account all of the parasitic and stray circuit elements then KVL still holds up if you have a vaying magnetic field, however, you've got a different circuit than what you started with. Both Lewin and Electroboom are right, all depends on whether you model in the parasitics. KVL still holds for varying magnetic field if and only if you model in the parasitic circuit elements
Title: Re: Does Kirchhoff's Law Hold? Disagreeing with a Master
Post by: HackedFridgeMagnet on November 06, 2018, 07:25:55 am
Thanks HackedFridgeMagnet, "KVL is based on the assumption that there is no fluctuating magnetic field linking the closed loop" which is true and KVL only holds up for a static field. However, if you take into account all of the parasitic and stray circuit elements then KVL still holds up if you have a vaying magnetic field, however, you've got a different circuit than what you started with. Both Lewin and Electroboom are right, all depends on whether you model in the parasitics. KVL still holds for varying magnetic field if and only if you model in the parasitic circuit elements

Hmm.. it seems to say it differently below.
Apparently it needs to be in a conservative vector field but I can't even see that the field within a resistor is a conservative vector field because it is dissipating energy.
Quote
KVL is based on the assumption that there is no fluctuating magnetic field linking the closed loop. This is not a safe assumption for high-frequency (short-wavelength) AC circuits.[2] In the presence of a changing magnetic field the electric field is not a conservative vector field. Therefore, the electric field cannot be the gradient of any potential. That is to say, the line integral of the electric field around the loop is not zero, directly contradicting KVL.

It is often possible to improve the applicability of KVL by considering "parasitic inductances" (including mutual inductances) distributed along the conductors.[2] These are treated as imaginary circuit elements that produce a voltage drop equal to the rate-of-change of the flux.

So is the line integral of the electric field around the loop zero in all cases (KVL) or not?
If it is non zero (Maxwell Faraday) then how would you measure it? Because measuring it with a scope would seem to prove KVL.
Of course there is also the unlikely scenario where Wikipedia is wrong.  ;)
Title: Re: Does Kirchhoff's Law Hold? Disagreeing with a Master
Post by: riyadh144 on November 06, 2018, 09:49:33 am
I have thought about this question way too much since high school, but I always convinced myself that KVL doesn't hold under external EMF.

BUT Walter's setup is really bad probing.
I will be working on getting a good experiment.
Title: Re: Does Kirchhoff's Law Hold? Disagreeing with a Master
Post by: EEVblog on November 06, 2018, 10:08:51 am
Maybe the demonstration by Walter Lewin doesn't really show what he is trying to show?

That's possible. He could be right in theory, but may be using a poorly thought out experiment to try and show it.
And maybe it's deliberately a bit dodgy because it's not easy to demonstrate?, and maybe he knows that?

Kinda reminds me of my current flowing through a capacitor video. That was bit of a troll on my part in order to show a "controversial" way of thinking about theoretic current and maxwells law. The old theoretical physicist vs practical engineer viewpoint discrepancy.
Title: Re: Does Kirchhoff's Law Hold? Disagreeing with a Master
Post by: Berni on November 06, 2018, 10:48:52 am
I would also say that both are right in a certain way.

Kirchoffs law doesn't hold in a magnetic field if you don't model the effect of the field on the inductance. But does work if you model the parasitics of the wire as instructors.

If you have taken the DC voltage source in his lecture example and replaced it with a 1GHz AC source then Kirchoffs law again would not hold because your schematic is ignoring parasitic effects that the real circuit can't simply ignore.

A similar argument to this is how to calculate kinetic energy in physics. Its commonly used that kinetic energy E = (m*v^2)/2 and it turns out this works perfectly in practical experiments. However due to relativistic effects things appear heavier as they get closer to the speed of light, this is sort of a 'parasitic' effect that we are ignoring in that formula because the speeds we normally work with are so low that the effect is practically zero and everything works great. But when you get to these higher speeds this effect starts to contribute more and more to the total energy and then yes that equation is broken. But its not broken because the equation is wrong. Its just ignoring a insignificant detail of the physics model that turned out to become significant in this particular set of circumstances.

But i do have to say that Dr. Lewin explains it a slightly odd direction that is kinda misleading. Its not that this special case with magnetic fields breaks Kichhoffs law, but its just that the mathematical model of the cirucit ignores magnetic effects that should not be ignored in this case. Ideal wires with 0 voltage drop would need to also be 0 units long in the physical world so that they don't have inductance. If all the wires are 0 units long this means the physical circumference of the circuit must also be 0 (Resistors are considered to be 0 length too or they would also be inductors). A circle with 0 circumference must also have a surface area of 0 as such it has 0 magnetic loop area and as such can't get a voltage induced in it no matter how strong or fast changing of a magnetic field you place it in. With that that the voltage on both resistors would calculate to be 0V as you would get trough the analysts of the circuit because the circuit contains no voltage sources.

The correct model for his physical circuit would include inductors on all lengths of wire(including probes going to the scope) and all these inductors should have a dot marked on them showing if they go clockwise or counterclockwise along the magnetic field. Additionally arrows should be drawn between all of them to show they are all coupling together via a magnetic field and the coupling factor written on each arrow. This circuit now has a voltage source in it, its the voltage source that is powering the inductor representing the solenoid. You could then calculate the voltages at each point of the circuit and you would get results that are very close to what the scope is showing.

This is an excellent paradox to get people thinking. But Dr. Lewin should explain that the problem is a bad circuit model ignoring what should not be ignored rather than the ever so useful Kirchhoffs law being wrong.
Title: Re: Does Kirchhoff's Law Hold? Disagreeing with a Master
Post by: nctnico on November 06, 2018, 11:09:09 am
I would also say that both are right in a certain way.

Kirchoffs law doesn't hold in a magnetic field if you don't model the effect of the field on the inductance. But does work if you model the parasitics of the wire as instructors.

This is an excellent paradox to get people thinking. But Dr. Lewin should explain that the problem is a bad circuit model ignoring what should not be ignored rather than the ever so useful Kirchhoffs law being wrong.
I agree. Especially with the last part. This has been discussed before and it seems Dr. Lewin is just trolling.
Title: Re: Does Kirchhoff's Law Hold? Disagreeing with a Master
Post by: SparkyFX on November 06, 2018, 11:11:30 am
I tend to approach this problem in the following way:
KVL holds true for infinite small dimensioning or as a logical concept - thats when you not happen to use a diagram that includes all inductances and capacitances in the circuit.

For everything else the dimensions and field strength would need to be specified, the logical concept of a circuit designed in a loop does not mean that it physically needs to be a flat loop.

Both are right by themselves, the discrepancy stems from taking the logical concept and converting it directly into a specific physical design.
Title: Re: Does Kirchhoff's Law Hold? Disagreeing with a Master
Post by: madires on November 06, 2018, 11:14:42 am
It's easy to forget that wire is a component too, and also the scope with its probes. If you add a changing magnetic field you cant ignore the wire any longer.
Title: Re: Does Kirchhoff's Law Hold? Disagreeing with a Master
Post by: SparkyFX on November 06, 2018, 11:34:46 am
All logical concepts ask for ideal elements, this means that side effects can be cancelled out somehow and you are left with the concept itself as the effect.
Ideal switches, ideal wires, ideal ... do obviously not exist, which makes it practically the base of the whole electronics trade to put concepts in physical reality without unwanted/inacceptable side effects (errors or mishaps) by design.

But don´t go ad hominem because of that, as it is just the kind of discrepancy that is thought provoking in a good way, presented as a right/wrong approach although the concept left the realm it was meant to be used in.
Title: Re: Does Kirchhoff's Law Hold? Disagreeing with a Master
Post by: Berni on November 06, 2018, 12:09:57 pm
I would also say that both are right in a certain way.

Kirchoffs law doesn't hold in a magnetic field if you don't model the effect of the field on the inductance. But does work if you model the parasitics of the wire as instructors.

This is an excellent paradox to get people thinking. But Dr. Lewin should explain that the problem is a bad circuit model ignoring what should not be ignored rather than the ever so useful Kirchhoffs law being wrong.
I agree. Especially with the last part. This has been discussed before and it seems Dr. Lewin is just trolling.

Yeah i get the feeling that he explains it in a way that doesn't give the full picture on purpose. Using it as sort of a way to see what the student are thinking when they try to explain it and perhaps find the rare bright student who figures out the real reason behind this effect.

But it could also be just a case of being exposed to too much theory and too little practical electronics work. I noticed this with some teachers that they get a different perception of a certain subject due to approaching it from pure theory and sort of settle in to certain specific ways of mathematical problem solving that they personally find really neat.

One of my favorite electronics teachers turned out to be a old guy who has been repairing TVs and other equipment for many years. He had an incredible depth of practical knowledge of useful electronic circuits and knew exactly how they work and how to design them. I'm really not saying theory is a waste of time but instead that every bit of theory should be connected to something physical as well, otherwise you just end up going down a rabbit hole full of math describing mythical ideal components. Once enough math abstractions are stacked on top of each other you can get so far away from the physical world that it can be hard to find your way back to what a voltmeter is showing on the bench (Especially when the teacher in question only ever touched a physical voltmeter once a year).
Title: Re: Does Kirchhoff's Law Hold? Disagreeing with a Master
Post by: MiDi on November 06, 2018, 03:13:57 pm
I am wondering if there is one example KVL does not apply when every "parasitics" and "outer elements" are modeled?

Someone suggested superconductor without giving an example.
So maybe there is an edge case/singularity where KVL does not apply  :-//
Title: Re: Does Kirchhoff's Law Hold? Disagreeing with a Master
Post by: b_force on November 06, 2018, 03:28:39 pm
I find it a bit nit picky.
Newtons laws also don't work everywhere, that doesn't make them false all of a sudden?
They have their limits, so has Kirchhoffs laws.
Nothing new, can we go on now with the real stuff?
Title: Re: Does Kirchhoff's Law Hold? Disagreeing with a Master
Post by: RoGeorge on November 06, 2018, 03:41:16 pm
https://www.eevblog.com/forum/chat/kirchhoff_s-loop-rule-is-for-the-birds/ (https://www.eevblog.com/forum/chat/kirchhoff_s-loop-rule-is-for-the-birds/)
https://www.eevblog.com/forum/beginners/are-kirchoff-laws-useless/msg920903/#msg920903 (https://www.eevblog.com/forum/beginners/are-kirchoff-laws-useless/msg920903/#msg920903)
https://www.eevblog.com/forum/beginners/walter-lewin-8-02-lect-16-super-demo-(correction)/ (https://www.eevblog.com/forum/beginners/walter-lewin-8-02-lect-16-super-demo-(correction)/)

Internet Rule #0: Search before you post  ;D
Title: Re: Does Kirchhoff's Law Hold? Disagreeing with a Master
Post by: T3sl4co1l on November 06, 2018, 04:04:53 pm
I'll repeat the comment I made on the video (which I'm sure has been utterly buried under a torrent of less extensive comments, or suppressed outright by algorithm):

The way I see it is structural:

He's using a DC circuit in an AC field, and claiming that the DC circuit still holds, while forcing an AC behavior upon it.

As is always the case in proof by contradiction: we have only proven that our premises were wrong.

It's a similar fiction as the conservation of charge vs. energy when connecting charged capacitors together: if energy is conserved, where does it go?  Well, it turns out that you can't simply short capacitors together without taking account of their resistance, or inductance.  Or, more generally, of the loop area between them, which gives rise to both elements.  So the charge is conserved (a more fundamental quantity), and energy is conserved whether or not you've written in a way for it to do so.

It would be more illustrative if he phrased it as a riddle to the student, to figure out where the disconnect is.


You are quite correct that merely adding a transformer, and keeping track of the probe wires in the field, is all that is missing!

As for failings of Kirchoff's laws: radio waves.  One must get ever more particular about where (spatially speaking) one applies them.  The current flowing into the feedpoint of a dipole antenna, for example, does not equate with the current conducted out of the element tips, which is zero.  The disconnect here is concentrating on conduction, while ignoring displacement current.  In effect, equivalent capacitance carries the current into free space.  But more accurately, it's carried into the fields around every point of the antenna.


At their most general (but perhaps least useful), KVL/KCL reduce to a single point only: they are a differential relation, which must be integrated over the space of interest.  (This, of course, is painful to do by hand for all but the simplest arrangements, so we usually have computers divide the space into millions of finite elements and apply the laws to them, for us.  Hence, FEA (finite element analysis) tools.)

This, in turn, drive home another point about schematics: what we draw is an abstraction, a fiction, a model.  The points are connected instantaneously in time and space, with no concept of distance, or the speed of light (namely, that the distance is effectively zero, or the speed of light is infinite).  To build a realistic model, when the speed of light is relevant, we must introduce enough parameters (whether L and C approximations, or real transmission line elements) to match this.

Tim
Title: Re: Does Kirchhoff's Law Hold? Disagreeing with a Master
Post by: Berni on November 06, 2018, 05:22:40 pm
Just for the fun of it, here is a LT Spice simulation recreating this cirucit.

The graph shows that the actual voltage between those two points is an average between the two scope readings(Since this cancels out the voltage added and subtracted by the test leads going one or the other way)

EDIT: Do note that this is not a accurate simulation as it still ignores stray capacitance and assumes a perfect coupling between all coils. In reality the coupling between the wires would be slightly less than 1 due to the wires not being able to exist in the same physical location and the coupling to the solenoid coil in the middle should be even less due to it being far away from the current path of all the other wires.

(https://www.eevblog.com/forum/chat/does-kirchhoffs-law-hold-disagreeing-with-a-master/?action=dlattach;attach=565039;image)
Title: Re: Does Kirchhoff's Law Hold? Disagreeing with a Master
Post by: IanMacdonald on November 06, 2018, 06:02:20 pm
The reason for the apparent difference in readings is that the magnetic field also induces a voltage into the meter or scope leads. Kirchhoff's Law still holds if you take this into account.

As a relevant point of interest, you cannot have a half-turn transformer winding. It is always an integer number of turns. You have to connect the ends of your 'half turn' winding together somehow, for current to flow. Making the wire longer, as in including the test leads of a meter for example, does not alter the fact that the circuit still completes the same magnetic path as a complete turn would do. The longer wire intercepts a more diffuse magnetic field but over a longer distance, and the induced EMF is the same as if it were a tightly wound turn. 

In this case a fail, but unless people challenge established science there will never be any more breakthroughs. The problem these days is that science has become more like a religion whose tenets MUST NOT be questioned. This is just as bad as allowing complete pseudoscience.
Title: Re: Does Kirchhoff's Law Hold? Disagreeing with a Master
Post by: SiliconWizard on November 06, 2018, 06:44:12 pm
Interesting question, but I find Dr. Lewin's experimental setup kind of a disgrace for an MIT professor. Anecdotally, the MIT revoked his "professor emeritus" title, but apparently not based on anything related to his scientific merits. Anyway.

I think he hasn't proved anything here. Is there anything else than induced voltages in the probing wires and scope probe in play?
Devising a completely neutral probing setup seems pretty difficult to do, and you won't achieve it with a couple flying wires (however twisted they may be) and basic scope probes.

That said, even if we consider a perfect setup and a real discrepancy, my take here would be that KVL could still perfectly apply. We would just have to consider the loop having extra voltage sources from any induced voltage. That wouldn't defeat it, but make us consider that our circuit model is incomplete.

Just my 2 cents.


Title: Re: Does Kirchhoff's Law Hold? Disagreeing with a Master
Post by: In Vacuo Veritas on November 06, 2018, 07:56:51 pm
A lot of older academics become kooks or cranks because they have been sniffing their own farts so long and no one dares to say they stink. (Because that would jeopardize their own path towards their auto-fart-sniffing career)

Old people should be viewed with suspicion, especially academics. It's sad but true. Look at the ideas that come out of 20-something brains versus the ossified crap and ego-stroking narcissism from older people.
Title: Re: Does Kirchhoff's Law Hold? Disagreeing with a Master
Post by: ataradov on November 06, 2018, 08:57:06 pm
It is not like he taught advanced physics. From what I've seen his lectures were more like physically themed performances rather than actual lectures. This was fine for the first year EEs that have never seen physics before.
Title: Re: Does Kirchhoff's Law Hold? Disagreeing with a Master
Post by: T3sl4co1l on November 06, 2018, 09:40:03 pm
That said, even if we consider a perfect setup and a real discrepancy, my take here would be that KVL could still perfectly apply. We would just have to consider the loop having extra voltage sources from any induced voltage. That wouldn't defeat it, but make us consider that our circuit model is incomplete.

Simply put:

You don't get to pick and choose when you follow a law and when you don't.

This is physics, not politics~~

Tim
Title: Re: Does Kirchhoff's Law Hold? Disagreeing with a Master
Post by: SiliconWizard on November 07, 2018, 02:41:25 am
That said, even if we consider a perfect setup and a real discrepancy, my take here would be that KVL could still perfectly apply. We would just have to consider the loop having extra voltage sources from any induced voltage. That wouldn't defeat it, but make us consider that our circuit model is incomplete.

Simply put:

You don't get to pick and choose when you follow a law and when you don't.

This is physics, not politics~~

Tim

Not sure I got your point, nor that you got mine.

I've yet to agree with KVL not being met, until I get a consistent and formal proof of that.

Any electrical circuit made of physical, non-ideal components and non-zero length, non-zero impedance connections between them will get inductive and capacitive coupling with its surroundings. If you devise the real, physical circuit that it actually is, including its surrounding environment, you get equivalents of transformers and capacitors added to the ideal circuit. Taking those into account makes up the real circuit IMO, and KVL is most likely met in any case. I'm saying "most likely" because this is what makes the most sense to me, but as others here, I would of course consider a different view as long as it's rigorously proven.

Not considering those "parasitics" (in the sense that they are not part of our idea of the ideal circuit we designed) is ignoring basic laws of physics. Still don't get how KVL would not apply, until I get a formal explanation and a rigorous experiment that shows that the observed discrepancies are not due to basic inductive and capacitive coupling.
Title: Re: Does Kirchhoff's Law Hold? Disagreeing with a Master
Post by: ajb on November 07, 2018, 02:54:37 am
I have a hard time believing that Lewin is "just trolling" or trying to get students to think as some have suggested, after reading his responses to Mehdi and others on his original video.  I think this link will work, if not just look for ElectroBOOM's comment on the video, it should be near the top: https://goo.gl/JsKHb8.

If Lewin has a more subtle point he's trying to make, he's doing a good job of hiding it.

[EDIT: link shortened to avoid the automatic Youtube embed]
Title: Re: Does Kirchhoff's Law Hold? Disagreeing with a Master
Post by: T3sl4co1l on November 07, 2018, 04:58:30 am
Not sure I got your point, nor that you got mine.

I've yet to agree with KVL not being met, until I get a consistent and formal proof of that.

Any electrical circuit made of physical, non-ideal components and non-zero length, non-zero impedance connections between them will get inductive and capacitive coupling with its surroundings. If you devise the real, physical circuit that it actually is...

Precisely -- you can't construct a supposed circuit (following one law), then probe it with another circuit (following a different law).  While you can find such corruption in some domains, physics is a domain where this is strictly prohibited. :)

Tim
Title: Re: Does Kirchhoff's Law Hold? Disagreeing with a Master
Post by: RoGeorge on November 07, 2018, 05:25:14 am
https://www.youtube.com/watch?v=b7i2uMx7gHo (https://www.youtube.com/watch?v=b7i2uMx7gHo)
Title: Re: Does Kirchhoff's Law Hold? Disagreeing with a Master
Post by: SparkyFX on November 07, 2018, 06:42:54 am
physics is a domain where this is strictly prohibited. :)

Well, it is in violation of the law!  :-DD
Title: Re: Does Kirchhoff's Law Hold? Disagreeing with a Master
Post by: ANTALIFE on November 07, 2018, 07:30:10 am
Neato, I would imagine you need to remove the effects of the wires coming back to the scope to get a clearer picture. The only way I can think of is by using something like an LED. Would need to do a bunch of tests to figure out the LED intensity as a function of peak EMF induced voltage
Title: Re: Does Kirchhoff's Law Hold? Disagreeing with a Master
Post by: RoGeorge on November 07, 2018, 08:30:35 am
Just to be clear, the Professor is correct.

To understand why in the intuitive way, remember what is voltage: By definition, voltage between points A and B is the work necessary to move a unit of charge from A to B.

1. In a constant (conservative) field, it does NOT matter the path I choose to walk my charge between A and B. It will cost me the same amount of energy, no matter how straight or how twisted my way from A to B was. Imagine dropping a ball from a mountain. No matter the path of the ball from top to the bottom, the ball will gain the same energy. All it matters is the height between the top and the bottom. In electric engineering language we say, it does NOT matter how I twist or coil the leads of my voltmeter, the measured voltage will be the same.

2. In a variable (non-conservative) field, it DOES matter the path I choose to walk between A and B. Imagine the same ball and the same mountain from case 1, except this time, while the ball is going downhill, somebody is tweaking the knob of "gravity". Now, the final energy of the ball will vary not only with the height between top and bottom, but also with how much and when the "gravity knob" was adjusted, so it is not the same any more if the ball take a shorter or a longer path.
 
In electric engineering terms we say, it DOES matter how I twist or coil the leads of my voltmeter.

For case 2 we say "in a variable field a voltage is induced in the voltmeter's leads", or "the leads act as voltage sources, too", or whatever, which is nothing more than (a wrong way of) saying that "the path DOES matter when moving charges in a variable (non-conservative) field".
Title: Re: Does Kirchhoff's Law Hold? Disagreeing with a Master
Post by: EEVblog on November 07, 2018, 10:46:41 am
I have a hard time believing that Lewin is "just trolling" or trying to get students to think as some have suggested, after reading his responses to Mehdi and others on his original video.  I think this link will work, if not just look for ElectroBOOM's comment on the video, it should be near the top: https://goo.gl/JsKHb8.

If Lewin has a more subtle point he's trying to make, he's doing a good job of hiding it.

Wow, the spam link responses  :o
I posted a comment, will see if I'll get the spam link reply honor too.
Title: Re: Does Kirchhoff's Law Hold? Disagreeing with a Master
Post by: EEVblog on November 07, 2018, 10:49:35 am
In electric engineering terms we say, it DOES matter how I twist or coil the leads of my voltmeter.

In the real world it can (and in this case demonstrably does) matter how you twist or coil the leads. There is transformer coupling happening which is not shown on your theoretical circuit.
Title: Re: Does Kirchhoff's Law Hold? Disagreeing with a Master
Post by: KL27x on November 07, 2018, 11:05:48 am
Just trying to scroll through all the replies, I have learned something thru osmosis.
...in an induced EMF...
....loop...
...determined by path....
...Kirchoff's law is not valid....
...Special case...
...Thus, Faradays Law is always valid...
...basic physics...
...not going to argue about it....

see lecture 8, 5, 11, 7, 15
Title: Re: Does Kirchhoff's Law Hold? Disagreeing with a Master
Post by: EEVblog on November 07, 2018, 01:32:08 pm
He'll do a video when he gets back from vacation in a week  :popcorn:

(https://i.imgur.com/6bDVfdr.png)
Title: Re: Does Kirchhoff's Law Hold? Disagreeing with a Master
Post by: BrianHG on November 07, 2018, 02:25:57 pm
He'll do a video when he gets back from vacation in a week  :popcorn:

(https://i.imgur.com/6bDVfdr.png)
Thanks for you polite request on our behalf.
Yes, Dr. Lewin's says he will do a video, but, the way he worded his response, it is as if he is clearly refusing to watch or acknowledge ElectroBoom's video in any way.  This may be understandable as it is clearly possible that Dr. Lewin's may have been hit with so many disruptive responses over the years on the subject which he may have won his argument so many times that he is now immune to any new views on what ElectroBoom's has measured and it may unfortunately be a pure one tone response.  I hope he points out truly where ElectroBoom has made an error from his point of view, otherwise, we will most likely see a repeat of his early videos.

Title: Re: Does Kirchhoff's Law Hold? Disagreeing with a Master
Post by: RoGeorge on November 07, 2018, 04:01:56 pm
In electric engineering terms we say, it DOES matter how I twist or coil the leads of my voltmeter.

In the real world it can (and in this case demonstrably does) matter how you twist or coil the leads. There is transformer coupling happening which is not shown on your theoretical circuit.

Good point, let's simplify the circuit. Let's get rid of the voltmeter. We will use an electron to probe the voltage for each half of our loop. Even more, let's look only at the sign of the voltage for each half of the loop.

1. We have our loop of 2 resistors in an increasing magnetic field.
2. Electrons will flow through our loop, let's say clockwise.
3. Let's measure the voltage. By definition, voltage is the work required to move the unit of charge between our measuring points.
4. We don't have a voltmeter, so we grab an electron, and start moving it through each half of the loop, in order to see how much work do we need to accomplish that - or other said to probe the voltage for each half of the loop, left-hand half, and right-hand half.
5. Starting from top, when we circulate our grabbed electron through the left-hand half of the loop, we will need to put some work to move our electron against the flow of all the other electrons in the loop, so negative voltage on the left-hand half.
6. Starting from top, when we circulate our grabbed electron through the right-hand side of the loop, we don't need to put any work, our electron will move by itself, it will go with the flow of all the other electrons, it will generate some work, so positive voltage on the right-hand half.
7. From 5 and 6 we observe the voltage between the same points is once positive, once negative, depending on which half of the loop we measure. The sum of voltages in our closed loop is Vpositive - Vnegative, which is NOT zero. E.g. 3V - (-5V) = 8V.
8. We just seen the sum of voltages for our loop is NOT zero, yet Kirchhoff's Voltage Law predicts it to be ZERO, so Kirchhoff is broken for our setup.
Title: Re: Does Kirchhoff's Law Hold? Disagreeing with a Master
Post by: jancumps on November 07, 2018, 04:23:14 pm
He'll do a video when he gets back from vacation in a week  :popcorn:

(https://i.imgur.com/6bDVfdr.png)
There may be moderation. I see the prof’s answer in the thread, but not your comment.
Title: Re: Does Kirchhoff's Law Hold? Disagreeing with a Master
Post by: wraper on November 07, 2018, 04:33:37 pm
As a relevant point of interest, you cannot have a half-turn transformer winding. It is always an integer number of turns. You have to connect the ends of your 'half turn' winding together somehow, for current to flow. Making the wire longer, as in including the test leads of a meter for example, does not alter the fact that the circuit still completes the same magnetic path as a complete turn would do. The longer wire intercepts a more diffuse magnetic field but over a longer distance, and the induced EMF is the same as if it were a tightly wound turn.
Fractional turns can be done. http://citeseerx.ist.psu.edu/viewdoc/download?doi=10.1.1.187.2764&rep=rep1&type=pdf (http://citeseerx.ist.psu.edu/viewdoc/download?doi=10.1.1.187.2764&rep=rep1&type=pdf)
Title: Re: Does Kirchhoff's Law Hold? Disagreeing with a Master
Post by: rfeecs on November 07, 2018, 06:45:21 pm
Dr. Lewin repeats the same answer in the comments:

Quote
In the case of an induced emf the potentials in a circuit are no longer determined, they depend on the path and thus Kirchhoff's Loop Rule is not valid. Kirchhoff's Loop Rule is a special case of Faraday's Law (namely when phi/dt=0). Thus Faraday's Law is always valid.

The thing is, he's not wrong, and it is fundamental. 

That's the beginning and end of his argument.  He's not interested in "you are not measuring it right".

He even takes this argument so far to the point of saying that Kirchoff's voltage law does not apply to circuits with inductors, because they have a changing magnetic field:
http://web.mit.edu/8.02/www/Spring02/lectures/lecsup4-1.pdf (http://web.mit.edu/8.02/www/Spring02/lectures/lecsup4-1.pdf)

He claims this is where almost all the electrical engineering textbooks are dead wrong.

As Dave says, a case of theoretical physics vs practical engineering approaches.

Title: Re: Does Kirchhoff's Law Hold? Disagreeing with a Master
Post by: KL27x on November 07, 2018, 08:03:21 pm
Quote
For case 2 we say "in a variable field a voltage is induced in the voltmeter's leads", or "the leads act as voltage sources, too", or whatever, which is nothing more than (a wrong way of) saying that "the path DOES matter when moving charges in a variable (non-conservative) field".

He says "two voltmeters connected to the same two points can have two different readings." And he does an experiment where this is not only the case, but these two different reading are repeatable. If this was caused by consistently placing the probe wires (either intentionally or unwittingly) in a different orientation, most of the world, including physicists, would say this is a wrong way of saying that the path DOES matter when moving charges in a variable (non-conservative) field." Especially when he does not include the reason the scopes give the consistently (if you believe the results) different readings. For someone interested in the theory, as an educator or physicist, you would not make this comment (two voltmeters connected to the same two points can have two different readings) in the first place, unless the topic at hand is the potential deficiencies in common voltage measuring devices. You would say the voltage between those two points is in fact quantifiable and repeatable at any given point in time in the experiment.

Professor stretched things. If this were intentionally done to make his students think about the problem, his video would congratulate the student who figures out the omission/trick of how he produced this result. Apparently we shall see if/when the promised video is published.

If you want to play this "path" game, and insist that voltage is not by default the lowest energy path between the two points, then you have to say that the voltage between those point is undefinable, no?
Title: Re: Does Kirchhoff's Law Hold? Disagreeing with a Master
Post by: JimRemington on November 07, 2018, 09:02:46 pm
Quote
If this were intentionally done to make his students think about the problem, his video would congratulate the student who figures out the omission/trick of how he produced this result

I agree completely, but Lewin's subsequent behavior suggests that he managed to fool himself, as well as the unwary student.

I always thought that the failure to take into account the probe placement was the problem, and am pleased to see this excellent video making that case!
Title: Re: Does Kirchhoff's Law Hold? Disagreeing with a Master
Post by: SiliconWizard on November 07, 2018, 09:42:35 pm
Come to re-think of it, and as he's clearly not an idiot, I was then willing to believe that he actually did that on purpose, just to make young students aware of the question: using simplistic models while thinking they hold true in the real world, which is a very common pitfall. This would be all good if he made it clear in the end that it was his intent instead of making it even more confusing, to the point that he even managed to confuse some very experienced engineers, using his position of authority.

Now if he was genuinely trying to instill advanced physics notions in young heads, I think this was a very bad way of doing it from a pedagogical standpoint.
Title: Re: Does Kirchhoff's Law Hold? Disagreeing with a Master
Post by: SiliconWizard on November 07, 2018, 09:58:45 pm
He even takes this argument so far to the point of saying that Kirchoff's voltage law does not apply to circuits with inductors, because they have a changing magnetic field:
http://web.mit.edu/8.02/www/Spring02/lectures/lecsup4-1.pdf (http://web.mit.edu/8.02/www/Spring02/lectures/lecsup4-1.pdf)

Thanks for the reference. This helps making it much clearer what he was meaning - especially on page 3.

From what I got and summing it up quickly, his fundamental point is that we EE use an "adapted" version of Kirchhoff's law which actually describes things correctly but is NOT what Kirchhoff intended.
This is why we don't seem to agree - we actually do agree for all practical matters but for a professor in physics, we're just abusively using the term "KVL". The lecture extract that rfeecs posted explains that a lot more clearly than Lewin's oral lectures and misleading experiments.

Title: Re: Does Kirchhoff's Law Hold? Disagreeing with a Master
Post by: RoGeorge on November 07, 2018, 10:14:21 pm
If you want to play this "path" game, and insist that voltage is not by default the lowest energy path between the two points, then you have to say that the voltage between those point is undefinable, no?

Yes.
I'm intrigued, where can I find this definition of voltage as "the lowest energy path"? (With the battery, any path between two given points requires the same energy.)

The wires and the resistors are nothing more than a way to force the charges to flow through a specific path.  When there is no variable magnetic field, the path does not matter.  When there is a variable magnetic field, the path is essential.

A good intuitive understanding is the old analogy with the water flow.  If we have our loop inside the variable field, all the water will flow through our loop in a circle, always in the same direction.  If we swim from the top to the bottom of our loop against the water flow, we will spend a lot of energy.  If we swim again from top to bottom, but this time the other way around (with the water flow) we will gain some energy from the water flow.  The energy spent or gained is our voltage.  Once is negative, once is positive.  Obviously they are not equal, so it is important if we choose to swim clockwise or counterclockwise.

For the case where the current flow is caused only by the battery, it doesn't matter if we choose to swim clockwise or counterclockwise.  It matters only the start point and the end point.
Title: Re: Does Kirchhoff's Law Hold? Disagreeing with a Master
Post by: rfeecs on November 07, 2018, 10:22:01 pm
He even takes this argument so far to the point of saying that Kirchoff's voltage law does not apply to circuits with inductors, because they have a changing magnetic field:
http://web.mit.edu/8.02/www/Spring02/lectures/lecsup4-1.pdf (http://web.mit.edu/8.02/www/Spring02/lectures/lecsup4-1.pdf)

Thanks for the reference. This helps making it much clearer what he was meaning - especially on page 3.

We've discussed this many times in previous threads.  Dr. Lewin gave his world famous SUPER DEMO as he refers to it in 2002, I guess.  But he didn't invent it.  It is an exact recreation of the experiment in this 1982 paper:
http://www.phy.pmf.unizg.hr/~npoljak/files/clanci/guias.pdf (http://www.phy.pmf.unizg.hr/~npoljak/files/clanci/guias.pdf)

In the paper, everything is explained very simply without drama.  It's no mystery.  The meter wires are part of the circuit and the orientation of the wires determines the results.
Title: Re: Does Kirchhoff's Law Hold? Disagreeing with a Master
Post by: ajb on November 07, 2018, 10:29:24 pm
He even takes this argument so far to the point of saying that Kirchoff's voltage law does not apply to circuits with inductors, because they have a changing magnetic field:
http://web.mit.edu/8.02/www/Spring02/lectures/lecsup4-1.pdf (http://web.mit.edu/8.02/www/Spring02/lectures/lecsup4-1.pdf)

From that same supplement, emphasis mine:
Quote
Suppose you put the probes of a voltmeter across the terminals of an inductor (with very small resistance) in a circuit. What will you measure? What you will measure on the meter of the voltmeter is a "voltage drop" of Ldi/dt. But that is not because there is an electric field in the inductor! It is because putting the voltmeter in the circuit will result in a time changing magnetic flux through the voltmeter circuit, consisting of the inductor, the voltmeter leads, and the large internal resistor in the voltmeter


I think that shows pretty clearly that he is not making a measurement mistake, and that he understands exactly why he got the result he did in his demonstration.  So I guess he's so dismissive of KVL not because it's wrong, but because it gets the right answer via an insufficiently rigorous method, even when lumped inductors are added to his resistor model.

Quote
a case of theoretical physics vs practical engineering approaches.


Just so.
Title: Re: Does Kirchhoff's Law Hold? Disagreeing with a Master
Post by: RoGeorge on November 07, 2018, 10:35:33 pm
The meter wires are part of the circuit and the orientation of the wires determines the results.

What happens if we remove the voltmeters?  How do we measure the voltage without the voltmeters?  We will need to take an electric charge and drag it through the circuit from A to B.  How much energy do we need to move our charge from A to B?

The answer is: it depends.  If we go from A to B through the left side, or through the right side.  Not only that the energy is different for each half of the circuit, but it has an opposite sign.  No chance to get a zero for the whole loop.
Title: Re: Does Kirchhoff's Law Hold? Disagreeing with a Master
Post by: SiliconWizard on November 07, 2018, 10:50:03 pm
He even takes this argument so far to the point of saying that Kirchoff's voltage law does not apply to circuits with inductors, because they have a changing magnetic field:
http://web.mit.edu/8.02/www/Spring02/lectures/lecsup4-1.pdf (http://web.mit.edu/8.02/www/Spring02/lectures/lecsup4-1.pdf)

Thanks for the reference. This helps making it much clearer what he was meaning - especially on page 3.

We've discussed this many times in previous threads.  Dr. Lewin gave his world famous SUPER DEMO as he refers to it in 2002, I guess.  But he didn't invent it.  It is an exact recreation of the experiment in this 1982 paper:
http://www.phy.pmf.unizg.hr/~npoljak/files/clanci/guias.pdf (http://www.phy.pmf.unizg.hr/~npoljak/files/clanci/guias.pdf)

In the paper, everything is explained very simply without drama.  It's no mystery.  The meter wires are part of the circuit and the orientation of the wires determines the results.

Absolutely, and this is exactly what several of us have been saying all along.
The experiment itself and the drama add no value.

There's still one valid point in what he says in how EEs interpret Kirchhoff's 2nd law. This doesn't change anything in practice.
Title: Re: Does Kirchhoff's Law Hold? Disagreeing with a Master
Post by: rfeecs on November 07, 2018, 11:02:23 pm
The meter wires are part of the circuit and the orientation of the wires determines the results.

What happens if we remove the voltmeters?  How do we measure the voltage without the voltmeters?  We will need to take an electric charge and drag it through the circuit from A to B.  How much energy do we need to move our charge from A to B?

The answer is: it depends.  If we go from A to B through the left side, or through the right side.  Not only that the energy is different for each half of the circuit, but it has an opposite sign.  No chance to get a zero for the whole loop.

Yes, the wires are determining the path from A to B.

So you are supposed to get the amount of work by multiplying the charge by the E field and integrating that over the path.  But that only works for electrostatics.  We have a changing magnetic field so the electrostatic potential is not defined.  We can make up a potential that still works for electrodynamics and Faraday's law, the magnetic vector potential:
https://en.wikipedia.org/wiki/Electric_potential (https://en.wikipedia.org/wiki/Electric_potential)

As the article points out, there is confusion over the language.  What is meant by potential, voltage drop, potential difference, EMF?  They tend to be used interchangeably but they can mean different things.
Title: Re: Does Kirchhoff's Law Hold? Disagreeing with a Master
Post by: RoGeorge on November 07, 2018, 11:11:07 pm
http://www.phy.pmf.unizg.hr/~npoljak/files/clanci/guias.pdf (http://www.phy.pmf.unizg.hr/~npoljak/files/clanci/guias.pdf)

Question:
If we remove the voltmeters and their leads, what is the voltage between A and B?

An even simpler question:
No voltmeters, no leads, no wires. Is the voltage between A and B positive, or negative?  <- drama, it's both!   :o
Title: Re: Does Kirchhoff's Law Hold? Disagreeing with a Master
Post by: ArthurDent on November 07, 2018, 11:12:02 pm
As a relevant point of interest, you cannot have a half-turn transformer winding.

Other than the previous link on fractional turns dated 2003, an engineer named Franklin d'Entremont who worked for GE in Somersworth, NH got a patent in 1942 for fractional turns in line frequency transformers.

http://www.freepatentsonline.com/2284406.html (http://www.freepatentsonline.com/2284406.html)

An easy way to get a half turn on a winding with multiple turns is to have 2 windings in parallel with 1 turn difference between the two windings. If one winding has 100 turns and the other has 101 turns, the average is 100.5 turns. It does waste a little power but if there are quite a few turns for the two windings it won't make much difference.   

 
Title: Re: Does Kirchhoff's Law Hold? Disagreeing with a Master
Post by: rfeecs on November 07, 2018, 11:56:59 pm
http://www.phy.pmf.unizg.hr/~npoljak/files/clanci/guias.pdf (http://www.phy.pmf.unizg.hr/~npoljak/files/clanci/guias.pdf)

Question:
If we remove the voltmeters and their leads, what is the voltage between A and B?

An even simpler question:
No voltmeters, no leads, no wires. Is the voltage between A and B positive, or negative?  <- drama, it's both!   :o

OK, no voltmeter, no wires.  Calculate the voltage between A and B (using physics, Maxwell's equations or whatever).  You will probably need to know the length of the wire segments.  Assume 4 equal segments and the resistors have negligible length.  Do you get more than one number?  No.
Title: Re: Does Kirchhoff's Law Hold? Disagreeing with a Master
Post by: bsfeechannel on November 08, 2018, 12:31:37 am
The problem with Kirchhoff is that he's not thinking fourth-dimensionally. The strict adherence to Kirchhoff's laws is what makes it difficult for people to grasp RF.
Title: Re: Does Kirchhoff's Law Hold? Disagreeing with a Master
Post by: Simon on November 08, 2018, 08:49:06 am
It is not like he taught advanced physics. From what I've seen his lectures were more like physically themed perfo rmances rather than actual lectures. This was fine for the first year EEs that have never seen physics before.

Yes I have to admit that the videos I have seen don't go into anything too profound and I guess with the more public profile that he has if that is what he is into then it's far enough for educating the average person. He has always striked me as a fairly decent fellow and of his lectures that I have seen I enjoyed most the ones that were indeed more performance in nature. I think he did an excellent job at giving physics a practical angle and perhaps inspiring people. I found his parting public lecture very entertaining and a little informative and greatly enjoyed the lecture he gave to children on the nature of sounds.

In this case though he has suffered the beginner mistake of poor probing and allowing his probe to become part of the circuit.

It is fairly a common problem with people that don't have to deal with the hard physics to miss "real world" factors like stray capacitance. I tried in vain to explain this to a guy at work that has just gotten into radio and is going by rules he is being taught in radio class. He thought that he could solve my EMC issue with standard aerial chokes until I explained what common mode noise is and that that inductor in real life is not an inductor but a tuned circuit and that for RFI purposes 2 inductors of the same value could behave differently because of this thing called stray capacitance.
Title: Re: Does Kirchhoff's Law Hold? Disagreeing with a Master
Post by: KL27x on November 08, 2018, 10:58:20 am
Quote
What happens if we remove the voltmeters?  How do we measure the voltage without the voltmeters?  We will need to take an electric charge and drag it through the circuit from A to B.  How much energy do we need to move our charge from A to B?

The answer is: it depends.  If we go from A to B through the left side, or through the right side.  Not only that the energy is different for each half of the circuit, but it has an opposite sign.  No chance to get a zero for the whole loop.

Well, when measuring voltage, why are we "dragging the electron" where it doesn't want to go? Isn't the usual way to do this to let the electron go where it wants? If you have to drag it one way and input energy, and the opposite is true for the other direction, it will take the other way every time. This is why the electrons go in a circle, here, right? From A to B via route 1, and B to A via route 2? You make it sound like there's some random chance electricity will spontaneously take the uphill direction against the merry-go-round and is therefore the voltage undefined. The electron is not going to go A to B via route 2, because it doesn't want to. Oh.. wait. yeah. i'm in way over my head, here. I think I'm starting to see where I have no idea what the grown ups are talking about.  >:D

I should probably watch lectures 8, 5, 11, 7 and 15.
Title: Re: Does Kirchhoff's Law Hold? Disagreeing with a Master
Post by: In Vacuo Veritas on November 08, 2018, 02:17:11 pm

We've discussed this many times in previous threads.  Dr. Lewin gave his world famous SUPER DEMO as he refers to it in 2002, I guess.  But he didn't invent it.  It is an exact recreation of the experiment in this 1982 paper:
http://www.phy.pmf.unizg.hr/~npoljak/files/clanci/guias.pdf (http://www.phy.pmf.unizg.hr/~npoljak/files/clanci/guias.pdf)

In the paper, everything is explained very simply without drama.  It's no mystery.  The meter wires are part of the circuit and the orientation of the wires determines the results.

LOL this Lewin guy is sounding more and more like the typical academic bitter old crank. Not attributing previous work and acting like a fool? Time to push him out on an ice floe, seems to me.
He's the Leo Kronecker of the physics world...
Title: Re: Does Kirchhoff's Law Hold? Disagreeing with a Master
Post by: RoGeorge on November 08, 2018, 03:06:06 pm
Quote
What happens if we remove the voltmeters?  How do we measure the voltage without the voltmeters?  We will need to take an electric charge and drag it through the circuit from A to B.  How much energy do we need to move our charge from A to B?

The answer is: it depends.  If we go from A to B through the left side, or through the right side.  Not only that the energy is different for each half of the circuit, but it has an opposite sign.  No chance to get a zero for the whole loop.

Well, when measuring voltage, why are we "dragging the electron" where it doesn't want to go? Isn't the usual way to do this to let the electron go where it wants? If you have to drag it one way and input energy, and the opposite is true for the other direction, it will take the other way every time. This is why the electrons go in a circle, here, right? From A to B via route 1, and B to A via route 2?

Very good idea, looks much clear if we move the probing electron as you say.

Let's drag our probing electron with the flow. From A to B via route 1, and B to A via route 2.  Our electron will gain some energy on route 1, then will gain some more energy on route 2.  After circling a full loop, from A to A, we end up with some extra energy in our probing electron.  That energy is the voltage around the loop.  So, the sum of voltages around the loop is NOT zero, as Kirchhoff Voltage Law predicted.

This is the contradiction that makes the professor saying "Kirchhoff is for the birds" here.
Title: Re: Does Kirchhoff's Law Hold? Disagreeing with a Master
Post by: KL27x on November 08, 2018, 11:08:48 pm
so hmm. it this a close analogy?

A mobius strip doesn't have a front and a back.... So therefore all the dummies using the term front and back are really talking about a special case (which is pretty much everything except a contrived thought experiment). So it is therefore an OUTRAGE for textbooks to be incorrectly stating that things have fronts and backs when it is not universally the case?
Title: Re: Does Kirchhoff's Law Hold? Disagreeing with a Master
Post by: SiliconWizard on November 08, 2018, 11:12:39 pm
so hmm. it this a close analogy?

A mobius strip doesn't have a front and a back.... So therefore all the dummies using the term front and back are really talking about a special case (which is pretty much everything except a contrived thought experiment). So it is therefore an OUTRAGE for textbooks to be incorrectly stating that things have fronts and backs when it is not universally the case?

You're pushing it a little too far, but that was quite fun. :-DD
Title: Re: Does Kirchhoff's Law Hold? Disagreeing with a Master
Post by: KL27x on November 09, 2018, 12:06:11 am
Ok, trying to be serious, and sticking my neck out. Is this close at all?

Kirchoff's law is not valid in a changing magnetic field. This is given fact. It's accepted. But this is only because of what we define as a closed circuit. We can add "invisible strings" in the form of magnetic flux, and by convention, the circuit is still closed. Which would be sorta like doing a momentum analysis between two colliding steel balls without figuring the effect of a magnet under the table. But by convention, this is the case, and thus Kirchoff's Law is considered invalid in a magnetic flux.

The good Doc has chosen a strange example to make this point, when a must better example would have been to show one loop in a transformer (well that's not a closed circuit!) another example where you could include in the effect of magnetic flux to show how that make Kirchoffs law correct. In fact, the example/experiment is obfuscating to the actual point, and further weirded by completely ignoring the magnet under the table. But it does make you go ooh, and want to watch lectures 8, 5, 7, 11, and 15.

If he makes more of it that this, then he knows something the rest of the world doesn't? Or he is making a mobius strip out of a molehill? (Hmm, maybe that should be "mountain out of a mobius strip?")
Title: Re: Does Kirchhoff's Law Hold? Disagreeing with a Master
Post by: Simon on November 09, 2018, 07:34:06 am
oh for christs sake how hard is it to see that voltage is also induced into the probe wires that were being ignored as part of the circuity are. In magnetic's any length of wire cannot be ignored like we do in DC, in AC it can start to be a problem and in magnetic's well tin foil hat time and don't breath the wrong way.
Title: Re: Does Kirchhoff's Law Hold? Disagreeing with a Master
Post by: RoGeorge on November 09, 2018, 09:51:49 am
[Kirchhoff's Voltages Law (KVL) failure] would be sorta like doing a momentum analysis between two colliding steel balls without figuring the effect of a magnet under the table.
  :-+

That's exactly our kind of problem.

Of course, we were trained to identify and deal with it.  But it was not so obvious that, in fact, we are "patching" the KVL by adding imaginary batteries to our circuit loop, isn't it?  If it were so obvious, we wouldn't bother to talk about it.  We are doing that in order to include the external influences of a variable flux.  KVL was not meant to include the external influences.  Kirchhoff derived the KVL for circuits with batteries and no external fields.

The other Kirchhoff's law, Kirchhoff's Currents Law (KCL) still works just fine no matter the externally magnetic fields.  Only KVL doesn't hold, and only in a variable magnetic flux.  KVL in a constant flux, again no problem.

Now, is the professor correct, or not?  Does Kirchhoff derived his KVL before, or after adding the virtual sources representing induced voltages caused by a variable flux?  All the clues indicates that Kirchhoff was not concerned about induced voltages from external fields.  He was not even looking for KVL.  Kirchhoff was trying to find a way to calculate all the currents in a mesh of linear wires, so he was looking for KCL, not KVL.  If I understood it correctly, Kirchhoff was thinking about telegraph wires when he derived KCL and, unintendly, KVL too.  (see https://www.jstor.org/stable/20021539 (https://www.jstor.org/stable/20021539) starting from the last paragraph)

Quote
Before taking his degree, Kirchhoff had begun his work in original
 research, and published a remarkable paper on electrical conduction in
 a thin plate, especially a circular one. His problem was to find the
 current in any branch of a network of linear conductors. Starting

 from Ohm's familiar law, he derived two results long recognized in
 electrical science as Kirchhoff 's laws.

I couldn't find the original papers with the KCL and KVL published by Kirchhoff.  All I could find is a followup of the KCL and KVL paper, (which, by the way, seems to be the first analysis of a transmission line: https://www.ifi.unicamp.br/~assis/Apeiron-V19-p19-25(1994).pdf (https://www.ifi.unicamp.br/~assis/Apeiron-V19-p19-25(1994).pdf) ). In this followup paper, Kirchhoff started from a real problem of those times: What happens in underwater telegraph wires.  Again, the influence caused by an external variable magnetic flux was not a concern for the problem of submarine telegraph cables.  All clues indicates that KCL and KVL were originally meant to be used for normal circuits, with batteries, and without considering external induced voltages.

I'll say the professor was correct when he said we can not always apply KVL.  Of course, if we first transform the real circuit into a lumped circuit, where we add the externally induced voltages as voltage sources internal to our circuit, then we obtain a new circuit that obeys KVL.
Title: Re: Does Kirchhoff's Law Hold? Disagreeing with a Master
Post by: MT on November 09, 2018, 04:23:08 pm
Come to re-think of it, and as he's clearly not an idiot, I was then willing to believe that he actually did that on purpose, just to make young students aware of the question: using simplistic models while thinking they hold true in the real world, which is a very common pitfall. This would be all good if he made it clear in the end that it was his intent instead of making it even more confusing, to the point that he even managed to confuse some very experienced engineers, using his position of authority.

Now if he was genuinely trying to instill advanced physics notions in young heads, I think this was a very bad way of doing it from a pedagogical standpoint.
Precisely.

The attitude of some professors to initially troll up some drama then later express the correct explanation in their lecturers but newer really make it clear is just crap mentality and bad pedagogics just causing confusion on everyone else expense. I have seen Prof Leonard Susskind do the same in some of his quantum physics lectures.
Title: Re: Does Kirchhoff's Law Hold? Disagreeing with a Master
Post by: bsfeechannel on November 09, 2018, 05:22:48 pm
I'll say the professor was correct when he said we can not always apply KVL.  Of course, if we first transform the real circuit into a lumped circuit, where we add the externally induced voltages as voltage sources internal to our circuit, then we obtain a new circuit that obeys KVL.

We engineers do that all the time. We tend to reduce a complicated problem to a simpler one for practical purposes. What Professor Lewin is probably trying to do is to call the attention to the fact that you'll have a hard time if you always think that way.

You see, Faraday-Maxwell is not easy. It involves vector calculus and a bunch of non intuitive concepts. That kind of study takes several semesters of an engineering course. Many consider that theory impenetrable.

For those who have Kirchhoff as second nature, this is an additional difficulty. Since Kirchhoff is simpler and easier to apply, Faraday-Maxwell seems unnecessarily complicated and a pain to reconcile.

He probably noticed that in his students and decided to demonstrate with an experiment what kind of confusion this may lead to. But as someone has pointed out, it ended up causing more confusion than convergence.
Title: Re: Does Kirchhoff's Law Hold? Disagreeing with a Master
Post by: HackedFridgeMagnet on November 09, 2018, 10:21:47 pm

I'll say the professor was correct when he said we can not always apply KVL.  Of course, if we first transform the real circuit into a lumped circuit, where we add the externally induced voltages as voltage sources internal to our circuit, then we obtain a new circuit that obeys KVL.

They are always just going to be two mathematical models of the same circuit, the real circuit is in the lab.

It would be necessary to have the formal definition of the law in front of us to see if it is disproved. If anyone can track that down it would be very informative.
Short of that we can use Wikipedia due to it being the most common interpretation of the law.  https://en.wikipedia.org/wiki/Kirchhoff%27s_circuit_laws (https://en.wikipedia.org/wiki/Kirchhoff%27s_circuit_laws)

No one has yet produced a demonstration that disproves KVL in a dynamic magnetic field.
I would like to see it.
As others have stated if you model the system correctly you can predict results using KVL and reproduce them in a lab. If there is an exception to this I would like to have it demonstrated.

Also I think the way some people are interpreting Maxwell-Faraday is wrong.
They are thinking the sum of voltages around a closed loop which is short circuited will be equal to the rate of change of the magnetic field. I doubt this is the case. Think of a single loop of copper. How would you even measure the two different voltages at the same point.
I am fairly sure Maxwell-Faraday describes a case where the loop is magnetically closed but electrically open circuit.

Title: Re: Does Kirchhoff's Law Hold? Disagreeing with a Master
Post by: ejeffrey on November 09, 2018, 11:08:12 pm
Wow. I was not even aware that there is such a controversy over Kirchhoff's law.

I though Kirchhoff's laws were derivable from Faraday's law and Maxwell equations in general. But quick search shows that it is not very easy.

KVL is only derivable from maxwells equations when dB/dt = 0.  It is basically a statement that the force on an electric charge is representable by a conservative field.  That means that the energy to more a particle between two locations is path independent, or in particular that moving a charge in a loop requires zero net work regardless of path. You can mathematically show that a conservative field can be integrated to create a scalar potential field, which in the case of electric circuits is just the voltage.

You can't do that with changing magnetic fields.  Curl-E = -dB/dt, so there is no scalar potential that describes electron motion.  That is what Walter Lewin was showing.  Everything else is just noise.
Title: Re: Does Kirchhoff's Law Hold? Disagreeing with a Master
Post by: rfeecs on November 09, 2018, 11:31:00 pm
Dr. Lewin is discussing two apparent problems with Kirchoff's voltage law when a changing magnetic field cuts through the surface of the loop:
1.  The voltage between two points depends on the path you take.
2.  The voltages around the loop don't add up to zero.

He is strictly defining voltage as the integral of E dot dl.  (This is the definition of electrostatic potential so obviously there is a problem here with a changing magnetic field.)

So he goes from point A to point B on one side and adds up the integral of E dot dl.  It's just IR1.  Then he goes from point A to point B on the other side and adds up the integral of E dot dl.  Its just -IR2.   OMG! They are not equal!  They are not even the same sign!  This demonstrates point number 1.

So add up the voltages all the way around the loop.  He says this is IR1 + IR2 and it doesn't equal zero!  So KVL is for the birds.  In fact it equals the inducted EMF around the loop.  So it agrees with Faraday's law:
IR1 + IR2 = EMF
This demonstrates point 2.

He then does his SUPER demo and blows your mind.  End of lecture.

Now people see the YouTube video and start saying:
"As for your demo, you are measuring things wrong.  Your test leads are forming a loop around the magnetic field and that is giving you a false measurement.
KVL still works!  The EMF appears across the ends of the wires connecting the resistors!  It's just like a transformer.  If you add up the voltages,
IR1 + IR2 - EMF = 0
The voltages around the loop sum to zero."

Dr. Lewin's response to this is:  You can't do that!  You can't just move the EMF from one side of the equation to the other!  That's dead wrong!  That's criminal. (https://bit.ly/2qzwkh0 (https://bit.ly/2qzwkh0))

This is the reasoning you are arguing against.
Title: Re: Does Kirchhoff's Law Hold? Disagreeing with a Master
Post by: ajb on November 10, 2018, 01:38:24 am
Dr. Lewin's response to this is:  You can't do that!  You can't just move the EMF from one side of the equation to the other!  That's dead wrong!  That's criminal. (https://bit.ly/2qzwkh0 (https://bit.ly/2qzwkh0))

Wait, one of the top comments on that video is from Mehdi, with responses from Lewin from A YEAR ago.  Then three days ago, Lewin added a new response with the same bunch of links he was putting all over the lecture video comments.  Wow.  He really does get worked up about where you put that EMF, doesn't he?
Title: Re: Does Kirchhoff's Law Hold? Disagreeing with a Master
Post by: EEVblog on November 10, 2018, 11:38:09 am
You can't do that with changing magnetic fields.  Curl-E = -dB/dt, so there is no scalar potential that describes electron motion.  That is what Walter Lewin was showing.  Everything else is just noise.

Yeah, but that "noise" is a very big claim that two points on the same circuit measure differently, he states that as a fact and uses a flawed demonstration to try and prove it. This is why many people have a big problem with this.
Title: Re: Does Kirchhoff's Law Hold? Disagreeing with a Master
Post by: RoGeorge on November 10, 2018, 12:29:29 pm
... is a very big claim that two points on the same circuit measure differently... This is why many people have a big problem with this.

Yes, two points can measure differently.

Why do you think this is not OK?  What rule does it violates?
Title: Re: Does Kirchhoff's Law Hold? Disagreeing with a Master
Post by: gildasd on November 10, 2018, 01:11:39 pm
The prof might be right in theory, but in this case Mehdi is right.

Loops can cancel or add to themselves in a electromagnetic situation, and (as I understand it) Mehdi points that this was not considered properly.
It’s not because something appears on a oscilloscope screen and you have Einstein’s hairdresser that it automatically sustains your postulate.

I have worked in a electromotor maintenance factory and I think i would be laughed out if i tried to sense a coil with so sloppily and diagnose the motor with the result.

That said, I cannot fault his other stuff, and he might be right in theory for specific situations.
Title: Re: Does Kirchhoff's Law Hold? Disagreeing with a Master
Post by: rfeecs on November 10, 2018, 05:19:47 pm
He posted a teaser video, sounds familiar:

https://youtu.be/qAtqgSaEU4Y (https://youtu.be/qAtqgSaEU4Y)
Title: Re: Does Kirchhoff's Law Hold? Disagreeing with a Master
Post by: bsfeechannel on November 10, 2018, 08:16:55 pm
https://www.youtube.com/embed/oXZa89Hv8Bo?start=79&end=81 (https://www.youtube.com/embed/oXZa89Hv8Bo?start=79&end=81)
Title: Re: Does Kirchhoff's Law Hold? Disagreeing with a Master
Post by: bsfeechannel on November 11, 2018, 01:29:52 pm
Let me hazard some thoughts.

The figure below shows a voltage source and two voltmeters. One is connected as closely as possible to the source and another one at a distance very, very far from it. The switch is open, so the second voltmeter reads 0 volts. All conductors are ideal, i.e., resistance is 0 ohms.

(https://www.eevblog.com/forum/chat/does-kirchhoffs-law-hold-disagreeing-with-a-master/?action=dlattach;attach=568870;image)

Now we close the switch and see the electric field propagating towards the second voltmeter, which still indicates 0V. We can see that, during this transient, Kirchhoff doesn't hold. Because if you add up all the voltages in the loop they will not result 0.

(https://www.eevblog.com/forum/chat/does-kirchhoffs-law-hold-disagreeing-with-a-master/?action=dlattach;attach=568876;image)

But, and there's always a but, you can REDUCE the long line of conductors to a series of inductors and capacitors, and now Kirchhoff holds. You can explain why the faraway voltmeter is indicating 0V by the fact that, along the line, some capacitor is still uncharged, while some inductor is reacting to the change of its current with an EMF equal to the source voltage.

(https://www.eevblog.com/forum/chat/does-kirchhoffs-law-hold-disagreeing-with-a-master/?action=dlattach;attach=568882;image)

However, there are no such inductors or capacitors. If we consider their existence, they must be infinitesimal. There are infinite infinitely tiny inductors and capacitors, and for Kirchhoff to hold you will have to apply it to infinite meshes.

This is just a simple example, but we can see that Kirchhoff is not adequate to model situations like that.

For a finite number of lumped components where space and time can be disconsidered, Kirchhoff is fine.

But add space and time and you'd be better off with a theory that takes that into consideration, and that is Faraday-Maxwell.
Title: Re: Does Kirchhoff's Law Hold? Disagreeing with a Master
Post by: ArthurDent on November 11, 2018, 06:33:19 pm
There may be an infinite number of ways to model this to show what you want to prove and this is my take. The circuit above isn't like the original problem at all partly because the left part of the circuit isn't switched the same way as the right half. The two halves are two different problems.

I'm lumping the induced voltage producer to one area, a transformer secondary instead of a distributed magnetic field, which is one part of the loop that includes a left an a right resistor of equal value. A very astute instructor I had once said: "Things that are in series are in series with each other." This means that the right resistor can be moved anywhere along the entire length of the wire connecting the left resistor and the transformer and it will always read the same voltage.

The video with the leads being moved from left to right to 'prove' there is a difference is the same as placing another induced A.C. voltage (or transformer secondary) in series with the test points either aiding or opposing the induced voltage in the coil.  Phasing is important. 
Title: Re: Does Kirchhoff's Law Hold? Disagreeing with a Master
Post by: T3sl4co1l on November 11, 2018, 08:04:55 pm
However, there are no such inductors or capacitors. If we consider their existence, they must be infinitesimal. There are infinite infinitely tiny inductors and capacitors, and for Kirchhoff to hold you will have to apply it to infinite meshes.

This is just a simple example, but we can see that Kirchhoff is not adequate to model situations like that.

For a finite number of lumped components where space and time can be disconsidered, Kirchhoff is fine.

But add space and time and you'd be better off with a theory that takes that into consideration, and that is Faraday-Maxwell.

Correct.  As I noted at the start of this thread, you must integrate over space to apply KVL and KCL for the general -- wave mechanical -- case.  For the 1-D case, this effectively gives the differential RLC transmission line element used above, and when solved, gives the Telegrapher's equations.  For the 3D case, you get field solutions of course.

We apply Kirchhoff's laws on coarser elements (e.g., whole circuit loops), when it is justifiable to do so, for example when the signals of interest are slower than the physical scale of the system (so that we need not consider wave mechanics as such).  In that case, a netlist (an abstract schematic drawing) and finitely many L and C can be used.  At lower and lower frequencies, the number of L and C required drops, until at DC, L and C go away completely and we need only consider the network of resistances.

We can likewise consider wave mechanics alone, when it is justifiable to do so.  For example, building active circuitry in as small a form factor as possible, then interconnecting the circuits with transmission lines.  KVL and KCL are broken between the ends of the transmission line, but we can still consider them locally, i.e. at each port of the line.

This is a very powerful design approach, allowing the designer to greatly simplify the circuit: one need not consider every possible coupling between elements in the circuit, but only those close enough together (including self-coupling, i.e., LF-equivalent stray L and C).

Tim
Title: Re: Does Kirchhoff's Law Hold? Disagreeing with a Master
Post by: Berni on November 11, 2018, 08:56:57 pm

However, there are no such inductors or capacitors. If we consider their existence, they must be infinitesimal. There are infinite infinitely tiny inductors and capacitors, and for Kirchhoff to hold you will have to apply it to infinite meshes.

This is just a simple example, but we can see that Kirchhoff is not adequate to model situations like that.

For a finite number of lumped components where space and time can be disconsidered, Kirchhoff is fine.

But add space and time and you'd be better off with a theory that takes that into consideration, and that is Faraday-Maxwell.

And this is exactly why we lump together these stray inductance and capacitance into lumped form that approximates the behavior of an infinitely large mesh as close as possible. Makes math a whole lot easier.

Its not only a problem of a wire. The resistors also have physical dimensions and as such have parasitics. To perfectly accurately model the resistor we would need to have a infinitely large mesh of resistors capacitors indusctors just to model the internal construction of say a metal film type troughhole resistor. Can you simplify this infinite mesh down to a single RLC circuit and still have it perform close enough to give us near perfect results once we do math to it? Yep we sure can so we do it, because doing math to 3 components is easier than to an infinite number of components.

If we tried to calculate the behavior of perfect non simplified models of a circuit we would need a computer that is much much larger than the largest supercomputers we have since we would basically need to simulate every single electron and every single atom of of the circuit. Yet using lumped component models we can get nearly the same result using the computing power of a PC from the 90s.
Title: Re: Does Kirchhoff's Law Hold? Disagreeing with a Master
Post by: rfeecs on November 12, 2018, 12:25:21 am
OK, well take the original demo, but instead of two resistors, make the whole loop one big resistor.  That is, a loop made out of resistive material:

(https://www.eevblog.com/forum/chat/does-kirchhoffs-law-hold-disagreeing-with-a-master/?action=dlattach;attach=569548;image)

Say the EMF induced in the loop is 1V.  Say the total resistance is 1 ohm, so you have 1 amp flowing around the loop.  Take any point in the loop and go around the loop adding up the IR drop.  Go all the way around the loop.  You always end up with 1V, not zero.

So how are you going to model this to make Kirchoff's law work?  An infinite number of resistors dR and an infinite number of voltage sources dV? (I don't think so.)
Title: Re: Does Kirchhoff's Law Hold? Disagreeing with a Master
Post by: HackedFridgeMagnet on November 12, 2018, 12:32:20 am
Say the EMF induced in the loop is 1V.  Say the total resistance is 1 ohm, so you have 1 amp flowing around the loop.  Take any point in the loop and go around the loop adding up the IR drop.  Go all the way around the loop.  You always end up with 1V, not zero.

So how are you going to model this to make Kirchoff's law work?  An infinite number of resistors dR and an infinite number of voltage sources dV? (I don't think so.)
No in this case you will end up with 0v if you measure it.
You're misinterpreting Faraday-Maxwell.
Title: Re: Does Kirchhoff's Law Hold? Disagreeing with a Master
Post by: rfeecs on November 12, 2018, 02:26:20 am
Say the EMF induced in the loop is 1V.  Say the total resistance is 1 ohm, so you have 1 amp flowing around the loop.  Take any point in the loop and go around the loop adding up the IR drop.  Go all the way around the loop.  You always end up with 1V, not zero.

So how are you going to model this to make Kirchoff's law work?  An infinite number of resistors dR and an infinite number of voltage sources dV? (I don't think so.)
No in this case you will end up with 0v if you measure it.
You're misinterpreting Faraday-Maxwell.

I am fairly sure Maxwell-Faraday describes a case where the loop is magnetically closed but electrically open circuit.
I'm fairly sure you are misinterpreting it.

But maybe I wasn't clear.  Say you have the resistive loop.  If you could measure say a section 1/10th of the way around.  (I admit it is not easy to accurately measure in the presence of the magnetic field.)  So that section has resistance 1/10th of an ohm and has current 1A flowing.  You should measure 0.1V.  So there are 10 pieces, and if you add them up, you get 1V, not 0V.  You could divide it up in other numbers of sections with the same result. 

How do you get it to add up to 0V?
Title: Re: Does Kirchhoff's Law Hold? Disagreeing with a Master
Post by: KL27x on November 12, 2018, 03:22:36 am
Quote
But maybe I wasn't clear.  Say you have the resistive loop.  If you could measure say a section 1/10th of the way around.  (I admit it is not easy to accurately measure in the presence of the magnetic field.)  So that section has resistance 1/10th of an ohm and has current 1A flowing.  You should measure 0.1V.  So there are 10 pieces, and if you add them up, you get 1V, not 0V.  You could divide it up in other numbers of sections with the same result. 
In your example you are measuring just one section of the loop. In the thought experiment, you are measuring the voltage across two points which are connected by two half loops of wire.
 
So in your example, if you measured a 1/10th section, you should measure [0.8V]. 0.9V in one direction, 0.1V in the other direction. It's a closed circuit with two parallel branches. But Lewin very clearly states that is possible for the voltmeter to have two different readings. And he suggests that it would read either [0.1V] or [0.9V] in the other direction, but he doesn't explicitly state this would be due to inductance in the probe wires. In this example you made, I think it would be reasonable to assume a voltmeter that is connected properly would read [0.8V], which is part of what Mehdin seems to have shown, but maybe I am completely wrong. A conundrum in this example is what happens are you move your two points closer together until they approach zero distance apart? Will you essentially measure [1.0V] with the probes essentially touching? That would be cool. Lewin would say the voltage is undefined in the ideal experiment in all these cases. But clearly Medhin can get meaningful voltage measurements in the real world experiment using real copper wire and regular resistors.

In the actual thought experiment, it's not the resistance of the wire that is causing drop in the wire, it's the inductance. The wire has negligible resistance, at least in comparison with the two resistors that were put in the loop. Now the inductance apparently can't even exist if the wire were an actual ideal superconductor, according to Lewin's teaser video.

I'm sure I'm completely wrong and look forward to the corrections. But more or less that's the way I tend to think about it, right now.
Title: Re: Does Kirchhoff's Law Hold? Disagreeing with a Master
Post by: Berni on November 12, 2018, 06:35:51 am
OK, well take the original demo, but instead of two resistors, make the whole loop one big resistor.  That is, a loop made out of resistive material:

(https://www.eevblog.com/forum/chat/does-kirchhoffs-law-hold-disagreeing-with-a-master/?action=dlattach;attach=569548;image)

Say the EMF induced in the loop is 1V.  Say the total resistance is 1 ohm, so you have 1 amp flowing around the loop.  Take any point in the loop and go around the loop adding up the IR drop.  Go all the way around the loop.  You always end up with 1V, not zero.

So how are you going to model this to make Kirchoff's law work?  An infinite number of resistors dR and an infinite number of voltage sources dV? (I don't think so.)

Interestingly yes you would get zero volts in this case!

(https://www.eevblog.com/forum/chat/does-kirchhoffs-law-hold-disagreeing-with-a-master/?action=dlattach;attach=569761;image)

It turns out you can poke any two points in this circle and have the points sit at 0V despite there being 0.8mA peak of current flowing in this simulation. This happens because the voltages on each inductor and resistor pair balance out. Spice follows Kirchhoffs rule so you need the voltages to add up. Since the current is the same means the voltage drop on all resistor is the same, the rest of the drop is in the inductors and since they are the same it distributes equally. This makes the voltage on the resistor chunks the same as on the inductor chunks so each one of these lumped inductive resistor models has 0V across itself all around. But there is voltage within the model across the resistor or inductor alone.

This effect is broken as soon as your ring is not completely even around its radius. If you ware to make the resistive material thicker at some point this upsets the balance and you see a voltage across the thinnest part of the ring.

And yes the inductive probing leads again would mess things up, but again you could fix the probing problem by simply averaging what the left and the right scope sees.

EDIT: Updated the test leads to 2.5uH to reflect them being the same as 1/4 of the loop.
Title: Re: Does Kirchhoff's Law Hold? Disagreeing with a Master
Post by: Berni on November 12, 2018, 06:56:15 am
Oh and i want to add that the same thing would happen in the physical demonstration with 2 resistors if you made the resistors say both 100 Ohm rather than one being 100 and other 900. This makes the demonstration a bit less exiting because then both scopes see the same shape and magnitude except that one scope is seeing the inverted waveform.

As long as the circuit is symmetrical around the two points you measure this is always the result. The same current flows around it so an identical circuit will produce the same voltage drop on both symmetrical sides, but since one of the halves is having the current running the opposite way it also generates an opposite voltage. So once summed up around the loop you basically are subtracting two voltages of the same size so you get 0V.
Title: Re: Does Kirchhoff's Law Hold? Disagreeing with a Master
Post by: Sredni on November 12, 2018, 07:31:54 am
@RoGeorge, just to let you know that you are not alone.

The only critic I would move to Lewin is in its use of the word "potential" even when the quantity is not defined because it is multivalued. I wonder if his demo would have had a better reception had he used the word "glorp" or "multivalued potential" to describe it.

In his followup video titled "Believing and Science are Very Different" (youtube code watch?v=wz_GqO-Urk4) he also shows a conceptual diagram (I am not calling it a schematic on purpose) demonstrating how the equations give exactly that result: the 'multivalued potential' at the very same pair of points to have two different distinct values at the same time.

It is also possible to push that diagram even further by dragging a voltmeter (and its probes) inside the loop and seeing how the 'multivalued potential' between those two points varies with continuity from +0.9V to -0.1V, as the fraction of the area of the loop containing the voltmeter path goes from 0% to 100% of the area of the loop with both resistors (and the changing magnetic field). And there is also a video made by a detractor (sorry, maybe this is not the right term, opponent might be better) of Lewin that shows just this [note1]... actually confirming Lewin's modeling of the system.

I cannot but wonder what causes this resistance... pun intended... to accept the consequence of the loss of irrotationality (is this even a word?)

I guess one reason is that we place too much faith in our instruments and we tend to believe the numbers they show have a meaning no matter what.
Another might be the difficulty in realizing that the resistor are INSIDE the secondary of the imaginary transformer. So, goodbye lumped component model...
Or probably engineering courses have to cram too much material and leave out too much of the basics, concentrating on the more pragmatic parts.


EDIT: corrected some of my lousy grammar and syntax, but not all.
Also, I would like to add that as long as the probes are outside of the loop (more specifically outside of the region at varying B field) there is no  probing problem at all (you might have to worry about capacitive coupling with the mains, and RFI from RadioBoomBoom, but not about the field that is confined inside the loop. Also self inductance is negligible, as shown by Lewin in the aforementioned video.
 
[note1] Ironically, when we take the measurement path inside the loop, that's where we can interpret the results (changing from +0.9V to -0.1V) as the effect of 'bad probing' (now I have to take into account the intercepted flux).  I guess that's the reason Lewin is staying outside, to avoid confusion.

Finally, maybe getting rid of the voltmeters and their probes, resorting to some other way to measure the 'voltage' across each resistor (placing an ammeter in the loop and using ohm's law?, using calorimetric measurements to infer the dissipated power? substituting the resistors with a voltage dependent light source and measuring the light output?) could help in removing the confusion).

Personally I like to think tiny angels are pushing electrons in the loop, producing a 1mA (oscillating) current. That would cause a 0.1V drop on the 100 ohm resistor and a 0.9V drop on the 900 ohm one. The fact that it's angels and not a generator allows the loop to be closed without anything between the resistors, and that show exactly why KVL is no longer valid. You have to mend it somehow by adding a distributed emf, but that does not remove the fact that KVL is broken.
If you do not believe in tiny angels, well you could use a varying magnetic field that stays all within the loop. A toroidal transformer, maybe?
Title: Re: Does Kirchhoff's Law Hold? Disagreeing with a Master
Post by: rfeecs on November 12, 2018, 06:15:40 pm
OK, well take the original demo, but instead of two resistors, make the whole loop one big resistor.  That is, a loop made out of resistive material:

(https://www.eevblog.com/forum/chat/does-kirchhoffs-law-hold-disagreeing-with-a-master/?action=dlattach;attach=569548;image)

Say the EMF induced in the loop is 1V.  Say the total resistance is 1 ohm, so you have 1 amp flowing around the loop.  Take any point in the loop and go around the loop adding up the IR drop.  Go all the way around the loop.  You always end up with 1V, not zero.

So how are you going to model this to make Kirchoff's law work?  An infinite number of resistors dR and an infinite number of voltage sources dV? (I don't think so.)

Interestingly yes you would get zero volts in this case!

(https://www.eevblog.com/forum/chat/does-kirchhoffs-law-hold-disagreeing-with-a-master/?action=dlattach;attach=569761;image)

It turns out you can poke any two points in this circle and have the points sit at 0V...

That is wrong.  Your model is wrong.  It led you to the wrong conclusion.

I realize I went down a dark path when I said the word "voltage".  As has been pointed out, "voltage" and "potential" are words that are not defined well in this situation.  The E field is defined.  The E field in this case forms circular loops.  The integral of E dot dl is also defined here.  As RoGeorge said, charge moving around the loop  experiences a force from E over the distance moved, dl.  That involves energy.  That is defined also.  It is not coming from imaginary coils.  It's coming from the E field which is coming from the changing magnetic field.

Like a lot of people, I have a tendency to say that "the magnetic field induces a voltage between the ends of the wires".  This resistive loop is an example of what happens when there are no wires.

Now I'm not opposed to making this kind of model.  It may be appropriate in some situations.  The model of just an ideal transformer with a single lumped R would be appropriate in some situations.  It depends on what you are trying to model.

As for actually measuring with a voltmeter, if you kept your measurement leads twisted tightly together and arranged the ends tight next to the 0.1 ohm section that you are trying to measure, then you have no magnetic field passing through your measurement loop.  You would measure 0.1V, not zero volts.  That's just Faraday's law. 

Title: Re: Does Kirchhoff's Law Hold? Disagreeing with a Master
Post by: RoGeorge on November 12, 2018, 06:54:22 pm
@Sredni
Indeed, mathematically speaking it doesn't make sense to talk about Potential in a non-conservative field.

Another big confusion is created because in some modern books the definition of Voltage is considered
Code: [Select]
1) Voltage1 = Potential in point A - Potential in point B
2) Voltage2 = The energy required to move a unit of charge between A and B

For conservative fields (e.g. in Electrostatic, or in DC circuits) the two definitions are equivalent, and Voltage1 is the same thing as Voltage2.  The path does not matter.

In non-conservative fields (e.g. Electrodynamics, or in AC circuits) Voltage1 is undefined.  Potential is undefined.  Only Voltage2 makes sense, but beware of the path!  Different paths will give us different Voltage2 values.

NOTE: Our most beloved voltmeters and oscilloscopes measure the Voltage2 type of voltage. They measures the energy to move the unit charge.  They do NOT measure the Voltage1 type of voltage, they do not measure a potential difference.





Mathematically speaking, the notion of Electric Potential doesn't make sense for e.g. AC circuits.
Go figure, Electric Potential doesn't exist for AC!

P.S. Never say that at a job interview.  ;D
Title: Re: Does Kirchhoff's Law Hold? Disagreeing with a Master
Post by: ogden on November 12, 2018, 07:15:46 pm
It turns out you can poke any two points in this circle and have the points sit at 0V...

That is wrong.  Your model is wrong.  It led you to the wrong conclusion.

Do you find following model as matching your "whole loop one big 1 Ohm resistor" ?

(https://www.eevblog.com/forum/chat/does-kirchhoffs-law-hold-disagreeing-with-a-master/?action=dlattach;attach=570035;image)

Title: Re: Does Kirchhoff's Law Hold? Disagreeing with a Master
Post by: rfeecs on November 12, 2018, 08:35:12 pm

Do you find following model as matching your "whole loop one big 1 Ohm resistor" ?

(https://www.eevblog.com/forum/chat/does-kirchhoffs-law-hold-disagreeing-with-a-master/?action=dlattach;attach=570035;image)

No, that still would have each voltage source + resistor pair cancelling out to 0V.

I'm not sure how to make a model with lumped components that violates KVL.
Title: Re: Does Kirchhoff's Law Hold? Disagreeing with a Master
Post by: Berni on November 12, 2018, 09:17:18 pm
I do have some resistance wire around here that i could weld into a continuous loop and do the experiment.

Just not quite sure how to keep the probe wires from picking up the magnetic field since we are interested in the actual voltage at the test point. My average out the reading for going left and right only works when probing exact opposite points on the circle.

Using something like a lamp or LED as the 'probe' sounds like a good idea at first but then you realize you still need some wire to connect it to the two points and thus has the same problem. Running it along the ring would give similar results that you get a different voltage depending on the path the wire takes. This essentially makes the magnetically induced voltage disappear and you get to see the voltage drops on sections of resistance again. But what happens if we instead remove the restive ring but leave the probe wires connecting to the same two points in space connected to nothing? Do we measure nothing?
Title: Re: Does Kirchhoff's Law Hold? Disagreeing with a Master
Post by: rfeecs on November 12, 2018, 09:41:41 pm
But what happens if we instead remove the restive ring but leave the probe wires connecting to the same two points in space connected to nothing? Do we measure nothing?

I'm thinking you have no current path through the meter, so you you measure nothing.
Title: Re: Does Kirchhoff's Law Hold? Disagreeing with a Master
Post by: ogden on November 12, 2018, 09:59:18 pm
No, that still would have each voltage source + resistor pair cancelling out to 0V.

Yes, it cancels to 0V. In every test point labelled "A, B, C, D..." voltage against ground is 0V. That's exactly what I am showing.

Your original "model" contains single 1V EMF source in form of single, rounded 1 Ohm resistor dissipating/cancelling that 1V of EMF - that's OK, but model with 10 sources and 10 resistors in series is not ok anymore? - Apply integral to my 10x0.1V EMF sources and 10x0.1 Ohm resistors to get your single loop back. Do you see what I mean now?

[edit] For those who did not pay attention whole thread, it is advised to read this post (https://www.eevblog.com/forum/chat/does-kirchhoffs-law-hold-disagreeing-with-a-master/msg1945138/#msg1945138), especially to sentence "KVL/KCL reduce to a single point only: they are a differential relation, which must be integrated over the space of interest".

Quote
I'm not sure how to make a model with lumped components that violates KVL.

LOL. You basically said: "you are wrong, but I cannot prove it"  :-DD
Title: Re: Does Kirchhoff's Law Hold? Disagreeing with a Master
Post by: rfeecs on November 12, 2018, 11:03:29 pm

Quote
I'm not sure how to make a model with lumped components that violates KVL.

LOL. You basically said: "you are wrong, but I cannot prove it"  :-DD

KVL holds for lumped circuits.  This isn't a lumped circuit.
I can prove those models are wrong because the voltage around the loop in the model adds up to zero.
Using Faraday's law on the resistive loop, integrating E dot dl around the loop doesn't equal zero.
Title: Re: Does Kirchhoff's Law Hold? Disagreeing with a Master
Post by: ogden on November 12, 2018, 11:58:10 pm
Using Faraday's law on the resistive loop, integrating E dot dl around the loop doesn't equal zero.

Yes. Nonzero current flows in case of electrically short loop. Note that ideal conductor have zero resistance, so in case of short superconductive loop there will be zero volts no matter how, where and using how many scopes you measure. Kirchhoff's Law Holds in this case BTW ;)

Quote
KVL holds for lumped circuits.  This isn't a lumped circuit.

Why not? If you cut 0-resistance loop open and connect 1 Ohm resistor in the opening, then in our 1 Ohm & 1 A example case there will be 1V drop on the resistor and 1V EMF voltage generated in the loop. Loop is voltage source and resistor is load - what's the problem with Kirchhoff's Law in this lumped circuit?

[edit] The same considerations apply to two 0.5 Ohm resistors and two halves of the loop.
Title: Re: Does Kirchhoff's Law Hold? Disagreeing with a Master
Post by: HackedFridgeMagnet on November 13, 2018, 12:38:15 am
Pls keep the thread polite and leave out the emoticons as they can distract.

Back to the thread title.

Does Kirchhoff's Law Hold?
My belief is if you can model something and if you can verify this with measurements then your model holds.
Every model has limitations but what are the limitations of KVL?

So AFAICT it will hold in Time varying magnetic fields.
People use it in steady state analysis all the time.
But not so easy in Transmission lines.
Teslacoil suggested you can use it at either end.

So to dispprove KVL within Time varying magnetic fields I suggest you need to prove it in the lab.
IMO I don't think Dr Lewin did this.
Title: Re: Does Kirchhoff's Law Hold? Disagreeing with a Master
Post by: ogden on November 13, 2018, 12:48:55 am
So to dispprove KVL within Time varying magnetic fields I suggest you need to prove it in the lab.
IMO I don't think Dr Lewin did this.

Agreed. Rfeecs already mentioned paper explaining "Super Demo" experiment and it's results:

We've discussed this many times in previous threads.  Dr. Lewin gave his world famous SUPER DEMO as he refers to it in 2002, I guess.  But he didn't invent it.  It is an exact recreation of the experiment in this 1982 paper:
http://www.phy.pmf.unizg.hr/~npoljak/files/clanci/guias.pdf (http://www.phy.pmf.unizg.hr/~npoljak/files/clanci/guias.pdf)

In the paper, everything is explained very simply without drama.  It's no mystery.  The meter wires are part of the circuit and the orientation of the wires determines the results.
Title: Re: Does Kirchhoff's Law Hold? Disagreeing with a Master
Post by: bsfeechannel on November 13, 2018, 02:54:51 am
Using Faraday's law on the resistive loop, integrating E dot dl around the loop doesn't equal zero.

It doesn't. However, every infinitesimal gain of energy represented by E · dl is lost as heat in the infinitesimal resistor dR.

(https://www.eevblog.com/forum/chat/does-kirchhoffs-law-hold-disagreeing-with-a-master/?action=dlattach;attach=570248;image)

(https://www.eevblog.com/forum/chat/does-kirchhoffs-law-hold-disagreeing-with-a-master/?action=dlattach;attach=570254;image)
Title: Re: Does Kirchhoff's Law Hold? Disagreeing with a Master
Post by: HackedFridgeMagnet on November 13, 2018, 03:24:01 am
[edit]Apparently this is the setup Faraday used.

Not sure, but wouldn't this be a better setup to what ElectroBoom's/Lewins experiment should be using? [/edit]


(https://upload.wikimedia.org/wikipedia/commons/2/2a/Faraday_emf_experiment.svg)

The torroid would capture most of the stray flux.
So instead of worry about bad probing, you know what flux is linked.



Title: Re: Does Kirchhoff's Law Hold? Disagreeing with a Master
Post by: HackedFridgeMagnet on November 13, 2018, 03:26:51 am
couldn't link image
Title: Re: Does Kirchhoff's Law Hold? Disagreeing with a Master
Post by: Berni on November 13, 2018, 06:35:20 am
But what happens if we instead remove the restive ring but leave the probe wires connecting to the same two points in space connected to nothing? Do we measure nothing?

I'm thinking you have no current path through the meter, so you you measure nothing.

Well if you have both meters connected simultaneously just like in Dr. Lewins experiment you actually get the same voltage pulse on both meters no matter if the resistive ring is connected or not (At least inside LtSpice). This clearly points to a problem with the experimental setup since we shouldn't be measuring voltage in a circuit with 0 components. Yes if you leave only one meter connected you will indeed read 0V because the probe wires are floating. Both meters connected provides a return path trough each meter so they can measure something. This shows that the meters are not correctly showing the voltage at points A and B but a voltage somewhere else (Like across the meters terminals)

And that makes sense since the ring is supposed to have 0V across any two points on it. Connecting to nothing also produces 0V so the readings on the meters are the same in both cases.

I do want to do this continuous resistive ring as a physical experiment, but i first need to figure out the correct way to probe this.
Title: Re: Does Kirchhoff's Law Hold? Disagreeing with a Master
Post by: ogden on November 13, 2018, 01:42:40 pm
I do want to do this continuous resistive ring as a physical experiment, but i first need to figure out the correct way to probe this.

Toroidal iron core AC mains transformer will greatly help to contain magnetic flux. AC mains seems to be good enough "test signal" source as well. All you need to do - add test winding and have a fun. Beauty of using AC voltage here - you don't even need scope. Any multimeter is good enough.
Title: Re: Does Kirchhoff's Law Hold? Disagreeing with a Master
Post by: Sredni on November 13, 2018, 04:11:10 pm
"Well if you have both meters connected simultaneously just like in Dr. Lewins experiment you actually get the same voltage pulse on both meters no matter if the resistive ring is connected or not"

This is a good point. You will have a loop with the meters' internal resistances taking the place of the resistors. With 10 meg input impedance and a 1V emf, the current in the loop would be 1/20 of a microamp. If the internal impedances are the same, the voltmeters will measure the same voltage (edit: the problem with a sinusoidal stimulus is that we lose the sign, but this adds drama). But what happens if the internal resistance are different, say 10 meg, 1 meg? Would they not measure different voltages, despite being connected to the same points?


EDIT
To get the same voltage ratio as in Lewin's experiment the internal impedances (at the test signal frequency) should be 9meg, 1meg.

Probably this is where we reach the language barrier between physics and engineering. An engineer that has different readings from instruments connected to the very same poiints will try to interpret this as a measurement error (load effects) or as wrong probing (in this case using the emf to make the trusted KVL to balance again). A physicist, on the other hand, knows that there is no problem at all, it's just that the so called 'voltage' is no longer positional: it does not depend only on the points where you place the probes, it also depends on the path followed by the probes.
The engineer will say: "see? KVL checks out: one instrument measures 0.1V, the other 0.9V, and then there's the emf of 1V, when I take all signs into account, the result is zero. KVL rulez!". A physicist will say: "see? the voltage is path dependent! If I sum the voltages around this loop I get 1V. That's the emf. KVL is for the birds!"

You might think both are right, but... the engineer is getting his balance check still assuming voltage is positional - and that's not true. The physicist gets his balance check and has the satisfaction to add his impression of Walter Kronkite saying "...and that's the way it is".
Title: Re: Does Kirchhoff's Law Hold? Disagreeing with a Master
Post by: bsfeechannel on November 13, 2018, 05:32:39 pm
I do want to do this continuous resistive ring as a physical experiment, but i first need to figure out the correct way to probe this.

Lemme give it a burl using what I think I know about Maxwell.

Let's suppose we have a changing magnetic field B confined to the area Σ, and we connect a voltmeter like in the picture below. This can be achieved for example when you have a ferromagnetic core that manages to concentrate most of the lines of B in its transversal area. So the very few lines of magnetic field in the area Σ' can be considered negligible.

(https://www.eevblog.com/forum/chat/does-kirchhoffs-law-hold-disagreeing-with-a-master/?action=dlattach;attach=570917;image)

Now let's remove the right hand side of the loop between the voltmeter probes. We end up with a new loop with an area Σ+Σ'.

(https://www.eevblog.com/forum/chat/does-kirchhoffs-law-hold-disagreeing-with-a-master/?action=dlattach;attach=570923;image)

Faraday-Maxwell says that the emf generated by this loop will be the derivative relative to time of the integral of the scalar product of B and dA on the surface Σ+Σ`. dA is the infinitesimal vector element of surface perpendicular to the surface Σ+Σ`. However B is zero over Σ'. So the scalar product over Σ' will be zero. This means that the voltage measured by the voltmeter will be that as if the loop covered only the area Σ.

(https://www.eevblog.com/forum/chat/does-kirchhoffs-law-hold-disagreeing-with-a-master/?action=dlattach;attach=570929;image)

Now let's remove the left arc and replace the right arc of the loop.

(https://www.eevblog.com/forum/chat/does-kirchhoffs-law-hold-disagreeing-with-a-master/?action=dlattach;attach=570935;image)

We can see that B is totally outside of this new loop and, according to Faraday-Maxwell, will not induce a voltage. So the voltmeter is going to show 0V.

Now that we know what is going on, we can REDUCE the problem to Kirchhoff and put everything together.

(https://www.eevblog.com/forum/chat/does-kirchhoffs-law-hold-disagreeing-with-a-master/?action=dlattach;attach=570941;image)

In relation to the voltmeter, the left arc is a generator in series with half of the total resistance of the loop. The right arc is the other half and is just a load. It doesn't contribute to the emf the voltmeter sees.

If you move the voltmeter to the left side of the loop, the left arc is now the load and the right side is voltage source in series with an internal resistance. Notice that source is inverted.

(https://www.eevblog.com/forum/chat/does-kirchhoffs-law-hold-disagreeing-with-a-master/?action=dlattach;attach=570947;image)
(https://www.eevblog.com/forum/chat/does-kirchhoffs-law-hold-disagreeing-with-a-master/?action=dlattach;attach=570953;image)

But, of course, we are smart people. We place a voltmeter immune to a magnetic field in the center of the loop. And we have the equivalent Kirchhoff reduction.

(https://www.eevblog.com/forum/chat/does-kirchhoffs-law-hold-disagreeing-with-a-master/?action=dlattach;attach=570959;image)

And bingo! We have our 0V.

However, if we change the position of the voltmeter and of the leads we are going to have whatever voltages we want between 0 and 1V, in absolute terms.

So what do we take from that? Kirchhoff doesn't always hold. Kirchhoff does not expect that the readings of your voltmeter varies depending on the position of your test gear in relation to your device under test. You need to inspect what you have under Faraday-Maxwell and then reduce it to Kirchhoff, i.e., you have to define things that are not predicted by Kirchhoff before you apply it.

We are accustomed to measuring the open-circuit voltage of secondary of transformers and we do not care about Faraday-Maxwell, but what we are really doing is an implicit reduction to Kirchhoff because we always have the configuration shown in the second picture of this post.
Title: Re: Does Kirchhoff's Law Hold? Disagreeing with a Master
Post by: Arznei on November 13, 2018, 08:15:21 pm
To be honest, I don't understand what all the fuss is about. What Dr. Lewin is pointing out with his experiment is, as he has repeated often by now, basic physics. The most basic definition of a voltage between two points is expressed as the line integral of the electric field from point A to point B. It is obvious that this only works properly as long as the line integral is _independent_ of the path chosen. If it becomes, for whatever reason, dependend on the path chosen you can no longer define a potential function for that electric field. You don't even need physics for that, thats just basic math at this point. With no potential function the term "voltage" becomes imprecise, because it does not take into account the path of measurement.

Now, if you design an experiment to include a changing magnetic field you get curl E != 0 at some area of the experiment making the very definition of voltages and potentials meaningless. There is this very nice paper [1], that has been shared on this forum multiple times, explaining to the last detail how it is that even with perfect measuring tools you will get two different voltage readings between two points. Note that in this paper bad probing is not a problem. The area where the probes are have no magnetic field, meaning there is _no_ voltage induced into the probes. I think this very fact of voltage not beeing defined is what Dr. Lewin wants to bring across, and I can understand him getting frustrated when people continue to disagree with him there. I was scared of this paper at first, but believe me there is no complicated math involved.

Now to the part of KVL not holding any more: it seems people have different understandings of what KVL means. If you know it as "summed up voltages across a loop in a circuit are zero" then _yes_ it does not hold in this case. Because in the experiment shown by Dr. Lewing the voltages do not sum up to zero. And I will trust him when he says that at MIT they did the experiment with superconductors _which cannot have an electric field inside them_ so there is no voltage across the "ring" that magically makes KVL hold.

If you interpret KVL to say "summed up voltages across a loop in a circuit sum up to the magnetically induced voltage" then this version of KVL holds. It is not nearly as practical as the first one though. You can, for example, not use it in a schematic. Because a schematic does not define the physical location of a component - and therefore, you don't know where the wires are and you don't know what the induced EMF of the loop will be.

Now if you are an engineer, most often you will not deal with "real physical devices" but you will _model_ them to make a equivalent circuit. That is where the voltage source, that many people want to add to the circuit, comes into play. Yes, as long as you are careful with your circuit you can model the effects of EMF into a voltage source. But the very fact people start to talk about "bad probing" all the time shows why this is now only a model of the real experiment. If this were a voltage source it wouldn't matter how I place my probes. They will always read the same voltage. With an EMF they do not. The concept of "bad probing" is that the measurements depend on the way you place your probes. Hence, the voltage you get is *path dependent*

It is of course a common problem, that a model of reality will fail at some point. So to still work with your model, you need to _know how and why_ it can fail. So, as long as you know what you do you can use KVL, but you need to keep in mind that it will only work as long as your actual physical device is carefully designed to match your model (or the other way around).

[1] http://www.phy.pmf.unizg.hr/~npoljak/files/clanci/guias.pdf (http://www.phy.pmf.unizg.hr/~npoljak/files/clanci/guias.pdf)
Title: Re: Does Kirchhoff's Law Hold? Disagreeing with a Master
Post by: ogden on November 13, 2018, 08:17:16 pm
However, if we change the position of the voltmeter and of the leads we are going to have whatever voltages we want between 0 and 1V, in absolute terms.

No. We can't measure whatever voltage we want between 0 and 1V. You shall check model of ring split ring into 10 parts or 4 parts each having 1/4V and 1/4R accordingly. BTW we talked about such case in this thread already.

Quote
So what do we take from that?

That you shall double-check your theory.

Quote
Kirchhoff doesn't always hold.

I'm afraid that you did not prove that.
Title: Re: Does Kirchhoff's Law Hold? Disagreeing with a Master
Post by: Sredni on November 13, 2018, 08:32:44 pm
Quote
No. We can't measure whatever voltage we want between 0 and 1V.

Almost right. If the emf is 1V, with equal resistances in the loop you can measure any voltage you want between -0.5V and +0.5V.
This goes from intercepting the whole flux in one sense to intercepting the whole flux in the opposite sense.

Quote
You shall check model of ring split ring into 10 parts or 4 parts each having 1/4V and 1/4R accordingly. BTW we talked about such case in this thread already.

Trying to localize in lumped components an inherently distributed phenomena will lead you to contradictions. Trust me, I know because I too did it, before seeing the light. :-)
Title: Re: Does Kirchhoff's Law Hold? Disagreeing with a Master
Post by: ogden on November 13, 2018, 08:33:23 pm
Now to the part of KVL not holding any more: it seems people have different understandings of what KVL means. If you know it as "summed up voltages across a loop in a circuit are zero" then _yes_ it does not hold in this case. Because in the experiment shown by Dr. Lewing the voltages do not sum up to zero.

Experiment of the Dr.Lewin does not show it (voltages do not sum up to zero). It is explained in the paper you BTW refer to: http://www.phy.pmf.unizg.hr/~npoljak/files/clanci/guias.pdf (http://www.phy.pmf.unizg.hr/~npoljak/files/clanci/guias.pdf).

Quote
And I will trust him when he says that at MIT they did the experiment with superconductors _which cannot have an electric field inside them_ so there is no voltage across the "ring" that magically makes KVL hold.

We are doing experiment with superconductor loop and single, lets' say 100 Ohm resistor. What will be EMF voltage on particular resistor during experiment?
Title: Re: Does Kirchhoff's Law Hold? Disagreeing with a Master
Post by: ogden on November 13, 2018, 08:40:38 pm
Almost right. If the emf is 1V, with equal resistances in the loop you can measure any voltage you want between -0.5V and +0.5V.

:palm:

Prove it. Download spice (LTspice) and try to model it with more than few lumped elements.

Quote
Trying to localize in lumped components an inherently distributed phenomena will lead you to contradictions. Trust me, I know because I too did it, before seeing the light. :-)

Try me. Please explain  why your model of two batteries and two resistors is OK to show that you read 0V, but mine with 10x0.1V batteries and 10x0.1R resistors (https://www.eevblog.com/forum/chat/does-kirchhoffs-law-hold-disagreeing-with-a-master/msg1960373/#msg1960373) is not?

Title: Re: Does Kirchhoff's Law Hold? Disagreeing with a Master
Post by: Arznei on November 13, 2018, 08:46:31 pm
Yes, he did show exactly that. Open up a physics book of your choice and it should tell you the KVL in presence of magnetic fields is that the sum of all voltages equals the induced EMF. At least from what I can tell that is also what any professor at university taught me.

If you take a ring of superconducting material and connect it to a resistor at one side, the voltage across the resistor will read whatever is appropriate according to faraday. If, however, you measure across the superconducting ring you will read 0 Volts.
Title: Re: Does Kirchhoff's Law Hold? Disagreeing with a Master
Post by: ogden on November 13, 2018, 08:50:14 pm
If you take a ring of superconducting material and connect it to a resistor at one side, the voltage across the resistor will read whatever is appropriate according to faraday. If, however, you measure across the superconducting ring you will read 0 Volts.

Wait... Please explain. Im afraid that I do not follow you here. Are you saying that if I route my test leads on resistor side, I will read voltage whatever is appropriate according to Faraday, but when I place my test leads on superconducting ring side, I read 0V?
Title: Re: Does Kirchhoff's Law Hold? Disagreeing with a Master
Post by: Arznei on November 13, 2018, 08:51:37 pm
Yes.

Edit: Now I'm sorry, this is not a chat but a forum so i should probably explain myself a little. What you get from faraday is the voltage *across the closed loop*. So thats Superconducting Ring + Resistor. From the resistance of the total loop (which is 0R for super conductor + resistance of resistor) you calculate the the current. Now take the resistance of the super condcutor, which is 0, and multiply it with the current. You will get 0V across the super conductor.
Title: Re: Does Kirchhoff's Law Hold? Disagreeing with a Master
Post by: Sredni on November 13, 2018, 09:04:33 pm
"Prove it"

That's what Lewin did. See his video "Science and believing are different things"  and follow his mesh analysis.
He is doing it with two voltmeters outside the main loop where B is changing. As long as you are outside you can always see two meshes with one of the voltmeter inside: one that do not contain the B field, and another one that contains it. As far as the B-encircling mesh go, in one case you have a positive orientation passing trought the small resistor, in the other one a negative orientation passing trough the big resistor. And this gives rise to the different voltages shown by the two external voltmeters.

But you can expand on that and bring the voltmeter inside the B-encircling loop. In this case for the inner voltmeter you will see two meshes that are both inside the B field: one positively oriented passing through the small resistor and the other negatively oriented passing through the big resistor. Depending on what fraction of the area is intercepted in the small or big R mesh, you end up with a different voltage reading.

You should interpret the square mesh as conceptual diagram, the actual area intercepted depends on the path of the probes in the real wold, but still goes from intercepting all the B-varying region in one sense, to intercepting all the B-varying region in the other. In between you get all the values of voltage comprised between the reading of the voltmeter on the left and the reading of the voltmeter on the right. All of this from the same two points!

Do the math, solve the circuit and you will see the light.

Now, I understand the urge to choose, among all these values the one that makes you feel better, namely the one from a path the equally splits the area (I should say the normal flux to be precise) between the two meshes, but that's just an arbitrary decision.

EDIT to answer the question posted below: removed another extra space at the bottom.
Also to clarify: I have zero batteries in my circuit, just the emf. And the two resistors are lumped component part of the physical circuit, not a tentative to model a distributed component.
As for your attempt to model a distributed circuit (where voltage is non single-valued) with a lumped circuit (where voltage is single-valued) to show that voltage is not multi-valued, I guess it's logically flawed.

Solve the mesh circuit with the emf, do the math. You will see the light.
Title: Re: Does Kirchhoff's Law Hold? Disagreeing with a Master
Post by: ogden on November 13, 2018, 09:10:26 pm
Yes.

Edit: Now I'm sorry, this is not a chat but a forum so i should probably explain myself a little. What you get from faraday is the voltage *across the closed loop*. So thats Superconducting Ring + Resistor. From the resistance of the total loop (which is 0R for super conductor + resistance of resistor) you calculate the the current. Now take the resistance of the super condcutor, which is 0, and multiply it with the current. You will get 0V across the super conductor.

Oh, really? When it is convenient, you just forget about EMF.

We take 1.0V battery with 0.001 Ohm  internal resistance, connect it to 1 Ohm resistor. 1A current will flow. Now take the resistance of the battery  which is 0.001, and multiply it with the current. Result is 0.001V. So battery miraculously is not 1V anymore but 0.001V?  :palm:
Title: Re: Does Kirchhoff's Law Hold? Disagreeing with a Master
Post by: ogden on November 13, 2018, 09:12:48 pm
"Prove it"

That's what Lewin did.

TL;DR. I did not find answer to my very simple question I asked:

Please explain  why your model of two batteries and two resistors is OK to show that you read 0V, but mine with 10x0.1V batteries and 10x0.1R resistors is not?

[edit] Dr. Lewin just re-created experiment http://www.phy.pmf.unizg.hr/~npoljak/files/clanci/guias.pdf (http://www.phy.pmf.unizg.hr/~npoljak/files/clanci/guias.pdf). Unformtunately for fanboys of Dr. Lewin, that experiment does not disprove KVL.
Title: Re: Does Kirchhoff's Law Hold? Disagreeing with a Master
Post by: Arznei on November 13, 2018, 09:18:53 pm
Yes.

Edit: Now I'm sorry, this is not a chat but a forum so i should probably explain myself a little. What you get from faraday is the voltage *across the closed loop*. So thats Superconducting Ring + Resistor. From the resistance of the total loop (which is 0R for super conductor + resistance of resistor) you calculate the the current. Now take the resistance of the super condcutor, which is 0, and multiply it with the current. You will get 0V across the super conductor.

Oh, really? When it is convenient, you just forget about EMF.

We take 1.0V battery with 0.001 Ohm  internal resistance, connect it to 1 Ohm resistor. 1A current will flow. Now take the resistance of the battery  which is 0.001, and multiply it with the current. Result is 0.001V. So battery miraculously is not 1V anymore but 0.001V?  :palm:

No, the result of 0.001V is the voltage across the internal resistance of the battery, not the internal voltage source. Which is why the voltage across the terminals of the battery will have dropped by 0.001V, what are you trying to get at now?

I did not forget about EMF, how do you come to that conclusion? Faraday gives us the voltage across the closed loop. You have a loop consisting of 2 components, one is superconducting and will not have any voltage across it. The other is a resistor, which can have a voltage across it if there is current flowing through it. So, naturally, all voltage in the circuit will be across the resistor.

Think of the way this voltage is created: the change of magnetic field puts a force on the charges in the ring+resistor. Now inside a superconductor it doesn't take energy to move a charge (hence resistance of 0R) so your electric field is 0. Inside the resistor it does tage energy to move the charges across, so you will have an electric field of nonzero, therefore a voltage across the resistor.
Title: Re: Does Kirchhoff's Law Hold? Disagreeing with a Master
Post by: ogden on November 13, 2018, 09:33:44 pm
I did not forget about EMF, how do you come to that conclusion? Faraday gives us the voltage across the closed loop. You have a loop consisting of 2 components, one is superconducting and will not have any voltage across it. The other is a resistor, which can have a voltage across it if there is current flowing through it. So, naturally, all voltage in the circuit will be across the resistor.

"all voltage in the circuit will be across the resistor" .. where superconductor loop terminals are connected meaning same voltage will be present on the both ends of the loop as well. So you do not measure different voltages by manipulating with test leads, this is dumb. Actual effect of test lead placement is explained in the document (http://www.phy.pmf.unizg.hr/~npoljak/files/clanci/guias.pdf) linked here so many times already.
Title: Re: Does Kirchhoff's Law Hold? Disagreeing with a Master
Post by: Arznei on November 13, 2018, 09:48:41 pm
I did not forget about EMF, how do you come to that conclusion? Faraday gives us the voltage across the closed loop. You have a loop consisting of 2 components, one is superconducting and will not have any voltage across it. The other is a resistor, which can have a voltage across it if there is current flowing through it. So, naturally, all voltage in the circuit will be across the resistor.

"all voltage in the circuit will be across the resistor" .. where superconductor loop terminals are connected meaning same voltage will be present on the both ends of the loop as well. So you do not measure different voltages by manipulating with test leads, this is dumb. Actual effect of test lead placement is explained in the document linked here so many times already.

No, this is precisely what this is all about. If you measure at the same two points of a circuit you can get different readings on a voltmeter in the presence of an EMF. The reason this is the case and is *not dumb* is that in the presence of EMF the electric field is not a conservative vector field. Your voltage will vary depending on which path you take.

You want to know the voltage across the superinductor, you integrate along a line through the superconductor. Your voltage will be zero, because inside an ideal conductor the electric field is always zero.

You want to know the voltage across the resistor, you integrate along a line through the resistor. Your voltage will not be zero.

I hope we agree, that the voltage induced by the changing magnetic field will be across the resistor. Take any textbook, look up the introdcution to magnetic indcution. You will find an "almost closed loop" split up at some point and a resistor attached to that "split". Every textbook will tell you the voltage across the resistor is the closed-loop-voltage as dictated by faraday.

Now assuming for a second that the same voltage is also present across the superinductor. In that case the "closed-loop" voltage will be zero. That directly contradicts faradays law.
Title: Re: Does Kirchhoff's Law Hold? Disagreeing with a Master
Post by: ejeffrey on November 13, 2018, 11:14:00 pm
You can't do that with changing magnetic fields.  Curl-E = -dB/dt, so there is no scalar potential that describes electron motion.  That is what Walter Lewin was showing.  Everything else is just noise.

Yeah, but that "noise" is a very big claim that two points on the same circuit measure differently, he states that as a fact and uses a flawed demonstration to try and prove it. This is why many people have a big problem with this.

There is no way to fix the demonstration that keeps the character because the whole point is that "voltage" is undefined in the circuit in question.  So there is no consistent way to define a measurement of it.  I don't get why this is hard to understand.

Yes, there are always good and bad measurement techniques and you always have to worry about measurement technique causing errors.  But you always assume there is some underlying "true" value you are trying to get at.  Here, the "flawed" measurement is actually intrinsic to the problem.
Title: Re: Does Kirchhoff's Law Hold? Disagreeing with a Master
Post by: RoGeorge on November 13, 2018, 11:40:55 pm
First I want to point out that SPICE is for simulating, not for proving.

Second, SPICE is not aware of the electric and magnetic fields.  SPICE is for solving circuits using Kirchhoff, and it can work only after you, the SPICE user, converts the geometry of a real circuit into a lumped circuit, with equivalent coils and capacitors.  You, the user, have the responsibility to take the real circuit where Kirchhoff might fail, and convert it into an equivalent lumped circuit with all kinds of coils and capacitors and transformers, and transmission lines, so SPICE can crunch the numbers using only Kirchhoff and no Maxwell.  It is the SPICE user who must take into considerations not only the geometric size and shapes, but also the permeability and the dielectric properties of the surrounding materials. That is a lot left outside SPICE, and even more, the topology of the lumped circuit needs to be changed when the external conditions change.

A lumped circuit is an equivalent circuit, where you, the user, are responsible for finding the correct topology of coils and capacitors and their values in such a way that Kirchhoff's law can be applied.
https://en.wikipedia.org/wiki/Lumped_element_model

Spice is solving its I/V matrix equations based on Kirchhoff and lumped circuits.  That is why in SPICE you won't find any circuit that does not obey Kirchhoff.

In real circuits, any piece of wire is a capacitor, a coil and a resistor in the same time.  This is crazy, and makes it almost impossible to calculate a lumped equivalent that holds with the real world.  We have some dedicated shapes for capacitors and coils where the formulas for L and C are well known, but that's all.  For a random piece of wire there is no clear formula without fields and material constants, and Maxwell.  If we go into transmission lines or antennas, things goes even more complicated.  There are no antennas in SPICE.  There is no causality in SPICE.  Propagation in lumped circuits is considered instantaneous, which in the real world is not true.

If we go further, let's say to calculate the beam of a particles accelerator, SPICE can not help.  Spice does not know Maxwell, and does not know relative speeds and Special Relativity, SPICE knows only Kirchhoff.

If we go even further, for satellites working in different gravitational fields, SPICE is again unaware of General Relativity.  It is us who need to take Special or General Relativity into consideration, and invent a lumped circuit in such a way that we capture all these effects into our imaginary lumped circuit.

Maxwell's equations stand no matter what.  That is why they are so praised.  Maxwell's equations does not need inductors and capacitors.  Kirchhoff does.  Again, Kirchhoff is great for static fields (AKA conservative), but can not be applied for non-conservative fields.  It was not meant for such non-conservative fields.

In order to use Kirchhoff for non-conservative (loosely speaking "variable" fields), we found a workaround, a trick.  We stuff the real circuit with all kinds of imaginary parts, and not only we add those imaginary parts, but the values and the topology of the lumped circuit keeps changing based on external fields and external surrounding of the real circuit, yet our real circuit never changed during this, only the surroundings changed.  We call this stuffed imaginary circuit a lumped equivalent circuit of the real thing.

This fake stuffing we do in order to obtain a lumped circuit is very, very fake and unnatural thing to do, yet we considered it NORMAL, because we are used to it, and because it makes our calculations easier.

In fact, we did it so often that the lumped circuit became to us a second nature, while the real circuit is looked rather as a fussy oddity that needs a lot of care (like electric and magnetic shielding) for it in order to work as expected.  We got so used with our imaginary fake lumped circuits that we now consider reality an anomaly.  We call reality parasitic hum, parasitic inductance coupling, parasitic capacitive coupling, parasitic ground loops, parasitic antennas, parasitic transmission lines, parasitic everything.

Suddenly, reality become for us just a parasitic effect, and we like to consider it that way just to accommodate our distorted lumped circuits with the cruel reality.  That has a lot to do with how human mind works, and from here we are in the realm of human psychology and philosophy.

"A fake repeated over and over becomes reality" <- just an old saying.

Peace!
Title: Re: Does Kirchhoff's Law Hold? Disagreeing with a Master
Post by: ejeffrey on November 14, 2018, 12:27:27 am
To confirm what RoGeorge said:

Once you get to the point you can't make a sufficiently accurate lumped element circuit model, you have to resort to field solvers such as Sonnet, ADS/Momentum, HFSS, or COMSOL.  Those model what is actually going on in a geometrical model of your circuit.  You can't extract a voltage from those.  I actually tried this once indirectly -- I was trying to make a nice heat map of a crosstalk simulation result.  When it had weird discontinuities I looked at what I was doing and realized it made no sense.
Title: Re: Does Kirchhoff's Law Hold? Disagreeing with a Master
Post by: ogden on November 14, 2018, 02:22:39 am
I did not forget about EMF, how do you come to that conclusion? Faraday gives us the voltage across the closed loop. You have a loop consisting of 2 components, one is superconducting and will not have any voltage across it. The other is a resistor, which can have a voltage across it if there is current flowing through it. So, naturally, all voltage in the circuit will be across the resistor.

"all voltage in the circuit will be across the resistor" .. where superconductor loop terminals are connected meaning same voltage will be present on the both ends of the loop as well. So you do not measure different voltages by manipulating with test leads, this is dumb. Actual effect of test lead placement is explained in the document linked here so many times already.

No, this is precisely what this is all about. If you measure at the same two points of a circuit you can get different readings on a voltmeter in the presence of an EMF. The reason this is the case and is *not dumb* is that in the presence of EMF the electric field is not a conservative vector field. Your voltage will vary depending on which path you take.

You want to know the voltage across the superinductor, you integrate along a line through the superconductor. Your voltage will be zero, because inside an ideal conductor the electric field is always zero.

You want to know the voltage across the resistor, you integrate along a line through the resistor. Your voltage will not be zero.

Ok. I am still trying to connect theory to my real-world knowledge of transformers. Dr. Lewin do not use superconductor in his transformer. So we can take transformer w/o superconductor as well - AC mains transformer. So you say that my AC mains transformer with 0.1 Ohms secondary winding does not actually output 12VAC that I measure with my (10MOhm input) RMS multimeter?  :-//
Title: Re: Does Kirchhoff's Law Hold? Disagreeing with a Master
Post by: ogden on November 14, 2018, 02:57:56 am
ogden, first I want to point out that SPICE is for simulating, not for proving.

Second, SPICE is not aware of the electric and magnetic fields.

I did not ask to prove Maxwell's equations using SPICE  :palm: Before you even consider to spread your wizdom, you really shall read what we actually were talking about.
Title: Re: Does Kirchhoff's Law Hold? Disagreeing with a Master
Post by: Berni on November 14, 2018, 06:50:17 am
There is another way of looking at it. Circuits analysis calls things like this a over-defined circuit. In the real world these circuits don't happen but can happen in diagrams with ideal components.

You don't even need magnetic fields to create such a circuit. Lets go break Kirchhoffs law again! :box:

For example this is an over-defined circuit:
(https://www.eevblog.com/forum/chat/does-kirchhoffs-law-hold-disagreeing-with-a-master/?action=dlattach;attach=571382;image)

What is the voltage across A and B? Well looking from one side its 1V, looking from the other side is 2V. And look Kirchhoffs voltage law is wrong again! The voltages don't add up anymore.

What happens if you do this in real life? You just get lots of current and the 1V that Kirchhoffs is missing to make it work is in the internal resistances of the voltage sources and wires. This is essentially a battery charger circuit.

Okay okay everyone knows you can't just parallel together voltage sources or series together current sources. That's like dividing by zero in math. So lets take a different circuit then:
(https://www.eevblog.com/forum/chat/does-kirchhoffs-law-hold-disagreeing-with-a-master/?action=dlattach;attach=571388;image)

When the switch is open its pretty easy to solve the voltages everywhere. A to B is 1V and C to D is 0V (Assuming the capacitor was not given any energy upon its creation in the universe). But then lets close the switch and ask what are the voltages at that moment.

So lets look at it from the left side first:
Voltage source is putting 1V there so A to B must be 1V. And switch is closed so A=C and B=D. So in that case the voltage between C and D is also 1V. Solved

Now lets try solving it from the right side:
If we know the voltage across the capacitor then we can know the voltage between C and D. We do have a formula for the voltage on a capacitor:
(https://wikimedia.org/api/rest_v1/media/math/render/svg/bbfc2bb45579b0a0c8e1604bd898ca18871c6909)

So the 1/C part is simple, we know its 1uF so that works out to 1e6. We also know V(t0) is 0V. All we need is the integral of the current. Since the switch has been closed for 0 seconds means the integral is also 0 so from this we conclude the voltage on C and D is 0V....wait didn't we say it was 1V before?Aha! Dr. Lewin is on to something, it does matter from what direction you look at it!

Alright fine the switch basically didn't do anything because no time has passed. Lets fix out question a bit then "What is the average voltage on the nodes in the span of 1ms of the switch closing". Okay now we are integrating from t=0 to t=0.001 .This time we do need to calculate the current, since this is one loop this is easy. We just sum up all the voltages and resistances and use Ohms law. So we get I = 1V / 0 Ohm ... yeah we can't divide by zero so the current is undefined. So the integral is undefined. So the voltage on C and D is also undefined. Okay fine we could say the current is infinite since approaching division by zero limits towards infinity. Well then the integral is also infinite and we find C and D have infinitely large voltage. But as soon as we introduce a resistance of not 0 in there we can calculate it just fine and it becomes a mathematically valid circuit.

This is the same as trying to solve:
x2 = -3
x = ?
Yes i know this has a complex number answer, but when doing DC circuit analysis what does a current of 2+j3 Amps look like in DC?

Circuit analysis methods break when you introduce these over-defined circuits, its not just SPICE that will throw a angry error message at you for trying to simulate one. Analyzing it by hand simply gives you weird results like 0, undefined or infinity in places where there should be a sensible number, or you get multiple results for the same voltage or current depending on what path you calculate.

Dr. Lewins circuit is also such a circuit because he is forcing a current around while the resistors try to define there own current. Its not only Kirchhoffs law that breaks in such circuits, you can break many other laws since the math simply does not come together.
Title: Re: Does Kirchhoff's Law Hold? Disagreeing with a Master
Post by: RoGeorge on November 14, 2018, 09:51:54 am
ogden, first I want to point out that SPICE is for simulating, not for proving.

Second, SPICE is not aware of the electric and magnetic fields.

I did not ask to prove Maxwell's equations using SPICE  :palm: Before you even consider to spread your wizdom, you really shall read what we actually were talking about.

I think SPICE was used in the previous pages to prove Kirchhoff holds, you used it in page 4, but maybe I misunderstood why you used it there.  My apologies if it was so.

Anyway, I shouldn't have started a 2AM rambling speech with a name, in the first place.  Name removed.
Title: Re: Does Kirchhoff's Law Hold? Disagreeing with a Master
Post by: Sredni on November 14, 2018, 04:30:56 pm
@ogden

you probably missed my answer because I added it to my previous post to keep the number of posts down.
Here it is again:

As for your attempt to model a distributed circuit (where voltage is non single-valued) with a lumped circuit (where voltage is single-valued) to show that voltage is not multi-valued, I guess it's logically flawed.
Solve the mesh circuit with the emf, do the math. You will see the light.

So to avoid wasting this post space with just a repetition, let me add this

Forget the circuit with two resistors and the voltmeter. That appears to be too complex.
Think of just two voltmeters hooked up to form a loop. Place the coil inside the loop. The instruments will measure voltage in opposition of phase with amplitudes partitioned according to the ratio of their internal impedance (suppose one with 9 Mohm impedance, the other with 1 Mohm impedance at the frequency of the test signal - ok to have decent reading with homelab created magnetic field we'd probably need far lower impedences, but consider this to be a thought experiment).
So, which one is probing 'the right way'? (please don't say "the right one" :-) )
Are they both wrong? And yet it checks out: the sum of the voltages around the loop is not zero, it's the emf.
Engineers like to think that to be a 'generalized' KVL. The sum of voltages and emfs around a loop gives zero, but somehow some of them freak out when they see two voltmeters, attached to the very same two endpoints, reading two different values (that add up exactly to what they expect!!!) and call that 'bad probing'. Go figure.
Title: Re: Does Kirchhoff's Law Hold? Disagreeing with a Master
Post by: Vtile on November 14, 2018, 05:28:05 pm
This is a good thread to ask. Does anyone know the original publication of these two Kirchoff laws 1st and 2nd, not voltage and current laws as usually wrongly tittled.

There is probably also pointed out that this our typical "ideal" circuit analysis way of drawing circuits is also ideal and centralized model that brakes down miserably when you go smaller in time domain, or go further in component size (ie. cross continent powerlines).
Title: Re: Does Kirchhoff's Law Hold? Disagreeing with a Master
Post by: ogden on November 14, 2018, 05:44:53 pm
As for your attempt to model a distributed circuit (where voltage is non single-valued) with a lumped circuit (where voltage is single-valued) to show that voltage is not multi-valued, I guess it's logically flawed.
Solve the mesh circuit with the emf, do the math. You will see the light.

As you disagree with my model where between any two points of 1Ohm ring where 1A current is running, there's always is 0V, you shall demonstrate mesh circuit with emf, do the math to prove that I am wrong and prove that you are right:

Almost right. If the emf is 1V, with equal resistances in the loop you can measure any voltage you want between -0.5V and +0.5V.

Show me the light you so often refer to. - If you can.
Title: Re: Does Kirchhoff's Law Hold? Disagreeing with a Master
Post by: Berni on November 14, 2018, 05:45:19 pm
Yes the voltmeters would read different values in a 9 to 1 ratio because the EMF voltage is in the wires used to connect them.

But along the way i found another fun way to break Kirchhoffs law without using any magnetic field:
(https://www.eevblog.com/forum/chat/does-kirchhoffs-law-hold-disagreeing-with-a-master/?action=dlattach;attach=571631;image)

Here the orange wire is made out of constantan(55% Cu, 45% Ni) while other wires are made out of copper. This creates a thermocouple joint at both ends of the constantan wire. If we now heat the indicated corner to 100°C while leaving all other parts of the circuit at ambient temperature we get the shown readings on both voltmeters.

3.03mV + 0.34mV = 3.37mV

And Kirchhoffs voltage law is broken again because we should be getting 0V when we sum the voltages in the loop. We could again fix it by adding a lumped thermocouple model in the corners where the missing hidden voltage source is located, but if we are not adding any lumped elements to the circuit to describe the magnetic properties of the wire then i suppose we also wouldn't add lumped elements to describe thermocouple effects. The thermoelectic effect after all does not need electric fields to push electrons around, just like the magnetic field does not (But magnetic fields do sort of create 'virtual electric fields' trough the effects of special relativity).
Title: Re: Does Kirchhoff's Law Hold? Disagreeing with a Master
Post by: free_electron on November 14, 2018, 06:16:44 pm
Current induced in the measuring wires.
it would be interesting to try this with untrimmed surface mounted film resistors
Those thru-hole resistors are most likely spiral cut ( which forms inductors by itself ... )
Title: Re: Does Kirchhoff's Law Hold? Disagreeing with a Master
Post by: Sredni on November 14, 2018, 06:50:03 pm
Yes, current induced in the measuring wires. But the point is that now we read two different values from the same points.
This is not a measurement issue, it is exactly what is predicted by the theory. And in fact, the values confirm that the sum of the voltage read around the loop do not give zero. They give the value of the emf.

Even with the original Lewin circuit you had current induced in the measuring wires. It was deemed negligible because the resistances in the circuit were much lower than the multimeter's impedances. But still, despite the probes not linking the flux there was always a mesh that enclosed the whole area with variable field. In one case with a positive orientation and containing a high valued R, in the other with a negative orientation and containing a low valued R. This is what gives rise to the different reading.

When you take the probes path inside the variable flux area, you end up partitioning it and now both meshes are linking the flux with opposite orientation and variable proportion (determined by how you place the probes). You can then get a reading of all voltage values included between the extreme values of 0.9V and -0.1V depending on which of the two meshes cuts more flux. From the same two endpoints!

Now, it seems that someone would like to single out, among the infinite values of V, the one where the contribution from the two meshes cancel out, put it on a pedestal and worship it as the "true voltage". I have no problems with that. You can find that same value from the same two endpoints for an infinite number of other probe path configurations, no need to twist and shield: just jerrymander the cables so that you get the value you want.

But the point is that voltage is no longer defined in this context.
And that's exactly what the math predicted: if the integral of the E field across any closed path is not zero, then V is not determined by the endpoints only, but also by the shape of the path.
This is not even basic physics. It's basic mathematics.

Title: Re: Does Kirchhoff's Law Hold? Disagreeing with a Master
Post by: ogden on November 14, 2018, 07:00:51 pm
Yes, current induced in the measuring wires.

 :palm:
Title: Re: Does Kirchhoff's Law Hold? Disagreeing with a Master
Post by: Sredni on November 14, 2018, 07:05:00 pm
@ogden

Thou doth facepalm too much.

Solve the circuit shown in Lewin's "Science and believing are different things", do the math.
Now I am no longer convinced you will see the light, but it's still worth a shot.

EDIT to answer the post below: I am not your tutor. Lewin has already solved it in his video, I did check it on my own but I am not wasting my time translating and formatting (I even tried to upload a pic two days ago, but gave up on tinypic and its enter code here and there) it for someone who does not even want to watch a video.
Maybe next week I will tell you a fairytale about an island with a hill and an instrument to measure the energy per unit mass required to move a mass from bottom to top, with and without a curly wind. You might understand what potential really is.


edit: correct mispelled name
Title: Re: Does Kirchhoff's Law Hold? Disagreeing with a Master
Post by: ogden on November 14, 2018, 07:32:24 pm
Solve the circuit shown in Lewin's "Science and believing are different things", do the math.
Now I am no longer convinced you will see the light, but it's still worth a shot.

Trying to shift discussion off-subject?  :-DD

I already did show simulation using lumped elements (10x0.1V EMF sources and 10x0.1R resistance). You disagreed. So now you shall solve the mesh circuit with the emf, do the math - to disprove my approach.

[edit] I remind that all I ask you - prove following words not using  :bullshit: :blah: but solving the mesh circuit with the emf, doing the math:

Quote
No. We can't measure whatever voltage we want between 0 and 1V.

Almost right. If the emf is 1V, with equal resistances in the loop you can measure any voltage you want between -0.5V and +0.5V.
Title: Re: Does Kirchhoff's Law Hold? Disagreeing with a Master
Post by: rfeecs on November 14, 2018, 07:43:10 pm
This is a good thread to ask. Does anyone know the original publication of these two Kirchoff laws 1st and 2nd, not voltage and current laws as usually wrongly tittled.

From here:
https://books.google.com/books?id=a434DAAAQBAJ&pg=PA3&lpg=PA3&dq=translation+of+%E2%80%9CUeber+den+Durchgang+eines+elektrischen+Stromes+durch+eine+Ebene,+insbesonere+durch+eine+kreisf%C3%B6rmige&source=bl&ots=JSjEAnl4cu&sig=REI3wpeFihBgq4PaU6gCX_VTeac&hl=en&sa=X&ved=2ahUKEwjFhOK_0NTeAhXEwMQHHWYkC90Q6AEwAnoECAgQAQ#v=onepage&q&f=false (https://books.google.com/books?id=a434DAAAQBAJ&pg=PA3&lpg=PA3&dq=translation+of+%E2%80%9CUeber+den+Durchgang+eines+elektrischen+Stromes+durch+eine+Ebene,+insbesonere+durch+eine+kreisf%C3%B6rmige&source=bl&ots=JSjEAnl4cu&sig=REI3wpeFihBgq4PaU6gCX_VTeac&hl=en&sa=X&ved=2ahUKEwjFhOK_0NTeAhXEwMQHHWYkC90Q6AEwAnoECAgQAQ#v=onepage&q&f=false)

The citation is note 4:

(https://www.eevblog.com/forum/chat/does-kirchhoffs-law-hold-disagreeing-with-a-master/?action=dlattach;attach=571763;image)

(https://www.eevblog.com/forum/chat/does-kirchhoffs-law-hold-disagreeing-with-a-master/?action=dlattach;attach=571781;image)
Title: Re: Does Kirchhoff's Law Hold? Disagreeing with a Master
Post by: ogden on November 14, 2018, 07:56:01 pm
EDIT to answer the post below: I am not your tutor. Lewin has already solved it in his video

I can't find video where he solves 1 Ohm ring loop problem where as you say - you can read any voltage between -0.5V to + 0.5V. It is not about tutoring. It's about proving that your words are not BS.
Title: Re: Does Kirchhoff's Law Hold? Disagreeing with a Master
Post by: Sredni on November 14, 2018, 08:36:10 pm
You should have read my post more carefully

"EDIT to answer the question posted below: removed another extra space at the bottom.
Also to clarify: I have zero batteries in my circuit, just the emf. And the two resistors are lumped component part of the physical circuit, not a tentative to model a distributed component. "

This is what I was talking about: Lewin's so much debated setup.
Your problem is even worse from the standpoint of spice simulation, because everything is distributed but still, off the top of my head, the voltage between two points forming an angle alpha with the center will be different when you measure it from one side out of the loop |V| = (alpha/2pi) * emf and from the other side of the loop |V| = (2pi - alpha)/2pi * emf (you need to fix the signs).
If you go inside you will end up will all values comprised between these two values (with adjusted signs). It all depends on how you cut the varying flux area.

EDIT
Added formulae, corrected one of the seventyfour typos and also to add:

So, your spice model shows one out of those infinite values, and that should prove what?

Title: Re: Does Kirchhoff's Law Hold? Disagreeing with a Master
Post by: Berni on November 14, 2018, 08:54:58 pm
Yep you can get any of the infinitely many values by rearranging the wires. But then the model should capture that so that you can then solve the circuit for that particular one.

The arrangement of wires i am interested in is the one where both probing wires summed together produce a EMF voltage of 0V. This means that the end where the voltmeter is at is at the same voltage as the two points we are trying to measure since this way the probes are not affecting the measurement. This collapses the circuit down to a single solution for the voltage.
Title: Re: Does Kirchhoff's Law Hold? Disagreeing with a Master
Post by: Sredni on November 14, 2018, 09:08:18 pm
@Berni

Yep, and that's totally fine with me. I know that's what some called "correct probing", but you should know that you are just deceving yourself if you think that voltage has a physical meaning outside the context of the particular path you have found. As I wrote before there is no need to twist and shield your cables - just jerrymander them until the instrument shows the value you like (which for the symmetric setup of Lewin's circuit will be the average of the values, considering the signs, so... 0.4V IIRC).

The fact that that is not the only voltage is not only a matter of definition. In my fairytale about the hill, there are Conservatives and SUPERDEMOcrats that have to ship medicine to their relatives at the top of the hill. Conservatives laugh at SUPERDEMOcrats, thinking that they believe the height of the hill changes with wind, but they end up all dead because their conservative "true value" of the energy required to shoot the package up is either too high or too low to safely reach the destination when there is net wind along the path.

PS
no politics please, it's just that the names were so fitting...
Title: Re: Does Kirchhoff's Law Hold? Disagreeing with a Master
Post by: ogden on November 14, 2018, 09:31:46 pm
Your problem is even worse from the standpoint of spice simulation, because everything is distributed but still, off the top of my head, the voltage between two points forming an angle alpha with the center will be different when you measure it from one side out of the loop |V| = (alpha/2pi) * emf and from the other side of the loop |V| = (2pi - alpha)/2pi * emf (you need to fix the signs).

Where did you lost 1 Ohm distributed resistance? You also forgot that loop we are talking about is electrically short!

Let's view at obvious case where alpha equals PI. As two identical halves of the loop we have one +0.5V and one -0.5V EMF source and two 0.5 ohm resistors connected in parallel series to each of them. How do we measure any voltage here between those two points?

- By "scientifically" manipulating voltmeter leads? ;)
Title: Re: Does Kirchhoff's Law Hold? Disagreeing with a Master
Post by: Sredni on November 14, 2018, 10:03:39 pm
Where did I lose the resistance? I didn't. If the emf is 1V, the current is 1V/1ohm = 1amp.
Why do you think the measured voltage should depend on the value of the resistance? Is it not a uniform resistance loop?
What do you expect by raising the total resistance to 2 ohms?

EDIT
also with 1V emf, and the measure points on a diameter, you will measure -0.5V on one side, from outside the loop, + 0.5V from the other side, still outside the loop, and any value you wish in between when you go inside the loop, depending on how much of the flux you cut in the two parts you are partitioning the area.
If you want to read the 0V you like so much, you place your inner voltmeter along a diameter. Or, you can go yin-yang.

EMF is still 1V, why should that change?
Title: Re: Does Kirchhoff's Law Hold? Disagreeing with a Master
Post by: ogden on November 14, 2018, 10:18:32 pm
Where did I lose the resistance? I didn't. If the emf is 1V, the current is 1V/1ohm = 1amp.
Why do you think the measured voltage should depend on the value of the resistance? Is it not a uniform resistance loop?
What do you expect by raising the total resistance to 2 ohms?

Just realized mistake in a hurry in previous post, corrected. - Each half of the loop is 0.5V EMF generator having 0.5 Ohm internal resistance, so obviously 0.5 Ohms is connected in series to 0.5V EMF source.

Quote
also with 1V emf, and the measure points on a diameter, you will measure -0.5V on one side, from outside the loop, + 0.5V from the other side

Only if loop is not shorted. We talk about shorted loop!
Title: Re: Does Kirchhoff's Law Hold? Disagreeing with a Master
Post by: Sredni on November 14, 2018, 10:35:10 pm
"Each half of the loop is 0.5V EMF generator"

EMF on half loop?
What's next? Magnetic monopoles?
I am sorry, I can't help you fill those gaps.
Continue facepalming.

EDIT after your following post: my last remark, I'll leave you to emoticons after this.
When your endpoints are on a diameter you have +0.5V measuring outside on the left (whatever); -0.5 V measuring on the other side, still with voltmeter outside the loop. Total, following the signs +0.5 - (-0.5) = 1V. That's the emf. It does not show up as a lumped generator, it's distributed. Your error is trying to localize it and compute fractions of it when you consider only part of the circuit.
Title: Re: Does Kirchhoff's Law Hold? Disagreeing with a Master
Post by: ogden on November 14, 2018, 10:53:10 pm
"Each half of the loop is 0.5V EMF generator"
EMF on half loop?

What's next? Magnetic monopoles?
I am sorry, I can't help you fill those gaps.
Continue facepalming.

LOL. Nice try. When it's convenient, you just pretend that you do not understand what I am talking about - partition (half) of the closed loop. Now suddenly your formula "|V| = (alpha/2pi) * emf" giving 0.5V with alpha=PI and emf=1V is not true anymore as well. [edit] In same post I say "we talk about closed loop".
Title: Re: Does Kirchhoff's Law Hold? Disagreeing with a Master
Post by: ogden on November 14, 2018, 11:27:02 pm
Just watched "Science and believing are different things", did the math. Left meter reads -900mV, right 100mV. In both - calculation and "lumped simulation". Funny that LTspice does not scream at me ;)

(https://www.eevblog.com/forum/chat/does-kirchhoffs-law-hold-disagreeing-with-a-master/?action=dlattach;attach=571904;image)

[Edit] agrees to paper http://www.phy.pmf.unizg.hr/~npoljak/files/clanci/guias.pdf (http://www.phy.pmf.unizg.hr/~npoljak/files/clanci/guias.pdf)  as well.
Title: Re: Does Kirchhoff's Law Hold? Disagreeing with a Master
Post by: Sredni on November 15, 2018, 05:28:37 am
Just watched "Science and believing are different things", did the math. Left meter reads -900mV, right 100mV. In both - calculation and "lumped simulation". Funny that LTspice does not scream at me ;)

That's amazing! I did not know Spice could do that!
Can you please highlight on the circuit the two (2) nodes that give two different values of voltage between them?
Maybe one is GND, so what is the other node?
Title: Re: Does Kirchhoff's Law Hold? Disagreeing with a Master
Post by: RoGeorge on November 15, 2018, 07:43:34 am
The explanation video promised by Prof. Walter Lewin.

https://www.youtube.com/watch?v=AQqYs6O2MPw (https://www.youtube.com/watch?v=AQqYs6O2MPw)
Title: Re: Does Kirchhoff's Law Hold? Disagreeing with a Master
Post by: KL27x on November 15, 2018, 09:39:13 am
So... Dr. Lewin just discovered impedance.

And he dropped his two scope, mobius strip with induction battery and improper probing experiment.

What a waste of time.

I still don't get why he says Kirchoffs works when the voltmeter is attached. Unless he is saying that the voltmeter has a resistance (conductance), and thus the voltmeter is what is dropping the voltage. At T approaches zero and the inductor's impedance is higher, yeah, that could be true and you could say that the voltemeter is part of the circuit; but between T= 0 and T= Tsaturation, most of the current is through the inductor and the voltage is dropped across the inductor, and the voltmeter will be pretty much not significant part of the circuit. I don't get the point he was making with that.
Title: Re: Does Kirchhoff's Law Hold? Disagreeing with a Master
Post by: bd139 on November 15, 2018, 10:04:22 am
Just watched all three videos.

These are all abstraction tools. Some tools you pick for some jobs. Some tools you pick for others.

One thing you do is pick the right tool for the job. KVL works fine for the use cases we use.

Two things you don't do are claim people are religious and fuck up your test environment and then wiggle out of it.
Title: Re: Does Kirchhoff's Law Hold? Disagreeing with a Master
Post by: ogden on November 15, 2018, 10:14:10 am
Can you please highlight on the circuit the two (2) nodes that give two different values of voltage between them?

So you are believer.

These are all abstraction tools. Some tools you pick for some jobs. Some tools you pick for others.

Well said. He integrate over time to disprove Kirchhoff's Law but to show special case when Kirchhoff's Law holds, he uses voltmeter with high time resolution :)

What a waste of time indeed.
Title: Re: Does Kirchhoff's Law Hold? Disagreeing with a Master
Post by: KL27x on November 15, 2018, 11:18:01 am
Kirchoff reduced his observations empirically and mathematically to produce kirchoffs laws. Later, someone studied a curious thing dubbed impedance/reactance, and Faraday was able to correlate impedance with magnetic flux, mathematically, using integrals.

Dr. Lewin used these equations of giants to re-discover impedance and has amazed several college students.
Title: Re: Does Kirchhoff's Law Hold? Disagreeing with a Master
Post by: bd139 on November 15, 2018, 11:34:58 am
That's basically it.

Let's look at the insignficance of the entire point too....

Avoiding worrying about this is the most powerful tool in your arsenal. Avoiding any problem is.

Extrapolating my point earlier, now I have a few minutes, the whole point of the "tree of knowledge" is that you pick a branch which allows you to solve your problem without having to understand the underlying abstractions if you can help it. Your objective is to solve the problem and move on. How many people in the modern world require knowledge of simple things like integrals, derivatives and logarithms? Not many. When it comes to applied knowledge, we almost never need to care about Maxwell's Equations or vector calculus. And thank goodness for that. They're horrid. I spent a good while scratching my head over these going through Feynman's books.

An example. Just last week I was building an opamp integrator. I couldn't be bothered to work it out as an integral (it was late and I'd had way too little coffee that day and the magic integral button on my calculator didn't work) so I picked a higher level abstraction which is a current source (input resistor to non-inverting input) and ideal behaviour of an opamp (lifts output to keep non inverting at zero here). Thus you can solve it using I = C * dv/dt ... I knew I wanted to go 1v/second and had a 1uF cap sitting there staring at me waiting to be used so I knew I. I know input voltage (saturated near rail) so picked an R for that using ohms law. Did maths in head, picked closest value, job done.

Not once did I look at integrals, maxwell, even kirchoff here. Just a rule of thumb which was dervied from this entire stack of knowledge.

At university, many hours did I spend sitting there doing page after page of kirchoff, solving simultanous equations to the point I wrote a linear equation solver on my calculator so I didn't have to do it any more because my arms were so heavy that the thought of doing more made me want to just quit and go and do a BA in arts or something.

And to make a mockery of it all, two points: I never used it. Not even once.  TAOE dedicates about 1/3 of a page to it because that's all the attention it needs in the real world.

So this is an hour+ of my life I have wasted on people arguing about technicalities which change nothing, unless you're a research physicist, which I'm not and most of us aren't.

Lewin: Look my abstraction is more right than yours!

Who gives a shit?
Title: Re: Does Kirchhoff's Law Hold? Disagreeing with a Master
Post by: bsfeechannel on November 15, 2018, 02:41:09 pm
The explanation video promised by Prof. Walter Lewin.

https://www.youtube.com/watch?v=AQqYs6O2MPw (https://www.youtube.com/watch?v=AQqYs6O2MPw)

Case closed.

(https://www.eevblog.com/forum/chat/does-kirchhoffs-law-hold-disagreeing-with-a-master/?action=dlattach;attach=572381;image)

(https://www.eevblog.com/forum/chat/does-kirchhoffs-law-hold-disagreeing-with-a-master/?action=dlattach;attach=572399;image)
Title: Re: Does Kirchhoff's Law Hold? Disagreeing with a Master
Post by: bd139 on November 15, 2018, 02:46:59 pm
Well duh  :-DD

(which is the whole point here)
Title: Re: Does Kirchhoff's Law Hold? Disagreeing with a Master
Post by: bsfeechannel on November 15, 2018, 03:11:43 pm
Duh, indeed.

(https://www.eevblog.com/forum/chat/does-kirchhoffs-law-hold-disagreeing-with-a-master/?action=dlattach;attach=572420;image)
Title: Re: Does Kirchhoff's Law Hold? Disagreeing with a Master
Post by: Sredni on November 15, 2018, 08:49:36 pm
Can you please highlight on the circuit the two (2) nodes that give two different values of voltage between them?
So you are believer.

I am sorry, it seems you do not have sarcasm in Latvia.
Let me rephrase it:

Show the id of the two nodes (not three or four, two) whose potential difference gives both 0.1V AND 0.9V.
If you can't, because you need at least 3 nodes, or more likely 4, it's because Spice can only give you an answer with a single-valued potential.

And if your aim was to find an equivalent lumped circuit where you could see 0.1V and 0.9V you should have not bothered to add all those inductances. Just uses a lumped secondary coil giving you 1V and put the two resistor sin series. There you go: in this case you need three nodes to show two voltages.

Please re-post your schematics indicating with TWO arrows the TWO nodes (not two pairs of nodes, just two nodes) that give you two separate potential differences at the same time.
My bet is that you cannot.


Quote
What a waste of time indeed.

You simply did not understand it.
It's not a crime, after all this is very counterintuitive stuff, like everything that has its roots in relativity.
Do not give up, study a bit more and you will see that Lewin is indeed right.


PS
Wanna see something nice: put two batteries with different voltage in parallel in spice. You will find that spice will give a single-valued voltage, namely the average of their voltages. Why? Because it cannot handle multivalued potentials. So it assumes a finite internal resistance to avoid impossible systems of equations.
I guess it's written in the manual.

But nowadays, who read the... fine manual anymore?
Title: Re: Does Kirchhoff's Law Hold? Disagreeing with a Master
Post by: ogden on November 15, 2018, 10:29:16 pm
And if your aim was to find an equivalent lumped circuit where you could see 0.1V and 0.9V you should have not bothered to add all those inductances.

My aim was to show opposite. -That miraculous reading of two voltages in single point is just bad probing and it can be explained using simplified model of  EMF sources and loads. BTW Berni showed more detailed model (https://www.eevblog.com/forum/chat/does-kirchhoffs-law-hold-disagreeing-with-a-master/msg1945312/#msg1945312) of what actually happens. Where do you see inductances? You think R1 and R2 in Dr.Lewin's circuit (picture attached) are inductances as well?

Honestly I do not see any reason to continue this forum chat with you. I agree to disagree and stop internet ink waste.
Title: Re: Does Kirchhoff's Law Hold? Disagreeing with a Master
Post by: RoGeorge on November 15, 2018, 11:23:17 pm
ogden, the "miraculous reading" of two voltages in single point is NOT about bad probing.

Please look at this picture made by bsfeechannel.
bsfeechannel perfectly summarizes the whole debate:

(https://www.eevblog.com/forum/chat/does-kirchhoffs-law-hold-disagreeing-with-a-master/?action=dlattach;attach=572381;image)

The first equation is Kirchhoff's KVL law.  In words, "the sum of all voltages in a closed loop = zero".

In the second equation, the term on the right side of the "=" is the induced EMF.  In words, "the sum of all voltages in a closed loop = induced EMF"

Kirchhoff says the sum of voltages in a loop is zero.
Faraday says the sum of voltages in a loop is EMF.
Contradiction.  Why?

When there is no induced voltages, Kirchhoff is true.
If you have induced voltages in your circuit, Kirchhoff is false.

Q: So, Kirchhoff was stupid, and the Physics is broken?
A: No.  Kirchhoff was a genius, but when he made those laws of him, Kirchhoff was talking only about circuits with NO induced voltages.
Title: Re: Does Kirchhoff's Law Hold? Disagreeing with a Master
Post by: Sredni on November 15, 2018, 11:23:43 pm
@odgen It was a lapsus. Change inductances with resistances and the gist does not change: the lumped circuits you can model in Spice cannot give you two different voltage readings from the very same two points (edit: and that's why Spice did not scream at you). BUT the system in the real world CAN.

As for your model to show two readings, yes, I sent you there just to show where those readings came from: the different flux linkage. Where did you guys think this multivalued potential came from? An alternate universe?
It is written clearly in Faraday's law.

What you call 'bad probing' is an inherent and irremovable characteristic of the physical system. Voltage is no longer 'positional' so if your aim is to create a map of 'voltage' values on the ring, you should know you can have any kind of mapping you want depending on how you place your probes.

(Edit: If your definition of 'true voltage' is what you read when the probe paths partition the area of the loop in order to give cancelling contributes...) ...what happens when the flux is not spatially uniform? You have to follow strange paths in the ortogonal section of the loop area. And if the loop is not circular symmetric, same thing. And if the flux change is relative not only to the module of the B field but also to its spatial distribution? You have to make your probes dance with the field to cancel the contribute of the flux linked by the two inner meshes.

Oh well, I guess you have the right to remain ignorant.
And I start to see why Lewin is giving those copy-pasted comments on youtube. (edit, posted after reading a post below) He was literally drag to post video after video and now people are saying "he's just repeating himself".

edit: typos and additions, as shown
further edit: typos typos typos. They're coming out of the effing walls!
Title: Re: Does Kirchhoff's Law Hold? Disagreeing with a Master
Post by: rfeecs on November 15, 2018, 11:27:28 pm
So in this latest video, Dr. Lewin again says that almost all the textbooks are wrong.  One he specifically calls out is Halliday and Resnick.  So just for fun, what does it say about inductors?  From Halliday and Resnick "Fundamentals of Physics"  9th edition, section 30-8:

Quote
In Section 30-6 we saw that we cannot define an electric potential for an
electric field (and thus for an emf) that is induced by a changing magnetic flux.
This means that when a self-induced emf is produced in the inductor of Fig. 30-13,
we cannot define an electric potential within the inductor itself, where the flux
is changing. However, potentials can still be defined at points of the circuit that
are not within the inductor-points where the electric fields are due to charge
distributions and their associated electric potentials.

Moreover, we can define a self-induced potential difference VL across an
inductor (between its terminals, which we assume to be outside the region of
changing flux). For an ideal inductor (its wire has negligible resistance), the magnitude
of VL is equal to the magnitude of the self-induced emf EL.

If, instead, the wire in the inductor has resistance r, we mentally separate the
inductor into a resistance r (which we take to be outside the region of changing
flux) and an ideal inductor of self-induced emf EL. As with a real battery of emf
E and internal resistance r, the potential difference across the terminals of a real
inductor then differs from the emf. Unless otherwise indicated, we assume here
that inductors are ideal.

So it doesn't attribute anything to Kirchoff's law.  It specifically says we can't define a potential from the E field produced by the inductor's changing magnetic flux.  It says we can still define a voltage outside the changing magnetic field of the inductor and use this as the potential difference across the inductor.  And we can make a "mental" lumped model of the inductor with internal resistance as an EMF in series with a resistor.

None of that sounds criminal.

He's made his point several times over in four or five videos.  At this point he's just repeating himself.
Title: Re: Does Kirchhoff's Law Hold? Disagreeing with a Master
Post by: Berni on November 15, 2018, 11:41:11 pm
After watching the response video I do have to apologies that I missed the point. Yes I agree that Kirchhoffs voltage law is not describing the actual underlying physics. If you need to describe that go and use maxwells equations, I'm sure they work great. The video explains the difference nicely. :-+

However I still don't agree with the demo experiment. But I do agree with some of it. There is indeed 0.9V and 0.1V on those resistors, it's Ohms law after all. Where I stop agreeing is the claim that the mid point of the circle is both at 0.9V and 0.1V at the same time. It's only at those voltages at the ends of the wire where it touches the resistors, but the voltage gradualy changes between the two as you move the measurement point along the wire. So i thing the experiment is misleading in the way it is explained.

The magnetic field is pushing those electrons down the wire because the two are moving in relation to each other. However the electrons want to equalize along the wire so they push back. At some point the two forces balance out and you end up with a certain number of extra electrons towards one end of the wire. More electrons means more charge so the voltage is higher there (actually lower since they are negative charges, but you get the point). If you provide another path for the electrons to equalize towards, such as placing a voltmeter between the two nodes in question they will happily flow towards it, the meter simply senses how eager those electrons are to equalize. Where the tricky part comes is that wires need to carry the electrons to the meter and electrons in these wires are also susceptible to being pushed around by that magnetic field, this creates the extra voltage we measure. The voltage between the two probing points is defined by the difference in charge density and can only be one number. This is the voltage that I refer to as the true voltage between the nodes. Since the probe wires are not supposed to be part of the circuit (but are a nececerry evil to be able to connect the meter) this means they should be used in a way that does not produce extra voltages. These are the voltages that push electrical current trough circuits (even if they are not the direct fault of a electric field)

SPICE is showing these voltages in its results and yes it will throw an over defined circuit error if you parallel two voltage sources due to there being no way for standard circuit analysis methods to resolve that. If it does simulate it fine then the simulator added a parasitic resistance to the voltage source. Often simulators apply some default (but overridable) parasitic values because it makes circuits act closer to what we expect in real life.

Kirchhoffs voltage and current laws are still incredibly useful tools for circuit analysis and they always work for that. It's a matter of the right tool for the job. These circuit analysts methods do a very good job of predicting what would happen in a real circuit. This is what science is about, making theories and then rigorously testing them to the limit with experiments. The ones that match experimental results are considered to be more valid and can then be of incredible use to engineers from all fields to help them predict the performance of there designs before they are built. You don't want to build a bridge just to test if it will collapse or not. Abstractions are great and all but they don't always give the whole picture even if the math works out so nicely (And yes Kirchhoffs laws ride on top of quite a few stacked up abstraction layers, it also relies on these layers to work in a certain way)
Title: Re: Does Kirchhoff's Law Hold? Disagreeing with a Master
Post by: Sredni on November 16, 2018, 12:01:44 am
There is indeed 0.9V and 0.1V on those resistors, it's Ohms law after all.

Well, yes and no.
There is 0.9V if you measure with the voltmeter on one side, and there is 0.1 V if you measure with the voltmeter on the other side of the loop. Even if you place the probe across THE SAME resistor.
Look at the meshes, can you see that in one case you are linking the flux with one orientation inside the mesh with the big resistor, and in the other case you link the flux with the opposite orientation inside the mesh with the small resistor?

Quote
Where I stop agreeing is the claim that the mid point of the circle is both at 0.9V and 0.1V at the same time.

Ay, there's the rub. You still think voltage is a property of the points on the circuit. It is not. Not anymore, it's not!
It depends on the points A and B AND on the PATH.
So you can have both readings at the same time, there is no 'quantum superposition of voltage states' so to speak.

Quote
It's only at those voltages at the ends of the wire where it touches the resistors, but the voltage gradualy changes between the two as you move the measurement point along the wire. So i thing the experiment is misleading in the way it is explained.

You are still under the spell of the positional voltage.
But you are this close to see the light.
Come to the bright side!!!
We literally own the light! [note]

[note] as a matter of fact, when the magnetic flux changes it concatenates a circulating electric field - you do not even need electrons or any kind of matter. And that varying electric field concatenates a variable magnetic field, and... well, it's the bright side, no?


Title: Re: Does Kirchhoff's Law Hold? Disagreeing with a Master
Post by: bd139 on November 16, 2018, 12:04:17 am
In a world of spherical cows, the path does not exist. What then?

The only reason the measurement is different is because the path is different.
Title: Re: Does Kirchhoff's Law Hold? Disagreeing with a Master
Post by: ogden on November 16, 2018, 02:00:47 am
The first equation is Kirchhoff's KVL law.  In words, "the sum of all voltages in a closed loop = zero".

It is attempt to apply Kirchhoff's KVL law to superconductive loop placed in the changing magnetic flux. Kirchoff's law does not hold for such. Do not recall anybody arguing that. Interesting stuff (https://en.wikipedia.org/wiki/Flux_pumping). Excerpt: "An electric current flowing in a loop of superconducting wire can persist indefinitely with no power source."

Debate was about flawed experiment which had cut loop as EMF source and resistor(s) as load. On resistor terminals you can measure voltage and there are no two different values. Also kinda obvious that nobody tries to apply Kirchoff's law to circuit with single element, not to mention shorted battery!
Title: Re: Does Kirchhoff's Law Hold? Disagreeing with a Master
Post by: rfeecs on November 16, 2018, 06:04:12 am
However I still don't agree with the demo experiment. But I do agree with some of it. There is indeed 0.9V and 0.1V on those resistors, it's Ohms law after all. Where I stop agreeing is the claim that the mid point of the circle is both at 0.9V and 0.1V at the same time. It's only at those voltages at the ends of the wire where it touches the resistors, but the voltage gradualy changes between the two as you move the measurement point along the wire. So i thing the experiment is misleading in the way it is explained.

Maybe consider that potential or voltage is a mathematically calculated quantity that doesn't necessarily correspond to something physically real.  In this case, it is not defined at all unless you define a path over which to do the calculation.

So consider real quantities instead:

The current is real.  The current has to be the same everywhere around the loop because of the continuity equation.

The magnetic field is real.  The only magnetic field is in the middle of the loop.  There is no magnetic field anywhere else in this simple two dimensional scenario.  So there is no magnetic field in the wires or in the resistors.

The electric field is real.  The electric field is caused by the changing magnetic field.  There is no field inside the perfect (or near perfect) conducting wires.  The only field is present in the resistors.  As you said, there is a charge distribution that maintains zero field in the wires and enhances the field in the resistors.

Energy is real.  So consider the charge that moves around the loop due to the current.  When it goes through the wires it doesn't gain or lose energy because there is no field inside the wire.  Where it passes through the resistors, it loses energy due to heat, and this energy is provided by the electric field.

So how could the voltage in the wire gradually change from one end to the other?  Wouldn't that require the charge to gain or lose energy as it moved from one end to the other?

There's no contradiction here.  All the real quantities are consistent, energy is conserved, charge is conserved. 

But voltage is not a real quantity in this case.

The experiment is not saying that the midpoint of the loop is both at 0.9V and 0.1V at the same time.  It is saying one meter is reading 0.9V and the other is reading 0.1V.  This is because the meter leads follow different paths.  Simple as that.
Title: Re: Does Kirchhoff's Law Hold? Disagreeing with a Master
Post by: Berni on November 16, 2018, 11:07:03 am
Sorry for the long post, but it is worth a read since i think it finds a good middle ground between Lewin and ElectroBoom


My explanations leans on this:
https://en.wikipedia.org/wiki/Electromotive_force#Formal_definitions
Quote
Inside a source of emf that is open-circuited, the conservative electrostatic field created by separation of charge exactly cancels the forces producing the emf. Thus, the emf has the same value but opposite sign as the integral of the electric field aligned with an internal path between two terminals A and B of a source of emf in open-circuit condition

If you have a open circuit length of wire in a moving field (Or vice versa) you do get a different amount of charge density on each of the ends that corresponds to the EMF voltage. The longer the wire is the more electrons there are for the magnetic field to tug along so as you go along the wire they cumulatively get pushed more and more. Much like a vertical column of water getting pulled on by gravity, the bottom ends up with more pressure than the top (Yes i know water is not the same as electricity but the idea is similar). Here this effect is called charge separation.

This works even in superconductors. At first it sounds wrong because more electrons at one end would create a electric field inside of a superconductor, but the magnetic field that shoved the electrons over to the end did so using its 'virtual electric field' (Well it is a real electric field, but its not caused by a charged object). So the two fields put together are again zero. Once you connect it into a loop they are free to move so the field disappears and the current they cause opposes the outside field so then you have no electric field induced by charge separation and the field caused by the magnetic fields cancel out to zero too. The current and field sustain each other so the current flows forever and the field stays forever. Very useful for making incredibly strong magnets and is used extensively for this in things like MRI machines and particle accelerators.

If we instead close the loop by putting a resistor in series then we get a case of both. We need an electric field to push electrons trough that stubborn resistance inside the resistor so this effect of open loop charge separation on the wire puts extra electrons on one side of the resistor so they can force themselves trough using there own electric field. But because now electrons are flowing trough the resistor we have a current in the loop so the loop makes its own opposing magnetic field. So far it looks like a closed loop superconductor again, but the resistors don't allow the electrons to flow freely so they can't make it around the loop fast enough to fill the 'electron void' on the other side of the resistor. As a result some electrons are left behind on one end of the resistor and continue to experience charge separation, thus making the wire look like it has voltage and this voltage appears as a smooth gradient across the length of the superconductor. Due to the resistor limiting the amount of current the magnetic field it creates around the loop is smaller than the outside magnetic field and so the 'virtual electric field' it creates in the loop does not fully subtract out the one caused by the outside field. The field that steps in to fill the missing part is the electric field caused by charge separation and gets the sum of fields inside the superconductor back to zero as it should be.

This means that if we connect a wire between two points on the superconductor and route it in a way that generates no EMF on the wire we get current flow that is proportional to the voltage on the two points and the wire resistance.(But only if this superconductors loop is closed with a resistor in series). This gives the two points a set voltage between them that is a single value.

So lets see the definition of voltage then:
https://en.wikipedia.org/wiki/Voltage
Quote
Voltage is the difference in electric potential between two points. The difference in electric potential between two points (i.e., voltage) in a static electric field is defined as the work needed per unit of charge to move a test charge between the two points.

Wait... ??? Yeah this is what throws the wrench in the works.

So if you integrate the total electrical fields around the path you get zero volts inside the superconductor and all the rest of the voltage on the resistor. Since the path goes trough a different resistor depending on what way around you go you also get a different voltage. So by definition of voltage it checks out. This is why this is such a big argument, Dr. Lewin is not saying anything wrong.


So where is the problem then?
We don't have a voltmeter that drags an electron around and logs the work needed to do so. Tho if someone did make one id love to see it cause it sounds really cool. So because of this we can't measure the voltage in the exact way it is defined. What we have to do instead is tap off the voltage with extra wires and bring that voltage to the voltmeters input port. When the wires are run in such a way that they don't get affected by the magnetic field only get the charge separation effect pushing electrons trough the voltmeter so the voltmeter ends up measuring electron charge density between the points. The voltmeter becomes part of the circuit and the voltage drop on the 10MOhm resistor inside the voltmeter is this voltage we see. (Doing this adds a third possible solution to the node)

TLDR starts here:

Okay our voltmeters suck... so really what is the problem?
The experiment is never explained how the voltmeter 'selects' what voltage it can see. There is no mention given to the importance of the path that the voltmeters probe wires take and why they are routed in that exact way. It just leaves you head scratching how is it possible to see two different voltages at the same point. It demolishes your intuitive notion of voltage in circuits. Many electronics engineers after university are likely still confused as to how it works.

The whole thing is explained with a schematic and using some basic circuit analysis tools. Any voltmeters in the schematics are assumed to tap off the electric field integral of the loop you want to see and ignore others. It would have been much better to explain it on the level of electric fields and electrons moving around if the goal was to show the underlying physics. Using a schematic and then simply using the ideal wire model out of circuit analysis methods and then talking about the fields inside a wire is confusing. You ether don't use  the abstraction of circuit node analysts methods at all and focus on electrons in a wire (so you can interact your magnetic fields with them), or you go all the way with circuit analysis and create an equivalent circuit that shows the magnetic effects as inductors. One or the other ways of explaining it makes sense and works great! Kirchhoffs law is a circuit analysis tool and it works (for circuit analysis), its not a law that governs how the universe works.

So why does circuit analysis not agree with physical electrons moving in wires? Because that's not the point of this abstraction. The goal of circuit analysis methods is to make it as easy as possible to predict the behavior of a circuit with as little math as possible. So to not complicate something as simple as a wire it simply cuts the concept of voltage down to the effect the voltage has on components (including the effect the voltage has on a voltmeter). In this simplified world Kirchhoffs law works perfectly and because the abstraction uses voltages that we can observe in real life means that the results of these circuits analysis methods also work in real circuits with electrons running trough them.

Many simple equations you have been taught in your first few years of physics are actually set inside an abstracted world where for example the speed of light is infinite and our atmosphere is a perfect vacuum. So are they wrong? Well... in theory yes they are wrong, but they work just fine in the abstracted world. The math is much easier and faster in this abstracted world, yet when done carefully still gives results that are very close to real ones you would get in the real world.

Please use a hammer for hammering nails rather than screws.
Title: Re: Does Kirchhoff's Law Hold? Disagreeing with a Master
Post by: rfeecs on November 16, 2018, 06:39:23 pm
You are defining a "true" measurement as one where the fields don't interfere with the meter leads.  We are taught that the leads are not supposed to be part of the measurement.  If there is a voltage drop in the leads, that is an error.  So you have to eliminate or account for the error to make an accurate measurement.

Fair enough.

So to measure from A to B, arrange your leads vertically so they are parallel to the magnetic field and perpendicular to the E field.  Or in some other way shield the leads from the fields.  So now apply Faraday's law.  First integrate the flux over the surface of your measurement loop.  There is no net flux through the area surrounded by the measurement leads and the line from A to B, because of the way the wires are arranged or shielded.  So you can pick either surface in the current loop going from A to B.  You are splitting the loop in half, so they each have the same flux which is half the total flux.  So either way you pick, there is an EMF of 0.5V.  Then integrate around the path of the measurement leads and either half of the loop.  If you go one way, you get 0.5V-0.1V=0.4V.  If you go the other way, 0.9V-0.5V=0.4V.  OK.  So the true voltage is is 0.4V.

And yes, if you move the test points toward one of the resistors, you would "slice the pie" of the surface of the current loop differently, and the voltage you measured would continuously change and end up just the voltage across the resistor.

So your model, using lumped coupled coils or transformers, works.  It gives you the "true" answer.  I get it.  It agrees with Electroboom and it agrees with Faraday's law.  There's nothing wrong with it.

This is basically the same thing bsfeechannel did in his analysis back in reply #106.  Some of the reactions were "you are just picking a path that gives you the answer you want".

But based on the standard way that electrical engineers look at measurements, this is the correct path.  Again, fair enough.

edit:  And yes, if you choose this path, the resistive ring always measures 0V.
Title: Re: Does Kirchhoff's Law Hold? Disagreeing with a Master
Post by: Sredni on November 17, 2018, 05:19:09 am
So to measure from A to B, arrange your leads vertically so they are parallel to the magnetic field and perpendicular to the E field.  Or in some other way shield the leads from the fields. 

So now apply Faraday's law.  First integrate the flux over the surface of your measurement loop.  There is no net flux through the area surrounded by the measurement leads and the line from A to B, because of the way the wires are arranged or shielded.

I am not sure to understand this.
You do know that the flux depends on the area orthogonal to the field that is enclosed by the contour, right?
How on earth do you plan to place or shield your leads to avoid intercepting the flux when you are partitioning a disk? You should shield the area, but then forget connecting to a circuit immersed in the field.

But let's say we have find a probe placing such that...
Quote
You are splitting the loop in half, so they each have the same flux which is half the total flux.  So either way you pick, there is an EMF of 0.5V.  Then integrate around the path of the measurement leads and either half of the loop.  If you go one way, you get 0.5V-0.1V=0.4V.  If you go the other way, 0.9V-0.5V=0.4V.  OK.  So the true voltage is is 0.4V.

Ok, let's call 'true' the voltage measured along paths that split the area in equal parts. This is not the only possibile logic choice but let's go with this.

Quote
And yes, if you move the test points toward one of the resistors, you would "slice the pie" of the surface of the current loop differently, and the voltage you measured would continuously change and end up just the voltage across the resistor.

If you call true potential the one along paths that split the area in equal parts, no. You have to gerrymander the probe to give two identical areas when your endpoints are not on a diameter.

Quote
So your model, using lumped coupled coils or transformers, works.  It gives you the "true" answer.  I get it.  It agrees with Electroboom and it agrees with Faraday's law.  There's nothing wrong with it.

Ahem, but let's suppose that we have agreed to use a certain class of possible paths (they are not unique for the same measurement - for example a measure with endpoints on a diameter can be done using the diameter as a path, or the yin-yang path, but let's brush this aside for the moment)

Quote
This is basically the same thing bsfeechannel did in his analysis back in reply #106.  Some of the reactions were "you are just picking a path that gives you the answer you want".

But based on the standard way that electrical engineers look at measurements, this is the correct path.  Again, fair enough.

I beg to differ. And here's another reason why.
The standard way that engineers look at measurements, voltage is addittive. If you want to find VAB you can find it by summing VAC and VCB.
Also, you might like the property VAB = -VBA.
Well, with path dependent measurements you have to give up both of these properties.

The latter one can easily be amended. If we agree to use the same path, only reversed, for BA and AB, then we gave to change

VAB + VBA = 0   (KVL)

in

VAB + VBA = EMF   (Faraday, or as engineers sometime call it, generalized KVL)

What happens when you want to sum two voltages? Each of them might require a different path to give the "true" potential reading. In general the three paths connecting the points A, B and C will enclose a portion of space that non necessarily encloses ALL of the flux. You you'll end up with

VAB+VBC+VCA = a fraction of emf corresponding to the varying flux linked by that area.

And what area is that? There is more than one path that can give you the 'true' potential (remember? diameter and yin yang). So, your sum of 'true potentials' is... whatever you want it to be.

Quote
edit:  And yes, if you choose this path, the resistive ring always measures 0V.

This is right. :-)


PS
I wrote this after having slept nine hours in three days. So, check it.
Title: Re: Does Kirchhoff's Law Hold? Disagreeing with a Master
Post by: GeorgeOfTheJungle on November 17, 2018, 08:59:30 am
Hey guys, I've solved it for ya:

(https://www.eevblog.com/forum/chat/does-kirchhoffs-law-hold-disagreeing-with-a-master/?action=dlattach;attach=573734;image)

Vab = -I*R = I*9R

Are these resistors in series, in parallel, or both?
Title: Re: Does Kirchhoff's Law Hold? Disagreeing with a Master
Post by: riyadh144 on November 17, 2018, 10:43:16 pm
https://www.youtube.com/watch?v=ototTU5NUNA (https://www.youtube.com/watch?v=ototTU5NUNA)

Dr. Lewin just uploaded another video.
Title: Re: Does Kirchhoff's Law Hold? Disagreeing with a Master
Post by: rfeecs on November 17, 2018, 10:56:00 pm
So to measure from A to B, arrange your leads vertically so they are parallel to the magnetic field and perpendicular to the E field.  Or in some other way shield the leads from the fields. 

So now apply Faraday's law.  First integrate the flux over the surface of your measurement loop.  There is no net flux through the area surrounded by the measurement leads and the line from A to B, because of the way the wires are arranged or shielded.

I am not sure to understand this.
You do know that the flux depends on the area orthogonal to the field that is enclosed by the contour, right?
How on earth do you plan to place or shield your leads to avoid intercepting the flux when you are partitioning a disk? You should shield the area, but then forget connecting to a circuit immersed in the field.

I'm thinking 3 dimensionally.  The contour is bent 90 degrees along the line from A to B.  The idea was to try to keep the test leads parallel to the magnetic field and perpendicular to the electric field as I said.  But yes, it is probably not physically possible.  For shielding, I was thinking more like coaxial shields around the test leads.

The other way is to account for the "error" introduced by the EMF induced in the loop of the test leads.  Measure the EMF separately in a calibration step, and then subtract it out the measurement.  The result is the same.

I was trying to rationalize the lumped element circuit that people are automatically coming up with as a valid interpretation of a "correct" measurement.

Personally, I think this demo is a fairly simple demonstration of Faraday's law, and trying to come up with the lumped circuit just adds unnecessary complications.

Electronics people automatically react strongly because of Dr. Lewin's use of a schematic in a confusing way and his lack of detailed explanation of what is really happening.I'm sure it was covered in more detail for his students in discussion sections and lecture supplements. And then there's his provocative "Kirchoff's law is for the birds." 

I haven't watched the latest video he just posted, yet.  We'll see.

 
Title: Re: Does Kirchhoff's Law Hold? Disagreeing with a Master
Post by: ogden on November 18, 2018, 02:10:18 am
For shielding, I was thinking more like coaxial shields around the test leads.

Just put whole experiment into transformer core like this:

(http://www.edn.com/ContentEETimes/Images/01Steve%20T/LivAnaPotCores140/Pot%20Cores%201.jpg)
Title: Re: Does Kirchhoff's Law Hold? Disagreeing with a Master
Post by: rfeecs on November 18, 2018, 05:23:57 am
For shielding, I was thinking more like coaxial shields around the test leads.

Just put whole experiment into transformer core like this:

(http://www.edn.com/ContentEETimes/Images/01Steve%20T/LivAnaPotCores140/Pot%20Cores%201.jpg)

Yes, put the current loop where the "windings" are, and route the wires connected to points A and B outside and measure.  Now you can move the meter from one side to the other and measure the same value.  Or two meters, one on each side, both connected to A and B would both measure the same.
Title: Re: Does Kirchhoff's Law Hold? Disagreeing with a Master
Post by: GeorgeOfTheJungle on November 18, 2018, 07:57:33 am
Dr. Lewin just uploaded another video.

(https://www.eevblog.com/forum/chat/does-kirchhoffs-law-hold-disagreeing-with-a-master/?action=dlattach;attach=574943;image)

He's got the current wrong :-)
Title: Re: Does Kirchhoff's Law Hold? Disagreeing with a Master
Post by: Berni on November 18, 2018, 09:41:11 am
Ah yeah i did think that current was pretty high for 140 Ohms but never went to check it. Yeah the decimal place is in the wrong spot and should be 0.064 A. Ah well can happen to anyone.
Title: Re: Does Kirchhoff's Law Hold? Disagreeing with a Master
Post by: GeorgeOfTheJungle on November 18, 2018, 11:57:33 am
In the last video 1) He's totally avoiding the crux of the matter with electroboom: that the voltmeter wires are part of the circuit, and when considered properly, explain the (seemingly bizarre) results he gets.

And 2) Kirchoff's KVL says "the sum of the emfs in any closed loop is equivalent to the sum of the potential drops in that loop", he leaves the induced EMF of the loop out of the equation to prove that KVL is wrong? Well, yeah, right.
Title: Re: Does Kirchhoff's Law Hold? Disagreeing with a Master
Post by: EEVblog on November 18, 2018, 12:39:23 pm
Okay our voltmeters suck... so really what is the problem?
The experiment is never explained how the voltmeter 'selects' what voltage it can see. There is no mention given to the importance of the path that the voltmeters probe wires take and why they are routed in that exact way. It just leaves you head scratching how is it possible to see two different voltages at the same point. It demolishes your intuitive notion of voltage in circuits. Many electronics engineers after university are likely still confused as to how it works.

Most simply don't care  ;D
Title: Re: Does Kirchhoff's Law Hold? Disagreeing with a Master
Post by: Arznei on November 18, 2018, 05:18:07 pm
Okay our voltmeters suck... so really what is the problem?
The experiment is never explained how the voltmeter 'selects' what voltage it can see. There is no mention given to the importance of the path that the voltmeters probe wires take and why they are routed in that exact way. It just leaves you head scratching how is it possible to see two different voltages at the same point. It demolishes your intuitive notion of voltage in circuits. Many electronics engineers after university are likely still confused as to how it works.

Most simply don't care  ;D

But they should to be honest. I mean, you should at least *know whats going on* and why voltage as a concept can break down in the presence of an induced EMF. You may practicaly never encounter it or have ways do deal with it, which surely is the case for most. But at least know the basic physics of your job and know the limits of the models you use in your daily work. I mean, you people have gone through university and have a masters degree, at least show some respect to your field of education.

You behave like a general practitioner not knowing how to perform a tracheotomy or what it even is. Yeah, he may never have to do one. But when he does, I want him to have at least a general idea of how to go on.
Title: Re: Does Kirchhoff's Law Hold? Disagreeing with a Master
Post by: Berni on November 18, 2018, 05:37:41 pm
Okay our voltmeters suck... so really what is the problem?
The experiment is never explained how the voltmeter 'selects' what voltage it can see. There is no mention given to the importance of the path that the voltmeters probe wires take and why they are routed in that exact way. It just leaves you head scratching how is it possible to see two different voltages at the same point. It demolishes your intuitive notion of voltage in circuits. Many electronics engineers after university are likely still confused as to how it works.

Most simply don't care  ;D

Well to be honest i wouldn't be surprised. Most people in university just memorise things to pass the test and are not even interested in understanding it.

It takes lots of enthusiasm for electronics to really get yourself to understand the field.
Title: Re: Does Kirchhoff's Law Hold? Disagreeing with a Master
Post by: Arznei on November 18, 2018, 05:56:52 pm

Well to be honest i wouldn't be surprised. Most people in university just memorise things to pass the test and are not even interested in understanding it.

It takes lots of enthusiasm for electronics to really get yourself to understand the field.

Yeah, and those are probably the ones jerking off the most about how they are STEM educated and making fun of any non-STEM students during university. Which is funny, because if you just leave all the complicated stuff out of EE you can break it down to get pretty simple in practice.
Title: Re: Does Kirchhoff's Law Hold? Disagreeing with a Master
Post by: Vtile on November 19, 2018, 05:29:08 pm

Well to be honest i wouldn't be surprised. Most people in university just memorise things to pass the test and are not even interested in understanding it.

It takes lots of enthusiasm for electronics to really get yourself to understand the field.

Yeah, and those are probably the ones jerking off the most about how they are STEM educated and making fun of any non-STEM students during university. Which is funny, because if you just leave all the complicated stuff out of EE you can break it down to get pretty simple in practice.
...And then they become walkers searching lost beans in some corporation, ruining everyones work with their idiocy.
Title: Re: Does Kirchhoff's Law Hold? Disagreeing with a Master
Post by: SiliconWizard on November 19, 2018, 05:37:16 pm
I still don't think Lewin uses the right way to make his point nor quite the right examples, but I must admit he at least opened our eyes on something we tend to use without actually really *caring* about all the underlying theory, as Dave said. When we do our work properly, it still doesn't change anything much, but now we know we don't use KVL the way it was intended, and that our definition of "voltage" is in all aspects more practical than theoretical.

Title: Re: Does Kirchhoff's Law Hold? Disagreeing with a Master
Post by: rfeecs on November 20, 2018, 06:05:23 pm
Yet another short video.  He mentions Electroboom this time.

https://youtu.be/d_XqrZo5_7Y (https://youtu.be/d_XqrZo5_7Y)
Title: Re: Does Kirchhoff's Law Hold? Disagreeing with a Master
Post by: KL27x on November 20, 2018, 08:44:37 pm
Summary:
1. Title is "my sincere apologies"
2. 90% of video is arguing against the nonexistent strawman who is apparently suggesting that Farady's law is incorrect, and Kirchoffs is always correct. Take great glee in squashing this strawman by repeating the same thing he has stated like a broken record for the past week.
3. State that anyone with a masters in electronics who suggests that the different readings are only* due to placement of the probing leads is an idiot.
4. Clarifies that the "sincere apology" is for being too blunt in laying out his 100% correct argument against the fairy tale strawman while completely brushing off the actual point of contention.

*but let's forget that I was the one to apply real oscilloscpes and their readings to a real test circuit. If we applied these real oscilloscopes to my theoretical model, we would apparently learn that [start broken record]

Super shorter summary:
"I can see the sailboat, and you can't."

Next up:
"Newton's laws are just a special case of relativity. It's a crime that we call these things laws, when it is really Newton's Loopholes."
Title: Re: Does Kirchhoff's Law Hold? Disagreeing with a Master
Post by: Sredni on November 20, 2018, 11:33:14 pm
Take two square clocks and put them side by side so that they have a common side. Paint it red to make it stand out.
Do you find it suprising that that same red side is seeing the hand of the left clock going down and the hand of the right clock going up?

Would you call that a measurement error? Or bad glancing?
Title: Re: Does Kirchhoff's Law Hold? Disagreeing with a Master
Post by: HackedFridgeMagnet on November 21, 2018, 04:56:23 am
Maybe a bad analogy?
Title: Re: Does Kirchhoff's Law Hold? Disagreeing with a Master
Post by: free_electron on November 21, 2018, 12:52:18 pm
https://www.youtube.com/watch?v=vZYAHGwS3mA (https://www.youtube.com/watch?v=vZYAHGwS3mA)
Title: Re: Does Kirchhoff's Law Hold? Disagreeing with a Master
Post by: EEVblog on November 21, 2018, 01:33:17 pm
I still don't think Lewin uses the right way to make his point nor quite the right examples

I think the same. Lewin's not wrong, but his experiment to explain his point is poor IMO.
Because he's not a practical engineer, I think he's incapable of seeing it from that aspect. He lives in the world of physical theory and knows it so intimately it's all he can see. And there is nothing wrong with that of course, it's important stuff in it's own right, and he's right to point it out.

Quote
, but I must admit he at least opened our eyes on something we tend to use without actually really *caring* about all the underlying theory, as Dave said. When we do our work properly, it still doesn't change anything much, but now we know we don't use KVL the way it was intended, and that our definition of "voltage" is in all aspects more practical than theoretical.

How many of us have ever really thought about it? I doubt there is a single one of us. KVL is a practical theory that works and holds for all but the most obscure aspects of the entire EE field.

It's like trying to argue that Newtons laws are "for the birds" and are just a special case of general relativity. You aren't wrong by saying that, but geeze, try going into NASA and telling all the space probe engineers that Newtons laws are "for the birds", you'd get laughed out of the room.
Title: Re: Does Kirchhoff's Law Hold? Disagreeing with a Master
Post by: Sredni on November 21, 2018, 02:18:52 pm
Maybe a bad analogy?

Do you really not see any relevance of the two clocks example with electromagnetism?
It exemplifies the convention used to define positive oriented areas in vector integral calculus.
The orientation of the path defines the orientation of the area.

Now, that is instrumental in computing the flux of a vector field. And the flux of a function of said field, like its time derivative.
Even the flux of the rotor of a vector field.

And what is the definition of rotor? Basically, it's the circulation around a tiny closed path around a point. So the flux of the rotor can be computed by summing, integrating, all those tiny contributions.
And what happens to the contours? Well, thanks to that 'bad' clock analogy they all cancel out except for those on the external border (look at figure 3.9 here http://www.feynmanlectures.caltech.edu/II_03.html (http://www.feynmanlectures.caltech.edu/II_03.html)). That's what Stokes theorem tells you: the surface integral of the rotor of a field on an oriented area is the circulation of that field along the closed contour encircling it, with the convention of the right hand rule. Like the hand of a clock.

Add in Faraday-Maxwell equation that tells you that the rotor of E is the time derivative of B (ok, there's a sign but that does not change anything since it's always the same) and you should now see the relevance of that analogy.

In fact, if you split a finite area in two parts with a common side, just like those two clocks, you will find that despite the orientation of the areas be the same, the orientation of the common path will come out reversed. Namely, if we assume the same flux configuration, you will get opposite contributes to the circulation along that path depending on which loop it is considered to be part of.
This 'reversal' of the integral of E dot dl (which in an electrostatic situation we would call "the" potential) is not because of bad probing or a measurement error, it is just a consequence of that inversion you see along the common side of two adjacent clocks.

I doubt anyone would find it surprising, just as they would find perfectly normal that the hand of the right clock is seen going up along the common side, while the hand of the left clock is seen as going down.

And there is no escape from this, no matter how small you choose the adjacent contours.
And when you consider finite contours, you can still divide them in two with a common side: the result is the same: if the same flux is intercepted by both partial areas (edit: actually this is not even required, what it counts is that the E field is the same along the common side and well, it has to be since it's the same set of points), the contribute to the emf on the common path will be reversed.
Would you call that measurement error, or probing error?
I would call that "that's just the way it is", Kronkite style.

This is the same inversion that comes out with Lewin's experiment when both resistor are the same.
Would you call that probing error?
To me, that's just how EM works. And the roots of this behavior go down to both that orientation behavior and the fact the circulation of one quantity is related to (a function of) the flux of another one.

No, I would not call that a bad analogy. But that's just me.

(of course, if you remove the link with the flux because the function of B is always zero you get a very special situation where this weird shit does not happen)

EDIT: changed "convention" with "behavior", because its the correlation between area orientation and path orientation that makes all this happen (clockwise-countclockwise vs up and down in this case)
EDIT: changed "infinitesimal" with "tiny" for not riling up mathematicians. Added link to picture. Corrected some minor typos.
EDIT: changed path with contour where relevant to avoid ambiguities.
Title: Re: Does Kirchhoff's Law Hold? Disagreeing with a Master
Post by: Sredni on November 21, 2018, 03:40:27 pm

It's like trying to argue that Newtons laws are "for the birds" and are just a special case of general relativity. You aren't wrong by saying that, but geeze, try going into NASA and telling all the space probe engineers that Newtons laws are "for the birds", you'd get laughed out of the room.

I'm sorry Dave, but it's not like that.
Faraday's law is not a refinement of the theory used in the 'generalized KVL' engineers use to analyze circuits with inductances, transformers and motors. It is the very same theory.

The only reason the 'generalized KVL' works, giving 'single-valued potentials' is because we are not allowed to cut through the fluxes that make inductances, transformers and motors work. All the weird stuff happens inside, and since our physical circuit cannot repartition the area where the variable flux is linked in the device, nor can it go around it the other way (think of a toroidal transformer, you can go around it any way you want, but you cannot cut through the flux since it is all contained inside - same happens with normal iron core transformers, yes, there may be leakage, but you can't cut through the core) [edit]... we get single valued potentials, like when we measure Lewin circuit from one side OR from the other.
If we avoid that repartitioning and cutting through, we're fine. We can still delude ourselves into believing that our voltages are uniquely determined by endpoints only. But that's just an illusion that helps in making circuit analysis more streamlined. Nothing more than that.

But if we are presented with a circuit that cuts through or allows for encircling with opposite orientation a varying flux region, then we can no longer indulge in that illusion and we must fall back to the reality of electromagnetism, where that linking between fluxes and circulations is deeply rooted down to the bone.
Saying that the multivalued potential we are witnessing can be ascribed to a 'probing error' is denying what Maxwell's equations and basic vector calculus are telling you.

Edit: added "and basic vector calculus".
Edit: added conclusion to sentence after very long parentheses. I had forgot to finish it.
I also wanted to add the voice of Hal saying "I'm sorry, Dave. I can't let you do that, Dave" but did not find a way to do that.

Title: Re: Does Kirchhoff's Law Hold? Disagreeing with a Master
Post by: Berni on November 21, 2018, 07:28:32 pm
Calm down a bit. Nobody is claiming that Maxwell equations are wrong or that KVL describes the underlying physics.

Maxwells equations describe the underlying physics perfectly fine. And yes KVL is indeed in theory wrong.

But this does not mean that KVL is "for the birds" as these feathery creatures have no use for it, while in contrast engineers use it all the time, sometimes even daily. Try to find an engineer who claims to have used Maxwells equations in the last 12 months (And i don't mean use the idea behind it, but actually calculated something with it). So why are we all using the clearly inferior KVL when we could all be using proper Maxwells stuff? Well... turns out KVL is simply more useful for everyday use.

Maxwells equations involve fields and geometry that needs to know the exact dimensions and locations of everything in 3D space. This brings about a lot of extra complicated math that just slows you down when you are trying to get actual work done and drastically increases the potential for human error. The way engineering has solved this problem is to introduce an abstraction called circuit schematics. For the case of magnetic components we can simplify all of this geometry and fields down to a single inductance number in units of Henry. We have pre-baked equations (That do eventually involve Maxwell if you drill deep enough) that quickly turn common geometrical segments of wire into inductance values, or the same inductance value of a real life circuit can be measured quicker and more accurately than its geometry. This single number is then inserted in the equivalent inductor model and is used for all further calculation. An extra benefit of doing this is that it not only abstracts the magnetic effects but it even abstracts voltage in to a sort of "effective voltage" that always has 1 defined value. An even better bonus is that now KVL works for all cases, but as a consequence the voltage it operates with and gives as a result are these "effective voltages" rather than the real definition of voltage(Intergal of fields around a path). Getting a result of "the voltage here is 1.4 V" is more useful than the result being multiple values and you need to use the correct one depending on what you are doing with the voltage. In order to give a single number it is not simply throwing away the other values and cherry picking this one. The math deep down works out in a way where the positive and negative signs of all the numbers line up in a way that gives the same result regardless of the path. The results of these abstracted calculations can be verified with experiments and they match up with measured results. This is the same kind of thing as using Complex numbers in calculations, we could have 4+j3 Volts across two points. So when a voltmeter in real life shows 5V is it wrong? No its not, its just that our math was using imaginary numbers as an abstraction of phase to make math much easier, we still need to know how to interpret the abstraction.

KVL is not a special case of Faradays law even if it looks similar. Instead it is a tool used in analysis of abstracted schematic circuits. If you are to unravel all of these mathematical circuit modeling tools far enough you would eventually get to Maxwells equations, but its not as simple as sticking an extra voltage into KVL, you would end up with multiple pages of math before you get there. Our circuit schematics are the equivalent of "spherical cows in vacuum" in physics, its an abstraction that optimizes and simplifies the math for quick and easy computation. In physics the well known F=m*a equation(Often called Newtons 2nd law) is also wrong in certain cases (Rockets and mass–energy equivalence) and we still use it with its limitations in mind rather than saying its for the birds.

So is it bad probing? I think we need a definition of what that is first. Its hard to find a formal definition of it but lets say we use something like this:
Quote
Bad probing is the result of performing a measurement of a desired quantity in such a way that the resulting measured value differs from the actual value of the quantity by more than than our defined error margin allows for.

So by this definition Dr. Lewin is not doing bad probing since he is indeed measuring the voltage he is after within tolerances required by the particular demonstration. The one who is doing bad probing is ElectroBoom since he is after the "apparent voltage" on the two points but is doing so in a way that cause the probes to significantly affect the reading. He should likely have used the setup suggested by this post: https://www.eevblog.com/forum/chat/does-kirchhoffs-law-hold-disagreeing-with-a-master/msg1972940/#msg1972940 (https://www.eevblog.com/forum/chat/does-kirchhoffs-law-hold-disagreeing-with-a-master/msg1972940/#msg1972940) .This would cause the inductive coupling coefficient to the probe wires to become negligibly low, giving the voltage that he is after and so solving the bad probing error. Solving the same circuit setup using Maxwells equations would return a similar result, except that the two values it gives for each path would be nearly identical (Keep in mind we are solving for voltage across the voltmeter terminals, not the voltage inside the ferrite shield).


So who is wrong about what?
ElectroBoom is wrong about using a oscilloscope to measure voltage as formally defined in literature. He needed to use that elusive voltmeter that drags an electron around and measures the mechanical work taken to do so, it would have given him the result Dr. Lewin predicted. But he is using Kirchhoffs law correctly by using it for circuit analysis and not to describe electric fields. He is also using it correctly to determine the voltage across the oscilloscope terminals by correctly interpreting the "apparent voltage" result of the abstracted circuit analysis math.

Dr. Lewin is wrong about using Kirchoffs law for describing what happens to electric fields. Kirchhoffs laws actually describe the behavior of ideal electrical circuits (That are merely an simplified but accurate abstraction of what happens in real circuits with actual fields and electrons). If you look up KVL on wikipedia it explicitly calls them in the title "Kirchhoffs's cirucit laws" ( https://en.wikipedia.org/wiki/Kirchhoff%27s_circuit_laws (https://en.wikipedia.org/wiki/Kirchhoff%27s_circuit_laws) ). He is however correct in using Faradays law to determine the voltage as per formal definition on the two points, but he never includes the oscilloscope and its connections into his calculation and does not explain why the voltage seen on the terminals of the oscilloscope is equal to the voltage at the two nodes of interest.

Its all a case of comparing apples to oranges. They are simply not the same thing even if they seam to fit in the same basket. Part of it is because Kirchhoffs circuit laws predate Maxwells equations and its likely that his laws ware used to describe the physics until Maxwell came around with his more elegant and physically accurate way simply because that's the best theory they had at the time. This is just how science works.
Title: Re: Does Kirchhoff's Law Hold? Disagreeing with a Master
Post by: In Vacuo Veritas on November 21, 2018, 07:37:31 pm
Seems to me it's simple to resolve, put the meter HALFWAY and see what you read...
Title: Re: Does Kirchhoff's Law Hold? Disagreeing with a Master
Post by: firewalker on November 21, 2018, 08:12:12 pm
He still doesn;t undestand the mistake he is making.
Title: Re: Does Kirchhoff's Law Hold? Disagreeing with a Master
Post by: GeorgeOfTheJungle on November 21, 2018, 08:48:20 pm
Seems to me it's simple to resolve, put the meter HALFWAY and see what you read...

When you do that, what are you supposed to read? Because there's two different resistors in parallel that are in series too, with the same current going through both... I*R1? -I*R2?
Title: Re: Does Kirchhoff's Law Hold? Disagreeing with a Master
Post by: ogden on November 21, 2018, 09:33:01 pm
Seems to me it's simple to resolve, put the meter HALFWAY and see what you read...

When you do that, what are you supposed to read? Because there's two different resistors in parallel that are in series too, with the same current going through both... I*R1? -I*R2?

If two meters that are part of super experiment circuit shows 0.1V and 0.9V accordingly, then such "halfway" meter shall read 0.4V. It is discussed/explained in this thread many times by the way
Title: Re: Does Kirchhoff's Law Hold? Disagreeing with a Master
Post by: EEVblog on November 21, 2018, 10:56:22 pm
KVL is not a special case of Faradays law even if it looks similar. Instead it is a tool used in analysis of abstracted schematic circuits. If you are to unravel all of these mathematical circuit modeling tools far enough you would eventually get to Maxwells equations, but its not as simple as sticking an extra voltage into KVL, you would end up with multiple pages of math before you get there. Our circuit schematics are the equivalent of "spherical cows in vacuum" in physics, its an abstraction that optimizes and simplifies the math for quick and easy computation. In physics the well known F=m*a equation(Often called Newtons 2nd law) is also wrong in certain cases (Rockets and mass–energy equivalence) and we still use it with its limitations in mind rather than saying its for the birds.

It's kinda similar with that "current flowing through a capacitor" video. Electrons don't actually flow through the capacitor, but that interpretation along with AC impedance is the practical and easiest way to design and analyse circuits and that's why it's used and is a perfectly valid way of thinking. Ironically, it's Maxwell's displacement current theory that helps validate the concept here. So the deeper into the theory you go, the more it backs up the "incorrect" current flow viewpoint.

Quote
So is it bad probing? I think we need a definition of what that is first. Its hard to find a formal definition of it but lets say we use something like this:
Quote
Bad probing is the result of performing a measurement of a desired quantity in such a way that the resulting measured value differs from the actual value of the quantity by more than than our defined error margin allows for.
So by this definition Dr. Lewin is not doing bad probing since he is indeed measuring the voltage he is after within tolerances required by the particular demonstration.

I'd call it "deceptive" probing  ;D
Title: Re: Does Kirchhoff's Law Hold? Disagreeing with a Master
Post by: rfeecs on November 21, 2018, 11:21:34 pm
It's kinda similar with that "current flowing through a capacitor" video. Electrons don't actually flow through the capacitor, but that interpretation along with AC impedance is the practical and easiest way to design and analyse circuits and that's why it's used and is a perfectly valid way of thinking. Ironically, it's Maxwell's displacement current theory that helps validate the concept here. So the deeper into the theory you go, the more it backs up the "incorrect" current flow viewpoint.

Yes, not only is KVL for the birds, but KCL is too!

Here's a rough derivation of Kirchoff's laws from Maxwell's equations:  KVL from Faraday's law, KCL from Ampere's law:
https://bit.ly/2Fy2svN (https://bit.ly/2Fy2svN)

Quote
Let’s look at when are Kirchoff’s Laws would be violated.  Notice that both laws assume the
time rate of change of something is approximately 0.  This means they are true in the ‘static’
or ‘low frequency’ limit.  The circuits that you will deal with in this course work under this
limit.  However, it’s interesting to note that KCL is violated inside the capacitor because
charge can build up on the capacitor plates, so if part of your closed surface passes between
the plates the net charge changes significantly with time.  However if you consider the
capacitor as a whole ‘lumped element’ and include both plates inside the closed surface, the
charges on opposite plates cancel out and KCL will still work.
  Similarly, KVL is violated
inside the inductor
because a significant magnetic flux passes through the coil, so if your
closed path runs along the coil’s wiring a voltage drop occurs even across the perfect metal
wire.  However if you consider the inductor as a whole ‘lumped element’ and recognize its
total voltage as the induced electromotive force due to Lenz’s Law, KVL will still work.
Title: Re: Does Kirchhoff's Law Hold? Disagreeing with a Master
Post by: GeorgeOfTheJungle on November 21, 2018, 11:21:45 pm
Seems to me it's simple to resolve, put the meter HALFWAY and see what you read...
When you do that, what are you supposed to read? Because there's two different resistors in parallel that are in series too, with the same current going through both... I*R1? -I*R2?
If two meters that are part of super experiment circuit shows 0.1V and 0.9V accordingly, then such "halfway" meter shall read 0.4V.

It's -0.1V and 0.9V.

Quote
It is discussed/explained in this thread many times by the way

Oh, oh, ok. This, but with only one voltmeter in the center:

(https://www.eevblog.com/forum/chat/does-kirchhoffs-law-hold-disagreeing-with-a-master/?action=dlattach;attach=571904;image)

Thanks.
Title: Re: Does Kirchhoff's Law Hold? Disagreeing with a Master
Post by: ogden on November 22, 2018, 12:17:33 am
It's -0.1V and 0.9V.

Yes. I omitted signs, sorry

Quote
Oh, oh, ok. This, but with only one voltmeter in the center:

Yes. You can refer to more detailed model shown in this post (https://www.eevblog.com/forum/chat/does-kirchhoffs-law-hold-disagreeing-with-a-master/msg1945312/#msg1945312) as well.
Title: Re: Does Kirchhoff's Law Hold? Disagreeing with a Master
Post by: bsfeechannel on November 22, 2018, 12:46:07 am
Yes, not only is KVL for the birds, but KCL is too!

Somebody finally said it.
Title: Re: Does Kirchhoff's Law Hold? Disagreeing with a Master
Post by: Berni on November 22, 2018, 07:11:50 am
Yep capacitors also break Kirchhoffs current law when you think about what electrons are actually doing.

But we are not in arguing about it because like Kirchhoffs voltage law we know it applies to circuit analysis where the capacitor is regarded as a black box with a large capacitance across the terminals. The reason we are arguing about KVL is that Dr. Lewin is trying to apply KVL to electric fields rather than circuits and is then blaming KVL for being wrong just because he is using it wrong.

There is a similar theoretical electrical circuit paradox involving two capacitors:
https://en.wikipedia.org/wiki/Two_capacitor_paradox
(https://upload.wikimedia.org/wikipedia/commons/thumb/8/86/Two_capacitor_paradox.svg/220px-Two_capacitor_paradox.svg.png)

Just throwing circuit analysis math at it and analyzing what happens in the circuit when the two differently charged capacitors are shorted together seams to cause capacitors to violate themodynamics by violating the law of conservation of energy. Go read trough it, its pretty interesting.

So of course the reason why this happens is that this is a circuit that can't exist in our universe. Same thing as Dr. Lewins circuit with two resistors and a mysterious current forced trough it without having a circuit component doing the actual current pushing. It breaks the circuit abstraction by doing things that are not supposed to happen and as a result breaks the math used to analyze circuits (Like KVL). In this circuit with two capacitors we simply need to add the parasitic inductance and resistance of the wires and suddenly the circuit acts like it would in real life. It would oscillate back and forth trough the stray inductance while gradually reducing in amplitude as the energy (that we seamed to miss before) got turned into heat on the resistance.

I can see how Dr. Lewin would throw Maxwells equations and Kirchhoffs laws into the same basket. He is a physicist and has likely not done enough circuit analysis and modeling to see that KVL is not just Faradays law with a missing voltage.

I do have to admit he did make me look at voltage is a bit of a different light. I think should continue to do this lecture and demonstration as it is quite dramatic. BUT he should not go to a conclusion of "Kirchhoff is for the birds". Instead he should explain inside the same lecture that Kirchhoffs laws are a circuit analysis tool rather than a physics equation that describes the working of the universe. Briefly explain the importance of correct circuit modeling and understanding how the math you use works. Leave it there and continue on with whatever other physics you have to teach.
Title: Re: Does Kirchhoff's Law Hold? Disagreeing with a Master
Post by: Sredni on November 22, 2018, 03:08:41 pm
The reason we are arguing about KVL is that Dr. Lewin is trying to apply KVL to electric fields rather than circuits and is then blaming KVL for being wrong just because he is using it wrong.

He is not using it wrong. And you say so, too:

Quote
Dr. Lewins circuit with two resistors and a mysterious current forced trough it without having a circuit component doing the actual current pushing. It breaks the circuit abstraction by doing things that are not supposed to happen and as a result breaks the math used to analyze circuits (Like KVL).

Yes, KVL works for lumped circuit abstraction only. When the premises for lumped circuit analysis are missing, KVL is for the birds.

Quote
He is a physicist and has likely not done enough circuit analysis and modeling to see that KVL is not just Faradays law with a missing voltage.

No. In order to see KVL as Faraday's law with a missing voltage, lumped circuit assumptions need to be met. All fluxes should be confined inside the lumped components and you should not be allowed to run circles around them. It's as easy as that. No refinement of the theory (the concept of inductance used in the 'extended KVL' you mention is based on Faraday's law), no need to add parasitics, no nothing.

An engineer should know the limits of their tools and theories.
KVL is lumped circuits stuff. Circuit not lumped ----> KVL for the birds.

Edit: changed highlighted part, able->allowed
Title: Re: Does Kirchhoff's Law Hold? Disagreeing with a Master
Post by: ogden on November 22, 2018, 05:59:41 pm
Dr.Lewin's demo in lecture #16 is bird trap for parrots, not KVL.
Title: Re: Does Kirchhoff's Law Hold? Disagreeing with a Master
Post by: Berni on November 22, 2018, 07:30:23 pm
He is not using it wrong. And you say so, too:

Can you show me a source that claims that KVL is allowed to be used outside of lumped cirucit schematics? And if it does use it as a simplified version of Faradays law does it ever say its valid in a non DC scenario?

Yes, KVL works for lumped circuit abstraction only. When the premises for lumped circuit analysis are missing, KVL is for the birds.

Yep exactly. Without lumped circuits KVL does not exist as it contains a summa operator () that operates on voltages across components. Without lumped circuits there are no components to put inside the equation (You can't strictly define down to the atom level a point in space where a resistor begins and where it ends). If you are operating on real life components made out of actual atoms you need the integral operator found inside Faradays law because real components have physical size. When you see an integral inside what looks like KVL, that is just a unfinished derivation from a special case of Faradays law, this is not KVL.

In the same way Faradays law does not exist in lumped circuits as the integral takes in physical dimensions that do not exist in lumped circuits, trying to apply it there would also make it appear broken, much like KVL appears broken in the real world. That would be a case of using Faradays law wrong.

No. In order to see KVL as Faraday's law with a missing voltage, lumped circuit assumptions need to be met. All fluxes should be confined inside the lumped components and you should not be allowed to run circles around them. It's as easy as that. No refinement of the theory (the concept of inductance used in the 'extended KVL' you mention is based on Faraday's law), no need to add parasitics, no nothing.

An engineer should know the limits of their tools and theories.
KVL is lumped circuits stuff. Circuit not lumped ----> KVL for the birds.

Edit: changed highlighted part, able->allowed

There are are other laws that are made for use inside lumped circuits and this includes Ohms law in the form we use! We are not extending KVL by adding inductance. The inductance is simply how the math behind circuit analysis takes in account the electrical effects of a wire having non zero length. The calculation of this inductance value involves Faradays law deep down, this calculation condenses the potentially incredibly complex geometry of a wire down to a single number that can simply be plugged into the other math rather than dragging whole Faradays law along for the ride and causing all further math to grow exponentially more complex. If you used Faradays law directly you would still be adding the effects of the wire as a extra piece of area in the loop, if you don't add it in then the wire appears to have zero length just like in the lumped circuit without the inductor added in. The only difference being that circuit modeling gives it a name of "parasitic inductance" (Because its usually undesired in circuit design) while Faradays law adds it in just the same but does not give it a specific name, instead just assuming it as extra loop area.

Can you explain why using a lumped circuit is bad?
Title: Re: Does Kirchhoff's Law Hold? Disagreeing with a Master
Post by: GeorgeOfTheJungle on November 22, 2018, 08:50:41 pm
And what can electroboom say in reply, now?
Title: Re: Does Kirchhoff's Law Hold? Disagreeing with a Master
Post by: Sredni on November 22, 2018, 09:09:44 pm
And what can electroboom say in reply, now?

He could say he was looking for a particular set of potential differences among the infinitely many you can find inside a varying flux region. If you look at the symmetry of the problem, a uniformly variable flux directed along the z axis will generate a non conservative induced E field tangential to concentric circumferences. If you choose radial paths to reach any couple of points A and B on the circuit circumference, the contribute of the induced field will be non zero only on the arc. This will give you a nice set of potentials that end up being additive and (possibly) uniquely defined. This is essentially what Cyriel Mabilde has done.

EDIT: actually I have yet to check the E field goes around in concentric circles, but seems reasonable.

(note, that this is not equivalent to intercept equal and opposite fluxes in both the meshes the disk has been partitioned into - it's quite the opposite: intercepting different fluxes so as to give a nice set of values in a way to give VAC = VAB+VBC).

BUT, and it's a big butt-- I mean but, you can measure them only by specifying a particular class of paths. Confirming what Faraday has always said: the voltage depends on the path.

(I suppose that by offsetting the circuit with R1 and R2 with respect to the main coil you should make your probes exit the circle not from the center of the circuit but from the center of the coil).

Edit: specified addittivity, made dependance on area explicit
Title: Re: Does Kirchhoff's Law Hold? Disagreeing with a Master
Post by: Sredni on November 22, 2018, 09:45:28 pm
Can you show me a source that claims that KVL is allowed to be used outside of lumped cirucit schematics? And if it does use it as a simplified version of Faradays law does it ever say its valid in a non DC scenario?

Lewin is not using KVL wrong, because he is not using it. He is using Faraday. Watch his video "Science and believing..." and follow the mesh analysis. You might have been confused by the fact that he drew what appears to be a circuit with lumped elements (and in fact, many posts ago, I said "I am not calling it schematics on purpose") but that curly arrow magnetic field representation inside the central mesh is telling it all. He is using Faraday.
And, in fact, he is getting path depending voltages.
No incongruences whatsoever.

And why is he using Faraday and not KVL?
Because he is analyzing a circuit that is NOT LUMPED. He is INSIDE the frigging coil!!!
If you insist in using KVL, for example at the mesh with the two voltemeters, you find and impossible result.
Impossible for Kirchhoff, but not for Faraday.

As for the rest of your post, I sense confusion. I don't know how to say it. KVL works only in the lumped component assumption. Real world components like inductors, coupled coils and transformers can be modeled with lumped components AS LONG AS YOU DO NOT MESS WITH THEIR FLUXES.  In this case you can mend KVL in 'generalized KVL'.
 If in your circuit you are able to mess with the flux, say goodbye to KVL and 'generalized KVL'. You need to apply Faraday, or you will find inconsistent results.

Why?

Because the voltage depends on the path.


Also, you seem to mixing togheter external and internal inductance. You might want to have a look at "Fields and Waves in Communication Electronics", by Ramo, Whinnery and Van Duzer.


Edit: IIRC, Lewin used the magnetic field representation with little crosses or dots, and not the curly arrow. Either way, that's not lumped circuits language.
Title: Re: Does Kirchhoff's Law Hold? Disagreeing with a Master
Post by: ogden on November 22, 2018, 10:03:26 pm
Because he is analyzing a circuit that is NOT LUMPED. He is INSIDE the frigging coil!!!

Right. Those two resistors put inside that frigging coil does not count. Dr.Lewin can ignore internal resistance of EMF source in his equations just because Dr.Lewin is always right.

https://www.youtube.com/watch?v=uAXtO5dMqEI
 (https://www.youtube.com/watch?v=uAXtO5dMqEI)
Title: Re: Does Kirchhoff's Law Hold? Disagreeing with a Master
Post by: Vtile on November 22, 2018, 10:09:26 pm
He is not using it wrong. And you say so, too:

Can you show me a source that claims that KVL is allowed to be used outside of lumped cirucit schematics? And if it does use it as a simplified version of Faradays law does it ever say its valid in a non DC scenario?

I think these two were answered in the original 'quote' of Kirchoff work some trillion posts ago.  ::)
Title: Re: Does Kirchhoff's Law Hold? Disagreeing with a Master
Post by: EEVblog on November 22, 2018, 10:14:34 pm
An engineer should know the limits of their tools and theories.
KVL is lumped circuits stuff. Circuit not lumped ----> KVL for the birds.

And that is the entire reason why Dr Lewin's experiment is flawed. He's trying to use a practical lumped circuit to show how his non-lumped "inside the inductor" Faraday thinking is right.
And that's wrong, or at best misleading, even though what's he's saying and explaining is not wrong.
Title: Re: Does Kirchhoff's Law Hold? Disagreeing with a Master
Post by: Zucca on November 22, 2018, 10:27:02 pm
In real life if two voltmeters connected to the same two points are reading different values, what to do then?

0) check if was not a mistake to use two instruments to perform the same measure
1) check if one of the two is broken or need calibration (or need some new fresh battery  ;))
2) check if there is an induced emf to the leads going to the DVMs, by moving the leads you should see a change in the readings

Can you tell me what I forgot? Thanks!

PS: Yes, I believe in God!
Title: Re: Does Kirchhoff's Law Hold? Disagreeing with a Master
Post by: bsfeechannel on November 22, 2018, 11:33:21 pm
An engineer should know the limits of their tools and theories.
KVL is lumped circuits stuff. Circuit not lumped ----> KVL for the birds.

And that is the entire reason why Dr Lewin's experiment is flawed. He's trying to use a practical lumped circuit to show how his non-lumped "inside the inductor" Faraday thinking is right.
And that's wrong, or at best misleading, even though what's he's saying and explaining is not wrong.

I think what Prof. Lewin is trying to convey is that no circuit is really lumped.

The "lumpiness" comes only after you make certain assumptions, which most of the time we do implicitly and unconsciously.

If I could translate "Kirchhoff is for the birds", I would say, "Please, put Kirchhoff circuital laws aside for a moment, while I introduce you to a more fundamental theory that not only will explain a lot of phenomena that Kirchhoff can't, but will explain Kirchhoff itself".

I think that, if he came up otherwise so politely with a soothing voice, people wouldn't pay attention to his advice and would try to derive Faraday from Kirchhoff, which is a mistake, instead of the other way around.

So I think that's why he decided to add some hyperbole to his rhetoric. Not sure if it worked though.
Title: Re: Does Kirchhoff's Law Hold? Disagreeing with a Master
Post by: joeqsmith on November 23, 2018, 01:14:35 am
I had seen Dr Lewin's  demonstration a few years ago but he did not do a good job showing his setup and I always assumed he had made a mistake.   I later saw another MIT lecture with a slight twist to the demonstration but again, I felt a bit letdown by the lack of details surrounding his setup.   I don't have a problem with what he presents on the board, well except the miss on ohms law.  lol.   Figured I would watch them all again as it had been so long.  Rare I watch EBs channel but after about 8 minutes in, I thought he did a good job with it.   

I really don't have much to add.  I repeated the test using some coax.  Braid terminated at one side, core at the other.   Then made both sections symmetrical.  My mind wasn't blown.  It behaves as I would expect.  As someone else said, take into account all the loops.   

There are a few things to note.  I didn't take a lot of care to try and get good numbers as I didn't think that was important.   Where both Dr Lewin and EB use a mechanical switch,  I am driving the coil and you can see both halves of the cycle.   


***************

I finished Romer's paper.  Had I been aware of it at the time I had first seen Dr Lewin's demonstration, I would have understood what he was  trying to show.   I am now watching Dr Lewin's latest videos where he is going over Romer's paper.   Still he misses the opportunity to discuss the various ways the circuit could be arranged as Romer's paper covers.   I feel as if I an being expertly trolled by an 80 year old!   :-DD 
Title: Re: Does Kirchhoff's Law Hold? Disagreeing with a Master
Post by: Sredni on November 23, 2018, 02:22:48 am
An engineer should know the limits of their tools and theories.
KVL is lumped circuits stuff. Circuit not lumped ----> KVL for the birds.

And that is the entire reason why Dr Lewin's experiment is flawed. He's trying to use a practical lumped circuit to show how his non-lumped "inside the inductor" Faraday thinking is right.
And that's wrong, or at best misleading, even though what's he's saying and explaining is not wrong.

I'm not sure I'm understanding this.
Where did Lewin used a "practical lumped circuit"?

In the demo at the end of the famous lecture 16
he used a physical system constituted by two resistors connected in a loop of metal wire. The resistors themselves are inside the loop, or as engineers like to call it, the coil. That alone should have alerted the student and made him think: "Oh, wait a minute... I have always seen coils made of wire only, and then the rest of the circuit attached to its terminals. This is something different".
Then he used two oscilloscopes as 'voltmeters', still in the physical world to show that they gave different readings despite being connected to the very same two points.
That should be mind blowing for a second year university student used to probe circuits that do not mess with the flux. Now, the student should have started to use the matter between the ears and ask himself "how is this possible?" and should have started questioning the assumptions made.
And that is the purpose of the demo. Stimulate thinking. Lewin did many similar thought-provoking demos in his lectures. He never gave an explanation on purpose, not to spoon-feed the students.
That's good teaching, I see no flaws here. (but that's me) [note1]

In the explanation and subsequent videos
He explained why the instruments in the real world gave that apparently impossible result. Because that circuit, for the fact of having resistors inside the coil and the instruments linking opposite flux depending on how they encircled the coil (you can't do otherwise when they are part of the coil itself EDIT: unless you want to cross the flux, which is bad as Venkman would tell you) could not be considered a lumped circuit where KVL works.
And in fact he solved it using Faraday. If you look at his 'schematic' there are symbols representing the magnetic field in the middle of the main mesh representing the metallic loop with the resistors. There is no such thing in lumped circuit theory: you draw that wiggly symbol that represents an inductance or the secondary of a transformer and you are sure the magic flux-voodoo happens inside and do not draw the crosses or dots that represents the direction of the magnetic field.
EDIT: Also, the equations he penned are Faraday's, and yes they obviously look like 'generalized KVL' but the source of emf is no longer localized in a lumped element (if you do that, you end up with single valued potentials). (end EDIT)
There is, therefore, no 'practical lumped circuit'. Neither in the real world, nor on the blackboard (or whiteboard).

If the student does not understand it, how should it be Lewin's fault?
But even if we agree Lewin should have spoon-fed his students (which are MIT students, so I guess he had somewhat higher than average expectations), the 'he did not explain it well enough' excuse could work for about 10-15 minutes. After that, well, it's no longer Lewin's fault.
And this thing has been going on for much longer than that.

(There is a silver lining, though: many people have been learning about Faraday better than ever)

(Note0: when I said 'inside the coil' I meant the loop with the two resistors. As for the primary coil - the one generating the field - it could have been substituted by a falling magnet, or a nuclear explosion...)
[Note1: I found this comment by Lewin in one of his videos
Quote
>>>asks me to find a mistake on my own>>
that is by far the best way to teach Trust me I have been teaching Physics for 58 years. If present a student on a silver plate what (s)he did wrong they will quickly forget. But if they have to put in the effort to find their mistake (after I have sent them my lectures which address their topic) they will never forget.
Edit: typos (some of them) and added part on equations on board.
Edit: added silver lining and Lewin teaching advice
Title: Re: Does Kirchhoff's Law Hold? Disagreeing with a Master
Post by: @rt on November 23, 2018, 06:15:09 am
So nobody posted the proper pounding yet?

edit.. already posted.

“Sincere Apology” nice one. Sorry you’re wrong! :D
Title: Re: Does Kirchhoff's Law Hold? Disagreeing with a Master
Post by: Berni on November 23, 2018, 07:19:24 am
Lewin is not using KVL wrong, because he is not using it. He is using Faraday. Watch his video "Science and believing..." and follow the mesh analysis. You might have been confused by the fact that he drew what appears to be a circuit with lumped elements (and in fact, many posts ago, I said "I am not calling it schematics on purpose") but that curly arrow magnetic field representation inside the central mesh is telling it all. He is using Faraday.
And, in fact, he is getting path depending voltages.
No incongruences whatsoever.

And why is he using Faraday and not KVL?
Because he is analyzing a circuit that is NOT LUMPED. He is INSIDE the frigging coil!!!
If you insist in using KVL, for example at the mesh with the two voltemeters, you find and impossible result.
Impossible for Kirchhoff, but not for Faraday.

Well at some point in the video he does add up all the voltages according to KVL and points out you don't get zero. This is his reasoning for saying "its for the birds". So one of the flowing cases is happening:
a) Assuming a circuit is lumped: He is claiming the circuit drawn is an lumped equivalent circuit of the real circuit so KVL should work inside it but it does not.
b) KVL in non lumped circuit: He is wrongfully applying KVL to a non lumped circuit so it does not work because its not meant to work
c) Inaccurate lumped circuit: His lumped lumped circuit model is describing a physical circuit consisting of zero length wires that can't exist in real life

Mesh analysis is a big part of the circuit analysis toolkit and is used on lumped circuits. This is another reason why i get the feeling he is mixing lumped and real circuits as if they are the same thing. As soon as you draw a resistor symbol you are creating a lumped model of a resistor. This resistor has a physical size of zero, acts perfectly according to Ohms law and has exactly 2 strictly defined connection terminals. A real resistor has a physical size larger than zero, stops following Ohms law when taken outside of operating limits and has a volume of material representing the connection terminals rather than a infinitely small point in space.

Lumped circuits are not ignoring Maxwell. They are fully embracing it and using Maxwell in a way that hides it away under abstraction in order to make math easier because 3d space integrals are hard. Its a math shortcut.

So rather than saying KVL is wrong he should have said ether "KVL can't be applied here because this is not a lumped cirucit" or "Here is a lumped equivalent circuit that is required to apply KVL"


As for the rest of your post, I sense confusion. I don't know how to say it. KVL works only in the lumped component assumption. Real world components like inductors, coupled coils and transformers can be modeled with lumped components AS LONG AS YOU DO NOT MESS WITH THEIR FLUXES.  In this case you can mend KVL in 'generalized KVL'.
 If in your circuit you are able to mess with the flux, say goodbye to KVL and 'generalized KVL'. You need to apply Faraday, or you will find inconsistent results.

Yes that is the main point of using a lumped circuit. You don't have to mess with the fluxes. All the flux interaction are calculated using Maxwell into a inductance and coupling coefficient number. Giving you inductors that act like the real one by simply multiplying its values with the correct things rather than having to calculate flux in 3d space every time. All of the flux interactions are modeled in that single coupling coefficient.

The only case where i can see lumped models of wires being a bad idea is when the wires are physically moving during the circuits operation. This would make you need to recalculate the inductance and coupling coefficients as well as the induced EMF according to Maxwell for every timestep of the calculation. Since you are calculating Maxwells equations in 3D space every time it makes sense to just use Maxwells equations directly, because the lumped circuit equivalent of that wire is no longer a math shortcut if you can only reuse the values once. So if you are calculating what happens inside a electric motor its better use Maxwell directly.

Additionally all off the shelf passive electronic components are specified in Ohms, Farads or Henrys. The manufacturer has calculated Maxwell for you (Tho more likely measured with an instrument) and created a equivalent model of the component. You can no longer stick this component in a non lumped circuit. You would need to disassemble the inductor and accurately measure the path the wire takes inside and then use that in your non lumped circuit model.

Nobody is stopping you from only analyzing circuits with non lumped models, but don't expect anyone else to do so. Most people enjoy calculating the result in a few lines of math rather than a few pages (And still get the same result).
Title: Re: Does Kirchhoff's Law Hold? Disagreeing with a Master
Post by: ogden on November 23, 2018, 07:38:54 am
There is, therefore, no 'practical lumped circuit'. Neither in the real world, nor on the blackboard (or whiteboard).

So wrong. Indeed there is practical lumped circuit (for coil loop with resistors) - consisting of as many fragments of the coil as needed for purpose and two resistors. Actually kind of lumped approach is used in Romer's paper stating that 1/2 of the coil have 1/2 of the EMF induced. Dr.Lewin forgot about Faradays law in lecture about Faradays law :) It is obvious that he had "ups, I fracked up" because in the apology "To Agree or Not to Agree with the Master" video circuit he did not use superconductive wire anymore- to further smear things up and make his error(s) less obvious for those (parrots and cultists) who learn physics just by memorizing it.
Title: Re: Does Kirchhoff's Law Hold? Disagreeing with a Master
Post by: sectokia on November 23, 2018, 08:07:38 am
Summary:
1. Title is "my sincere apologies"
2. 90% of video is arguing against the nonexistent strawman who is apparently suggesting that Farady's law is incorrect, and Kirchoffs is always correct. Take great glee in squashing this strawman by repeating the same thing he has stated like a broken record for the past week.
3. State that anyone with a masters in electronics who suggests that the different readings are only* due to placement of the probing leads is an idiot.
4. Clarifies that the "sincere apology" is for being too blunt in laying out his 100% correct argument against the fairy tale strawman while completely brushing off the actual point of contention.

*but let's forget that I was the one to apply real oscilloscpes and their readings to a real test circuit. If we applied these real oscilloscopes to my theoretical model, we would apparently learn that [start broken record]

Super shorter summary:
"I can see the sailboat, and you can't."

Next up:
"Newton's laws are just a special case of relativity. It's a crime that we call these things laws, when it is really Newton's Loopholes."

You have to remember that Electroboom is making a really really bad claim here. Electroboom is claiming that induction in the probe wires is what causes the result.

This is easy to prove incorrect experimentally: You can use shielded grounded coxial cable for the meter probes - the volt meters still show different voltage.

They show different voltages because there is different currents in the left and right loop because of the induction in the centre loop. Lewin explains why this is with actual math and has done detailed videos of 30min+ length about this multiple times.

Mehdi makes motherhood statements, uses napkin diagrams, and refers to analogies about voltage, fields and inductions.

At the end Mehdi shows a circuit which he claims is the 'correct' way. What he actually shows is a circuit of two meter probe loops that DO have induction. So he actually changes the circuit from 3 to two loops, both of which now have induction, and he basically claims that this is the correct way to not have induction. He is totally backwards!

If you look at Mehdis answer, I bet he pulled it from somewhere else, because his solution at the end doesn't follow at all from what he had been blabbing about in the minutes leading up to it, and this is evidence by the fact he says he isn't 100% sure, makes no mention of the magnetic field nor even the position of the probes relative to the soleniod. His solution involves re-arranging circuit so there is induction in the meter probe loops, something that he said was causing the incorrect readings. Hence the only reasonable explination is: he is wrong. It it his is 'probing' that is wrong.

Sorry but Mehdi got owned here.... if he is going to do a video like that he should brush up first. As Lewin said, this is embarrassing for him.
Title: Re: Does Kirchhoff's Law Hold? Disagreeing with a Master
Post by: ogden on November 23, 2018, 12:33:19 pm
You have to remember that Electroboom is making a really really bad claim here. Electroboom is claiming that induction in the probe wires is what causes the result.

What is your explanation then?

Quote
Sorry but Mehdi got owned here.... if he is going to do a video like that he should brush up first. As Lewin said, this is embarrassing for him.

You just embarrassed yourself - because induced EMF in the probe wires is actual cause of "surprising" result. Dr.Lewin explains it in the video "Believing and Science are Very Different".
Title: Re: Does Kirchhoff's Law Hold? Disagreeing with a Master
Post by: sectokia on November 23, 2018, 01:25:04 pm
Fine... well I will do a full dissection of the all the mistakes Mehdi makes:

At 5:47 Mehdi claims that in a twisted wire there is induced current inbetween each twist that cancels out the one in the next twist. This is false. The cables cross over for each twist, they do not connect, so there is no induction loop in the 'circle' of each twist. The reason twisting cables stops induction is because as you twist the cable, you are flipping the area of the loop over. So the magnetic field is now effectively passing though the next twist upside down. So the magnetic field through the first twist is in one direction, and the magnetic field through the next twist is in the other direction - when you consider the plain the loop made up by the wire run. So by twisting you can make the net field through the overall loop zero - therefore no induction.


At 8:18 Mehdi measures the current 'across both resistors'. Mehdi then concludes that voltage R1 plus voltage R2 is the same as voltage across the loop. In adding the volt meter across R1 and R2, Mehdi has changed the topology of the circuit. It now has 4 loops instead of 3.  Most importantly, the meter is now in the inductive loop, not the resistors. Mehdi is oblivious that he has done this. In this case the voltage around the loop is the voltage shown on the meter. By adding the voltage meter here there is no longer any induction in the smaller loop containing the resistors, as that smaller loop does not enclose any of the magnetic field.

At 9:10 he admits that if the meter is put on the out side, re-forming the original induction loop with the resistors in it, that the voltage again goes to zero. He then asks who is correct.  Obviously it is Lewin who is correct, as Mehdi is oblivious the he changed the topology of the loops when he measured what he is calling the 'loop voltage'. With the circuit back in its proper arrangement, he measures as per Lewin. Ironically it is Mehdi who is doing 'incorrect probing' here.

At 9:27 his inability to think of the inductive loop is shown further at 9:27, when he claims to be 'measuring across the gap'. Of course he is just completing a loop with his meter.... there is no gap. Once again he is obvious to his own incorrect probing....

At 9:42 he then runs the meter the other way, so there is no loop around the magnetic field, and he reads zero. He later attributes this to 'cancelling out', but there is nothing to cancel out, as he has formed no loop around the magnetic field.

At 9:49 he claims if he makes the sense lines 'a little bit shorter' than the loop, but without crossing (or encircling) the magnetic field in soleniod, and says "I will have voltage jumps". This shouldn't happen of course because he doesn't have any loop around the magnetic field inside the soleniod. The reason he reads a bizzare wave form here is because there is a weak magnetic OUTSIDE the soleniod (imaging it come out the top stretching to infinity as it spills down and comes back in the bottom of the soleniod). The thing he is measuring at this point is actually the magnetic field 'falling' through his loop in the opposite direction as the magnetic field through the solneiod. This is why his wave form is opposite to when he just measures around the soleniod. He incorrectly attributes this to the meter probes no longer 'canceling out' properly. You can see plainly that he has made a fork shape loop, and he puts his loop in the worst possible position: over the end of the soleniod while not enlcosing the magnetic field inside the soleniod. Lewin explains how you need at least a 30cm soleniod with your loop half way down, the point of this is to ensure the magnetic field outside the soleniod is for practical purposes zero. Mehdi has a complete failure of experiment setup here. You can replicate it yourself, if you have any fork shape loop around the soleniod (but not enclosing it) you read zero. Start moving it up to the end of the soleniod and you will start getting induction in the OPPOSITE direction compared to when you enclose the soleniod.

At 10:00 to 10:27 Mehdi makes the extraordinary claim that a non-closed loop will have half V induction, cancelled out by half V the other way. He never produces a single measurement to show this is true. If his claim was correct, he could measure the Half V by putting meter across left and right sides of the 'semi circle' he shows. This is of course complete garbage. There is no loop formed around the magnetic field, so there is no induction.

At 11:18 Mehdi claims "the same voltage of the loop wire are induced on the sense line". The sense lines are on the left and right side loops, which enclose no magnetic field, so there is no induction involving them. The induction loop is just the inner loop with the resistors. He is 100% wrong about induction 'in' the sense lines.

At 12:02 the proof of Mehdi being wrong is made final when he shows a circuit claiming to show "properly" how to measure the voltage across the loop. Once again this just changes the topology of the circuit. By bringing the probes through the middle he is now creating two induction loops, the left loop and the right loop, each enclosing half the magnetic field. Only this time HIS METER PROBES ARE PART OF THE INDUCTION LOOP. So he has gone from the original Lewin circuit that had no magnetic field through the left and right loops that contain the meter probes, to a circuit with only two loops, both of which form an inductive loop by using the meter probes. Yet he claims it is Lewin who incorrectly probed :-DD

He overall main mistake is that for some reason he thinks voltage must exist across the wire portion of the induction loop. This is because he is used to thinking of circuit elements in conservative fields, which have a voltage drop across them. He out right says it himself when he claims Lewin 'forgot' the 'transformer'. He has no idea at all when it comes to non conservative fields, not even the most basic requirement of a closed loop around the magnetic field being required for induction.... Throughout his video he is completely obviously when he creates new induction loops and changes the topology of the circuit.

In all seriousness... has anyone checked this guy even is actually qualified? Is it just a show? His videos are entertaining but his video on this is absurd junk science. What is scary is the amount of people on here, and youtube, and reddit, who are siding with Mehdi...



Title: Re: Does Kirchhoff's Law Hold? Disagreeing with a Master
Post by: Sredni on November 23, 2018, 02:35:24 pm
b) KVL in non lumped circuit: He is wrongfully applying KVL to a non lumped circuit so it does not work because its not meant to work

It's not KVL, it's Faraday.

Quote
Mesh analysis is a big part of the circuit analysis toolkit and is used on lumped circuits. This is another reason why i get the feeling he is mixing lumped and real circuits as if they are the same thing.

I called it mesh analysis to make the sentence short. And in fact he is analyzing the meshes, applying Faraday. Where's the big difference with lumped circuit analysis? That the emf is not localized. This is best seen when you bring the voltmeter inside the loop and you probe the points on a diameter. Now, if you use Faraday and the emf is not localized, the emf contribute appears in BOTH sub-meshes, proportional to the area encircled by the sub-mesh. If you localize it as if it were a lumped circuit, the emf contribute appears only in one sub-mesh, no matter the area.

Quote
As soon as you draw a resistor symbol you are creating a lumped model of a resistor.

And where is it written that that lumpiness is contagious?
Come on, what should he have done? Invent a new symbology? I've never seen that in any of the EM books I've read.
You should have enough mental flexibility to understand what is going on.

Quote
So rather than saying KVL is wrong he should have said ether "KVL can't be applied here because this is not a lumped cirucit" or "Here is a lumped equivalent circuit that is required to apply KVL"
Good if you want your students to think in a compartmentalized way. Like technicians. But if you are forming scientists, or even engineers, you should teach them to question the limits of the tools (practical and theoretic) they use.

Quote
Yes that is the main point of using a lumped circuit. You don't have to mess with the fluxes.
[...]
The only case where i can see lumped models of wires being a bad idea is when the wires are physically moving during the circuits operation.

Lewin's circuit IS A CASE WHERE A LUMPED MODEL IS INAPPROPRIATE.
How can this be so hard to grasp?

Quote
Nobody is stopping you from only analyzing circuits with non lumped models, but don't expect anyone else to do so. Most people enjoy calculating the result in a few lines of math rather than a few pages (And still get the same result).

Seriously? After all these umptillion pages of discussion you still have not realized that Lewin's circuit cannot be analyzed with lumped circuit theory?
Nobody is saying that you have analyze all circuits with 'non lumped models' only. You just have to do that only when it is necessary. Such is the case of Lewin's circuit.

Oh, for the love of... Physics! :-)
Title: Re: Does Kirchhoff's Law Hold? Disagreeing with a Master
Post by: Sredni on November 23, 2018, 02:43:54 pm
You just embarrassed yourself - because induced EMF in the probe wires is actual cause of "surprising" result. Dr.Lewin explains it in the video "Believing and Science are Very Different".

ogden, I promise I will not interact with you again on this matter (I did it before when you refuse to watch the video, but then you watched it so I turned back on my steps, but now no conditions).

I will leave you with this question.

When the voltmeter is outside the loop, for example measuring 0.9 V, does it matter the area its probes are enclosing in excess of the loop area? I mean, assuming the field is all confined inside the two resistors loop, does it even matter to twist the probes? Will you read, for example, a different voltage when that excess area enclosed by the probes changes from nearly zero (twisted probes) to say, half the area of the loop, equal to the area of the loop, twice the area of the loop, ten times the area of the loop?

That's all.
Now, believe what you wish.

Edit: typos, syntax, grammar, oh Lord! Somebody get me a dictionary and a basic English course!
Title: Re: Does Kirchhoff's Law Hold? Disagreeing with a Master
Post by: ogden on November 23, 2018, 03:10:56 pm
Sredni, seriously? If you did not watch video, here's screenshot from it. Pay attention to EMF equation of loop4 and what loop4 actually is.  And watch the Funny video! Also reading Romer's paper will not hurt BTW.

(https://www.eevblog.com/forum/chat/does-kirchhoffs-law-hold-disagreeing-with-a-master/?action=dlattach;attach=578696;image)
Title: Re: Does Kirchhoff's Law Hold? Disagreeing with a Master
Post by: ogden on November 23, 2018, 03:50:28 pm
He overall main mistake is that for some reason he thinks voltage must exist across the wire portion of the induction loop.

Here we go AGAIN. Groundhog Day (1993) movie. Why don't you read the forum first? Read Romer's paper as well.
Title: Re: Does Kirchhoff's Law Hold? Disagreeing with a Master
Post by: Siwastaja on November 23, 2018, 04:40:15 pm
In all seriousness... has anyone checked this guy even is actually qualified? Is it just a show? His videos are entertaining but his video on this is absurd junk science. What is scary is the amount of people on here, and youtube, and reddit, who are siding with Mehdi...

I think it's far more alarming that people ridicule the desire to understand and learn, and start talking in "siding with" terms when a younger engineer wants to question a professor and ask for clarification in order to gain and spread understanding, and does it in a completely civil way, with a lot of more thought and actual experiments put into it than what goes to usual lecture question. I would be extremely happy for such well-formed and scientifically sound questioning from someone outside the formal scientific circles.

Even more alarming is that an academic person who has been actually teaching is completely unable to handle this kind of situation, which should be everyday practice when it comes to science and learning.

It's also interesting how new accounts pop up in the EEVblog forum just to write such passive-aggressive "taking side" comments like this.

Thank you for your analysis, though.

If I needed to side with someone, it would definitely be Mehdi, regardless of who is being "right". One is clearly encouraging scientific engineering process (which includes questioning, experimenting, and desire to understand), while the other is actively hurting the process (by scaring, denying proper discussion, generally being a total asshole). I understand there are fanboys for the both "cultures".

Luckily, no one is forcing anyone to "side with" anyone.
Title: Re: Does Kirchhoff's Law Hold? Disagreeing with a Master
Post by: Berni on November 23, 2018, 08:36:32 pm
It's not KVL, it's Faraday.

Yes he got his results with Faradays law. But to prove that KVL is wrong he shows that the voltages don't add up to zero. Voltages adding up to zero is KVL and he was applying it to a circuit model that is not valid in normal circuit analysis methods where KVL is meant to be used. This is his proof before saying its for the bids.

Just like usual Newtonian kinetic energy equation ( E = (m*v^2) / 2 ) breaks if you are approaching the speed of light since that form of the equation works with the assumption that speed of light is infinite or that Einsteins relativity effects don't exist, yet we don't say that Newton is for the birds. It works when used correctly


I called it mesh analysis to make the sentence short. And in fact he is analyzing the meshes, applying Faraday. Where's the big difference with lumped circuit analysis? That the emf is not localized. This is best seen when you bring the voltmeter inside the loop and you probe the points on a diameter. Now, if you use Faraday and the emf is not localized, the emf contribute appears in BOTH sub-meshes, proportional to the area encircled by the sub-mesh. If you localize it as if it were a lumped circuit, the emf contribute appears only in one sub-mesh, no matter the area.

Lumped model still works with the voltmeter inside the loop, you simply have to recalculate the values of the inductors that represent wire segments so that they match the crossection contribution and you again get accurate results from a lumped model. You can use an inductor to represent a fractional turn in order to model just one section of wire around the loop. If you want to know whats going on in the middle of that section of wire then simply split it into two half valued inductors and you get a node with the voltage and current at that point in the loop.

The lumped model does not make it necessary to put all the inductance in the loop inside one inductor. This inductance can be split over as many inductors as you want to expose the voltage and current at any point along a wire. It may seam strange to have this fractional turn section of wire since Faradays law requires a loop, but it does work and can be imagined as taking a slice of cake out of the loop and attributing that slice to its accompanying length of wire.

You have the freedom to construct a lumped model in a way that makes the desired measurements easy to access while not changing the behavior of the circuit.


And where is it written that that lumpiness is contagious?
Come on, what should he have done? Invent a new symbology? I've never seen that in any of the EM books I've read.
You should have enough mental flexibility to understand what is going on.

Sorry if that was misunderstood. I was mostly using it as an example of how quickly and often lumped component models sneak up when doing math with electricity. These lumped models have there own set of limitations, just like KVL and all other circuit analysis tools. It makes sense to use a resistor symbol, but you have to realize that it does introduces "lumpynes" into the model.


Lewin's circuit IS A CASE WHERE A LUMPED MODEL IS INAPPROPRIATE.
How can this be so hard to grasp?

I have created a lumped element model that behaves identically to Dr. Lewins experimental circuit on the first page of this thread:
https://www.eevblog.com/forum/chat/does-kirchhoffs-law-hold-disagreeing-with-a-master/msg1945312/#msg1945312 (https://www.eevblog.com/forum/chat/does-kirchhoffs-law-hold-disagreeing-with-a-master/msg1945312/#msg1945312)

It uses the above described "slice of flux cake" way of determining inductance values for sections of wire. Due to the lumped model it makes KVL work perfectly fine in it and allows any other circuit mesh analysis tool to be applied to it correctly.

Can you explain why a lumped model is inappropriate given that it is required in order to apply KVL to it?



Seriously? After all these umptillion pages of discussion you still have not realized that Lewin's circuit cannot be analyzed with lumped circuit theory?
Nobody is saying that you have analyze all circuits with 'non lumped models' only. You just have to do that only when it is necessary. Such is the case of Lewin's circuit.

Oh, for the love of... Physics! :-)

Well that's because so far nobody has shown me what exactly i am doing wrong in my lumped model linked above. It reproduces the same waveform as Dr. Lewins experiment and does not have any cherry picked or tweaked component values in it.

I am not saying you have to analyze everything non lumped. But if you want to use KVL correctly you need to do it lumped.

If someone thinks this model is a lucky fluke i can also show it adjusted to probe other points on the loop or put the voltmeter inside the loop.
Title: Re: Does Kirchhoff's Law Hold? Disagreeing with a Master
Post by: Sredni on November 23, 2018, 11:18:31 pm

I have created a lumped element model that behaves identically to Dr. Lewins experimental circuit on the first page of this thread:
https://www.eevblog.com/forum/chat/does-kirchhoffs-law-hold-disagreeing-with-a-master/msg1945312/#msg1945312 (https://www.eevblog.com/forum/chat/does-kirchhoffs-law-hold-disagreeing-with-a-master/msg1945312/#msg1945312)

I will answer to this part only, because I cannot repeat the same things over and over again. Forgive the use of capital letters, but at this point I want to emphasize the message.

Your model DOES NOT BEHAVES IDENTICALLY to Dr. Lewins experimental circuit.
In Lewin's circuit you can have two different voltage readings from the VERY SAME TWO NODES.
Your circuit cannot do that. You get two different reading from TWO DIFFERENT COUPLES OF NODES. I do not even have to check it because I know that Spice can not tolerate that kind of ambiguity. I have already written about it, some twelve billions posts ago. And if you want to see -0.1V and 0.9V on those two resistors, you could have placed just one coil, with the two resistors in series. Et voilà, the magic dual reading. Except it isn't referred to the same two nodes.

Also, note that the 'extended KVL' is just Faraday under disguise. The original KVL cannot even account for lumped inductances and trasformers. It is indeed for the birds when you have these kind of components in the circuit. But we can still save the appearances by moving the emfs on the other side and pretend it's a voltage drop or a voltage generator (so to speak) and happily simulate our circuits in Spice - it won't scream at you because in this case you can still have single-valued potentials.
But that WORKS only if the components are lumped, i.e. you do not mess with their fluxes. When you can mess with the fluxes, you can no longer pretend to have lumped components, so even the extended KVL is for the birds - you need to account for the distributed emf 'manually'. You can simulate a different, similar circuit in Spice to help out in solving the equations, but you won't see that magic trick of the voltage across the very same two points assuming different values at the same time.

Come back when Spice can give TWO different voltages from the very same TWO NODES. Not three, not four. TWO.
I'll be waiting for you in my igloo in Hell.

In any case the key point is: having voltages depending not only on the endpoints but also on the path, is no magic at all. It is basic electromagnetism - it is a behavior that goes down to the bone of EM structure. Starting from the definition of rotor, passing through Stokes theorem and adding in a pinch of experimental result (Faraday's law).
And that is really basic physics.
Title: Re: Does Kirchhoff's Law Hold? Disagreeing with a Master
Post by: sectokia on November 24, 2018, 12:08:38 am
I was wrong.

Lewin is the fraud.

Title: Re: Does Kirchhoff's Law Hold? Disagreeing with a Master
Post by: Berni on November 24, 2018, 12:19:40 am
Your model DOES NOT BEHAVES IDENTICALLY to Dr. Lewins experimental circuit.

My circuit model produces a graph identical to what what Dr. Lewins two oscilloscopes show, so in what way does it behave differently to the real circuit?


In Lewin's circuit you can have two different voltage readings from the VERY SAME TWO NODES.
Your circuit cannot do that. You get two different reading from TWO DIFFERENT COUPLES OF NODES. I do not even have to check it because I know that Spice can not tolerate that kind of ambiguity. I have already written about it, some twelve billions posts ago. And if you want to see -0.1V and 0.9V on those two resistors, you could have placed just one coil, with the two resistors in series. Et voilà, the magic dual reading. Except it isn't referred to the same two nodes.

So how do you get a real life voltmeter to show you two different voltages at the same time? (Without flipping wires around as that effectively connects the voltmeter to a different part of the cirucit)

Also, note that the 'extended KVL' is just Faraday under disguise. The original KVL cannot even account for lumped inductances and trasformers. It is indeed for the birds when you have these kind of components in the circuit. But we can still save the appearances by moving the emfs on the other side and pretend it's a voltage drop or a voltage generator (so to speak) and happily simulate our circuits in Spice - it won't scream at you because in this case you can still have single-valued potentials.
But that WORKS only if the components are lumped, i.e. you do not mess with their fluxes. When you can mess with the fluxes, you can no longer pretend to have lumped components, so even the extended KVL is for the birds - you need to account for the distributed emf 'manually'. You can simulate a different, similar circuit in Spice to help out in solving the equations, but you won't see that magic trick of the voltage across the very same two points assuming different values at the same time.

I do actually agree to all of that. KVL doesn't have to account for inductance, or even resistance, other circuit analysis tools do it so it doesn't have to. You indeed can't mess with fluxes because they don't exist in lumped models so no worry there. And finally SPICE will indeed always give one voltage, that's why i like it, its always nice to get a concrete answer to a question.



Come back when Spice can give TWO different voltages from the very same TWO NODES. Not three, not four. TWO.
I'll be waiting for you in my igloo in Hell.

In any case the key point is: having voltages depending not only on the endpoints but also on the path, is no magic at all. It is basic electromagnetism - it is a behavior that goes down to the bone of EM structure. Starting from the definition of rotor, passing through Stokes theorem and adding in a pinch of experimental result (Faraday's law).
And that is really basic physics.

I sure hope i don't see SPICE giving two voltages, i don't like bugs in my software.

I completely agree with you that the nodes in Dr. Lewins have two different voltages across it according to the formal definition of voltage. I even explained it in one post how that works on the level of electrons and fields in a wire.

But do try to understand circuit analysis and lumped circuits a bit better. Then you will see how KVL indeed works when applied correctly (And no it still won't give you two voltages because the "effective voltages" used in circuit analysis always have exactly one value unlike "formal definition voltages" that Maxwell operates with).

So when do i get to see a real life voltmeter showing two voltages?
Title: Re: Does Kirchhoff's Law Hold? Disagreeing with a Master
Post by: ogden on November 24, 2018, 12:42:24 am
The voltage across the meter reads 0V.
This video at 9:30 shows this experimentally:

Indeed it shall read 0V between A&D points - if both resistors are equal (100 Ohms in this case). BTW thank you for providing video which at 12:30 confirms discussed in this thread behavior of the circuit - that actual voltage between A&D in Dr.Lewin's experiment is 0.4V.
Title: Re: Does Kirchhoff's Law Hold? Disagreeing with a Master
Post by: sectokia on November 24, 2018, 12:53:20 am
I was wrong.

Lewin is the fraud.
Title: Re: Does Kirchhoff's Law Hold? Disagreeing with a Master
Post by: Sredni on November 24, 2018, 01:07:36 am
So when do i get to see a real life voltmeter showing two voltages?

I see you excluded going around the flux. Naughty boy! :-)
But I can do this with an arm tied behind my back. Bring the voltmeter inside Lewin's two resistors loop, solder its probes on opposite points on the diameter. Wiggle the probes around.
There you go. You can read anything you like from -0.1V to +0.9V (ok, it's either AC or pulsed so the sign is more about phase in the AC case).

Don't tell me you want to see a voltmeter giving two readings from the same two points and the same path.

Am I the only one sensing a shifting goalpost, here?

I am afraid I cannot help you further in solving your confusion. All I can do is suggest you read Ramo, Whinnery, Van Duzer. It will clarify a lot about lumped circuits, distributed circuits and field analysis for waveguides, resonant cavities and antennas. There's a whole world out there, beyond Kirchhoff's columns.
Title: Re: Does Kirchhoff's Law Hold? Disagreeing with a Master
Post by: joeqsmith on November 24, 2018, 02:30:13 am
So when do i get to see a real life voltmeter showing two voltages?

 :-//  If you really want to see it in real life,  your best bet may be to take a few minutes and set it up yourself.  If you lived next door, I could invite you to see my setup in person.   Dr Lewin did make a video using volt meters as well as a scope but I can understand wanting to see it for yourself.   Some wire, a bolt, battery, couple of resistors and a couple of cheap analog meters.   
Title: Re: Does Kirchhoff's Law Hold? Disagreeing with a Master
Post by: rfeecs on November 24, 2018, 02:55:16 am
I'm not sure why sectokia posted a link to this video, then deleted his posts.  But I think it's worth watching if you are interested and have half an hour to kill.

Some people will say he found the proper way to make this measurement.  Others will say that he is just choosing a measurement path that gives him what he wants:

https://youtu.be/JpVoT101Azg (https://youtu.be/JpVoT101Azg)
Title: Re: Does Kirchhoff's Law Hold? Disagreeing with a Master
Post by: The_Paya on November 24, 2018, 03:08:03 am
Quote from: rfeecs on Today at 13:55:16 (https://www.eevblog.com/forum/index.php?topic=149278.msg1986992#msg1986992)
I'm not sure why sectokia posted a link to this video, then deleted his posts.
Because it kind of proves EB right, contrary to his critique?
Edit: NVM.
I was wrong.

Lewin is the fraud.
Title: Re: Does Kirchhoff's Law Hold? Disagreeing with a Master
Post by: sectokia on November 24, 2018, 05:28:39 am
We can all agree that meter probe when completely off to side reads -.1v and  when off the other side completely reads 0.9v.

If the meter probe comes down through the magnetic field splitting it exactly in to two halves of equal area  it reads 0.4v.

You can get any value you want from -0.1 to 0.9 by changing the ratio of the left and right area formed by the meter probes cutting the magnetic field.

You can attach the meter  directly across the left resistor and read anything from -0.1 to 0.4,  it depends solely on the ratio of the two areas that the meter probes cut the solenoid field into.

The same is true when putting the Probes across the other resistor in the other side.

The same is true when putting the probes on any point between the resistors to any point between the resistors on the other side.

In all those cases you can arrange the probes to read anything from  -0.1 to 0.9V.
 
I can connect 3 meters across the resistor on the left and have one read -0.1v, one read 0.4V and one read 0.9v.
Title: Re: Does Kirchhoff's Law Hold? Disagreeing with a Master
Post by: bsfeechannel on November 24, 2018, 06:06:47 am
I'm not sure why sectokia posted a link to this video, then deleted his posts.  But I think it's worth watching if you are interested and have half an hour to kill.

Some people will say he found the proper way to make this measurement.  Others will say that he is just choosing a measurement path that gives him what he wants:

https://youtu.be/JpVoT101Azg (https://youtu.be/JpVoT101Azg)

Nice try. But why did he take half an hour to measure the voltage of a single loop connected to two resistors? Because he had to go into long considerations about Faraday's law of induction before he measured the voltages the "right" way.

Well, if you have to use another theory to validate yours, it means that your theory is not as fundamental.

Make no mistake. Kirchhoff is cool, but for flux sake, learn the doggone Maxwell. There you gon' understand Kirchhoff and much more.
Title: Re: Does Kirchhoff's Law Hold? Disagreeing with a Master
Post by: Berni on November 24, 2018, 10:17:45 am
So when do i get to see a real life voltmeter showing two voltages?

I see you excluded going around the flux. Naughty boy! :-)
But I can do this with an arm tied behind my back. Bring the voltmeter inside Lewin's two resistors loop, solder its probes on opposite points on the diameter. Wiggle the probes around.
There you go. You can read anything you like from -0.1V to +0.9V (ok, it's either AC or pulsed so the sign is more about phase in the AC case).

Don't tell me you want to see a voltmeter giving two readings from the same two points and the same path.

Am I the only one sensing a shifting goalpost, here?

I am afraid I cannot help you further in solving your confusion. All I can do is suggest you read Ramo, Whinnery, Van Duzer. It will clarify a lot about lumped circuits, distributed circuits and field analysis for waveguides, resonant cavities and antennas. There's a whole world out there, beyond Kirchhoff's columns.

Sure you can exclude the flux in the lumped model if you want. All you need to do is set the voltmeters wires to have 0 Henrys of inductance.(This makes the circuit impossible to construct in real life tho). I chose to include the flux so that it does behave like Dr. Lewins real circuit.

Nope not a shifting goalpost. You asked to see a lumped model show two voltages different across two points, voltmeters show this same "effective voltage" as used in lumped circuits so to have two voltages in the same point a voltmeter needs to show two different readings simultaneously too.

You make the different voltages show by changing the path of the wires. The model does the same by simply updating its probe wire inductance values to match the new wire path. Both get the same result on the voltmeter. So how are these results different?


I'm not sure why sectokia posted a link to this video, then deleted his posts.  But I think it's worth watching if you are interested and have half an hour to kill.

Some people will say he found the proper way to make this measurement.  Others will say that he is just choosing a measurement path that gives him what he wants:

<YOUTUBE>

That video does a great job of explaining what is happening. To top it off all of his claims have an accompanying experiment to prove it, rather than just saying "This is how it is, chose to believe it or be wrong"


:-//  If you really want to see it in real life,  your best bet may be to take a few minutes and set it up yourself.  If you lived next door, I could invite you to see my setup in person.   Dr Lewin did make a video using volt meters as well as a scope but I can understand wanting to see it for yourself.   Some wire, a bolt, battery, couple of resistors and a couple of cheap analog meters.   

I did the experiment a few days ago just because i thought it would be interesting, but i haven't posted photos since there ware no unusual results, it works the same as Dr. Lewins experiment as expected.
Title: Re: Does Kirchhoff's Law Hold? Disagreeing with a Master
Post by: HackedFridgeMagnet on November 24, 2018, 11:10:02 am
I'm not sure why sectokia posted a link to this video, then deleted his posts.  But I think it's worth watching if you are interested and have half an hour to kill.

Some people will say he found the proper way to make this measurement.  Others will say that he is just choosing a measurement path that gives him what he wants:

https://youtu.be/JpVoT101Azg (https://youtu.be/JpVoT101Azg)
Great video.  Well worth watching.

He uses a much, much better experimentation methodology and gives much better explanation of what is happening than Dr Lewin does.
He also understands how Maxwell-Faraday interacts with reality too rather than just seeing it as an equation.
He credits someone called Kirk McDonald for his explanation of the "Lewin paradox".
Title: Re: Does Kirchhoff's Law Hold? Disagreeing with a Master
Post by: Berni on November 24, 2018, 11:48:01 am
Here are the photos of my recreation that i did last week but never posted photos. I still had it set up so i just cleared the bench a bit and made some photos and scope captures.

I used a MOSFET driver circuit i had laying around to pulse the current to the coil using a waveform generator so that i had a repeatable waveform with a trigger signal. The recreation of the experiment was mostly done because i thought it was a cool experiment and i don't get to play with such magnetic effects often.

Results are identical to Dr. Lewins experiment and are identical to my lumped circuit model ( https://www.eevblog.com/forum/chat/does-kirchhoffs-law-hold-disagreeing-with-a-master/msg1945312/#msg1945312 (https://www.eevblog.com/forum/chat/does-kirchhoffs-law-hold-disagreeing-with-a-master/msg1945312/#msg1945312) ) that i have put together before even doing the experiment.


So my conclusions stay the same:

1) Faradays law works and does show two different voltages across two points in a circuit. The two voltages appear because we are calculating only one section of the whole loop, what voltage you get depends on what loop section you are going along (The whole voltage is path dependent). There indeed is -0.1V and 0.9V across A and B in Dr. Lewins cirucit, but these are voltages along a incomplete loop (Probe wires complete it into a closed loop to then produce a single result on the voltmeter). Its more of a math technicality and NOT something like Schrodingers cat(Being both dead and alive simultaneously) where both voltages are somehow physically existing in two forms. Its only as real as imaginary numbers.

2) Kirchhoffs voltage law works too. When applied to a lumped circuit mesh representing the real circuit it shows the sum of voltages is zero and makes a prediction that matches Dr. Lewins experiment. Note that the lumped circuit includes the probe wires as part of the circuit (Dr. Lewins application of Faradays law does not include probe wires, hence why it does not fit together)

3) Dr. Lewin is right about everything in his videos except his claims about KVL are wrong. He is applying KVL incorrectly to his circuit. It should instead have been said that KVL is a circuit analysis tool and like all other such tools requires a circuit mesh model to work on (This model includes probe wires). Additionally KVL is defined as being "algebraic sum of all the voltages around any closed loop in a circuit is equal to zero" this is not the same as an integral of the electric field around the loop as he writes it, you require lumped components to be able to apply the summa operator() in the equations of KVL

4) Electroboom is technically wrong about bad probing since Dr. Lewin was measuring the voltage he set out to measure. However he recognizes correctly that the probing method is where the double voltage phenomenon comes from and has shown the underlying reasons for why. There are indeed some minor aspects of his video that could be improved, but he comes to the correct conclusion that KVL always works when used correctly.

4) This is a difficult to explain topic and its very hard to make an explanation that everyone understands.


I would love to see a 3 minute video that carefully condenses the information and explanation from this thread, but i am not a youtube creator.
Title: Re: Does Kirchhoff's Law Hold? Disagreeing with a Master
Post by: bsfeechannel on November 24, 2018, 03:59:49 pm
He also understands how Maxwell-Faraday interacts with reality too rather than just seeing it as an equation.

The equation summarizes reality. That's why you, I and everyone in this forum need to study Maxwell.

Maxwell didn't come up with his equations out of an exercise of math. He collected experimental data from Faraday, Ampere and Gauss, tried to figure out what was going on, i.e., what the REALITY was, and proposed the equations that better describe it.

More important, after he grasped the deep meaning of that REALITY, he could predict, among other amazing things, the existence of something that REALITY was not making so obvious in his time: the propagation of radio waves.

Fifteen or so years later Heinrich Hertz proved with an EXPERIMENT that the equations were right.

When you see people treating Maxwell just "as an equation" it is because they understand the solidity of the theory, have proved its efficacy in practice and have absolute confidence in its predictions.
Title: Re: Does Kirchhoff's Law Hold? Disagreeing with a Master
Post by: ogden on November 24, 2018, 04:19:24 pm
He credits someone called Kirk McDonald for his explanation of the "Lewin paradox".

Good read BTW: http://www.hep.princeton.edu/~mcdonald/examples/lewin.pdf (http://www.hep.princeton.edu/~mcdonald/examples/lewin.pdf)

3) Dr. Lewin is right about everything in his videos except his claims about KVL are wrong.

He is wrong in other claim as well - that 1/2 of the (superconductive) loop has no EMF and potential difference voltage drop between points A1 & A2 at the moment of observation is 0V (video: "Kirchhoff's Loop Rule Is For The Birds", @3:40). This assumption is kinda naīve as properly noted by Dr. Kirk McDonald.
Title: Re: Does Kirchhoff's Law Hold? Disagreeing with a Master
Post by: bsfeechannel on November 24, 2018, 06:55:20 pm
In all seriousness... has anyone checked this guy even is actually qualified? Is it just a show? His videos are entertaining but his video on this is absurd junk science. What is scary is the amount of people on here, and youtube, and reddit, who are siding with Mehdi...

I think it's far more alarming that people ridicule the desire to understand and learn, and start talking in "siding with" terms when a younger engineer wants to question a professor and ask for clarification in order to gain and spread understanding, and does it in a completely civil way, with a lot of more thought and actual experiments put into it than what goes to usual lecture question. I would be extremely happy for such well-formed and scientifically sound questioning from someone outside the formal scientific circles.

Even more alarming is that an academic person who has been actually teaching is completely unable to handle this kind of situation, which should be everyday practice when it comes to science and learning.

I think I understand sectokia's rant. Instead of taking this opportunity to show the limitations of Kirchhoff and encourage people to learn Maxwell, people are transforming the discussion into a libel against the old professor.

This kind of attitude, we've got to admit, is disgusting and will only make half-assed engineers of all of us.
Title: Re: Does Kirchhoff's Law Hold? Disagreeing with a Master
Post by: HackedFridgeMagnet on November 24, 2018, 10:26:22 pm
He also understands how Maxwell-Faraday interacts with reality too rather than just seeing it as an equation.

The equation summarizes reality. That's why you, I and everyone in this forum need to study Maxwell.

Maxwell didn't come up with his equations out of an exercise of math. He collected experimental data from Faraday, Ampere and Gauss, tried to figure out what was going on, i.e., what the REALITY was, and proposed the equations that better describe it.

More important, after he grasped the deep meaning of that REALITY, he could predict, among other amazing things, the existence of something that REALITY was not making so obvious in his time: the propagation of radio waves.

Fifteen or so years later Heinrich Hertz proved with an EXPERIMENT that the equations were right.

When you see people treating Maxwell just "as an equation" it is because they understand the solidity of the theory, have proved its efficacy in practice and have absolute confidence in its predictions.

I don't think anyone here ever said they doubted Maxwell-Faraday.

But from what I can see Dr Lewin has absolute confidence in it's predictions, to the point he thinks he can measure the voltage difference at one set of two points, twice (using a CRO) and get two different answers.
Whereas Cyriel Mabilde shows he only reads two different voltages, because of the different fluxes linked in the leads.
Cyriel Mabilde in doing this shows us how we can measure emf properly.  Incidentally he does it using a sinusoid instead of a transient to show the sign of the voltage better.

Then Dr Lewin goes on to conclude KVL is for the birds.
This is sensationalist rubbish. KVL is successfully used everyday.
The proof is you can easily apply KVL to his demonstration with predictable experimental results.
The predictable results shows KVL works in this case too. Quite a few people have.

Title: Re: Does Kirchhoff's Law Hold? Disagreeing with a Master
Post by: Siwastaja on November 24, 2018, 11:04:21 pm
Instead of taking this opportunity to show the limitations of Kirchhoff and encourage people to learn Maxwell, people are transforming the discussion into a libel against the old professor.

Bullshit. sectokia's comment was directed to Mehdi specifically, in a clearly malicious manner; but Mehdi's not responsible for other "people" transforming the discussion into any "libel". He couldn't have been more appropriate, and to the point.
Title: Re: Does Kirchhoff's Law Hold? Disagreeing with a Master
Post by: rfeecs on November 24, 2018, 11:07:46 pm
In Cyriel Mabilde's video, he is claiming that Dr. Lewin says "there is no EMF in the wires".  He says this is wrong and that his measurements remove all doubt about the existence of EMF in the wires.

To say there is "EMF in the wires" makes no sense.  According to Faraday's law, in this case the EMF is the  time rate of change of the magnetic flux through a surface.  The surface defines the EMF.  It is not located at specific points in the path that defines the surface.

When he makes his wedge shaped measurements, he is defining a surface outlined by the wedge.  That determines a quantity of magnetic flux through that surface, and that determines an EMF.  Obviously the EMF changes when he moves his wedge sides to different positions around the loop, because he is changing the size of the surface.  It doesn't mean the EMF is "in the wires".

I think this confusion happens because of the lumped model we use for induced EMF in inductors and transformers.  You have to stick a voltage source that represents the EMF somewhere in series with the wires.  You can stick it anywhere that makes sense.  We get so used to this model that we think that the EMF is "in the wires" of the inductor or "in the turns" of the transformer.  In fact it is just a model to give us the right voltage at the terminals of the transformer, and is not an actual voltage source in series with the wires in a specific location.

That being said, I think his video is a nice demonstration of Faraday's law and this particular setup.  It shows the effect of different measurement path choices and explains things fairly clearly.
Title: Re: Does Kirchhoff's Law Hold? Disagreeing with a Master
Post by: bsfeechannel on November 24, 2018, 11:45:11 pm
but Mehdi's not responsible for other "people" transforming the discussion into any "libel".

Thank you for admitting that people are really turning the discussion into a libel against Lewin instead of realizing the limitations of Kirchhoff or being encouraged to learn Maxwell.

Admitting the problem is the first step to the solution.
Title: Re: Does Kirchhoff's Law Hold? Disagreeing with a Master
Post by: Zucca on November 25, 2018, 01:12:14 am
But I think it's worth watching if you are interested and have half an hour to kill.

This was excellent. Thanks rfeecs, I feel like now I understand everything and the earth is one more time not flat.
Title: Re: Does Kirchhoff's Law Hold? Disagreeing with a Master
Post by: ogden on November 25, 2018, 01:27:02 am
To say there is "EMF in the wires" makes no sense. According to Faraday's law, in this case the EMF is the time rate of change of the magnetic flux through a surface.  The surface defines the EMF.  It is not located at specific points in the path that defines the surface.

Wait... So you say that there is no EMF induced in the straight wire which is located in the changing magnetic field and only complete/closed loop results in EMF?
Title: Re: Does Kirchhoff's Law Hold? Disagreeing with a Master
Post by: rfeecs on November 25, 2018, 02:03:15 am
To say there is "EMF in the wires" makes no sense. According to Faraday's law, in this case the EMF is the time rate of change of the magnetic flux through a surface.  The surface defines the EMF.  It is not located at specific points in the path that defines the surface.

Wait... So you say that there is no EMF induced in the straight wire which is located in the changing magnetic field and only complete/closed loop results in EMF?
I'm saying this is Faraday's law:

(https://www.eevblog.com/forum/chat/does-kirchhoffs-law-hold-disagreeing-with-a-master/?action=dlattach;attach=579944;image)
(https://www.eevblog.com/forum/chat/does-kirchhoffs-law-hold-disagreeing-with-a-master/?action=dlattach;attach=579950;image)

The right hand side of the equation is the EMF.  It is defined by a surface bounded by a closed contour.  The closed contour doesn't have to be a wire.  It can be any closed contour.

For the EMF to do work, you need a circuit.  So that forms a closed contour.  A straight wire by itself is not a closed contour.  You can't say it results in an EMF all by itself, or that it contains an EMF.

So what can you say about a straight wire that is located in a changing magnetic field?  There will be a defined charge distribution in the wire.  There may even be current in the wire as the charge moves back and forth.  There will be an electric field around the wire, induced by the magnetic field.  The current in the wire may induce it's own magnetic field.  You can say lots of things, but not that it contains an EMF.

You can connect a meter across the ends of the wire.  Now you may measure a voltage and call that an EMF.  Because you created a closed contour with the wires of the meter.
Title: Re: Does Kirchhoff's Law Hold? Disagreeing with a Master
Post by: sectokia on November 25, 2018, 12:36:13 pm
To say there is "EMF in the wires" makes no sense. According to Faraday's law, in this case the EMF is the time rate of change of the magnetic flux through a surface.  The surface defines the EMF.  It is not located at specific points in the path that defines the surface.

Wait... So you say that there is no EMF induced in the straight wire which is located in the changing magnetic field and only complete/closed loop results in EMF?

That's exactly right.

And this is something Mehdi gets wrong when he measures the emf across the loop with the gap in it. By closing the gap with the meter probes only now does the emf exist.

Those is not just a theoretical thing. It is verifiable. If there was an emf there would be a static build up at each end of the wire that you could measure the e field of. But there isn't. It only becomes measurable when you close the loop. If you close it with a resistor there is a static build up on either side of it that you can measure.


Title: Re: Does Kirchhoff's Law Hold? Disagreeing with a Master
Post by: GeorgeOfTheJungle on November 25, 2018, 01:53:38 pm
Those is not just a theoretical thing. It is verifiable. If there was an emf there would be a static build up at each end of the wire that you could measure the e field of. But there isn't. It only becomes measurable when you close the loop. If you close it with a resistor there is a static build up on either side of it that you can measure.

I think the charge moves in the wire even if it's not a closed loop, if you could measure it you'd see a + q in one end and a minus q in the other end.
Title: Re: Does Kirchhoff's Law Hold? Disagreeing with a Master
Post by: Berni on November 25, 2018, 02:48:49 pm
Those is not just a theoretical thing. It is verifiable. If there was an emf there would be a static build up at each end of the wire that you could measure the e field of. But there isn't. It only becomes measurable when you close the loop. If you close it with a resistor there is a static build up on either side of it that you can measure.

I think the charge moves in the wire even if it's not a closed loop, if you could measure it you'd see a + q in one end and a minus q in the other end.

Yes the charges move in a open loop of wire too. Just that they quickly bunch up at the end of the wire, this causes a electric field inside the wire that starts pulling the changes back into a equalized state. At some point the force put on the changes by the magnetic field balances out with the force from the electric field and the charges stop moving. This means the electric field is equal to the magnetic EMF. So there is indeed a electric field inside the wire (Happens even on a superconducting wire), but you can't directly use Faradays law to calculate it as it requires a loop. You have to calculate it using more fundamental math of applying forces to electrons (Faradays law is mostly an application of that fundamental math in a more useful form). It all depends on how you think about voltage. Yes there are more bunched up electrons on one end of the wire and if you generated a few kV of EMF you would even have the electrons fly off into the air and ionise it, but there is also an opposing magnetic EMF present. So if you add up all forms of EMF (electric and magnetic) you indeed get 0V. Electrons feel both types of EMF so the formal definition of voltage across the ends of the wire is 0V. However if you connect the two ends using a wire that travels in such a way that it generates no magnetic EMF you will get current flow proportional to the electric field of those bunched up electrons. This wire would close the loop in such a way that Faradays law would calculate a EMF voltage equal to the voltage of the electric field the bunched up electrons created.

So yes the open piece of wire does push electrons much like a open circuit battery would, but due to the definition of voltage its still 0V because the magnetic EMF is included.

The way definitions are set up we get confusing things when dealing with open loops of wire, but that's kinda okay since a open loop of wire can't do anything useful, it needs to be connected to something on the ends to do something meaningful and at that point the loop got closed anyway.
Title: Re: Does Kirchhoff's Law Hold? Disagreeing with a Master
Post by: Sredni on November 25, 2018, 03:19:36 pm
I think the charge moves in the wire even if it's not a closed loop, if you could measure it you'd see a + q in one end and a minus q in the other end.

And which end will have a plus and which will have a minus?
Imagine your AB segment in a uniformly distributed time-varying B field. It is all the same, spatially.
Which end will get the plus, and which end will get the minus?

Can't decide?
Well, let's create a square loop with AB as its side. Use the right hand rule to find out how the current will flow with that flux varying configuration. Now you can tell me which extreme of AB is plus and which is minus, correct?
Except...

That it all depends which 'side of the loop' AB is on.

Think of two square clocks with a common AB side. Is the seconds hand going up or down? Does it depends on which clock the hand belongs to?
Title: Re: Does Kirchhoff's Law Hold? Disagreeing with a Master
Post by: GeorgeOfTheJungle on November 25, 2018, 03:34:19 pm
With a strong enough (variable) magnetic field and/or enough loops, you should (in theory) be able to light a LED (for example, to say something) even with the ends of the loop open, simply because there's charge being pushed/moving along into the wire and that's what a current is.
Title: Re: Does Kirchhoff's Law Hold? Disagreeing with a Master
Post by: Berni on November 25, 2018, 03:34:33 pm
For those that are disappointed to not see Kirrhhoffs laws be broken here is a way to break them.

(https://www.eevblog.com/forum/chat/does-kirchhoffs-law-hold-disagreeing-with-a-master/?action=dlattach;attach=580238;image)

This is the schematic of the experiment for it. We have a AC signal source that powers a circuit of two resistors. Circuit analysis tells us this is essentially a 50% restive divider. So we expect the blue voltmeter to always read half of what the yellow voltmeter is showing.



(https://www.eevblog.com/forum/chat/does-kirchhoffs-law-hold-disagreeing-with-a-master/?action=dlattach;attach=580244;image)

Here is the test setup. Voltmeters are again represented by an oscilloscope. The RF signal generator (set to +10dBm) is out of the shot, but you can see the coax cable coming in from it. The red coiled up wire is a 1m crocodile clip lead to serve as the 1m monopole antenna.




(https://www.eevblog.com/forum/chat/does-kirchhoffs-law-hold-disagreeing-with-a-master/?action=dlattach;attach=580250;image)

Here is the closeup of the test circuit. It is built using SMD resistors to provide better behavior at RF frequencies while trace lengths are kept to a minimum. Active scope probes ( 0.6pF loading) are used to probe the circuit without drastically affecting it. Additionally a 6dB attenuator is used before the signal enters the board to prevent standing wave issues in the long coax cable to the RF synthesizer.



Antenna disconected
(https://www.eevblog.com/forum/chat/does-kirchhoffs-law-hold-disagreeing-with-a-master/?action=dlattach;attach=580256;image)

This is what we get once we turn it on.

Acording to cirucit analysis we should be getting
504mV / 2 = 252mV

We measure 259mV so an error of 2.8% .This is well within reason given resistor, probe and scope tolerances.




Antenna connected
(https://www.eevblog.com/forum/chat/does-kirchhoffs-law-hold-disagreeing-with-a-master/?action=dlattach;attach=580262;image)

So now we connect the antenna to the midpoint between the resistors and let it hang over the edge of the table so that it is far away from objects it could potentially capacitively couple to. Other end is not connected to anything and is floating in mid air.



(https://www.eevblog.com/forum/chat/does-kirchhoffs-law-hold-disagreeing-with-a-master/?action=dlattach;attach=580268;image)

This results in the flowing image on the scope. The overall loading caused the input signal to sag a bit so we need to recalculate it:
470mV / 2 = 235mV

We measured it to be 150mV this is an error of 57%. This is certainly not inside a reasonable margin of error! Clearly the currents in the two resistor can't be identical if the voltages across them are so different. Well the solution is easy, the missing current is simply flowing into the antenna.

Okay, but where is that current returning? The antenna has only one connection. All currents flow in loops, so where does this loop return the current back?

The frequency of 76MHz was not just randomly picked. The calculated quarter wavelength for a 1m long monopole is 71MHz. But mine ended up measuring to be 76MHz due to the way its bent, its insulation, evnivorment... etc. This means that 1m piece of wire is very good at radiating energy out as radio waves for that particular frequency. Its sucking energy out of our circuit to be able to do so.



(https://www.eevblog.com/forum/chat/does-kirchhoffs-law-hold-disagreeing-with-a-master/?action=dlattach;attach=580274;image)

But what if this is just capacitive loading from the wire dragging down the signal? Well capacitors always create a phase shift. If you look at the previous screenshot you can see the two signals are still perfectly in phase with each other. So to prove this is not the case we leave the wire connected to change the frequency to 152MHz. This is double the frequency before and as a result turns out quarter wavelength of wire into a half wavelength of wire. This makes it very difficult for the wire to radiate the energy because all of it is simply bouncing back into it.

So we do the math again for the expected result:
472mV / 2 = 236mV

And we measured it to be 233mV so an error of 1.3%. This is once again well within our margin of error. So the circuit is again acting like circuit analysis tells us. This proves that capacitive loading of the wire was not at fault here. But because it no longer was able to radiate out energy as radio waves means it was no longer stealing energy out of the circuit so it was allowed to operate as usual.


Conclusion:
Kirchhoffs laws indeed stop working when your circuit starts to emit radio waves. This is very difficult to model and is very frequency dependent. This is not caused by parasitic inductance and capacitance so the circuit model can't be fixed by simply adding those in.

This is the reason why a lot of literature states that Kirchhoffs laws only work in low frequency AC circuits. The low frequency here is considered to be one witch has a wavelength significantly shorter than the circuits physical size. This prevents parts of the circuit becoming antennas and radiating away energy. This is a known limitation of his law.

So are you happy now all of you that want to see Kirchhoffs being wrong?
Title: Re: Does Kirchhoff's Law Hold? Disagreeing with a Master
Post by: Berni on November 25, 2018, 03:49:16 pm
I think the charge moves in the wire even if it's not a closed loop, if you could measure it you'd see a + q in one end and a minus q in the other end.

And which end will have a plus and which will have a minus?
Imagine your AB segment in a uniformly distributed time-varying B field. It is all the same, spatially.
Which end will get the plus, and which end will get the minus?

Can't decide?
Well, let's create a square loop with AB as its side. Use the right hand rule to find out how the current will flow with that flux varying configuration. Now you can tell me which extreme of AB is plus and which is minus, correct?
Except...

That it all depends which 'side of the loop' AB is on.

Think of two square clocks with a common AB side. Is the seconds hand going up or down? Does it depends on which clock the hand belongs to?

Sure you can decide. You follow the usual rules of direction with magnetic fields.

(http://hyperphysics.phy-astr.gsu.edu/hbase/magnetic/imgmag/genwir.gif)

This example shows the wire moving trough the magnetic field, but you can just as well keep the wire stationary and move the magnetic field instead.

So i see no issue with determining what end is positive, its the one that has current is flowing towards it.
Title: Re: Does Kirchhoff's Law Hold? Disagreeing with a Master
Post by: Sredni on November 25, 2018, 04:27:27 pm
I think the charge moves in the wire even if it's not a closed loop, if you could measure it you'd see a + q in one end and a minus q in the other end.

And which end will have a plus and which will have a minus?
Imagine your AB segment in a uniformly distributed time-varying B field. It is all the same, spatially.
Which end will get the plus, and which end will get the minus?

Can't decide?
Well, let's create a square loop with AB as its side. Use the right hand rule to find out how the current will flow with that flux varying configuration. Now you can tell me which extreme of AB is plus and which is minus, correct?
Except...

That it all depends which 'side of the loop' AB is on.

Think of two square clocks with a common AB side. Is the seconds hand going up or down? Does it depends on which clock the hand belongs to?

Sure you can decide. You follow the usual rules of direction with magnetic fields.


And you are the one who does not shift goalposts, eh?

There is no motional emf in Lewin's circuit.
In your example, the velocity of the moving bar is breaking the symmetry.
Please answer the question:

How do you decide which extreme of the bar has the plus and which has the minus when your bar is STATIONARY with respect to a SPATIALLY UNIFORM but TIME-VARYING B field?
Are you capable of answering THIS question without changing the problem?

Quote
This example shows the wire moving trough the magnetic field, but you can just as well keep the wire stationary and move the magnetic field instead.

Not the same problem. Please answer the question above.

Example, if I ask
"What it 9 divided by 4"
you should not answer
"9 divide by 3 is 3".

How do you decide which side is plus and which is minus in the case of a bar, stationary with a spatially uniform, time-varying magnetic field?

Same question applies to GeorgeoftheJungle, of course. He too did not answer.

EDIT: As for the antenna example, of course that breaks KVL as well, but we are trying to keep things simple here. So our circuit is in the domain of quasi-static electrodynamics, where the d/dt of the field is so small that the concatenation of B and E fields dies off in a very short distance. We are in fact disregarding the displacement current in Ampere-Maxwell's equation. So, no radiation.

EDIT: removed emoticon.
Title: Re: Does Kirchhoff's Law Hold? Disagreeing with a Master
Post by: Berni on November 25, 2018, 05:16:16 pm

And you are the one who does not shift goalposts, eh?

There is no motional emf in Lewin's circuit.
In your example, the velocity of the moving bar is breaking the symmetry.
Please answer the question:

How do you decide which extreme of the bar has the plus and which has the minus when your bar is STATIONARY with respect to a SPATIALLY UNIFORM but TIME-VARYING B field?
Are you capable of answering THIS question without changing the problem?

Sorry, my mistake there. I forgot about the uniform varying field in the original question.

In this case you can simply apply the left hand rule to determine the positive end. Align your thumb with the field and the fingers show the direction of current.

The only case where this is problematic is when a wire is straight since we can't determine if a straight line is bending towards the clockwise or counter clockwise direction. But such a straight wire would not generate any EMF so it doesn't have a direction.

If you prefer to solve this with Faradays law you can also just connect the ends of the wire with a straight line in this case. That straight line will not generate any EMF but will close the loop so that you can take an integral of the field going trough its surface area, so whatever the result of Faradays law is the EMF of that wire segment.(This only works in a uniform field tho as otherwise you can get EMF on a straight wire)

EDIT: Made a mistake here on how open loops work, see this thread for explanation: https://www.eevblog.com/forum/chat/does-kirchhoffs-law-hold-disagreeing-with-a-master/msg1993139/#msg1993139 (https://www.eevblog.com/forum/chat/does-kirchhoffs-law-hold-disagreeing-with-a-master/msg1993139/#msg1993139)

EDIT: As for the antenna example, of course that breaks KVL as well, but we are trying to keep things simple here. So our circuit is in the domain of quasi-static electrodynamics, where the d/dt of the field is so small that the concatenation of B and E fields dies off in a very short distance. We are in fact disregarding the displacement current in Ampere-Maxwell's equation. So, no radiation.

I do agree it is a more complex example and is more difficult to reproduce, but it does show a case of something that you can't properly model using lumped circuit meshes. Because you can't produce a lumped circuit that also means you can't use Kirchhoffs laws.

Dr. Lewins example can be easily modeled accurately using lumped circuit meshes. Once you have a lumped circuit you can apply Kirchhoffs laws and they work perfectly fine.

So i suppose RF engineers can indeed say "Kirchhoffs law is for the birds", most other engineers, not so much.
Title: Re: Does Kirchhoff's Law Hold? Disagreeing with a Master
Post by: GeorgeOfTheJungle on November 25, 2018, 05:25:04 pm
How do you decide which side is plus and which is minus in the case of a bar, stationary with a spatially uniform, time-varying magnetic field?
Same question applies to GeorgeoftheJungle, of course. He too did not answer.

A varying magnetic field pushes q in one direction, that's how you know where q is going to move to. Berni has even drawn it for you. What am I missing?
Title: Re: Does Kirchhoff's Law Hold? Disagreeing with a Master
Post by: Berni on November 25, 2018, 05:46:03 pm
How do you decide which side is plus and which is minus in the case of a bar, stationary with a spatially uniform, time-varying magnetic field?
Same question applies to GeorgeoftheJungle, of course. He too did not answer.

A varying magnetic field pushes q in one direction, that's how you know where q is going to move to. Berni has even drawn it for you. What am I missing?

Yes but that picture is not very helpful for a varying uniform field. When you have a varying non uniform field you get induction in that straight piece of wire because the field lines appear to be moving in relation to the wire.

When you have a varying uniform field its only the magnitude of the field that changes, the actual field lines stay in the same place. This field will try to push electrons in a circle around the field lines. This can be imagined as every field line trying to get electrons to circle around it at the same time. On a straight wire this makes the electron get pulled in both directions simultaneously since some field lines are on one side and some on the other side of the wire. Once you put a bend in the wire this makes the electrons easier to move in the direction the wire is bending. So as a result the fields on the outside of the bend are mostly pushing the electrons into the side of the wire while the fields on the inside of the curve are pushing electrons more along the direction of the wire. This makes the fields on the inside of the bend win out and start moving electrons along the wire according to the left hand rule.

Sorry if this explanation is not very scientific but i think it makes the concept easier to grasp.

EDIT: Made a mistake here on how open loops work, see this thread for explanation: https://www.eevblog.com/forum/chat/does-kirchhoffs-law-hold-disagreeing-with-a-master/msg1993139/#msg1993139 (https://www.eevblog.com/forum/chat/does-kirchhoffs-law-hold-disagreeing-with-a-master/msg1993139/#msg1993139)
Title: Re: Does Kirchhoff's Law Hold? Disagreeing with a Master
Post by: GeorgeOfTheJungle on November 25, 2018, 06:37:26 pm
How do you decide which side is plus and which is minus in the case of a bar, stationary with a spatially uniform, time-varying magnetic field?
Same question applies to GeorgeoftheJungle, of course. He too did not answer.

A varying magnetic field pushes q in one direction, that's how you know where q is going to move to. Berni has even drawn it for you. What am I missing?

Yes but that picture is not very helpful for a varying uniform field. When you have a varying non uniform field you get induction in that straight piece of wire because the field lines appear to be moving in relation to the wire.

When you have a varying uniform field its only the magnitude of the field that changes, the actual field lines stay in the same place. This field will try to push electrons in a circle around the field lines. This can be imagined as every field line trying to get electrons to circle around it at the same time. On a straight wire this makes the electron get pulled in both directions simultaneously since some field lines are on one side and some on the other side of the wire. Once you put a bend in the wire this makes the electrons easier to move in the direction the wire is bending. So as a result the fields on the outside of the bend are mostly pushing the electrons into the side of the wire while the fields on the inside of the curve are pushing electrons more along the direction of the wire. This makes the fields on the inside of the bend win out and start moving electrons along the wire according to the left hand rule.

Sorry if this explanation is not very scientific but i think it makes the concept easier to grasp.

Oh, ok, (I think) I get it. Thanks!

When I said "the charge moves in the wire even if it's not a closed loop, if you could measure it you'd see a + q in one end and a minus q in the other end" I was thinking in ElectroBooms' setup @ 9m13s: youtu.be/0TTEFF0D8SA?t=9m13s
Title: Re: Does Kirchhoff's Law Hold? Disagreeing with a Master
Post by: Berni on November 26, 2018, 06:50:32 am
You do realize you are making up rules on the fly, do you?
So, let me get this straight (pun intended).
Let's consider a square loop, with AB side perfectly straight. According to your model, there is no 'partial emf' on that side, right? And if you bend it a bit, the sign of the 'partial emf' will change according to the curvature of that side?
Or, let's just consider an AB segment alone: according to your rule: when it is straight there is no charge build up, but if it is bent in one way the charges are + on A and - on B, while if it is bent the other way the charges are - on A and + on B, correct?

Man, am I glad I do not live in the same universe as you. Looks pretty much more complicated than the universe I am in.

Its not the curvature itself that causes voltage, but the overall trend the wire is taking. A square is still bending around to form a loop that creates the usual direction. An example that seams to go both ways is a S shaped line. If you connect the ends of it you get a clockwise and counterclockwise loop so the surface area integrates to zero if both are symmetrical. If you did connect the ends you would also get zero current due to the area being zero. Electrons in the positive side of the S curve can't magically know about the ones in the negative side and not move because of it. The path the wire takes causes fields on the inside of the curves overall trend to overlap causing one side of the wire to start having more effect than the other.

I will admit i don't fully understand the underlying magic that determines why things move in the specific directions inside fields but the overall effects this causes seam to point towards this.

I would love to live in a simpler universe but magnetic and electric effects are linked trough the effects of Einsteins relativity and that stuff does all sorts of weird things.

KVL breaks in the case of radiation, correct. But it also breaks in the case of induction.

It worked just fine for me in the case of Dr. Lewins experiment so do we perhaps need a different experiment to show it not working with induction?



So for the case of open loops still producing charge separation i propose the flowing experiment:
(https://www.eevblog.com/forum/chat/does-kirchhoffs-law-hold-disagreeing-with-a-master/?action=dlattach;attach=580757;image)

We break a loop by putting a capacitor in series with it. This capacitor can be formed by two metal plates just like the symbol so there is a real air gap between the ends of the wire. No electrons can jump the gap between the plates. The capacitor requires a change in voltage to push electrons onto the plate and off the other plate in order to create the illusion of current flowing trough it.

So what would happen in this circuit when the uniform magnetic field is suddenly turned on?



(https://www.eevblog.com/forum/chat/does-kirchhoffs-law-hold-disagreeing-with-a-master/?action=dlattach;attach=580763;image)
But one could argue that that's just a small gap so we could just ignore it. So lets take it a step further and introduce a 2nd capacitor on the other side to completely cut the loop in half.

Can the 2 completely separated segments of wire generate a voltage on the capacitor to get the current flowing?


By the way this example is also possible to build as a real life experiment. There is even a way to measure the current in this loop without actually connecting any test equipment to the loop. Circuit analysis tells us that this is a LC circuit, if it indeed is one it will hold on to the pulse of energy and use it to oscillate back and forth, generating an AC magnetic field around the loop even after we remove the original field. We can measure this field as proof of current in the loop.
Title: Re: Does Kirchhoff's Law Hold? Disagreeing with a Master
Post by: Sredni on November 26, 2018, 03:47:19 pm
Its not the curvature itself that causes voltage, but the overall trend the wire is taking. A square is still bending around to form a loop that creates the usual direction.

It looks like you are trying to say you can define emf only if you can identify an area to associate to it.
Well, that's progress.  That's what Faraday has tried to tell you since the nineteenth century.

Quote
I will admit i don't fully understand the underlying magic that determines why things move in the specific directions inside fields but the overall effects this causes seam to point towards this.

Yes, it points towards an area, encircled by a closed (possibly fictional, in the sense it does not have to be all inside real conductors) loop.
As the formula stating Faraday's law has always said.

Quote
I would love to live in a simpler universe but magnetic and electric effects are linked trough the effects of Einsteins relativity and that stuff does all sorts of weird things.

Correct, but not as weird as you think.
I suggest to brush up your physics on some good book, like Purcell (Electricity and Magnetism, second volume of the Berkeley physics series), and then look up the practical applications of the basic concepts in books like Ramo, Whinnery, VanDuzer (Fields and Waves in Communication Electronics).

As for the further goalpost shifting at the end of your post, please... Leave caps out - we are trying to keep things simple here. If we are having trouble understanding each other with a simple circuit like that, what do you think would happen if you introduce another paradox generating element, like the two caps back to back?

And no, KVL did not work with Lewin's circuit. You had to introduce that magical emf term to make your numbers check. That's Faraday at work. In fact, you cannot locate that voltage anywhere with a voltmeter, can you? I am talking about that circuit, do not try to modify it. Let down those scissors, I tell you!!!

EDIT: Repetitia juvant.
What happens if we pull the resistors out of the loop and make sure we cannot interfere with the flux that is generating the emf? That we have a series of two resistors and a black box with two terminals. Now you can call that the secondary of a hidden transformer. Now you can located the voltage it 'generates' with a voltmeter. It's right there, at its two terminals! Now you can delude yourself KVL works, and call it, instead of Faraday's Law, "extended KVL" or "modified KVL" or "modern KVL". Lumped circuit theory works, all voltages we can measure are uniquely defined. Now the quarrel "KVL vs Faraday" is just a language barrier.
But when the resistors are inside the loop, say goodbye to lumped circuit theory and uniquely defined voltages. You have to take paths into account.
Title: Re: Does Kirchhoff's Law Hold? Disagreeing with a Master
Post by: Berni on November 26, 2018, 07:44:12 pm

It looks like you are trying to say you can define emf only if you can identify an area to associate to it.
Well, that's progress.  That's what Faraday has tried to tell you since the nineteenth century.

Alright i have to apologies because i did get one detail about voltage in an open loop wrong. Straight wires require a special case to produce no EMF in the field (I will go back and add a note about this in my previous posts)

I finally found an article that explains how voltages in stationary open loops work when exposed to a varying uniform field:
Induced voltage in an open wire by K. Morawetz, M. Gilbert, A. Trupp
www.eevblog.com/forum/chat/does-kirchhoffs-law-hold-disagreeing-with-a-master/?action=dlattach;attach=581270 (https://www.eevblog.com/forum/chat/does-kirchhoffs-law-hold-disagreeing-with-a-master/?action=dlattach;attach=581270)


(https://www.eevblog.com/forum/chat/does-kirchhoffs-law-hold-disagreeing-with-a-master/?action=dlattach;attach=581273;image)

It is indeed solved by closing the loop using a wire flowing a path that generates no EMF. However such a wire does not directly connect the ends of the open loop wire segment. What is required instead is two straight wires that travel to the geometric origin of the uniform magnetic field. Such straight wires that pass trough or touch the geometric origin point of the field generate no EMF. This was causing problems for me because i did not realize that even ideal uniform fields have a center point. However in the case of a close loop coil the center point becomes irrelevant because it affects the entire loop and averages out by the time you get all the way around. So technically the beloved closed loop is just a special case of a open loop that closes in on itself(Or the open loop being a special case of a closed loop, whatever way around you want to think about it).

So there you do get voltage induced in a open loop of wire (Proven using Maxwell even).



Correct, but not as weird as you think.
I suggest to brush up your physics on some good book, like Purcell (Electricity and Magnetism, second volume of the Berkeley physics series), and then look up the practical applications of the basic concepts in books like Ramo, Whinnery, VanDuzer (Fields and Waves in Communication Electronics).

I do have at least some understanding on all of those areas, just that some i don't know well in to the detail and especially not down into the deep math behind it. Its just not something i deal with on a regular basis. Electronics engineering has so many abstractions in place that there is no need to delve this deep into the fundamental math under it all. Hence why most electronics engineers know about Maxwell and what he did, but they never used his equations on the job. Its just easier and faster to use the derived "easy bake" equations for calculating everything you would need, but if you dig down and dissect a lot of those equations you tend to find some Maxwells equation somewhere in there.

Contrary to popular belief engineers are mostly not math geniuses, they are just really good at looking up the right equation and quickly punching it into a calculator. Its simply the fastest way to get work done on a deadline.


As for the further goalpost shifting at the end of your post, please... Leave caps out - we are trying to keep things simple here. If we are having trouble understanding each other with a simple circuit like that, what do you think would happen if you introduce another paradox generating element, like the two caps back to back?

The capacitor in an inductive loop is just my proposed experiment to show that an open loop can generate a voltage. Circuit analysis is well understood for RLC circuits so we can easily use it to predict the behavior, then actually do the experiment and see if the results match. Capacitors don't create any more of a paradox than inductors.  Can you propose a simpler experiment that shows or disproves the presence of voltage in open loops of wire (aka fractional turns)? If the experiment can be done with equipment and materials found in a reasonable electronics lab i will recreate it.

The purpose of this experiment was to show that fractional turns can indeed take part in a circuit and pick up EMF just like complete loops can.

And no, KVL did not work with Lewin's circuit. You had to introduce that magical emf term to make your numbers check. That's Faraday at work. In fact, you cannot locate that voltage anywhere with a voltmeter, can you? I am talking about that circuit, do not try to modify it. Let down those scissors, I tell you!!!
Its not introducing a magical emf out of nowhere.

The cirucit mesh model simply needs to know about the properties of a wire. You do agree that a coil of wire with 100 turns placed across two points in a circuit is modeled using an inductor symbol right? Well these inductors are not closed loops as they have two terminals that connect to other components of a circuit (just like a straight wire).

A straight piece of wire is basically the same thing except with much less inductance since magnetic flux is not being reused multiple times on the same wire by coiling. This website provides a helpful calculator for this: https://www.eeweb.com/tools/wire-inductance (https://www.eeweb.com/tools/wire-inductance) . Among other things it also provides a calculator for loop inductance that comes useful later(mutual inductance)

If you place two such 100 turn coils in close proximity you can get some of the same magnetic field passing trough both coils. This turns them into coupled inductors where they not only have self inductance, but also something called a mutual inductance (This is essentially a transformer). The value of self and mutual inductance for each is all that is needed to describe the magnetic properties of them. Any number of coils can be added to this magnetically coupled inductor, not just two. The coupled inductor model is the "mathematical adapter" that brings Maxwells equations into a form that fits into circuit analysis theory. Once it fits inside the circuit analysis abstraction all other circuit analysis tools can be applied(Kirchhoff being only one of them). Its sort of like a software API, but with math rather than code. By putting an inductor into the mesh we simply create an instance of the inductor model that deals with magnetic effects for us.

So since a straight piece of wire is simply a inductor with less turns than a coil of wire we can model a piece of wire in the exact same way. Tho due to it having essentially zero turns means the self inductance is pretty low(but NOT zero) while the mutual inductance is likely significantly larger as soon as it forms a larger loop with other components of the circuit that it connects to. This mutual inductance is where the loop inductance is if you connect multiple segments of wire together into a complete loop.

We are still using Maxwells equations and Faradays law deep inside the equation that calculated the inductance value in Henrys. So why is modeling a length of wire as an inductor incorrect? Is using Maxwells equations as part of another equation forbidden?


EDIT: Repetitia juvant.
What happens if we pull the resistors out of the loop and make sure we cannot interfere with the flux that is generating the emf? That we have a series of two resistors and a black box with two terminals. Now you can call that the secondary of a hidden transformer. Now you can located the voltage it 'generates' with a voltmeter. It's right there, at its two terminals! Now you can delude yourself KVL works, and call it, instead of Faraday's Law, "extended KVL" or "modified KVL" or "modern KVL". Lumped circuit theory works, all voltages we can measure are uniquely defined. Now the quarrel "KVL vs Faraday" is just a language barrier.
But when the resistors are inside the loop, say goodbye to lumped circuit theory and uniquely defined voltages. You have to take paths into account.

Yes that is correct, see its not that hard to think in terms of circuit mesh models. You can indeed fix things by adding a black box transformer into the circuit. However Dr. Lewins experiment is about the voltage on points A and B. Once we lump all of the loop inductance into one black box we loose points A and B.

But wait! We can fix that. Instead of lumping all of it into one black box we can just lump each wire segment into its own black box. This way we get 4 such black boxes that are located between the resistors terminals and the points of interest.

(https://www.eevblog.com/forum/chat/does-kirchhoffs-law-hold-disagreeing-with-a-master/?action=dlattach;attach=581279;image)

This way points A and B are maintained while each black box is now one of the 4 secondary coils to the solenoid coil creating the original field. They all know about each other trough mutual inductance. If we suddenly also want a point C that's halfway between A and the left resistor we simply cut the black box into two blackboxes with half the inductance value. Point C now pops out as the midpoint between the two new blackboxes. This is why i was trying to explain above that open wire segments can interact with magnetic fields just as much as wire loops can, the black boxes are models of a open wire segment.

As i said KVL is not just a different form of Faradays law. Its is like comparing an apple to a car, they are two completely different things. Faradays law calculates the relationship between voltage and magnetic flux change in loops. Kirchhoffs voltage law just calculates voltage relationships in abstract electrical circuit meshes. It has nothing to do with magnetic or electric fields. Its just a law that is part of circuit analysis methods, those will then call upon Faradays law whenever circuits have to deal with inductance. Circuit analysis wraps Faradays law into the form of an inductor. The process of mesh analysis eventually marries together the equation for an inductor model (That contains Faradays law deep inside it) with Kirchhoffs voltage and current laws to produce a mathematical model of the circuit. Both KVL and Faradays law exists together in that resulting circuit equation, you can't see it in the form that Faradays law is written but if you expand the equations backwards far enough you could eventually get it to pop out Faradays law in textbook form. KVL can't deal with anything other than voltages so it relies on other laws to do it instead. It can't even understand resistance, it needs help from Ohms law to tell it the voltage on a resistor. Because of that we don't say KVL is for the birds because Ohms law handles it better.

So the only thing that Faradays law and KVL have in common is that during circuit analysis KVL makes use of Faradays law to be able to understand magnetic fields. They are great friends even if circuit analysis sometimes demands it to do unusual things such as fractional loop segments(That still work fine, see above). Is it forbidden to plug the results from one law into another law?

Circuit models are not meant to describe the underlying physics, they are about as real as the imaginary part of complex numbers is real. But circuit models use just enough physics to accurately describe the behavior of the circuit on a macroscopic level.
Title: Re: Does Kirchhoff's Law Hold? Disagreeing with a Master
Post by: radioactive on November 27, 2018, 11:17:29 pm
I've modified one of the openEMS examples to try and simulate this experiment numerically via the openEMS FDTD field solver.  In the attached image of the geometry, you can see the physical model of the excitation coil with feedline (copper) and resistor,  the wire loop (copper), two resistors (100/900 ohms)  / voltage/current probe elements, and two infinite resistance voltage probes measuring the voltage on the wire segments on either side of the resistors. The infinite resistance flags openEMS not to add any material properties for these probes (so they are unaffected by the fields and cause no loading).  The mesh is something close to 4 million cells which is probably a bit overkill, but wanted to make sure everything got integrated.  The simulation is configured to dump the volume of the magnetic field (H-field) and results in about 12GB of vtr files that you can view as an animation in paraview.  The attached matlab file can be used to create the geometry file and run the openEMS simulation.

[edit] removed sim files.  I'm an idiot.  I'l post a working sim when I have it finished.
Title: Re: Does Kirchhoff's Law Hold? Disagreeing with a Master
Post by: bsfeechannel on November 28, 2018, 12:41:01 am
I will admit i don't fully understand the underlying magic that determines why things move in the specific directions inside fields but the overall effects this causes seam to point towards this.

I would love to live in a simpler universe but magnetic and electric effects are linked trough the effects of Einsteins relativity and that stuff does all sorts of weird things.

Our brains are not hardwired to admit space being "warped" by any kind of changing field, be it magnetic, electric, gravitational or whatever.

Fortunately our brains are equipped with reason, so we can painstakingly break free from our basic intuitions and build a reasoning that will lead us to new levels of understanding. It takes time and effort, but it's worth the journey.

You might know that Maxwell is a theory that accounts for our four little happy dimensions of the spacetime in which we are immersed. The road to Maxwell starts with calculus, which is nothing more than the mathematical (i.e. formal) study of change. You know, we live in an ever changing universe. So someone had to invent a theory to give us tools to deal with that.

I happen to have a happy little video about calculus.

Calculus For Young Players - BSFEEChannel #2

https://www.youtube.com/watch?v=mT6FGdfeCNo (https://www.youtube.com/watch?v=mT6FGdfeCNo)

That won't make anyone an expert but is going to give viewers an idea of what it is and how to immediately apply it to simple electronic circuits. However, if the viewer got interested in learning more, a good introduction to calculus is the series of 31 lectures given by Professor Richard Delaware at UMKC, which inspired my video above.

https://www.youtube.com/watch?v=CtRAHmeWSC0&list=PL65FC200C611F0E27 (https://www.youtube.com/watch?v=CtRAHmeWSC0&list=PL65FC200C611F0E27)

Then you'll come to vector calculus, also known as vector analysis. This is the theory that will give you tools to deal with change over whatever lines, surfaces, and volumes, because things are not all in the same place at the same time. There is no absolute simultaneity in the universe (blame Einstein).  A good start is the book Electromagnetic Waves and Radiating Systems by Edward C. Jordan and Keith G. Balmain. The first chapter is dedicated to introduce you to vector analysis. Interestingly enough, the first section discusses the limitations of circuit theory (i.e. Kirchhoff) without neglecting its importance, while demonstrates why the more complicated field theory (a.k.a Maxwell) is worth the effort.

Now that you upgraded your brain to think fourth-dimentionally, you can tackle Maxwell with ease. And that's what that book does when you reach chapter 4.

The cool thing about electromagnetism is that, since the electromagnetic force is 10³⁶ times stronger than gravity, mind boggling things that defy common sense happen right on top of your bench at human scale compared to gravitational phenomena that are only relevant at astronomical scale. This video shows just an example.

https://www.youtube.com/watch?v=sENgdSF8ppA (https://www.youtube.com/watch?v=sENgdSF8ppA)
Title: Re: Does Kirchhoff's Law Hold? Disagreeing with a Master
Post by: bsfeechannel on November 28, 2018, 12:56:32 am
I do have at least some understanding on all of those areas, just that some i don't know well in to the detail and especially not down into the deep math behind it. Its just not something i deal with on a regular basis. Electronics engineering has so many abstractions in place that there is no need to delve this deep into the fundamental math under it all. Hence why most electronics engineers know about Maxwell and what he did, but they never used his equations on the job. Its just easier and faster to use the derived "easy bake" equations for calculating everything you would need, but if you dig down and dissect a lot of those equations you tend to find some Maxwells equation somewhere in there.

Contrary to popular belief engineers are mostly not math geniuses, they are just really good at looking up the right equation and quickly punching it into a calculator. Its simply the fastest way to get work done on a deadline.

If we engineers deliberately neglect the fundamentals, how can we criticize Lewin who is teaching them?
Title: Re: Does Kirchhoff's Law Hold? Disagreeing with a Master
Post by: radioactive on November 28, 2018, 01:04:27 am
The explanation video promised by Prof. Walter Lewin.

https://www.youtube.com/watch?v=AQqYs6O2MPw (https://www.youtube.com/watch?v=AQqYs6O2MPw)

I haven't made it all the way through this entire thread yet, but that response video was excellent.  I still need to watch his lecture #20.  This quick video should clear things up for anyone who watches it though.
Title: Re: Does Kirchhoff's Law Hold? Disagreeing with a Master
Post by: ogden on November 28, 2018, 02:25:35 am
If we engineers deliberately neglect the fundamentals, how can we criticize Lewin who is teaching them?

I would not call high level abstraction as negligence. It is common sense. We criticize Dr.Lewin because he neglect fundamentals himself (read "Lewin’s Circuit Paradox" by Dr. Kirk T. McDonald - you'll see). I do not see anybody who is against Maxwell's equations or saying that Kirchhoff's law *always* hold. Those who do not agree (to "KVL for the birds") say that Kirchhoff’s loop equations apply to Lewin’s circuit.
Title: Re: Does Kirchhoff's Law Hold? Disagreeing with a Master
Post by: bsfeechannel on November 28, 2018, 02:37:35 am
If we engineers deliberately neglect the fundamentals, how can we criticize Lewin who is teaching them?

I would not call high level abstraction as negligence. It is common sense. We criticize Dr.Lewin because he neglect fundamentals himself (read "Lewin’s Circuit Paradox" by Dr. Kirk T. McDonald - you'll see). I do not see anybody who is against Maxwell's equations or saying that Kirchhoff's law *always* hold. Those who do not agree (to "KVL for the birds") say that Kirchhoff’s loop equations apply to Lewin’s circuit.

So, you criticize Lewin because someone else wrote an article criticizing him? Not because you yourself master the fundamentals and is in a position to confront him? What kind of engineers do we want to be? Just a bunch of dilettantes ranting at random in forums? Let's shut up and do our homework. Thank you.
Title: Re: Does Kirchhoff's Law Hold? Disagreeing with a Master
Post by: ogden on November 28, 2018, 03:00:48 am
We criticize Dr.Lewin because he neglect fundamentals himself (read "Lewin’s Circuit Paradox" by Dr. Kirk T. McDonald - you'll see). I do not see anybody who is against Maxwell's equations or saying that Kirchhoff's law *always* hold. Those who do not agree (to "KVL for the birds") say that Kirchhoff’s loop equations apply to Lewin’s circuit.

So, you criticize Lewin because someone else wrote an article criticizing him? Not because you yourself master the fundamentals and is in a position to confront him? What kind of engineers do we want to be?

I already provided my position here in this thread. That's why I just refer to article I agree to and do not repeat what is already said. Don't blame me if you did not read thread or do not remember what I did say or whatever.

Quote
Just a bunch of dilettantes ranting at random in forums? Let's shut up and do our homework. Thank you.

You better behave
Title: Re: Does Kirchhoff's Law Hold? Disagreeing with a Master
Post by: bsfeechannel on November 28, 2018, 03:59:13 am

I already provided my position here in this thread. That's why I just refer to article I agree to and do not repeat what is already said. Don't blame me if you did not read thread or do not remember what I did say or whatever.

Yeah, you said it clearly. And I quote.

Those who do not agree (to "KVL for the birds") say that Kirchhoff’s loop equations apply to Lewin’s circuit.

No they don't. Suppose that Kirchhoff didn't know about Faraday's law of induction. I show a loop of wire with two resistors and hide the solenoid under the table. I ask Kirchhoff to measure the voltages with a voltmeter. Kirchhoff wouldn't know how to explain how a loop of wire with two resistors and no voltage source has some voltage on them. Worse, he wouldn't know how to explain why the voltmeter shows diferent voltages depending on the position of the voltmeter.

Kirchhoff wouldn't know how to explain how two pieces of wire hanging out in the breeze (that today we call dipole antenna) suddenly have voltages and currents without no visible voltage source connected to them.

Although Faraday demonstrated the phenomenon of induction in 1831, he couldn't find a mathematical formulation and therefore his theories were rejected by the scientists of the time. Meanwhile in 1845 Kirchhoff came up with his law that do not account for any kind of varying field for that matter. In 1865, Maxwell could finally produce his now famous equations that gave a mathematical formulation to Faraday's law.

Kirchhoff died in october 1887. Hertz published his first paper confirming Maxwell's equations in november 1887.

So there is no paradox in Lewin's explanations. Kirchhoff died absolutely ignorant of the confirmation of Maxwell's theory so that's why his theory doesn't account for that. Period.

So who told you that the loop is a secondary of a transformer? Certainly not Kirchhoff. Because when that theory could finally be confirmed, he was DEAD. End of story.

Quote
You better behave

Here's a dollar, kid. Go get yourself a better education.
Title: Re: Does Kirchhoff's Law Hold? Disagreeing with a Master
Post by: ogden on November 28, 2018, 05:26:22 am
Kirchhoff died absolutely ignorant of the confirmation of Maxwell's theory so that's why his theory doesn't account for that. Period.

You think that Kirchoff's circuit laws shall explain electromagnetism? - Better save your dollar to get better education yourself, kid:

Here's a dollar, kid. Go get yourself a better education.
Title: Re: Does Kirchhoff's Law Hold? Disagreeing with a Master
Post by: Siwastaja on November 28, 2018, 05:48:27 am
Here's a dollar, kid. Go get yourself a better education.

Don't you see how detrimental this attitude is to science and understanding?

IMHO, this is also the actual mistake Lewin is making.

Honest questioning and discussion should be valued.

Not everyone is as strong as a person as you or me or ogden. Not everyone wants to fight. Some may actually get upset about they way Lewin arrogantly attacks their "qualifications" when they just asked honest questions, especially if you are in the role of a student. They won't ask more questions, but instead, do as you teach them to do: close up your desire for understanding, just shut up and "learn". But, just reading isn't the way you learn science. You need to really understand, and for that, asking questions, yes, even questioning your professor - would be the right thing to do.

But I guess we'll disagree on this. And, I think you are genuinely a bad person for not even trying to think about this side of the coin - and the life goes on :).
Title: Re: Does Kirchhoff's Law Hold? Disagreeing with a Master
Post by: Berni on November 28, 2018, 07:14:27 am

So, you criticize Lewin because someone else wrote an article criticizing him? Not because you yourself master the fundamentals and is in a position to confront him? What kind of engineers do we want to be? Just a bunch of dilettantes ranting at random in forums? Let's shut up and do our homework. Thank you.

Okay i know there are a lot of opinions in this thread. But i personally was never trying to say that Dr. Lewin is a bad teacher. Id love to have him as my physics teacher when compared to all the teachers i have had. He does a great job of explaining things in a interesting way to help you understand it. I watched some of his other lectures and i enjoyed them.

His statements about KVL is the only part where i get to disagree with him. Nobody is saying Maxwell or Faraday is wrong, nobody is trying to prove them being wrong (You would have one hell of a time trying to do that). It just seams to me that Dr. Lewin has a different idea of what KVL actually is, this is what is leading him to the conclusion of KVL being just a special case of Faradays law with no magnetic field (Its not, you can't even stick an electric field into KVL).


No they don't. Suppose that Kirchhoff didn't know about Faraday's law of induction. I show a loop of wire with two resistors and hide the solenoid under the table. I ask Kirchhoff to measure the voltages with a voltmeter. Kirchhoff wouldn't know how to explain how a loop of wire with two resistors and no voltage source has some voltage on them. Worse, he wouldn't know how to explain why the voltmeter shows diferent voltages depending on the position of the voltmeter.

Kirchhoff wouldn't know how to explain how two pieces of wire hanging out in the breeze (that today we call dipole antenna) suddenly have voltages and currents without no visible voltage source connected to them.

Although Faraday demonstrated the phenomenon of induction in 1831, he couldn't find a mathematical formulation and therefore his theories were rejected by the scientists of the time. Meanwhile in 1845 Kirchhoff came up with his law that do not account for any kind of varying field for that matter. In 1865, Maxwell could finally produce his now famous equations that gave a mathematical formulation to Faraday's law.

Kirchhoff died in october 1887. Hertz published his first paper confirming Maxwell's equations in november 1887.

So there is no paradox in Lewin's explanations. Kirchhoff died absolutely ignorant of the confirmation of Maxwell's theory so that's why his theory doesn't account for that. Period.

So who told you that the loop is a secondary of a transformer? Certainly not Kirchhoff. Because when that theory could finally be confirmed, he was DEAD. End of story.

Kirchhoff indeed did not know about Faraday or Maxwell at the time. He came up with his laws quite a few years before them. He did not understand induction and was not even trying to. He was dealing with circuits of resistors powered by batteries. Science at the time did not understand the underlying principle of how magnetism and electricity interact, but they knew very well that they do interact and experimentally determined a lot of the rules it follows. Its only when Maxwell came around with his equations that they had a concrete mathematical explanation of how the magnetic and electric worlds interact. They knew about induction and loop area and even had equations that can calculate it before that time, but those equations simply came from applying the best fitting equation onto the experimental results. Maxwells equations turned out to fit perfectly in all of those experimentally determined equations so he knew he was onto something big (Its incredibly unlikely this would happen to fit so many equations by chance).

Maxwell didn't go "Hah suck it Faraday, your math is wrong cause mine works so much better". He was instead using the work of Faraday and many other scientists to be able to come up with his equations. He surely went trough a lot of trial and error with mental models and math before he found a set of equations that fit in. Much like other scientist did before him to make other famous equations. Science builds on top of itself.

In science we can't know what is right and what is wrong. But we can use scientific methods to find the theories that seam to fit our universe the best. Every so often a new theory comes up that explains something better and so its adopted as the new best thing. Heck for a long time we didn't even know what atoms and molecules look like, we went trough increasingly better ideas of what they are. Eventually we even figured out that something as fundamental as a electron can be taken apart into even more fundamental building blocks.

Kirchhoffs has many other contributions in other fields, but in the electrical field his contribution is used for circuit analysis, much like Thevenins or Nortons theorem. These things don't deal with electrical and magnetic fields, inches, permeability etc... They ONLY deal with volts and amps flowing trough ideal components. So Maxwells equations don't really have much to do with KVL. Where Maxwells equations touch circuit analysis laws is during circuit modeling. Maxwells equations are used deep down to turn a physical coil of wire into how many Henrys an ideal inductor model should have to act like that coil. After that step circuit analysis uses the inductor model without knowing about Maxwell even tho Maxwells equations are hidden inside that inductance value.
Title: Re: Does Kirchhoff's Law Hold? Disagreeing with a Master
Post by: bsfeechannel on November 29, 2018, 03:16:05 am
Don't you see how detrimental this attitude is to science and understanding?

Now we're talking.

Quote
IMHO, this is also the actual mistake Lewin is making.

Honest questioning and discussion should be valued.

Not everyone is as strong as a person as you or me or ogden. Not everyone wants to fight. Some may actually get upset about they way Lewin arrogantly attacks their "qualifications" when they just asked honest questions, especially if you are in the role of a student. They won't ask more questions, but instead, do as you teach them to do: close up your desire for understanding, just shut up and "learn". But, just reading isn't the way you learn science. You need to really understand, and for that, asking questions, yes, even questioning your professor - would be the right thing to do.

Unfortunately people are not questioning honestly. An honest question requires an open mind to accept the answer.

Lewin, I and numerous others on this forum and perhaps on other places spent time and effort patiently explaining that a lot of phenomena are out of reach of Kirchhoff's theory. We wrote texts, drew sketches, made videos, recommended lectures, books, etc.

What is the reaction? Cool, I didn't know about this, I am going to study?

No. The reaction was variously like I am a "practical" engineer dedicated to "high level abstractions", which is just a wanky name for "I don't know the fundamentals and I don't care" and I want you to come with an explanation that conforms to my limited understanding of physics.

Sorry, but that ain't gonna happen. It's impossible to dumb Maxwell down. You'll have to make room in your head to accept a new idea, a.k.a., learn.

Quote
But I guess we'll disagree on this. And, I think you are genuinely a bad person for not even trying to think about this side of the coin - and the life goes on :).

Forum member rfeecs published a link to a video by Cyriel Mabilde, who said he'd prove that Lewin was "wrong". My comment then was polite.

https://www.eevblog.com/forum/chat/does-kirchhoffs-law-hold-disagreeing-with-a-master/msg1987190/#msg1987190 (https://www.eevblog.com/forum/chat/does-kirchhoffs-law-hold-disagreeing-with-a-master/msg1987190/#msg1987190)

The fact is that that demonstration is a train wreck. Some of the errors were pointed out by rfeecs himself. Why is that?

At 33:17 Mabilde says: Invoking the theory of path dependency [i.e. Faraday/Maxwell] is not the right answer for explaining these simple measurements.

So he cannot interpret what he is measuring because he rejects the only theory capable of explaining what is going on right in front of his nose. To add insult to injury, his denial goes on: After all, spending more time and energy in a misinterpreted demonstration, I think it is time to tackle more serious problems, problems in this world, that concern all of us, such as the climate, injustice, violence, drugs, war and refugees.

He conveniently forgot to name ignorance as a world problem. Why? Because he chose ignorance. He chose not to learn. He chose to discredit Lewin instead. Because that's the easy path.

His video is a crime against humanity.

Who is being arrogant, after all? Mabilde and all those who recalcitrantly refuse to learn, or Lewin who dedicated an entire life to teaching?

This stubborn attitude is what is getting under our skin.

So don't fool yourself. You're not doing science a favor. If you really love science do as we all do: humbly learn.
Title: Re: Does Kirchhoff's Law Hold? Disagreeing with a Master
Post by: bsfeechannel on November 29, 2018, 04:19:57 am
It just seams to me that Dr. Lewin has a different idea of what KVL actually is, this is what is leading him to the conclusion of KVL being just a special case of Faradays law with no magnetic field (Its not, you can't even stick an electric field into KVL).

Kirchhoff (KVL) IS a special case of Faraday.

What does Maxwell say? That the EMF along a closed line in space is a function of how the surface integral of a magnetic field across an arbitrary surface bounded by the closed line varies in relation to time.

The magnetic field can vary in intensity and direction. The surface can vary in size, direction and/or shape (including the closed line).

When that surfce integral does not vary with time, EMF is zero. And that coincides exactly with what Kirchhoff said: that the sum of the voltages around a mesh (a closed line) is zero.

In what circumstances does the surface integral equal zero? When the magnetic field AND the surface are constant, for example, among many other cases.

So Kirchhoff ended up being a subset Faraday. I even drew it as a Venn diagram many messages ago in this thread.

(https://www.eevblog.com/forum/chat/does-kirchhoffs-law-hold-disagreeing-with-a-master/?action=dlattach;attach=583322;image)
Title: Re: Does Kirchhoff's Law Hold? Disagreeing with a Master
Post by: Berni on November 29, 2018, 07:39:32 am

Kirchhoff (KVL) IS a special case of Faraday.

What does Maxwell say? That the EMF along a closed line in space is a function of how the surface integral of a magnetic field across an arbitrary surface bounded by the closed line varies in relation to time.

The magnetic field can vary in intensity and direction. The surface can vary in size, direction and/or shape (including the closed line).

When that surfce integral does not vary with time, EMF is zero. And that coincides exactly with what Kirchhoff said: that the sum of the voltages around a mesh (a closed line) is zero.

In what circumstances does the surface integral equal zero? When the magnetic field AND the surface are constant, for example, among many other cases.

Yes this is what i find to be the issue. Faraday and Kirchhoff are put into the same basket while they describe different things.

Here is a definition of Faradays law:
"The electromotive force induced in a circuit by variation of the magnetic flux through the circuit is proportional to the negative of the time rate of change of the magnetic linkage"

Here is a definition of Kirchhoff voltage law:
"The algebraic sum of all the voltages around any closed loop in a circuit is equal to zero"

So Faradays law only talks about the induced EMF, it does not say anything about other voltages (Like voltage drops on components or batteries pushing voltage). This is very useful because we can use it to calculate the induced EMF. Kirchhoff on the other hand does talk about the voltages summing to zero. Notice that it says "all the voltages" so this implies induced EMF too as that's a voltage inside the loop. Also notice that it says "algebraic sum", this infers that the circuit mentioned afterwards is a circuit mesh model rather than a real circuit. Had it been a real circuit means that he would be using an integral of electric fields around the circuit, rather than just summing together voltages on components.

So if "all the voltages" in Kirchhoff law includes induced EMF voltage then KVL needs to know how big this voltage is. And how do we calculate that? Well we use Faradays law of course.

In a circuit mesh schematic a wire has zero length, zero resistance and zero reactance. These wires are immune to all field effects. Building the two resistor circuit with such wires in the real world results in a circuit that must have a circumference of zero, this means it also must have a loop area of zero, zero area means no induced EMF and KVL works. This however has a problem because Dr. Lewins circuit clearly has a loop area larger than zero so it makes the circuit mesh model act different(You get 0V on both resistors). We fix this by replacing out ideal wires with inductors. This plugs in Faradays law and tells it about how big the loop area is. Now the circuit model does get voltage induced in it and KVL still appears to work.

YES I KNOW that Kirchhoffs voltage law does indeed break in certain special cases, i'm not saying it always works. I know why it breaks in those special cases. But Dr. Lewins experimental circuit is not one of these special cases since i could throw together a equivalent lumped model in 5 minutes, it worked exactly like his experiment first try.

Can you explain in what way is it a special case of Faradays law, if the two laws describe two separate concepts?


So Kirchhoff ended up being a subset Faraday. I even drew it as a Venn diagram many messages ago in this thread.

(https://www.eevblog.com/forum/chat/does-kirchhoffs-law-hold-disagreeing-with-a-master/?action=dlattach;attach=583322;image)

That diagram i fully agree with. Kircchhoffs laws are an abstracted application of Maxwells equations and are meant to be only used on lumped circuit meshes.

Dr. Lewin is not applying KVL to a lumped circuit hence why it does not work for him. Works perfectly fine for me when i apply KVL to my lumped equivalent version of his circuit. Is my lumped circuit model wrong in some way? (I always welcome constructive criticism)
Title: Re: Does Kirchhoff's Law Hold? Disagreeing with a Master
Post by: radioactive on November 29, 2018, 02:38:36 pm
I've updated the openEMS FDTD simulation (3D field solver for Maxwell's equations).   More turns to the coil,  Added Perfect-Matched-Layer (8 levels) to the boundary conditions.  Added two more voltage probes right next to each other at points approximately 90 degrees around the loop from the resistors.  I did some experimentation with a stepped pulse instead of the nice Gaussian, but still working on that.

I really don't see how Kirchhoff's law could possibly be used to analyze something like this (and truly be successful).  Not saying that to be controversial,  I just don't see how KVL could be useful for much other than static DC analysis and this circuit is definitely not that.   

[edit] removed sim files.  I'm an idiot.  I'll post a good sim when I have it ready.
Title: Re: Does Kirchhoff's Law Hold? Disagreeing with a Master
Post by: Berni on November 29, 2018, 03:58:59 pm
Are you perhaps using a current source to power the dense coil in the middle?

The parasitic capacitance between the turns and the inductance of the coil are likely forming a parallel LC tank circuit. Parallel tank circuits are easily exited into oscillation by a current source due to the current source providing freedom to the voltage to swing as much as it wants. Similar thing happens if a voltage source step powers a series LC tank.

To test out my theory i replaced the voltage source in my LT Spice model with a current source and assigned the solenoid coil some parasitic capacitance. And yes this does appear to be the case. Try powering the inner coil with a voltage step instead.
Title: Re: Does Kirchhoff's Law Hold? Disagreeing with a Master
Post by: ogden on November 29, 2018, 04:04:23 pm
I did some experimentation with a stepped pulse instead of the nice Gaussian, but still working on that.

You shall try to make pulse which results in waveforms demonstrated by Dr.Lewin in his SUPER DEMO. Also voltages on resistors (< 1mV) are way too small - does not match those seen in SUPER DEMO.

Quote
I really don't see how Kirchhoff's law could possibly be used to analyze something like this (and truly be successful).

Of course! Kirchhoff's law can't be used to analyze electromagnetism :) As your results do not look like those shown by Dr.Lewin, shall I conclude that Maxwell's equations can't be used as well? [kidding] :D
Title: Re: Does Kirchhoff's Law Hold? Disagreeing with a Master
Post by: radioactive on November 29, 2018, 04:59:52 pm
I'll have to work on this more later, but here is some output from an attempt at using a stepped voltage source  (vs. Gaussian current source).   I will see if there is a way to scale things on the voltages, but I think is pretty standard for excitation pulses to be 1 Volt in these sims.  For now, I have just been looking at how one might come to the conclusion that loop voltages add up to zero.  I thought maybe there was something special about this configuration.  I'm not particularly impressed by the triggered scope captures, but would be nice to see something close to that.  This output might come closer to that depending on what point in time / voltage trigger level you look at I guess.

Anyway,  I probably have it screwed up.  Let me know if somebody knows how to fix this sim up.   I have used openEMS for designing UHF microstrip planar BPF filters in the past (that work),  so I have confidence in the solver itself.

[edit]  I did have it all screwed up.  I'll post a better sim when I have it ready.
Title: Re: Does Kirchhoff's Law Hold? Disagreeing with a Master
Post by: Berni on November 29, 2018, 05:17:57 pm
Yeah the image of those scopes is a bit fuzzy in the video but i got pretty much the same results when recreating the experiment myself:
https://www.eevblog.com/forum/chat/does-kirchhoffs-law-hold-disagreeing-with-a-master/msg1987562/#msg1987562 (https://www.eevblog.com/forum/chat/does-kirchhoffs-law-hold-disagreeing-with-a-master/msg1987562/#msg1987562)

The purple trace at the bottom is the voltage on the solenoid coil, i was using that as a stable trigger signal.

So you need to apply a similar voltage step response across your solenoid coil (Not just a pulse). Also your time scale appears to be very short in the simulation. The pulse you applied seams to last only a few picoseconds, this gives it a bandwidth of >100GHz and hence why you get funny behavior as you are mostly simulating radio waves traveling around your scene. The whole simulation only lasting what appear to be around half a nanosecond. My experiment had the pulse last 500 microseconds so about 1 000 000 times longer than your simulation time.

I don't have any experience with that EM simulation tool but it does look pretty cool.
Title: Re: Does Kirchhoff's Law Hold? Disagreeing with a Master
Post by: ogden on November 29, 2018, 06:11:08 pm
So you need to apply a similar voltage step response across your solenoid coil (Not just a pulse).

Right. Lewin uses just positive step. If possible, magnetic core shall be added as well.

I think is pretty standard for excitation pulses to be 1 Volt in these sims.

0dBV for RF PCB simulations is good default choice, but this is solenoid :) We know that EMF of experiment is 0.1V or so.
Title: Re: Does Kirchhoff's Law Hold? Disagreeing with a Master
Post by: bsfeechannel on November 29, 2018, 08:45:17 pm
Yes this is what i find to be the issue. Faraday and Kirchhoff are put into the same basket while they describe different things.

Kirchhoff lives inside Faraday. So, if you put Faraday in a basket, Kirchhoff will be there too. The guy who realized that was Maxwell.

Quote
Here is a definition of Faradays law:
"The electromotive force induced in a circuit by variation of the magnetic flux through the circuit is proportional to the negative of the time rate of change of the magnetic linkage"

Here is a definition of Kirchhoff voltage law:
"The algebraic sum of all the voltages around any closed loop in a circuit is equal to zero"

What happens if your EMF is ZERO? Doesn't that ring you a bell? Where did you see ZERO before?

Ahh! KIRCHHOFF! He says that "The algebraic sum of all the voltages around any closed loop in a circuit is equal to ZERO"!!!!!!!

Quote
Kirchhoff on the other hand does talk about the voltages summing to zero. Notice that it says "all the voltages" so this implies induced EMF too as that's a voltage inside the loop.

Here is where your problem lies. When Kirchhoff says all the voltages, he is not considering any kind of EMF. In Kirchhoff's wonderful world there is absolutely NO EMF whatso-fluxing-ever!

Quote
In a circuit mesh schematic a wire has zero length, zero resistance and zero reactance. These wires are immune to all field effects. Building the two resistor circuit with such wires in the real world results in a circuit that must have a circumference of zero, this means it also must have a loop area of zero, zero area means no induced EMF and KVL works.

Philosophical question: if Kirchhoff only applies to circuits that cannot exist, why do we need his stupid theory?

Quote
This however has a problem because Dr. Lewins circuit clearly has a loop area larger than zero so it makes the circuit mesh model act different(You get 0V on both resistors).  We fix this by replacing out ideal wires with inductors.

If, however, you have a constant magnetic flux, Kirchhoff holds. So Lewin has no problem with Kirchhoff.

Quote
This plugs in Faradays law and tells it about how big the loop area is.

Faraday is not a plug-in, it is the chassis. It is the base upon which Kirchhoff stands. There's no Kirchhoff without Faraday.

Quote
YES I KNOW that Kirchhoffs voltage law does indeed break in certain special cases, i'm not saying it always works.

Kirchhoff's law breaks MOST of the cases. Faraday is the rule. Kirchhoff is the exception.

Quote
Can you explain in what way is it a special case of Faradays law, if the two laws describe two separate concepts?

We've done it several times along this thread.

Quote
(https://www.eevblog.com/forum/chat/does-kirchhoffs-law-hold-disagreeing-with-a-master/?action=dlattach;attach=583322;image)

That diagram i fully agree with. Kircchhoffs laws are an abstracted application of Maxwells equations and are meant to be only used on lumped circuit meshes.

You do not only have a problem with Kirchhoff, Maxwell and Lewin. You also have a problem with Georg Cantor.

Between you and me. Forget all you know about circuits. Follow the guidelines I published in a previous message. Study calculus, vector analysis and a good book on electromagnetism. You'd be better off than struggling with theories you do not master.
Title: Re: Does Kirchhoff's Law Hold? Disagreeing with a Master
Post by: HackedFridgeMagnet on November 29, 2018, 10:37:34 pm
Quote
Since the mid-20th century, it has been understood that Maxwell's equations are not exact, but a classical limit of the fundamental theory of quantum electrodynamics.

from https://en.wikipedia.org/wiki/Maxwell%27s_equations#Limitations_for_a_theory_of_electromagnetism (https://en.wikipedia.org/wiki/Maxwell%27s_equations#Limitations_for_a_theory_of_electromagnetism)

you might have to fix your Venn diagram.

Does that mean Maxwell "is for the birds"?
Title: Re: Does Kirchhoff's Law Hold? Disagreeing with a Master
Post by: ogden on November 30, 2018, 12:43:22 am
When Kirchhoff says all the voltages, he is not considering any kind of EMF.

WTF are you talking about here?  :-//

Quote
If, however, you have a constant magnetic flux, Kirchhoff holds. So Lewin has no problem with Kirchhoff.

Try to wrap discrete time measurement into your head. Levin even uses term "voltage at given time" and demonstrates experiment using oscilloscopes. Obviously "at given time" - during infinitely small time moment of observation, magnetic flux does not change. You just said that in such case Kirchhoff holds. That' it, case closed.
Title: Re: Does Kirchhoff's Law Hold? Disagreeing with a Master
Post by: bsfeechannel on November 30, 2018, 01:06:14 am
Quote
Since the mid-20th century, it has been understood that Maxwell's equations are not exact, but a classical limit of the fundamental theory of quantum electrodynamics.

from https://en.wikipedia.org/wiki/Maxwell%27s_equations#Limitations_for_a_theory_of_electromagnetism (https://en.wikipedia.org/wiki/Maxwell%27s_equations#Limitations_for_a_theory_of_electromagnetism)

you might have to fix your Venn diagram.

If we are restricted to classical physics, which is the physics of circuit analysis, that won't be necessary. But if we consider QED, which applies to phenomena at the subatomic scale, that won't change the status of Kirchhoff as being owned by Faraday.

To really understand the limits of Maxwell, you'll have to study quantum mechanics. But do you believe that the people who are lazy enough to study Maxwell will have the cojones to study QM? I doubt it.

Quote
Does that mean Maxwell "is for the birds"?

I don't know. Do birds study calculus, vector analyses and electromagnetism?
Title: Re: Does Kirchhoff's Law Hold? Disagreeing with a Master
Post by: Berni on November 30, 2018, 10:31:20 am
That depends on the symmetry of the induced field. Let me give you my take on that paper (who appears to be a draft considering there is at least a minor error and a (?) meaning what, exactly? Do you know if this was ever published and where is the definitive version?)
Well, it's interesting but it is nonetheless amazing that to compute the emf on an open path, the area always pops out.

Well this paper is what i could find on the topic freely accessible on the internet. Flowing sources and searching on the topic shows more work on the topic but most of it is locked behind paywalls (The usual thing with scientific publications). I'm not a university student anymore to have access to those for free. Feel free to dig deeper into it if you want.

It makes sense to me that the geometric origin of the field would be important since that's the single point where nothing happens to the magnetic field lines as the flux changes. The integral of an area just happens to be a good way to capture the fact that a combination of two spatial dimensions affect the result. Since both Faradays law and a wire segment require it they both use an area (And besides its both magnetic induction so you expect something similar to happen). Area integrals pop up a lot in electronics just because how useful they are.

But yeah the question weather there is voltage on the open loop of wire depends on how you look at it. The EMF will balance out with the electric field of charge separation so technically the voltage is zero but there are more electrons on one end than the other. Circuit analysis makes use of this concept for making wire segments in the form of inductors.


What happens if your EMF is ZERO? Doesn't that ring you a bell? Where did you see ZERO before?

Ahh! KIRCHHOFF! He says that "The algebraic sum of all the voltages around any closed loop in a circuit is equal to ZERO"!!!!!!!

Quote
Kirchhoff on the other hand does talk about the voltages summing to zero. Notice that it says "all the voltages" so this implies induced EMF too as that's a voltage inside the loop.

Here is where your problem lies. When Kirchhoff says all the voltages, he is not considering any kind of EMF. In Kirchhoff's wonderful world there is absolutely NO EMF whatso-fluxing-ever!

Quote
In a circuit mesh schematic a wire has zero length, zero resistance and zero reactance. These wires are immune to all field effects. Building the two resistor circuit with such wires in the real world results in a circuit that must have a circumference of zero, this means it also must have a loop area of zero, zero area means no induced EMF and KVL works.

Philosophical question: if Kirchhoff only applies to circuits that cannot exist, why do we need his stupid theory?

Quote
(https://www.eevblog.com/forum/chat/does-kirchhoffs-law-hold-disagreeing-with-a-master/?action=dlattach;attach=583322;image)

That diagram i fully agree with. Kircchhoffs laws are an abstracted application of Maxwells equations and are meant to be only used on lumped circuit meshes.

You do not only have a problem with Kirchhoff, Maxwell and Lewin. You also have a problem with Georg Cantor.

Between you and me. Forget all you know about circuits. Follow the guidelines I published in a previous message. Study calculus, vector analysis and a good book on electromagnetism. You'd be better off than struggling with theories you do not master.

Well Faradays law says that with no flux change the induced EMF voltage equals zero. There are other voltages possible in circuits that are not coming from induced EMF, batteries also produce voltage that is NOT induced. So saying that the induced EMF is zero does not automatically mean the sum of all voltages is zero as induced EMF is not the only voltage possible in a circuit

Kirchhoffs laws are a circuit analysis tool, not a law that governs how the universe works. Circuit analysis is just an practical application of physics, while physics is a practical application of mathematics.

Physics has no use for Kirchhoffs law since it doesn't deal with anything physical. Maxwells equations and the things that come from them (like for example Faradays law) are completely sufficient to explain what is going on. Okay we did discover even more fundamental quantum effects a layer deeper than Maxwell explains, but Maxwell still works perfectly on a macroscopic level so that's good enough, we can't say its wrong because of that (Just slightly abstracted that's all)

So then why do we even need Kirchhoffs law anyway if it appears to be useless in physics? Heck why do we even need circuit analysis if physics can already describe anything electrical? Lets just use physics instead!

Well.. we could. The problem is that calculating all of this for a real physical circuit involving only a few components would already result in a LOT of math. Engineers regularly deal with circuits that involve 10 to 1000 components, not just 3, this causes the math complexity to skyrocket and it simply becomes unpractical to calculate. Have a look at how hard EM simulations are on computers. They take a ton of memory and take a significant time to compute even on a modern PC.

Turns out engineers often have to calculate the behavior of circuits as part of there work. When they wonder how a RC low pass filter circuit acts at audio frequencies they simply don't care what happens with magnetic and electric fields around there circuit. To solve this problem the science of circuit analysis was created. This cherry picks the effects from physics that matter and condenses them down into a simpler form of math.

The engineer can now chose if he wants to take in account the magnetic fields or not, rather than it simply being part of the process that must be included for everything to function. Our universe breaks down if you suddenly have magnetic fields disappear, circuit mesh models keep working. If the engineer wants to ignore magnetic effects they simply leave them out and the circuit continues to work as if they are not there. However the circuit now behaves as if it is made out of these mythical ideal wires. In most cases this results in identical circuit behavior. If these effects cause a significant difference in behavior(Such as Dr. Lewins circuit) then the engineer must realize this and chose to add them. In this case this is done by adding an equivalent model of the wire in the form of inductor. The inductor model then calls upon Maxwell to only solve that single wire, this is quick to do because we have generalized easybake equations already prepared from Maxwell.

So KVL only interacts with Faradays law when circuit mesh modeling deems it necessary. Its all just part of a elaborate mathematical shortcut that we call "circuit analysis". Engineers use it to calculate stuff faster and you apply it outside of that there is no grantee it will work. It might still work in special cases outside of circuit meshes but that's just a special case, not intended use.
Title: Re: Does Kirchhoff's Law Hold? Disagreeing with a Master
Post by: bsfeechannel on November 30, 2018, 11:57:00 am
Physics has no use for Kirchhoffs law since it doesn't deal with anything physical.

Kirchhoff, one of the most important physicists of the 19th century, must be rolling over in his grave.

Quote
The problem is that calculating all of this for a real physical circuit involving only a few components would already result in a LOT of math.

You don't get it. You have an impressive equipment on your bench all of it making extensive use of Maxwell and yet you have no clue about the theory used to design it.

In Kirchhoff's world you wouldn't have coaxial cables, ground planes, canned circuits, EMI certification, impedance matching, delay lines, coupling, decoupling, transformers, inductors, motors, generators, radiation, etc., etc., etc.

Kirchhoff is from a time when the only source of electrical energy widely available were batteries. There were no changing fields. No transients. No kind of interference. Only well-behaved DC circuits. Today, even toys using batteries have SMPSs with inductors and transformers, these specifically making use of Faraday's law.

So you can't escape Maxwell. It's everywhere these days. When we say Kirchhoff, we are in reality saying Maxwell, or Faraday in case of induction, under certain VERY special conditions. But it is still Maxwell.

Don't fool yourself or you'll end up like Cyriel Mabilde: an advocate of ignorance.

Quote
So KVL only interacts with Faradays law when circuit mesh modeling deems it necessary.

So you're saying that on your planet the laws of Nature obey the desires of the engineer? I'm moving there right now.
Title: Re: Does Kirchhoff's Law Hold? Disagreeing with a Master
Post by: Berni on November 30, 2018, 02:07:33 pm
Physics has no use for Kirchhoffs law since it doesn't deal with anything physical.

Kirchhoff, one of the most important physicists of the 19th century, must be rolling over in his grave.

Sorry if i worded that a bit too broadly. I did not mean to include any of Kirchhoffs other laws! Those are indeed very important for physics.

I meant to say that only for Kirchhoffs circuit laws. Maxwells equations can already describe the behavior found in Kirchhoffs cirucit laws, so physics doesn't really need KVL and KCL. In that way i guess i could say that Dr. Lewin could say that KVL is "for the birds"

But for circuit analysis really does need KVL and KCL to work.


Quote
The problem is that calculating all of this for a real physical circuit involving only a few components would already result in a LOT of math.

You don't get it. You have an impressive equipment on your bench all of it making extensive use of Maxwell and yet you have no clue about the theory used to design it.

In Kirchhoff's world you wouldn't have coaxial cables, ground planes, canned circuits, EMI certification, impedance matching, delay lines, coupling, decoupling, transformers, inductors, motors, generators, radiation, etc., etc., etc.

Kirchhoff is from a time when the only source of electrical energy widely available were batteries. There were no changing fields. No transients. No kind of interference. Only well-behaved DC circuits. Today, even toys using batteries have SMPSs with inductors and transformers, these specifically making use of Faraday's law.

So you can't escape Maxwell. It's everywhere these days. When we say Kirchhoff, we are in reality saying Maxwell, or Faraday in case of induction, under certain VERY special conditions. But it is still Maxwell.

Don't fool yourself or you'll end up like Cyriel Mabilde: an advocate of ignorance.


I was never trying to say that Maxwell is useless. I was going among the lines that its rare that you absolutely need to get down and dirty with electric and magnetic fields. Yes Kirchhoff is not going to do much in helping you understand how a coax cable works, but for 99% of use cases i can just think of a coax cable as a delay line with some attenuation and characteristic impedance, wack that into the equivalent circuit model and done.

Why doesn't everyone write software in a hex editor? Its the most fundamental way of doing it after all. Its just simply more practical to work on a higher level with a compiler. Yes you are missing out on some fine details with all these high level language abstractions, but in 99% of cases it doesn't matter and it does get the job done significantly faster.

Not saying one should ignore the underlying physics. One should indeed understand how it works underneath for the sake of the big picture. I was making an example of why circuit analysis is useful in order to justify why one might want to use KVL rather than the more the accurate alternatives (the answer is deadlines). This is my argument for why KVL is not "for the birds"

Im not trying to get away from Maxwell. His equations do an amazing job in explaining how electricity works. Just showing where KVL fits in to it all.

Quote
So KVL only interacts with Faradays law when circuit mesh modeling deems it necessary.

So you're saying that on your planet the laws of Nature obey the desires of the engineer? I'm moving there right now.

Nah sadly the same stubborn laws of nature apply to the planet i live on. But in the world of mesh circuits they do indeed obey the desires of the engineer, so far no airline is offering one way trips into that world so i guess we are out of luck. But at least the bent laws help things calculate faster in my real world, the trick is bending them just right so that it acts almost the same as in the real world while only taking 1% of the math to get there. Bending these laws just this right way is the responsibility of the engineer.

Applying KVL without modeling the effects of wires to Dr Lewins experimental circuit is an example of bending the "circuit mesh world" laws too far, creating a circuit that behaves significantly different compared to real life where all the laws apply all the time.

Title: Re: Does Kirchhoff's Law Hold? Disagreeing with a Master
Post by: Zucca on November 30, 2018, 02:33:16 pm
If I believe that in two same points there will be the same voltage ALWAYS (I cut the part with the two identical multimeters) regardless the "path", does it implies I do not undesrtand Mr. Maxwell?

I don't get it. Sorry I am an idiot.
Title: Re: Does Kirchhoff's Law Hold? Disagreeing with a Master
Post by: rfeecs on November 30, 2018, 06:43:05 pm
Looking at a circuit theory textbook, "Basic Circuit Theory" by Desoer and Kuh, they devote the first 10 pages of a 900 page book to "Lumped circuits and Kirchoff's laws".  They state:

Quote
KVL applies to any lumped electric circuit; it does not matter whether the circuit elements are linear, nonlinear, active, passive, time-varying, time-invariant, etc.  In other words, KVL is independent of the nature of the elements.

You can't leave out the word lumped in that statement!  They also point out that the entire book only applies to lumped circuits.

Yes, you can make a lumped circuit model for just about anything.  You can even model a circuit with an antenna by replacing the antenna with a lumped element that represents it's terminal impedance.

But for this example, why bother?  For a circular magnetic flux region, you can easily calculate the electric field.  It goes around in circles:
(https://www.eevblog.com/forum/chat/does-kirchhoffs-law-hold-disagreeing-with-a-master/?action=dlattach;attach=584825;image)
If you insert the loop of wire with resistors into the electric field loop, you will get a current.  Also, the charges in the wire will arrange themselves to cancel the electric field in the wire.  (this also would happen to an unconnected segment of wire in the electric field).  So the wire has no (tangential) electric field inside it or at it's surface, but it does have a charge distribution.

Some of these papers are just defining the voltage across a section of the wire as the electric scalar potential.  This is saying: take the electric field caused by the charge distribution only and integrate it along the path of the wire.  Call this the "voltage".  Here's another paper by Kirk McDonald where he is doing this:
http://www.hep.princeton.edu/~mcdonald/examples/volt.pdf (http://www.hep.princeton.edu/~mcdonald/examples/volt.pdf)

So they are making up an imaginary voltage that would be created by the charge distribution in the wire.  But actually there is no field there, because it is cancelled by the induced electric field.  So you can't measure that "voltage" unless you could freeze the charge distribution and place it outside the field.  Or if you could form a probing loop that is unaffected by the field, then you could measure the effect of that charge distribution because it would cause a field in your meter.  I suppose this is what Cyriel Mabilde is trying to do.
Title: Re: Does Kirchhoff's Law Hold? Disagreeing with a Master
Post by: bsfeechannel on December 01, 2018, 12:38:27 am
so physics doesn't really need KVL and KCL.

And I thought KVL and KCL were descriptions of physical phenomena. My life is a lie.

Quote
Why doesn't everyone write software in a hex editor? Its the most fundamental way of doing it after all. Its just simply more practical to work on a higher level with a compiler. Yes you are missing out on some fine details with all these high level language abstractions, but in 99% of cases it doesn't matter and it does get the job done significantly faster.

Abstraction and neglect of the fundamentals are orthogonal things.

Abstraction means to take away from the designer tasks and routines that are necessary but will not have any kind of influence on the result. So, if you have a menial task to do, you hand it down to a computer, or another person, or company, or the compiler, etc., so that you can concentrate on the specifics of your project. However this doesn't remove from the designer the responsibility to have full mastery of the fundamental concepts of his field.

If abstraction were the same as neglect of the fundamentals, any computer science or software engineering course will be just a class on some stupid "high level" language and nothing more.

Quote
I was making an example of why circuit analysis is useful in order to justify why one might want to use KVL rather than the more the accurate alternatives (the answer is deadlines).

So. You have, say, a radar to design. You have a tight deadline. You turn to your colleague and say "Bugger that! Let's use Kirchhoff and go home."

Quote
But at least the bent laws help things calculate faster in my real world, the trick is bending them just right so that it acts almost the same as in the real world while only taking 1% of the math to get there.

The math required for the work with Kirchhoff requires a decade or so of learning. You do not learn Kirchhoff in the first grade, do you? If the math required for Kirchhoff were so simple, you wouldn't have spice programs out there to solve them for you.

The  basic math required to work with Maxwell requires less than 100 hours of study. And if the equations get really complex, you also have proper software to deal with them.

So, never try anymore to hide your lack of knowledge behind excuses like that. Convince yourself and others of the need to be ready to learn something new every day.
Title: Re: Does Kirchhoff's Law Hold? Disagreeing with a Master
Post by: ogden on December 01, 2018, 02:25:51 am
So, never try anymore to hide your lack of knowledge behind excuses like that.

You are troll. Shame on you and respect to Berni.
Title: Re: Does Kirchhoff's Law Hold? Disagreeing with a Master
Post by: Berni on December 01, 2018, 10:09:07 am

Abstraction and neglect of the fundamentals are orthogonal things.

Abstraction means to take away from the designer tasks and routines that are necessary but will not have any kind of influence on the result. So, if you have a menial task to do, you hand it down to a computer, or another person, or company, or the compiler, etc., so that you can concentrate on the specifics of your project. However this doesn't remove from the designer the responsibility to have full mastery of the fundamental concepts of his field.

If abstraction were the same as neglect of the fundamentals, any computer science or software engineering course will be just a class on some stupid "high level" language and nothing more.

And this is exactly why KVL is an abstraction of Maxwells equations. It makes things easier while not influencing the result (when used correctly).

Don't worry i seen plenty of programmers that don't understand how a computer works and consider C being too low level. Some even stick to purely interpreted languages like javascript, php or python. I do think they should at least conceptually understand the inner workings of computers. Large software projects are often massive cobbled together messes with chunks of code that nobody understands why they are needed and how they work but if you touch them things break horribly so everyone stays away from them and works around them. Tight deadlines encouraging to just cobble on more rushed code until the project becomes unmaintainable. But that's a topic for another day.



So. You have, say, a radar to design. You have a tight deadline. You turn to your colleague and say "Bugger that! Let's use Kirchhoff and go home."


Actually i was working on some phased array stuff recently and i used even more abstraction than just Kirchhoff.

I was trying to calculate the directionality of a phased array and i wanted to do it fast so that i could later use iterative optimization methods on it. I could have simulated each point in space and its interaction to the neighboring points so that i get a simulation of wave propagation trough the medium. Takes a lot of computation to do and im too lazy to program all of that. Well... instead i just did trigonometry to find the distance to each element and pretended there is an ideal delay line with its delay proportional to the distance. Worked great and it would spit out a directionality graph for all possible angles in the blink of an eye(Without even trying to optimize the code). Is this magical delay line what is actually happening on a phased array? Hell no. Does it act similar enough? For my use case plenty enough.

When the abstraction works i will definitely say "Bugger that!" to the more complicated alternatives. If the abstraction doesn't work then i will go down the long path. I don't work in an academic institution to needlessly waste time obsessing about the underlying mechanisms for cases where they are simply not important.


The math required for the work with Kirchhoff requires a decade or so of learning. You do not learn Kirchhoff in the first grade, do you? If the math required for Kirchhoff were so simple, you wouldn't have spice programs out there to solve them for you.

The  basic math required to work with Maxwell requires less than 100 hours of study. And if the equations get really complex, you also have proper software to deal with them.

So, never try anymore to hide your lack of knowledge behind excuses like that. Convince yourself and others of the need to be ready to learn something new every day.

Well i was introduced to Kirchhoffs laws before high school. Then actually had to use them in high school to manually solve circuit meshes. Inside the circuit mesh abstraction its really easy to use.

Maxwell on the other hand i only had it mentioned in high school without really explaining what it is. In university i did get it explained to me why Maxwell is so important, but ultimately never used his equations directly to calculate a real practical example with actual numbers. We dealt with calculating electric and magnetic fields, induced EMF etc... in real examples but it was all with equations that are derived from Maxwell to calculate a certain situation (Such are Faradays law when dealing with induction in a loop).

We heavily used circuit analysis methods in university to manually calculate the behavior of all sorts of circuits. Not only ones with batteries and resistors (We did that in high school) but circuits that involve capacitors, inductors and even semiconductors (dioes, BJTs, FETs, tryristors, diacs etc). Circuit analysis can handle all of this by simply plugging in appropriate models. Try calculating a circuit with transistors using only Maxwells equations, its going to be a major pain.

Heck we even used circuit analysis methods on non electrical things. We used a circuit model to simulate heat propagating trough a wall (thermal conductance being a resistor and thermal mass being a capacitor), to calculate hydraulic flows trough pipe networks, or even turning magnetic circuits into mesh schematics to more easily calculate them. Circuit mesh analysis (and the Kirchhoffs laws used in it) worked great in all these situations. Its all a matter of modeling your circuit correctly. So go ahead and say "Kirchhoffs laws are the plumbers" instead.
Title: Re: Does Kirchhoff's Law Hold? Disagreeing with a Master
Post by: Berni on December 01, 2018, 10:41:47 am
Oh and i was to add that with the using schematic circuits to calculate heat flow trough a wall, water flow trough a pipe or magnetic flow trough a core is mostly something that electrical engineers do when they have to deal with something non electrical. Its sort of just swapping out the abstraction layer underneath so they can keep using familiar analysis methods. Im pretty sure other kinds of engineers don't "misuse" schematics for this.

But this trick does show how in nature everything appears to have two intertwined quantities:
-Electricity: Volt and Amps
-Mechanics: Force and Distance
-Rotational mechanics: Torue and RPM
-Fluids: Pressure and Flowrate
-Magnetics: Magnetic density(H) and Magnetic flux(B)
-Thermal: Temperature and Thermal flux
etc...

In all of these cases if you multiply these two intertwined quantities you get power(P) in Watts. Ohms law works for all of these(Tho units are not in ohms anymore) and each of these has a inductor/capacitor equivalent that provides time dependence.

All of these act the same, just use different units because they are different physical things. Because of this cirucit mesh analysis also works on all of these as long as you know how to model it as a schematic. Once its a schematic you can apply KVL and KCL to it just fine and it still works fine, just like all the other circuit analysis tools such as Thenevnins and Nortons theorem. Circuit schematics ware never designed to do this but it just happens to work due to how nature works.

I found this realization of intertwined units quite eye opening back in university. It can help you understand how non electrical things work a lot faster.
Title: Re: Does Kirchhoff's Law Hold? Disagreeing with a Master
Post by: bsfeechannel on December 01, 2018, 03:48:59 pm
And this is exactly why KVL is an abstraction of Maxwells equations. It makes things easier while not influencing the result (when used correctly).

Because in many of your "abstractions" you used KVL instead of Maxwell, you think that KVL is an abstraction of Maxwell. Don't say that anymore.

Saying that means that every time you have a problem solvable by Maxwell, you can immediately apply KVL "used correctly" and that's it.

It implies that you can get away without understanding the underlying phenomena. No, you can't.

That's what Cyriel Mabilde disastrously did. That's what Lewin is desperately trying to warn you about.

Quote
Don't worry i seen plenty of programmers that don't understand how a computer works and consider C being too low level. Some even stick to purely interpreted languages like javascript, php or python. I do think they should at least conceptually understand the inner workings of computers. Large software projects are often massive cobbled together messes with chunks of code that nobody understands why they are needed and how they work but if you touch them things break horribly so everyone stays away from them and works around them. Tight deadlines encouraging to just cobble on more rushed code until the project becomes unmaintainable. But that's a topic for another day.

Why you can get away with that in computing? Because those languages, including machine code, are all equivalent in computing power. So you can get a program working without knowing the details under the hood at the expense of a messy code.

Kirchhoff and Maxwell are NOT equivalent. If Kirchhoff and Maxwell were languages, Kirchhoff would describe a machine with LESS computing power than Maxwell.

Quote
Actually i was working on some phased array stuff recently and i used even more abstraction than just Kirchhoff.

I was trying to calculate the directionality of a phased array and i wanted to do it fast so that i could later use iterative optimization methods on it. I could have simulated each point in space and its interaction to the neighboring points so that i get a simulation of wave propagation trough the medium. Takes a lot of computation to do and im too lazy to program all of that. Well... instead i just did trigonometry to find the distance to each element and pretended there is an ideal delay line with its delay proportional to the distance. Worked great and it would spit out a directionality graph for all possible angles in the blink of an eye(Without even trying to optimize the code). Is this magical delay line what is actually happening on a phased array? Hell no. Does it act similar enough? For my use case plenty enough.

When the abstraction works i will definitely say "Bugger that!" to the more complicated alternatives. If the abstraction doesn't work then i will go down the long path. I don't work in an academic institution to needlessly waste time obsessing about the underlying mechanisms for cases where they are simply not important.

Again. Who told you that you could reduce the problem to KVL? Kirchhoff? Certainly not. Kirchhoff is a bird. He doesn't know anything about propagation, delay lines, fields, etc.

You had to use MAXWELL, and you did it almost unconsciously, to reduce to problem to Kirchhoff. So you confirm what I said in one of my first messages on this thread about how we engineers are so used to that practice that we forget that we are in fact using Maxwell and implicitly reducing the problem to Kirchhoff.

You can abstract, but you cannot use this as an excuse to ditch the fundamentals. What people are doing is not even trying to study Maxwell, consequently not understanding what the flux is going on and criticizing Lewin for THEIR ignorance.

This is the most stupid educational move I've seen in decades.

Quote
Well i was introduced to Kirchhoffs laws before high school. Then actually had to use them in high school to manually solve circuit meshes. Inside the circuit mesh abstraction its really easy to use.

...[snip]

I found this realization of intertwined units quite eye opening back in university. It can help you understand how non electrical things work a lot faster.

TLDR.

Now that I've made you a convert, let's help others to avoid saying stupid things like "Physics has no use for Kirchhoffs law since it doesn't deal with anything physical."

I was almost using that quote as a signature. But then I decided to give you a second chance.
Title: Re: Does Kirchhoff's Law Hold? Disagreeing with a Master
Post by: HackedFridgeMagnet on December 01, 2018, 10:17:13 pm
The thing is.

Berni, Electroboom and Cyriel Mabilde know how to measure the voltage in the demonstration circuit.
They all demonstrated it properly.

Clearly bsfeechannel and Dr Lewin don't know how to measure the voltage.

The moment you put voltmeter on as Dr Lewin did, you are measuring a scalar voltage. He did that.
But unfortunately he did it incorrectly and drew the wrong conclusion.

Maxwell-Faraday is a model the same as any other. It has limitations, and can be used successfully if it fits within them.
It is not magic. It isn't even particularly complex.

btw bsfeechannel please update your venn diagram to fix your mistake, people will get the wrong idea.
Title: Re: Does Kirchhoff's Law Hold? Disagreeing with a Master
Post by: bsfeechannel on December 02, 2018, 01:18:00 am

Berni, Electroboom and Cyriel Mabilde know how to measure the voltage in the demonstration circuit.
They all demonstrated it properly.

They have no idea what they're measuring.

Quote
Clearly bsfeechannel and Dr Lewin don't know how to measure the voltage.

I have a series of videos where I design and build an isolation transformer. The first video starts with an explanation why a transformer works. It obviously employs Maxwell/Faraday, because transformers are a direct result of Faraday's research. After I understand what to expect, I reduce the problem to Kirchhoff and then I design, build, test and characterize it.

It was a very successful project. The transformer met all the specs and works a treat.

https://www.youtube.com/watch?v=B9OhDcLnGFY&list=PLd-j8HcWkeZqf_bG3ve0Fes8lrlKvlAv2 (https://www.youtube.com/watch?v=B9OhDcLnGFY&list=PLd-j8HcWkeZqf_bG3ve0Fes8lrlKvlAv2)

Quote
The moment you put voltmeter on as Dr Lewin did, you are measuring a scalar voltage. He did that.
But unfortunately he did it incorrectly and drew the wrong conclusion.

Isn't it annoying when Nature does not conform to our preconceived notions of reality?

Quote
Maxwell-Faraday is a model the same as any other. It has limitations, and can be used successfully if it fits within them.
It is not magic. It isn't even particularly complex.

btw bsfeechannel please update your venn diagram to fix your mistake, people will get the wrong idea.

As I said, I don't need to upgrade anything for circuit analysis. There is no mistake. The theory is sound. When you study it, as I, Lewin, and many others on this forum did, you'll understand it as clear as crystal. If you need any assistance in upgrading your knowledge, we are here to help.
Title: Re: Does Kirchhoff's Law Hold? Disagreeing with a Master
Post by: HackedFridgeMagnet on December 02, 2018, 01:54:55 am
Your venn diagram was wrong because you had Maxwell covering in all other cases.
It's an important point because it glosses over the limitations of modelling natural phenomena.

Dr Lewin seemingly had no idea what he was measuring, I make this judgement because he said it would blow their minds.
It certainly confused people but didn't blow minds.

Quote
Isn't it annoying when Nature does not conform to our preconceived notions of reality?
I guess it would be.
If Berni, Electroboom and Cyriel Mabilde got it wrong then let us know what you think voltages are on the loop in Dr Lewins experiment.
Title: Re: Does Kirchhoff's Law Hold? Disagreeing with a Master
Post by: HackedFridgeMagnet on December 02, 2018, 02:02:29 am
But on a brighter note we seem to agree that KVL is at lease useful for characterising transformers.

ps. I would have liked to have watched your video further but the robot voice was hurting me.
Maybe someone on EEBblog could do an English (or other language) voice over?
Title: Re: Does Kirchhoff's Law Hold? Disagreeing with a Master
Post by: bsfeechannel on December 02, 2018, 02:50:22 am
But on a brighter note we seem to agree that KVL is at lease useful for characterising transformers.

ps. I would have liked to have watched your video further but the robot voice was hurting me.
Maybe someone on EEBblog could do an English (or other language) voice over?

I wrongly thought that since my videos are directed to a technical audience, my viewers would not be bothered by a computer-generated voice. If someone, especially a native speaker of English, could do the voice-over that would be wonderful. It would be a voluntary work, though, since my channel is not monetized. I was mustering the courage to do it myself. But I am not sure how the viewers would react.

Anyway, thanks for watching.
Title: Re: Does Kirchhoff's Law Hold? Disagreeing with a Master
Post by: GeorgeOfTheJungle on December 02, 2018, 09:36:57 am
Hey, bsfeechannel, where do you come from?
Title: Re: Does Kirchhoff's Law Hold? Disagreeing with a Master
Post by: Berni on December 02, 2018, 10:25:01 am
And this is exactly why KVL is an abstraction of Maxwells equations. It makes things easier while not influencing the result (when used correctly).

Because in many of your "abstractions" you used KVL instead of Maxwell, you think that KVL is an abstraction of Maxwell. Don't say that anymore.

Saying that means that every time you have a problem solvable by Maxwell, you can immediately apply KVL "used correctly" and that's it.

It implies that you can get away without understanding the underlying phenomena. No, you can't.

That's what Cyriel Mabilde disastrously did. That's what Lewin is desperately trying to warn you about.

That's because abstractions also have there limits. So do Maxwells equations when you get down to really small scales where quantum effects take over. You have to know about those limits and simply make sure you don't use that particular abstraction when outside of them.




Why you can get away with that in computing? Because those languages, including machine code, are all equivalent in computing power. So you can get a program working without knowing the details under the hood at the expense of a messy code.

Kirchhoff and Maxwell are NOT equivalent. If Kirchhoff and Maxwell were languages, Kirchhoff would describe a machine with LESS computing power than Maxwell.

In theory yes any truing complete language can do anything.

In practice the capabilities of programing languages vary a lot. Some languages are simply faster to compute a given task, no matter how well you optimize your program (While others complete it with less program code). But there is also certain low level functionality that many languages are simply not capable of. Modern CPUs often have special instructions that most compilers don't know how to use so they don't. There are special locked registers in CPUs that require a sequence of instructions that a lot of high level languages can't reproduce. These special things are usually not something that most programs need to do, but operating systems or hypervisors and such really need it, as they have to do things like set up the MMU, manage execution privilege levels, perform context switching, switch the CPU from 8086 compatibility mode into the full instruction set on boot etc... Normal programs running under a OS also make use of some special features such as JIT compiling where the whole program is basically self modifying code that compiles itself on the fly as it runs by jumping back into the compiler whenever needed. All of this is simply not possible in high level languages like python (Well apart from loading raw binary data into memory and then crashing the program in just the right way that the CPU ends up executing that area by mistake, but that's basically bypassing the language and using a hex editor to program)

This is much how Kirchhoff and Maxwell are NOT equivalent. Kirchhoff can do most things one would normally need to do, but not everything. The things it can do it usually does in a way that is move convenient than the alternative, for the things it can't do then you have no choice but to use the alternative.


Again. Who told you that you could reduce the problem to KVL? Kirchhoff? Certainly not. Kirchhoff is a bird. He doesn't know anything about propagation, delay lines, fields, etc.

You had to use MAXWELL, and you did it almost unconsciously, to reduce to problem to Kirchhoff. So you confirm what I said in one of my first messages on this thread about how we engineers are so used to that practice that we forget that we are in fact using Maxwell and implicitly reducing the problem to Kirchhoff.

You can abstract, but you cannot use this as an excuse to ditch the fundamentals. What people are doing is not even trying to study Maxwell, consequently not understanding what the flux is going on and criticizing Lewin for THEIR ignorance.

This is the most stupid educational move I've seen in decades.

Kirchhoff was not involved in in that phased array. I was making an example why wave propagation doesn't automatically make Maxwell necessary, or even make physics necessary.

I was making use of physics of wave propagation to abstract the problem down to just geometry. Waves traveling in a constant uniform medium always travel at the same speed, this leads to a conclusion that the time delay from the transmitter to the receiver is only a function of distance. With that i can craft simple factor to multiply with in order to translate distance into time delay. With this number in hand i can then predict the waveform this element will receive and feed that on into the phased array beam steering math. No physics involves what so ever, only geometry.

The results it gave matched up with other tools and with experimental results.

You only have to understand enough of the underlying physics to determine what abstraction is appropriate (if any). No need to calculate the whole thing using fundamental physics first. The understanding is more valuable than being able to blindly put numbers into famous equations. Sticking numbers into equations blindly without trying to understand what they are is ignorance. Applying understanding of the subject to form a simpler abstraction to make things easier and faster is instead called "getting stuff done".


TLDR.

Now that I've made you a convert, let's help others to avoid saying stupid things like "Physics has no use for Kirchhoffs law since it doesn't deal with anything physical."

I was almost using that quote as a signature. But then I decided to give you a second chance.

Made me convert to what?

Kirchhoffs cirucit laws still are not some fundamental law of the universe or something. Its just one of the sets of laws that make circuit meshes work. I have yet to see Kirchhoffs laws be wrong when they are used as intended. Its a great abstraction that helps you make sense of physical things.

You have demonstrated in the transformer video how useful the circuit mesh abstraction is. Transformers don't have additional winding that make leakage inductance, they don't have a physical resistor inside them that causes core losses. Yet it acts pretty much like that was the case so that's why the real transformer model uses it. It makes things much simpler to work with while acting close enough. You even use such simplifications before you get to the equivalent circuit model. For example you consider two turns in a coil as simply being 2 times a single turn, while showing segments of wire that go up diagonally to connect the two and a set of wires coming out and then showing a voltage across the two wires without closing them into a loop. I'm not saying its wrong to do this, it makes perfect sense to do it, but for these same reasons is why other people had issues with my lumped model of Dr. Lewins experimental circuit. For some reason i was not allowed to insert inductors into the equivalent model and not allowed to have a voltage on a non closed loop wire segment.

Absolutely nothing wrong in that video (Okay maybe apart from the voice)
Title: Re: Does Kirchhoff's Law Hold? Disagreeing with a Master
Post by: ogden on December 02, 2018, 11:10:28 am
For some reason i was not allowed to insert inductors into the equivalent model and not allowed to have a voltage on a non closed loop wire segment.

It's privilege of physics [self-acclaimed] gurus only. As you did not worship Dr.Lewin - you are not worthy to use inductor models.

Quote
Absolutely nothing wrong in that video (Okay maybe apart from the voice)

Gloves :D

Hey, bsfeechannel, where do you come from?

My guess would be: Brazil?
Title: Re: Does Kirchhoff's Law Hold? Disagreeing with a Master
Post by: HackedFridgeMagnet on December 03, 2018, 12:47:06 am
Sorry you still don't get it.
The only thing the oscilloscope reads is the voltage at its terminals.
It displays a representation of that scalar voltage.
It doesn't show the voltage at at A and D even though the probes are connected to A and D.

If you want the oscilloscope to read the voltage at A and D you must not add extra flux into the loop.
Cyriel Mabilde demonstrated how to do this.
It is not magic.
It is measurable.
He even knows the value.

I think 15 year olds would understand this better than some older people seem to be able to do.
Title: Re: Does Kirchhoff's Law Hold? Disagreeing with a Master
Post by: bsfeechannel on December 03, 2018, 02:26:58 am
You have demonstrated in the transformer video how useful the circuit mesh abstraction is. Transformers don't have additional winding that make leakage inductance, they don't have a physical resistor inside them that causes core losses. Yet it acts pretty much like that was the case so that's why the real transformer model uses it. It makes things much simpler to work with while acting close enough.

I had to make a lot of implicit assumptions to REDUCE the model. That is stated in the video. These assumptions were only possible because Maxwell told me what is going on. Kirchhoff can't explain them in any way. For instance, where does the leakage inductance come from? What exactly produces it? How can it be modeled as a lumped component? Why do we have a magnetizing current on the primary even without a load on the secondary. Is that a "parasitic" current? Can we reduce it? If we can, how? How will this affect the other parameters?

While I think about that and derive the calculations to model the device, Kirchhoff sits in the corner like Jack Horner. I only call him to the party in the last minute, when all the components of my model are figured out.

So Kirchhoff is just a fancy interface in the end to hide a complicated mechanism. It serves no purpose in the design of the transformer.

Quote
You even use such simplifications before you get to the equivalent circuit model. For example you consider two turns in a coil as simply being 2 times a single turn, while showing segments of wire that go up diagonally to connect the two and a set of wires coming out and then showing a voltage across the two wires without closing them into a loop.

The arrow indicates where you are going to place your voltmeter, and then the loop will be closed. If the diagonal wire bothers you, just rotate the top loop clockwise a little until the connecting wire becomes perpendicular.

Quote

For some reason i was not allowed to insert inductors into the equivalent model and not allowed to have a voltage on a non closed loop wire segment.

Because my transformer and Lewin's experiment have a crucial difference. While the transformer has a fixed topology, Lewin's experiment doesn't. And Maxwell showed that topology is everything in electromagnetism.

If you move the voltmeter an isty bitsy tiny little femtometer, it will measure a different voltage. Your voltmeter may not be sensitive enough to catch very small variations, but they will be there. There's simply no right way to measure voltages on Lewin's experiment. It is undefined.

Since Kirchhoff knows nothing about how your circuit is arranged in space, if you want to reduce Lewin's experiment to Kirchhoff you have to define the topology, either explicitly, or implicitly like in the case of a transformer, but once you do that, it will not be Lewin's experiment anymore.

For more details, revisit this post:

https://www.eevblog.com/forum/chat/does-kirchhoffs-law-hold-disagreeing-with-a-master/msg1962869/#msg1962869 (https://www.eevblog.com/forum/chat/does-kirchhoffs-law-hold-disagreeing-with-a-master/msg1962869/#msg1962869)

Quote
Absolutely nothing wrong in that video (Okay maybe apart from the voice)

Thanks.
Title: Re: Does Kirchhoff's Law Hold? Disagreeing with a Master
Post by: bsfeechannel on December 03, 2018, 03:55:29 am
If Berni, Electroboom and Cyriel Mabilde got it wrong then let us know what you think voltages are on the loop in Dr Lewins experiment.

After all these posts, you still don't get it.

We've been discussing this topic for a month now and, although apparently the vast majority of EEs on this forum understands what is at stake, there are still a few people who think that Mehdi is a hero, Lewin is a charlatan, I am a troll, and Maxwell is just a complicated version of Kirchhoff.

The problem seems to be that Lewin's "Kirchhoff is for the birds" rant is being perceived as an attack on those who learned to rely on Kirchhoff. But Lewin's rant is directed at some of those who TEACH circuit analysis.

What he is exposing is not new. Somewhere on the thread I recommended the book "Electromagnetic Waves and Radiating Systems" by Jordan and Balmain. Its first edition is from 1950. The very first section of the first chapter approaches the problem using very well chosen words and taking care not to offend susceptibilities.

But the issue is the same. Circuit analysis (a.k.a Kirchhoff) is being taught by some as an equivalent theory to Maxwell. Worse than that: it is being taught as a theory based on different postulates, as if Maxwell had nothing to do with Kirchhoff. Worse still: Maxwell is an extension of Kirchhoff for very limited cases where Kirchhoff, surprisingly, doesn't work. Even worse still: Kirchhoff is a series of tricks to solve circuits, it has nothing to do with physics. Maxwell has nothing to do with practical engineering: it is a thing of interest to physicists disconnected from the reality of practical engineering.

All of this is absolutely far from the truth. And the attempts at remedying it are timid.

We must admit that Lewin had a lot of balls to defy the educational establishment and maintain his integrity even after having retired from teaching.
Title: Re: Does Kirchhoff's Law Hold? Disagreeing with a Master
Post by: HackedFridgeMagnet on December 03, 2018, 04:48:39 am
Quote
you must not add extra flux into the loop
Quote
So, let me ask you again: what is 'extra flux'?

Not sure why you cant understand me.
I mean any non zero net flux that is intercepted by the act of probing the points A and D.
Watch Mabilde he knows how to probe a voltage correctly.
The reason why I call it a voltage and not an emf is because the oscilloscope can only probe voltages.

Quote
You do realize you are saying that the reading does not depends on the endpoints only but also on the path?
Yes
Quote
So, you are saying Lewin is right.
No
Quote
And yet, you still talk about "voltage on the loop" as if it were a property of the loop.
?? I talked about measuring the voltages at A and D  but only because I think that  is what Dr Lewin is trying to do.

Anyway I think I have said my piece.
Thanks for the discussion all.

ps . I like Dr Lewins lectures. I have only seen a few though.
Title: Re: Does Kirchhoff's Law Hold? Disagreeing with a Master
Post by: Berni on December 03, 2018, 07:11:04 am
Nobody here was trying to say that Maxwells equations or Faradays law are wrong or even just an extension of Kirchhoffs laws.

I don't have anything against Dr. Lewin and i think he makes some amazing lectures. Its just his explanation of Kirchhoffs law that i don't agree with. Not that i have some secret love relationship with Kirchhoff, but from my experience with the electronics engineering field i never saw this way of using Kirchhoffs voltage law to be the correct use for it, hence why it does not work in his case.



I had to make a lot of implicit assumptions to REDUCE the model. That is stated in the video. These assumptions were only possible because Maxwell told me what is going on. Kirchhoff can't explain them in any way. For instance, where does the leakage inductance come from? What exactly produces it? How can it be modeled as a lumped component? Why do we have a magnetizing current on the primary even without a load on the secondary. Is that a "parasitic" current? Can we reduce it? If we can, how? How will this affect the other parameters?

While I think about that and derive the calculations to model the device, Kirchhoff sits in the corner like Jack Horner. I only call him to the party in the last minute, when all the components of my model are figured out.

So Kirchhoff is just a fancy interface in the end to hide a complicated mechanism. It serves no purpose in the design of the transformer.

Leakage inductance are simply magnetic field lines that don't pass trough both coils (Including connecting wires to the coils). The field has to be simulated in space to find this inductance computationally since it heavily depends on the coil placement and core design. It behaves like extra uncoupled inductance so can be modeled as a inductor. Magnetizing current is determined by the core properties(or in an abstract way by inductance and frequency). It can be reduced by changing the number of turns or swapping out the core for a higher permeability one. Alternatively the reactive part of this current can be completely removed using a compensation capacitor while the real current can only be reduced using lower resistance coils and lower loss material in the core. It affects other parts of the transformer depending on what you do but in most cases makes the transformer larger and heavier or more expensive to produce(Ferrite core).

All of this is indeed all Maxwell. I never said Maxwell was useless. Just that its not always the best choice if there are simpler alternatives.

Never said that Kirchhoff could do everything. It also can't calculate how many miles per gallon my car can do. Its simply not what Kirchhoff is for. Its only job is to describe how currents and voltages work inside ideal circuit meshes. That's all it does, it doesn't do anything more than that.


The arrow indicates where you are going to place your voltmeter, and then the loop will be closed. If the diagonal wire bothers you, just rotate the top loop clockwise a little until the connecting wire becomes perpendicular.

Yes but the path you take when connecting your voltmeter matters. So the sensible way to connect it is in a way that makes no EMF on the wires, as i have did before. Nothing wrong with that method, yet it was somehow wrong when i used it.


Because my transformer and Lewin's experiment have a crucial difference. While the transformer has a fixed topology, Lewin's experiment doesn't. And Maxwell showed that topology is everything in electromagnetism.

If you move the voltmeter an isty bitsy tiny little femtometer, it will measure a different voltage. Your voltmeter may not be sensitive enough to catch very small variations, but they will be there. There's simply no right way to measure voltages on Lewin's experiment. It is undefined.

Since Kirchhoff knows nothing about how your circuit is arranged in space, if you want to reduce Lewin's experiment to Kirchhoff you have to define the topology, either explicitly, or implicitly like in the case of a transformer, but once you do that, it will not be Lewin's experiment anymore.

For more details, revisit this post:

https://www.eevblog.com/forum/chat/does-kirchhoffs-law-hold-disagreeing-with-a-master/msg1962869/#msg1962869 (https://www.eevblog.com/forum/chat/does-kirchhoffs-law-hold-disagreeing-with-a-master/msg1962869/#msg1962869)

Lewins experiment had no moving parts, the wires to the resistor and wires to both oscilloscopes remained stationary throughout the experiment.

Maxwell also would need to know the exact femtometer accurate path the wire takes to make an accurate prediction. Such measurements ware not provided so i assumed the wires to run perfectly in the same path as does his math. In my case Maxwell was used to obtain the inductance of the wire segments. After all of it i had a circuit that acts within the margin of error like Dr. Lewins experiment.

I fully agree that there are two voltages over points A and B in Dr. Lewins circuit. The math does work out that way.

I just don't agree in his application of Kirchhoffs law to prove its wrong. It needs a lumped circuit mesh to work right and i was able to turn his circuit into a valid and accurately behaving circuit mesh in 5 minutes where it shows that Kirchhoff indeed works.

So let me ask you then what is the correct use of Kirchhoffs laws?
Title: Re: Does Kirchhoff's Law Hold? Disagreeing with a Master
Post by: Sredni on December 03, 2018, 06:18:06 pm
I just don't agree in his application of Kirchhoffs law to prove its wrong. It needs a lumped circuit mesh to work right

? If you are referring to the mesh shown in "Science and believing..." (or what is the name), you are mistaken.
It might look like a lumped circuit mesh, but it's not. In fact, the emf is not lumped anywhere.
This is why we keep saying he's using Faraday.

Quote
and i was able to turn his circuit into a valid and accurately behaving circuit mesh in 5 minutes where it shows that Kirchhoff indeed works.

I will get to the shortcomings of the lumped representation later, but for now, let me ask you: how did you dimension the probe's inductors? Would those values change if the probes' cables were twice as long, and covered a different area?
Title: Re: Does Kirchhoff's Law Hold? Disagreeing with a Master
Post by: MiDi on December 03, 2018, 06:23:11 pm
https://youtu.be/Q9LuVBfwvzA (https://youtu.be/Q9LuVBfwvzA)
Title: Re: Does Kirchhoff's Law Hold? Disagreeing with a Master
Post by: Berni on December 03, 2018, 06:32:22 pm
Yes that video does indeed clear up a lot of things. Its also worth taking a look at the document that Dr. John W. Belcher wrote on the topic of his videos.

I will get to the shortcomings of the lumped representation later, but for now, let me ask you: how did you dimension the probe's inductors? Would those values change if the probes' cables were twice as long, and covered a different area?

I got it from seeing how his experiment is set up and applied Maxwells concepts to give wire segments a realistic inductance. Since all of these wire segments take the same path 1/4 turn around the circle means they all have to have  the same inductance value (All of it being coupled inductance rather than self inductance)

Of course they would change if you move the wires. Its the inductor values that are capturing the physical and magnetic properties of a circuit. So changing the wire paths changes the circuits behavior so its a new different circuit. Update the inductance values to match it and the equivalent lumped circuit will again behave the same as the real one.

You have to put new updated numbers into Maxwells equations too if you change the path, right?
Title: Re: Does Kirchhoff's Law Hold? Disagreeing with a Master
Post by: dr.diesel on December 03, 2018, 07:24:24 pm
It's sad to hear Dr. Lewin attack Mehdi with the education jab, just so unnecessary. 

Mehdi's approach and professional comments on this topic clearly show he is of the highest moral character and only wishes to constructively discuss the topic.  A very well done to him.

Anxious to hear Lewin's response to the last video.
Title: Re: Does Kirchhoff's Law Hold? Disagreeing with a Master
Post by: bsfeechannel on December 03, 2018, 11:57:13 pm
It's sad to hear Dr. Lewin attack Mehdi with the education jab, just so unnecessary. 

The title of his first video is "Disagreeing with a master". Science has no word of authority. It seems that Mehdi wanted to provoke that kind of reaction and Lewin fell prey beautifully.

If you go to the comment section of Mehdi latest video, the most popular comments are all bashing the octogenarian professor, with hearts conceded by Mehdi. It gives us the impression that that's what Mehdi was after the whole time. But it is just an impression and I may be wrong.

Quote
Mehdi's approach and professional comments on this topic clearly show he is of the highest moral character and only wishes to constructively discuss the topic.  A very well done to him.

I read Belcher's paper. Nowhere he says KVL always holds or that Lewin is wrong.

He says: In this sense [i.e. not always], KVL holds, as argued by Mehdi Sadaghdar, but one must always remember that the voltage difference across the inductor is defined in a very different way compared to the voltage difference across the other three elements.

What is this different way Belcher refers to? It is described in The Feynman Lectures on Physics Vol II, p22‐2.

Feynman says:  Suppose that we have a coil like an inductance except that it has very few turns, so that we may neglect the magnetic field of its own current. This coil, however, sits in a changing magnetic field such as might be produced by a rotating magnet, as sketched in Fig. 22-5. (We have seen earlier that such a rotating magnetic field can also be produced by a suitable set of coils with alternating currents). Again we must make several simplifying assumptions. The assumptions we will make are all the ones that we described for the case of the inductance. In particular, we assume that the varying magnetic field is restricted to a definite region in the vicinity of the coil and does not appear outside the generator in the space between the terminals.

(https://www.eevblog.com/forum/chat/does-kirchhoffs-law-hold-disagreeing-with-a-master/?action=dlattach;attach=587351;image)

It is still true, however, that the line integral of E around a complete loop, including the return from b to a outside the generator, must be zero, because there are no changing magnetic fields.


One thing that Mehdi omits is that Feynman actually derives Kirchhoff's rules (not laws)--by the way, Lewin also calls them rules, not laws--from Maxwell [Section 22-3]. And doesn't say anywhere that Kirchhoff always holds. What he says is: With those two rules it is possible to solve for the currents and voltages in any network.

Network of LUMPED elements, i.e., no changing fields outside of the components and this is the abc of Kirchhoff.

Feynman uses the line integral around a circuit just like Lewin does.

Quote
Anxious to hear Lewin's response to the last video.

Mehdi is not completely honest with his audience and the literature he recommends doesn't disprove in any way Lewin's experiment.

And that is what is really sad.
Title: Re: Does Kirchhoff's Law Hold? Disagreeing with a Master
Post by: ogden on December 04, 2018, 12:15:34 am
I read Belcher's paper. Nowhere he says KVL always holds or that Lewin is wrong.

You can troll twist it as you like, but Belcher says that KVL holds for Lewin's circuit (page 16), at the same time agreeing to Mehdi.
Title: Re: Does Kirchhoff's Law Hold? Disagreeing with a Master
Post by: Sredni on December 04, 2018, 12:19:58 am
We've been discussing this topic for a month now and, although apparently the vast majority of EEs on this forum understands what is at stake, there are still a few people who think that Mehdi is a hero, Lewin is a charlatan, I am a troll, and Maxwell is just a complicated version of Kirchhoff.

The problem seems to be that Lewin's "Kirchhoff is for the birds" rant is being perceived as an attack on those who learned to rely on Kirchhoff. But Lewin's rant is directed at some of those who TEACH circuit analysis.

I am afraid the problem is deeper than that.
I had a look at Mehdi's latest video (the "conclusion") and the ensuing comments and it's beyond sad. It's scary.
December the third should mark the birth of scientific populism.
And he even links an essay by a professor telling him what the field is inside the conductors! (who's gonna break it to Mabilde, now?)
All he did was to pretend the beef was about what they decided to call KVL. Then showed what it intended with 'extended KVL' and showed that it works (duh!) for lumped circuits. And even Feynman says so!
And when he tries to apply to a non-lumped circuit he tries to locate v/2 on the conductor as if potential difference had a meaning and says... "now that's not wrong but it's a bit misleading...".

"A bit misleading". "Not wrong".

And all the fanboys chanting "lock Lewin up!" (metaphorically speaking - some say he's so old he lost his wits, other point out he has an ego problem, others highlight why he was let go from MIT, others expect this to be called ElectroBoom's law...).
And he linked a document saying "if the inductor wires are perfectly conducting, this integral is zero because there is no electric field in the wires" and showing how the superposition of the coulomb and induced field is such to give (almost) zero field in the conductors and charge accumulation on the resistors (just as Lewin always said).

Science validated by upvoting.
This is worrisome.
Title: Re: Does Kirchhoff's Law Hold? Disagreeing with a Master
Post by: bsfeechannel on December 04, 2018, 12:44:49 am
Mehdi fought the old professor and lost. But made a video to appear that he won. He can fool his supporters (after all he needs their money), but not those who are experienced and versed in the theory.

Although his comment section is a despicable scene, and he has nothing really to contribute, at least he makes me laugh with his antics.
Title: Re: Does Kirchhoff's Law Hold? Disagreeing with a Master
Post by: ogden on December 04, 2018, 01:14:38 am
Science validated by upvoting.
This is worrisome.

Come on. When there's paper with real arguments you suddenly [conveniently] talk about youtube comments??

Prof. Belcher's document (link below) shall be considered as peer review of Lewin's experiment.

Mehdi fought the old professor and lost. But made a video to appear that he won.

You really shall read Prof. Belcher's document which says that Mehdi won his argument.

http://www.electroboom.com/share/FaradaysLaw_Mehdi.pdf (http://www.electroboom.com/share/FaradaysLaw_Mehdi.pdf)
Title: Re: Does Kirchhoff's Law Hold? Disagreeing with a Master
Post by: Sredni on December 04, 2018, 01:30:29 am
When there's paper with real arguments you talk about youtube comments??

I promised not to interact with you on this matter, but... are you asking me where is the paper stating what I put into quotes in my previous post?
Well, it's Belcher's document.

Quote
You really shall read Prof. Belcher's document

Yup. You should indeed.

Quote
which says that Mehdi won his argument.

Of course, reading is necessary but not sufficient condition to understand what it says.
Title: Re: Does Kirchhoff's Law Hold? Disagreeing with a Master
Post by: ogden on December 04, 2018, 01:38:26 am
I promised not to interact with you on this matter, but... are you asking me where is the paper stating what I put into quotes in my previous post?
Well, it's Belcher's document.

Well, then in case you missed that - same document states "KVL holds, as argued by Mehdi Sadaghdar"
Title: Re: Does Kirchhoff's Law Hold? Disagreeing with a Master
Post by: electrodacus on December 04, 2018, 01:47:47 am
You can not measure two different voltages when you measure in the exact same two points at the exact same point in time.
I will give it a go at explaining the problem but I'm to lazy to draw something so I will rely on my bad English and your capability to reconstruct the proposed experiments in your mind.

1) Imagine a copper loop say it is a ring with a resistance of 1Ohm and an emf of 1V thus creating a round 1A current trough the ring.
Now get an ideal voltmeter and probe two opposite points on the ring so that the ring is exactly split in two equal parts by this two points.
What do you think the reading will be ?  It will be 0V.
It will not make any sense to try and probe with two voltmeters there is nothing to be proven with that as they will read the exact same thing.

2) While point 1 should be enough to demonstrate my point let me get another example.
I imagine the same ring but this time half of the ring has 0.9Ohm and the other half has 0.1Ohm. (same 1V / 1A).
Same ideal voltmeter connected at the points where those two half rings intersect will read what ? :)
Answer will be the same as if you have an equivalent circuit with two 0.5V ideal sources one on each half and with a 0.1Ohm series resistor on one side and 0.9Ohm resistor on the other side.
So 400mV (the sign will also be fixed but to keep things simple I did not provided all the info).
Title: Re: Does Kirchhoff's Law Hold? Disagreeing with a Master
Post by: Sredni on December 04, 2018, 03:08:43 am
You can not measure two different voltages when you measure in the exact same two points at the exact same point in time.

Aaaaand... we're back to square one.

Quote
I will give it a go at explaining the problem but I'm to lazy to draw something

Let me guess, you are too lazy to read the other posts in the previous thirteen pages of this thread as well.

<sigh>


Quote from: ogden
Well, then in case you missed that - same document states "KVL holds, as argued by Mehdi Sadaghdar"

Thank you for proving my point about the disjointness of reading and understanding.
Maybe if you read it again you will see that section 11 KVL is about lumped circuits. You too should go back read all past posts.
I'm done talking with walls.
Title: Re: Does Kirchhoff's Law Hold? Disagreeing with a Master
Post by: electrodacus on December 04, 2018, 04:52:47 am
You can not measure two different voltages when you measure in the exact same two points at the exact same point in time.

Aaaaand... we're back to square one.

Please point out what part of my short statement is wrong.
And if that is not wrong then what was the point of that experiment?
Title: Re: Does Kirchhoff's Law Hold? Disagreeing with a Master
Post by: Berni on December 04, 2018, 06:40:12 am
You can not measure two different voltages when you measure in the exact same two points at the exact same point in time.

Aaaaand... we're back to square one.

Please point out what part of my short statement is wrong.
And if that is not wrong then what was the point of that experiment?

I do have to agree with Sredini this time.

The problem is that this ideal voltmeter still needs wires to connect to the points of interest on the circle and that's how you can get different voltages depending on what path these wires take, they are as much part of the circuit as the loop itself.

If you take the formal definition of voltage that says that the voltage is the integral of all forces acting on electrons along a path. Then you do get a different number depending on what path do you take trough the loop. By definition there are indeed two possible voltages across the two points. This is because both the electric field of charge separation and the magnetic EMF count towards the total voltage.

Voltmeters only read the charge separation part of the voltage because that's what drives current trough its internal resistance. If you connect the voltmeter with the wires taking such a path that the magnetically induced EMF is zero you get to measure that charge separation and you get the single result you expect (The result is indeed 0.4V). This is the same result that circuit mesh analysis will give (that is called KVL here, but its more than just KVL).

The voltage from charge separation is always defined as a single number for all points in any circuit. Its simply how many electrons are sitting there. More electrons more negative the voltage. Its only the magnetic EMF part of the voltage that depends on what path you take, this is because that EMF is pulling the electrons into a certain direction. This makes electrons motivated to move in that direction even if there is already the same amount of electrons there, but only in that direction.

So to conclude yes there are two voltages across A and B in Dr. Lewins circuit according to the formal definition of voltage, but this is not useful voltage that can be harnessed, its just a incomplete calculation of voltage that requires the rest of the loop to be added in too and that then gives you a single result. In your case that is a single result of voltage across the voltmeter terminals.

These multiple voltage across two points in electrical engineering do sort of the same thing as complex numbers in math. The imaginary part of complex numbers don't really physically exist, but it is there to make the math work out that otherwise wouldn't be possible(Such as square roots of negative numbers). Same goes here with Dr. Lewins example. The voltages are indeed there according to the math, but its not a real voltage you can "physically touch" in the real world. Much like a voltage of 6 V or 5.66+j2 V look the same to a voltmeter but are not the same in the math.

EDIT: Note there are no complex numbers involved in Dr. Lewins example. I was just making a comparison to math. Tho if you want you can still stick complex values of voltage and current into Kirchhoffs circuit laws and it still works(very useful when you have AC sources and reactive components)
Title: Re: Does Kirchhoff's Law Hold? Disagreeing with a Master
Post by: beanflying on December 04, 2018, 06:45:54 am
I think that regardless of absolute right or wrong Mehdi is to be commended for questioning (politely) an academic of note, creating a reasoned hypothesis of his own, testing and working through his thoughts in a practical manner on a difficult concept in a way to not turn off those not as technically minded. :-+

Some of the comments on youtube and the initial ego driven reply from Lewin did nothing to help solve the question. There is scope for rigorous scientific experimental verifiable testing of this to maybe get closer to a resolution, living room maths on a whiteboard and playing on a kitchen bench isn't going to do it.

Musing to myself over a Beer Crazy Beard to with guests of Crazy Hair and Crazy Unibrow at some point in the very distant future on proper measurement and probing techniques would be a fun watch too 8)

https://www.youtube.com/watch?v=2vzvWUqUtb8&t=2282s (https://www.youtube.com/watch?v=2vzvWUqUtb8&t=2282s)
Title: Re: Does Kirchhoff's Law Hold? Disagreeing with a Master
Post by: Sredni on December 04, 2018, 06:47:36 am
Please point out what part of my short statement is wrong.
And if that is not wrong then what was the point of that experiment?

Allow me to apologize for being so blunt, but I was really disappointed by Electroboom's latest video and the reaction of his fans.
The answer to your question, though, is in the many many posts of this thread. Please, browse through it and you will see that it is indeed possible for two voltmeters whose probes' tips are attached to the very same two points to read different values at the same time. There are also youtube video showing this, if you do not believe it.
The reason is in the different flux intercepted by the two meshes the circuit is partitioned into.

If you are curios, there are thirteen pages awaiting for you.
Title: Re: Does Kirchhoff's Law Hold? Disagreeing with a Master
Post by: electrodacus on December 04, 2018, 07:27:47 am
I do have to agree with Sredini this time.

The problem is that this ideal voltmeter still needs wires to connect to the points of interest on the circle and that's how you can get different voltages depending on what path these wires take, they are as much part of the circuit as the loop itself.

What will be the point of a real voltmeter in this example ? They will need to be added as components to the circuit and will only complicate the demonstration but not change the result if you correctly add them.   


If you take the formal definition of voltage that says that the voltage is the integral of all forces acting on electrons along a path. Then you do get a different number depending on what path do you take trough the loop. By definition there are indeed two possible voltages across the two points. This is because both the electric field of charge separation and the magnetic EMF count towards the total voltage.

The result will be the same 0V for first example and 0.4V for the second example and it the second case not just the value will be the same but also the polarity. We are talking about a fixed moment is time and the real voltage not the indication of two real voltmeters that are part of the circuit and inside the magnetic field measuring something else than the real voltage between those two points.


Allow me to apologize for being so blunt, but I was really disappointed by Electroboom's latest video and the reaction of his fans.
The answer to your question, though, is in the many many posts of this thread. Please, browse through it and you will see that it is indeed possible for two voltmeters whose probes' tips are attached to the very same two points to read different values at the same time. There are also youtube video showing this, if you do not believe it.
The reason is in the different flux intercepted by the two meshes the circuit is partitioned into.

If you are curios, there are thirteen pages awaiting for you.

See the replay above made to Berni.  I did read most of this tread (about 70%) and seen the a few videos from both sides.
Is not about believing is about real world as we are currently able to understand and in this universe there can be only one voltage at a fix moment in time.

The reason is in the different flux intercepted by the two meshes the circuit is partitioned into.

That is what I was answering above as in you are not measuring the actual voltage at the tip of the probes.
You can even completely remove the ring made of two resistors and keep just the two voltmeter's and you will read something but not what it will be at the tip of those two voltmeters.
Title: Re: Does Kirchhoff's Law Hold? Disagreeing with a Master
Post by: ogden on December 04, 2018, 08:14:24 am
Maybe if you read it again you will see that section 11 KVL is about lumped circuits. You too should go back read all past posts.

I thought that this forum agreed about lumped circuit of Dr.Lewin's experiment, yet you woke up. Aaaaand... we're back to square one. To talk about Maxwell's equations make sure you know laws of nature as well. Seems, I am done with this thread disregarding what sofa experts like you say. Your copy/paste skills are not even entertaining anymore.  :=\
Title: Re: Does Kirchhoff's Law Hold? Disagreeing with a Master
Post by: bsfeechannel on December 04, 2018, 09:15:38 am
I think that regardless of absolute right or wrong Mehdi is to be commended for questioning (politely) an academic of note, creating a reasoned hypothesis of his own, testing and working through his thoughts in a practical manner on a difficult concept in a way to not turn off those not as technically minded. :-+

Politeness is commendable, but we are engineers. We design serious stuff: buildings, bridges, cars, airplanes, life-supporting systems, telecommunication systems, power systems.

If we give up our integrity, people die.
Title: Re: Does Kirchhoff's Law Hold? Disagreeing with a Master
Post by: beanflying on December 04, 2018, 09:23:00 am
I think that regardless of absolute right or wrong Mehdi is to be commended for questioning (politely) an academic of note, creating a reasoned hypothesis of his own, testing and working through his thoughts in a practical manner on a difficult concept in a way to not turn off those not as technically minded. :-+

Politeness is commendable, but we are engineers. We design serious stuff: buildings, bridges, cars, airplanes, life-supporting systems, telecommunication systems, power systems.

If we give up our integrity, people die.

Not maintaining Integrity? Explain please?

I see NO where he didn't maintain his integrity at the highest level which has little or nothing being proven correct or incorrect in the long run. If English is not your first language then best you check the definition of the word. Your attacking of the mans integrity and not debating the technicalities of the subject is out of order!
Title: Re: Does Kirchhoff's Law Hold? Disagreeing with a Master
Post by: bsfeechannel on December 04, 2018, 10:24:31 am
I think that regardless of absolute right or wrong Mehdi is to be commended for questioning (politely) an academic of note, creating a reasoned hypothesis of his own, testing and working through his thoughts in a practical manner on a difficult concept in a way to not turn off those not as technically minded. :-+

Politeness is commendable, but we are engineers. We design serious stuff: buildings, bridges, cars, airplanes, life-supporting systems, telecommunication systems, power systems.

If we give up our integrity, people die.

Not maintaining Integrity? Explain please?

I see NO where he didn't maintain his integrity at the highest level which has little or nothing being proven correct or incorrect in the long run. If English is not your first language then best you check the definition of the word. Your attacking of the mans integrity and not debating the technicalities of the subject is out of order!

The first thing I did was to analyze his latest video from the technical standpoint. And I've been discussing the technical points of Lewin's experiment from my post #1 in this thread.

I've been an engineer for long enough to know what integrity means and I don't need the English language to tell me that.
Title: Re: Does Kirchhoff's Law Hold? Disagreeing with a Master
Post by: beanflying on December 04, 2018, 10:47:37 am
Clearly you do need a dictionary and some basic decency and manners. Attacking the integrity of the person and not the ideas as you have just done is what in Australia we would call a c... act amongst others! In particular where the person in question is not here to defend themselves. You clearly can't show where Mehdi has compromised his 'integrity'!

interesting

A quick check shows you have maybe 40 posts in this thread so clearly you have a major bee in your bonnet and in a chunk of those posts you have decided to play the man not the subject. I will leave others to decide what this says about your character.
Title: Re: Does Kirchhoff's Law Hold? Disagreeing with a Master
Post by: bsfeechannel on December 04, 2018, 11:27:33 am
Please point out what part of my short statement is wrong.

All of it.

Suppose that we have the circuit of the schematic below.

(https://www.eevblog.com/forum/chat/does-kirchhoffs-law-hold-disagreeing-with-a-master/?action=dlattach;attach=587654;image)

All the components are lumped, and we only have batteries. The real value of the voltages and resistances are not important. However, to simplify our calculations, let's suppose the two resistors have the same value (it could be any other known proportion). Connected at the same two points in the circuit we have five voltmeters.

We agree that the five voltmeters will measure exactly the same voltage.

Now let's remove the batteries and apply a magnetic field that rises linearly in intensity with time, points towards you and is confined in the total area of the circuit, i.e., A1 + A2 + A3 + A4. 

(https://www.eevblog.com/forum/chat/does-kirchhoffs-law-hold-disagreeing-with-a-master/?action=dlattach;attach=587660;image)

According to Maxwell, this field will generate a constant EMF that will be proportional to the area enclosed by a circuit and whose polarity will be defined by the corkscrew rule. In short, the topology now counts. To simplify our calculations, let's suppose that the EMF generated by the loop enclosed by all four areas is 1V.

The voltages seen by the voltmeters will now be like that (in volts):

#1
-0.5

#2
( A1 - ( A2 + A3 + A4 ) ) / ( 2 * ( A1 + A2 + A3 + A4 ) )

#3
( ( A1 + A2 ) - ( A3 + A4 ) ) / ( 2 * ( A1 + A2 + A3 + A4 ) )

#4
( ( A1 + A2 +  A3 ) - A4 ) / ( 2 * ( A1 + A2 + A3 + A4 ) )

#5
+0.5

So, what voltmeter is measuring the "correct" voltage?
Title: Re: Does Kirchhoff's Law Hold? Disagreeing with a Master
Post by: bsfeechannel on December 04, 2018, 11:40:14 am
Clearly you do need a dictionary and some basic decency and manners. Attacking the integrity of the person and not the ideas as you have just done is what in Australia we would call a c... act amongst others! In particular where the person in question is not here to defend themselves. You clearly can't show where Mehdi has compromised his 'integrity'!

You're young. You need a hero. I'm old. I do not have time for this kind of nonsense.

Quote
A quick check shows you have maybe 40 posts in this thread so clearly you have a major bee in your bonnet

I like to contribute. Is that a problem?

Quote
and in a chunk of those posts you have decided to play the man not the subject. I will leave others to decide what this says about your character.

I'm not the subject of this thread. Mehdi and his (now debunked) claims are.
Title: Re: Does Kirchhoff's Law Hold? Disagreeing with a Master
Post by: BrianHG on December 04, 2018, 11:47:04 am
All the components are lumped, and we only have batteries. The real value of the voltages and resistances are not important. However, to simplify our calculations, let's suppose the two resistors have the same value (it could be any other known proportion). Connected at the same two points in the circuit we have five voltmeters.

We agree that the five voltmeters will measure exactly the same voltage.

Now let's remove the batteries and apply a magnetic field that rises linearly in intensity with time, points towards you and is confined in the total area of the circuit, i.e., A1 + A2 + A3 + A4. 

(https://www.eevblog.com/forum/chat/does-kirchhoffs-law-hold-disagreeing-with-a-master/?action=dlattach;attach=587660;image)

According to Maxwell, this field will generate a constant EMF that will be proportional to the area enclosed by a circuit and whose polarity will be defined by the corkscrew rule. In short, the topology now counts. To simplify our calculations, let's suppose that the EMF generated by the loop enclosed by all four areas is 1V.

The voltages seen by the voltmeters will now be like that (in volts):

#1
-0.5

#2
( A1 - ( A2 + A3 + A4 ) ) / ( 2 * ( A1 + A2 + A3 + A4 ) )

#3
( ( A1 + A2 ) - ( A3 + A4 ) ) / ( 2 * ( A1 + A2 + A3 + A4 ) )

#4
( ( A1 + A2 +  A3 ) - A4 ) / ( 2 * ( A1 + A2 + A3 + A4 ) )

#5
+0.5

So, what voltmeter is measuring the "correct" voltage?
With your example, you are either claiming that both resistor values in you example are equal in value, or identical current is flowing on each side like a perfect mirror, otherwise the EMF field will be lop-sided due to the different load of the resistors.

What I would like to see is having modify the original loop so that it bends inwards to the center at the measurement point, insert a tiny 6 pin sot-23 MCU right on a watch battery directly in the middle sampling the voltage at 10msps, not connection anywhere else, and optically feeding out the readings.
Title: Re: Does Kirchhoff's Law Hold? Disagreeing with a Master
Post by: sectokia on December 04, 2018, 12:15:39 pm

With your example, you are either claiming that both resistor values in you example are equal in value, or identical current is flowing on each side like a perfect mirror, otherwise the EMF field will be lop-sided due to the different load of the resistors.

What I would like to see is having modify the original loop so that it bends inwards to the center at the measurement point, insert a tiny 6 pin sot-23 MCU right on a watch battery directly in the middle sampling the voltage at 10msps, not connection anywhere else, and optically feeding out the readings.

Have a look at this:
http://www.hep.princeton.edu/~mcdonald/examples/lewin.pdf (http://www.hep.princeton.edu/~mcdonald/examples/lewin.pdf)

See equation 17 on page 5.

Here you will get the answer as per doing what you describe above.

There is actually no need for the meter to be 'in the middle' of the magnetic field. Remember its the loop path enclosing the magnetic flux that matters. Thus putting the meter in the middle of the magnetic field and having the probes divide it is no different to the meter being outside the magnetic field and having one probe divide it up the middle, as shown in the link above on page 5 circuit diagram.

In the case of Lewins original circuit you will read 0.4V so long as the area containing the flux is enclosed equally on both sides. As you move the meter from left to right you will read -0.1 to +0.9 simply depending on the ratio of the flux encircled on each side. It really is that simple.
Title: Re: Does Kirchhoff's Law Hold? Disagreeing with a Master
Post by: beanflying on December 04, 2018, 12:33:52 pm
You're young. You need a hero. I'm old. I do not have time for this kind of nonsense.

Seriously Not young at all and yet you decide this based on what? As to 'nonsense' you have time to attack Mehdi's integrity but not tell us where didn't maintain his integrity? Still waiting?

Quote
I like to contribute. Is that a problem?
I'm not the subject of this thread. Mehdi and his (now debunked) claims are.

When you descend into criticism of the person or others here  you open yourself up to becoming a subject of discussion for your behavior and personal attacks.

And Mehdi's claims are 'debunked' as decided by you? WOW your verbosity and frequency of posting must have made it so awesome work you have me instantly convinced....
Title: Re: Does Kirchhoff's Law Hold? Disagreeing with a Master
Post by: bsfeechannel on December 04, 2018, 12:37:35 pm
Quote
With your example, you are either claiming that both resistor values in you example are equal in value, or identical current is flowing on each side like a perfect mirror, otherwise the EMF field will be lop-sided due to the different load of the resistors.

EMF has nothing to do with the value of the resistors according to Maxwell, only with the varying field and the geometry of the circuit. If you change the proportion of the resistors, the voltmeters will read different values from what you see above, but they will all show different values among themselves.

In particular, if you choose the make the left resistor nine times greater than the right resistor, you will have the following voltages.

#1
-0.9

#2
( A1 - 9 * ( A2 + A3 + A4 ) ) / ( 10 * ( A1 + A2 + A3 + A4 ) )

#3
( ( A1 + A2 ) - 9 * ( A3 + A4 ) ) / ( 10 * ( A1 + A2 + A3 + A4 ) )

#4
( ( A1 + A2 +  A3 ) - 9 * A4 ) / ( 10 * ( A1 + A2 + A3 + A4 ) )

#5
+0.1

Quote
What I would like to see is having modify the original loop so that it bends inwards to the center at the measurement point, insert a tiny 6 pin sot-23 MCU right on a watch battery directly in the middle sampling the voltage at 10msps, not connection anywhere else, and optically feeding out the readings.

So you are suggesting to change the TOPOLOGY of the circuit to measure what you would like to see? What if you can't do that?

What if the wires are in fact the traces of a PCB? Or any other kind of installation where you cannot move them?
Title: Re: Does Kirchhoff's Law Hold? Disagreeing with a Master
Post by: bsfeechannel on December 04, 2018, 01:39:35 pm
Seriously Not young at all and yet you decide this based on what?

You looked young to me.

Quote
As to 'nonsense' you have time to attack Mehdi's integrity but not tell us where didn't maintain his integrity? Still waiting?

He tried to pass the impression that professor Belcher endorsed his claims against Lewin's demonstration. He even invoked the "ghost" of Feynman to bless his "honest questioning".

We read the literature Mehdi recommended and found nothing to discredit Lewin. Actually we realized that Lewin employs the same terminology and concepts Feynman uses. And that everything we discussed here that proves that Lewin's demonstration is right is in perfect accordance to what Feynman describes. No wonder. We, Feynman and Lewin studied Maxwell.

The comment section of his videos is full of people making personal attacks against Lewin. This means nothing because Youtube comments mean precisely dick (I don't know what this means but I found the phrase cute).

However, Mehdi congratulates with those who post that kind of comment and even capitalizes on all the brouhaha (Is that an English word?).

The title of his latest video on the subject is absolutely misleading. "Kirchhoff vs. Faraday"? Can we have a "Newton vs. Einstein?" Or perhaps a "Euclid vs. Hilbert"?

So you judge his integrity. You're right. I do not have time for that.

Quote
When you descend into criticism of the person or others here  you open yourself up to becoming a subject of discussion for your behavior and personal attacks.

Fair enough. Mehdi and his "integrity" are targets, because he decided to target Lewin personally. He cleverly does not do that in his videos, but his comment section betrays him.

Quote
And Mehdi's claims are 'debunked' as decided by you?

No. Maxwell.

Quote
WOW your verbosity and frequency of posting must have made it so awesome work you have me instantly convinced....

Good to read that.
Title: Re: Does Kirchhoff's Law Hold? Disagreeing with a Master
Post by: electrodacus on December 04, 2018, 07:36:04 pm
So, what voltmeter is measuring the "correct" voltage?

Correct voltage in your example assuming those two resistors are equal and circuit is symmetrical (uniform magnetic field from the middle of the circuit) will be 0V same as in my example number 1.
And if you want you can add those voltmeters to the circuit but then they will be part of the circuit. Each will measure a different thing but not the voltage between those two points of interest except for the one in the middle number 3 that could read the correct value of 0V assuming it was shielded since a real voltmeter will likely not be perfectly symmetrical in internal construction.
Title: Re: Does Kirchhoff's Law Hold? Disagreeing with a Master
Post by: bsfeechannel on December 05, 2018, 04:17:26 am
Each will measure a different thing but not the voltage between those two points of interest except for the one in the middle number 3 that could read the correct value of 0V assuming it was shielded since a real voltmeter will likely not be perfectly symmetrical in internal construction.

Excellent. So let's get rid of the other voltmeters and stick to the voltmeter #3. Let's suppose that its display has seven digits (pretty common on modern bench multimeters) and let's suppose it is properly "shielded", etc.

I hadn't defined the areas, but now let's suppose that their common side is 10cm, and that x+y = 20cm like in the picture below.

(https://www.eevblog.com/forum/chat/does-kirchhoffs-law-hold-disagreeing-with-a-master/?action=dlattach;attach=588280;image)

So, V3 = ( ( A1 ) - ( A2 ) ) / ( 2 * ( A1 + A2 ) ), x + y = 0.2 m and, since A1 = x * 0.1 and A2 = y * 0.1, and A1 + A2 = 0.02m², we will have that V3 in volts:

V3 = ( ( x * .1 ) - ( y * .1) ) / ( 0.04 ) = ( x - y)/0.4

If x = y, then V3 should read 0.000000V.

But now, let's imagine for a moment that this circuit doesn't have the voltmeter and then suddenly someone decides to connect it.

Look what happens if the unfortunate engineer assigned with that task misses the exact point of connection and places the voltmeter 200nm to the right. 200 nanometers. In that case x = 0.1000002 m, and y = 0.0999998 m.

V3 = (0.1000002 - 0.0999998) / 0.4 = 0.000001 V

So it's affecting the precision of the measurement. It's no biggie in this case because we know what to expect ( 0V). But what if the resistors didn't have exactly the same value? How do I know that this error is not due to a resistor mismatch? Because I could nanometrically place the voltmeter in the "correct" spot, adjusting it so that its voltage read the expected 0.000000V. But what if a mismatch in the value of the resistors is giving me a false reading?
Title: Re: Does Kirchhoff's Law Hold? Disagreeing with a Master
Post by: electrodacus on December 05, 2018, 04:30:04 am
Each will measure a different thing but not the voltage between those two points of interest except for the one in the middle number 3 that could read the correct value of 0V assuming it was shielded since a real voltmeter will likely not be perfectly symmetrical in internal construction.

Excellent. So let's get rid of the other voltmeters and stick to the voltmeter #3. Let's suppose that its display has seven digits (pretty common on modern bench multimeters) and let's suppose it is properly "shielded", etc.

I hadn't defined the areas, but now let's suppose that their common side is 10cm, and that x+y = 20cm like in the picture below.

(https://www.eevblog.com/forum/chat/does-kirchhoffs-law-hold-disagreeing-with-a-master/?action=dlattach;attach=588280;image)

So, V3 = ( ( A1 ) - ( A2 ) ) / ( 2 * ( A1 + A2 ) ), x + y = 0.2 m and, since A1 = x * 0.1 and A2 = y * 0.1, and A1 + A2 = 0.02m², we will have that V3 in volts:

V3 = ( ( x * .1 ) - ( y * .1) ) / ( 0.04 ) = ( x - y)/0.4

If x = y, then V3 should read 0.000000V.

But now, let's imagine for a moment that this circuit doesn't have the voltmeter and then suddenly someone decides to connect it.

Look what happens if the unfortunate engineer assigned with that task misses the exact point of connection and places the voltmeter 200nm to the right. 200 nanometers. In that case x = 0.1000002 m, and y = 0.0999998 m.

V3 = (0.1000002 - 0.0999998) / 0.4 = 0.000001 V

So it's affecting the precision of the measurement. It's no biggie in this case because we know what to expect ( 0V). But what if the resistors didn't have exactly the same value? How do I know that this error is not due to a resistor mismatch? Because I could nanometrically place the voltmeter in the "correct" spot, adjusting it so that its voltage read the expected 0.000000V. But what if a mismatch in the value of the resistors is giving me a false reading?

I do not disagree with anything you mentioned in you last replay. Correct measurement will read a single value and that is the real 0V no multiple voltages at a single defined point in time.
Of course if you move the measurement point you get a different reading that is normal in any circuit not just this particular case.
Title: Re: Does Kirchhoff's Law Hold? Disagreeing with a Master
Post by: Berni on December 05, 2018, 06:46:03 am
Hence why understanding how your probing works is important. Especially when expecting that sort of accuracy from the measurement.

It is why i thought it would be important for Dr. Lewin to explain the path the voltmeter wires take. His experiment only gives these results when the voltmeter wires hug the circuit loop wires tightly. You can get nearly any other voltage you want if you move the voltmeter wires around.

Once you start regularly using oscilloscopes for things >10MHz or with fast changing large currents you encounter all sorts of probing anomalies. Its part of the engineers job to determine if the measurement they did is accurate enough to be useful.

Yes according to the math there are two voltages across A and B, but this is not a voltage that can be used for anything until wires are run to it to connect it to something(Such as a voltmeter). Once that is done the path is defined completely and the resulting voltage is only a single well defined voltage appearing across the voltmeter you connected it to. What sort of EMF the voltmeters wires experience is totally up to you, but usually its most usefully that there is zero EMF because you can then consider the wire as being an ideal connection between two circuit nodes.
Title: Re: Does Kirchhoff's Law Hold? Disagreeing with a Master
Post by: electrodacus on December 05, 2018, 06:59:53 am
Yes according to the math there are two voltages across A and B, but this is not a voltage that can be used for anything until wires are run to it to connect it to something(Such as a voltmeter). Once that is done the path is defined completely and the resulting voltage is only a single well defined voltage appearing across the voltmeter you connected it to. What sort of EMF the voltmeters wires experience is totally up to you, but usually its most usefully that there is zero EMF because you can then consider the wire as being an ideal connection between two circuit nodes.

What is the math for the above example and what will those two different voltages be ?  -0V and +0V :)
From my understanding of physics in this universe (going to assume there is no parallel universe with same experiment running at the exact same time) there will always be a single voltage between any two points at a fixed moment in time no mater if there is a measurement device in there or not.
Title: Re: Does Kirchhoff's Law Hold? Disagreeing with a Master
Post by: Berni on December 05, 2018, 07:22:39 am
Yes according to the math there are two voltages across A and B, but this is not a voltage that can be used for anything until wires are run to it to connect it to something(Such as a voltmeter). Once that is done the path is defined completely and the resulting voltage is only a single well defined voltage appearing across the voltmeter you connected it to. What sort of EMF the voltmeters wires experience is totally up to you, but usually its most usefully that there is zero EMF because you can then consider the wire as being an ideal connection between two circuit nodes.

What is the math for the above example and what will those two different voltages be ?  -0V and +0V :)
From my understanding of physics in this universe (going to assume there is no parallel universe with same experiment running at the exact same time) there will always be a single voltage between any two points at a fixed moment in time no mater if there is a measurement device in there or not.

This is the reason why its so confusing.

First of all this is not a weird quantum mechanics thing like Schrodingers cat thought experiment that the cat is both dead and alive simultaneously until you look at it. Instead it is actually the fault of how voltage is defined in textbooks.

The more common way of thinking about voltage is that more electrons there are in one place the more negative that point is, connect this area with lots of electrons to an area with few electrons and you get current between them as the charges want to even out.

The way voltage is actually formally defined is "An integral of all forces working on charges along a path between two points". These forces include the electric field that bunched together electrons create, but it also includes the magnetic forces pushing electrons around (Any charged particle is affected by a changing magnetic field). This force is dependan't on where you are in the magnetic field. This results in a different result of the integral of the force and hence a different voltage.

In Dr. Lewins example you get the magnetic EMF adding to the resistors voltage if you go around one way, but subtracting from the resistors voltage if you go around the other way. Hence the integral is different and there is different voltage. Its just how the math ends up working out. The actual electron charge density at the points A and B is always a single well defined number. The voltage you measure by connecting a voltmeter as you shown will measure this electron density. Hence why the voltmeter shows one voltage.

The two different voltages result from Dr. Lewins example could be sort of a incomplete result of the voltage so far, you need to include the rest of the circuit to properly define the voltage. Think of the two voltages as sort of like complex number math, they don't necessarily exist in the real world but they make the math work.
Title: Re: Does Kirchhoff's Law Hold? Disagreeing with a Master
Post by: ogden on December 05, 2018, 08:13:36 am
This is the reason why its so confusing.

It's made to be confusing as presented by Dr.Lewin and his cultists, but it's not. It confuses only those who can't understand that voltmeter wire can be EMF source. The same "path dependent voltage" mind tricks can be played using other EMF sources like chemical batteries or photovoltaic cells, but those are not so confusing because "such batteries cannot be easily hidden in the wires" :)
Title: Re: Does Kirchhoff's Law Hold? Disagreeing with a Master
Post by: electrodacus on December 05, 2018, 08:47:58 am
This is the reason why its so confusing.

First of all this is not a weird quantum mechanics thing like Schrodingers cat thought experiment that the cat is both dead and alive simultaneously until you look at it. Instead it is actually the fault of how voltage is defined in textbooks.

The more common way of thinking about voltage is that more electrons there are in one place the more negative that point is, connect this area with lots of electrons to an area with few electrons and you get current between them as the charges want to even out.

The way voltage is actually formally defined is "An integral of all forces working on charges along a path between two points". These forces include the electric field that bunched together electrons create, but it also includes the magnetic forces pushing electrons around (Any charged particle is affected by a changing magnetic field). This force is dependan't on where you are in the magnetic field. This results in a different result of the integral of the force and hence a different voltage.

In Dr. Lewins example you get the magnetic EMF adding to the resistors voltage if you go around one way, but subtracting from the resistors voltage if you go around the other way. Hence the integral is different and there is different voltage. Its just how the math ends up working out. The actual electron charge density at the points A and B is always a single well defined number. The voltage you measure by connecting a voltmeter as you shown will measure this electron density. Hence why the voltmeter shows one voltage.

The two different voltages result from Dr. Lewins example could be sort of a incomplete result of the voltage so far, you need to include the rest of the circuit to properly define the voltage. Think of the two voltages as sort of like complex number math, they don't necessarily exist in the real world but they make the math work.

I know it has nothing to to with Schrodingers cat :) but it sounds like that is what you are trying to say.
I asked what those two different voltages are, at a fixed moment in time and I do not see that in your replay.
Title: Re: Does Kirchhoff's Law Hold? Disagreeing with a Master
Post by: Berni on December 05, 2018, 12:40:36 pm
Ah okay you want to know the voltages. Given that the total EMF is 1V the result is -0.5V on one side and + 0.5V on the other side. This is because wires show up as having no voltage while the resistors show a voltage according to Ohms law. Since the current trough the loop is identical everywhere this means both resistors have to show the same drop (given they are the same value). Sine the current is flowing upwards in one resistor and down the other you get opposite voltage polarity. So you do get two voltages.

To be honest tho such a result is not very useful when you are trying to understand what the circuit does. Dr Lewins math is not wrong about this (Its wrong when it comes to KVL)

The more useful way of analyzing this circuit is using lumped circuit mesh analysis. This is thought in every EE Highschool and gives more useful results. Here all lengths of wire are modeled as coupled inductors. Since we are interested in the voltage at only one point in time we can calculate the voltage using Faradays law and just pretend there is a battery there. With this, the loop on the left and right of the voltmeter get a 0.5V battery (each loop is half of the whole loop). Because the resistors also have a 0.5V drop the result is 0V for both halves and you get the result of the voltmeter reading 0V (Not +0V and -0V, just a single well defined 0V). If you want to connect the voltmeter to other points you have to recalculate the loop areas accordingly.KVL holds just fine here.

In the experiment he just connects the scope with wires in just the right way to expose the two voltages. If you cosider his scope wires as part of the circuit you will get the same result with regular circuit mesh analysis. The two scopes are connected to two different parts of the circuit since each connects via its own wire. The EMF in those wires added up is the exact amount the readings on the two scopes differs. The actual voltage on the two points stays the same (It's the average of the two scopes when probing the middle like that)
Title: Re: Does Kirchhoff's Law Hold? Disagreeing with a Master
Post by: alanb on December 05, 2018, 01:08:50 pm
I don't know if someone has previously posted this link. Its worth watching.

https://youtu.be/JpVoT101Azg (https://youtu.be/JpVoT101Azg)
Title: Re: Does Kirchhoff's Law Hold? Disagreeing with a Master
Post by: bsfeechannel on December 05, 2018, 01:22:12 pm
I do not disagree with anything you mentioned in you last replay. Correct measurement will read a single value and that is the real 0V no multiple voltages at a single defined point in time.
Of course if you move the measurement point you get a different reading that is normal in any circuit not just this particular case.

Well, in case of the first circuit supplied by batteries with no varying magnetic flux, you agreed with me that all five voltmeters would read the same, i.e. the real position of the voltmeters didn't matter. But let's forget that for the moment.

Because I agree with you that in this particular case, any misplacement of the voltmeter will reflect on the precision of the measuring.

But my question remains unanswered.

How do I know that this imprecision is due to the voltmeter not being exactly in the place it should be, and not due to a resistor mismatch?

You know, resistors change their values, either with temperature, or age, or both, etc.
Title: Re: Does Kirchhoff's Law Hold? Disagreeing with a Master
Post by: ogden on December 05, 2018, 03:23:08 pm
I don't know if someone has previously posted this link. Its worth watching.

Yes indeed it was posted.
Next time please do read the thread or at least use search.
Title: Re: Does Kirchhoff's Law Hold? Disagreeing with a Master
Post by: electrodacus on December 05, 2018, 08:11:24 pm
Ah okay you want to know the voltages. Given that the total EMF is 1V the result is -0.5V on one side and + 0.5V on the other side. This is because wires show up as having no voltage while the resistors show a voltage according to Ohms law. Since the current trough the loop is identical everywhere this means both resistors have to show the same drop (given they are the same value). Sine the current is flowing upwards in one resistor and down the other you get opposite voltage polarity. So you do get two voltages.
You can only read -0.5V and +0.5V if you cut the circuit in half on those two measurement points and then measure each of the half circuits but then that is a completely different circuit and it is no different from a circuit where you have batteries. A real battery has the internal impedance distributed trough the battery is not like there is an ideal source inside and then a separate impedance as it is represented in a diagram.

Wires do have a resistance unless they are supercondutors and my original example specifically had only wires that had a stated resistance was 1Ohm in both examples just that first had a loop made of same type of wire half the ring was 0.5Ohm and the other half 0.5Ohm while the second example had a ring made of two half rings soldered together one with 0.1Ohm and the other with 0.9Ohm resistance.
My examples where meant to get rid of the confusion of having wires and separate resistors but any example will work the same.
Also my example allows you to imagine swiping the virtual voltmeter probes around the ring similar to a potentiometer.

You can use KVL to find out voltage between any two points on those example rings.

If you make an infinitely small cut in any of those two example rings so that there is no current then using the same potentiometer method to measure any two points you will get the same result for both example rings as resistance no longer plays a role.


Well, in case of the first circuit supplied by batteries with no varying magnetic flux, you agreed with me that all five voltmeters would read the same, i.e. the real position of the voltmeters didn't matter. But let's forget that for the moment.
Because I agree with you that in this particular case, any misplacement of the voltmeter will reflect on the precision of the measuring.
But my question remains unanswered.
How do I know that this imprecision is due to the voltmeter not being exactly in the place it should be, and not due to a resistor mismatch?
You know, resistors change their values, either with temperature, or age, or both, etc.

Not sure what sort of point you want to make. The discussion here is that there is only one voltage between the two defined points at a fixed moment in time and that KVL applied correctly will give you the correct result.
Of course real circuits are not perfect and that is why you have tolerances but you add those tolerances in your calculation and your result will have a margin of error proportional with those tolerances.
Title: Re: Does Kirchhoff's Law Hold? Disagreeing with a Master
Post by: Sredni on December 05, 2018, 09:10:08 pm
My examples where meant to get rid of the confusion of having wires and separate resistors but any example will work the same.
[...]
You can use KVL to find out voltage between any two points on those example rings.

Ok, let's take a loop made of two big resistors - physically big - and some copper wire. Let's say the resistors are shaped into an arc spanning 45 degrees. One is 0.1 ohm, the other one is 0.9 ohm. Emf in the loop is 1V.
What is the real and only voltage across the resistors?
What is the real and only voltage across the remaining two portions of wire (say it's copper)?

Edit: added plurals.
Title: Re: Does Kirchhoff's Law Hold? Disagreeing with a Master
Post by: bsfeechannel on December 05, 2018, 10:07:15 pm
Not sure what sort of point you want to make.

I'm just trying to understand what you said in your first post. Of course if you please.

Quote
The discussion here is that there is only one voltage between the two defined points at a fixed moment in time and that KVL applied correctly will give you the correct result.

OK. That's exactly what I am discussing too.

Quote
Of course real circuits are not perfect and that is why you have tolerances but you add those tolerances in your calculation and your result will have a margin of error proportional with those tolerances.

Fine. So let's suppose now that we don't know the value of the resistors. We don't and we can't know ( for some reason, doesn't matter). The "correct" voltage will obviously be measured when we find the exact right place that in our case demands a nanometric precision.

Let's suppose that the voltmeter now indicates something like 0.437582 V. How can we be sure that this is the correct value? Because now, since the resistances and their relation are unknown, we don't know what to expect. How can we know that we are not a couple of hundreds of nanometers off?
Title: Re: Does Kirchhoff's Law Hold? Disagreeing with a Master
Post by: electrodacus on December 05, 2018, 10:23:17 pm
Ok, let's take a loop made of two big resistors - physically big - and some copper wire. Let's say the resistors are shaped into an arc spanning 45 degrees. One is 0.1 ohm, the other one is 0.9 ohm. Emf in the loop is 1V.
What is the real and only voltage across the resistors?
What is the real and only voltage across the remaining two portions of wire (say it's copper)?

The copper wire is also a resistor so you need to provide the resistance of those in order for me to be able to give you the exact answer.
But all you did was split the ring in to 4 equal parts and have 4 resistors two with equal low resistance (the copper wires) and the other two with higher resistance 0.1Ohm and 0.9Ohm
If you assume the wires are made of superconducting material then result is same as in my original experiment 0.4V across the same two opposite points (middle of each copper wire).
In case you measure across the quarter of the ring you will measure say +650mV across the 0.9Ohm resistor and then -150mV across the 0.1Ohm resistor this values will be slightly different if those are not superconductors but copper wires.
The equivalent circuit to make the calculations will be made of 4 voltage sources each equal with 0.25V and each with a series resistance corresponding to whatever that quarter resistance is.

Still there is no point in your slightly more complex example as the voltage on any of the points will be clearly defined and you will never have two different values at a fixed moment in time.
Title: Re: Does Kirchhoff's Law Hold? Disagreeing with a Master
Post by: Sredni on December 05, 2018, 10:41:48 pm
But all you did was split the ring in to 4 equal parts

Not equal (360 - 2x45 = 270; 270/2 = 135 != 45), but that's immaterial.

Quote
and have 4 resistors two with equal low resistance (the copper wires) and the other two with higher resistance 0.1Ohm and 0.9Ohm

Correct. So, let's make the resistors span 90 degrees instead of 45 so we can use your numbers.

Quote
you will measure say +650mV across the 0.9Ohm resistor and then -150mV across the 0.1Ohm resistor

The only and true voltage across the 0.9 ohm resistor is .65 V. Ohm would say there's a current of  .65/.9 = .72 amps
The only and true voltage across the 0.1 ohm resistor is .15 V. Ohm would say there's a current of .15/.1 = 1.5 amps

Now, .65 + .15 = .80 V. EMF is 1V, I suppose you want to put that into 'modified KVL', so did you bungle the calculation or are you assuming there are 0.20 voltage drop on the copper wires? If we assume 0 resistance that would mean ohm would think there's infinite current.
But let's leave ohm alone for the moment because you might want to put some EMF here and there.

Can you put KVL into a formula with numbers? Please make the (possibly corrected, if required) numbers of all the 'true' voltage drops in the loop and show that KVL balance.

Title: Re: Does Kirchhoff's Law Hold? Disagreeing with a Master
Post by: electrodacus on December 05, 2018, 11:01:59 pm
But all you did was split the ring in to 4 equal parts

Not equal (360 - 2x45 = 270; 270/2 = 135 != 45), but that's immaterial.

Quote
and have 4 resistors two with equal low resistance (the copper wires) and the other two with higher resistance 0.1Ohm and 0.9Ohm

Correct. So, let's make the resistors span 90 degrees instead of 45 so we can use your numbers.

Quote
you will measure say +650mV across the 0.9Ohm resistor and then -150mV across the 0.1Ohm resistor

The only and true voltage across the 0.9 ohm resistor is .65 V. Ohm would say there's a current of  .65/.9 = .72 amps
The only and true voltage across the 0.1 ohm resistor is .15 V. Ohm would say there's a current of .15/.1 = 1.5 amps

Now, .65 + .15 = .80 V. EMF is 1V, I suppose you want to put that into 'modified KVL', so did you bungle the calculation or are you assuming there are 0.20 voltage drop on the copper wires? If we assume 0 resistance that would mean ohm would think there's infinite current.
But let's leave ohm alone for the moment because you might want to put some EMF here and there.

Can you put KVL into a formula with numbers? Please make the (possibly corrected, if required) numbers of all the 'true' voltage drops in the loop and show that KVL balance.

Sorry I confused your 45 degree with 90 degree but I guess that still makes the point you wanted to make.

The formula is very simple each quarter of the ring (90 degree) will see a quarter of that 1V so 0.25V
Thus depending on the direction of the current you have +0.9V across the 0.9Ohm resistor but you subtract 0.25V thus +650mV
On the other side of the ring (again my assumption the other resistor is on the other half) you have 0.1V on the resistor and subtract 0.25V so -150mV
The copper wires are treated the same as the resistors so a 0.25V source and whatever resistance those wires have say is 1mOhm for each quarter segment in series
Then total ring resistance will be 1.002Ohm and thus the current will be smaller 0.998A thus the values calculated before will be slightly influenced by this but not much.

If you disagree with this please provide your numbers and how you got to them.
Title: Re: Does Kirchhoff's Law Hold? Disagreeing with a Master
Post by: ogden on December 05, 2018, 11:24:11 pm
Mehdi posted a follow-up video:

Well, thank you for coming with old news (https://www.eevblog.com/forum/chat/does-kirchhoffs-law-hold-disagreeing-with-a-master/msg2010251/#msg2010251). Thread is "boiling" about it few days already ;) Maybe next time read the thread please?
Title: Re: Does Kirchhoff's Law Hold? Disagreeing with a Master
Post by: Sredni on December 05, 2018, 11:37:55 pm
The formula is very simple each quarter of the ring (90 degree) will see a quarter of that 1V so 0.25V
Thus depending on the direction of the current you have +0.9V across the 0.9Ohm resistor but you subtract 0.25V thus +650mV
On the other side of the ring (again my assumption the other resistor is on the other half) you have 0.1V on the resistor and subtract 0.25V so -150mV
The copper wires are treated the same as the resistors so a 0.25V source and whatever resistance those wires have say is 1mOhm for each quarter segment in series

Ok, so you are Mabilde-like.
Hence you believe that the emf is located inside the wire in the form of a distributed voltage generator.
Can we summarize your KVL balance as

net voltage drop across 0.9 ohm + voltage gain across copper + net voltage drop across 0.1 ohm + voltage gain across copper = 0

(-0.9 +0.25) + 0.25 + (-0.1 + 0.25) + 0.25 = 0

Does that mean that there is an electric field inside the copper conductor? (either perfect or as you wish with 1mohm resistance per arc)?

Quote
If you disagree with this please provide your numbers and how you got to them.

I do not believe in a 'true' voltage (actually it's not a belief, the formulas tell me). In this case it all depends on how you measure it - I can get your numbers by suitably partitioning the disk with the probes, or many other values. But I tell you how I can balance Faraday here:
 EMF = 1V, total loop resistance 1 ohm, current 1 amp
 integral of E dl in 0.9 ohm + integral of E dl in 0.1 ohm + nearly nothing in copper = EMF
 0.9 + 0.1 + 0 = 1

(signs come about when you consider the correct conventions)

My ohm's law still work. I had to give up uniqueness of voltage between two points, though.
Most importantly, there is practically no field inside the copper conductor, as predicted by Maxwell's equations.

In your case, well, what is the field inside the copper parts, if you have 0.25 V across each of them?
Title: Re: Does Kirchhoff's Law Hold? Disagreeing with a Master
Post by: ogden on December 05, 2018, 11:56:56 pm
I do not believe in a 'true' voltage (actually it's not a belief, the formulas tell me). In this case it all depends on how you measure it - I can get your numbers by suitably partitioning the disk with the probes, or many other values. But I tell you how I can balance Faraday here:
 EMF = 1V, total loop resistance 1 ohm, current 1 amp
 integral of E dl in 0.9 ohm + integral of E dl in 0.1 ohm + nearly nothing in copper = EMF
 0.9 + 0.1 + 0 = 1

Right. Summary E field in resistors including wire resistance is equal to EMF of the loop, 1V. You shall not ignore EMF in the big, long resistors or we can even name them segments of resistive wire. This is where you add and subtract 0.25V EMF in them accordingly.

Quote
In your case, well, what is the field inside the copper parts, if you have 0.25 V across each of them?

If superconductive, then E field is 0.0V. In this case it is 0.002 V because wire is specified as 0.001 Ohms per segment.

[edit] We don't measure E field or EMF between two points of the circuit. We usually measure potential difference.
Title: Re: Does Kirchhoff's Law Hold? Disagreeing with a Master
Post by: electrodacus on December 06, 2018, 12:18:58 am
Quote
If you disagree with this please provide your numbers and how you got to them.

I do not believe in a 'true' voltage (actually it's not a belief, the formulas tell me). In this case it all depends on how you measure it - I can get your numbers by suitably partitioning the disk with the probes, or many other values. But I tell you how I can balance Faraday here:
 EMF = 1V, total loop resistance 1 ohm, current 1 amp
 integral of E dl in 0.9 ohm + integral of E dl in 0.1 ohm + nearly nothing in copper = EMF
 0.9 + 0.1 + 0 = 1

(signs come about when you consider the correct conventions)

My ohm's law still work. I had to give up uniqueness of voltage between two points, though.
Most importantly, there is practically no field inside the copper conductor, as predicted by Maxwell's equations.

In your case, well, what is the field inside the copper parts, if you have 0.25 V across each of them?

So are you saying that voltage across the 0.9Ohm resistor in the current example is 0.9V ? because that sure is not the case unless the resistor has an infinitely small size not a quarter of the ring (90 degree arc).
The copper parts are no different from the resistors as they are basically all resistors so same rule will apply.

You may want to think on what voltages you will read if the ring was open no current.
Maybe think on the same ring (no matter if the resistors are there or not) and you make a small cut (so no current) and insert there the world smallest voltmeter with almost infinite internal impedance. What do you think the reading will be ?
If you guest approximately 1V you will be right. That will be the case even if the ring was all made of superconducting material so how can you explain the measured 1V (assuming you agree).

So across the 0.9Ohm resistor will be 0.650V and not 0.900V and also as important there will be only one voltage there 0.650V and no multiple voltages at the same point in time as implied in the Lewin experiment.

If superconductive, then E field is 0.0V. In this case it is 0.002 V because wire is specified as 0.001 Ohms per segment.

[edit] We don't measure E field or EMF between two points of the circuit. We usually measure potential difference.

Thanks I'm worse on expressing myself :)  so your replay is shorter and more to the point.
Title: Re: Does Kirchhoff's Law Hold? Disagreeing with a Master
Post by: bsfeechannel on December 06, 2018, 12:22:50 am
I'd like to kindly ask the Kirchhoff experts what tools do Kirchhoff rules give me to calculate the "right" voltage of the loop below. Except for the irregular perimeter, all conditions are the same as for my rectangular loop above.

(https://www.eevblog.com/forum/chat/does-kirchhoffs-law-hold-disagreeing-with-a-master/?action=dlattach;attach=589078;image)

I need to know the exact points where to connect the voltmeter and its precise location. Any reply will be appreciated.
Title: Re: Does Kirchhoff's Law Hold? Disagreeing with a Master
Post by: Sredni on December 06, 2018, 12:36:33 am
So are you saying that voltage across the 0.9Ohm resistor in the current example is 0.9V ?
If I measure from outside, and without crossing the flux varying region, yes.
Note that I did not use 'voltage drop' but the integral of E.dl in my balance.

Quote
because that sure is not the case unless the resistor has an infinitely small size

No, no, that's the case and it is due to the fact that charge accumulate at the resistors end, where there is a gradient in conductivity. This makes the field strong along the resistors, and nearly zero in the copper conductor, just as expected by Maxwell's equations, the equation of continuity and the constitutive relation in the conductor.
Basically, all the EMF falls across the resistors.
It's a matter of charge displacement and the ensuing superposition of field.

Quote
If superconductive, then E field is 0.0V. In this case it is 0.002 V because wire is specified as 0.001 Ohms per segment.

[edit] We don't measure E field or EMF between two points of the circuit. We usually measure potential difference.

Thanks I'm worse on expressing myself :)  so your replay is shorter and more to the point.

So, you measure electric field in volts?
You need to brush up on basic physics.

Please, humor me and try again: what is the field inside the copper conductor and how do you justify - with formulas - that there is a 0.25 volts drop difference across it?
Title: Re: Does Kirchhoff's Law Hold? Disagreeing with a Master
Post by: Sredni on December 06, 2018, 12:40:36 am
I'd like to kindly ask the Kirchhoff experts what tools do Kirchhoff rules give me to calculate the "right" voltage of the loop below. Except for the irregular perimeter, all conditions are the same as for my rectangular loop above.

(https://www.eevblog.com/forum/chat/does-kirchhoffs-law-hold-disagreeing-with-a-master/?action=dlattach;attach=589078;image)

I need to know the exact points where to connect the voltmeter and its precise location. Any reply will be appreciated.

bsfeechannel, you dirty bastard!  :-DD

Title: Re: Does Kirchhoff's Law Hold? Disagreeing with a Master
Post by: ogden on December 06, 2018, 12:55:06 am
So, you measure electric field in volts?
You need to brush up on basic physics.

I did not mention any units at all. You need to check your vision.

Quote
Please, humor me and try again: what is the field inside the copper conductor and how do you justify - with formulas - that there is a 0.25 volts drop across it?

You really need to check your vision. I said there is 0.25 volts EMF and 0.002V drop (in 0.002 ohms conductor). Don't you know Ohms law? BTW you did use it yourself to calculate drop across 0.1 and 0.9 Ohms resistors.
Title: Re: Does Kirchhoff's Law Hold? Disagreeing with a Master
Post by: electrodacus on December 06, 2018, 01:14:16 am
So are you saying that voltage across the 0.9Ohm resistor in the current example is 0.9V ?
If I measure from outside, and without crossing the flux varying region, yes.
Note that I did not use 'voltage drop' but the integral of E.dl in my balance.

V = IR - EMF  so voltage at the terminals of that resistor is as mentioned 0.650V (for the 0.9Ohm resistor) The EMF will only be zero for a infinitely small resistor and as that is not the case in our example where resistor is a quarter of the loops size thus 0.250V
Same of course apply to a section of copper say 0.001Ohm with 0.001V drop thus voltage measured will be -0.249V
I sort of feel like I deal with trolls and I hope that is not the case.
Title: Re: Does Kirchhoff's Law Hold? Disagreeing with a Master
Post by: ogden on December 06, 2018, 01:19:36 am
I sort of feel like I deal with trolls

Me too. What a coincidence :-DD
Title: Re: Does Kirchhoff's Law Hold? Disagreeing with a Master
Post by: Sredni on December 06, 2018, 01:52:37 am
So, you measure electric field in volts?
You need to brush up on basic physics.

I did not mention any units at all. You need to check your vision.

You agreed with ogden, who expressed electric field in volts. You should have read better, before saying that his reply was more to the point.
But, anyway, ok.
I know you specified the emf, but I am asking you about the electric field in the copper.
Sorry I used 'voltage drop', I corrected my self shortly after with 'difference', but you were already answering and quoted my previous sentence.
So, let me ask you again, because this is important:

What is the field inside the copper conductor and how do you justify - with formulas - that there is a (0.25-0.002) volts difference across it?

Assume standard conductivity for copper, say 5.8 10^7 mhos per meter and a copper section of 1 mm in diameter (or any real world value you can attribute to a circuit similar to those shown by Lewin, Mehdi or Mabilde - it's about 10 cm diameter loop, suppose half of it is allocated by the big resistors, but it's not important).

I am asking for the electric field E inside the conductor - to be more precise, the tangential component that contribute to the integral of E.dl .
I can tell you that in my case it would be in the mV/m range.
What value do you get in your case?
Title: Re: Does Kirchhoff's Law Hold? Disagreeing with a Master
Post by: bsfeechannel on December 06, 2018, 02:16:54 am
I sort of feel like I deal with trolls and I hope that is not the case.

What are you afraid of? If your claims are sound, you would instantly know the answer for the questions we've posed. Even for the one Sredni found amusing. If you think that that question is out of proportion,  think again. That could very well be traces on a PCB or wires on any real installation. If your Kirchhoff only works for perfectly rectangular or round loops, with known resistors within tolerances, it is a useless theory.

And here is one more challenge for your Kirchhoff. You know the drill. We need to know the correct way to measure the correct voltage on this loop. The B field is now concentrated on half of the area, but all other conditions hold. Good luck.

(https://www.eevblog.com/forum/chat/does-kirchhoffs-law-hold-disagreeing-with-a-master/?action=dlattach;attach=589189;image)
Title: Re: Does Kirchhoff's Law Hold? Disagreeing with a Master
Post by: HackedFridgeMagnet on December 06, 2018, 02:19:37 am
Have you discovered magnetic monopoles? Amazing.  ;)
Title: Re: Does Kirchhoff's Law Hold? Disagreeing with a Master
Post by: bsfeechannel on December 06, 2018, 03:12:49 am
Have you discovered magnetic monopoles?

No.
Title: Re: Does Kirchhoff's Law Hold? Disagreeing with a Master
Post by: electrodacus on December 06, 2018, 03:30:53 am
I am asking for the electric field E inside the conductor - to be more precise, the tangential component that contribute to the integral of E.dl .
I can tell you that in my case it would be in the mV/m range.
What value do you get in your case?

OK I think I understand what part you do not understand so I will try to find a bit of a different example to explain the problem.


Imagine you have a ring (closed loop) made of superconducting material (set flat on a table) and somewhere above it you have a magnet.
As long as magnet and ring are stationary there will be no EMF so of course also no current in that ring.
Now I drop that magnet that will start to accelerate thus there is a change in magnetic field and there will be a current induced inside the ring that will create a field that opposes the magnetic field from the magnet.
This opposing magnetic field will slow down the magnet and in this particular case it will be slowed down to zero thus you will end up with a levitating magnet.
Now what happened is that energy from the falling magnet induced a current in the coil that created an opposing magnetic field and since coil is made of superconducting material there is no IR loss as R=0 so that energy remains conserved (you can even slowly remove the magnet and that current will remain in that ring).

If we repeat the experiment but instead of the superconducting ring we now use a copper conductor but say is fairly think very low resistance the magnet will still slow down but since there is a bit of IR loss magnet will not levitate and fall all the way down until will hit the table or floor.

Now third experiment you can have a copper cor super conductor ring but it will be an open loop so a very small small cut and you can install a voltmeter there but should be with almost infinite impedance.
In this case magnet will not slow down at all (we ignore the air resistance) but voltmeter will read a voltage that is basically the EMF as IR is zero I=0

EMF = Velocity x B-flux x length  so if you want to increase EMF you will need to increase the speed in this case you need to apply some additional force to the magnet or drop it from a higher place or increase the length of the loop but then you also need to increase the size of the magnet so B-flux remains the same.

We never mentioned in earlier examples the B-flux or the length of our loop as we had enough other information like we established that current trough the loop was 1A and the loop resistance was 1Ohm this was to have small round numbers.

All this discussion is off topic and not sure it helped but the the Lewin claim was that at the same moment in time you can have two completely different voltages on the same exact two points. That is just bad measurement methods as he did not considered the voltmeter leads as part of the circuit.

What are you afraid of? If your claims are sound, you would instantly know the answer for the questions we've posed. Even for the one Sredni found amusing. If you think that that question is out of proportion,  think again. That could very well be traces on a PCB or wires on any real installation. If your Kirchhoff only works for perfectly rectangular or round loops, with known resistors within tolerances, it is a useless theory.


I'm afraid of you being a troll and wasting my time.
Read the replay above as it may be relevant if you are not a troll.
Shape of the loop will make no difference as long as B-flux is uniform but if that is not the case then you will not be able to calculate that with just pen and paper (you may be able to approximate something) but you will need a computer simulation tool to solve that and of course all details to scale.
And even if flux is uniform you will need to know the total length of that loop and the length between the two points you want to make the measurement then calculation is the same as for the simple ring model as shape alone makes no difference.   
Title: Re: Does Kirchhoff's Law Hold? Disagreeing with a Master
Post by: Sredni on December 06, 2018, 03:54:30 am
I am asking for the electric field E inside the conductor - to be more precise, the tangential component that contribute to the integral of E.dl .
I can tell you that in my case it would be in the mV/m range.
What value do you get in your case?

OK I think I understand what part you do not understand so I will try to find a bit of a different example

Can you or can you not compute this electric field?
Our circuit is stationary, nothing is moving.
What is the electric field inside your conductor?

You are not telling because you do not know how to compute it, or because you cannot justify a 0.25 V (yeah, minus 0.002V) voltage difference at its ends?
Because that's the point.
Title: Re: Does Kirchhoff's Law Hold? Disagreeing with a Master
Post by: electrodacus on December 06, 2018, 05:34:48 am
Can you or can you not compute this electric field?
Our circuit is stationary, nothing is moving.
What is the electric field inside your conductor?

You are not telling because you do not know how to compute it, or because you cannot justify a 0.25 V (yeah, minus 0.002V) voltage difference at its ends?
Because that's the point.
This tread is here to discuss about the silly idea that you can read two different voltages measured between the exact same two points at the exact same moment in time.

But good news :).
I came up with an experiment that you can do to understand EMF
It involves either a transformer or if you prefer a brush-less motor/generator (maybe you have one small servo motor around).

Say it is the servo motor as it is more visual and shows a bit more information.
EMF output 100V at 1000RPM and coil resistance 10Ohm
You just connect a voltmeter (you can ignore the 1Mohm) and while the generator spins at 1000RPM you will read 100V
Now you connect at 100Ohm restive load and again spin at 1000RPM what do you think the voltmeter will read now at the output terminal (in parallel with the 100Ohm load) ?
It will of course take more work to spin at 1000RPM when the 100Ohm load is connected but the EMF will be the same 100V

So you have 100V and the close loop will have 100 + 10 = 110Ohm thus current will be 0.909A
IR Voltage drop on the internal 10Ohm coil will be 9.09V thus the voltmeter will read 100V - 9.09V = 90.9V

If you do not believe my calculations are correct then you can experiment. Fortunately the motor and transformer are both shielded so it will not mess with your measurement device.

Edit: This was my last attempt as there is not much else I can do.
I have to go back at creating equipment's that work based in part on what I tried to explain above. They are all open source as I think knowledge should be free and shared, and people should understand how things work.
Title: Re: Does Kirchhoff's Law Hold? Disagreeing with a Master
Post by: Berni on December 06, 2018, 06:23:39 am
Yes that generator example is spot on.

The last two pages of arguing in this thread seam to be all because of different definitions of voltage.

The correct formal definition used by Dr Lewin: Voltage is the integral of all forces on charges along a given path

The common definition used in circuit analysis: Voltage is the difference in charge density between two points

The formal definition is what results in the two voltages because it handles magnetic EMF differently. The EMF is path dependent and this makes the voltage path depend an as well. Because some part of the path is missing this gives multiple results until you close the path using a voltmeter. In this definition a superconductor can't have any voltage across it.

The other definition just focuses on how many electrons there are in that spot. It essentially ignores any magnetic EMF and instead observes the charge separation effect that the magnetic fields cause. Since there can only be a single number for how many electrons are there means this never gives multiple voltages as a result. In this definition superconductors can have a voltage across them if they are a open loop, the magnetic forces bunch up the electrons to one end of the wire and you can measure this voltage with a voltmeter. This is because voltmeters read this definition of voltage. I call this "aparrant voltage"

So I suggest that you make it clear in further discussion what kind of voltage you are talking about.

Edit: Autocorect mistakes
Title: Re: Does Kirchhoff's Law Hold? Disagreeing with a Master
Post by: Sredni on December 06, 2018, 06:46:57 am
Yes that generator example is spot on.

The last two pages of arguing in this thread seam to be all because of different definitions of voltage.

The correct formal definition used by Dr Lewin: Voltage is the integral of all forces on charges along a given path

The common definition used in circuit analysis: Voltage is the difference in charge density between two points

[...]
So I suggest that you make it clear in further discussion what kind of voltage you are talking about.

Just out of curiosity...
Have you ever heard the term "dimensional analysis"?

(Also, I guess you won't tell us what the field inside the conductor is... Kirchhoffians are allergic to electric fields, it appears)

I am not sure I can sort this mess.
Title: Re: Does Kirchhoff's Law Hold? Disagreeing with a Master
Post by: beanflying on December 06, 2018, 07:05:52 am
I sort of feel like I deal with trolls

Me too. What a coincidence :-DD

More like my probing technique is bigger in 'theory' than yours  ;)

Far to much theoretical waffle and going around in circles or in some cases non circular circuits. Given this is a forum for EE's not theoretical Physicists with very average probing technique, get on design the experiment, test it and prove or disprove it and then get it repeated by others.
Title: Re: Does Kirchhoff's Law Hold? Disagreeing with a Master
Post by: ogden on December 06, 2018, 07:35:17 am
You agreed with ogden, who expressed electric field in volts. You should have read better, before saying that his reply was more to the point.

Well, ok. I admit that ogden (*me) did not answer your trolling question "what is the field inside the copper conductor" correctly. Expressing Volts, I missed to mention "integral of E.dl". Hopefully it is resolved now and we can return to the roots of our conversation.

Quote
What is the field inside the copper conductor and how do you justify - with formulas - that there is a (0.25-0.002) volts difference across it?

BTW wire fragment resistance is 0.001 Ohms so it is (0.25-0.001) Volts, not (0.25-0.002).

Inside our conductor there are two E fields: E.induced and E.coloumb. Total electric field E = E.coloumb + E.induced. Coulomb electric field in the wire is opposite the direction of the induced electric field - that's the justification of voltage difference. Potential difference (integral of E.dl) at the ends of that copper conductor you calculate using same formula as for 0.25V chemical battery that has 0.001 Ohm internal resistance and 1A current load. Answer is mentioned already here in this thread.

Quote
Assume standard conductivity for copper, say 5.8 10^7 mhos per meter

Do not introduce new unnecessary conditions.
Title: Re: Does Kirchhoff's Law Hold? Disagreeing with a Master
Post by: Sredni on December 06, 2018, 07:49:42 am
Quote
What is the field inside the copper conductor and how do you justify - with formulas - that there is a (0.25-0.002) volts difference across it?
BTW wire fragment resistance is 0.001 Ohms so it is (0.25-0.001) Volts, not (0.25-0.002).
Ok.
Quote
Inside our conductor there are two E fields: E.induced and E.coloumb. Total electric field E = E.coloumb + E.induced. Coulomb electric field in the wire is opposite the direction of the induced electric field

Ok, this is real progress.

Quote
- that's the justification of voltage difference.

Please clarify.
The field inside the copper conductor is the sum of E.coloumb with E.induced, you said (and I agree). How do you think the copper can tell which is which? The copper sees the net, resulting, field. (and THIS is the point)
What is this sum?

Quote
Potential difference (integral of E.dl) at the ends of that copper conductor you calculate using same formula as for 0.25V chemical battery that has 0.001 Ohm internal resistance and 1A current load. Answer is mentioned already here in this thread.
Please, indulge me. Give me the number in V/m (volts per meter).
Quote
Quote
Assume standard conductivity for copper, say 5.8 10^7 mhos per meter
Do not introduce new unnecessary conditions.
I am looking forward to seeing how you compute the field and how well it fits with the 0.25-0.001 volts difference at the extremes.
Title: Re: Does Kirchhoff's Law Hold? Disagreeing with a Master
Post by: Berni on December 06, 2018, 07:51:55 am
Yes that generator example is spot on.

The last two pages of arguing in this thread seam to be all because of different definitions of voltage.

The correct formal definition used by Dr Lewin: Voltage is the integral of all forces on charges along a given path

The common definition used in circuit analysis: Voltage is the difference in charge density between two points

[...]
So I suggest that you make it clear in further discussion what kind of voltage you are talking about.

Just out of curiosity...
Have you ever heard the term "dimensional analysis"?

(Also, I guess you won't tell us what the field inside the conductor is... Kirchhoffians are allergic to electric fields, it appears)

I am not sure I can sort this mess.

Yes i have its getting units in order in equations.

What so problematic about the field? You get a electric field caused by charge separated electrons that is precisely proportional to the amount of charge separation. There is also an apparent electric field caused by the magnetic field that is exactly the same size and opposite in direction than the electric field  from before (Given this conductor is an open loop or a superconductor)
Title: Re: Does Kirchhoff's Law Hold? Disagreeing with a Master
Post by: Berni on December 06, 2018, 07:53:30 am
I'd like to kindly ask the Kirchhoff experts what tools do Kirchhoff rules give me to calculate the "right" voltage of the loop below. Except for the irregular perimeter, all conditions are the same as for my rectangular loop above.

(https://www.eevblog.com/forum/chat/does-kirchhoffs-law-hold-disagreeing-with-a-master/?action=dlattach;attach=589078;image)

I need to know the exact points where to connect the voltmeter and its precise location. Any reply will be appreciated.

Oh and also i solved your curvy wire example.

Here is where you have to put the voltmeter for it to read 0V

(https://www.eevblog.com/forum/chat/does-kirchhoffs-law-hold-disagreeing-with-a-master/?action=dlattach;attach=589408;image)

Solved using Solidworks Fusion 360 due to it having a convenient area measurement tool.
Title: Re: Does Kirchhoff's Law Hold? Disagreeing with a Master
Post by: ogden on December 06, 2018, 08:31:38 am
The field inside the copper conductor is the sum of E.coloumb with E.induced, you said (and I agree). How do you think the copper can tell which is which?

What are you smoking?

Quote
The copper sees the net, resulting, field. (and THIS is the point)
What is this sum?

Integral of E.dl where E = E.coloumb + E.induced. EMF of wire segment is EMF.total/4 (because segment is 1/4 of loop) = 1/4V and voltage drop due to current is 0.001Ohm*1A = 0.001V. So, this sum is 0.25+(-0.001) Volts. What's the point to ask question so many times?

Quote
Quote
Potential difference (integral of E.dl) at the ends of that copper conductor you calculate using same formula as for 0.25V chemical battery that has 0.001 Ohm internal resistance and 1A current load. Answer is mentioned already here in this thread.
Please, indulge me. Give me the number in V/m (volts per meter).

 :-// With same success you can ask me weight of the wire used in experiment. Before asking V/m, make sure you give enough data to calculate such (https://www.eevblog.com/forum/chat/does-kirchhoffs-law-hold-disagreeing-with-a-master/msg2016091/#msg2016091)  :palm:

[p.s.] Trolls... One is shifting goalposts all over the place, I wonder when he will touch chemical composition of the copper wire? Another is stuck into self-inflicted mental loop of proving that Kirchoff's rules cannot be used in place of Maxwells equations. Fun!

[edit] Added reply to Q "What is this sum?"
Title: Re: Does Kirchhoff's Law Hold? Disagreeing with a Master
Post by: bsfeechannel on December 06, 2018, 12:59:08 pm
Oh and also i solved your curvy wire example.

Here is where you have to put the voltmeter for it to read 0V

Solved using Solidworks Fusion 360 due to it having a convenient area measurement tool.

Wonderful. Much appreciated. Now we need to measure the voltage indicated by the calculations so as to confirm that they are right. But, alas, in the real circuit there is a physical obstruction, that in no way affects the magnetic field. This obstruction goes all the way with the field while it is perpendicular to the surface.

How can we measure measure that voltage? Thanks in advance for your kind reply.

(https://www.eevblog.com/forum/chat/does-kirchhoffs-law-hold-disagreeing-with-a-master/?action=dlattach;attach=589588;image)
Title: Re: Does Kirchhoff's Law Hold? Disagreeing with a Master
Post by: bsfeechannel on December 06, 2018, 01:18:09 pm
I sort of feel like I deal with trolls

Me too. What a coincidence :-DD

More like my probing technique is bigger in 'theory' than yours  ;)

Far to much theoretical waffle and going around in circles or in some cases non circular circuits. Given this is a forum for EE's not theoretical Physicists with very average probing technique, get on design the experiment, test it and prove or disprove it and then get it repeated by others.

By the choice of your words you sense that there is probably something wrong with your "probing technique". It's not sponsored by any electronics engineering fundamentals which pretty much describes tried and true experimental phenomena along the past two centuries up to this day. You only rely on a couple of 10 min or so videos on the internet without even questioning their content. Any serious trade like ours upon which the lives of people depend deserves a little more rigor.
Title: Re: Does Kirchhoff's Law Hold? Disagreeing with a Master
Post by: Berni on December 06, 2018, 02:25:59 pm
Oh and also i solved your curvy wire example.

Here is where you have to put the voltmeter for it to read 0V

Solved using Solidworks Fusion 360 due to it having a convenient area measurement tool.

Wonderful. Much appreciated. Now we need to measure the voltage indicated by the calculations so as to confirm that they are right. But, alas, in the real circuit there is a physical obstruction, that in no way affects the magnetic field. This obstruction goes all the way with the field while it is perpendicular to the surface.

How can we measure measure that voltage? Thanks in advance for your kind reply.

(https://www.eevblog.com/forum/chat/does-kirchhoffs-law-hold-disagreeing-with-a-master/?action=dlattach;attach=589588;image)

That's just an engineering problem at this point.

1) Make a length of wire that would fit across those two points as if the pesky barrier was not there
2) Make a rigid wire structure that goes around the barrier as needed and connects to a voltmeter on the other end
3) Make another copy of the structure from 2, but short it using the piece of wire from 1
4) Place the structure from 2 onto the circuit to tap the voltage and place the structure from 3 anywhere near by
5) Subtract the readings of the voltmeters.

The compensation structure from 3 can be used multiple times to ensure the field is indeed uniform all around so that we know the placement of the structure has shown valid readings.

Alternatives are to just calculate the voltage of the compensation structure if you already know the exact properties of the field, or in that case if you know the properties of the field and the path of the wire you want to measure you can just apply Faradays law to the whole thing.

Remember im not trying to disprove anything about Faraday or Maxwell. Just saying that Kirchhoffs laws are convenient to use in some cases (And they do work when used correctly).
Title: Re: Does Kirchhoff's Law Hold? Disagreeing with a Master
Post by: ogden on December 06, 2018, 02:58:30 pm
Wonderful. Much appreciated. Now we need to measure the voltage indicated by the calculations so as to confirm that they are right. But, alas, in the real circuit there is a physical obstruction, that in no way affects the magnetic field. This obstruction goes all the way with the field while it is perpendicular to the surface.

How can we measure measure that voltage? Thanks in advance for your kind reply.

Just to be sure rules are known - both resistors are equal and positioned symmetrically against midpoint of the the loop, right?

Sure - due to obstruction and EMF we can't measure potential difference between given points directly. First we measure EMF induced in the voltmeter test leads - by shorting them on far side and routing them around the obstruction, making sure our test lead loop is symmetrical and centered against outer loop. Then we can leave one lead where it is (connected to far side midpoint of the outer loop) and bring another to near midpoint, measure voltage across the outer loop noting that it is impaired by EMF in one of test leads. Then just either add or subtract 1/2 of test leads EMF voltage from measurement - depending on which test lead receives EMF and direction of magnetic field. What's the point of all this?

[p.s.] Let's name shape of this loop or maybe even whole experiment as "trail of the troll"...
Title: Re: Does Kirchhoff's Law Hold? Disagreeing with a Master
Post by: In Vacuo Veritas on December 06, 2018, 03:21:47 pm
Does Ohm's Law still work? I've got this LED I have to turn on and I need to know which side of the LED to put the resistor...
Title: Re: Does Kirchhoff's Law Hold? Disagreeing with a Master
Post by: ogden on December 06, 2018, 03:41:52 pm
Does Ohm's Law still work? I've got this LED I have to turn on and I need to know which side of the LED to put the resistor...

LOL. You can rest assured - complex laws do not invalidate basic laws of physics and nature easily.

Thou in case you want detailed answer - you better start new thread in "n00bs" section ;)
Title: Re: Does Kirchhoff's Law Hold? Disagreeing with a Master
Post by: Sredni on December 06, 2018, 04:42:32 pm
The field inside the copper conductor is the sum of E.coloumb with E.induced, you said (and I agree). How do you think the copper can tell which is which?
What are you smoking?

ogden, I promised not to interact with you, but you are making this promise very hard to keep. If you interfere with my exchanges with other posters I have to reply to you as well.

Quote from: ogden
Integral of E.dl where E = E.coloumb + E.induced. EMF of wire segment is EMF.total/4 (because segment is 1/4 of loop) = 1/4V and voltage drop due to current is 0.001Ohm*1A = 0.001V. So, this sum is 0.25+(-0.001) Volts. What's the point to ask question so many times?

Because you guys keep telling me the voltage. I want to know the electric field.

Quote from: Sredni
Please, indulge me. Give me the number in V/m (volts per meter).
Quote from: ogden
:-// With same success you can ask me weight of the wire used in experiment. Before asking V/m, make sure you give enough data to calculate such (https://www.eevblog.com/forum/chat/does-kirchhoffs-law-hold-disagreeing-with-a-master/msg2016091/#msg2016091)  :palm:

I gave all the data, and also allowed for some freedom in choosing the parameters. Go back and read what I wrote in Reply #379, yesterday at 12:52:37 pm

So, let me ask you again, because this is important:
What is the field inside the copper conductor and how do you justify - with formulas - that there is a (0.25-0.002) volts difference across it?

Assume standard conductivity for copper, say 5.8 10^7 mhos per meter and a copper section of 1 mm in diameter (or any real world value you can attribute to a circuit similar to those shown by Lewin, Mehdi or Mabilde - it's about 10 cm diameter loop, suppose half of it is allocated by the big resistors, but it's not important).

I am asking for the electric field E inside the conductor - to be more precise, the tangential component that contribute to the integral of E.dl .
I can tell you that in my case it would be in the mV/m range.
What value do you get in your case?

And still, no answer.


Edit: typo, added slant and bold.
Title: Re: Does Kirchhoff's Law Hold? Disagreeing with a Master
Post by: ogden on December 06, 2018, 05:05:12 pm
Quote from: ogden
Integral of E.dl where E = E.coloumb + E.induced. EMF of wire segment is EMF.total/4 (because segment is 1/4 of loop) = 1/4V and voltage drop due to current is 0.001Ohm*1A = 0.001V. So, this sum is 0.25+(-0.001) Volts. What's the point to ask question so many times?
Because you guys keep telling me the voltage. I want to know the electric field.

I said (E = E.coloumb + E.induced). Are you satisfied now?

Quote
Quote from: Sredni
Assume standard conductivity for copper, say 5.8 10^7 mhos per meter and a copper section of 1 mm in diameter (or any real world value you can attribute to a circuit similar to those shown by Lewin, Mehdi or Mabilde - it's about 10 cm diameter loop, suppose half of it is allocated by the big resistors, but it's not important).

I am asking for the electric field E inside the conductor - to be more precise, the tangential component that contribute to the integral of E.dl .
I can tell you that in my case it would be in the mV/m range.
What value do you get in your case?

And still, no answer.

You can either provide solution yourself and tell what you want to say with it or stick that tangential component where it hurts. I do not see the point of solving your tasks. "Trail of the troll" was at least funny.
Title: Re: Does Kirchhoff's Law Hold? Disagreeing with a Master
Post by: Sredni on December 06, 2018, 05:16:58 pm
Because you guys keep telling me the voltage. I want to know the electric field.
I said (E = E.coloumb + E.induced). Are you satisfied now?

No, I want to know the value in V/m (or in J/C if you prefer).

Quote
You can either provide solution yourself and tell what you want to say with it or stick that tangential component where it hurts. I do not see the point of solving your tasks. "Trail of the troll" was at least funny.

Ok, you have no idea on how to compute the electric field inside a conductor. It's not a crime. Maybe all that facepalming has interfered with your mental processes but, fine.
Any other Kirchhoffian who believes that the 'real' voltage across the 0.9 ohm resistor is 0.65 V and the real voltage across one of the two arcs of copper is 0.25-0.001 V care to tell us what the electric field is inside said copper?
Title: Re: Does Kirchhoff's Law Hold? Disagreeing with a Master
Post by: SiliconWizard on December 06, 2018, 05:32:03 pm
Does Ohm's Law still work? I've got this LED I have to turn on and I need to know which side of the LED to put the resistor...

This question is actually a lot more relevant than it appears here (due to its ironic nature), as the basic Ohm's law links voltage, resistance and current. Now what is voltage again? ;D
Incidentally, Kirchhoff (not him again!) reformulated Ohm's law as: J = sigma.E
So, may be on to something.
Title: Re: Does Kirchhoff's Law Hold? Disagreeing with a Master
Post by: In Vacuo Veritas on December 06, 2018, 05:45:49 pm
Does Ohm's Law still work? I've got this LED I have to turn on and I need to know which side of the LED to put the resistor...

This question is actually a lot more relevant than it appears here (due to its ironic nature),

Thanks, at least someone appreciates my work here.
Title: Re: Does Kirchhoff's Law Hold? Disagreeing with a Master
Post by: ogden on December 06, 2018, 05:58:07 pm
Ok, you have no idea on how to compute the electric field inside a conductor. It's not a crime. Maybe all that facepalming has interfered with your mental processes but, fine.
Any other Kirchhoffian who believes that the 'real' voltage across the 0.9 ohm resistor is 0.65 V and the real voltage across one of the two arcs of copper is 0.25-0.001 V care to tell us what the electric field is inside said copper?

Yes, I have to dig into it to solve it. So what. Original Dr.Lewins experiment assumed that conductors have no resistance, so no coloumb E-field. Now you are modifying it to prove what exactly?- That your debate opponents cannot calculate something during time they are willing to spend, so this is proof that you are right? BTW this is typical tactic of internet trolls - derail discussion into personal attacks.

Better tell your E-field number and make your point. Educate Kirchoff believers, don't let them compute what you can do in a snap.
Title: Re: Does Kirchhoff's Law Hold? Disagreeing with a Master
Post by: Berni on December 06, 2018, 06:03:46 pm
Does Ohm's Law still work? I've got this LED I have to turn on and I need to know which side of the LED to put the resistor...

This question is actually a lot more relevant than it appears here (due to its ironic nature), as the basic Ohm's law links voltage, resistance and current. Now what is voltage again? ;D
Incidentally, Kirchhoff (not him again!) reformulated Ohm's law as: J = sigma.E
So, may be on to something.

Or maybe its just using metric volts instead of imperial volts. We already crashed a probe into mars because of this shit. ;D
(You do have a valid point there tho)
Title: Re: Does Kirchhoff's Law Hold? Disagreeing with a Master
Post by: Sredni on December 06, 2018, 06:33:36 pm
Ok, you have no idea on how to compute the electric field inside a conductor. It's not a crime. Maybe all that facepalming has interfered with your mental processes but, fine.
Any other Kirchhoffian who believes that the 'real' voltage across the 0.9 ohm resistor is 0.65 V and the real voltage across one of the two arcs of copper is 0.25-0.001 V care to tell us what the electric field is inside said copper?

Yes, I have to dig into it to solve it. So what. Original Dr.Lewins experiment assumed that conductors have no resistance, so no coloumb E-field.


No, so no resultant E field at all.

Quote
Now you are modifying it to prove what exactly?- That your debate opponents cannot calculate something during time they are willing to spend, so this is proof that you are right? BTW this is typical tactic of internet trolls - derail discussion into personal attacks.

Coming from the person who constantly facepalms, calls other posters trolls and asks what they smoke, this is hilarious.
I've been extremely restrained with you, but you are like one of those small dogs that keep jumping and barking when their owners are trying to have a conversation.

Quote
Better tell your E-field number and make your point. Educate Kirchoff believers, don't let them compute what you can do in a snap.

I already told you my number. It's in the mV/m range. With 1 amp in a 1mm diameter copper wire it's about 30 mV/m; with 10 mA as in the original Lewin experiment is about 30 uV/m. And it is perfectly consistent with the constitutive equation j = sigma E.

Also, I already told what my aim is in one of my previous post: to show that since there could not be a significant electric field inside the copper (it is zero in a perfect conductor) it is nonsense thinking that you can still have an induced field capable of producing a 0.25V voltage at the extremes (much more in the case of Lewin's experiment, since there the resistors were much smaller).
The induced E field inside the conductor is compensated by the field caused by redistribution of charge. All the resultant electric field is located in the resistor region, with nothing left in the conductor.

This is the point.

Now, dig in and try to compute that field. Then try to explain why you have to give up on j = sigma E as well.

Edit: added "resultant electric", and some plurals I had missed
Title: Re: Does Kirchhoff's Law Hold? Disagreeing with a Master
Post by: electrodacus on December 06, 2018, 06:42:08 pm
Ok, you have no idea on how to compute the electric field inside a conductor. It's not a crime. Maybe all that facepalming has interfered with your mental processes but, fine.
Any other Kirchhoffian who believes that the 'real' voltage across the 0.9 ohm resistor is 0.65 V and the real voltage across one of the two arcs of copper is 0.25-0.001 V care to tell us what the electric field is inside said copper?

Have you read my replay with the servo motor or transformer experiment.  Please do that experiment as there your measurement device will be outside of the changing magnetic field and so you will get the correct result.
Even just reading that proposed experiment you should know that those will be the results that you will get and that they use the same logic that resulted in 0.65V on the 0.9Ohm resistor and 0.250-0.001V on the two 90 degree copper arcs situated in an uniform magnetic flux.
A piece of wire is a low value resistor so you introducing that in my simplified example did not helped with anything other than complicating the example.
Maybe is worth noting that if you shield the 0.9Ohm resistor the EMF at the same point in time will drop from 1V to 0.75V and since it is uniform it will be distributed equally on the other 3 quarters of the circuit. I leave it to you to calculate what will be the voltage on the 0.9Ohm in this case.
Don't care what you use to solve this as long as the result is correct.
Title: Re: Does Kirchhoff's Law Hold? Disagreeing with a Master
Post by: Sredni on December 06, 2018, 06:58:14 pm
Have you read my replay with the servo motor or transformer experiment.

Sorry, I am not allowing mutatio controversiae. Let's stick to the ring and keep it as simple as possible, because heaven knows what excuses you people could come up with with slightly more complex systems.

Quote
Please do that experiment as there your measurement device will be outside of the changing magnetic field and so you will get the correct result.

So, when the voltmeter is outside the loop in the two resistor experiment, and measures 0.9V across the 0.9ohm resistors, that is the correct result?
Outside the loop there is no changing magnetic field.
Your rule does not apply here?

And, just to be clear, I know that if you measure the voltage on the ring following radial path as Mabilde did, the  voltmeter will read the value you say. What I am trying to tell yuo is that such value is perfectly compatible with the application of Faraday's law and does not require for any resultant E-field inside the conductor.
On the other hand, you need that field to justify 0.25 volts across 7-8 cm of copper wire.

Title: Re: Does Kirchhoff's Law Hold? Disagreeing with a Master
Post by: ogden on December 06, 2018, 08:01:27 pm
Also, I already told what my aim is in one of my previous post: to show that since there could not be a significant electric field inside the copper (it is zero in a perfect conductor) it is nonsense thinking that you can still have an induced field capable of producing a 0.25V voltage at the extremes

Wait... What you just said? - That in ideal conductor can't be EMF (induced field)? I am speechless to be honest. We are back to square one where you stop posting and go watch videos of Dr.Lewin. He is brilliant teacher BTW.

Quote
The induced E field inside the conductor is compensated by the field caused by redistribution of charge.

This is exactly what I was telling multiple times already, E = E.coloumb + E.induced.
Title: Re: Does Kirchhoff's Law Hold? Disagreeing with a Master
Post by: Berni on December 06, 2018, 08:18:36 pm
Also, I already told what my aim is in one of my previous post: to show that since there could not be a significant electric field inside the copper (it is zero in a perfect conductor) it is nonsense thinking that you can still have an induced field capable of producing a 0.25V voltage at the extremes

Wait... What you just said? - That in ideal conductor can't be EMF (induced field)? I am speechless to be honest. We are back to square one where you stop posting and go watch videos of Dr.Lewin. He is brilliant teacher BTW.

Quote
The induced E field inside the conductor is compensated by the field caused by redistribution of charge.

This is exactly what I was telling multiple times already, E = E.coloumb + E.induced.

Yes i noticed the two being thrown into the same basket and considered as one thing all too often in this thread. The real electric field caused by charge separated electrons is a different thing that the apparent electric field that the electrons feel due to the magnetic interaction with them. The two have very different underlying mechanisms behind them.

Its especially important because voltmeters can only see the first kind of "electron pusher".

Oh and user electrodacus seams to be using the electron charge density kind of voltage everywhere. His claims are correct when you consider that. Tho i think he should look into what the other magnetic EMF part of the voltage looks like to see the whole picture in this thread.
Title: Re: Does Kirchhoff's Law Hold? Disagreeing with a Master
Post by: ogden on December 06, 2018, 10:35:54 pm
Also, I already told what my aim is in one of my previous post: to show that since there could not be a significant electric field inside the copper (it is zero in a perfect conductor) it is nonsense thinking that you can still have an induced field capable of producing a 0.25V voltage at the extremes

Wait... What you just said? - That in ideal conductor can't be EMF (induced field)?

No, I said that in a perfect conductor there cannot be a non-zero resultant E field. While in copper you get a small field compatible with j = sigma E.

This is not what you said. You correctly say that E.coloumb in ideal conductor is zero, then you imply that it means that it is nonsense to have 0.25V induced field (EMF). Read your own words for god's sake: "since there could not be a significant electric field inside the copper (it is zero in a perfect conductor) it is nonsense thinking that you can still have an induced field capable of producing a 0.25V voltage at the extremes".

Quote
And still you can't see.

What I shall see? Enlighten me.

Quote
What is E, then?

E is sum of two fields, E.induced + E.coloumb - you can do the math and calculate (do integral over E.dl) potential difference at the ends of wire segment that is subject to both E-fields. This part is explained by Lewin himself BTW.
Title: Re: Does Kirchhoff's Law Hold? Disagreeing with a Master
Post by: Sredni on December 06, 2018, 10:56:30 pm
Also, I already told what my aim is in one of my previous post: to show that since there could not be a significant electric field inside the copper (it is zero in a perfect conductor) it is nonsense thinking that you can still have an induced field capable of producing a 0.25V voltage at the extremes

Wait... What you just said? - That in ideal conductor can't be EMF (induced field)?

No, I said that in a perfect conductor there cannot be a non-zero resultant E field. While in copper you get a small field compatible with j = sigma E.

This is not what you said. You correctly say that E.coloumb in ideal conductor is zero, then you imply that it means that it is nonsense to have 0.25V induced field (EMF). Read your own words for god's sake: "since there could not be a significant electric field inside the copper (it is zero in a perfect conductor) it is nonsense thinking that you can still have an induced field capable of producing a 0.25V voltage at the extremes".

When there is the primary coil only, say in vacuum, you can find the induced E field in all space. It is directed in circles and has a module that grows with the distance r from the center inside the coil, while it decreases as 1/r outside of it.
Ok, there you can see the field, in vacuum.

Now you put your copper loop with the two resistors.
Do you still think that inside the copper there will be the same induced field? No, charges will be displaced, producing a coloumbian field that will compensate this field, actually erasing it inside the conductor. All that remains is zero in a perfect conductor and that tiny little field compatible with j = sigma E in a real conductor.
Your induced field is no mas. Obliterated by the field produced by the displaced charges - that will accumulate at the resistors ends and in general wherever there are gradients in conductivity and permeability.

So I confirm my words: "no significant electric field inside the copper (it is zero in a perfect conductor)" and that if you think that you can still have your unaltered induced field inside the copper you are thinking nonsense.
Just like thinking that you can have a radially direct E-field inside a conductor placed nearby a point charge.

Quote
E is sum of two fields, E.induced + E.coloumb - you can do the math and calculate (do integral over E.dl) potential difference at the ends of wire segment that is subject to both E-fields. This part is explained by Lewin himself BTW.

So, after all this posts you still have to answer what is the value of the resulting E field in V/m in copper.


Edit: added the gradient eps and gradient sigma parts.
Title: Re: Does Kirchhoff's Law Hold? Disagreeing with a Master
Post by: ogden on December 06, 2018, 11:40:45 pm
Do you still think that inside the copper there will be the same induced field?
if you think that you can still have your unaltered induced field inside the copper you are thinking nonsense.

I don't think so and never did. You just discovered EMF? We are long over this part, kid.

Quote
So, after all this posts you still have to answer what is the value of the resulting E field in V/m in copper.

If you want value of field *inside* copper segment, then this is all you get: (I*R)/length. Better just watch Dr.Lewin's famous video where he explains those questions
Title: Re: Does Kirchhoff's Law Hold? Disagreeing with a Master
Post by: electrodacus on December 06, 2018, 11:52:41 pm
So, after all this posts you still have to answer what is the value of the resulting E field in V/m in copper.

You always ask silly questions that have no relevance to the problem. Showing that you do not understand the mechanisms behind all this.

Original problem had this as data

A closed loop made of two different resistors connected with some pieces of wire (nit that relevant as wire is also a resistor with lower value).
EMF = 1V (I think this was the same in my example and in Lewin example).
Resistor values 0.9Ohm and 0.1Ohm in my example not sure but I think it was 1000x more in Lewin example (not relevant).

Noting else was given and there is no need for anything else to find out what the voltage on those two symmetrically opposed points will be.

So let me explain to you what a 1V EMF means as you do not seem to understand (likely my fault as my examples should have been very clear).

You can have a wire open circuit no matter what shape how long or what type of conductor as long as you mention that EMF is 1V then you will measure 1V across that wire (this is a snapshot in time as of course there will be a variable magnetic field that generates this). There is also the assumption that magnetic field is uniform across the loop meaning that you will read 0.5V if you measure two points at half the wire length between them anywhere on the wire.

Say you close this wire in a loop (again not relevant what shape the loop has and is the exact same moment in time thus EMF=1V) then there will be a current generated in this loop that will be result of EMF divided by loop resistance.
Keep in mind that this is the case because the EMF = 1V was given as input for the closed loop meaning that the magnetic field was strong enough to generate that 1V EMF no matter how much current went trough the closed loop.

So in my example with a total loop resistance of around 1Ohm current was 1A and I think Lewin experiment had 1mA but there is no difference for the explanation.

Now if the loop was all made of copper wire with 1Ohm resistance it seems you do not have a problem with the fact that 1A will travel trough this wire as EMF is defined as 1V (am I correct in assuming that).

Your problem seems to be understanding why if say a quarter of this loop has 0.999Ohm and the remaining 3 quarters have 0.001Ohm

a) If this new loop is open circuit you will agree that you will read 1V between ends and not only that but you will read 0.5V on any half part of the loop.
b) If I cut this loop in two open parts one quarter size 0.999Ohm it will read 0.25V and the other 3 quarter size 0.001Ohm will read 0.75V
c) Each of this two separately can be equivalent with a battery with the respective internal resistance

Now can you understand why is simple to calculate the voltage in any two points on this close loop made of basically two resistors (you can add 10 different resistors in the loop if you want).
And you do not need to know anything else to calculate the voltage between any two points.

All your unrelated questions are useless and just show you do not understand what EMF is.
Just hope this made it clear to you and others as I love sharing and acquiring knowledge.  If I'm not able to explain it so anyone can understand it means I do not understand it.
Title: Re: Does Kirchhoff's Law Hold? Disagreeing with a Master
Post by: beanflying on December 07, 2018, 12:40:35 am

By the choice of your words you sense that there is probably something wrong with your "probing technique". It's not sponsored by any electronics engineering fundamentals which pretty much describes tried and true experimental phenomena along the past two centuries up to this day. You only rely on a couple of 10 min or so videos on the internet without even questioning their content. Any serious trade like ours upon which the lives of people depend deserves a little more rigor.

There is no real rigor left in this thread what you have all descended into is tit for tat point making among 5 or 6 of you and then arguing who has the biggest probe er I mean best resolution to the point then disagreeing on that point and going around again in a circle!

Attacking the person for having a different opinion is not ever going to solve anything! Attack the technicalities just might get a resolution but in this threads case I doubt it very very much.

'Any serious trade like ours upon which the lives of people depend deserves a little more rigor.' Yep because projects like the Manhattan one 'saved lives' with Physics and Engineering. You have used this same statement a few times and while it can be true it is also a logical falsehood.
Title: Re: Does Kirchhoff's Law Hold? Disagreeing with a Master
Post by: Sredni on December 07, 2018, 12:52:56 am
There is no real rigor left in this thread

What kind of rigor can you expect from people who wriggle like eels and refuse to compute the field in their circuit?
I have seen lots and lots of words to go around that simple question. And the reason is that they will end up with inconsistent results.
They believe they can have .25 V across a piece of copper 7cm long, 1mm diameter with nearly zero field inside. Or non-negligible field (much much higher than that allowed by the constitutive equation) inside a good conductor. What rigor can you expect?

This is why Lewin stopped answering questions about this matter. Flat-earthers always come up with new excuses, no matter what.
Now there is 'apparent electric field'.
Title: Re: Does Kirchhoff's Law Hold? Disagreeing with a Master
Post by: electrodacus on December 07, 2018, 01:06:19 am
There is no real rigor left in this thread

What kind of rigor can you expect from people who wriggle like eels and refuse to compute the field in their circuit?
I have seen lots and lots of words to go around that simple question. And the reason is that they will end up with inconsistent results.
They believe they can have .25 V across a piece of copper 7cm long, 1mm diameter with nearly zero field inside. Or non-negligible field (much much higher than that allowed by the constitutive equation) inside a good conductor. What rigor can you expect?

This is why Lewin stopped answering questions about this matter. Flat-earthers always come up with new excuses, no matter what.
Now there is 'apparent electric field'.

Have you even read my last replay?  Do you still do not understand the problem and the parameters given ?
Nothing else is needed to solve the problem and nothing else was specified in my examples or Lewin's
There was no mention anywhere about the loop size, diameter of the wires or magnetic field as they are not required to solve the problem and you can have an infinite combination of those to get the spec EMF but just the EMF was needed.
Please make sure you read that else it makes no sense for me to replay to you.
Title: Re: Does Kirchhoff's Law Hold? Disagreeing with a Master
Post by: ogden on December 07, 2018, 01:13:03 am
Flat-earthers always come up with new excuses, no matter what.

When they are out of arguments in existing discussion, they invent new useless challenges - just like you.
Title: Re: Does Kirchhoff's Law Hold? Disagreeing with a Master
Post by: Sredni on December 07, 2018, 01:18:37 am
Electrodacus
I read it and it shows that you have a high school student mentality. You see it as an exercises with 'givens' from the prof. and you have to find a value that coincides with the prof. result.
But getting the right number does not mean you are getting the theory behind it. In fact, Mabilde is getting the same number I would expect based on Faraday, the nearly zero field inside the copper and no little generators dispersed along the wire.

What I am doing here is trying to point out inconsistencies in your (erroneous) view of the phenomena.
If you compute the darn field inside your copper conductor you will find that you either have to give up j = sigma E or you have to renounce having that 0.25V located there.
And I have to resort to field theory (EDIT: to do that, and that should not be a problem: my view is perfectly consistent with that, can you say the same for yours?).

I can show you exactly how the charge distribution is affected by gradients in conductivity and in permeability, but that would require vector calculus and if you do not understand as basic a concept as the superposition of fields, what hope is there that you will understand that?
Stop making up other examples where you can find the right number. Focus on the underlying theory and principles and show me you do not find inconsistencies. My view has no inconsistency whatsoever: the field inside the copper is perfectly abiding j = sigma E. Can you say the same?

Also, did you answer my question about the voltmeter measuring 0.9V from outside the loop? No, you didn't.
You keep making up new examples to avoid facing the inconsistencies of your view.
While I have always been focused on the two resistor loop of Romer-Lewin.
Try to do the same.
Title: Re: Does Kirchhoff's Law Hold? Disagreeing with a Master
Post by: bsfeechannel on December 07, 2018, 01:25:04 am
When they are out of arguments in existing discussion, they invent new useless challenges - just like you.

A challenge is what distinguishes a professional from a wannabe. The versed from the amateur. The authentic from the impostor.
Title: Re: Does Kirchhoff's Law Hold? Disagreeing with a Master
Post by: ogden on December 07, 2018, 01:47:57 am
A challenge is what distinguishes a professional from a wannabe. The versed from the amateur. The authentic from the impostor.

LOL. Who would say so. BTW you also came with useless challenge. Now both of you can challenge each other until it hurts. This thread became :horse: - like both of you wanted.
Title: Re: Does Kirchhoff's Law Hold? Disagreeing with a Master
Post by: Berni on December 07, 2018, 12:14:28 pm

Quote from: Berni
Yes i noticed the two being thrown into the same basket and considered as one thing all too often in this thread.

Yes, and you should ask yourself why.

Quote
The real electric field caused by charge separated electrons is a different thing that the apparent electric field that the electrons feel due to the magnetic interaction with them. The two have very different underlying mechanisms behind them.

Do you think the copper can tell the difference?
Or will it just experience the superposition of both fields?
I gave up my hopes on ogden, but you might make it.

Here's a hint, from electrostatics.
The field produced from a point charge is radially directed and goes as 1/r^2.
Now put a piece of copper near it.
Will the field inside the piece of copper be still radially directed from the source?
Or maybe, the free charges in the conductor will distribuite themselves in such a way to compensate for that radial field, so that the resulting field will be zero inside the conductor?

Does it matter the underlying mechanism that produced the various contributions to the total field?
Where is it written that superposition of electric fields only works for... 'same mechanism origin' fields only?


Copper doesn't really care about the difference, it just enjoys its electrons roaming about wherever they want as metals tend to do.

And yes a similar effect happens with external electrostatic fields, but with the difference is that these fields act as a conservative field. Path does not matter with them and as such they are not capable of pushing current around a circuit loop. Due to this the wires used to connect the voltmeter subtract this effect back out and once again cause the voltmeter incapable of detecting it even tho it is essentially measuring the charge density (across its terminals, not across the ends of its probe wire)

One could reformulate Dr. Lewins two resistor experiment to work with electrostatics by simply removing the solenoid and placing the circuit between the plates of a large capacitor, then ramping the voltage across this capacitors terminals. You will again see voltage and currents in the circuit, so circuit mesh analysis will need a parasitic capacitor added in to work right, but both scopes would show the same voltage so it wouldn't be as cool of a demo.

So how does a voltmeter tell the difference if it only shows the field caused by electron density and not the magnetic EMF? (Hint: It does show some EMF too but not where you would typically want)

If voltmeters treat the two separately, why should we treat them as the same thing? We are trying to calculate what the voltmeter would show after all.

I'm not trying to pick sides here, or say anything negative about anyone. To me it seams that most people in this thread are not saying anything wrong for the most part, but the disagreement seams to stem from using a slightly different definition of things and more rarely a bit from just having a different thought process about this thing.
Title: Re: Does Kirchhoff's Law Hold? Disagreeing with a Master
Post by: bsfeechannel on December 07, 2018, 04:05:30 pm
That's just an engineering problem at this point.

1) Make a length of wire that would fit across those two points as if the pesky barrier was not there
2) Make a rigid wire structure that goes around the barrier as needed and connects to a voltmeter on the other end
3) Make another copy of the structure from 2, but short it using the piece of wire from 1
4) Place the structure from 2 onto the circuit to tap the voltage and place the structure from 3 anywhere near by
5) Subtract the readings of the voltmeters.

The compensation structure from 3 can be used multiple times to ensure the field is indeed uniform all around so that we know the placement of the structure has shown valid readings.

Alternatives are to just calculate the voltage of the compensation structure if you already know the exact properties of the field, or in that case if you know the properties of the field and the path of the wire you want to measure you can just apply Faradays law to the whole thing.

Remember im not trying to disprove anything about Faraday or Maxwell. Just saying that Kirchhoffs laws are convenient to use in some cases (And they do work when used correctly).

After trying to follow your instructions, I am not sure if I understand what you mean without a drawing. So I decided to simplify the challenge. Let's suppose that the whole internal area of the loop is completely occupied by the obstruction.

(https://www.eevblog.com/forum/chat/does-kirchhoffs-law-hold-disagreeing-with-a-master/?action=dlattach;attach=590755;image)

Perhaps that way it becomes easier for me to understand.
Title: Re: Does Kirchhoff's Law Hold? Disagreeing with a Master
Post by: bsfeechannel on December 07, 2018, 10:26:57 pm
LOL. Who would say so. BTW you also came with useless challenge. Now both of you can challenge each other until it hurts. This thread became :horse: - like both of you wanted.

I told you: go get yourself a better education. Now you can see that praising Mehdi and Mabilde and bashing Lewin didn't get you smarter.

Next time, listen to the voice of experience.
Title: Re: Does Kirchhoff's Law Hold? Disagreeing with a Master
Post by: beanflying on December 07, 2018, 11:28:19 pm
LOL. Who would say so. BTW you also came with useless challenge. Now both of you can challenge each other until it hurts. This thread became :horse: - like both of you wanted.

I told you: go get yourself a better education. Now you can see that praising Mehdi and Mabilde and bashing Lewin didn't get you smarter.

Next time, listen to the voice of experience.

Being a loudmouth (or loud typist) and attempting to belittle others is not educational or helpful to a debate it is no more than behaving like a petty bully in the schoolyard. But as you claim to be an adult with the ability to read this may be of benefit but I doubt you would understand it. https://www.psychologytoday.com/au/blog/neurosagacity/201702/how-tell-youre-dealing-malignant-narcissist (https://www.psychologytoday.com/au/blog/neurosagacity/201702/how-tell-youre-dealing-malignant-narcissist)
Title: Re: Does Kirchhoff's Law Hold? Disagreeing with a Master
Post by: Sredni on December 07, 2018, 11:40:57 pm
LOL. Who would say so. BTW you also came with useless challenge. Now both of you can challenge each other until it hurts. This thread became :horse: - like both of you wanted.

I told you: go get yourself a better education. Now you can see that praising Mehdi and Mabilde and bashing Lewin didn't get you smarter.

Next time, listen to the voice of experience.

Being a loudmouth (or loud typist) and attempting to belittle others is not educational or helpful to a debate it is no more than behaving like a petty bully in the schoolyard. But as you claim to be an adult with the ability to read this may be of benefit but I doubt you would understand it. https://www.psychologytoday.com/au/blog/neurosagacity/201702/how-tell-youre-dealing-malignant-narcissist (https://www.psychologytoday.com/au/blog/neurosagacity/201702/how-tell-youre-dealing-malignant-narcissist)

Nah, I don't think ogden is a malignant narcissist. He's just ignorant and proud.
You can tell from his numerous non-technical posts, whose sole intention is to belittle the efforts of the few people trying to bring a little physics in this 'technicians' den'.

But why did you quote Bsfeechannel exasperation post to tell him that?  ;D
Title: Re: Does Kirchhoff's Law Hold? Disagreeing with a Master
Post by: beanflying on December 07, 2018, 11:52:57 pm

But why did you quote Bsfeechannel exasperation post to tell him that?  ;D

Not even close to funny.

Being a bully in person or online isn't ok ever and when done under the guise of 'education and learning' it is just pathetic and nothing will be learned.

Like I posted yesterday attack the persons ideas or challenge a theory by all means people can learn from this sort of discourse.
Title: Re: Does Kirchhoff's Law Hold? Disagreeing with a Master
Post by: electrodacus on December 08, 2018, 12:21:05 am

And this is why it is nonsense to believe that you can locate a field big enough to give 0.25V voltage (integral of E.dl along the conductor) in the copper parts of the loop.
You would break the constitutive equation in copper. You would count twice the effects of the induced field.


I do not remember if I answered this concern you have in a direct way so I will do that now.
You asked several times about the size of the loop and I probably mentioned that is irrelevant for the problem so let me explain a bit better (I think).
emf = B * l * v
In the examples we were discussing the emf was a given none of the others after the equal sign where needed to calculate the potential difference between two points that was the topic of interest.
Now you are thinking on real world examples and somehow imagine a loop of a few centimeter (nowhere I ever mentioned anything about absolute length and neither what that the case in Lewin's example).
But looking at the formula you can understand that you can manipulate any of the 3 parameters and get the same emf (magnetic field, wire length and speed).

Any length of copper (or other conductor) will also be an inductor thus a energy storage device sort of the reverse of a capacitor but for magnetic field.
So using the analog of a battery that has an emf and an internal resistance is the correct way to view and simplify a piece of wire in a variable magnetic field when you consider this for a fixed moment in time else you need to integrate.
Title: Re: Does Kirchhoff's Law Hold? Disagreeing with a Master
Post by: Sredni on December 08, 2018, 12:40:41 am
Quote from: beanflying
Not even close to funny.

It wasn't meant to be funny. It was a sarcastic grin.
I suggest you go back reading ogden's posts in this thread and revise your judgment. All he did was to facepalm, LOL, belittle and tell other people they were trolls, to stick it where it hurts and so on.
But no, we should put up with that shit and give him an award for just showing up and saying bullshit.
Sorry, not all posters in this forum align to the politically correctness craze that is so endemic to the US.
I tried not to respond to provocations, but to fault bsfeechannel for telling him to go study, is in my eye a bit excessive.
He is the incarnation of the people who do not know, but think they know Lewin was talking about.
Enough is enough.


Quote from: electrodacus
You asked several times about the size of the loop and I probably mentioned that is irrelevant for the problem

Physical size is relevant to get the E field and current density j values.
I am beginning to suspect you think those are mytical quantities. Like unicorns.
I'll try to be more specific: if I know the current and I want to find the current density, I have to divide by the area of the wire's section. Also, voltage along a path is the integral of the electric field, do you think that knowing how long the path you are integrating on can have some relevance?



Title: Re: Does Kirchhoff's Law Hold? Disagreeing with a Master
Post by: beanflying on December 08, 2018, 01:22:54 am

It wasn't meant to be funny. It was a sarcastic grin.
I suggest you go back reading ogden's posts in this thread and revise your judgment. All he did was to facepalm, LOL, belittle and tell other people they were trolls, to stick it where it hurts and so on.
But no, we should put up with that shit and give him an award for just showing up and saying bullshit.
Sorry, not all posters in this forum align to the politically correctness craze that is so endemic to the US.
I tried not to respond to provocations, but to fault bsfeechannel for telling him to go study, is in my eye a bit excessive.
He is the incarnation of the people who do not know, but think they know Lewin was talking about.
Enough is enough.

Nowhere have I defended any of you including Ogden that have resorted to name calling and petty or even malicious taunts. This has nothing to do with political correctness so don't use that as an excuse to continue disrespectful conduct! Do you behave like this professionally or just here behind a keyboard?

Stick to the subject and there may be hope!
Title: Re: Does Kirchhoff's Law Hold? Disagreeing with a Master
Post by: Sredni on December 08, 2018, 01:39:55 am

It wasn't meant to be funny. It was a sarcastic grin.
I suggest you go back reading ogden's posts in this thread and revise your judgment. All he did was to facepalm, LOL, belittle and tell other people they were trolls, to stick it where it hurts and so on.
But no, we should put up with that shit and give him an award for just showing up and saying bullshit.
Sorry, not all posters in this forum align to the politically correctness craze that is so endemic to the US.
I tried not to respond to provocations, but to fault bsfeechannel for telling him to go study, is in my eye a bit excessive.
He is the incarnation of the people who do not know, but think they know Lewin was talking about.
Enough is enough.

Nowhere have I defended any of you including Ogden that have resorted to name calling and petty or even malicious taunts. This has nothing to do with political correctness so don't use that as an excuse to continue disrespectful conduct! Do you behave like this professionally or just here behind a keyboard?

Stick to the subject and there may be hope!

Disrespectful conduct?
What do you expect people to do? Hold hands and sing kumbaya when people tell you to stick it where it hurts, wasting space with useless comments that only expose their ignorance? The least you can expect is for those remarks to be sent back to the sender.

I was trying to stick to the subject, but I tell you - paraphrasing a certain Roy - I feel pretty much unappreciated.
Title: Re: Does Kirchhoff's Law Hold? Disagreeing with a Master
Post by: electrodacus on December 08, 2018, 01:42:35 am
Physical size is relevant to get the E field and current density j values.
I am beginning to suspect you think those are mytical quantities. Like unicorns.
I'll try to be more specific: if I know the current and I want to find the current density, I have to divide by the area of the wire's section. Also, voltage along a path is the integral of the electric field, do you think that knowing how long the path you are integrating on can have some relevance?

I have a hard time understanding your questions.
None of your questions are relevant to the problem (as far as I can see).
It sort of seems to suggest you disagree with the result's I got on those experiments. Is that the case ?
Or do you agree with the results but want to make some sort of point that I fail to see ?
Title: Re: Does Kirchhoff's Law Hold? Disagreeing with a Master
Post by: Berni on December 08, 2018, 11:07:21 am
That's just an engineering problem at this point.

1) Make a length of wire that would fit across those two points as if the pesky barrier was not there
2) Make a rigid wire structure that goes around the barrier as needed and connects to a voltmeter on the other end
3) Make another copy of the structure from 2, but short it using the piece of wire from 1
4) Place the structure from 2 onto the circuit to tap the voltage and place the structure from 3 anywhere near by
5) Subtract the readings of the voltmeters.

The compensation structure from 3 can be used multiple times to ensure the field is indeed uniform all around so that we know the placement of the structure has shown valid readings.

Alternatives are to just calculate the voltage of the compensation structure if you already know the exact properties of the field, or in that case if you know the properties of the field and the path of the wire you want to measure you can just apply Faradays law to the whole thing.

Remember im not trying to disprove anything about Faraday or Maxwell. Just saying that Kirchhoffs laws are convenient to use in some cases (And they do work when used correctly).

After trying to follow your instructions, I am not sure if I understand what you mean without a drawing. So I decided to simplify the challenge. Let's suppose that the whole internal area of the loop is completely occupied by the obstruction.

(https://www.eevblog.com/forum/chat/does-kirchhoffs-law-hold-disagreeing-with-a-master/?action=dlattach;attach=590755;image)

Perhaps that way it becomes easier for me to understand.

Same procedure still works, just needs longer probe wires to get around the larger obstruction.

Explaining it more simply you just measure your probe wires in the same field first by shorting them, then remove the short and actually connect them to the circuit, measure again and subtract out the first measurement to get the result. That way probe wires are compensated out. If you want to double check you can repeat the whole thing with probe wires taking a different path and the result will be the same.

Tho a lot of this thread has seam to have devolved into insults (Nothing towards me but towards others) rather than creative discussion so il probably stop participating in it if this continues.
Title: Re: Does Kirchhoff's Law Hold? Disagreeing with a Master
Post by: ogden on December 08, 2018, 12:15:35 pm
Tho a lot of this thread has seam to have devolved into insults

From very beginning "Kirchoff for the birds" fans arrogantly insulted nearly everybody who disagree with them. Later rather than sooner it resulted in opposite reaction. I was part of it and not proud about it. Apologies to anyboody who got hurt in the process. Most likely I shall stay away from this thread which is/was just baseless "arguing (https://www.eevblog.com/forum/chat/does-kirchhoffs-law-hold-disagreeing-with-a-master/msg1979633/#msg1979633) against the nonexistent strawman who is apparently suggesting that Farady's law is incorrect, and Kirchoffs is always correct", like broken record.
Title: Re: Does Kirchhoff's Law Hold? Disagreeing with a Master
Post by: bsfeechannel on December 08, 2018, 01:58:33 pm
Same procedure still works, just needs longer probe wires to get around the larger obstruction.

Thank you for your reply. However I'm still not sure what you mean, perhaps due to my limited fluency in English. When you say the probe goes around the obstruction, where exactly do you place your voltmeter? I understand that, in the area of the obstruction, it'll be impossible.

Quote
Tho a lot of this thread has seam to have devolved into insults (Nothing towards me but towards others) rather than creative discussion so il probably stop participating in it if this continues.

Apparently, encouraging people to get smarter is offensive for some.
Title: Re: Does Kirchhoff's Law Hold? Disagreeing with a Master
Post by: bsfeechannel on December 08, 2018, 03:39:15 pm
From very beginning "Kirchoff for the birds" fans arrogantly insulted nearly everybody who disagree with them.

There's no such thing as "Kirchhoff for the birds" fans. I myself was considered a king of circuit analysis when I was in college. I got my first job as an engineer because of that ability.

However, I know darn well that Kirchhoff can only be applied to lumped circuits and it doesn't work for anything under a varying electromagnetic field. Because, under that condition, a circuit becomes an "antenna", and antennas are everything but lumped. You'll have induction everywhere so it will be impossible to solve a circuit using Kirchhoff which does not provide the tools for either designing or probing your circuits.

How do I know that? By my own experience. When I got my first job as an engineer, digital circuits were leaving the domains of Kirchhoff and foraying into the realm of electromagnetism. Frequencies were such that the wavelengths were becoming as short as the size of the PCBs.

We had problems with the companies that designed our PCBs at the time, because they were kirchhoffalwaysholders (the cute name I decided to bestow upon those who think that Kirchhoff always holds) not by choice, but because they were used to low frequency circuits, where Kirchhoff is good enough.

We had to invite the help of our colleagues from the radio division, for whom Maxwell was second nature, and use their experience to instruct our designers how to properly design a PCB for higher frequencies, which is commonplace today.

This kirchhoffalwaysholdery sucks, therefore, because not only it is a pseudo-scientific doctrine, it is an encumbrance for any serious electronics engineering today.

And apparently it is now taking the shape of a religious movement where its converts get very offended when it is shown in theory or practice that their nonsense dogma is false.

As Lewin says, "this tells you something about them".
Title: Re: Does Kirchhoff's Law Hold? Disagreeing with a Master
Post by: bsfeechannel on December 08, 2018, 03:56:41 pm
I'm afraid of you being a troll and wasting my time.

Fear not. According to the dictionary, a troll is someone who posts deliberately inflammatory articles on an internet discussion board. I may rant or show I am indignant at some absurd fact sometimes, but I never troll. I don't have the time or the motivation for hanging around in forums posting provocative assertions just to see the world burn.

I'm just questioning your claims. That's what forums are for.

Quote
Read the replay above as it may be relevant if you are not a troll.

I did, thank you.

Quote
Shape of the loop will make no difference as long as B-flux is uniform but if that is not the case then you will not be able to calculate that with just pen and paper (you may be able to approximate something) but you will need a computer simulation tool to solve that and of course all details to scale.
And even if flux is uniform you will need to know the total length of that loop and the length between the two points you want to make the measurement then calculation is the same as for the simple ring model as shape alone makes no difference.   

Thanks again for your reply. I thought of many other "challenges", but I won't tire you with an infinite list of questionings. I just wanted to understand the basic principles of your claim.
Title: Re: Does Kirchhoff's Law Hold? Disagreeing with a Master
Post by: rfeecs on December 08, 2018, 11:17:15 pm

So how does a voltmeter tell the difference if it only shows the field caused by electron density and not the magnetic EMF? (Hint: It does show some EMF too but not where you would typically want)

If voltmeters treat the two separately, why should we treat them as the same thing? We are trying to calculate what the voltmeter would show after all.

I'm not trying to pick sides here, or say anything negative about anyone. To me it seams that most people in this thread are not saying anything wrong for the most part, but the disagreement seams to stem from using a slightly different definition of things and more rarely a bit from just having a different thought process about this thing.

I'm not clear what you are talking about.  So the two voltmeters in this experiment are not affected by the EMF?  Then why do they read different voltages?

Anyway, voltmeters don't read the field caused by electron density.  They don't read the electrostatic potential.  Take the example of a PN junction diode.  It clearly has different electron densities in the P and N depletion region.  There is an electrostatic potential difference due to the charge separation.  But a voltmeter measures zero volts when connected to the leads of the diode.

If you integrate E dot dl through an unbiased diode, you get a voltage!  Diodes violate KVL!

(https://i.stack.imgur.com/SaSH6.jpg)

Obviously using integral of E dot dl has a problem.  Circuits with diodes would be another KVL fail according to Dr. Lewin's definition.
Title: Re: Does Kirchhoff's Law Hold? Disagreeing with a Master
Post by: ogden on December 09, 2018, 12:07:16 am
If you integrate E dot dl through an unbiased diode, you get a voltage!  Diodes violate KVL!

Indeed. Chemical battery violates KVL as well. Resistor and any other lone component violates KVL. Kirchoff's Circuit Laws requires closed Circuit. As some insist that Dr.Lewin's loop cannot be split into lumped elements, then all this conversation is futile. When we agree that 1/4 of the Dr.Lewin's experiment (inner) loop receives EMF/4 and can be treated as lumped element meaning Berni model (https://www.eevblog.com/forum/chat/does-kirchhoffs-law-hold-disagreeing-with-a-master/msg1945312/#msg1945312) is correct - then there's ground for conversation. Honestly I do not see such time coming.

Some (you know who) can prove me wrong  :popcorn:
Title: Re: Does Kirchhoff's Law Hold? Disagreeing with a Master
Post by: rfeecs on December 09, 2018, 12:26:27 am
If you integrate E dot dl through an unbiased diode, you get a voltage!  Diodes violate KVL!

Indeed. Chemical battery violates KVL as well. Resistor and any other lone component violates KVL. Kirchoff's Circuit Laws requires closed Circuit.

That's not my point.  You can make a closed circuit by connecting the diode leads together with a wire.  Go around the loop and calculate the integral of E dot dl.  For the wire it is zero, for the diode, it is not zero.  So the integral of E dot dl around the loop is not zero.  According to Dr. Lewin's definition, Kirchoff's Loop Rule is violated.  So are Maxwell's equations?  The world is flat.

But everyone knows the voltage across the diode is zero, and the voltage across the wire is zero.  So KVL is OK, and the world is not flat.
Title: Re: Does Kirchhoff's Law Hold? Disagreeing with a Master
Post by: ogden on December 09, 2018, 01:38:37 am
You can make a closed circuit by connecting the diode leads together with a wire.  Go around the loop and calculate the integral of E dot dl.

No. Kirchoff's Circuit Laws requires closed circuit of lumped elements. So diode is one lumped element of the circuit and wire supposedly with it's internal resistance - another. So two elements. When you do circuit analysis - you don't go around the loop integrating everything in the path. This is not how CIRCUIT analysis shall be done. You do E dot dl over diode and make it lumped element - black box with two terminals, you can name it as voltage source when subject to light for example. Then you do E dot dl with wire and again make it black box with two terminals, name it load. Both are lumped elements of our circuit. Connect those together and *then* check against Kirchoff's Laws using voltage/current measurements.

Shall I repeat? Pay close attention to term "lumped element":

When we agree that 1/4 of the Dr.Lewin's experiment (inner) loop receives EMF/4 and can be treated as lumped element meaning Berni model (https://www.eevblog.com/forum/chat/does-kirchhoffs-law-hold-disagreeing-with-a-master/msg1945312/#msg1945312) is correct - then there's ground for conversation.

[edit] Better stick to electromagnetism.
Title: Re: Does Kirchhoff's Law Hold? Disagreeing with a Master
Post by: Sredni on December 09, 2018, 02:40:35 am
Take the example of a PN junction diode.  It clearly has different electron densities in the P and N depletion region.  There is an electrostatic potential difference due to the charge separation.  But a voltmeter measures zero volts when connected to the leads of the diode.

The reason for that is that to connect to the silicon you have to create ohmic contacts (non-rectifying contacts) and...
Nah, I'll use the first link.
https://www.quora.com/Why-we-cant-measure-the-barrier-potential-existing-across-a-p-n-junction-by-connecting-voltmeter-across-the-p-n-junction (https://www.quora.com/Why-we-cant-measure-the-barrier-potential-existing-across-a-p-n-junction-by-connecting-voltmeter-across-the-p-n-junction)
Title: Re: Does Kirchhoff's Law Hold? Disagreeing with a Master
Post by: rfeecs on December 09, 2018, 03:21:25 am
Take the example of a PN junction diode.  It clearly has different electron densities in the P and N depletion region.  There is an electrostatic potential difference due to the charge separation.  But a voltmeter measures zero volts when connected to the leads of the diode.

The reason for that is that to connect to the silicon you have to create ohmic contacts (non-rectifying contacts) and...
Nah, I'll use the first link.
https://www.quora.com/Why-we-cant-measure-the-barrier-potential-existing-across-a-p-n-junction-by-connecting-voltmeter-across-the-p-n-junction (https://www.quora.com/Why-we-cant-measure-the-barrier-potential-existing-across-a-p-n-junction-by-connecting-voltmeter-across-the-p-n-junction)

Don't believe everything you read on the internet.

The voltmeter reads the difference in Fermi levels between the two contacts:

https://en.wikipedia.org/wiki/Fermi_level (https://en.wikipedia.org/wiki/Fermi_level)

Quote
Sometimes it is said that electric currents are driven by differences in electrostatic potential (Galvani potential), but this is not exactly true.[2] As a counterexample, multi-material devices such as p–n junctions contain internal electrostatic potential differences at equilibrium, yet without any accompanying net current; if a voltmeter is attached to the junction, one simply measures zero volts.[3] Clearly, the electrostatic potential is not the only factor influencing the flow of charge in a material—Pauli repulsion, carrier concentration gradients, electromagnetic induction, and thermal effects also play an important role.

In fact, the quantity called voltage as measured in an electronic circuit has a simple relationship to the chemical potential for electrons (Fermi level). When the leads of a voltmeter are attached to two points in a circuit, the displayed voltage is a measure of the total work transferred when a unit charge is allowed to move from one point to the other. If a simple wire is connected between two points of differing voltage (forming a short circuit), current will flow from positive to negative voltage, converting the available work into heat...

Maybe this is straying too far off topic.  The point was supposed to be that a voltmeter doesn't measure electrostatic potential, and what a voltmeter actually measures and what is the definition of potential and voltage are more complicated than we usually think.
Title: Re: Does Kirchhoff's Law Hold? Disagreeing with a Master
Post by: Sredni on December 09, 2018, 04:01:25 am
The reason for that is that to connect to the silicon you have to create ohmic contacts (non-rectifying contacts) and...
Nah, I'll use the first link.
https://www.quora.com/Why-we-cant-measure-the-barrier-potential-existing-across-a-p-n-junction-by-connecting-voltmeter-across-the-p-n-junction (https://www.quora.com/Why-we-cant-measure-the-barrier-potential-existing-across-a-p-n-junction-by-connecting-voltmeter-across-the-p-n-junction)
Don't believe everything you read on the internet.
The voltmeter reads the difference in Fermi levels between the two contacts:
https://en.wikipedia.org/wiki/Fermi_level (https://en.wikipedia.org/wiki/Fermi_level)

Well, wikipedia is on the Internet, so I shouldn't believe it. But maybe you misquoted it.
Besides, the difference in Fermi levels is the barrier potential (if we agree on how to treat the sign). I guess you were the one saying that you cannot read it with a voltmeter.

But if you want a reference that is not on the Internet, you might want to read page 242 of "Semiconductor Physics and Devices" by Donald Neamen.

"This potential difference across the junction cannot be measured with a voltmeter because new potential barriers will be formed between the probes and the semiconductor that will cancel V_bi"

This is the first book I took off my shelf, but I'm pretty sure I could find something along the same line on Sze, or on Streetman, or on Muller Kamins.
Oh my.
Title: Re: Does Kirchhoff's Law Hold? Disagreeing with a Master
Post by: rfeecs on December 09, 2018, 06:00:49 am
The reason for that is that to connect to the silicon you have to create ohmic contacts (non-rectifying contacts) and...
Nah, I'll use the first link.
https://www.quora.com/Why-we-cant-measure-the-barrier-potential-existing-across-a-p-n-junction-by-connecting-voltmeter-across-the-p-n-junction (https://www.quora.com/Why-we-cant-measure-the-barrier-potential-existing-across-a-p-n-junction-by-connecting-voltmeter-across-the-p-n-junction)
Don't believe everything you read on the internet.
The voltmeter reads the difference in Fermi levels between the two contacts:
https://en.wikipedia.org/wiki/Fermi_level (https://en.wikipedia.org/wiki/Fermi_level)

Well, wikipedia is on the Internet, so I shouldn't believe it. But maybe you misquoted it.
Besides, the difference in Fermi levels is the barrier potential (if we agree on how to treat the sign). I guess you were the one saying that you cannot read it with a voltmeter.
With no bias, in equilibrium, the Fermi levels on both terminals are equal, so zero voltage.  Refer to figure 7.3 of Neamen.
Quote

But if you want a reference that is not on the Internet, you might want to read page 242 of "Semiconductor Physics and Devices" by Donald Neamen.

"This potential difference across the junction cannot be measured with a voltmeter because new potential barriers will be formed between the probes and the semiconductor that will cancel V_bi"
You have to wonder if "potential barrier" is the right phrase for an ohmic contact, which should have little or no barrier.
Quote
This is the first book I took off my shelf, but I'm pretty sure I could find something along the same line on Sze, or on Streetman, or on Muller Kamins.
Oh my.
So what's your point?  Are you saying that there is no net electrostatic potential across the diode terminals?
Title: Re: Does Kirchhoff's Law Hold? Disagreeing with a Master
Post by: ogden on December 09, 2018, 09:04:41 am
"This potential difference across the junction cannot be measured with a voltmeter because new potential barriers will be formed between the probes and the semiconductor that will cancel V_bi"

Right. This is where I fully agree with you :)

So what's your point?

What's *your* point to talk about semiconductors in discussion about Dr.Lewin's lecture explaining electromagnetism?

Observer effect of quantum theory while looking at bare PN junction is way too huge stretch off the rails of said discussion. We shall stick to Dr.Lewin's original experiment conditions where circuit element terminals/lugs are made out of conductor (not semiconductor) and voltmeter measures potential difference using Ohms law - by running current through it's internal resistance.

So if such "voltmeter vulgaris" measure 0V on lumped element terminals, we say it's E dot dl is zero and disregard quantum or chemical or whatever phenomena inside it.
Title: Re: Does Kirchhoff's Law Hold? Disagreeing with a Master
Post by: Berni on December 09, 2018, 09:45:41 am
I'm not clear what you are talking about.  So the two voltmeters in this experiment are not affected by the EMF?  Then why do they read different voltages?

Anyway, voltmeters don't read the field caused by electron density.  They don't read the electrostatic potential.  Take the example of a PN junction diode.  It clearly has different electron densities in the P and N depletion region.  There is an electrostatic potential difference due to the charge separation.  But a voltmeter measures zero volts when connected to the leads of the diode.

If you integrate E dot dl through an unbiased diode, you get a voltage!  Diodes violate KVL!

(https://i.stack.imgur.com/SaSH6.jpg)

Obviously using integral of E dot dl has a problem.  Circuits with diodes would be another KVL fail according to Dr. Lewin's definition.

This diagram shows a diode with a current flowing trough it. In such a case all semiconductors show a voltage drop that can indeed be measured with a voltmeter.

In a rest state any voltage created on the junction is subtracted back out once the semiconductor connects to the copper pins. If a diode was to create a voltage in such a conduction this would mean i will also have to be capable of pushing current in that direction of voltage. Once you have both you have power being output from the diode and this would violate conservation of energy. That being said it is possible to use a diode to generate a voltage. If you are to heat up one end of a diode and cool the other you can get a strong thermocouple effect that converts some of that heat into output power on the pins. Additionally if this is a glass encapsulated diode or a LED then shining the right wavelengths of light on the diodes junction will also cause it to operate like a solar cell and produce power. In all of these cases external energy had to be put in to make it do that.

Again KVL has no way of dealing with diodes as that's not part of its job. But circuit analysis theory makes it work by having lumped models for all these semiconductor devices (Diodes, BJTs, FETs, IGBTs, SCRs...). There lumped models often contain parametric current sources and volt/amp meters inside of them and they vary in complexity depending on how accurate you need it to be. This allows for circuit analysis to be used on circuits with active components without any issues.

So what does a voltmeter measure? Well it actually measures the current trough its internal resistance and then displays what voltage it takes to push such a current. Notice how in Dr. Lewins example the voltage across the resistors is always defined as a single value. In the same way it is defined to have a single value across the terminals of a voltmeter. Since an ideal resistor has zero physical dimension, means that it is impossible to generate any magnetic EMF across it (It can't be part of a surface area edge as it has no length) and a external electrostatic field can't produce a gradient sharp enough to pull electrons along. So the only "electron pusher" that remains to convince electrons to flow trough the resistor is the difference in charge density on the resistors terminals. The crowded electrons on one end want to get trough to the not as crowded electrons on the other end. Hence why the voltmeter ends up showing a difference in charge density across its terminals.

But it is possible to have EMF generated on a resistor that has physical length. Its basically the combination of a wire and a resistor (And can be lump modeled as such if desired). In the same way a voltmeter that's longer than zero will read EMF across itself. But its only the EMF induced in the section that the voltmeters size occupies. So the larger the voltmeter the more EMF it will show on the display. This just makes things more confusing so we say voltmeters have zero size so they don't read any EMF.
Title: Re: Does Kirchhoff's Law Hold? Disagreeing with a Master
Post by: rfeecs on December 09, 2018, 05:24:13 pm
This diagram shows a diode with a current flowing trough it. In such a case all semiconductors show a voltage drop that can indeed be measured with a voltmeter.

In a rest state any voltage created on the junction is subtracted back out once the semiconductor connects to the copper pins. If a diode was to create a voltage in such a conduction this would mean i will also have to be capable of pushing current in that direction of voltage.

Actually, the diagram is in equilibrium, meaning no net current is flowing.

I fully agree that the diode contacts, which are not shown on the diagram, will have their own potential difference, E-field and charge that compensates for the voltage across the diode junction.  Maxwell's equations still work.

So I guess the only point of bringing up the diode is to point out that E field is not the only "pusher" of charge.  The concentration gradient at the diode junction is another.  You have electrochemical potential as well as electrostatic potential and induced EMF that can all move charge.  The voltmeter can't tell the difference between them.

Quote
So what does a voltmeter measure? Well it actually measures the current trough its internal resistance and then displays what voltage it takes to push such a current. Notice how in Dr. Lewins example the voltage across the resistors is always defined as a single value. In the same way it is defined to have a single value across the terminals of a voltmeter. Since an ideal resistor has zero physical dimension, means that it is impossible to generate any magnetic EMF across it (It can't be part of a surface area edge as it has no length) and a external electrostatic field can't produce a gradient sharp enough to pull electrons along. So the only "electron pusher" that remains to convince electrons to flow trough the resistor is the difference in charge density on the resistors terminals. The crowded electrons on one end want to get trough to the not as crowded electrons on the other end. Hence why the voltmeter ends up showing a difference in charge density across its terminals.

But it is possible to have EMF generated on a resistor that has physical length. Its basically the combination of a wire and a resistor (And can be lump modeled as such if desired). In the same way a voltmeter that's longer than zero will read EMF across itself. But its only the EMF induced in the section that the voltmeters size occupies. So the larger the voltmeter the more EMF it will show on the display. This just makes things more confusing so we say voltmeters have zero size so they don't read any EMF.

So you are still saying that EMF is located at specific segments of the loop.  A zero length voltmeter won't show any EMF on the display?  I thought it would show the EMF of the whole loop that includes the voltmeter and the test leads and the path connecting the two points you are measuring.

We're all just stuck in an endless loop now.  break;
Title: Re: Does Kirchhoff's Law Hold? Disagreeing with a Master
Post by: ogden on December 09, 2018, 08:20:39 pm
You have electrochemical potential as well as electrostatic potential and induced EMF that can all move charge.  The voltmeter can't tell the difference between them.

What does it prove? - That all electrons are equal and you can't mark them?

So you are still saying that EMF is located at specific segments of the loop.

Seems, you are alone denying this here. Remember time when this thread talked about resistive ring (introduced by you BTW)?

https://www.eevblog.com/forum/chat/does-kirchhoffs-law-hold-disagreeing-with-a-master/msg1961348/#msg1961348 (https://www.eevblog.com/forum/chat/does-kirchhoffs-law-hold-disagreeing-with-a-master/msg1961348/#msg1961348)
Title: Re: Does Kirchhoff's Law Hold? Disagreeing with a Master
Post by: radioactive on December 20, 2018, 04:27:01 am
I probably shouldn't dig this thread back up, but wanted to (humbly) share the sim that I worked on such as it is.  I've seen some of the comments referring indirectly to my previous posts as trolls, idiot, etc. First off, let me start by saying that I'm not trying to *prove* anything here.  I don't care who is right or wrong.  I'm just someone who had some time recently to play with openEMS and see if I could reproduce something similar to the Romer experiment and thought I would share it in case someone else might see how cool FDTD sims are.  I feel bad for screwing up the initial sims and posting them.  It may still have issues, but I think I figured out what was wrong in the first couple of posts.  The mesh you see in the geometry viewer is not necessarily how the materials get discretized before being passed on to the FDTD engine  (I had forgotten about this).  It is really important to use the --debug-PEC option to view the mesh in Paraview in order to verify materials didn't disappear too much after the engine matches materials up on the mesh sub-volumes.   This sim is still not the right scale for Romer because it is just too much to discretize a meshed coil with sub-mm wire radius.   I still think it is worth sharing.  If for no other reason, then just to give some more exposure to the really nice open/free software called openEMS  http://www.openems.de/start/index.php (http://www.openems.de/start/index.php)

As for the crazy voltages / currents in my previous post that some obviously found funny,  I should have explained more on that.   I was testing several different voltages for the excitation with highly scaled up geometry to try and see if I got a log response (I did).  I noticed the thing with the feed coil and posted some images while the voltage was something like 10e6 or something crazy like that while trying to make an unrelated point (unconvincingly).   Part of the issue may have been with the mesh on that one... or it may have been find at that point... not sure.  Anyway, hopefully that makes sense.  I understand some of the comments.  Sorry if I wasted your time or offended in previous posts. 

The images and videos are from a sim with a triangular waveform (like Romer), but with a DC bias (not like Romer).  This gives a slow rise DC component in the beginning of the excitation.  Note that due to the time it takes to simulate a complex geometry like this, the excitation is at a higher frequency (above SRF and much higher than Romer).  I also ran the sim for 1 cycle below SRF (still much shorter wavelength than Romer) and will post voltage measurements for that.   The videos are generated with Paraview.   Also another very nice free software.

SRF of the coil was determined by excitation with a bandwidth-limited Gaussian pulse which generates a nice flat stimulus in the frequency domain  (think using a wideband noise source with an FFT spectrum analyzer).   The simulation generates time domain voltages / currents in a text file.  If you process that with FFT, then you can analyze arbitrary materials / structures in the frequency domain.   

Another much-easier-to-simulate-with-amazing-accuracy example would be a planar microstrip filter made of copper, vias, dielectric, airbox, etc.

Time steps in the videos should match time steps in the plots.

https://www.youtube.com/watch?v=84wCYzqA5a8 (https://www.youtube.com/watch?v=84wCYzqA5a8)

https://www.youtube.com/watch?v=Y57rDnQfjlw (https://www.youtube.com/watch?v=Y57rDnQfjlw)

Another thing that someone might find interesting...  was talking to a guy who does almost nothing but RF sims.   He told me about another method.  MoM  (method of moments) that can be used to mesh geometries and convert them to equivalent reactive components for simulating in a standard analytical circuit simulator  (e.g. Spice).  I couldn't find any open-source projects that looked like something worth investigating.  Would be very interested this if someone is aware of any.
Title: Re: Does Kirchhoff's Law Hold? Disagreeing with a Master
Post by: ogden on December 20, 2018, 12:44:38 pm
radioactive, I wanted to double check your previous simulation and posts  - they all gone, deleted. This is disrespect to people who spent their time answering.

I'm just someone who had some time recently to play with openEMS and see if I could reproduce something similar to the Romer experiment

Thank you for investing your time trying. You shall show time and voltage units used in X&Y scales. Now we can only guess - excitation pulse period is 100 seconds, 100 femtoseconds or what? It is very important to simulate experiment as close as possible to the original which is pure electromagnetism because of huge solenoid and comparably slow impulse where electric fields and antenna effects can be ignored. Your simulations seems far from that. Besides my suggestion to add core material to form solenoid, I would like to remind following comment as well:

So you need to apply a similar voltage step response across your solenoid coil (Not just a pulse). Also your time scale appears to be very short in the simulation. The pulse you applied seams to last only a few picoseconds, this gives it a bandwidth of >100GHz and hence why you get funny behavior as you are mostly simulating radio waves traveling around your scene. The whole simulation only lasting what appear to be around half a nanosecond. My experiment had the pulse last 500 microseconds so about 1 000 000 times longer than your simulation time.
Title: Re: Does Kirchhoff's Law Hold? Disagreeing with a Master
Post by: radioactive on December 20, 2018, 03:10:46 pm
Ogden,

Quote
Besides my suggestion to add core material to form solenoid,
  See the Romer paper where "iron core" is specifically mentioned (and not used).

As for the instantaneous step,  I think this would be a good simulation for you to try now that you have the source.  You might be surprised at the results.

Title: Re: Does Kirchhoff's Law Hold? Disagreeing with a Master
Post by: ogden on December 20, 2018, 10:16:25 pm
Quote
Besides my suggestion to add core material to form solenoid,
  See the Romer paper where "iron core" is specifically mentioned (and not used).

I did miss that, apologies. Actually no big deal because see the Romer's paper where it is said that solenoid is wound with 444 turns of wire, it's inductance is 1.8 mH (quite big number) and test frequency is 300 Hz. What is inductance of your "solenoid"? What is frequency of your signal?

Quote
I think this would be a good simulation for you to try now that you have the source.

You are so kind, but thank you. I think would be good if you fix/update your simulation. First thing you shall fix is presentation of your waveform graphs:

You shall show time and voltage units used in X&Y scales. Now we can only guess - excitation pulse period is 100 seconds, 100 femtoseconds or what?
Title: Re: Does Kirchhoff's Law Hold? Disagreeing with a Master
Post by: mhz on December 22, 2018, 03:17:29 am
<Knock knock>

Is this thing still on?

The super demo has long bothered me since I first watched it back in 2004 or so (when I was first learning Physics). I recently came back to it a few months ago and spent many late nights reading physicsforums threads of people arguing over it. Even though Mehdi (and Mabilde) are ultimately wrong and misleading many people, I'm really happy about all the drama surrounding this because it surfaced up all the extra pieces I needed to understand (I feel like I'm at maybe 80~90 % comfortability with this now), namely Lewin's response videos (which included the crucial mesh analysis), the Romer paper, the McDonald paper (which admittedly includes some Physics concepts beyond my current level but includes crucial footnotes and KVL history), the Feynman chapter, and finally the Belcher writeup.

I spent a lot of time working on this by myself and then found this thread fearing it would be more of the same in Electroboom's comment section but I was happy to find some voices of reason that supported my conclusions in sredni and bsfeechannel. Thank you for your patience in these past 19 pages of threads.

I tried to organize my thoughts about the whole thing here https://grumpyengineering.wordpress.com/ (only one post there for now) if you want to read them.
Title: Re: Does Kirchhoff's Law Hold? Disagreeing with a Master
Post by: radioactive on December 22, 2018, 06:17:28 pm
Quote
I tried to organize my thoughts about the whole thing here https://grumpyengineering.wordpress.com/ (only one post there for now) if you want to read them.

I'm looking forward to reading more of your thoughts if you continue.  Very easy to read.  I also echo your thanks to sredni and bsfeechannel, and especially Lewin for their responses/patience to the challenge put forth.  Combing those responses with working on an EM sim and being able to see the mag/dir of the fields over time/space definitely gave me a much better intuitive feel for it.

Title: Re: Does Kirchhoff's Law Hold? Disagreeing with a Master
Post by: ogden on December 24, 2018, 09:31:08 pm
I tried to organize my thoughts about the whole thing here https://grumpyengineering.wordpress.com/ (https://grumpyengineering.wordpress.com/) (only one post there for now) if you want to read them.

When you talk about advanced stuff like Faraday's law, you shall not ignore other laws like law of conservation of energy (https://en.wikipedia.org/wiki/Conservation_of_energy).

Pay close attention to following post made by proponent of Dr.Lewin (https://www.eevblog.com/forum/chat/does-kirchhoffs-law-hold-disagreeing-with-a-master/msg1961348/#msg1961348) to see where exactly you and Dr.Lewin made mistake.
Title: Re: Does Kirchhoff's Law Hold? Disagreeing with a Master
Post by: mhz on December 24, 2018, 09:53:36 pm
I tried to organize my thoughts about the whole thing here https://grumpyengineering.wordpress.com/ (https://grumpyengineering.wordpress.com/) (only one post there for now) if you want to read them.

When you talk about advanced stuff like Faraday's law, you shall not ignore other laws like law of conservation of energy (https://en.wikipedia.org/wiki/Conservation_of_energy).

I agree.

Quote
Pay close attention to following post made by proponent of Dr.Lewin (https://www.eevblog.com/forum/chat/does-kirchhoffs-law-hold-disagreeing-with-a-master/msg1961348/#msg1961348) to see where exactly you and Dr.Lewin made mistake.

I don't see an issue here, but feel free to point it out if you like.
Title: Re: Does Kirchhoff's Law Hold? Disagreeing with a Master
Post by: ogden on December 24, 2018, 10:10:56 pm
Quote
Pay close attention to following post made by proponent of Dr.Lewin (https://www.eevblog.com/forum/chat/does-kirchhoffs-law-hold-disagreeing-with-a-master/msg1961348/#msg1961348) to see where exactly you and Dr.Lewin made mistake.

I don't see an issue here, but feel free to point it out if you like.

Your (and Dr.Lewin's) equation does not separate EMF (energy) source from load, incorrectly and blatantly saying that KVL is as follows:

(https://s0.wp.com/latex.php?latex=%5Coint+%5Cvec+E+%5Ccdot+d%5Cvec+l+%3D+0&bg=ffffff&fg=333333&s=4)

Kirchsoffs CIRCUIT law requires circuit consisting of energy source and load. Correct equation would be:

(https://www.eevblog.com/forum/chat/does-kirchhoffs-law-hold-disagreeing-with-a-master/?action=dlattach;attach=570254;imag)
Title: Re: Does Kirchhoff's Law Hold? Disagreeing with a Master
Post by: mhz on December 24, 2018, 11:35:07 pm
Quote
Pay close attention to following post made by proponent of Dr.Lewin (https://www.eevblog.com/forum/chat/does-kirchhoffs-law-hold-disagreeing-with-a-master/msg1961348/#msg1961348) to see where exactly you and Dr.Lewin made mistake.

I don't see an issue here, but feel free to point it out if you like.

Your (and Dr.Lewin's) equation does not separate EMF (energy) source from load, incorrectly and blatantly saying that KVL is as follows:

(https://s0.wp.com/latex.php?latex=%5Coint+%5Cvec+E+%5Ccdot+d%5Cvec+l+%3D+0&bg=ffffff&fg=333333&s=4)

It's not "my" equation. It's how Lewin defines KVL. I'm not sure why you think it can't be rewritten to explicitly show sources and loads. It would look like this.
(https://s0.wp.com/latex.php?latex=%5Coint+_%7Bsources%7D+%5Cvec+E+%5Ccdot+%5Cvec+dl+%2B+%5Coint+_%7Bloads%7D+%5Cvec+E+%5Ccdot+%5Cvec+dl%3D+0+&bg=ffffff&fg=333333&zoom=2)

Quote
Kirchsoffs CIRCUIT law requires circuit consisting of energy source and load.

Not really. I can apply KVL just fine to a trivial network of resistors with no energy source and still get the correct answers.

Quote
Correct equation would be:
(https://www.eevblog.com/forum/chat/does-kirchhoffs-law-hold-disagreeing-with-a-master/?action=dlattach;attach=570254;imag)

Actually, I think there's a minus sign missing on the right side of the above equation, due to Lenz law, but I'm not really sure what you're arguing here. Are you saying that Lewin's definition of KVL doesn't work in this circuit? If so then yes, I agree, that's just about his entire point!
Title: Re: Does Kirchhoff's Law Hold? Disagreeing with a Master
Post by: ogden on December 25, 2018, 09:27:43 am
It's not "my" equation. It's how Lewin defines KVL. I'm not sure why you think it can't be rewritten to explicitly show sources and loads. It would look like this.
(https://s0.wp.com/latex.php?latex=%5Coint+_%7Bsources%7D+%5Cvec+E+%5Ccdot+%5Cvec+dl+%2B+%5Coint+_%7Bloads%7D+%5Cvec+E+%5Ccdot+%5Cvec+dl%3D+0+&bg=ffffff&fg=333333&zoom=2)

Right. BTW where I did say that I think it can't be rewritten? Look, whole idea of "KVL does not work" proof is based on statement that integral of E.dl for the loop equals zero, thus EMF equals zero which is as Dr.Lewin say impossible. Indeed it is impossible - because equation is incomplete, thus statement is futile. You just corrected it by writing EMF + ( -I*R ) = 0. If you agree then we are done. You disproved Dr.Lewin.
Title: Re: Does Kirchhoff's Law Hold? Disagreeing with a Master
Post by: mhz on December 25, 2018, 10:44:27 am
It's not "my" equation. It's how Lewin defines KVL. I'm not sure why you think it can't be rewritten to explicitly show sources and loads. It would look like this.
(https://s0.wp.com/latex.php?latex=%5Coint+_%7Bsources%7D+%5Cvec+E+%5Ccdot+%5Cvec+dl+%2B+%5Coint+_%7Bloads%7D+%5Cvec+E+%5Ccdot+%5Cvec+dl%3D+0+&bg=ffffff&fg=333333&zoom=2)

Right. BTW where I did say that I think it can't be rewritten?
When you said "Your (and Dr.Lewin's) equation does not separate EMF (energy) source from load". I suppose you didn't explicitly say it can't be rewritten, but you seemed to be implying that for some reason it couldn't. Anyways, I'm glad we agree on this.

Quote
Look, whole idea of "KVL does not work" proof is based on statement that integral of E.dl for the loop equals zero, thus EMF equals zero which is as Dr.Lewin say impossible.
In a typical batteries+resistors type circuit, the EMF is the batteries, and they are included in the E•dl integration. I wrote this out explicitly in my last post where the source and loads were specified in separate integrations (i'm not even sure if this is proper notation but I think you get the point). Saying that E•dl equals zero means the EMF must equal zero is not correct, and I don't believe Lewin ever said that (please link with timestamp if he did). Rather, the point is that using KVL (as defined by Lewin as E•dl = 0) will yield the wrong answer in the presence of time varying magnetic flux, and the reason it yields the wrong answer is that now we have an EMF that doesn't come from an electric field.
Quote
Indeed it is impossible - because equation is incomplete, thus statement is futile.
Of course it's incomplete, that's Lewin's point. And the way to complete it is to update it to Faraday's law. If you disagree I'd ask you how you would complete it.

Quote
You just corrected it by writing EMF + ( -I*R ) = 0. If you agree then we are done. You disproved Dr.Lewin.
As I mentioned in the previous post, there was a minus sign missing. In the drawing, the assumption is we're looking at a specific point in time where the emf from the solenoid is  1V. The value of the evenly distributed resistance will determine the value of the current (if it's total 1ohm, we get 1A, etc.). Either way, if you start from Faraday's law, you have 1V on both sides of the equation (both sides better be the same value otherwise we either screwed up or Faraday's law is somehow wrong). If you move that 1V over to the left then you're effectively saying 1V - 1V = 0V. Not a very enlightening statement and the spirit behind Lewin's "5 + 3 - 8 = 0" video.

I'm wondering if maybe this will help. Here I've rearranged Faraday's law for the bseechannel example to have the emf on left side (the source), and the load on the right side. Note both sides are still equal to E•dl and non-zero, and as always you can subtract the RHS side from both sides if you want to see zero there.
(https://ibin.co/w800/4RCvdo3HWTGP.png)

In addition to asking how you would complete Lewin's KVL (Int E•dl = 0) to make it correct (I agree that it is not universally correct), I think I should ask you to clarify exactly what it is that you think Lewin has done incorrectly so we can make some progress in understanding each other.
Title: Re: Does Kirchhoff's Law Hold? Disagreeing with a Master
Post by: ogden on December 25, 2018, 11:28:49 am
Of course it's incomplete, that's Lewin's point. And the way to complete it is to update it to Faraday's law. If you disagree I'd ask you how you would complete it.

That's the whole point - you cannot use incomplete equation to prove anything! Integral E.dl = 0 of Kirchoff's circuit rule includes *both* EMF source and load. Integral E.dl of Maxwell's equation includes/describes only EMF *source*. You completed it for me:

(https://ibin.co/w800/4RCvdo3HWTGP.png)

Quote
In addition to asking how you would complete Lewin's KVL (Int E•dl = 0) to make it correct (I agree that it is not universally correct), I think I should ask you to clarify exactly what it is that you think Lewin has done incorrectly so we can make some progress in understanding each other.

Move right side (load) of equation you just completed to left side and I am done showing where Dr.Lewin was wrong. It will be in front of your eyes contradicting with what you say in your blog:

(https://www.eevblog.com/forum/chat/does-kirchhoffs-law-hold-disagreeing-with-a-master/?action=dlattach;attach=605005;image)
Title: Re: Does Kirchhoff's Law Hold? Disagreeing with a Master
Post by: mhz on December 25, 2018, 12:46:00 pm
Of course it's incomplete, that's Lewin's point. And the way to complete it is to update it to Faraday's law. If you disagree I'd ask you how you would complete it.

That's the whole point - you cannot use incomplete equation to prove anything! Integral E.dl = 0 of Kirchoff's circuit rule includes *both* EMF source and load. Integral E.dl of Maxwell's equation includes/describes only EMF *source*. You completed it for me:

(https://ibin.co/w800/4RCvdo3HWTGP.png)

Quote
In addition to asking how you would complete Lewin's KVL (Int E•dl = 0) to make it correct (I agree that it is not universally correct), I think I should ask you to clarify exactly what it is that you think Lewin has done incorrectly so we can make some progress in understanding each other.

Move right side (load) of equation you just completed to left side and I am done showing where Dr.Lewin was wrong. It will be in front of your eyes contradicting with what you say in your blog:

If I move the right side of the equation to the left hand side, it will look like this:

(https://ibin.co/w800/4RDW0KhA0NN6.png)

So what?

It doesn't contradict anything I wrote in that blog. The point there is that something can't be simultaneously zero and non zero. This is how Lewin points out that EMFs don't have to come from batteries and then introduces Faraday's law. Maybe it will help if you look at this to get the full picture:

(https://ibin.co/w800/4RDKta0bGGlv.png)

I feel like perhaps your disconnect is that you feel that the EMF in the Lewin circuit must be able to be part of the int E•dl term, but it never will be because it comes from a time varying magnetic flux. int E•dl is *not* (necessarily) the same thing as the sum of all EMFs and voltage drops in the loop! It is however *always* equal to the negative time rate of change of the magnetic flux, which in this demo is the *only* emf, i.e. the only thing causing anything to happen (this is Faraday's law, and I am just assuming all this time that you agree with Faraday's law, please let me know if this is not the case). Think of the changing magnetic flux as the source, and int E•dl as the way you can figure out what is happening in the load. In this circuit we've been talking about int E•dl is nothing but currents through resistors. You can add a real battery in there and then you'll have two different kinds of EMF but that's not what we've been talking about up til now.

I'll ask once more for you to clearly articulate what it is that you think that Lewin got wrong. Please don't just respond to something I said here, state your objection clearly so that I can respond to it (tomorrow, we're in very different time zones). I'm still not totally clear on what your objection is but it feels like we might be able to come to an understanding.
Title: Re: Does Kirchhoff's Law Hold? Disagreeing with a Master
Post by: ogden on December 25, 2018, 01:40:36 pm
The point there is that something can't be simultaneously zero and non zero.

It can - when "something" is one thing in one case and completely different in another.

Take following as my objection you asked for: you cannot take in account law of conservation of energy in KVL case but ignore it in Maxwell's.

Shall I repeat & emphasize : Integral E.dl part of Kirchoff's circuit rule includes *both* EMF source and load. Integral E.dl of Maxwell's equation includes/describes only EMF *source*. You simply can't equal two (integrals), because they "look the same" (your words BTW). Following equation describes circuit of Dr.Lewin's experiment inner loop:

(https://ibin.co/w800/4RDW0KhA0NN6.png)

Don't you find it similar to equation of KVL you wrote?

(https://s0.wp.com/latex.php?latex=%5Coint+_%7Bsources%7D+%5Cvec+E+%5Ccdot+%5Cvec+dl+%2B+%5Coint+_%7Bloads%7D+%5Cvec+E+%5Ccdot+%5Cvec+dl%3D+0+&bg=ffffff&fg=333333&zoom=2)

I assume you agree to both. Me too.
Title: Re: Does Kirchhoff's Law Hold? Disagreeing with a Master
Post by: mhz on December 25, 2018, 08:52:51 pm

OK now we're getting somewhere, i think (i hope!). If I understand you correctly, you believe that Faraday's law implies a violation of conservation of energy!

"you cannot take in account law of conservation of energy in KVL case but ignore it in Maxwell's." I would call this a strawman, unless you can point me to where Lewin said that Maxwell's equations require ignoring conservation of energy.

It's his KVL that violates conservation of energy in his experiment because it can't possibly account for the source of energy as it knows nothing about magnetic flux!

I'm not sure I'm going to be able to bring you over without repeating things I've already said in the past few posts, but I'll try.

The point there is that something can't be simultaneously zero and non zero.

It can - when "something" is one thing in one case and completely different in another.
but.. it's not. int E•dl is int E•dl is int Ed•dl. It has the same meaning in both Lewin's KVL and Faraday's law. It means you go around a loop adding up each incremental bit of E field you encounter along your path. It doesn't matter whether it's an E field from a battery or an E field in a resistor or an E field arising from a changing magnetic flux. Do you believe that int E•dl has somehow a different meaning in the two equations?

Quote
Take following as my objection you asked for: you cannot take in account law of conservation of energy in KVL case but ignore it in Maxwell's.

Shall I repeat & emphasize : Integral E.dl part of Kirchoff's circuit rule includes *both* EMF source and load. Integral E.dl of Maxwell's equation includes/describes only EMF *source*.

I would say that as applied to this experiment, Faraday's law equates Int E•dl with the source (negative time rate of change of magnetic flux through the loop) of energy. It seems like you are unwilling to accept an EMF that doesn't arise from a source that is clearly related to an E field being maintained between two points, and that's what's causing you trouble, but that's the reality that Faraday describes.

Quote
You simply can't equal two (integrals), because they "look the same" (your words BTW).

I think you are misunderstanding my point. Let's use completely different symbols to hopefully make it clear. Let's look at two equations:

Y = 0

Y = F(x)

In equation one, we are saying that Y = 0. In equation two we are saying Y is equal to some function of x. There is only one way both can be valid   and that is the case where F(x) = 0 for any value of x. But we are asserting that F(x) also takes on non-zero values. Therefore Y = 0 is not a universal relation but just one possible point on the real general case of Y = F(x). 0 is just one possible value in the range of the function.

If it helps, think of Y as your height above the ground, and F(x) as a function that gives your current height above the ground. You might spend days, weeks or months walking around thinking that F(x) always equals 0 because you're always firmly on the ground. Then you get in an elevator, or an airplane for the first time. F(x) is no longer zero, and you must conclude that Y = 0 is a special case and not a general relation. This is mathematically, logically the same argument with different symbols.


Quote
Following equation describes circuit of Dr.Lewin's experiment inner loop:

(https://ibin.co/w800/4RDW0KhA0NN6.png)

Don't you find it similar to equation of KVL you wrote?

(https://s0.wp.com/latex.php?latex=%5Coint+_%7Bsources%7D+%5Cvec+E+%5Ccdot+%5Cvec+dl+%2B+%5Coint+_%7Bloads%7D+%5Cvec+E+%5Ccdot+%5Cvec+dl%3D+0+&bg=ffffff&fg=333333&zoom=2)

I assume you agree to both. Me too.

Yes, and I had to use Faraday's law to get there. Lewin's KVL could not have done it. That is one of his points.

[Edit: few clarifying words]
[Edit2: fix broken quoting]
Title: Re: Does Kirchhoff's Law Hold? Disagreeing with a Master
Post by: ogden on December 25, 2018, 09:19:18 pm

OK now we're getting somewhere, i think (i hope!). If I understand you correctly, you believe that Faraday's law implies a violation of conservation of energy!

"you cannot take in account law of conservation of energy in KVL case but ignore it in Maxwell's." I would call this a strawman, unless you can point me to where Lewin said that Maxwell's equations require ignoring conservation of energy.

It's his KVL that violates conservation of energy in his experiment because it can't possibly account for the source of energy as it knows nothing about magnetic flux!

Seems, you did not get it or just pretend that you do not understand what I did mean with that sentence.

Ok. Next try. I do not talk about abstract KVL and Maxwell equation "cases". I talk about equations that describes circuit of experiment. Everything seemingly is ok with KVL simple int E.dl = 0, yet I would prefer to split it into EMF source and load, as you already did - thank you for that. Problem arises when Dr.Lewin use plain Maxwell's equation and say that it miraculously tells everything about inner loop of his experiment. I disagree. Maxwell's equation is just EMF source part! Where's physics of load (resistors) in Maxwells equation? If you leave it like that, then it is indeed violation of conservation of energy. Plain Maxwell's equation can be used only to describe superconductive ring (w/o embedded resistors) placed in changing magnetic flux.

Quote
I would say that as applied to this experiment, Faraday's law equates Int E•dl with the source (negative time rate of change of magnetic flux through the loop) of energy. It seems like you are unwilling to accept an EMF that doesn't arise from a source that is clearly related to an E field being maintained between two points, and that's what's causing you trouble, but that's the reality that Faraday describes.

Oh my. You better take care of your own troubles first, ok? Seems, you are unwilling to accept that EMF energy is dissipated in the embedded resistors of the inner loop.
Title: Re: Does Kirchhoff's Law Hold? Disagreeing with a Master
Post by: mhz on December 25, 2018, 09:36:00 pm
If you don't believe Maxwell's equations always apply, or you think they violate conservation of energy, then I'm afraid we can't go anywhere from there. Maxwell's equations are axiomatic to any discussion about electricity and magnetism regardless of circuit configuration (regardless of whether the elements of the circuit can be  represented as lumped elements or not) and they always hold[1].

Still, thank you for the conversation. It's been helpful to me.

[edit: added bit in the parenthesis]
[edit2: minor clarifying edit]
[edit3: add bit about QED]

[1] Except, apparently in the domain of Quantum Electrodynamics, which is not what we're talking about here.
Title: Re: Does Kirchhoff's Law Hold? Disagreeing with a Master
Post by: ogden on December 26, 2018, 04:13:24 am
If you don't believe Maxwell's equations always apply, or you think they violate conservation of energy, then I'm afraid we can't go anywhere from there.

LOL. Here we go. Again. When out of arguments - just state that debate opponent does not understand Maxwell's equations.  :horse:

I never said or implied that I do not believe Maxwell's equations. Also I never said that they violate law of conservation energy. What I did say that you can't use EMF source equation alone to describe system of EMF source and load. If you cannot comprehend such a simple concept then further discussion indeed is pointless. Funny that you even did write proper equation which shall be used to describe/analyze circuit of experiment against "KVL apply to circuit or not", but when you most likely realized that it would destroy your whole system of beliefs/reasoning, you decided to completely ignore everything about it:

(https://ibin.co/w800/4RDW0KhA0NN6.png)
Title: Re: Does Kirchhoff's Law Hold? Disagreeing with a Master
Post by: mhz on December 26, 2018, 04:37:12 am
Quote
LOL. Here we go. Again. When out of arguments - just state that debate opponent does not understand Maxwell's equations.  :horse:

I never said or implied that I do not believe Maxwell's equations. Also I never said that they violate law of conservation energy.

You said:
"Maxwell's equation is just EMF source part! Where's physics of load (resistors) in Maxwells equation? If you leave it like that, then it is indeed violation of conservation of energy. Plain Maxwell's equation can be used only to describe superconductive ring (w/o embedded resistors) placed in changing magnetic flux."

I can't go anywhere from there. If you think that you can write something like "Plain Maxwell's equation can be used only to describe ..." then either you think that there are more fundamental equations or you simply don't understand how they work. Maxwell's equations are the starting point. Everything else can be derived and approximated from them. This is accepted science for the last 100+ years.

It's clear that you disagree with me and that's fine. I'm willing to end this discussion in disagreement. I don't think there's anything left for me to say, but as I said I appreciate the discussion.

p.s. the load (resistors) are in the int E•dl. it's all up there in the equations i've already posted.
[edit: p.p.s. i'd been waiting for the emoticons to come out. not a device i'd employ if I wanted to be taken seriously]
Title: Re: Does Kirchhoff's Law Hold? Disagreeing with a Master
Post by: ogden on December 26, 2018, 04:51:14 am
I can't go anywhere from there.

Yes. Please. Stop this :blah: nonsense of pretending that you do not understand what I mean. Our discussion looks like broken record.
Title: Re: Does Kirchhoff's Law Hold? Disagreeing with a Master
Post by: bsfeechannel on December 26, 2018, 05:33:29 am
LOL. Here we go. Again. When out of arguments - just state that debate opponent does not understand Maxwell's equations.  :horse:

You don't. But that's not your fault. Maxwell's equations show how Nature is much weirder than we may conceive. You'll have to reboot your brain to understand it. Just give it time. We all had our Maxwell crisis.

You're just having yours in public.
Title: Re: Does Kirchhoff's Law Hold? Disagreeing with a Master
Post by: mhz on December 26, 2018, 05:47:23 am
LOL. Here we go. Again. When out of arguments - just state that debate opponent does not understand Maxwell's equations.  :horse:

You don't. But that's not your fault. Maxwell's equations show how Nature is much weirder than we may conceive. You'll have to reboot your brain to understand it...

Or in my case literally spend a few months of sleepless nights reading/watching everything on the topic. There's still a gremlin in my understanding but I'll start a different thread for that one.

I can't help but feel all of the drama around this could have been avoided if electronics educators did a better job of adding caveats and asterisks to their materials and explanations. I went to UC Berkeley and I'm pretty sure not a single professor ever uttered the words "lumped circuit abstraction" in my entire undergrad career. Agarwal is doing God's (or rather Feynman's) work.

Title: Re: Does Kirchhoff's Law Hold? Disagreeing with a Master
Post by: electrodacus on December 26, 2018, 05:55:53 am
I can't go anywhere from there.

Yes. Please. Stop this :blah: nonsense of pretending that you do not understand what I mean. Our discussion looks like broken record.

Do not waste your time.
Title: Re: Does Kirchhoff's Law Hold? Disagreeing with a Master
Post by: bsfeechannel on December 26, 2018, 08:47:35 am
I can't help but feel all of the drama around this could have been avoided if electronics educators did a better job of adding caveats and asterisks to their materials and explanations. I went to UC Berkeley and I'm pretty sure not a single professor ever uttered the words "lumped circuit abstraction" in my entire undergrad career. Agarwal is doing God's (or rather Feynman's) work.

Exactly.

Kirchhoff is normally taught in high school. Maxwell is mentioned only en passant. No one says that Maxwell is the theory underlying Kirchhoff's laws. When you get to college, you are taught a lot of apparently meaningless and complicated math. No one says that this will be in preparation for Maxwell and other theories. Then they teach you electromagnetism mercilessly without explaining that the fluxing Kirchhoff is a special case of Maxwell.

Lewin is a critic of that way of teaching, given how that cripples understanding the basic concepts.

We could suggest a change. Teach a simplified version of calculus and vector analysis in high school. This is perfectly possible. My video "Calculus for young players" is aimed at high-schoolers. Then teach the basics of Maxwell, explaining that Kirchhoff is a special case of that theory.

When at college, they should say, now that you are going to be an engineer, you will be responsible for designing serious things. So you are going to learn this stuff with all the rigor these theories require. You'll also learn circuit analysis with even more detail and rigor, always having in mind that it is a special case of electromagnetism.

That would avoid the anger everybody feels when they discover the truth years later.
Title: Re: Does Kirchhoff's Law Hold? Disagreeing with a Master
Post by: Berni on December 26, 2018, 04:48:57 pm
This is proabobly the most important realization you made Mhz:
https://grumpyengineering.wordpress.com/
Quote
If you still disagree with the above I would love to hear from you, but I believe, most of the confusion results from disagreements about what KVL is.

Nobody in this thread is trying to prove that Maxwells equations are wrong, stop blaming people for that.

Most of the arguing between the "Kirchoff" and "Maxwell" sides is due to both sides having a different idea of what KVL is. I fully agree that if KVL is what Dr. Lewin explains it as being then its garbage as soon as you have changing magnetic fields. I have no idea where he got that definition of KVL, everywhere i see it defined as the flowing:
Quote
The algebraic sum of all the voltages around any closed loop in a circuit is equal to zero

It explicitly mentions all voltages, this includes the electric field as well as the EMF (I think we all agree EMF is a voltage). Also notice that it mentions an algebraic sum (This is not an integral!) since KVL is not a law of the universe but a tool for analyzing circuit mesh models.

Both the "Kirchoff" and "Maxwell" sides are correct! The only only reason that we are arguing is because the so called "Maxwell" side is using a less useful interpretation of KVL. The people on the "Kirchoff" side are not denying anything about Maxwells equations, the only thing this side is trying to say is that you can use KVL to solve Lewins paradoxic circuit just fine i you use KVL correctly. You indeed CAN NOT USE Kirhoffs laws for everything, but this particular circuit is not such a case.
 
The "Kirchoff" side should actually be named "Kirchoff and Maxwell" side. We are happy to use one or the other rather than swear by Maxwell only, just a matter of the right tool for the job. Is there something wrong with that?
Title: Re: Does Kirchhoff's Law Hold? Disagreeing with a Master
Post by: bsfeechannel on December 26, 2018, 07:08:27 pm
Nobody in this thread is trying to prove that Maxwells equations are wrong

But that is exactly what you do when you say that you can solve Lewin's circuit with Kirchhoff. Which, by the way, is not Lewin's. You find that circuit anywhere because it defines the phenomenon of induction.

Your claims lead immediately to a logical contradiction which indicates that your reasoning is false.  And that's why people keep "blaming" you, or better, warning you.
Title: Re: Does Kirchhoff's Law Hold? Disagreeing with a Master
Post by: Berni on December 26, 2018, 09:24:13 pm
Circuit analysis makes use of equations derivated from Maxwells equations in order to make inductors and capacitors work with Kirchoffs circuit laws. Is there some law that forbids the use of Maxwells equations in anything else but there raw form? Somehow making anything they are used in automatically wrong even if it gives the same result?

So is it a problem that Kirhhoffs cirucit laws work in lumped circuit meshes? Or is it a problem that circuit modeling has to be used to link the real world with circuit meshes?
Title: Re: Does Kirchhoff's Law Hold? Disagreeing with a Master
Post by: mhz on December 26, 2018, 10:02:26 pm
This is proabobly the most important realization you made Mhz:
https://grumpyengineering.wordpress.com/
Quote

    If you still disagree with the above I would love to hear from you, but I believe, most of the confusion results from disagreements about what KVL is.


Nobody in this thread is trying to prove that Maxwells equations are wrong, stop blaming people for that.
On the contrary, that's exactly where the previous discussion with ogden led, with them concluding that Maxwell's equations are only correct in specific situations (superconducting rings, I believe is what was said). That's another way of saying that sometimes they are wrong.

Quote
Most of the arguing between the "Kirchoff" and "Maxwell" sides is due to both sides having a different idea of what KVL is. I fully agree that if KVL is what Dr. Lewin explains it as being then its garbage as soon as you have changing magnetic fields. I have no idea where he got that definition of KVL, everywhere i see it defined as the flowing:
Quote

    The algebraic sum of all the voltages around any closed loop in a circuit is equal to zero

the problem with this definition, what I keep calling the "modified" KVL, which is what I think most of us think of as the "real" KVL,  is it starts to break down in situations where you're dealing with nonlumped elements. As I mentioned in the blog post, there is nothing keeping us from modeling things with lumped elements (we'd have to add a mutual conductance in the inner loop) to get thngs to sum to zero in a nice KVL way but at the expense of incorrectly localizing the effect.

Quote
It explicitly mentions all voltages, this includes the electric field as well as the EMF (I think we all agree EMF is a voltage). Also notice that it mentions an algebraic sum (This is not an integral!) since KVL is not a law of the universe but a tool for analyzing circuit mesh models.

right! the algebraic sum is the key to pointing out that it only works with lumped circuits, or circuits modeled with lumped elements.

Quote
Both the "Kirchoff" and "Maxwell" sides are correct! The only only reason that we are arguing is because the so called "Maxwell" side is using a less useful interpretation of KVL. The people on the "Kirchoff" side are not denying anything about Maxwells equations, the only thing this side is trying to say is that you can use KVL to solve Lewins paradoxic circuit just fine i you use KVL correctly. You indeed CAN NOT USE Kirhoffs laws for everything, but this particular circuit is not such a case.

I'd invite you to redraw the diagram and use the "modified" KVL on this circuit. See if you can model it in a way that doesn't falsely localize the effect of the mutual inductance but that allows you to get the correct answer with the "modified" KVL. It would help if you post the drawing of the updated model for us.

Quote
The "Kirchoff" side should actually be named "Kirchoff and Maxwell" side. We are happy to use one or the other rather than swear by Maxwell only, just a matter of the right tool for the job. Is there something wrong with that?

Nothing wrong with that at all. The real lesson of all this, to me at least, is that the "modified" KVL only works with lumped circuits. If Lewin had introduced the concept of lumped circuit analysis (as described expertly by Feynman and Agarwal) explicitly after the super demo I think there would be probably be a lot less confusion.
Title: Re: Does Kirchhoff's Law Hold? Disagreeing with a Master
Post by: Berni on December 26, 2018, 10:28:09 pm
Quote
I'd invite you to redraw the diagram and use the "modified" KVL on this circuit. See if you can model it in a way that doesn't falsely localize the effect of the mutual inductance but that allows you to get the correct answer with the "modified" KVL. It would help if you post the drawing of the updated model for us.

I already did that in the very first few posts on this thread:
https://www.eevblog.com/forum/chat/does-kirchhoffs-law-hold-disagreeing-with-a-master/msg1945312/#msg1945312 (https://www.eevblog.com/forum/chat/does-kirchhoffs-law-hold-disagreeing-with-a-master/msg1945312/#msg1945312)

You simply calculate the contributions each wire segment is providing and distribute the inductance accordingly (Note these are all coupled inductors since the all interact with the same flux). This way only parts of the circuit that are not interesting get lumped up while preserving all the points of interest along with there voltages. If you suddenly want to know whats happening half way along a wire you simply represent it as two lumps with correct proportions to expose the point we want.

Calculating the inductance is easy in this case because the circuit is very symmetrical and there is high coupling between the wires due to them flowing the same path. Had the wires followed arbitrary complex 3D paths trough space it would have been much harder to model. We would need to go trough Maxwell with fancy convoluted 3D space intergals in order to get the magical inductance values for the equivalent circuit. Or if we have access to the physical circuit simply measure them with a impedance analyzer and plonk the numbers into the equivalent circuit. No matter what the process is you end up with a cirucit mesh that acts like the real cirucit, but can be used with KVL just fine since its a lumped circuit mesh.

Modeling has nothing to do with KVL or even electronics. Its simply a process of turning a real thing into math that acts like the real thing so that we can run calculations or simulations on it. Its done with all sorts of things, be it mechanical, structural, thermal etc. These models all have there limitations. Just as a thermal model of a house will not be accurate if you try to simulate ambient being 10 000 °C in the same way your equivalent circuit will not be accurate at RF frequencies unless specifically modeled for it.
Title: Re: Does Kirchhoff's Law Hold? Disagreeing with a Master
Post by: ogden on December 26, 2018, 11:14:07 pm
On the contrary, that's exactly where the previous discussion with ogden led, with them concluding that Maxwell's equations are only correct in specific situations (superconducting rings, I believe is what was said). That's another way of saying that sometimes they are wrong.

Typical strategy of forum trolls and "sofa experts" BTW - take just single sentence out of context, twist it as much as possible, draw conclusions out of this new "twisted meaning" to prove own agenda.

For those unaware this is what I ACTUALLY said:

I do not talk about abstract KVL and Maxwell equation "cases". I talk about equations that describes circuit of experiment. Everything seemingly is ok with KVL simple int E.dl = 0, yet I would prefer to split it into EMF source and load, as you already did - thank you for that. Problem arises when Dr.Lewin use plain Maxwell's equation and say that it miraculously tells everything about inner loop of his experiment. I disagree. Maxwell's equation is just EMF source part! Where's physics of load (resistors) in Maxwells equation? If you leave it like that, then it is indeed violation of conservation of energy. Plain Maxwell's equation can be used only to describe superconductive ring (w/o embedded resistors) placed in changing magnetic flux.
Title: Re: Does Kirchhoff's Law Hold? Disagreeing with a Master
Post by: Psi on December 26, 2018, 11:17:06 pm
I disagree with the thread title.

Walter Lewin used to be a master. Then he started flinging poo at good people.
So i vote we take Master status from him.  :-DD
Title: Re: Does Kirchhoff's Law Hold? Disagreeing with a Master
Post by: beanflying on December 26, 2018, 11:21:47 pm
I disagree with the thread title.

Walter Lewin used to be a master. Then he started flinging poo at good people.
So i vote we take Master status from him.  :-DD

So is that why this thread travels toward flinging poo AGAIN emulating the Master?  :palm:

The last few pages and a lot of the thread have been really interesting and in between the poo this thread is more informative than my Uni lecturers way back in the dim dark past ever were.
Title: Re: Does Kirchhoff's Law Hold? Disagreeing with a Master
Post by: mhz on December 26, 2018, 11:31:56 pm
@berni Ah, perfect. Ok so my point is this, you've modeled the circuit with lumped elements and that gives something that looks correct, but if you set this experiment up in real life, do you really think you'll measure any significant voltage across any of L2, L3, L4 or L5 like spice would tell you is there? The coupled flux isn't confined in those points so you won't. This is where our "modified" KVL breaks down.

We saw Mehdi struggle with this in his first video. This is one of the biggest points of confusion, that there must be "voltage in the wire". I don't remember who, bsfeechannel?, was earlier arguing that this can't be modeled 100% correctly in spice because spice only knows about lumped elements. No matter how many inductors you split the mutual inductance into in spice, it will give an incorrect answer if you use it try to use it to measure a voltage along the wire. The reason is that in the actual experiment, there aren't any lumped inductors, the linked flux in the secondary is not confined to any specific two terminals.

Also, I'd argue that the leads should not be modeled as mutual inductance as they aren't supposed to link any of the flux in the center loop. (This is how Romer defines the experiment).

p.s. I'm not going to be bothering to respond to others that aren't attempting to have a productive conversation

[edit: clarify who I'm talking to since a few posts happened in the interim]
Title: Re: Does Kirchhoff's Law Hold? Disagreeing with a Master
Post by: ogden on December 26, 2018, 11:58:55 pm
Ok so my point is this, you've modeled the circuit with lumped elements and that gives something that looks correct, but if you set this experiment up in real life, do you really think you'll measure any significant voltage across any of L2, L3, L4 or L5 like spice would tell you is there? The coupled flux isn't confined in those points so you won't.

So you say that Kirk T. McDonald is wrong in chapter "2.3 Comments" (page 10) (http://www.physics.princeton.edu/~mcdonald/examples/lewin.pdf)? Please tell where and why he is wrong. Prove him wrong.

Excerpt:

(https://www.eevblog.com/forum/chat/does-kirchhoffs-law-hold-disagreeing-with-a-master/?action=dlattach;attach=605980;image)
Title: Re: Does Kirchhoff's Law Hold? Disagreeing with a Master
Post by: rfeecs on December 27, 2018, 12:48:13 am
Ok so my point is this, you've modeled the circuit with lumped elements and that gives something that looks correct, but if you set this experiment up in real life, do you really think you'll measure any significant voltage across any of L2, L3, L4 or L5 like spice would tell you is there? The coupled flux isn't confined in those points so you won't.

So you say that Kirk T. McDonald is wrong in chapter "2.3 Comments" (page 10) (http://www.physics.princeton.edu/~mcdonald/examples/lewin.pdf)? Please tell where and why he is wrong. Prove him wrong.

Excerpt:

(https://www.eevblog.com/forum/chat/does-kirchhoffs-law-hold-disagreeing-with-a-master/?action=dlattach;attach=605980;image)

Did you even read that section?
Quote
Suppose a voltmeter were connected to two points on the upper wire between resistors 1
and 2, as shown in the sketch below. The voltmeter loop is not coupled to the solenoid, so
there is no (or extremely little) EMF induced in this loop, and hence I1 = 0, and the meter
reading would be Vmeter = 0.

This agrees with what @mhz said.

In that section, Dr. McDonald is pointing out that a voltmeter does not read the difference in scalar potential.  He is making up his own definitions of "voltage drop" and "EMF" in the presence of a changing magnetic field.
Title: Re: Does Kirchhoff's Law Hold? Disagreeing with a Master
Post by: ogden on December 27, 2018, 12:49:54 am
Do you see the above two equations as equivalent term by term?

I did say "Don't you find it similar". I did not say equivalent, especially term by term. One more illustration of troll & "coach expert" tactics. You just cherrypick out of context whatever you find convenient for you, ignoring what I was ACTUALLY talking about: circuit that has EMF source and load, that both equations describes such and are similar in such sense.
Title: Re: Does Kirchhoff's Law Hold? Disagreeing with a Master
Post by: Sredni on December 27, 2018, 12:53:11 am
Besides, the difference in Fermi levels is the barrier potential (if we agree on how to treat the sign). I guess [NOTE: I should have written 'thought', instead of 'guessed') you were the one saying that you cannot read it with a voltmeter.
With no bias, in equilibrium, the Fermi levels on both terminals are equal, so zero voltage.  Refer to figure 7.3 of Neamen.
I was talking about the Fermi levels of the separated materials, what Neamen call 'intrinsic Fermi levels' when referring to the compound structure. At thermodynamic equilibrium there are no longer Fermi levelS. But yes, out of equilibrium the one Fermi level of the structure splits in separate levels and the voltage corresponding to that difference is what is measured.
Quote
You have to wonder if "potential barrier" is the right phrase for an ohmic contact, which should have little or no barrier.

I guess it is the right denomination, since there are potential barriers nonetheless. In some cases they are not very hard to overcome thanks to the field associated with the charge developed at the contact surface , in others they are very high and steep, but electrons can easily tunnel through them. See Muller and Kamins for an explanation of both versions (section 3.4 in the second edition).

Quote
So what's your point?  Are you saying that there is no net electrostatic potential across the diode terminals?

I am saying that whatever potential is there, it will be canceled by the contact potentials created by the placement of the probes.
But anyway, by putting the limelight on other sources of emf, you raised an interesting point that might benefit the original discussion: could it be that the emf due to changing dB/dt is different from all other emfs? Faraday's law seems to point there.
Title: Re: Does Kirchhoff's Law Hold? Disagreeing with a Master
Post by: ogden on December 27, 2018, 12:54:51 am
Did you even read that section?

Did you? Carefully enough?

Quote
Quote
Suppose a voltmeter were connected to two points on the upper wire between resistors 1
and 2, as shown in the sketch below. The voltmeter loop is not coupled to the solenoid, so
there is no (or extremely little) EMF induced in this loop, and hence I1 = 0, and the meter
reading would be Vmeter = 0.

This agrees with what @mhz said.

In that section, Dr. McDonald is pointing out that a voltmeter does not read the difference in scalar potential.

From which planet did you just arrive? We are not talking about scalar potentials but EMF. @mhz said that voltmeter will not measure any EMF as Spice do.

Title: Re: Does Kirchhoff's Law Hold? Disagreeing with a Master
Post by: Berni on December 27, 2018, 12:56:08 am
To not repeat a certain arrangement i will also answer this for both definitions of voltage:

A) For definition "Voltage is the integral of all forces pushing on electrons along a given path connecting two points":
The inductors L2 L3 L4 L5 have zero voltage across them at all times (Zero resistance). Any EMF voltage induced in the wire by the magnetic field is instantly countered by the charge separation of electrons.

B) For definition "Voltage is the difference in charge density between two points" (This is what real life voltmeters show)
The inductors L2 L3 L4 L5 have a voltage drop that sums up to the same voltage as the total voltage drop on the resistors.  This voltage in the wire is caused by charge separation pushing electrons towards one end of a wire, resulting in more electrons on one end hence higher voltage on one end.



The proper textbook definition (A) is the one that is used in Dr. Lewins example where he gets two different voltages across the same two points. This is fully correct and there are indeed two voltages there. The reason for the two solutions is that this voltage is including the EMF voltage from the magnetic field, yet the loop is not closed yet as the two points are in different locations in space. Depending on how this path is closed results in a different solution for the EMF voltage and this changes the result. Hence why voltage is path dependent.

So why are we using the other definition (B) if its clearly wrong? Well turns out in real life its rather tricky to measure the voltage according to that definition. Electrical components (such as resistors) represent only a small part of the path around the magnetic fields loop area, because of this all of the electron pushing work is done by the electrical field (caused by charge seperation). Turns out all the voltmeters are actually devices that measure current trough the internal resistance and display the voltage required to push that current. The density of electrons at a point in space can only have 1 single defined value hence why these voltages always have one value rather than multiple. All of this simply makes this definition (B) more useful and as such is used in circuit analysis and spice simulations. Since circuit analysis uses it that forces KVL to use it too.

In the absence of a changing magnetic field around the circuit both definitions of voltage have the same value so it doesn't matter what you use. But in a magnetic field it does matter a lot.

As for the lumped model, it only hides what you want it to hide. Many many tiny inductors in series act the same as one big inductor, so it makes it easier to use a lumped version. Once you lump a segment of a circuit all voltages within the lumped part become meaningless, this is why lumping the inductor as a single one causes problems in this example. The lumping procedure also lumped all our points of interest and messed them up, they no longer show true voltages. However anything outside the area we just lumped is preserved. The rest of the circuit doesn't care how many inductors there are, it just sees a set inductance value across the points. So by only lumping sections of the wire that have no points of interest we preserve all the points we want to measure. Hence why all the points on the ends of the inductors have the correct voltage values.

All wires have some amount of mutual inductance to each other as long as they are not placed at perfect right angles. In this case there is more to it however. The inductors L2 L3 L4 L5 are actually a single inductor (single whole turn of the loop) that has been sliced up in to 4 parts. Because they are part of the same inductors means they share the same magnetic flux and hence are highly coupled inductors. The inductors L6 L7 L8 L9 are another inductor that has been sliced up in to quarters, but since the wire follows the same path as the inner cirucit means that any flux passing trough that loop passes trough this one too. This means all of them are coupled to each other (aka a transformer). The solenoid coil in the middle is also having the same magnetic flux pass trough it hence why its also coupled. In my simulation it has a ideal coupling coefficient of 1, but in reality it would be lower because solenoid is smaller than the loop so some of the flux escapes.

So yeah we are mostly looking at two sides of the same coin here. Its two different ways of explaining the same thing.
Title: Re: Does Kirchhoff's Law Hold? Disagreeing with a Master
Post by: Sredni on December 27, 2018, 01:17:56 am
Do you see the above two equations as equivalent term by term?

I did say "Don't you find it similar". I did not say equivalent, especially term by term. One more illustration of troll & "coach expert" tactics. You just cherrypick out of context whatever you find convenient for you, ignoring what I was ACTUALLY talking about: circuit that has EMF source and load, that both equations describes such and are similar in such sense.

I am just trying to pinpoint what I believe to be the origin of your misconception.
Back in one of your exchanges with MHz, you wrote:

Quote
Integral E.dl = 0 of Kirchoff's circuit rule includes *both* EMF source and load. Integral E.dl of Maxwell's equation includes/describes only EMF *source*.

Have you ever tried to compute the integral of E.dl of a RLC circuit with a generator - a lumped circuit, just to see that Kirchhoff and field theory can agree if there is no dB/dt area enclosed by the circuit path? It might clear a lot of things up before trying to attack a non-lumped circuit such as Lewin's.
Title: Re: Does Kirchhoff's Law Hold? Disagreeing with a Master
Post by: ogden on December 27, 2018, 01:29:25 am
Have you ever tried to compute the integral of E.dl of a RLC circuit with a generator

No. Have you?

I use L(di/dt) with real inductors/transformers having inductance and saturation current specs. Works for me well.
Title: Re: Does Kirchhoff's Law Hold? Disagreeing with a Master
Post by: mhz on December 27, 2018, 01:35:15 am
Ok so my point is this, you've modeled the circuit with lumped elements and that gives something that looks correct, but if you set this experiment up in real life, do you really think you'll measure any significant voltage across any of L2, L3, L4 or L5 like spice would tell you is there? The coupled flux isn't confined in those points so you won't.

So you say that Kirk T. McDonald is wrong in chapter "2.3 Comments" (page 10) (http://www.physics.princeton.edu/~mcdonald/examples/lewin.pdf)? Please tell where and why he is wrong. Prove him wrong.

Excerpt:

(https://www.eevblog.com/forum/chat/does-kirchhoffs-law-hold-disagreeing-with-a-master/?action=dlattach;attach=605980;image)


I didn't say that I disagree with McDonald. Unlike many of us here, I'm not generally inclined to disagree with Physics professors. As rfeecs pointed out, McDonald echoes what I said to Berni, that the voltmeter will measure zero (whereas in his spice model, with the loop inductance split up into four lumped inductors, measuring across the inductor will give a nonzero value). 

To be honest, some of McDonald's writing is presented at a level beyond the one I'm at. I'm going to work through Griffiths and hopefully come back to it able to fully digest what he has to say. That said, his footnote number 8 in http://physics.princeton.edu/~mcdonald/examples/voltage.pdf (http://physics.princeton.edu/~mcdonald/examples/voltage.pdf) seems to be expressing what I've been trying to get across in my discussion with Berni.
Title: Re: Does Kirchhoff's Law Hold? Disagreeing with a Master
Post by: ogden on December 27, 2018, 01:44:52 am
As rfeecs pointed out, McDonald echoes what I said to Berni, that the voltmeter will measure zero

He says exactly opposite: "the result Vmeter = 0 is appealing in that we might naïvely expect the “voltage drop” to be zero between points along a good/perfect conductor.".

You really shall read last paragraph of mentioned chapter carefully.
Title: Re: Does Kirchhoff's Law Hold? Disagreeing with a Master
Post by: Sredni on December 27, 2018, 01:45:20 am
Have you ever tried to compute the integral of E.dl of a RLC circuit with a generator

No. Have you?

Yes. It clears things up. A lot.
You should too, before embarking in discussions such as this.
Luckily for you, you can find a well presented walk-through in Ramo, Whinnery and VanDuzer "Fields and Waves in Communication Electronics". Chapter 4, "The electromagnetics of circuits".
Find a library that has this book, and read the first few pages of chapter 4.

Quote
I use L(di/dt) with real inductors/transformers having inductance and saturation current specs.

You should try to see where that expression comes from.

Quote
Works for me well.

Does not look like that, from this side of the monitor.
Title: Re: Does Kirchhoff's Law Hold? Disagreeing with a Master
Post by: ogden on December 27, 2018, 01:54:35 am
Does not look like that, from this side of the monitor.

You shall visit optometrist then. Thank you for suggestions anyway ;)
Title: Re: Does Kirchhoff's Law Hold? Disagreeing with a Master
Post by: mhz on December 27, 2018, 02:10:03 am
To not repeat a certain arrangement i will also answer this for both definitions of voltage:

A) For definition "Voltage is the integral of all forces pushing on electrons along a given path connecting two points":
The inductors L2 L3 L4 L5 have zero voltage across them at all times (Zero resistance). Any EMF voltage induced in the wire by the magnetic field is instantly countered by the charge separation of electrons.

B) For definition "Voltage is the difference in charge density between two points" (This is what real life voltmeters show)
The inductors L2 L3 L4 L5 have a voltage drop that sums up to the same voltage as the total voltage drop on the resistors.  This voltage in the wire is caused by charge separation pushing electrons towards one end of a wire, resulting in more electrons on one end hence higher voltage on one end.

Where does definition B come from? Charge density and Voltage don't even have the same units. [C/m3] vs [J/C]

Quote
The proper textbook definition (A) is the one that is used in Dr. Lewins example where he gets two different voltages across the same two points. This is fully correct and there are indeed two voltages there. The reason for the two solutions is that this voltage is including the EMF voltage from the magnetic field, yet the loop is not closed yet as the two points are in different locations in space. Depending on how this path is closed results in a different solution for the EMF voltage and this changes the result. Hence why voltage is path dependent.

So why are we using the other definition (B) if its clearly wrong? Well turns out in real life its rather tricky to measure the voltage according to that definition. Electrical components (such as resistors) represent only a small part of the path around the magnetic fields loop area, because of this all of the electron pushing work is done by the electrical field (caused by charge seperation). Turns out all the voltmeters are actually devices that measure current trough the internal resistance and display the voltage required to push that current. The density of electrons at a point in space can only have 1 single defined value hence why these voltages always have one value rather than multiple. All of this simply makes this definition (B) more useful and as such is used in circuit analysis and spice simulations. Since circuit analysis uses it that forces KVL to use it too.

In the absence of a changing magnetic field around the circuit both definitions of voltage have the same value so it doesn't matter what you use. But in a magnetic field it does matter a lot.

As for the lumped model, it only hides what you want it to hide. Many many tiny inductors in series act the same as one big inductor, so it makes it easier to use a lumped version. Once you lump a segment of a circuit all voltages within the lumped part become meaningless, this is why lumping the inductor as a single one causes problems in this example. The lumping procedure also lumped all our points of interest and messed them up, they no longer show true voltages. However anything outside the area we just lumped is preserved. The rest of the circuit doesn't care how many inductors there are, it just sees a set inductance value across the points. So by only lumping sections of the wire that have no points of interest we preserve all the points we want to measure. Hence why all the points on the ends of the inductors have the correct voltage values.

I disagree. You can split the total mutual inductance M of the loop into two strings of as many inductors as you want in spice. The value you measure in spice will not be the  actual scalar voltage potential between the ends of the resistors (which is approximately zero as measured by the voltmeter). Lumping can't be done in this kind of circuit  in spice without creating false outcomes.

Quote
All wires have some amount of mutual inductance to each other as long as they are not placed at perfect right angles. In this case there is more to it however. The inductors L2 L3 L4 L5 are actually a single inductor (single whole turn of the loop) that has been sliced up in to 4 parts. Because they are part of the same inductors means they share the same magnetic flux and hence are highly coupled inductors. The inductors L6 L7 L8 L9 are another inductor that has been sliced up in to quarters, but since the wire follows the same path as the inner cirucit means that any flux passing trough that loop passes trough this one too. This means all of them are coupled to each other (aka a transformer). The solenoid coil in the middle is also having the same magnetic flux pass trough it hence why its also coupled. In my simulation it has a ideal coupling coefficient of 1, but in reality it would be lower because solenoid is smaller than the loop so some of the flux escapes.

I see now that you're trying to model the mutual inductance of the "outer loop" i.e. the path formed by the two measurement loops, but not going through R1 and R2. You've arranged the coupling dots in a way that the inner inductors and outer inductors cancel each other out in a way that satisfies there being no flux coupling in the two measurement loops.

Quote
So yeah we are mostly looking at two sides of the same coin here. Its two different ways of explaining the same thing.

Mostly, with perhaps some disagreements on how far you can go with lumped models of inherently non-lumped reality, and the part I'm not following up there about charge density.

[Edit1: fix quoting]
Title: Re: Does Kirchhoff's Law Hold? Disagreeing with a Master
Post by: mhz on December 27, 2018, 02:26:03 am
As rfeecs pointed out, McDonald echoes what I said to Berni, that the voltmeter will measure zero

He says exactly opposite: "the result Vmeter = 0 is appealing in that we might naïvely expect the “voltage drop” to be zero between points along a good/perfect conductor.".

You really shall read last paragraph of mentioned chapter carefully.

I have been, and I have to admit I'm quite perplexed with it. In particular equation 35 doesn't seem correct to me. For example, if we assume a point in time where I = 1mA, R1 = 100ohm, R2 = 900ohm, R = 1Mohm and I1 ≈ 0 (as he states) then he seems to be saying 0V = -1V. If someone can help clear this up for me I'd appreciate it.

(https://ibin.co/4ROe8sZbEx99.png)

[Edit 1: fix typos]
Title: Re: Does Kirchhoff's Law Hold? Disagreeing with a Master
Post by: ogden on December 27, 2018, 02:42:58 am
I have been, and I have to admit I'm quite perplexed with it. In particular equation 35 doesn't seem correct to me. For example, if we assume a point in time where I = 1mA, R1 = 100ohm, R2 = 900ohm, R = 1Mohm and I1 ≈ 0 (as he states) then he seems to be saying 0V = -1V. If someone can help clear this up for me I'd appreciate it.

Equation is correct indeed. Thou it is counterintuitive. Wire segment "a-b" is part of *both* loops - loop of leads and loop containing R1 and R2. Leads loop does not have any EMF induced, source of "voltage drop" in particular wire segment is EMF generated only in the loop containing resistors and it is obviously mirrored in the a-b segment of the leads loop.  All this is not that important. Important part is: voltage will be observed which is contrary to your statement.

[edit] No, you dont'use Ohms law here. You shall use Maxwell's equation to calculate EMF generated in wire segment "a-b"

[edit1] Seems, I know where your frustration comes from. By saying I1=0 he means that current induced by EMF is zero because there is no EMF in the voltmeter leads. On the other hand current will be flowing through voltmeter due to potential difference "voltage drop" between points a & b. This is my explanation. Hope it helps.

[edit2] Obviously I agree that it is kinda incorrect to indicate I1=0, at the same time saying that voltmeter having finite resistance measures something that differs from 0V.
Title: Re: Does Kirchhoff's Law Hold? Disagreeing with a Master
Post by: mhz on December 27, 2018, 03:03:21 am
I have been, and I have to admit I'm quite perplexed with it. In particular equation 35 doesn't seem correct to me. For example, if we assume a point in time where I = 1mA, R1 = 100ohm, R2 = 900ohm, R = 1Mohm and I1 ≈ 0 (as he states) then he seems to be saying 0V = -1V. If someone can help clear this up for me I'd appreciate it.

Equation is correct indeed. Thou it is counterintuitive. Wire segment "a-b" is part of *both* loops - loop of leads and loop containing R1 and R2. Leads loop does not have any EMF induced, source of "voltage drop" in particular wire segment is EMF generated only in the loop containing resistors.  All this is not that important. Important part is: voltage will be observed which is contrary to your statement.

[edit] No, you dont'use Ohms law here. You shall use Maxwell's equation to calculate EMF generated in wire segment "a-b"

[edit1] Seems, I know where your frustration comes from. By saying I1=0 he means that current induced by EMF is zero because there is no EMF in the voltmeter leads. On the other hand current will be flowing through voltmeter due to potential difference "voltage drop" between points a & b. This is my explanation. Hope it helps.

Thanks for the reply. I annotated his drawing to make my confusion clearer (heh).

(https://ibin.co/4ROonqRN5Cu3.png)

The equation in question, repeated is
(https://ibin.co/4ROe8sZbEx99.png)

Green is the path integral on the left side of the equation and red is the path integral on the right side of the equation. Would be great if you can fix it so that the equation balances. By all means use Maxwell's equations to get there.

I suspect I'm not going to fully understand McDonald until I've completely grokked his discussions on vector potential.

Title: Re: Does Kirchhoff's Law Hold? Disagreeing with a Master
Post by: mhz on December 27, 2018, 03:06:22 am
The most confusing thing about this is that he seems to be suggesting that there is a scalar potential here that is independent of the path, despite that pesky time varying B field.

I have been, and I have to admit I'm quite perplexed with it. In particular equation 35 doesn't seem correct to me. For example, if we assume a point in time where I = 1mA, R1 = 100ohm, R2 = 900ohm, R = 1Mohm and I1 ≈ 0 (as he states) then he seems to be saying 0V = -1V. If someone can help clear this up for me I'd appreciate it.

Equation is correct indeed. Thou it is counterintuitive. Wire segment "a-b" is part of *both* loops - loop of leads and loop containing R1 and R2. Leads loop does not have any EMF induced, source of "voltage drop" in particular wire segment is EMF generated only in the loop containing resistors.  All this is not that important. Important part is: voltage will be observed which is contrary to your statement.

[edit] No, you dont'use Ohms law here. You shall use Maxwell's equation to calculate EMF generated in wire segment "a-b"

[edit1] Seems, I know where your frustration comes from. By saying I1=0 he means that current induced by EMF is zero because there is no EMF in the voltmeter leads. On the other hand current will be flowing through voltmeter due to potential difference "voltage drop" between points a & b. This is my explanation. Hope it helps.

Thanks for the reply. I annotated his drawing to make my confusion clearer (heh).

(https://ibin.co/4ROonqRN5Cu3.png)

The equation in question, repeated is
(https://ibin.co/4ROe8sZbEx99.png)

Green is the path integral on the left side of the equation and red is the path integral on the right side of the equation. Would be great if you can fix it so that the equation balances. By all means use Maxwell's equations to get there.

I suspect I'm not going to fully understand McDonald until I've completely grokked his discussions on vector potential.
Title: Re: Does Kirchhoff's Law Hold? Disagreeing with a Master
Post by: ogden on December 27, 2018, 03:26:09 am
I suspect I'm not going to fully understand McDonald until I've completely grokked his discussions on vector potential.

(https://ibin.co/4ROonqRN5Cu3.png)


Oh, my... You use Ohms law to claim that voltage between a-b is 1V?  :palm:

I don't even know what to say. How dare you pretend that you mastered Maxwell's equations?!

First, you have to know azimuthal angle between two points, a & b. Let's assume it is PI/4 (45 degrees). According to your data EMF of the resistor loop is 1V. We put 1V and Pi/4 into equation (34): 1V*(Pi/4)/(2*Pi) = 1/8 V. Voltmeter shall show 0.125V in such case (when angle is 45 degrees).

[edit] Forget about "I1=0". It is misleading or even incorrect. All you shall know - there's no EMF induced in the voltmeter leads.
Title: Re: Does Kirchhoff's Law Hold? Disagreeing with a Master
Post by: mhz on December 27, 2018, 03:43:44 am
I suspect I'm not going to fully understand McDonald until I've completely grokked his discussions on vector potential.

(https://ibin.co/4ROonqRN5Cu3.png)


Oh, my... You use Ohms law to claim that voltage between a-b is 1V?  :palm:

I don't even know what to say. How dare you pretend that you mastered Maxwell's equations?!

First, you have to know azimuthal angle between two points, a & b. Let's assume it is PI/4 (45 degrees). According to your data EMF of the resistor loop is 1V. We put 1V and Pi/4 into equation (34): 1V*(Pi/4)/(2*Pi) = 1/8 V. Voltmeter shall show 0.125V in such case (when angle is 45 degrees).

[edit] Forget about "I1=0". It is misleading or even incorrect.

I don't pretend anything, and if you continue using these emoticons or abusive/patronizing language I will just start ignoring you again. Please drop them if you want to continue discussing like adults.

Int E•dl in the wires is 0V (no E fields in perfect conductors, or next to none in real conductors in which case we approximate to 0) and in the resistor is equivalent to I*R (if not then what do you think the contribution of Int E•dl through the resistor is?).

Your response contradicts several of the things McDonald says in his paper, namely that the voltmeter will read 0V (not 0.125V) and that I1 = 0 is misleading/incorrect. So now who should I believe, you or McDonald?

Anybody else wanna take a crack at this?
Title: Re: Does Kirchhoff's Law Hold? Disagreeing with a Master
Post by: ogden on December 27, 2018, 03:45:49 am
Have you ever tried to compute the integral of E.dl of a RLC circuit with a generator - a lumped circuit, just to see that Kirchhoff and field theory can agree if there is no dB/dt area enclosed by the circuit path? It might clear a lot of things up before trying to attack a non-lumped circuit such as Lewin's.

See discussion above about McDonald's paper, then think again how non-lumped circuit is Dr.Lewin's experiment.
Title: Re: Does Kirchhoff's Law Hold? Disagreeing with a Master
Post by: ogden on December 27, 2018, 03:58:50 am
Int E•dl in the wires is 0V (no E fields in perfect conductors, or next to none in real conductors in which case we approximate to 0) and in the resistor is equivalent to I*R (if not then what do you think the contribution of Int E•dl through the resistor is?).

Faradays law? ... Maybe? From your blog BTW:

(https://www.eevblog.com/forum/chat/does-kirchhoffs-law-hold-disagreeing-with-a-master/?action=dlattach;attach=606070;image)

Quote
Your response contradicts several of the things McDonald says in his paper, namely that the voltmeter will read 0V (not 0.125V) and that I1 = 0 is misleading/incorrect. So now who should I believe, you or McDonald?

Anybody else wanna take a crack at this?

I do not contradict with McDonald. You do. I will repeat again what he says: "the result Vmeter = 0 is appealing in that we might naïvely expect the “voltage drop” to be zero between points along a good/perfect conductor." He even shows equation (34) how to calculate voltage between a-b points.

I already explained what I think about I1=0:

[edit1] Seems, I know where your frustration comes from. By saying I1=0 he means that current induced by EMF is zero because there is no EMF in the voltmeter leads. On the other hand current will be flowing through voltmeter due to potential difference "voltage drop" between points a & b. This is my explanation. Hope it helps.
Title: Re: Does Kirchhoff's Law Hold? Disagreeing with a Master
Post by: rfeecs on December 27, 2018, 04:01:02 am
I have been, and I have to admit I'm quite perplexed with it. In particular equation 35 doesn't seem correct to me. For example, if we assume a point in time where I = 1mA, R1 = 100ohm, R2 = 900ohm, R = 1Mohm and I1 ≈ 0 (as he states) then he seems to be saying 0V = -1V. If someone can help clear this up for me I'd appreciate it.

(https://ibin.co/4ROe8sZbEx99.png)

[Edit 1: fix typos]

I agree equation 35 does not look correct, maybe it is a typo.  He seems to have left out the EMF term.  Perhaps by E he is refering to EV as in equation 18.
Title: Re: Does Kirchhoff's Law Hold? Disagreeing with a Master
Post by: ogden on December 27, 2018, 04:10:26 am
I agree equation 35 does not look correct, maybe it is a typo.  He seems to have left out the EMF term.

It's not a crime and it does not make it a typo. Dr.Lewin omits EMF vector potential terms all the time. Everybody does. Even mhz himself:

(https://www.eevblog.com/forum/chat/does-kirchhoffs-law-hold-disagreeing-with-a-master/?action=dlattach;attach=606070;image)
Title: Re: Does Kirchhoff's Law Hold? Disagreeing with a Master
Post by: mhz on December 27, 2018, 04:20:44 am
Int E•dl in the wires is 0V (no E fields in perfect conductors, or next to none in real conductors in which case we approximate to 0) and in the resistor is equivalent to I*R (if not then what do you think the contribution of Int E•dl through the resistor is?).

Faradays law? ... Maybe? From your blog BTW:

(https://www.eevblog.com/forum/chat/does-kirchhoffs-law-hold-disagreeing-with-a-master/?action=dlattach;attach=606070;image)

That is Faraday's law yes, how would you use it to calculate the contribution of the line integral of E•dl through the resistor?

Quote
Your response contradicts several of the things McDonald says in his paper, namely that the voltmeter will read 0V (not 0.125V) and that I1 = 0 is misleading/incorrect. So now who should I believe, you or McDonald?

Anybody else wanna take a crack at this?

Quote
I do not contradict with McDonald. You do. I will repeat again what he says: "the result Vmeter = 0 is appealing in that we might naïvely expect the “voltage drop” to be zero between points along a good/perfect conductor." He even shows equation (34) how to calculate voltage between a-b points.


You said "Voltmeter shall show 0.125V."
McDonald said "the result Vmeter = 0 ..."

I believe what you meant to say is that Vab = 0.125V (based on your use of equation 34), despite the fact that the Voltmeter shows 0V.
At any rate, I'm asking about equation 35 not 34*. You're fixating on the arc a-b when equation 35 doesn't include that arc in either of its path integrals (as shown in my color coded annotation).

I asked a question in the previous post. "what do you think the contribution of Int E•dl through the resistor is?" You seemed to disagree with me that it is equal to I*R.

*admittedly, equation 34 is confusing to me as well but I suspect I'm not going to follow McDonald there without a much deeper reading of this paper, rather than having it explained to me here.

Edit1: attempt at fix quotesing
Title: Re: Does Kirchhoff's Law Hold? Disagreeing with a Master
Post by: ogden on December 27, 2018, 04:30:02 am
That is Faraday's law yes, how would you use it to calculate the contribution of the line integral of E•dl through the resistor?

Why you suddenly introduce resistor here? We talk about Maxwell's equations and Faradays law. Note that wire segment a-b does not contain any resistor.

Quote
You said "Voltmeter shall show 0.125V."

Yes. I demonstrated you how to calculate voltage between a-b in case angle is 45 degrees. McDonald provides only formula, not actual calculation. Where's your problem to understand that?

Quote
McDonald said "the result Vmeter = 0 ..."

If you cannot comprehend that McDonald says that it is naïve to expect that voltmeter will show 0V, then our discussion is finished here and now. When you confirm that you can read - we may continue.
Title: Re: Does Kirchhoff's Law Hold? Disagreeing with a Master
Post by: mhz on December 27, 2018, 04:46:04 am
That is Faraday's law yes, how would you use it to calculate the contribution of the line integral of E•dl through the resistor?

Why you suddenly introduce resistor here? We talk about Maxwell's equations and Faradays law. Note that wire segment a-b does not contain any resistor.

I didn't suddenly introduce it. It's inside one of the path integrals in equation 35 which is what I've been talking with you about for these past several posts. I now see that you were still referring to the arc a-b and using equation 34 when you said "Oh, my... You use Ohms law to claim that voltage between a-b is 1V?  :palm:"

I'm fully able to plug the same numbers into equation 34 as you are. You keep responding to my questions about equation 35 with equation 34. I've already conceded that I don't understand McDonald's development to arriving at that equation, and I'm not particularly interested in discussing it here without a chance to do a deeper read of his paper. I have been talking about equation 35 for these past several posts and it's now clear that you've been ignoring them and persisting at talking about equation 34. We're not even talking about the same thing.

Quote

Quote
You said "Voltmeter shall show 0.125V."

Yes. I demonstrated you how to calculate voltage between a-b in case angle is 45 degrees. McDonald provides only formula, not actual calculation. Where's your problem to understand that?

Quote
McDonald said "the result Vmeter = 0 ..."

If you cannot comprehend that McDonald says that it is naïve to expect that voltmeter will show 0V, then our discussion is finished here and now. When you confirm that you can read - we may continue.

"our discussion is finished here and now"

Sounds good to me.
Title: Re: Does Kirchhoff's Law Hold? Disagreeing with a Master
Post by: ogden on December 27, 2018, 05:09:46 am
I didn't suddenly introduce it. It's inside one of the path integrals in equation 35 which is what I've been talking with you about for these past several posts. I now see that you were still referring to the arc a-b and using equation 34 when you said "Oh, my... You use Ohms law to claim that voltage between a-b is 1V?  :palm:"

I'm fully able to plug the same numbers into equation 34 as you are. You keep responding to my questions about equation 35 with equation 34.

Equation 34 is derivative of equation 35. They essentially both are explaining EMF voltage at given time of observation between points a & b. Thou there's enough data only for equation 34 to calculate anything. Note that Dr.Lewin also declares "EMF equals 1V" and do not actually calculate it (EMF voltage) using Maxwell's equation. So your questions about equation 35 and field inside resistors when it is defined that EMF is 1V and current flowing in the loop 1mA is... kinda pointless. You shall go straight to equation 34 and calculate it - as I did it already. Result was not 0V.

[edit]  BTW integral E.dl between points a & b does not have resistor in the path, your equation is incorrect in result:"

(https://www.eevblog.com/forum/chat/does-kirchhoffs-law-hold-disagreeing-with-a-master/?action=dlattach;attach=606097;image)

Quote
I have been talking about equation 35 for these past several posts and it's now clear that you've been ignoring them and persisting at talking about equation 34. We're not even talking about the same thing.

It is clear that you did not even understand that those two are essentially the same :)


Quote
"our discussion is finished here and now"

Sounds good to me.

I am glad that we can agree at least about something.
Title: Re: Does Kirchhoff's Law Hold? Disagreeing with a Master
Post by: ogden on December 27, 2018, 11:02:59 am
To not repeat a certain arrangement i will also answer this for both definitions of voltage:

A) For definition "Voltage is the integral of all forces pushing on electrons along a given path connecting two points":
The inductors L2 L3 L4 L5 have zero voltage across them at all times (Zero resistance). Any EMF voltage induced in the wire by the magnetic field is instantly countered by the charge separation of electrons.

B) For definition "Voltage is the difference in charge density between two points" (This is what real life voltmeters show)
The inductors L2 L3 L4 L5 have a voltage drop that sums up to the same voltage as the total voltage drop on the resistors.  This voltage in the wire is caused by charge separation pushing electrons towards one end of a wire, resulting in more electrons on one end hence higher voltage on one end.

Thank you for very good explanation of vector versus scalar potentials. I admire your patience explaining underlying basics to those who learn physics just by memorizing equations.
Title: Re: Does Kirchhoff's Law Hold? Disagreeing with a Master
Post by: Berni on December 27, 2018, 12:04:32 pm
Where does definition B come from? Charge density and Voltage don't even have the same units. [C/m3] vs [J/C]

Yes the units don't quite match up because they are not the same thing.

It comes from removing the effects of external fields. As you have seen in this thread circuit meshes don't handle external fields. The components that make up a mesh model act as if they have zero physical size (This includes all wires being 0m long). This means that no matter how strong a magnetic or electric field the circuit is exposed for there will never be any observable field gradient across a component. A loop with a area of 0 m2 can't have EMF induced in it no matter what you do.

So when external fields have no effects the only thing remaining pushing electrons around is the internal electric field caused by charge separation and this is basically different charge density between two points.

This section of the Wiki page for voltage explains the conflicting definitions:
https://en.wikipedia.org/wiki/Voltage#Definition

The kind of voltage i am talking about in definition (B) is what they call "Definition via decomposition of electric field". You can read up on the details by flowing the links. It uses a different way of treating the magnetic field to make charge separation via EMF visible as a voltage. This solves all the multiple result ambiguities of definition (A) and always gives a single number for every point in the circuit. By doing this it also causes a voltage to appear on a wire when in a changing magnetic field.

This is the source of argument for most of the recent posts in this thread. The answer to the above diagrams depends on what definition you are using.

I disagree. You can split the total mutual inductance M of the loop into two strings of as many inductors as you want in spice. The value you measure in spice will not be the  actual scalar voltage potential between the ends of the resistors (which is approximately zero as measured by the voltmeter). Lumping can't be done in this kind of circuit  in spice without creating false outcomes.

Well i was comparing my spice simulation results to experimental data made on youtube and on my own bench, they seamed to match pretty closely. So it does appear to work fine for circuits discussed here. If it is wrong then it looks like it takes a different or more complex circuit to cause it to break. Lumping is very commonly used even in RF circuits, its just a matter of lumping it correctly.

Do note that SPICE like all other circuit analysis uses the second definition (B) for voltage. The equation used for that is seen on the Wiki article above.



I see now that you're trying to model the mutual inductance of the "outer loop" i.e. the path formed by the two measurement loops, but not going through R1 and R2. You've arranged the coupling dots in a way that the inner inductors and outer inductors cancel each other out in a way that satisfies there being no flux coupling in the two measurement loops.

Yes the coupling dots are very important. The dots indicate that both loops are going clockwise around the center. So if you go around the loop you will always see them pointing the same way. If the dot points the other way that would signify the wire turning around and going counterclockwise, its allowed to do that if it wants and will still give correct results, however this circuit does not have the wire changing direction around the loop so they all point the same direction.

You have indeed identified correctly how this model works. Because the wire going to the resistor and the wire going to the voltmeter flow the same path this means they get the same voltage induced in them. Since you need to go clockwise to get to the circle midpoint and then back counterclockwise to get back to the meter the voltages are opposite in sign and they subtract out. This is called bifilar winding and is widely used for removing inductance when its not desired.

If you use the second definition (B) of voltage and as described in the Wiki article use the "magnetic vector potential" to think about the magnetic field you will find the exact same behavior in the real life circuit.

Its just a different way of thinking about it. Results are identical in the end, just how you get to them is slightly different.
Title: Re: Does Kirchhoff's Law Hold? Disagreeing with a Master
Post by: In Vacuo Veritas on December 27, 2018, 01:54:44 pm
Is it still OK to use a voltmeter? Or is every measurement suspect now?
Title: Re: Does Kirchhoff's Law Hold? Disagreeing with a Master
Post by: bsfeechannel on December 27, 2018, 02:44:52 pm
I disagree with the thread title.

Walter Lewin used to be a master. Then he started flinging poo at good people.
So i vote we take Master status from him.  :-DD

A mark of distinction of those who criticize Lewin is their utter and declared inability to teach Maxwell.
Title: Re: Does Kirchhoff's Law Hold? Disagreeing with a Master
Post by: In Vacuo Veritas on December 27, 2018, 03:58:29 pm
A mark of distinction of those who criticize Lewin is their utter and declared inability to teach Maxwell.

To be fair, he did issue an apology video. Don't know if it's already been linked to here.

https://www.youtube.com/watch?v=d_XqrZo5_7Y (https://www.youtube.com/watch?v=d_XqrZo5_7Y)
Title: Re: Does Kirchhoff's Law Hold? Disagreeing with a Master
Post by: ogden on December 27, 2018, 04:05:40 pm
Is it still OK to use a voltmeter? Or is every measurement suspect now?

It depends. Definitely you shall know what you are doing and how exactly your voltmeter measure voltages and how it may impact results. Sometimes you may want to use electrometer instead of voltmeter.
Title: Re: Does Kirchhoff's Law Hold? Disagreeing with a Master
Post by: boB on December 27, 2018, 04:16:17 pm
Is it still OK to use a voltmeter? Or is every measurement suspect now?

Those meter measurements were always suspect in certain situations.

Title: Re: Does Kirchhoff's Law Hold? Disagreeing with a Master
Post by: ogden on December 27, 2018, 04:22:32 pm
To be fair, he did issue an apology video. Don't know if it's already been linked to here.

Yes it was mentioned here "many pages of posts" ago. Dr.Lewin is one of greatest physics teachers known, he deserves all the titles received.

Punishment shall be proportionate. MIT may revoke his emeritus title, harassed women may seek justice in the court, but come on -  removing all his life's work, his courses and lectures from all MIT teaching platforms is way too much. MIT punished not only Dr.Lewin but many, many students. Luckily we have youtube and hopefully social justice warriors of MIT will not do anything about it. We shall not burn scientist with all his books/works/videos just because he made some mistake in his life.
Title: Re: Does Kirchhoff's Law Hold? Disagreeing with a Master
Post by: In Vacuo Veritas on December 27, 2018, 04:54:58 pm
Is it still OK to use a voltmeter? Or is every measurement suspect now?

Those meter measurements were always suspect in certain situations.

During lightning storms, I guess...
Title: Re: Does Kirchhoff's Law Hold? Disagreeing with a Master
Post by: bsfeechannel on December 27, 2018, 08:22:01 pm
Is it still OK to use a voltmeter?

That's a very good question. And trying to answer it is the path to understanding Maxwell and his dreaded equations.

Quote
Or is every measurement suspect now?

Why would Nature lie to you?
Title: Re: Does Kirchhoff's Law Hold? Disagreeing with a Master
Post by: ogden on December 28, 2018, 11:08:44 pm
Regarding McDonalds 'paper' (has that been published on a peer reviewed paper?), let me quote this comment George Hnatiuk from a youtube discussion

Quote
I found several errors in the "Lewin's Circuit Paradox" paper back in June to which I alerted Dirk and company and the paper has since been edited with NEW errors introduced and some of the older ones still present. It is very sloppy work at best and nothing I would expect from a university staff member.

Ok. Fair enough. Nobody's perfect. That's why peer review practice is so important and McDonald do error corrections. Could you provide (link to) Dr.Lewin's paper regarding subject, supposedly peer-reviewed?

Quote
As I said before, you should try to analyze how the lumped circuit simplifications come about before tackling non-lumped circuits like Lewin's ring.

As I said - when you read McDonald's paper you will see that Dr.Lewin's circuit can be analyzed as circuit of lumped elements. If you disagree, then prove opposite - tell where and how McDonald is wrong. [edit] Robert H. Romer in his paper do lumped analysis as well BTW.
Title: Re: Does Kirchhoff's Law Hold? Disagreeing with a Master
Post by: ogden on December 29, 2018, 12:15:40 am
EMF source equation alone does not describe system of EMF source *and* load. In the inner loop of Dr.Lewin's experiment E fields can be expressed as E = E.coloumb + E.induced. Wire loop is responsible for E.induced, we can say it is EMF source. As resistance of the wire is very small compared to resistors we ignore it so all the E.coloumb field is located in the resistors - those are load. In Laymans terms: EMF generated by the wire is dissipated in the resistors. Now question - what's the summary field (integral E.dl ) of the loop E = E.coloumb + E.induced? Before you answer remember energy conservation law - that all the energy generated by wire loop is dissipated in the resistors.
Title: Re: Does Kirchhoff's Law Hold? Disagreeing with a Master
Post by: Sredni on December 29, 2018, 12:46:46 am
Ok. Fair enough. Nobody's perfect. That's why peer review practice is so important and McDonald do error corrections. Could you provide (link to) Dr.Lewin's paper regarding subject, supposedly peer-reviewed?

Why, it's Romer's paper - it's funny you did not realize it, considering you are mentioning it in this very post. I can even cite a book that has the very same analysis - I mean literally quotes Romer - but I'll keep that to myself for the moment. You would not read it anyway, like you did not read Ramo Whinnery Van Duzer.

Quote
As I said - when you read McDonald's paper you will see that Dr.Lewin's circuit can be analyzed as circuit of lumped elements. If you disagree, then prove opposite - tell where and how McDonald is wrong.

Maybe you are misunderstanding McDonald as well.

Quote
In the inner loop of Dr.Lewin's experiment E fields can be expressed as E = E.coloumb + E.induced. Wire loop is responsible for E.induced, we can say it is EMF source. As resistance of the wire is very small compared to resistors we ignore it so all the E.coloumb field is located in the resistors - those are load.

Nah, the *resulting* field is all located in the resistor. E_coulomb kills E_induced in the wires giving a resultant E field compatible with j = sigma E.

Title: Re: Does Kirchhoff's Law Hold? Disagreeing with a Master
Post by: ogden on December 29, 2018, 01:00:00 am
Why, it's Romer's paper - it's funny you did not realize it

It's not. Dr.Lewin's lecture(s) do not follow Romer's paper by any stretch of imagination, you shall know it (maybe). Are you absolutely sure you want to go down this rabbit hole? ;)

Quote
Quote
As I said - when you read McDonald's paper you will see that Dr.Lewin's circuit can be analyzed as circuit of lumped elements. If you disagree, then prove opposite - tell where and how McDonald is wrong.

Maybe you are misunderstanding McDonald as well.

Instead of pointing out where McDonald is wrong you come-up with this BS? BTW John W. Belcher in his paper do lumped analysis as well.

Quote
E_coulomb kills E_induced in the wires

Right. In short: it sum is zero. End of discussion.
Title: Re: Does Kirchhoff's Law Hold? Disagreeing with a Master
Post by: ogden on December 30, 2018, 08:57:14 pm
Romer's paper is about the very same experiment made by Lewin, and it reaches the very same conclusions.

How is that? I do not find Romer mentioning birds in his conclusions. Do you mean conclusion "Faraday's law is very fascinating and puzzling"? Yes, they both are puzzled about Faraday's law :) - We all are. Especially when it is interpreted by Dr.Lewin. Ok, kidding. L(di/dt) is not that puzzling and it works well. Please name those "very same conclusions" you are talking about. I am excited to hear them.

Quote
So, since real life voltmeters have (to my knowledge) no way to tell the E_coulomb and the E_induced apart but only see the effect of the resulting total field, it makes little sense to ascribe to the [difference in the values of the] scalar potential any meaning besides that of the voltage measured along a very special class of paths.

TL;DR. Are you suggesting that voltmeter can be used to measure electric field? Question did not mention voltmeters, nor expected voltage as an answer: "what's the summary field (integral E.dl ) of the loop E = E.coloumb + E.induced?" You did not gave clear answer. Is it zero or not?

p.s. Of course all electrons are the same, no matter they were moved by electromagnetic induction, chemical reaction or just static electricity.
p.p.s Yes, I am sarcastic - because when I ask you simple question, you are making evasive moves like scammer who is caught.
Title: Re: Does Kirchhoff's Law Hold? Disagreeing with a Master
Post by: ogden on December 31, 2018, 12:09:59 am
And he didn't start with "Hello, hello, hello!" as well. Then they must have found completely different results.

As an argument I was referring to scientific paper having peer review and corrections. All you have in return is youtube video with introduction "Hello, hello, hello", no written content and no peer review? Whatta crooked mirrors world you are living in?!

Quote
What I see is that they both find that the voltage is dependent on the path, and that when placed on the outside of the loop the voltmeters - applied to the very same two points - give different and opposite phase reading.

What?!! Your whole proof is two voltmeters showing different signs?

Quote
Quote
"what's the summary field (integral E.dl ) of the loop E = E.coloumb + E.induced?" You did not gave clear answer. Is it zero or not?
Here's the answer, assuming that with 'summary' you mean circulation along the circuit's path: the circulation of E_total (integral of (E_coloumb + E_induced) . dl along the circuit's path is equal to minus the time derivative of the flux of B linked by said path.
Yes, it's Faraday's law.

So you refuse to name number because you either do not know it of refuse to acknowledge it being zero?
Title: Re: Does Kirchhoff's Law Hold? Disagreeing with a Master
Post by: rfeecs on December 31, 2018, 12:14:17 am
Please do yourself and anybody else a favor: get hold of a copy of "Fields and Waves in Communication Electronics" by Ramo, Whinnery and VanDuzer and read the first four pages of chapter 4 (The electromagnetics of circuits)...

I guess you really like that book.  It was my textbook in college 37 years ago for "Electromagnetic Fields and Waves"  EECS117A, B, and C.  The teacher was Theodore Van Duzer.

Gee, that was before Dr. Lewin first performed his SUPER DEMO.  My education was lacking.
Title: Re: Does Kirchhoff's Law Hold? Disagreeing with a Master
Post by: EEVblog on December 31, 2018, 01:59:40 am
To be fair, he did issue an apology video. Don't know if it's already been linked to here.

Yes it was mentioned here "many pages of posts" ago. Dr.Lewin is one of greatest physics teachers known, he deserves all the titles received.

Punishment shall be proportionate. MIT may revoke his emeritus title, harassed women may seek justice in the court, but come on -  removing all his life's work, his courses and lectures from all MIT teaching platforms is way too much. MIT punished not only Dr.Lewin but many, many students. Luckily we have youtube and hopefully social justice warriors of MIT will not do anything about it. We shall not burn scientist with all his books/works/videos just because he made some mistake in his life.

Woah  :o
What happened here?
Was he involved in some sexual scandal or something?

Edit?: found it:
https://www.insidehighered.com/news/2015/01/23/complainant-unprecedented-walter-lewin-sexual-harassment-case-comes-forward (https://www.insidehighered.com/news/2015/01/23/complainant-unprecedented-walter-lewin-sexual-harassment-case-comes-forward)

(https://i.imgur.com/luWSGsc.png)

I didn't know they removed all his lectures from their openMIT channel
Title: Re: Does Kirchhoff's Law Hold? Disagreeing with a Master
Post by: ogden on December 31, 2018, 02:18:55 am
Was he involved in some sexual scandal or something?

He became of #metoo target, due to his own human error. As I already said - I do not agree. Such issues shall be resolved in/by court, not by directorate of MIT - by literally burning all his work, by punishing his students, not actually punishing himself. [edit] AFAIK it was by consent - hot pics in exchange for passed exams.
Title: Re: Does Kirchhoff's Law Hold? Disagreeing with a Master
Post by: EEVblog on December 31, 2018, 02:37:28 am
He became of #metoo target, due to his own human error. As I already said - I do not agree. Such issues shall be resolved in/by court, not by directorate of MIT - by literally burning all his work, by punishing his students, not actually punishing himself.

That was my first thought, removing his (highly regarded) content is effectively punishing students.
Title: Re: Does Kirchhoff's Law Hold? Disagreeing with a Master
Post by: Sredni on December 31, 2018, 11:29:15 am
Please do yourself and anybody else a favor: get hold of a copy of "Fields and Waves in Communication Electronics" by Ramo, Whinnery and VanDuzer and read the first four pages of chapter 4 (The electromagnetics of circuits)...

I guess you really like that book.

It's a nice, no-bullshit, dense and somewhat rigorous textbook. Not perfect by any stretch, but one of my favorites nonetheless. It could use some additional 80-100 pages. All blank. Seriously, that book needs to put some space between paragraphs ;-).
The reason I'm bringing RWvD up so many times is that it has a really nice discussion on the origin of KVL and KCL from Maxwell's equations, and of course the fact that ogden seems impervious to reading it. I'll end up summarizing it here, I know... But pictures, and formulas... they are so time consuming to put in a post...

Quote
  It was my textbook in college 37 years ago for "Electromagnetic Fields and Waves"  EECS117A, B, and C.  The teacher was Theodore Van Duzer.

Woah, how was he?
Title: Re: Does Kirchhoff's Law Hold? Disagreeing with a Master
Post by: Sredni on December 31, 2018, 11:47:34 am
As an argument I was referring to scientific paper having peer review and corrections. All you have in return is youtube video with introduction "Hello, hello, hello", no written content and no peer review? Whatta crooked mirrors world you are living in?!

The peer-reviewed scientific paper backing Lewin's "Hello hello hello" videos is Romer's paper. I had hoped it was clear.
Lewin himself talks about the paper in one of his recent videos. He said "I may or may not have read it", meaning that since it was published prior to his first superdemo he cannot prove he came up independently with the same experiment. He might as well have, it's not something particularly difficult to conceive. What counts is that both the superdemo and Romer's paper are about the same circuit and the apparent paradox of two voltmeters attached to the very same two points reading two different voltages. (Yes, it's more like oscilloscopes showing waveforms since we're talking about time-varying fields).

Quote

Quote
What I see is that they both find that the voltage is dependent on the path, and that when placed on the outside of the loop the voltmeters - applied to the very same two points - give different and opposite phase reading.
What?!! Your whole proof is two voltmeters showing different signs?

The whole drama is about two voltmeters showing different readings when attached to the same points (along different paths).
Now you are pretending it isn't?

Quote
Quote
Quote
"what's the summary field (integral E.dl ) of the loop E = E.coloumb + E.induced?" You did not gave clear answer. Is it zero or not?
Here's the answer, assuming that with 'summary' you mean circulation along the circuit's path: the circulation of E_total (integral of (E_coloumb + E_induced) . dl along the circuit's path is equal to minus the time derivative of the flux of B linked by said path.
Yes, it's Faraday's law.

So you refuse to name number because you either do not know it of refuse to acknowledge it being zero?

Oh, for... ****'s sakes!
It's not zero. The value depends on the time-varying flux: it is equal to minus the time derivative of the flux of B linked by said path. Didn't I just tell you that?
You want a value? In the case of Lewin's experiment, IIRC, it's 1V.

left side - keeping track of the signs and considering perfect conducting wires
0.9V on one resistor, 0V in the wire, 0.1V on the other resistor, 0V in the wire ===> circuitation of E.dl = 1V
right side
-dfi(B)/dt = 1V

Please, find a copy of...
--never mind.

EDIT: fixed quoting, and typos and plurals - oh my!
Title: Re: Does Kirchhoff's Law Hold? Disagreeing with a Master
Post by: dayfall on December 31, 2018, 05:54:47 pm
I have a question.  Why wouldn't a simple loop of wire show that "Kirchhoff is for the birds"?   (Or even a straight wire)

The wire, as in Lewin's more complex circuit, has a resistance of zero.  We can easily create a situation in which we measure a voltage across that loop.  Afterwards a circuit, like Lewin's, can be shown to have different voltages across resistors that are shorted.  Kirchhoff defeated.

Also, in one of his videos does Lewin say something like "The oscillioscope on the left measures the voltage across the left resistor."  How does he know which one it is measuring?
Title: Re: Does Kirchhoff's Law Hold? Disagreeing with a Master
Post by: rfeecs on December 31, 2018, 06:00:41 pm

Quote
  It was my textbook in college 37 years ago for "Electromagnetic Fields and Waves"  EECS117A, B, and C.  The teacher was Theodore Van Duzer.

Woah, how was he?

He was a bit introverted, quiet, maybe the opposite of Lewin's teaching personality.  By the time we got to the last quarter of the course series, there were only about 5 students left.  I was losing interest myself.  Back then there was almost no discussion of computer simulation which has become so dominant in the field now.  We were probably using the first edition of the book.  I remember some of the course was tricks to solve specific problems that wouldn't be needed today.

I agree the book is pretty good.  I have referred back to it myself when I realize that I forgot some basic things and it immediately refreshed my memory with it's basic and direct approach.  It does require vector calculus, so maybe not for everyone.  That's just the nature of the subject.
Title: Re: Does Kirchhoff's Law Hold? Disagreeing with a Master
Post by: bsfeechannel on December 31, 2018, 09:02:46 pm
The reason I'm bringing RWvD up so many times is that it has a really nice discussion on the origin of KVL and KCL from Maxwell's equations, and of course the fact that ogden seems impervious to reading it.

Well, we have to thank ogden for his "imperviousness". Without him, you, mhz, rfeecs, RoGeorge, radioactive, et al., wouldn't have produced such brilliant expositions. I've learned a lot reading your posts.

I have a question.  Why wouldn't a simple loop of wire show that "Kirchhoff is for the birds"?   (Or even a straight wire)

Very good question. It would. However, Lewin wanted to work the other way around to emphasize the problem a lot of people have when trying to make sense of Faraday's law, having Kirchhoff always holds as a premise. What he did was the classical reductio ad absurdum. By assuming that Kirchhoff always holds, he arrived at a contradiction. Therefore the premise is false.

Quote
Also, in one of his videos does Lewin say something like "The oscillioscope on the left measures the voltage across the left resistor."  How does he know which one it is measuring?

You have to imagine the circuit with a battery that he drew on the blackboard with each oscilloscope probe connected across each respective resistor. He didn't demonstrate the circuit with a battery in practice because he believed that the understanding of Kirchhoff's law would be trivial. Then the battery is imaginarily removed. So the left oscilloscope measuring the left resistor is the same oscilloscope which would be measuring the left resistor when we had the battery.

I agree the book is pretty good.  I have referred back to it myself when I realize that I forgot some basic things and it immediately refreshed my memory with it's basic and direct approach.  It does require vector calculus, so maybe not for everyone.  That's just the nature of the subject.

We also used this textbook in college. And the pages recommended by Sredni are really good. However we had vector calculus as a separate course before we were introduced to electromag. That's why I recommended Jordan & Balmain, because it has a simplified review of vector analysis, a.k.a, vector calculus, for use with electromagnetism right in the first chapter. You'll need to study basic Calculus, though, but you can find it easily on the Interwebs.

You see, the biggest mistake of those who struggle with electromagnetism is to think that electricity and magnetism are a property of circuits. They are a property of space. So you need tools to study the properties of space. And that is vector analysis.
Title: Re: Does Kirchhoff's Law Hold? Disagreeing with a Master
Post by: ogden on January 01, 2019, 01:29:29 am
The peer-reviewed scientific paper backing Lewin's "Hello hello hello" videos is Romer's paper. I had hoped it was clear.

Fact that you say Romer's paper is backing Lewin's lectures does not magically create new reality. We are talking about Dr.Lewin's claims, not Romer's. It does not matter that Dr.Lewin is doing same experiment and voltmeters are showing similar "phenomena". He did not even made it clear - he know contents of paper or not!!! Math is not the same, claims differ, conclusions are far from equal. You are trying to make impression that you know how science works, but now you produced pure nonsense. There are no Dr.Lewin's scientific papers regarding Kirchoff's circuit rules vs Faradays law. Period. You better just leave it there.

Quote
Quote
"what's the summary field (integral E.dl ) of the loop E = E.coloumb + E.induced?" You did not gave clear answer. Is it zero or not?
Oh, for... ****'s sakes!
It's not zero. The value depends on the time-varying flux: it is equal to minus the time derivative of the flux of B linked by said path. Didn't I just tell you that?
You want a value? In the case of Lewin's experiment, IIRC, it's 1V.

I asked what is sum of conservative and nonconservative fields (E.coloumb + E.induced), your answer is 1V. I would like to address this *after* simple but important thought experiment:

We have transformer in a box with two twisted-pair wires coming out: primary & secondary. We also have tiny 1 KOhm resistor, it's so small that we ignore it's dimensions. We ignore internal resistance of transformer coils and wires as well. Also it is known that magnetic field of transformer is contained in the box and do not have any effect on wires coming out of it. When transformers primary winding is connected to signal generator, we measure 1V AC peak-peak voltage on it's secondary. Transformer is powerful enough so it is still same 1V AC peak-peak when 1K resistor is connected to secondary.

Three questions: 1) Do you agree that we can treat transformer's secondary winding as AC voltage source and resistor as a load? 2) Do you agree that those are two lumped elements, they form circuit and Kirchoff's circuit law holds here? 3) Do you agree that integral E.dl for this circuit is zero at any given time of observation?

Just tell - you agree or disagree. No evasive BS with pointers to literature this time, please. Part 2 will follow after you answer those questions.
Title: Re: Does Kirchhoff's Law Hold? Disagreeing with a Master
Post by: Sredni on January 01, 2019, 02:24:52 pm
Disclaimer: Since I wanted to start the new year with the right foot, I spent new year's eve and I am spending new year's day in bed with a fever. It's not high, but I might be omitting letters, signs (especially signs) and symbols. The good(?) news is that I've had the time to find a way to upload images on this site.

Ok, let's get to the questions.

EDIT: on second thought...

This post has been shortened and cleansed to avoid upsetting other children.
Whatever was written here can be found in one or more of the following books (in no particular order, and without mentioning the usual suspects Feynman, Purcell, Griffiths, Ohanian, Jackson):

Panofsky, Phillips
Classical Electricity and Magnetism 2nd ed

John Kraus
Electromagnetism 2nd to 4th ed

Ramo, Whinnery, VanDuzer
Fields and Waves in Communication Electronics 2nd or 3rd ed

Bleaney
Electricity and Magnetism 3rd ed

Nayfeh, Brussel
Electricity and Magnetism

Kip
Fundamentals of Electricity and Magnetism 2nd ed

Lorrain, Courson
Electromagnetic Fields and Waves 2nd ed

"Books" are static paper based documents that can be found in libraries. They are like smartphones, but (usually) bigger, with lots and lots of extremely thin flexible e-ink screens and a very long battery life. Libraries are...
Oh, never mind. Keep on pushing that square peg into that round hole. With a big enough hammer, it will fit.
Title: Re: Does Kirchhoff's Law Hold? Disagreeing with a Master
Post by: ogden on January 01, 2019, 09:38:12 pm
Quote
"3) Do you agree that integral E.dl for this circuit is zero at any given time of observation?"
Absolutely not.

Do you realize that the circuit you are proposing is just Lewin's ring with one resistor?

You wish I don't? :D

You disagreed that integral E.dl around loop of described circuit is zero, yet for resistor, wires and "black box" you wrote integral E.dl = R*I + 0 + EMF. Do you really claim sum is not zero?

Thing in a box may be piezoelectroc device or just generator - Faraday's law do not apply for such. Please do not stick it everywhere - needed or not. What is coming out of the box - AC 1V peak-peak. Do integral E.dl around the loop of described circuit is zero or not?

Happy and successful NY for everybody! May your every circuit works.
Title: Re: Does Kirchhoff's Law Hold? Disagreeing with a Master
Post by: ogden on January 01, 2019, 10:11:36 pm
After all, when you compute the circulation of E_total = E_conservative + E_induced you get the sum of the circulation of E_conservative which is zero and all that's left is the circulation of E_induced. Everything checks out.

Wait... E.conservative in the resistor is zero even when on it's terminals is 1V? Are you sure?
Title: Re: Does Kirchhoff's Law Hold? Disagreeing with a Master
Post by: Sredni on January 01, 2019, 10:19:41 pm
After all, when you compute the circulation of E_total = E_conservative + E_induced you get the sum of the circulation of E_conservative which is zero and all that's left is the circulation of E_induced. Everything checks out.

Wait... E.conservative in the resistor is zero even when on it's terminals is 1V? Are you sure?

I was talking about the circulation. You have to close the loop.
Fever has gone up, I will check again tomorrow, if I survive the night :-)
Title: Re: Does Kirchhoff's Law Hold? Disagreeing with a Master
Post by: ogden on January 01, 2019, 10:36:51 pm
After all, when you compute the circulation of E_total = E_conservative + E_induced you get the sum of the circulation of E_conservative which is zero and all that's left is the circulation of E_induced. Everything checks out.

Wait... E.conservative in the resistor is zero even when on it's terminals is 1V? Are you sure?

I was talking about the circulation. You have to close the loop.

Indeed to have voltage on resistor terminals, you have to connect it to voltage source. Unconnected, lone resistor for sure will have zero field inside, right? ;)

My way of looking at this box+resistor debate: if black box is able to produce AC 1V alone, I name it EMF source - disregarding it is transformer or just DC-fed AC generator. If I connect resistor, I close the loop, AC 1mA current is flowing and integral E.dl around the loop is zero. There is no magic, just quite simple logic and law of conservation of energy. Do you agree?
Title: Re: Does Kirchhoff's Law Hold? Disagreeing with a Master
Post by: Berni on January 02, 2019, 12:24:46 am
This is exactly why the other "less scientific" definition of voltage (The one about how many electrons there are in one spot) is used in circuit analysis and pretty much everywhere else where you need to actually calculate something.

With that definition of voltage the magnetic EMF looks exactly like a battery. There are no longer any paradoxes or ambiguities of what the voltage is inside a changing magnetic field. Put anything you want in that black box and the result is the same, no longer is a magnetic field a special case that must be treated differently. If you have 1V of magnetically induced EMF its the same as a 1V battery, both push the same current trough the resistor load according to Ohms law. Faradays law works perfectly fine with this definition too.

The only thing you have to do is imagine the magnetic field as a electric field that is circulating around the changing magnetic flux lines. Once you do that you have only electric fields acting on electrons and those are always conservative so you never get multiple answers for what voltage is between two points. And this voltage happens to be the exact same voltage voltmeters are showing.

Its just a different way of calculating the same thing, except that this way gives less confusing answers (Particularly in this kind of cirucit)
Title: Re: Does Kirchhoff's Law Hold? Disagreeing with a Master
Post by: bsfeechannel on January 02, 2019, 01:42:04 am
Fever has gone up, I will check again tomorrow, if I survive the night :-)

I hope you get better soon.
Title: Re: Does Kirchhoff's Law Hold? Disagreeing with a Master
Post by: radioactive on January 02, 2019, 02:10:39 am
Fever has gone up, I will check again tomorrow, if I survive the night :-)

I hope you get better soon.

Yes, get better soon Sredni.  Thank you for sharing your insights and knowledge in this thread.
Title: Re: Does Kirchhoff's Law Hold? Disagreeing with a Master
Post by: ogden on January 02, 2019, 02:54:50 am
Yes, get better soon Sredni.  Thank you for sharing your insights and knowledge in this thread.

Yes, get better soon.

Meanwhile I will sum-up our discussion about "1V AC box + resistor":

While measuring 1V AC voltage coming out of the box with voltmeter/scope, we cannot discern - source is transformer, piezo device or just AC generator powered by chemical battery (we agreed that electrons are the same long ago in this thread). Kirchoff's circuit law (KVL) holds for AC voltage source + resistor *only* when AC voltage is generated by anything but transformer or other kind contraption ruled by Faraday's law. When we have transformer in a box, KVL does not hold anymore. To know - KVL holds or not, we have to look inside the box, otherwise we may be mistaken. This is what you are claiming? Anybody else agreeing?
Title: Re: Does Kirchhoff's Law Hold? Disagreeing with a Master
Post by: rfeecs on January 02, 2019, 02:56:55 am
The only thing you have to do is imagine the magnetic field as a electric field that is circulating around the changing magnetic flux lines.
Why would you do that?  The changing magnetic field produces an electric field.  The electric field lines are circles around the magnetic field as shown in post #16:
https://www.eevblog.com/forum/chat/does-kirchhoffs-law-hold-disagreeing-with-a-master/msg2003414/#msg2003414 (https://www.eevblog.com/forum/chat/does-kirchhoffs-law-hold-disagreeing-with-a-master/msg2003414/#msg2003414)

Quote
Once you do that you have only electric fields acting on electrons and those are always conservative so you never get multiple answers for what voltage is between two points.
If you have electric field lines that form a loop, as in this case, you have a nonconservative field.

Quote
And this voltage happens to be the exact same voltage voltmeters are showing.
So you don't have two voltmeters connected to same point showing different voltages?  This is what happens in the experiment.  Or are we supposed to imagine that it doesn't?

The two voltmeters will only measure the same voltage if the voltmeters and their connecting wires are outside the field region or somehow shielded or arranged so that the field does not affect the measurement.  Fine, but that is not this experiment.  This experiment is set up to demonstrate Faraday's law and non conservative fields.

Quote
Its just a different way of calculating the same thing, except that this way gives less confusing answers (Particularly in this kind of cirucit)

You seem to be claiming that a voltmeter only measures an electric field caused by separation of charges.  For example, you have to move charges on the gate of the input FET of the voltmeter to make it read a voltage, I guess is what you mean.

But the voltmeter can't tell you what separated the charges.  It could be a battery, it could be a solar cell, it could be a thermocouple, it could be a hall effect device, it could be a loop of wire surrounding a changing magnetic flux, or it could be a loop of wire rotating in a static magnetic field, to name a few possibilities.  The voltmeter cannot distinguish the charge separation caused by the electrostatic scalar potential from the charge separation caused by all the different other types.

In this case, the wires connecting the voltmeter are passing through a region that has a non-zero electric field.  That electric field, interacting with the charge in the wire, will give you a different reading on the voltmeter depending on the path the wires take through that field.
Title: Re: Does Kirchhoff's Law Hold? Disagreeing with a Master
Post by: ogden on January 02, 2019, 03:11:49 am
The two voltmeters will only measure the same voltage if the voltmeters and their connecting wires are outside the field region or somehow shielded or arranged so that the field does not affect the measurement.  Fine, but that is not this experiment.  This experiment is set up to demonstrate Faraday's law and non conservative fields.

This time we are *not* talking about Dr.Lewin's experiment circuit but "transformer in a box + 1K resistor *outside it* (https://www.eevblog.com/forum/chat/does-kirchhoffs-law-hold-disagreeing-with-a-master/msg2081848/#msg2081848)".

Most voltmeters (including those Dr.Lewin uses) measure voltage as integral E.dl through their internal resistance. How voltmeters measure is not that important in this case BTW.
Title: Re: Does Kirchhoff's Law Hold? Disagreeing with a Master
Post by: bsfeechannel on January 02, 2019, 04:43:09 am
Meanwhile I will sum-up our discussion about "1V AC box + resistor":

While measuring 1V AC voltage coming out of the box with voltmeter/scope, we cannot discern - source is transformer, piezo device or just AC generator powered by chemical battery (we agreed that electrons are the same long ago in this thread). Kirchoff's circuit law (KVL) holds for AC voltage source + resistor *only* when AC voltage is generated by anything but transformer or other kind contraption ruled by Faraday's law. When we have transformer in a box, KVL does not hold anymore. To know - KVL holds or not, we have to look inside the box, otherwise we may be mistaken. This is what you are claiming? Anybody else agreeing?

Let's try to explain. In the "circuit" below V1 is whatever while V2 is literally zero. Why? Because the net flux in the area bounded by the path determined by the terminals of V2 is zero. So this is not a lumped circuit. Simply encasing this loop of wire that is under a varying magnetic field in a box won't lump it.

(https://www.eevblog.com/forum/chat/does-kirchhoffs-law-hold-disagreeing-with-a-master/?action=dlattach;attach=610816;image)

If you analyze other Maxwell's equations, there is one that shows that you can't have a magnetic monopole. If B is going into the screen, where does it come out? The answer: elsewhere. How could Lewin then create a magnetic monopole when this is not possible? The answer: he showed it a little earlier in the same video where he relegated Kirchhoff to the birds. The answer: solenoids. Solenoids generate a strong magnetic field inside their cores, but very weak outside. In practice we can consider it zero. This happens because outside of a solenoid the lines of magnetic field are very sparse, while in the core they're highly concentrated.

Therefore we need to find a way to lump our loop, so that it can be considered the secondary of a transformer. To do that, we need to gather all the return lines of B and make them cross the area between the loop and V2, like below. Now, bingo! V1 = V2. Now we can box up the loop with the returning lines of B.

(https://www.eevblog.com/forum/chat/does-kirchhoffs-law-hold-disagreeing-with-a-master/?action=dlattach;attach=610822;image)

Another way of lumping our loop is to make its area perpendicular to the area "seen" by V2. In that case the scalar product B · dA will be zero, once scalar products can be thought as the "shadow" one vector casts on another. If they are perpendicular, the "shadow" is zero.

(https://www.eevblog.com/forum/chat/does-kirchhoffs-law-hold-disagreeing-with-a-master/?action=dlattach;attach=610828;image)

You'll recognize these practical arrangements on real inductors. In the picture below, you have a solenoid (a). The area defined by the voltmeter is roughly perpendicular to the area of the solenoid. If you measure like in (b), the voltage will be the real voltage minus one turn. If N is not much grater than one, you can use a toroid as in (c) or other core with a closed magnetic path.

(https://www.eevblog.com/forum/chat/does-kirchhoffs-law-hold-disagreeing-with-a-master/?action=dlattach;attach=610834;image)
Title: Re: Does Kirchhoff's Law Hold? Disagreeing with a Master
Post by: bsfeechannel on January 02, 2019, 05:20:08 am
In short, the loop, the way it is, is unlumpable. Lumpable means that I can measure the same voltage regardless of the position of the voltmeters. What we have is a loop antenna. There is a reason why loop antennas are connected by two-wire transmission lines. So that you guarantee, among other things, that the voltage will always be measured like V1, and not like V2. If, instead of this loop antenna, we had, say a lumped generator, that would not be a problem, although we would need the transmission line for other reasons.

(https://www.eevblog.com/forum/chat/does-kirchhoffs-law-hold-disagreeing-with-a-master/?action=dlattach;attach=610846;image)
Title: Re: Does Kirchhoff's Law Hold? Disagreeing with a Master
Post by: Sredni on January 02, 2019, 08:13:43 am
This is exactly why the other "less scientific" definition of voltage (The one about how many electrons there are in one spot) is used in circuit analysis and pretty much everywhere else where you need to actually calculate something.

You keep talking of that as if it were a different physical quantity, measured in different units. It's not. The 'scalar potential' that is part of the (V, A) pair is just the path integral of the conservative part of the total electric field.
It's as if you decomposed a mathematical function into its odd and even parts and than claimed that the odd part is a different, 'less mathematic' definition of function.

You can easily see where that decomposition comes from by writing Faraday's law (use dS for the differential element of area to avoid confusion with the vector potential A, and use E_total instead of E to highlight that it is the resulting field, superposition of coulombian and induced fields). Then express B in terms of the vector potential A,  turn the surface integral on the right into a path integral along the surface contour using Stokes (or "the rotor's") theorem. Now take the integral on the right to the left, changing sign. Bring everything inside the same path integral. You are left with a field whose circulation along a closed path is always zero.

(https://i.ibb.co/XLHj6mg/screenshot-3.png)

Hey, that's a conservative field! Well of course, you have stripped the induced - non conservative - part from the total field.

(https://i.ibb.co/bmWzBWq/screenshot-2.png)
For a sanity check, refer to formula 11-3-3 on page 484 of Kraus' "Electromagnetics", 4th edition.

Congratulations, it can be very useful, but it's only half of the story.

I will come back to this post with formulas and drawings when I will be able to scan.
EDIT: yep, I did just that.
EDIT: fixed wrong double integral symbol.
Edit jan 22 - attached pictures
Title: Re: Does Kirchhoff's Law Hold? Disagreeing with a Master
Post by: ogden on January 02, 2019, 08:22:48 am
Quote
My way of looking at this box+resistor debate: if black box is able to produce AC 1V alone, I name it EMF source - disregarding it is transformer or just DC-fed AC generator. If I connect resistor, I close the loop, AC 1mA current is flowing and integral E.dl around the loop is zero. There is no magic, just quite simple logic and law of conservation of energy. Do you agree?

No. This is the root of your problem in understanding Faraday. You start with the assumption that integral of E.dl around a loop is zero to prove your thesis that integral of E.dl around the loop is zero.

*You* have problem, not me. You say that resistor with voltage drop have zero E field inside. That means that integral E.dl over it is zero meaning that it does not have voltage drop on it! It's paradox, don't you see?

Quote
KVL does not work anymore, and that's the point in saying that Kirchhoff is for the birds.

What?! TL;DR. You did not even answer. I will try again. Three Four questions: 1) Does KVL hold when inside box is DC battery? 2) Does KVL hold when inside box is DC-powered AC generator? 3) Does KVL hold when inside box is piezo-based 1V AC voltage generator? 4) Does KVL hold when inside box is transformer? I assume that I know your answer to last question and it is definite "no". I will appreciate your answers to remaining questions.
Title: Re: Does Kirchhoff's Law Hold? Disagreeing with a Master
Post by: ogden on January 02, 2019, 08:47:09 am
In short, the loop, the way it is, is unlumpable.

You just disproved yourself. Do you recall drawing lumped model of 1Ohm ring while ago - it was drawn as string of many series batteries + resistors.

Quote
Lumpable means that I can measure the same voltage regardless of the position of the voltmeters.

No! :) It does not mean only that! :)

Wikipedia is good enough https://en.wikipedia.org/wiki/Lumped_element_model (https://en.wikipedia.org/wiki/Lumped_element_model)

Quote
What we have is a loop antenna. There is a reason why loop antennas are connected by two-wire transmission lines.

Hold your horses! Don't introduce (radiating & leaky) antennas and transmission lines into talk about <= 300 Hz electromagnetism. [edit] Just for fun I tried first loop antenna calculator I can find. Results are: you can't put 300 Hz loop antenna on your table. Zero chances:
Quote
The specified conductor length of 1000000 meters is not ideal.
Conductor length should be between 121,250 and 242,500 meters at the specified frequency of 0.0003 MHz.


Other post (https://www.eevblog.com/forum/chat/does-kirchhoffs-law-hold-disagreeing-with-a-master/msg2084374/#msg2084374) of yours is unfortunately off-topic because it is not specified that there is Dr.Lewin's experiment in the box. It was defined that box is magnetically shielded and there is no EMF induced in the wires coming out of the box.
Title: Re: Does Kirchhoff's Law Hold? Disagreeing with a Master
Post by: Berni on January 02, 2019, 10:06:08 am
Why would you do that?  The changing magnetic field produces an electric field.  The electric field lines are circles around the magnetic field as shown in post #16:
https://www.eevblog.com/forum/chat/does-kirchhoffs-law-hold-disagreeing-with-a-master/msg2003414/#msg2003414 (https://www.eevblog.com/forum/chat/does-kirchhoffs-law-hold-disagreeing-with-a-master/msg2003414/#msg2003414)

Yes exactly this is the underlying effect due to Einsteins specials relativity, its what makes magnetism and radio waves work.

But notice how you do not need to have a closed loop in order to have a changing magnetic field induce a electric field in a wire. Depending on what definition of voltage you use you get that there is 0V across the ends of the wire (The textbook definition) or that the voltage is equal to the magnetic EMF (The engineering definition).

So if one was to separate the ends of the wire by 5mm and expose it to a magnetic field fast enough to induce 30kV of EMF in it, would the voltage arc over and jump the gap? Or does the loop have to be closed before current could start flowing and then slowly separated to draw an arc?

If you have electric field lines that form a loop, as in this case, you have a nonconservative field.

Yes if you keep it as a loop, but given the wire length has only one path inside of it (Its just a wire afterall) then you can unroll the loop along the length of the wire and you have something that looks exactly like a normal electrostatic field (That im sure we all agree are conservative).

Another way to think about it is that the path was strictly defined by the single wire so we now also have a strictly defined voltage along that path.

So you don't have two voltmeters connected to same point showing different voltages?  This is what happens in the experiment.  Or are we supposed to imagine that it doesn't?

The two voltmeters will only measure the same voltage if the voltmeters and their connecting wires are outside the field region or somehow shielded or arranged so that the field does not affect the measurement.  Fine, but that is not this experiment.  This experiment is set up to demonstrate Faraday's law and non conservative fields.

Well yes and no.

Yes i fully agree that the two voltmeters in Dr. Lewins experiment must show different voltages. I even did the experiment myself and i have shown the circuit working just fine in SPICE. And i fully agree with Dr. Lewins calculated result on the whiteboard where he shows two different voltages across points A and B. There indeed are two diferent voltages there.(In the textbook voltage definition!)

What i don't agree is that they are connected to the same two points. Because the voltmeter probe wires have a significant effect on the result means they can't be ignored in the circuit model. The voltmeters actually being connected to the ends of two different inductors (that represent probe wires), so in my view they are not actually connected to the same two points. A wire no longer acts as a ideal equipotential connection between two points when exposed to certain conditions such as this one (Even if its a superconductor).

You seem to be claiming that a voltmeter only measures an electric field caused by separation of charges.  For example, you have to move charges on the gate of the input FET of the voltmeter to make it read a voltage, I guess is what you mean.

But the voltmeter can't tell you what separated the charges.  It could be a battery, it could be a solar cell, it could be a thermocouple, it could be a hall effect device, it could be a loop of wire surrounding a changing magnetic flux, or it could be a loop of wire rotating in a static magnetic field, to name a few possibilities.  The voltmeter cannot distinguish the charge separation caused by the electrostatic scalar potential from the charge separation caused by all the different other types.

In this case, the wires connecting the voltmeter are passing through a region that has a non-zero electric field.  That electric field, interacting with the charge in the wire, will give you a different reading on the voltmeter depending on the path the wires take through that field.

Exactly that's the point i was making. Voltmeters have no idea what caused the voltage, they just see the effects of the voltage. The other "non textbook" definition is this "effective voltage". If you use that definition then suddenly it does not matter if the source of the voltage is a magnetic field, piezzo effect, thermocuple or a battery. If the black box is pushing 1A trough a 1Ohm resistor across its terminals then the answer is always there is 1V across the terminals.

This is why this "effective voltage" is used in circuit analysis, SPICE and every other case where you would want to actually do math on circuits.

None of this goes against Faraday or Maxwell, its just a slightly different mathematical path trough it that never gives ambiguous multiple results that give rise to this paradox.
Title: Re: Does Kirchhoff's Law Hold? Disagreeing with a Master
Post by: Sredni on January 02, 2019, 10:12:55 am
*You* have problem, not me. You say that resistor with voltage drop have zero E field inside. That means that integral E.dl over it is zero meaning that it does not have voltage drop on it! It's paradox, don't you see?

Indeed. But I never said that the resistor has zero E-field inside. I said that the circulation of the conservative part of the E field is zero.
All the resulting field is concentrated at the resistors (one in this case), I've said that many times in my previous posts. The circulation of the resulting field is not zero. The circulation of the conservative part of the resulting field, on the other hand, is zero. But that does not mean either that the path integral of the conservative part of the field is zero inside the resistor, as well.

Try to see it this way: the primary coil is generating a time-varying quasi-static magnetic B field (spatially uniform going up and down). This is turn, links a non conservative time-varying electric field (E_induced) in space. This field is directed along concentric circles (changing direction with time) and goes like r inside the primary coil boundary, and like 1/r outside of it.
Then you place to copper of the secondary coil with the resistor(s) in it.
The free electrons of the copper instantly (with relaxation times of the order or 10^-14 seconds) redistribute themselves in order to satisfy the constitutive equations j = sigma E which means that if sigma is infinite, we will end with a resulting E field in the copper that is zero. This means that the conservative part of the resultant E field is compensating, obliterating it completely, the induced part in the copper. In the resistors sigma is low, so there is a significant resultant E field inside the resistor.

The conservative part of the E field is stronger in the copper as well. It has to be in order to cancel the induced part.

I will come back with some drawings.

Quote
What?! TL;DR. You did not even answer.

Well, you did not even read!!!

Quote
  I will try again.
1) Does KVL hold when inside box is DC battery? 2) Does KVL hold when inside box is battery-powered AC generator? 3) Does KVL hold when inside box is piezo-based 1V AC voltage generator? 4) Does KVL hold when inside box is transformer?

1) yes, outside and inside
2) it depends. Does the generator have a time-varying B field inside ? Is so and if the flux is neatly tucked inside the box, then 'extended KVL' (which is Faraday under disguise) will appear to work outside, but won't work inside when you cross the flux-varying region.
3) I am not familiar with piezoelectric generators, but if there is no dphi/dt involved we probably can treat them as batteries.
4) 'new KVL' which is Faraday under disguise will appear to work outside and fail miserably inside if you attempt to cross the flux-varying region.
Title: Re: Does Kirchhoff's Law Hold? Disagreeing with a Master
Post by: Berni on January 02, 2019, 10:30:53 am
This is exactly why the other "less scientific" definition of voltage (The one about how many electrons there are in one spot) is used in circuit analysis and pretty much everywhere else where you need to actually calculate something.

You keep talking of that as if it were a different physical quantity, measured in different units. It's not. The 'scalar potential' that is part of the (V, A) pair is just the path integral of the conservative part of the total electric field.
It's as if you decomposed a mathematical function into its odd and even parts and than claimed that the odd part is a different, 'less mathematic' definition of function.

You can easily see where that decomposition comes from by writing Faraday's law (use dS for the differential element of area to avoid confusion with the vector potential A, and use E_total instead of E to highlight that it is the resulting field, superposition of coulombian and induced fields). Then express B in terms of the vector potential A,  turn the surface integral on the right into a path integral along the surface contour using Stokes (or "the rotor's") theorem. Now take the integral on the right to the left, changing sign. Bring everything inside the same path integral. You are left with a field whose circulation along a closed path is always zero.
Hey, that's a conservative field! Well of course, you have stripped the induced - non conservative - part from the total field.

Congratulations, it can be very useful, but it's only half of the story.

I will come back to this post with formulas and drawings when I will be able to scan.

You can have different results in the same quantity if that quantity is defined differently.

For example if we take something we all know well, Speed aka Velocity.
So you are running 10 m/s along a road. So your speed is 10m/s we can all agree
What if you are running 10 m/s along the back of a truck that's going 50m/s ? Okay now we need to actually use vectors to know how to add them together to get the speed along the ground. So suppose its in the same direction as the truck so we are doing 60m/s
But we are all on a planet that's going along the sun at 30500 m/s. Well okay now we have to define speed as being the difference in speed between the ground and you.
So we already have lots of different speeds we could be considered moving depending on how you look at the quantity of speed.
But then we start going faster, we send the truck along a tunnel of vacuum at 299792453 m/s (5m/s slower than the speed of light) and you run in the forward direction at 10m/s along the back of the truck and you suddenly are moving faster than the speed of light?! And yet again we have to redefine what speed is and take into account Einsteins special relativity, throwing out all the kinematics equations from highschool physics to get the right result. And the result depends on where you are standing and watching the guy running along the truck.
...
It gets messy even for something this simple (And this same velocity mess is partially responsible for making magnetic fields work trough the very same special relativity effect).


And yes exactly as you said i turned it into a conservative field by defining the path with a length of wire (More detail in 2nd response in previous post) and looking at what field would end up along the path given the magnetic field around it. Note that this doesn't need an area, just a path, but the whole thing can be simplified down to an area with Faradays law as long as the path is closed.
Title: Re: Does Kirchhoff's Law Hold? Disagreeing with a Master
Post by: ogden on January 02, 2019, 11:10:58 am
You say that resistor with voltage drop have zero E field inside. That means that integral E.dl over it is zero meaning that it does not have voltage drop on it! It's paradox, don't you see?

Indeed. But I never said that the resistor has zero E-field inside. I said that the circulation of the conservative part of the E field is zero.

You just pick one or another type of E field according to your agenda  :-DD

I think I have all the parts of the puzzle, all made by you. Now you can buckle-up for Part2.

Quote
The conservative part of the E field is stronger in the copper as well. It has to be in order to cancel the induced part.

Did you just say that Dr.Lewin is mistaken? - Because he claims that nonconservative field is zero inside copper coil (or secondary of transformer). Remember - resistance is so small that we ignore it, but there can't be conservative field in the conductor of zero resistance.

So, copper part have *only* nonconservative field, resulting voltage (integral E.dl) equals 1V. You agreed because you did not argue when I said that it is possible to measure 1V AC voltage on transformer secondary  w/o resistor connected. (It would be dumb to argue anyway).

In our "box + resistor" case resistor is outside magnetic field of the transformer, so Faraday's law cannot do anything about it, so there cannot be nonconservative fields in form of EMF inside it. What remains is conservative field which we already agreed exists in resistor, it's integral E.dl is 1V when connected to coil generating 1V.

According to logic above, sum of fields in copper and resistor, integral E_conservative.dl and integral E_nonconservative.dl is zero. [edit] At given time of observation obviously.

Quote
  I will try again.
1) Does KVL hold when inside box is DC battery? 2) Does KVL hold when inside box is battery-powered AC generator? 3) Does KVL hold when inside box is piezo-based 1V AC voltage generator? 4) Does KVL hold when inside box is transformer?

1) yes, outside and inside
2) it depends. Does the generator have a time-varying B field inside ? Is so and if the flux is neatly tucked inside the box, then 'extended KVL' (which is Faraday under disguise) will appear to work outside, but won't work inside when you cross the flux-varying region.
3) I am not familiar with piezoelectric generators, but if there is no dphi/dt involved we probably can treat them as batteries.
4) 'new KVL' which is Faraday under disguise will appear to work outside and fail miserably inside if you attempt to cross the flux-varying region.

Fact that you even try to answer those questions is hilarious by it's own :) We agreed that for voltmeter it does not matter what's inside the box, it cannot sort out electrons - they were moved by class-AB amplifier, little monkeys or Mr.Faraday.

[edit]  Typo correction using strikethrough
Title: Re: Does Kirchhoff's Law Hold? Disagreeing with a Master
Post by: Sredni on January 02, 2019, 12:23:13 pm
Did you just say that Dr.Lewin is mistaken? - Because he claims that nonconservative field is zero inside copper coil (or secondary of transformer).
He must have meant the total field.
Keep in mind that the left side of Faraday's equation, circulation of E.dl refers to the total field.

Of course it might happen that the conservative field be zero as well: if all fields are conservative, the conservative field is all you've got and in that case the conservative field will be zero inside a perfect conductor. It happens for example in electrostatics: point charge generates a conservative E-field, you place a piece of copper nearby, free charges on the copper surface redistribute to create a contribution that will erase the field inside the copper. So sum of conservative E field due to point charge plus conservative field due to free surface charge equals total conservative E field inside the conductor is zero. And I think it will work with batteries and a copper wire.
It's all about context.

Can you show where Lewin said that?

Quote
So, copper part have *only* nonconservative field, resulting voltage (integral E.dl) equals 1V. You agreed because you did not argue when I said that it is possible to measure 1V AC voltage on transformer secondary  w/o resistor connected. (It would be dumb to argue anyway).
No I do not agree and I have already explained how that can happen in one of my previous post (many pages back).
The fact that you can have a voltage at the open secondary does not imply that there is a net resultant E field in the conductor. Charge density varies wherever there are gradients of permeability and conductivity. This can be put into formulas but it require a bit of vector calculus you would refuse to read (IIRC I also said there was a bit of vector calculus involved). I will try nonetheless to scan a few formulas when I get off my bed.

Quote
In our "box + resistor" case resistor is outside magnetic field of the transformer, so Faraday's law cannot do anything about it, so there cannot be nonconservative fields in form of EMF inside it.

I have to stop you there: as long as you stay outside you can pretend that the contribute of -dphi/dt are either 'battery-like' emf of 'resistor-like' voltage drops. But when you get inside the box - and you have to get inside the box to compute the path integral of the total E-field along the circuit's path, you have to surrender this delusion and come to terms with Faraday.

Quote
According to logic above, sum of fields in copper and resistor, integral E_conservative.dl and integral E_nonconservative.dl is zero. [edit] At given time of observation obviously.

The above logic is flawed because it does not keep into account the right hand side of Faraday's equations.

circulation of E_conservative + circulation of E_nonconservative = -dphi/dt
circulation of E_conservative == 0 by definition!
circulation of E_nonconservative = -dphi/dt

Quote
Fact that you even try to answer those questions is hilarious by it's own :) We agreed that for voltmeter it does not matter what's inside the box, it cannot sort out electrons - they were moved by class-AB amplifier, little monkeys or Mr.Faraday.

We agree that voltmeters are not affected by the path of their probes when you are not allowed to cut through or run circles around the flux varying region. From the outside of a toroidal transformer it does not matter how I place my probes, I can even run circles around the transformer but since the flux is all inside, I will not be able to encircle any net flux: as much goes inside the surface delimited by my probes, will come out.

Title: Re: Does Kirchhoff's Law Hold? Disagreeing with a Master
Post by: ogden on January 02, 2019, 01:13:02 pm
Did you just say that Dr.Lewin is mistaken? - Because he claims that nonconservative field is zero inside copper coil (or secondary of transformer).
He must have meant the total field.
Keep in mind that the left side of Faraday's equation, circulation of E.dl refers to the total field.

Of course it might happen that the conservative field be zero as well: if all fields are conservative, the conservative field is all you've got and in that case the conservative field will be zero inside a perfect conductor. It happens for example in electrostatics: point charge generates a conservative E-field, you place a piece of copper nearby, free charges on the copper surface redistribute to create a contribution that will erase the field inside the copper. So sum of conservative E field due to point charge plus conservative field due to free surface charge equals total conservative E field inside the conductor is zero. And I think it will work with batteries and a copper wire.
It's all about context.

Can you show where Lewin said that?

You don't seem to understand what EMF actually is. Do you? Note that Dr.Lewin uses E-field to describe conservative field and EMF when he talks about nonconservative field. If you start "Is Kirchhoff's Loop Rule for the Birds?" video at around 15:10, you will hear it.

Quote
Quote
So, copper part have *only* nonconservative field, resulting voltage (integral E.dl) equals 1V. You agreed because you did not argue when I said that it is possible to measure 1V AC voltage on transformer secondary  w/o resistor connected. (It would be dumb to argue anyway).
No I do not agree and I have already explained how that can happen in one of my previous post (many pages back).

LOL. From which alternate universe your physics come from? When I measure 1V on the transformer terminals, you claim that it is not 1V actually?

Quote
Quote
In our "box + resistor" case resistor is outside magnetic field of the transformer, so Faraday's law cannot do anything about it, so there cannot be nonconservative fields in form of EMF inside it.

I have to stop you there: as long as you stay outside you can pretend that the contribute of -dphi/dt are either 'battery-like' emf of 'resistor-like' voltage drops. But when you get inside the box - and you have to get inside the box to compute the path integral of the total E-field along the circuit's path, you have to surrender this delusion and come to terms with Faraday.

Don't even say "Faraday" in area where there's no magnetic field! Also please leave resistor where it is - outside the box. I repeat: box is shielded and you don't know what's actually inside - monkey with piezo igniter, signal generator or transformer.

Quote
Quote
According to logic above, sum of fields in copper and resistor, integral E_conservative.dl and integral E_nonconservative.dl is zero. [edit] At given time of observation obviously.

The above logic is flawed because it does not keep into account the right hand side of Faraday's equations.

Leave Faraday's equations alone because their job is done when we know EMF voltage which is 1V. We are talking about fields now or integral E.dl of the various types of fields to be precise.

Quote
From the outside of a toroidal transformer it does not matter how I place my probes

Fine. No need to remind so often that voltmeter have probe wires.
Title: Re: Does Kirchhoff's Law Hold? Disagreeing with a Master
Post by: Sredni on January 02, 2019, 01:56:44 pm
Did you just say that Dr.Lewin is mistaken? - Because he claims that nonconservative field is zero inside copper coil (or secondary of transformer).
He must have meant the total field.
Keep in mind that the left side of Faraday's equation, circulation of E.dl refers to the total field.
--snip--
Can you show where Lewin said that?
If you start "Is Kirchhoff's Loop Rule for the Birds?" video at around 15:10, you will hear it.

What I hear is "electric field" and I agree: the total, resulting electric field in copper is negligible. Zero in a perfect conductor. As I have always said.
Where do you hear him saying that (in this particular circuit) the conservative electric field in copper is zero?

Quote
Quote
Quote
So, copper part have *only* nonconservative field, resulting voltage (integral E.dl) equals 1V. You agreed because you did not argue when I said that it is possible to measure 1V AC voltage on transformer secondary  w/o resistor connected. (It would be dumb to argue anyway).

No I do not agree and I have already explained how that can happen in one of my previous post (many pages back).
LOL. From which alternate universe your physics come from? When I measure 1V on the transformer terminals, you claim that it is not 1V actually?

No, I do not agree with the "copper part have *only* nonconservative field, resulting voltage (integral E.dl) equals 1V."
This is where you are mistaken.

Oh, the alternate universe I am in is the same universe Ramo, Whinnery and Van Duzer wrote that book that I keep suggesting to you. But it probably does not exist in your universe.

Quote
Don't even say "Faraday" in area where there's no magnetic field!

Why not?
There is no magnetic field outside the coils, and yet the voltmeters read different values when they are attached to the same points. That's Faraday in action. You can explain it with the curling electric field, the one that goes as 1/r. I really need to post a lot of pictures.
Title: Re: Does Kirchhoff's Law Hold? Disagreeing with a Master
Post by: ogden on January 02, 2019, 03:49:21 pm
If you start "Is Kirchhoff's Loop Rule for the Birds?" video at around 15:10, you will hear it.

What I hear is "electric field" and I agree: the total, resulting electric field in copper is negligible. Zero in a perfect conductor. As I have always said.
Where do you hear him saying that (in this particular circuit) the conservative electric field in copper is zero?

Where do you hear him saying in his videos *any* of two terms: "conservative" or "non-conservative"? He does not use such at all. You deleted/ignored my comment about EMF to suit your agenda and I am glad you did it.

In case anybody wonders why I do not continue discussion - because my job is done. Answer yourself simple question: what is expected result of "integral E.dl" between terminals of transformer or voltmeter? - Voltage. It was defined from the very beginning of "box + resistor" debate, 1V AC peak-peak. I just trapped Dr.Lewin's cultists and they took the bait.
Title: Re: Does Kirchhoff's Law Hold? Disagreeing with a Master
Post by: bsfeechannel on January 02, 2019, 04:25:55 pm
Other post[/url] of yours is unfortunately off-topic because it is not specified that there is Dr.Lewin's experiment in the box.

Oh! I didn't notice that the topic had suddenly become off-topic. My bad.
Title: Re: Does Kirchhoff's Law Hold? Disagreeing with a Master
Post by: Berni on January 02, 2019, 05:11:43 pm
To give things some better perspective i have put together a few images that graphically show the fields.

1) Circuit under test
(https://www.eevblog.com/forum/chat/does-kirchhoffs-law-hold-disagreeing-with-a-master/?action=dlattach;attach=611437;image)

R1 = 1 KOhm
R2 = 1 KOhm
Rwire = 0 Ohm

Yes this circuit is changed a bit but it makes the graphics easier since my colors don't quite have the dynamic range in available colors to make voltages visible that are an order of magnitude apart.

2) Charge Density
(https://www.eevblog.com/forum/chat/does-kirchhoffs-law-hold-disagreeing-with-a-master/?action=dlattach;attach=611443;image)

For a EMF of 1V the the scale shown has to be multiplied by 0.25V
The color shows charge density at every point in the circuit. This could also be considered a topographical map of the coulomb E field. The voltmeters are reading the difference in the E field across the terminals, so we can see that V1= -0.5V  and V2= +0.5V. So we do get the paradox behavior of two meters showing different numbers across the same two points.

These colors also directly correspond to the "non textbook" definition of voltage used in circuit analysis.

As you can also see the "ground reference" of the circuit is placed at node A. All voltages are measured in reference to A


3) Magnetic EMF
(https://www.eevblog.com/forum/chat/does-kirchhoffs-law-hold-disagreeing-with-a-master/?action=dlattach;attach=611449;image)

For 1V EMF the scaling factor here is 0.5V

So now lets also visualize the EMF in every point. This is a bit tricky because as we know its a non conservative field. To be able to show it the scale shown here actually wraps around (Similar as phase in phase diagrams). To get the voltage between two points you simply travel along the wire and check the difference between the starting and ending value. Due to the scale wraparound you can go around the circle multiple times and just keep adding more voltage each time you go around in that direction. If you go around it once you find you get 1V, go around twice you get 2V. These same voltages are induced in the outer probe loop.

Notice that the EMF voltage across the resistors and voltmeters is essentially zero due to them being physically small compared to the loop. This is an important fact and is the reason why voltmeters can't see EMF voltage. There is simply no significant noncosenrvative magnetically induced E field across them. Its the electrons on the wire that see most of it and get shoved into the terminals of the resistors and voltmeters, hence why only the end effect of this magnetic EMF is seen by components.

This is also where the gradients in the wires on Figure 2 have come from. They are simply the opposite of the EMF as the charge separation E field balances out with the EMF E field.


4) True voltage
(https://www.eevblog.com/forum/chat/does-kirchhoffs-law-hold-disagreeing-with-a-master/?action=dlattach;attach=611455;image)

So now lets combine the two and get the real voltage as formally defined.

Well... we have problems. Here is where the paradox comes up. Suddenly any piece of wire that is enclosing the magnetic flux is undefined (RED) or has multiple solutions. It depends how we traveled along the EMF Figure 3. To correctly color this in the pixels would need to have multiple colors at the same time (Sorry im not making an animation out of this)

The resistor and voltmeter have defined voltage across them because there is no EMF across them, so no matter what path we take the result is identical. As such i could color them in.

5) True-er? voltage
(https://www.eevblog.com/forum/chat/does-kirchhoffs-law-hold-disagreeing-with-a-master/?action=dlattach;attach=611461;image)

We can help ourselves a bit and use the claim that wires always have 0V across them, so if we have one part of a wire defined, then we can color in the rest of the wire with the same voltage and this is what we get. We still can't color in the small parts in the middle because suddenly multiple voltages meet there. And here is how we get to the multiple voltages at the same point result that Dr. Lewin demonstrated. It's a perfectly valid result and there are indeed two voltages present.

6) EMF in the bigger picture
(https://www.eevblog.com/forum/chat/does-kirchhoffs-law-hold-disagreeing-with-a-master/?action=dlattach;attach=611467;image)

Here i have shown the EMF outside the wires too.(This is part of how Fig 3 was created in the first place)

You can chose any two points (Here 1 and 2 for example) and read off the EMF voltage between them by just looking at the difference between them.

This can be also seen as a graphical demonstration of Faradays law:
(https://wikimedia.org/api/rest_v1/media/math/render/svg/0887c6b22873ec7cd5a3d9afe9f0fb38e2bd7930)

To calculate the same voltage with Faradays law you have to construct a surface area that you can put down there into the limits of that initgeral. This is done by first connect the two points to the magnetic fields origin (These are wires that have 0V EMF) and then draw any non intersecting shape of a line between those two points to close the loop. The size of the resulting E field according to Faradays law is this voltage shown by color in the picture.

Notice that the resistors never needed to be involved. This is because Faradays law only deals with the relationship between a changing magnetic and electric field. It has nothing to do with electric fields caused by charges, but since the integral of those fields around a closed loop is always zero it means it doesn't matter if you include them into Faradays law anyway. It still works fine. The reason it affects the coulomb electric field is that the electrons can't tell the two fields apart and will happily move themselves into equilibrium where they generate that opposite coulomb E field and this is what then drives electrons trough our voltmeters.


If anyone does not agree with the given diagrams you are welcome to modify them and color them in the way you think they should look.

EDIT: Embeded images properly
Title: Re: Does Kirchhoff's Law Hold? Disagreeing with a Master
Post by: ogden on January 02, 2019, 06:08:06 pm
Other post[/url] of yours is unfortunately off-topic because it is not specified that there is Dr.Lewin's experiment in the box.

Oh! I didn't notice that the topic had suddenly become off-topic. My bad.

Well, you cannot reply to discussion about shielded transformer with message that requires magnetically unshielded transformer!  :palm:

Again you demonstrated your tactic by removing part of my message that explains why your other post became off-topic:

Other pos of yours is unfortunately off-topic because it is not specified that there is Dr.Lewin's experiment in the box. It was defined that box is magnetically shielded and there is no EMF induced in the wires coming out of the box.
Title: Re: Does Kirchhoff's Law Hold? Disagreeing with a Master
Post by: rfeecs on January 02, 2019, 06:55:08 pm
To give things some better perspective i have put together a few images that graphically show the fields.

This is very nice.  I think we pretty much all agree that you can decompose the E field into conservative and non conservative parts.  Or fields due to charge separation and fields due to changing magnetic field.

You are actually not showing fields, you are trying to show potentials in different colors.  As you say, the potential is not defined in the case of a non conservative field.  Perhaps just showing E-fields would be easier, since they are defined everywhere and give all the information needed to describe what's happening.
Title: Re: Does Kirchhoff's Law Hold? Disagreeing with a Master
Post by: Berni on January 02, 2019, 07:26:10 pm
Yes its showing potentials hence why i was referring to it as sort of a topographical map of a E field.

All of these charts show E fields with the potentials in Volts. But the E field that is induced by a changing magnetic field is still non conservative hence why it was a bit tricky to show. Its E fields generated by charges that are conservative and always defined.

It also shows how you don't need a closed loop to have EMF in a wire. Hence why modeling each wire as a section around the loop is perfectly sensible. You only need a closed loop when you want to use Faradays law to directly calculate the EMF voltage. It doesn't mean that you can't have EMF without a closed loop just because Faradays law uses a loop area, it just means you have to use other laws to calculate your EMF in those cases.

All of this is why this circuit can be lump modeled just fine and used in circuit analysis(Where to the dismay of some in this thread KVL works)
Title: Re: Does Kirchhoff's Law Hold? Disagreeing with a Master
Post by: Sredni on January 02, 2019, 11:33:19 pm
WTF is the keyboard combination that can make you lose the whole post??
This site does not have a draft saving function? I was copying a ***** line and all went poof!
The ****! What the ****** *****!!!

Short version, then.

EDIT: No, not even that.

This post has been shortened and cleansed to avoid upsetting other children.
Whatever was written here can be found in one or more of the following books (in no particular order, and without mentioning the usual suspects Feynman, Purcell, Griffiths, Ohanian, Jackson):

Panofsky, Phillips
Classical Electricity and Magnetism 2nd ed

John Kraus
Electromagnetism 2nd to 4th ed

Ramo, Whinnery, VanDuzer
Fields and Waves in Communication Electronics 2nd or 3rd ed

Bleaney
Electricity and Magnetism 3rd ed

Nayfeh, Brussel
Electricity and Magnetism

Kip
Fundamentals of Electricity and Magnetism 2nd ed

Lorrain, Courson
Electromagnetic Fields and Waves 2nd ed
Title: Re: Does Kirchhoff's Law Hold? Disagreeing with a Master
Post by: Berni on January 03, 2019, 06:58:48 am
WTF is the keyboard combination that can make you lose the whole post??
This site does not have a draft saving function? I was copying a ***** line and all went poof!
The ****! What the ****** *****!!!

Yeah forum web software has not advanced much at all in the last 20 years.




In the case of the field produced by a primary cylindrical coil here is the induced e field magnitude:

(https://i.ibb.co/1RMrvkz/screenshot-11.png)

here is plotted with direction in a plane perpendicular to the axis of the infinitely long primary coil (here represented by the orange ring):

(https://i.ibb.co/2t2d8JF/screenshot-10.png)

Seems perfectly defined (of course it is time-varying but we can express exactly how - this is snapshot frozen in time). The voltage, on the other hand...


Sorry i did indeed word that a bit wrong.

I meant that the potential is undefined yes, it happens when fields form a loop like this.

I visualized the potential inside the field because its the easiest to see with color grading. If you want field vectors then just compare the color of a pixel to its neighbors to get your vector arrow. Drawing a sea of arrows wouldn't be as easy to see (and i don't have any quick way of doing that in CorelDraw while color grading is just a simple filter layer)


I'll skip the part regarding that paper you mentioned back when we had this discussion and that kept using AREA in computing the emf for a single line.
We're going 'full field' now, so forget areas - welcome boundary conditions!
You are able to plot the induced field only when you specify the boundary condition set by the coil generating the field.
After all Maxwell's equations in their differential form are... partial differential equations and if you do not specify boundary and initial conditions, how can you choose the solutions that suits your problem among the infinitely many? In their integral form, since the are essentially integral relations between areas and boundaries you have to specify... well, you know what. Maybe there is some equivalence hidden in there?

Quote
You only need a closed loop when you want to use Faradays law to directly calculate the EMF voltage. It doesn't mean that you can't have EMF without a closed loop just because Faradays law uses a loop area, it just means you have to use other laws to calculate your EMF in those cases.

Except the EMF is only half of the story.
When you put the secondary coil in, the charges in the conductors will rearrange to produce the coloumbian field and the resulting field depends on how you close (or do not close) the loop. After all E_total has the same direction of j. You can end up with a total E that is opposed to E_induced (it helps using a finite albeit big value for sigma). I had written something else in the lost post but **** it! Anyway I still need to scan my drawings so...

Alright then lets hear how you think one should calculate the charge density on the ends of a open wire in a changing magnetic field.

And so, what is the true voltage?
Because it seems to me that in your coloring analysis you skipped the Mabilde-McDonald voltage right away and went on calling "true voltage" other voltages.
If I understand correctly, what you call "Charge density" is the voltage definable as potential difference that is associated with the coloumbian aka conservative part of the total field. Why is not that the true voltage?

True voltage is voltage as per textbook definition of an integral of all forces acting on a electron along the path. This is definition you have to use to get the two voltages at one point paradox as Dr. Lewin demonstrates it.

I can't call the charge density approach true voltage because that is not how voltage is formally defined. But this is the voltage that all voltmeters detect and show as a result and it is never undefined so i tend to use this so called "effective voltage" or "conservative voltage" or "columbian voltage" or whatever you want to call it, just because its more useful to work with, while not breaking any of Faradays or Maxwells math.

Title: Re: Does Kirchhoff's Law Hold? Disagreeing with a Master
Post by: bsfeechannel on January 03, 2019, 10:33:08 pm
I can't call the charge density approach true voltage because that is not how voltage is formally defined. But this is the voltage that all voltmeters detect and show as a result and it is never undefined so i tend to use this so called "effective voltage" or "conservative voltage" or "columbian voltage" or whatever you want to call it, just because its more useful to work with, while not breaking any of Faradays or Maxwells math.

This post is wonderful. Because it is the declaration of what we suspected right from the beginning. The "effective/conservative/coulumbian" voltage is a matter of conviction. Of all infinite possibilities of detecting voltage between two points under a varying magnetic field, one is chosen as the "right" one because it's the one that coincides with what would be measured if you could replace the effect of the EMF by lumped generators (batteries, whatever).

Do you remember when I proposed the following "challenge" (https://www.eevblog.com/forum/chat/does-kirchhoffs-law-hold-disagreeing-with-a-master/msg2012129/#msg2012129)?

(https://www.eevblog.com/forum/chat/does-kirchhoffs-law-hold-disagreeing-with-a-master/?action=dlattach;attach=612775;image)

I asked one of the forum members to tell me what voltmeter was measuring the right voltage. He said V3. Because that coincided with what all voltmeters were measuring in this circuit I had proposed before.

(https://www.eevblog.com/forum/chat/does-kirchhoffs-law-hold-disagreeing-with-a-master/?action=dlattach;attach=612781;image)

Then I showed how difficult that would be (https://www.eevblog.com/forum/chat/does-kirchhoffs-law-hold-disagreeing-with-a-master/msg2014192/#msg2014192). Because a nanometric variation in the position of the meter would cause a detectable error on reasonably precise voltmeters. The voltage he thought to be correct could only be detected at an arbitrarily precise position.

(https://www.eevblog.com/forum/chat/does-kirchhoffs-law-hold-disagreeing-with-a-master/?action=dlattach;attach=612787;image)

Let's now change a little bit that picture, but not significantly in terms of its theoretical meaning.

(https://www.eevblog.com/forum/chat/does-kirchhoffs-law-hold-disagreeing-with-a-master/?action=dlattach;attach=612793;image)

V5 is how we measure the voltage on the secondary of any transformer under load since the beginning of its invention soon after Faraday discovered the phenomenon of induction in the early 19th century. I maintained V1 just because it is also physically possible, but that measurement has no meaning for the application in question. In this case we proposed that the resistors had the same value, so V1 = -V5.

Does it mean that we've been measuring the wrong voltage all this time? Because the voltage measured by V3 not only doesn't have any practical usefulness, but also would be impossible to be measured directly because we have an obstruction which is the solid core in the middle.

So that's what the "Kirchhoff always holds" and "the right way to measure a voltage" [under varying fields] leads to: absurd conclusions.

By the time people are trying to convince nature of the contrary (just this thread has been now running for 2 months), if this people had deigned to sit down, forgotten what they think they know about the subject, and studied electromagnetism for real, they'd be making sense of it without all that struggle.

People forget that the most important piece of equipment in any lab is the brain. And it needs constant upgrades.
Title: Re: Does Kirchhoff's Law Hold? Disagreeing with a Master
Post by: ogden on January 04, 2019, 05:20:43 am
In fact, when you bring a voltmeter inside the ring, you can measure any voltage you want between -0.1V and +0.9V

You can read +1V and -1V as well - if you short probe leads that go around solenoid. Some academic scientists and their worshippers may say "Look! I discovered that voltage is path-dependent" while actual "discovery" is just electromagnetic induction. Engineer will say: your probing sux :)

You can bend it as you want, truth is that voltmeter reading (but not actual voltage in the connection point) depends on the path of test leads, especially if they are placed in the varying magnetic flux. All this is very complicated way of teaching how to NOT measure voltage.

p.s. When you realize who your debate opponent is, you may see your discussion in a whole new light. (https://www.powerofpositivity.com/know-it-all-behaviors/)
Title: Re: Does Kirchhoff's Law Hold? Disagreeing with a Master
Post by: Berni on January 04, 2019, 07:11:40 am
Quote from: Berni
Yeah forum web software has not advanced much at all in the last 20 years.
Well, some web software has advanced. Stack Exchange for example, saves the drafts, upload images directly, has a nice TeX editor... Too bad they pissed me off with their censorship attitude. But, never mind.

Yeah true, but the strict moderation is what makes stack exchange so useful when you just want a quick, concise and correct answer to a technical question. The best answers float up to the top and some answers actually have a significant amount of effort behind them.


I am a bit at a loss, here. I thought you were looking for the true one and only voltage, the one that has is so uniquely defined that it can be expressed as a potential difference, independent of path. And yet you say that potential is undefined?
Are you referring only to the possibility to run more than one time around the core (in which case, we could overlook it, limiting ourselves to a full circle at most) or is this a more fundamental lack of uniqueness?
Because the way you used colors makes me believe you are selecting a particular class of paths to represent your color coded voltages, namely paths along the circle. You fix 0 at one node (A, IIRC) and then compute the path integral along the circle from A to another point P on the circle. Do we agree that this does not exclude the possibility that the voltage from A to P depends on the path?
Which one of the many voltages you have shown is the path-independent one?
(My guess is that it has to be the first one, the conservative one you called "charge density". And yet you say


What is considered true voltage is dependent on what side of a argument about KVL you are on.

I fully support both definitions of voltage. I was never trying to say that multiple voltages at the same points are impossible (I fully agree with Dr. Lewins two voltages when used with the appropriate definition of voltage). I was just trying to explain why that happens and why you can't use this kind of voltage in KVL.

It just so happens that the textbook definition of voltage is less useful since as you can see all my diagrams that use it there have some red(undefined) in it, while the dreadful "coulomb voltage" is always defined at every point while giving the exact same result on the voltmeters and resistors. It simply untangles the same math out of these undefined values by looking at potential in a different way.

Figure 3 has an explanation in text that you have to TRAVEL along the diagram and that the scale wraps around at +1 and -1. This is why you can go around multiple times and it matters in what direction you go. Going counter clockwise of course gives the opposite sign than going counterclockwise so path and direction DOES matter. This is why i could not simply overlay this on to Figure 4 to get true voltage. It matters what way around you went in Figure 3.

The textbook definition of voltage is indeed PATH DEPENDANT. No need to tell me that as i never claimed the opposite in this thread. Im claiming that columbian voltage is path independent. Its this difference between the two voltage definitions that causes the undefined potential problem.


Which seems to me an admission that... the true voltage is path dependent, as Lewin has always said. But let's forget about this for the moment. The part I am more interested in is the following

Quote
I can't call the charge density approach true voltage because that is not how voltage is formally defined. But this is the voltage that all voltmeters detect and show as a result and it is never undefined so i tend to use this so called "effective voltage" or "conservative voltage" or "columbian voltage" or whatever you want to call it, just because its more useful to work with, while not breaking any of Faradays or Maxwells math.

So to be clear, is this the voltage Mabilde is measuring? The one that Kirchhoffian call the 'true and one voltage'? You just decided to call it with another name (not a problem, we just need to be clear) but can you clarify that this is the case and that what you called "Charge Density" in the picture and call now "effective, conservative, coloumbian voltage" is the Mabilde-McDonald voltage?

Before addressing the meaning of said voltage, let me say that there is one problem with you statement above. You wrote that "that is the voltage that all voltmeters detect and show as a result". I am afraid you are mistaken.
To show the Mabilde-McDonald voltage you need a special measurement setup, consisting in a careful choice of your probes' path. The voltage all voltmeters detect is the one with the operational definition of path integral of E. The one that depends on the path of your probes when there is a changing magnetic field. In fact, when you bring a voltmeter inside the ring, you can measure any voltage you want between -0.1V and +0.9V and that is what the voltmeter show. Even outside you have two different values depending on how you place the probes around the core.
So, no. The 'effective, conservative, coloumbian voltage' is not the voltage that all voltmeters detect and show as a result. You did the experiment yourself!

But I think I know where you got that idea.
I will address that in a separate post, along with my answer to this


Yes Mabilde is measuring the "coulomb voltage" but also Dr. Lewin is measuring the "coulomb voltage" since that's the voltage that all oscilloscopes measure (Or did Tektronix make two special scopes for his experiment that measure both components of voltage together?).

Probes are part of the circuit and "coulomb voltage" is generated inside the probes wires as they are also exposed to the magnetic field, so no need to run your probes in a special way or put your voltmeter in a special spot. But you have to realize that voltage is there and needs to be considered in your results. If you are looking to get the potential at the ends of those wires rather than at the voltmeter terminals then you have to subtract out any voltage introduced by the probes. So the charge density at the end of those probe wires does indeed change as you have the probe wires take different paths, hence why the voltmeter shows a different voltage.

So if say voltmeters don't measure "coulomb voltage" what do they measure then? Maybe show it on an example circuit to give it some context (Like the simple single coil and single resistor we had before).


Quote
Alright then lets hear how you think one should calculate the charge density on the ends of a open wire in a changing magnetic field.


EDIT: fixed sentence I had left without conclusion.


Well? How do we calculate it then? Or is there no need to calculate it because it's zero perhaps?
Title: Re: Does Kirchhoff's Law Hold? Disagreeing with a Master
Post by: Sredni on January 05, 2019, 03:15:49 pm
Stack Exchange for example, saves the drafts, upload images directly, has a nice TeX editor... Too bad they pissed me off with their censorship attitude. But, never mind.
Yeah true, but the strict moderation is what makes stack exchange so useful when you just want a quick, concise and correct answer to a technical question. The best answers float up to the top and some answers actually have a significant amount of effort behind them.

Nah, the problem is twofold: first, science is not and should not be democratic - so the 'most voted answer is the best' is not necessarily true and the mechanism is flawed from the start. While it might work fine for some 90% of the content, when it comes to specialized stuff, it breaks miserably. This flaw is exacerbated by the editing power that one can exert even if he/she has no clue of what is being discussed (meaning the so called 'expert' in one branch might only have an approximate and sometimes erroneous knowledge of other branches). Imagine if you and ogden had the power to close this discussion because it's crystal clear that this is just a probing error (and you have more yellow square than me and MHz). Or to look if from the other side, if Mhz and I had the power to shut up Kirchhoffians by editing and removing their posts (and once they are removed, if the site has enough traffic there won't be enough people to notice or even care to reopen them).
Nobody Many readers in this blog would not have discovered the surprising role of surface charge in keeping the current within a conductor (I will get to that when I will have completed my drawings and scans).

I was happy with the way Physics SE is run: there is highly competent people making the selection there. Not so much in EE. Waste my time once, shame on you. Waste my time twice...
Scientific populism will be a problem a few years ahead.
But enough digressing.

Quote
Well? How do we calculate it then? Or is there no need to calculate it because it's zero perhaps?

We use Maxwell's equations. What else?
Zero? With an abrupt discontinuity at the ends and an induced field of known geometry? What makes you think that?
It is zero in a closed isotropic circular conducting torus perfectly aligned with the circular induced field. But even then, my guts say you just have to move it off axis to see charge pile up on the 'lateral' surface (well, is there any other surface on a torus?). And if you place portion of different resistance, you will certainly see charge pile up at the surface of separation.
I've found plenty of literature supporting my point of view. I need a little bit of time to sift through the best works and select a few images. Keep your popcorns in a warm place, I'm almost done with my flu.

edit: nobody was a truly bold statement. In my defense, it's all Jackson's fault. I'll explain later
Title: Re: Does Kirchhoff's Law Hold? Disagreeing with a Master
Post by: ogden on January 05, 2019, 03:58:55 pm
Suppose I put my electron in, with particular initial conditions (like velocity in a particular direction starting from A) and that I have recorded its trajectory from A to B. Would you agree in calling the voltage computed along that path the most meaningful voltage for this particular experiment?

No. I don't give a chit how fast that electron moves inside voltmeter. If it is faster than in the probe tip, then measurement is trash. I want to measure voltage between probe tips. If probe tips are shorted meaning voltage is 0V but I get 1V indication, I cannot call it meaningful by any stretch of imagination.

If voltage indication changes depending on how you manipulate with probe wires - you cannot trust your measurements not to mention make big scientific thing out of it.  |O

You can read +1V and -1V as well - if you short probe leads that go around solenoid. Some academic scientists and their worshippers may say "Look! I discovered that voltage is path-dependent" while actual "discovery" is just electromagnetic induction.

Oh, come on. I thought we were past that.
Voltage in non-conservative E-field IS path-dependent. This is not even up for discussion.

Yes. This is where I agree to you. I already said that and can repeat: "voltmeter reading (but not actual voltage in the connection point) depends on the path of test leads, especially if they are placed in the varying magnetic flux."
Title: Re: Does Kirchhoff's Law Hold? Disagreeing with a Master
Post by: Berni on January 05, 2019, 05:04:17 pm
Nah, the problem is twofold: first, science is not and should not be democratic - so the 'most voted answer is the best' is not necessarily true and the mechanism is flawed from the start. While it might work fine for some 90% of the content, when it comes to specialized stuff, it breaks miserably. This flaw is exacerbated by the editing power that one can exert even if he/she has no clue of what is being discussed (meaning the so called 'expert' in one branch might only have an approximate and sometimes erroneous knowledge of other branches). Imagine if you and ogden had the power to close this discussion because it's crystal clear that this is just a probing error (and you have more yellow square than me and MHz). Or to look if from the other side, if Mhz and I had the power to shut up Kirchhoffians by editing and removing their posts (and once they are removed, if the site has enough traffic there won't be enough people to notice or even care to reopen them).
Nobody Many readers in this blog would not have discovered the surprising role of surface charge in keeping the current within a conductor (I will get to that when I will have completed my drawings and scans).

I was happy with the way Physics SE is run: there is highly competent people making the selection there. Not so much in EE. Waste my time once, shame on you. Waste my time twice...
Scientific populism will be a problem a few years ahead.
But enough digressing.

Ah i never actually posted on there so i had no idea how that works, i just often take useful answers from there and i had good experience with that part.

Quote
Well? How do we calculate it then? Or is there no need to calculate it because it's zero perhaps?

We use Maxwell's equations. What else?
Zero? With an abrupt discontinuity at the ends and an induced field of known geometry? What makes you think that?
It is zero in a closed isotropic circular conducting torus perfectly aligned with the circular induced field. But even then, my guts say you just have to move it off axis to see charge pile up on the 'lateral' surface (well, is there any other surface on a torus?). And if you place portion of different resistance, you will certainly see charge pile up at the surface of separation.
I've found plenty of literature supporting my point of view. I need a little bit of time to sift through the best works and select a few images. Keep your popcorns in a warm place, I'm almost done with my flu.

edit: nobody was a truly bold statement. In my defense, it's all Jackson's fault. I'll explain later

Well i shown how to calculate the charge density on the end of a open wire with Faradays law, but apparently according to you that's wrong, so i would like to see what is the correct way of using Maxwells equations to find the result.

I was not claiming its zero, just guessing your answer because you keep saying wires always have no voltage across them (apart from restive drop). So you are now saying that wires can have voltage across them?
Title: Re: Does Kirchhoff's Law Hold? Disagreeing with a Master
Post by: bsfeechannel on January 05, 2019, 09:20:27 pm
Nah, the problem is twofold: first, science is not and should not be democratic - so the 'most voted answer is the best' is not necessarily true and the mechanism is flawed from the start. While it might work fine for some 90% of the content, when it comes to specialized stuff, it breaks miserably. This flaw is exacerbated by the editing power that one can exert even if he/she has no clue of what is being discussed (meaning the so called 'expert' in one branch might only have an approximate and sometimes erroneous knowledge of other branches). Imagine if you and ogden had the power to close this discussion because it's crystal clear that this is just a probing error (and you have more yellow square than me and MHz). Or to look if from the other side, if Mhz and I had the power to shut up Kirchhoffians by editing and removing their posts (and once they are removed, if the site has enough traffic there won't be enough people to notice or even care to reopen them).
Nobody Many readers in this blog would not have discovered the surprising role of surface charge in keeping the current within a conductor (I will get to that when I will have completed my drawings and scans).

Now I understand what you mean. This tendency is really dangerous and detrimental to any kind of knowledge.

Quote
But enough digressing.

No way. This is highly pertinent to the present discussion.

Quote
Keep your popcorns in a warm place, I'm almost done with my flu.

The things we do to help poor souls to walk on the path of salvation.
Title: Re: Does Kirchhoff's Law Hold? Disagreeing with a Master
Post by: bsfeechannel on January 05, 2019, 09:32:27 pm
If probe tips are shorted meaning voltage is 0V but I get 1V indication, I cannot call it meaningful by any stretch of imagination.

Maybe nature wants to tell you something you are ignoring.

Quote
If voltage indication changes depending on how you manipulate with probe wires - you cannot trust your measurements not to mention make big scientific thing out of it.  |O

Well, Michael Faraday, a scientist, had the same problem that you do way back in 1831. Then he banged his head, like you did, until he realized he was in front of a new phenomenon that no one had noticed before.

When he published his findings, people started to test his claims and found they were true. They soon realized the potential of that new discovery and started to take advantage of it, instead of trying to deny it.

And this changed the lifestyle of most of us since then.

Quote
I already said that and can repeat: "voltmeter reading (but not actual voltage in the connection point) depends on the path of test leads, especially if they are placed in the varying magnetic flux."

But if you don't like what your voltmeter is telling you because of the arrangement of its probes, you can replace it by a radio receiver or a lamp.

(https://www.eevblog.com/forum/chat/does-kirchhoffs-law-hold-disagreeing-with-a-master/?action=dlattach;attach=614485;image)
(https://www.eevblog.com/forum/chat/does-kirchhoffs-law-hold-disagreeing-with-a-master/?action=dlattach;attach=614491;image)
(https://www.eevblog.com/forum/chat/does-kirchhoffs-law-hold-disagreeing-with-a-master/?action=dlattach;attach=614497;image)
If Kirchhoff always held, you couldn't have the modern world and we wouldn't be able to have this very conversation. Learn to live with that.
Title: Re: Does Kirchhoff's Law Hold? Disagreeing with a Master
Post by: ogden on January 05, 2019, 10:28:19 pm
If probe tips are shorted meaning voltage is 0V but I get 1V indication, I cannot call it meaningful by any stretch of imagination.

Maybe nature wants to tell you something you are ignoring.

It's *you* who are ignoring, not me. I said (https://www.eevblog.com/forum/chat/does-kirchhoffs-law-hold-disagreeing-with-a-master/msg2089864/#msg2089864): "You can read +1V and -1V as well - if you short probe leads that go around solenoid. Some academic scientists and their worshippers may say "Look! I discovered that voltage is path-dependent" while actual "discovery" is just electromagnetic induction."

Quote
Well, Michael Faraday, a scientist, had the same problem that you do way back in 1831.

I say "it's electromagnetic induction", you say: "no! - it's electromagnetic induction!". You apparently have problem, not me.

Quote
If Kirchhoff always held, you couldn't have the modern world and we would be able to have this very conversation. Learn to live with that.

LOL. Have to repeat that you are "arguing (https://www.eevblog.com/forum/chat/does-kirchhoffs-law-hold-disagreeing-with-a-master/msg1979633/#msg1979633) against the nonexistent strawman who is apparently suggesting that Farady's law is incorrect, and Kirchoffs is always correct", like broken record.
Title: Re: Does Kirchhoff's Law Hold? Disagreeing with a Master
Post by: Berni on January 06, 2019, 12:30:09 am

LOL. Have to repeat that you are "arguing (https://www.eevblog.com/forum/chat/does-kirchhoffs-law-hold-disagreeing-with-a-master/msg1979633/#msg1979633) against the nonexistent strawman who is apparently suggesting that Farady's law is incorrect, and Kirchoffs is always correct", like broken record.

I have to agree with Ogden. Who are you actually arguing with?

To make things easier i will summarize most of my claims in a list:
1) There are indeed two voltages present at the measured points in Dr. Lewins experiment when using the formal textbook definition of voltage.
2) The formal definition of voltage is path dependant
3) Faradays law is correct, however the E field in that equation refers only to the non conservative E field generated by the magnetic field on the other side of the equation. No other E fields are included on the left side (Tho you can include the conservative E field too since it integrates to zero anyway).
4) Voltmeters can only measure charge density across the terminals and can't detect the non conservative component of voltage around the whole loop, only observe the effects of this non conservative field in the form of charge separation.
5) If voltmeters ware capable of measuring voltage as formally defined then measuring the voltage across a transformer secondary would always result in 0V with the secondary terminals open, or when the transformer is powering a load the voltage would be the restive drop in the wingdings.
6) In Dr. Lewins circuit charge density is always defined as a single number for all points at any given moment, giving every point of the circuit a defined "effective voltage"
7) Closed loops of wire with a defined area are not a requirement for having induction happen. Open loops of wire can experience induction and even act like LC tank circuits.
8 ) Open lengths of wire in a changing magnetic field indeed have zero textbook voltage along them, but in most cases do have a different charge density at the ends giving them "effective voltage" that is capable of being detected with voltmeters or in extreme cases make electrons arc across gaps.
9) Lengths of wire connecting the voltmeter to the probing points are part of the circuit and need to be analyzed along with the rest of the circuit. These wires transfer the voltage from the probing points to the voltmeter terminals where it is actually measured. If it is found that these wires generate a voltage that affects the voltmeters reading then this voltage must be subtracted out to get the voltage at the probe points. Failure to realize this, correct it, or compensate for it is considered as "bad probing".
10) Changing the path of the probe wires in Dr. Lewins circuit does change the voltmeter reading due to changing the charge density present on the voltmeter terminals. However when doing correct probing as mentioned above the result of the voltage at the probing points it always the same, regardless of wire path or voltmeter location (The effect is always substracted out).
11) Kirchhoffs circuit laws always work in circuit mesh models where all voltages use the "effective voltage" definition
12) Kirchhoffs cirucit laws can not be directly applied to just any real life circuit with the assumption of ideal wires, especially when high frequency AC signals are involved or significant magnetic effects are present
13) Kirchoffs voltage law does not contain an intergal of E as Dr. Lewin shows. Its actually a algebraic sum of all voltages on components and as such can only be used on a lumped model.
14) Kirchoffs cirucit laws do not go against Faradays law or Maxwells equations. All three can exist without conflict. Faradays law and KVL describe two different things and as such are not mutually exclusive.
15) Kirchoffs citucit laws have nothing to do with Maxwells equations, but they are used together whenever circuit analysis is used on reactive components such as inductors or capacitors.
16) The circuit from Dr. Lewins experiment can easily be lump modeled using multiple coupled inductors to represent wires. As such all common methods of circuit analysis can be applied to it including KVL to get results matching the real physical experiment
17) The inductor lump models can be split any number of times and distributed around the loop to expose any point of interest in the circuit.
18) Circuit mesh models assume there is no flux outside of individual components, however coupled inductors models can be used to get this flux sharing behavior when desired.
19) The "effective voltage" is just as real as the formal textbook voltage, yet more useful due to being always defined and shown by real life voltmeters.


I might have missed a few but these are the ones i remember right now. Any disagreement on these?
Title: Re: Does Kirchhoff's Law Hold? Disagreeing with a Master
Post by: ogden on January 06, 2019, 02:15:41 am
I might have missed a few but these are the ones i remember right now. Any disagreement on these?

I would add definition of voltage to 2 ). Version "True voltage is integral of all forces acting on a electrons along the path" is kinda tricky because some may think that electron(s) shall be carried all the way along the path for voltage to appear, which is untrue. IMHO worth to mention more straightforward "the work needed per unit of charge to move a test charge between the two points". It also "connects" better to volt = joule/coulomb equation.

I disagree with 8 ) because both "all forces on electrons along the path" and "work per unit of charge" voltage definition variants allows voltage to be present on terminals of zero resistance "open lengths of wire" or inductor. After all there's well-known equation: V = L(di/dt)

Kinda offtopic or maybe not. -How special relativity is related to this discussion:


https://youtu.be/1TKSfAkWWN0 (https://youtu.be/1TKSfAkWWN0)

p.s. How to (easily) avoid this gigantic thumbnail appearing when I include URL to YT video?
Title: Re: Does Kirchhoff's Law Hold? Disagreeing with a Master
Post by: Sredni on January 06, 2019, 02:34:41 am
Making the pictures is taking more time than I had thought and I have work to do.
EDIT: But then...

This post has been shortened and cleansed to avoid upsetting other children.
Whatever was written here can be found in one or more of the following books (in no particular order, and without mentioning the usual suspects Feynman, Purcell, Griffiths, Ohanian, Jackson):

Panofsky, Phillips
Classical Electricity and Magnetism 2nd ed

John Kraus
Electromagnetism 2nd to 4th ed

Ramo, Whinnery, VanDuzer
Fields and Waves in Communication Electronics 2nd or 3rd ed

Bleaney
Electricity and Magnetism 3rd ed

Nayfeh, Brussel
Electricity and Magnetism

Kip
Fundamentals of Electricity and Magnetism 2nd ed

Lorrain, Courson
Electromagnetic Fields and Waves 2nd ed
Title: Re: Does Kirchhoff's Law Hold? Disagreeing with a Master
Post by: Sredni on January 06, 2019, 03:00:55 am
To make things easier i will summarize most of my claims in a list:
1)
...
19)

I might have missed a few but these are the ones i remember right now. Any disagreement on these?

If you were a computer, I'd reboot you.  :)
(I've edited one of the posts in the previous page with the origin of the scalar and vector potentials, just in case it could help in sorting some of the points out)
Title: Re: Does Kirchhoff's Law Hold? Disagreeing with a Master
Post by: Berni on January 06, 2019, 10:12:38 am
I might have missed a few but these are the ones i remember right now. Any disagreement on these?

I would add definition of voltage to 2 ). Version "True voltage is integral of all forces acting on a electrons along the path" is kinda tricky because some may think that electron(s) shall be carried all the way along the path for voltage to appear, which is untrue. IMHO worth to mention more straightforward "the work needed per unit of charge to move a test charge between the two points". It also "connects" better to volt = joule/coulomb equation.

I disagree with 8 ) because both "all forces on electrons along the path" and "work per unit of charge" voltage definition variants allows voltage to be present on terminals of zero resistance "open lengths of wire" or inductor. After all there's well-known equation: V = L(di/dt)

Kinda offtopic or maybe not. -How special relativity is related to this discussion:

p.s. How to (easily) avoid this gigantic thumbnail appearing when I include URL to YT video?

Yeah good point 2) should have a definition of voltage included to make sure another definition doesn't get assumed. The wording "integral of all forces on electrons" doesn't assume a single electron but rather the average contribution from all of them, but yeah the wording of "work per unit charge needed to move along the path" is actually a bit more accurate and unambiguous.

For 8 ) Yes V = L(di/dt) is sort of a "patch" to make voltage appear on the terminals of transformers when using the textbook definition of voltage. It assumes a lumped inductor and places this voltage across its terminals even tho the voltage inside the inductor is ether zero, the resistive loss, or undefined. So by moving a nanometer into the transformer terminals you get the 0V solution but the moment you step on the edge of a transformer terminal this "patch formula" comes into effect and suddenly you have lots of voltage.
https://en.wikipedia.org/wiki/Voltage#Definition (see bottom of section "Definition as potential of electric field")
If you keep reading you get to the alternative definition of voltage that i call "effective voltage" or "charge density voltage". This definition does not require the patch formula from before to make voltage appear on transformer secondaries. There simply is a voltage all throughout the length of the transformers winding as well as on the terminals with no sharp transition anywhere. (Tho this L(di/dt) formula is still incredibly useful for modeling inductors in circuit analysis). This is a much more elegant solution in my opinion (And as a bonus is never undefined or path dependent).


Oh and yes that is an excellent video for explaining how and why magnetism works in a very intuitive way. Every electronics engineer should watch it at some point. (And no i don't think you can prevent embedding of youtube links)




...you keep saying wires always have no voltage across them (apart from restive drop). So you are now saying that wires can have voltage across them?

You can have voltage at the terminals, with zero field and zero voltage inside the secondary. (EDIT: of course in case sigma = infinity, so what I called copper has to be thought as a perfect conductor - otherwise, we would see minimal resistive losses and a negligible E complying with j = sigma E).
These are just the essential pictures of a long story that starts with electrostatics, so consider this a sneak preview (and please do not mind too much at signs, they were not my priority here)

Consider a single loop secondary and a closed IMAGINARY, MATHEMATICAL closed path going through the copper and joining the terminals. Like this

(https://i.ibb.co/QdgY9Hs/screenshot.png)

How does a transformer work? By applying Faraday. Not Kirchhoff, Faraday. I have a closed path, it defines an area. Let's do it!


Yes you are going to have lots of trouble calculating this with Kirchhoffs circuit laws, those laws don't care about inductors of any form. Its like trying to undo a bolt with a screwdriver, its simply not meant for doing that. All that Kirchhoffs cirucit laws are used for is defining the behavior of voltage and current in cirucits. Its the job of circuit modeling to plug the correct voltage equation into KVL when dealing with inductors to tell it what the voltage on a inductor is. (See my claim 15)


(https://i.ibb.co/WWbdXNT/screenshot-3.png)

Ok, nice formula, how do I fit it into my circuit?
Let's decompose the closed path into two partial open paths and see what we can get out of that:

(https://i.ibb.co/qBhYxxc/screenshot-4.png)

We've got this:

(https://i.ibb.co/hVCCqHC/screenshot-5.png)

and the notion that you can have zero field and zero voltage inside the wire of the secondary coil, while having voltage at the terminals. Try to explain this with KVL.


Yet if you use the conservative definition of voltages KVL suddenly works across all of this circuit without having to define a sharp point where the coil of wire stops being an inductor and becomes a normal wire where in a infinitesimally small area the voltage suddenly shoots from 0V to whatever voltage Faradays law says it should have. What if i move that transition point somewhere else?
Title: Re: Does Kirchhoff's Law Hold? Disagreeing with a Master
Post by: ogden on January 06, 2019, 10:39:52 am
For 8 ) Yes V = L(di/dt) is sort of a "patch" to make voltage appear on the terminals of transformers when using the textbook definition of voltage. It assumes a lumped inductor and places this voltage across its terminals even tho the voltage inside the inductor is ether zero, the resistive loss, or undefined. So by moving a nanometer into the transformer terminals you get the 0V solution but the moment you step on the edge of a transformer terminal this "patch formula" comes into effect and suddenly you have lots of voltage.

Let's put aside L(di/dt) which is not "patch" at all, but pay attention to what happens with electrons, thus charge in the coil during flux change. Charge is pushed to the one end of the coil. I can't see how resulting voltage on the terminals could be zero.
Title: Re: Does Kirchhoff's Law Hold? Disagreeing with a Master
Post by: Berni on January 06, 2019, 11:44:09 am
For 8 ) Yes V = L(di/dt) is sort of a "patch" to make voltage appear on the terminals of transformers when using the textbook definition of voltage. It assumes a lumped inductor and places this voltage across its terminals even tho the voltage inside the inductor is ether zero, the resistive loss, or undefined. So by moving a nanometer into the transformer terminals you get the 0V solution but the moment you step on the edge of a transformer terminal this "patch formula" comes into effect and suddenly you have lots of voltage.

Let's put aside L(di/dt) which is not "patch" at all, but pay attention to what happens with electrons, thus charge in the coil during flux change. Charge is pushed to the one end of the coil. I can't see how resulting voltage on the terminals could be zero.

Yes i also don't like how that works but that's how the formal textbook definition of voltage makes it work.

When you have a wire in a changing magnetic field that field can induce the non conservative E field along it. Because the electrons in a wire are free to move they start marching in the direction that field is pushing them. As a result they end up bunched up at one end of the wire. But when electrons are bunched up like that they create there own E field. This field opposes the magnetically induced E field and the electrons keep marching along until they are creating an electrostatic E field that exactly opposes the magnetically induced one. Now all the fields around the electrons sum up to zero so they stay still in there cozy equilibrium point.

So now if you take the formal definition of voltage (Work needed to move a unit of charge along a path) you will find that zero force is needed to move an electron along the wire because they is no force acting against you. As such by the formal definition of voltage there is 0V along the wire. If you connect a load to the terminals of this coil then the voltage becomes undefined because you get a different result if you travel between the terminals along the path going trough the coil and a different result if you traverse the path by going trough the load. This is the whole reason why Dr. Lewin is not wrong by saying there are two voltages across the points in his circuit and the root cause for what we are arguing about in this thread.

The alternative is to instead use the "effective voltage" definition that is basically just the difference in charge density at each point. This simply ignores the troublesome noncoservative E field along the wire and instead just sees the effect of it in the form of charge separation. If you look at the coil of wire again using this definition now you find that as you travel along the wire you find electrons bunched up together more and more and as such the voltage smoothly increases along the coil until reaching its maximum value at the terminals. Connecting a voltmeter to the terminals measures this exact voltage, or connecting a load to the terminals pushes a current trough it according to Ohms law, path does not matter anymore and we always have exactly one solution for the voltage across the coils terminals.

This is why i think the "effective voltage" definition makes more sense and is more useful for calculations. Under this definition KVL works just fine too.

The problem is that the first definition is considered to be the formal definition of voltage and as such i can't argue against using it. I can't just ignorantly say "That's not what voltage is" just because its not something i would want voltage to be defined as. So we have to just deal with having two definitions of voltage, one of them causes the paradox in Dr. Lewins experiment, the other does not. I fully support both definitions of it, but my personal honest opinion is that the "effective voltage" definition is more useful.
Title: Re: Does Kirchhoff's Law Hold? Disagreeing with a Master
Post by: ogden on January 06, 2019, 12:51:28 pm
When you have a wire in a changing magnetic field that field can induce the non conservative E field along it. Because the electrons in a wire are free to move they start marching in the direction that field is pushing them. As a result they end up bunched up at one end of the wire. But when electrons are bunched up like that they create there own E field. This field opposes the magnetically induced E field and the electrons keep marching along until they are creating an electrostatic E field that exactly opposes the magnetically induced one. Now all the fields around the electrons sum up to zero so they stay still in there cozy equilibrium point.

More electrons bunched at the one end than another equals potential difference. That "cozy equillibrium point" happens when capacitor connected to wire loop is finished charging. You may want to say "there's no capacitor" - read my comments below.

Quote
So now if you take the formal definition of voltage (Work needed to move a unit of charge along a path) you will find that zero force is needed to move an electron along the wire because they is no force acting against you. As such by the formal definition of voltage there is 0V along the wire.

Moving electrons to one end of the wire will create opposing magnetic field, opposing force. This means that to move electron, nonzero work shall be done. So there's your definition of voltage that predicts voltage.

Quote
If you connect a load to the terminals of this coil then the voltage becomes undefined because you get a different result if you travel between the terminals along the path going trough the coil and a different result if you traverse the path by going trough the load.

As wire loop can't be infinitely small (area enclosed by the loop shall be > 0), it will be some kind of capacitor as well - you like it or not. So there's your load - capacitor that becomes charged. When magnetic field does not change anymore, this "parasitic capacitor" immediately discharges through low resistance of the wire.
Title: Re: Does Kirchhoff's Law Hold? Disagreeing with a Master
Post by: Sredni on January 06, 2019, 01:38:43 pm

 :popcorn:







(my turn)
Title: Re: Does Kirchhoff's Law Hold? Disagreeing with a Master
Post by: Berni on January 06, 2019, 02:27:47 pm
More electrons bunched at the one end than another equals potential difference. That "cozy equillibrium point" happens when capacitor connected to wire loop is finished charging. You may want to say "there's no capacitor" - read my comments below.

Yes there is indeed capacitance there. This is why i claim a open length of wire can act as a LC circuit (As said in my claim no. 7)

My point is that since the magnetic EMF is distributed along the wire you also get a smooth gradient of charge density along the wire. Its not like the charge density along the wire is constant but then at the very end of the wire there are lots of bunched up electrons while on the other end there are missing electrons.

The textbook definition of voltage makes it seam like you only have excess electrons at the very end while they are perfectly evenly distributed between the ends. But this is not the case.

Moving electrons to one end of the wire will create opposing magnetic field, opposing force. This means that to move electron, nonzero work shall be done. So there's your definition of voltage that predicts voltage.

Yes moving an electron does create a magnetic field around it, but the sea of free electrons inside the wire gets pushed around in a way that creates an exactly opposing electrostatic E field. This is where the idea comes from that it is impossible to have a E field inside a superconductor. Any magnetically induced E field gets  canceled out by electrons rearranging to create a opposing electrostatic E field. Force exerted on electrons comes from the sum of all E fields and with the field being zero the force is also zero. Alternativly you could also explain it as the electrostatic E field acting on it while a magnetic field acts on it in the opposing direction. You can't have the induced E field and the magnetic field simultaneously acting on the electron. This induced E field is simply the interpretation of the magnetic fields effect on charges (due to Einsteins special relativity). This induced field is not a real E field, its sort of a virtual E field caused by the interaction of charges with magnetic fields. So you ether imagine the effect as moving charges experiencing force when moving trough a magnetic field, or as an a moving magnetic field creating this virtual E field trough the effects of special relativity and this E field then produces force on electrons.

Lets just let Sredini, bsfeechannel,rfeecs... etc explain and advocate for this kind of voltage definition. I honestly don't like this definition myself (its messy and confusing) so i rather not explain it more than necessary. I just wanted to say that when these forum members are talking about there being no voltage in a wire when in a changing magnetic field, they are actually correct if you use the formal textbook definition of voltage.


I'm not going to say they are wrong just because i don't agree with this being a good definition of voltage. It is a widely accepted definition written down in countless literature. I was just commenting on why i think this is not a very good definition of voltage and why the "effective voltage" makes more sense to me. I prefer the "effective voltage" definition due to making more sense, but im not going to say the other definition is wrong just because i don't like it.

Even if i don't agree with some other stuff they claim, im not going to say its wrong just because i don't like it. But i will certainly argue about things that i think are genuinely wrong according to other facts.


As wire loop can't be infinitely small (area enclosed by the loop shall be > 0), it will be some kind of capacitor as well - you like it or not. So there's your load - capacitor that becomes charged. When magnetic field does not change anymore, this "parasitic capacitor" immediately discharges through low resistance of the wire.

Yes i fully agree.

Title: Re: Does Kirchhoff's Law Hold? Disagreeing with a Master
Post by: Sredni on January 06, 2019, 03:14:28 pm
I might have missed a few but these are the ones i remember right now. Any disagreement on these?

I would add definition of voltage to 2 ). Version "True voltage is integral of all forces acting on a electrons along the path" is kinda tricky because some may think that electron(s) shall be carried all the way along the path for voltage to appear, which is untrue. IMHO worth to mention more straightforward "the work needed per unit of charge to move a test charge between the two points". It also "connects" better to volt = joule/coulomb equation.

This post has been shortened and cleansed to avoid upsetting other children.
Whatever was written here can be found in one or more of the following books (in no particular order, and without mentioning the usual suspects Feynman, Purcell, Griffiths, Ohanian, Jackson):


Kip
Fundamentals of Electricity and Magnetism 2nd ed

Lorrain, Courson
Electromagnetic Fields and Waves 2nd ed

John Kraus
Electromagnetism 2nd to 4th ed

Ramo, Whinnery, VanDuzer
Fields and Waves in Communication Electronics 2nd or 3rd ed

Panofsky, Phillips
Classical Electricity and Magnetism 2nd ed

Bleaney
Electricity and Magnetism 3rd ed

Nayfeh, Brussel
Electricity and Magnetism

Title: Re: Does Kirchhoff's Law Hold? Disagreeing with a Master
Post by: ogden on January 06, 2019, 05:28:26 pm
What do you see?
Or, better yet: what do you NOT see?

There are no voltmeters.
There are no probes.

There is no spoon.

And yet, the voltage between A and B can have two different values.

If you are right - then why there is voltage on transformer terminals? There's same voltage - you measure with 10MOhm voltmeter alone or with 1K resistor in parallel. Please explain why "Romer's loop" wire have 0V on it's ends but any other transformer nonzero voltage? It's question to explain transformer, not to profile me from psychological or educational point.
Title: Re: Does Kirchhoff's Law Hold? Disagreeing with a Master
Post by: Sredni on January 06, 2019, 05:58:38 pm
If you are right - then why there is voltage on transformer terminals?

I... I... I thought I just explained that.
Ok, let me answer with a question.

EDIT: On second thought, no.

This post has been shortened and cleansed to avoid upsetting other children.
Whatever was written here can be found in one or more of the following books (in no particular order, and without mentioning the usual suspects Feynman, Purcell, Griffiths, Ohanian, Jackson):

Panofsky, Phillips
Classical Electricity and Magnetism 2nd ed

John Kraus
Electromagnetism 2nd to 4th ed

Ramo, Whinnery, VanDuzer
Fields and Waves in Communication Electronics 2nd or 3rd ed

Bleaney
Electricity and Magnetism 3rd ed

Nayfeh, Brussel
Electricity and Magnetism

Kip
Fundamentals of Electricity and Magnetism 2nd ed

Lorrain, Courson
Electromagnetic Fields and Waves 2nd ed

"Books" are static paper based documents that can be found in libraries. They are like smartphones, but (usually) bigger, with lots and lots of extremely thin flexible e-ink screens and a very long battery life. Libraries are...
Oh, never mind. Keep on pushing that square peg into that round hole. With a big enough hammer, it will fit.


Title: Re: Does Kirchhoff's Law Hold? Disagreeing with a Master
Post by: Berni on January 06, 2019, 06:02:55 pm
I do have to agree with Sredini on his post( https://www.eevblog.com/forum/chat/does-kirchhoffs-law-hold-disagreeing-with-a-master/msg2096638/#msg2096638 (https://www.eevblog.com/forum/chat/does-kirchhoffs-law-hold-disagreeing-with-a-master/msg2096638/#msg2096638) ). This is indeed how voltage works when its used as is formally defined with "The work needed to move a unit of charge along a path"

The reason that the voltmeter is reading a voltage is that its essentially a amp meter with a 10MOhm resistor inside, because this resistor is only a tiny part of the whole loop (or is not even inside the magnetic field) means it is not being affected by the the non conservative E field induced by the magnetic field, but is seeing all of the electrostatic E field caused by charge separation at the transformers terminals. So if we integrate the E field across this 10M resistor we are basically only seeing the conservative electrostatic E field. This causes a current to flow trough it according to Ohms law and the voltmeter displays the voltage needed to push that current.

Its not wrong, its just a more complicated roundabout way of getting to the answer.

Alternatively you could simply use the "effective voltage" definition that only looks at the charge density and ignores the non conservative field all together. Then you always have voltage on the transformer terminals no matter what you do (apart from turning the transformer off). This voltage is spread over all the turns of the coil in the transformer and if desired you can tap into it at any point to give you any voltage in between. No need to coax this voltage into existence by connecting a load to the transformer. Its simply there all the time and you can poke your voltmeter at it to see it.

This is why i prefer the "effective voltage" flavor of voltage much like ogden does, it just simply makes more sense.

Title: Re: Does Kirchhoff's Law Hold? Disagreeing with a Master
Post by: bsfeechannel on January 06, 2019, 06:27:58 pm
Some academic scientists and their worshippers may say "Look! I discovered that voltage is path-dependent" while actual "discovery" is just electromagnetic induction."

Bad probing is the lame excuse that people not comfortable with electromagnetism give when academic scientists set up an experiment to explain exactly WHY voltage is path dependent under a varying magnetic field.

A probing technique that suppresses the very effect you are trying to demonstrate is called facepalm probing.
Title: Re: Does Kirchhoff's Law Hold? Disagreeing with a Master
Post by: ogden on January 06, 2019, 06:56:42 pm
If you are right - then why there is voltage on transformer terminals?

The field outside will not be zero, though.


Thank you.  :clap:
Title: Re: Does Kirchhoff's Law Hold? Disagreeing with a Master
Post by: jesuscf on January 06, 2019, 07:54:40 pm

And yet, the voltage between A and B can have two different values.


At the same exact time?  Maybe you have to take into account the magnetic field generated by the flowing current through the wire...
Title: Re: Does Kirchhoff's Law Hold? Disagreeing with a Master
Post by: bsfeechannel on January 06, 2019, 08:12:50 pm
I have to agree with Ogden. Who are you actually arguing with?

Read below.

Quote
To make things easier i will summarize most of my claims in a list:

Thank you.

Quote
1) There are indeed two voltages present at the measured points in Dr. Lewins experiment when using the formal textbook definition of voltage.

[snip]

9) Lengths of wire connecting the voltmeter to the probing points are part of the circuit and need to be analyzed along with the rest of the circuit. These wires transfer the voltage from the probing points to the voltmeter terminals where it is actually measured. If it is found that these wires generate a voltage that affects the voltmeters reading then this voltage must be subtracted out to get the voltage at the probe points. Failure to realize this, correct it, or compensate for it is considered as "bad probing".
10) Changing the path of the probe wires in Dr. Lewins circuit does change the voltmeter reading due to changing the charge density present on the voltmeter terminals. However when doing correct probing as mentioned above the result of the voltage at the probing points it always the same, regardless of wire path or voltmeter location (The effect is always substracted out).

The "correct probing" is a technique to avoid UNWANTED induction. But Lewin's experiment is exactly to show how voltage is dependent on the path under induction. So the voltage induced by the probes is PART of the experiment. You cannot subtract it out!

If you set up an experiment and employ a probing technique to suppress the very effect you are trying to demonstrate, you are on dope.

Quote
11) Kirchhoffs circuit laws always work in circuit mesh models where all voltages use the "effective voltage" definition
12) Kirchhoffs cirucit laws can not be directly applied to just any real life circuit with the assumption of ideal wires, especially when high frequency AC signals are involved or significant magnetic effects are present
13) Kirchoffs voltage law does not contain an intergal of E as Dr. Lewin shows. Its actually a algebraic sum of all voltages on components and as such can only be used on a lumped model.

Aw, man! Don't do that. What do you think an integral is? You clearly have no idea that integrating the electric field along a path of lumped components will result exactly in the algebraic sum of all voltages on the components.

For the record, Richard Feynman and others use the line integral with lumped circuits to demonstrate Kirchhoff's law.

Quote
14) Kirchoffs cirucit laws do not go against Faradays law or Maxwells equations. All three can exist without conflict. Faradays law and KVL describe two different things and as such are not mutually exclusive.

If they describe two different things, they are mutually exclusive.

Quote
15) Kirchoffs citucit laws have nothing to do with Maxwells equations, but they are used together whenever circuit analysis is used on reactive components such as inductors or capacitors.

Kirchhoff's law can be deduced from Maxwell's equations. This is classic electromagnetism.

Quote
16) The circuit from Dr. Lewins experiment can easily be lump modeled using multiple coupled inductors to represent wires. As such all common methods of circuit analysis can be applied to it including KVL to get results matching the real physical experiment

No circuit under varying magnetic fields can be lumped modeled. Please read the Feynman lectures recommended by Mehdi.

Quote
I might have missed a few but these are the ones i remember right now. Any disagreement on these?

We are not here to reach an agreement. We are here to ascertain the truths of electromagnetism.
Title: Re: Does Kirchhoff's Law Hold? Disagreeing with a Master
Post by: Sredni on January 07, 2019, 05:40:24 pm
And yet, the voltage between A and B can have two different values.
At the same exact time?

Yep. That's a snapshot at a given time.
The field will oscillate going one way, then the other, at - I do not remember exactly, maybe... 300 Hz?
At any rate, well within the limit of quasi static electrodynamics.

Quote
Maybe you have to take into account the magnetic field generated by the flowing current through the wire...

Nope, self-inductance is negligible, and there are no retardation effects.

The same two points, at the same moment in time, can have different voltages between them.
And there are no voltmeters, no probes, no measurement errors.
That's just the way it is.

Now, do I get to beat Mehdi, like in old Iran?  >:D
What would Jesus do?
Title: Re: Does Kirchhoff's Law Hold? Disagreeing with a Master
Post by: Berni on January 07, 2019, 06:45:00 pm

The "correct probing" is a technique to avoid UNWANTED induction. But Lewin's experiment is exactly to show how voltage is dependent on the path under induction. So the voltage induced by the probes is PART of the experiment. You cannot subtract it out!

If you set up an experiment and employ a probing technique to suppress the very effect you are trying to demonstrate, you are on dope.


Exactly it gets rid of unwanted effects such as the probe wires needing to follow a certain path, but it does not get rid of what you are measuring.

You still get the same result in Dr. Lewins experiment if you use the formal definition of voltage when subtracting out the probes. It just so happens that he set the probe wires in such a path that you need to subtract 0V to get the result. If you move the wires you get a diferent result on the voltmeters. Does that mean that the voltage across A and B has changed? No you just messed up your probing.

If you compensate out probing effects you can place both voltmeters on the left side in Dr. Lewins circuit and still get 2 different voltages as a result. If you do all your probe compensation math with textbook voltage you get two different values for voltage no matter where the voltmeters or the wires are.

If you use the "efective voltage" in the math to calculate the error voltage on the probes to subtract out you get the same result on both voltmeters no matter where they are. (Just like here you could just place one voltmeter in the middle for this error voltage to be 0V and thus make no need to compensate it out)

Correct probing practices don't break Dr. Lewins two voltages across A and B experiment, but given that the path the probe wires take in Dr. Lewins physical experiment is important it should be said why the probe wires take the path they do. This particular path requires no compensation of probe error for what he is trying to measure, all other paths do.

Aw, man! Don't do that. What do you think an integral is? You clearly have no idea that integrating the electric field along a path of lumped components will result exactly in the algebraic sum of all voltages on the components.

For the record, Richard Feynman and others use the line integral with lumped circuits to demonstrate Kirchhoff's law.


Well in the lecture where he talks about it he uses the summa operator:
http://www.feynmanlectures.caltech.edu/II_22.html#Ch22-S3 (http://www.feynmanlectures.caltech.edu/II_22.html#Ch22-S3)

He also explains why analyzing circuits as lumped is a good idea in the section above the one linked.


If they describe two different things, they are mutually exclusive.


They would be mutually exclusive if they would explain the SAME thing as being two different things.

Kirchhoffs circuit laws describe voltage and current relationships in circuit meshes. Maxwells equations describe the relationships of electromagnetic fields in our universe.


Kirchhoff's law can be deduced from Maxwell's equations. This is classic electromagnetism.


You certainly can, here is how: https://physics.stackexchange.com/questions/102458/how-can-kvl-kcl-be-derived-from-maxwell-equations (https://physics.stackexchange.com/questions/102458/how-can-kvl-kcl-be-derived-from-maxwell-equations)

However as you can see the equation you get as a result looks rather messy. This is sort of the physical world incarnation of Kirchhoffs law, but it does work with magnetic fields present, since the Maxwells equations that it came from also work fine with magnetic fields present.

Kirchhoff only stays so beautifully simple when you keep it within circuit meshes where it was meant to be used. Hence why it is so useful there.


No circuit under varying magnetic fields can be lumped modeled. Please read the Feynman lectures recommended by Mehdi.


I certainly agree for cases when the formal definition of voltage is used. Or in the case that you are not allowed to use coupled inductors in circuit models, i sure hope that is not the case since that makes modeling transformers really tricky (And Dr. Lewins experimental circuit is just a glorified transformer)

We are not here to reach an agreement. We are here to ascertain the truths of electromagnetism.

Well in that case we can close the thread cause Maxwell beat us to the goal of ascertaining the truths of electromagnetism by a good 150 years.
Title: Re: Does Kirchhoff's Law Hold? Disagreeing with a Master
Post by: Berni on January 08, 2019, 05:47:52 pm
What probe wires???
There are no probes wires in the computation of the path integrals I've shown above.
The two different values we get, for the two possibile path along the circuit, are the result of induction. But that's how the system is. If you remove the induced part of the field, you are analyzing a different (unrealistic) system.

It's as if you subtracted the field generated by the point charge near a piece of copper to come to the conclusion that there is a nonzero field inside the metal (and then came up with tiny generators inside the metal) that produce the observed surface charge.

The ones that connect his oscilloscope to points A and B, since the BNC connector on a scope does not conveniently have the exact contact spacing to touch the two points of interest.

Im not saying that the two voltages result is due to the probe wires. I am trying to say that the path that the probe wires take is important. In Dr. Lewins experiment if you move the probe wires into different paths you get a different result on the oscilloscope, hence why understanding the effects of probing is important. The path his wires take in that case is such that the formal voltage difference between the voltage of interest and the scope terminals is zero. If you take probing into account you can run the wires in any path you want and get the same result. Still two voltages across A and B.

In the case of point charges this extra field eventually sorts it self out since its conservative so it adds up to zero once you get around the loop.
Title: Re: Does Kirchhoff's Law Hold? Disagreeing with a Master
Post by: bsfeechannel on January 08, 2019, 08:48:48 pm
Now, do I get to beat Mehdi, like in old Iran?  >:D
What would Jesus do?

Well, the first crucifixion recorded by history was performed by the Persians in 522 a.C. So I guess Jesus may be a little bit furious as of now.

While Jesus calms down, perhaps it's time for Mehdi to issue a huge apology to the engineering and scientific community for trying to give credit to pseudo-scientific claims.
Title: Re: Does Kirchhoff's Law Hold? Disagreeing with a Master
Post by: bsfeechannel on January 08, 2019, 09:21:57 pm
Exactly it gets rid of unwanted effects such as the probe wires needing to follow a certain path, but it does not get rid of what you are measuring.

You still get the same result in Dr. Lewins experiment if you use the formal definition of voltage when subtracting out the probes. It just so happens that he set the probe wires in such a path that you need to subtract 0V to get the result. If you move the wires you get a diferent result on the voltmeters. Does that mean that the voltage across A and B has changed? No you just messed up your probing.

Here resides all your struggle with reality.

When you "right-probe" something you are not aware that what you are doing is exactly to make your probes follow a certain path.

In Lewin's experiment, we have a WANTED field. This is the field that's generating the EMF to power the resistors. So if you "right-probe" that circuit you will cancel out the effect of this EMF on your meter, treating this field as UNWANTED, and you are going measure nonsense.

Quote
If you compensate out probing effects you can place both voltmeters on the left side in Dr. Lewins circuit and still get 2 different voltages as a result. If you do all your probe compensation math with textbook voltage you get two different values for voltage no matter where the voltmeters or the wires are.

If you use the "efective voltage" in the math to calculate the error voltage on the probes to subtract out you get the same result on both voltmeters no matter where they are. (Just like here you could just place one voltmeter in the middle for this error voltage to be 0V and thus make no need to compensate it out)

Correct probing practices don't break Dr. Lewins two voltages across A and B experiment, but given that the path the probe wires take in Dr. Lewins physical experiment is important it should be said why the probe wires take the path they do. This particular path requires no compensation of probe error for what he is trying to measure, all other paths do.

And what is the significance of that voltage? Is it the EMF? No. Is it the voltage across one of the components? No. It's just an arbitrary voltage defined by an arbitrarily precise positioning of your probes. And that's exactly what Lewin wanted to prove.

You are confounding the concept with the technique. You take the "right probing" technique as a dogma, and you think that the technique can overrule the concept that created it. Unbelievable!

Quote
Well in the lecture where he talks about it he uses the summa operator:
http://www.feynmanlectures.caltech.edu/II_22.html#Ch22-S3 (http://www.feynmanlectures.caltech.edu/II_22.html#Ch22-S3)

Yes. He does.

(https://www.eevblog.com/forum/chat/does-kirchhoffs-law-hold-disagreeing-with-a-master/?action=dlattach;attach=617467;image)

And what is on the left side of the equals sign? A sea horse? A pregnant, elongated, slanted S? Or is it a line integral?

Any five year old kid knows that two things separated by an equals sign are equivalent. So the algebraic sum of the voltages around a circuit is equal to the line integral of the electric field around the path determined by the circuit (in the case where there are no varying fields as Feynman explains earlier).

Quote
He also explains why analyzing circuits as lumped is a good idea in the section above the one linked.

Provided you can LUMP THEM! You can only lump model a circuit if, and I quote, there is no magnetic field in the region outside the individual circuit elements. It's in the text. Didn't you read it? Or you need us to repeat it to you like a broken record every day? If you want to know why, read Feynman's previous chapters.

Lewin's circuit HAS a magnetic field outside the components, so it is UNLUMPABLE.

Quote
They would be mutually exclusive if they would explain the SAME thing as being two different things.

The text you linked is cool because it is here where Feynman derives Kirchhoff from Maxwell, showing that Kirchhoff is just a special case. All the complicated math he had already done in the previous chapters to come right to this point.

So you cannot argue anymore that Kirchhoff is one thing and Maxwell another. The proof is in the pudding.

Quote
Kirchhoffs circuit laws describe voltage and current relationships in circuit meshes. Maxwells equations describe the relationships of electromagnetic fields in our universe.

And where do you think our circuit meshes live? Outside the universe? Perhaps in the heavens? I can't believe I'm reading this.

Quote
You certainly can, here is how: https://physics.stackexchange.com/questions/102458/how-can-kvl-kcl-be-derived-from-maxwell-equations (https://physics.stackexchange.com/questions/102458/how-can-kvl-kcl-be-derived-from-maxwell-equations)

However as you can see the equation you get as a result looks rather messy. This is sort of the physical world incarnation of Kirchhoffs law, but it does work with magnetic fields present, since the Maxwells equations that it came from also work fine with magnetic fields present.

So we do have two Kirchhoff's laws now? One version is the "physical world incarnation of" it. The other is the regular one.

We have then three valid theories to describe exactly the same phenomenon: Maxwell, "physical" Kirchhoff and "regular" Kirchhoff. Wow! 

Quote
Kirchhoff only stays so beautifully simple when you keep it within circuit meshes where it was meant to be used. Hence why it is so useful there.

Wouldn't be beautifully simpler if we could understand that there's ONLY ONE theory to describe electricity and magnetism: Maxwell, and that Kirchhoff is just special case of it?

Quote
I certainly agree for cases when the formal definition of voltage is used.

Your brain has already realized that the cozy shell you created around your thoughts are no longer holding the water. The clear sign of it is the invention of the "effective voltage", "textbook-defined voltage", "physical Kirchhoff", "good probing technique" and other mental bodges to conciliate your set of axioms with the conflicting reality.

This thread is interminable because every time you try to make sense of your "theory" someone shows you a contradiction.

A theory can only hold if it is not contradictory.

Get rid of this as soon as possible, before someone gets hurt (including yourself).

Quote
Or in the case that you are not allowed to use coupled inductors in circuit models, i sure hope that is not the case since that makes modeling transformers really tricky (And Dr. Lewins experimental circuit is just a glorified transformer)

Every animal is vulnerable during the molt process. That happens to us when we need to get rid of concepts we thought were truths but aren't. Give time to yourself. Learn Maxwell piecemeal.

Quote
Well in that case we can close the thread cause Maxwell beat us to the goal of ascertaining the truths of electromagnetism by a good 150 years.

Cool! That's what I'm trying to do since 28 November when I told ogden to get better education (and I was subsequently called a troll). What we need to do now is to shut up and learn Maxwell (me included).
Title: Re: Does Kirchhoff's Law Hold? Disagreeing with a Master
Post by: Sredni on January 08, 2019, 10:20:14 pm
Now, do I get to beat Mehdi, like in old Iran?  >:D
What would Jesus do?
Well, the first crucifixion recorded by history was performed by the Persians in 522 a.C. So I guess Jesus may be a little bit furious as of now.

WHAT? IT'S MEHDI'S FAULT JESUS WAS KILLED???
GET HIM!

Jokes aside, I did not want to start a real religion war.
Maybe we should set for 100 lashes with a wet noodle.
But first I have to tie a few loose ends, and fix the signs in my integrals (also, I have to remove that circle from all surface integrals, what was I thinking?). Who knows, maybe I'll be able to convince Berni...

As you said, we have all had our Maxwell crisis. It has to take its course.
Title: Re: Does Kirchhoff's Law Hold? Disagreeing with a Master
Post by: bsfeechannel on January 09, 2019, 01:33:42 am
Maybe we should set for 100 lashes with a wet noodle.

Paraphrasing Get Smart, zis is EEVblog, ve don't lash here (I always wanted to use that catch phrase).

For those who will read this thread in the years to come it is good to make it clear that Sredni's joke is a reference to Mehdi's joke in one of his videos about not living in Iran and thus being exempt from corporal punishment and about the fact that Youtube is banned there.

Mehdi's claims about Kirchhoff's laws were promptly and peremptorily debunked by serious understanding of physics and engineering. That's how we "punish" around here.

Title: Re: Does Kirchhoff's Law Hold? Disagreeing with a Master
Post by: ogden on January 09, 2019, 06:55:48 am
Cool! That's what I'm trying to do since 28 November when I told ogden to get better education (and I was subsequently called a troll).

You think you are not worthy of this title? Look for yourself how many times you managed to insult Berni in single post! You were bitterly arrogant against him through most of this discussion, yet he never pushed back
Title: Re: Does Kirchhoff's Law Hold? Disagreeing with a Master
Post by: CM800 on January 09, 2019, 10:48:00 am
Arguing about unwanted and wanted fields... putting the 'P' in PhD.

I side with Mehdi on this.

The fields are effecting the probes, the only reason why the voltage changes is due to how the probe wires are affected by the probes. In such a way the laws are put in place, we talk of an ideal circuit.

If I was to make the same mistake while trying to measure a current sensor's voltage while wrapping the wires around the probe, I'd be laughed out of the room.
Title: Re: Does Kirchhoff's Law Hold? Disagreeing with a Master
Post by: bsfeechannel on January 09, 2019, 01:43:27 pm
You think you are not worthy of this title? Look for yourself how many times you managed to insult Berni in single post! You were bitterly arrogant against him through most of this discussion, yet he never pushed back

When Berni shows that you can deduce Kirchhoff from Maxwell and says that Kirchhoff doesn't have anything to do with Maxwell, it insults my intelligence. And makes me very angry.

It insults the intelligence of any reader of this forum.

Instead of sayin, hey, you know, I have a little difficulty understanding electromagnetism, could you help me with this? Or, I always thought that Kirchhoff always holds, can you explain why it isn't so? He keeps blurting stupid assertions like that to justify his gross errors and misconceptions and absolute lack of study of one of the staples of EE.

Sredni, I and others voluntarily spend time to try to bring the most accurate and didactic explanations.

So, who is arrogant? Who is the troll? Someone who makes personal sacrifices and sincerely wants to help a fellow engineer stop espousing nonsense, or someone who refuses to learn and on top of that invents the most incoherent excuses again and again to remain in ignorance?

I have a lot to learn about Maxwell and electromagnetism, but instead of  transforming my ignorance in some kind of precept to publish on forums, I do what you and Berni should do: I try to learn.
Title: Re: Does Kirchhoff's Law Hold? Disagreeing with a Master
Post by: bsfeechannel on January 09, 2019, 01:49:28 pm
Arguing about unwanted and wanted fields... putting the 'P' in PhD.

I used those terms for humorous effect. There are varying fields that I don't want to interfere with my measurement so I cancel out their effects when probing. But I can't cancel out the effect of the very field I want to measure.

This is stupid.

Quote
I side with Mehdi on this.

Mehdi is my friend. But truth is a better friend.

Quote
The fields are effecting the probes, the only reason why the voltage changes is due to how the probe wires are affected by the probes. In such a way the laws are put in place, we talk of an ideal circuit.

If I was to make the same mistake while trying to measure a current sensor's voltage while wrapping the wires around the probe, I'd be laughed out of the room.

Lewin's demonstration is a reductio ad absurdum. He assumes for a moment that Kirchhoff always holds, and then shows that if you keep thinking like that you are going to reach the wrong conclusions.

This thread is a monument to that truth.
Title: Re: Does Kirchhoff's Law Hold? Disagreeing with a Master
Post by: Berni on January 09, 2019, 05:38:58 pm
About this:
(https://www.eevblog.com/forum/chat/does-kirchhoffs-law-hold-disagreeing-with-a-master/?action=dlattach;attach=617467;image)

Yes in order to do this you need the components to be lumped. Something you claim can not be done for any circuit in a magnetic field.

As for "correct probing" no it does NOT mean canceling out the EMF on the voltmeter. Its about canceling out EMF on the wires leading to the meter. If you are trying to get voltage in the formal textbook definition then Dr. Lewins wire path is such a path that gives 0V over the probe wires, if you are trying to measure the conservative component of voltage then then such a path is both wires heading towards the origin of the field. If you calculate the EMF on the probe wires and subtract it then you can place the wires in any path you like and still get the correct result. So you ether place your wires so that there is no error due to probing, or you calculate the error and subtract it out. So Dr. Lewin is indeed using correct probing to measure the thing he wants to measure, but it is never explained that the probing is just as important as the circuit to get the correct result.

As for there being two forms of Kirchhoffs law:
(https://www.eevblog.com/forum/chat/does-kirchhoffs-law-hold-disagreeing-with-a-master/?action=dlattach;attach=618160;image)

This is what you get when you turn maxwells equations into KVL. Notice that it has an extra voltage in it. That is because Maxwells equations work fine with magnetic fields around so deriving KVL from it also gets you a form of KVL that works with magnetic fields. If you use this form of KVL in Dr. Lewins example it works out just fine, nothing surprising about that. It is very similar to the form of KVL we all know, but its not the same. When you are using it in cirucit meshes you have to throw away Vi because its impossible to calculate it due to circuit meshes not having any fields inside them. Without Vi you get the well known form of KVL that's meant for use in circuit meshes, but as Dr. Lewin has demonstrated breaks in the real world. Cirucit meshes are NOT supposed to be the underlying workings of the universe. they are just a close enough approximation that works pretty much the same, yet it faster to work with.

Its the similar as the classical kinematics equation we all know from physics:
d=s*t

This equation is WRONG! It only works in a universe where the speed of light is infinite or in one where space and time are not related to each other in the form of a spacetime field. So is that for the birds too? But the thing is that at any reasonable speeds we might encounter on earth the error in the result due to ignoring special relativity is pretty much in the parts per million or even smaller. So we use it anyway because it gives results that are still within margin of error, yet its much easier and faster to work with. In fact most physics equations we see in highschool only work in this fictional universe with infinite, speed of light, no atmosphere or drag and spherical cows. Yet a lot of these cut down formulas are still close enough to the real deal to be perfectly usable. Circuit meshes are the same sort of thing, not quite real but real enough for what they are supposed to do.

If you are going to use the classical simple form of KVL use it in circuit meshes where it indeed always works. If you want your circuit mesh to behave like the real circuit in the universe we live in then also use proper circuit modeling methods (where wires are modeled as having inductance). If you don't want to do that then don't just directly slap on KVL and expect it to work every time.

Instead you can use the version of KVL that is derived from Maxwell equations in the physical world, since that does work. Calculate it however you want, just don't carelessly mix formulas from our universe and formulas from circuit meshes. A lot of the times they work fine, but not always (As Dr. Lewin clearly demonstrates)

If you can't handle abstraction then just ignore circuit meshes and focus on pure Maxwells equations instead.

I can make sense of Dr. Lewins circuit both in the form of fields and in the form of a circuit mesh model. Both work just fine and give identical results. If you can't make sense of the circuit using a mesh model then try to learn how, otherwise don't complain about it being wrong just because you don't seam to understand it. I don't want to come across rude or anything, but any answer to why its wrong to mesh model this circuit is along the lines of "It can't be done because i said so" rather than getting an explanation why i am getting the right results out of my mesh model despite it being supposedly wrong for some mysterious reason.
Title: Re: Does Kirchhoff's Law Hold? Disagreeing with a Master
Post by: Berni on January 09, 2019, 05:48:20 pm
I do agree with Sredini here.

Tho yeah if your goal is to measure the voltage at the terminals of the current sensor then warping the probe wires around it is a rather non optimal way of doing it. Unless its a rather high gain type of sensor (Like a high ratio current transformer of a heavily amplified HAL), then it doesn't really matter since your scopes front end is probably too noisy and drifty to notice.
Title: Re: Does Kirchhoff's Law Hold? Disagreeing with a Master
Post by: ogden on January 09, 2019, 06:49:49 pm
EDIT: Bernie, it's Sredni, not Sredini.

I am Sredni Vashtar the beautiful.
My thoughts are red thoughts
and my teeth are white.
My enemies call for peace,
but I bring them death.


So try not to piss me off, uh?  :D

https://en.wikipedia.org/wiki/Narcissism (https://en.wikipedia.org/wiki/Narcissism)
Title: Re: Does Kirchhoff's Law Hold? Disagreeing with a Master
Post by: Berni on January 09, 2019, 07:00:10 pm
Ah sorry for the typo in your name Sredni
Title: Re: Does Kirchhoff's Law Hold? Disagreeing with a Master
Post by: In Vacuo Veritas on January 09, 2019, 07:29:12 pm
The real question is: would it measure the same in space, or indeed, Mars?

I mean the Earth is just a rock with nothing on it. Those rocks in space however, are the Future. We better figure out how to measure voltages there.
Title: Re: Does Kirchhoff's Law Hold? Disagreeing with a Master
Post by: Sredni on January 10, 2019, 11:58:04 am
The real question is: would it measure the same in space, or indeed, Mars?

I mean the Earth is just a rock with nothing on it. Those rocks in space however, are the Future. We better figure out how to measure voltages there.

Dude, I've heard Canada is experimenting with making marijuana legal.
How's it going so far?
 :D
Title: Re: Does Kirchhoff's Law Hold? Disagreeing with a Master
Post by: jesuscf on January 10, 2019, 03:58:43 pm
And yet, the voltage between A and B can have two different values.
At the same exact time?

Yep. That's a snapshot at a given time.
The field will oscillate going one way, then the other, at - I do not remember exactly, maybe... 300 Hz?
At any rate, well within the limit of quasi static electrodynamics.

Quote
Maybe you have to take into account the magnetic field generated by the flowing current through the wire...

Nope, self-inductance is negligible, and there are no retardation effects.

The same two points, at the same moment in time, can have different voltages between them.
And there are no voltmeters, no probes, no measurement errors.
That's just the way it is.

Whatever you say man.  I just want to point out what "Engineering Electromagnetics" by Hyat and Buck, Seventh Edition, says in page 94:

"Equation (21) (Curl integral of E.dl=0) is therefore just a more general form of Kirchhoff's circuital law for voltages, more general in that we can apply it to any region where an electric field exists and we are not restricted to a conventional circuit composed of wires, resistances, and batteries.  Equation (21) must be amended before we can apply it to time-varying fields.  We shall take care of this in Chapter 10, and in Chapter 13 we will them be able to establish the general form of Kirchhoff's voltage law for circuits in which currents and voltages vary with time."

Title: Re: Does Kirchhoff's Law Hold? Disagreeing with a Master
Post by: bsfeechannel on January 10, 2019, 06:03:52 pm
As for there being two forms of Kirchhoffs law:
(https://www.eevblog.com/forum/chat/does-kirchhoffs-law-hold-disagreeing-with-a-master/?action=dlattach;attach=618160;image)

Unfortunately that is not Kirhchhoff's law. That is Kirchhoff's law.

(https://www.eevblog.com/forum/chat/does-kirchhoffs-law-hold-disagreeing-with-a-master/?action=dlattach;attach=619306;image)

And it is not difficult to see why.

Let's get back to the infamous Lewin's circuit. Kirchhoff says that all voltages around a circuit add up to zero. So I'm going to do exactly what he says. I will "walk" around the circuit with my voltmeter. Since "bad probing" would give me the wrong results and I would probably be  laughed out of the room, I'm taking the proper precautions not to allow any stinking varying magnetic field to induce unwanted voltages on my probes. So, here we go.

(https://www.eevblog.com/forum/chat/does-kirchhoffs-law-hold-disagreeing-with-a-master/?action=dlattach;attach=619312;image)

So far so good. As an added precaution, I will measure the voltage across the wire, just to make sure it's what I expect.

(https://www.eevblog.com/forum/chat/does-kirchhoffs-law-hold-disagreeing-with-a-master/?action=dlattach;attach=619318;image)

And, bingo! Zero volts. No wonder. The wire is a dead short. Now it's time for the other resistor.

(https://www.eevblog.com/forum/chat/does-kirchhoffs-law-hold-disagreeing-with-a-master/?action=dlattach;attach=619324;image)

Notice that I'm maintaining the polarity of the meter coherent with the anticlockwise path that I chose. Now, let's check the voltage across the other piece of wire and add up the voltages.

(https://www.eevblog.com/forum/chat/does-kirchhoffs-law-hold-disagreeing-with-a-master/?action=dlattach;attach=619330;image)

Surprise! They do not add up to zero.

For Kirchhoff to hold there should be a V5 somewhere measuring exactly -1V. But I checked every corner of this circuit and didn't find any other voltage than the four I measured.

So, either Kirchhoff is a liar, or didn't see it coming. I prefer to believe in the second hypothesis.

But you are going to say, Aha! Gotcha! The loop is nothing more than the secondary of a transformer. But where exactly is that generator in the circuit? My measurements show that this generator is nowhere to be found.

So the logic conclusion is that voltages do not necessarily obey the observation made by Kirchhoff, petrified in his laws. There must be another phenomenon that, when present, brakes those laws.

Quote
Its the similar as the classical kinematics equation we all know from physics:
d=s*t

This equation is WRONG! It only works in a universe where the speed of light is infinite or in one where space and time are not related to each other in the form of a spacetime field. So is that for the birds too?

From the point of view of Einstein's relativity, yes, Newton is for the birds. You cling to Newton, you won't be able to predict what Einstein did. Newton reveals an even worse relationship to Einstein, than Kirchhoff to Maxwell. The speed of light can't be infinite in practice, as you said. While we can have a zero magnetic field.

Quote
But the thing is that at any reasonable speeds we might encounter on earth the error in the result due to ignoring special relativity is pretty much in the parts per million or even smaller. So we use it anyway because it gives results that are still within margin of error, yet its much easier and faster to work with. In fact most physics equations we see in highschool only work in this fictional universe with infinite, speed of light, no atmosphere or drag and spherical cows. Yet a lot of these cut down formulas are still close enough to the real deal to be perfectly usable. Circuit meshes are the same sort of thing, not quite real but real enough for what they are supposed to do.

Such approximations work fine because we live under a relatively low constant gravitational field. We would only notice something wrong at astronomical scale. In fact we did, already in the 19th century. And that's why we have relativity today.

However the electromagnetic force is 10³⁶ times stronger than gravity. Noticeable deviations from the approximations such as those that we do to deduce Kirchhoff can be noticeable at a scale of millimeters. So they must be taken care of with much more attention.

Quote
If you are going to use the classical simple form of KVL use it in circuit meshes where it indeed always works. If you want your circuit mesh to behave like the real circuit in the universe we live in then also use proper circuit modeling methods (where wires are modeled as having inductance). If you don't want to do that then don't just directly slap on KVL and expect it to work every time.

Instead you can use the version of KVL that is derived from Maxwell equations in the physical world, since that does work. Calculate it however you want, just don't carelessly mix formulas from our universe and formulas from circuit meshes. A lot of the times they work fine, but not always (As Dr. Lewin clearly demonstrates)

If you can't handle abstraction then just ignore circuit meshes and focus on pure Maxwells equations instead.

I can make sense of Dr. Lewins circuit both in the form of fields and in the form of a circuit mesh model. Both work just fine and give identical results. If you can't make sense of the circuit using a mesh model then try to learn how, otherwise don't complain about it being wrong just because you don't seam to understand it. I don't want to come across rude or anything, but any answer to why its wrong to mesh model this circuit is along the lines of "It can't be done because i said so" rather than getting an explanation why i am getting the right results out of my mesh model despite it being supposedly wrong for some mysterious reason.

You're absolutely right. I can't handle the "abstraction". I must admit. So I humbly ask you a favor. Please, show me how to solve the circuit below using exclusively Kirchhoff.

(https://www.eevblog.com/forum/chat/does-kirchhoffs-law-hold-disagreeing-with-a-master/?action=dlattach;attach=619336;image)

Thank you in advance.
Title: Re: Does Kirchhoff's Law Hold? Disagreeing with a Master
Post by: Berni on January 12, 2019, 03:37:01 pm

Unfortunately that is not Kirhchhoff's law. That is Kirchhoff's law.

(https://www.eevblog.com/forum/chat/does-kirchhoffs-law-hold-disagreeing-with-a-master/?action=dlattach;attach=619306;image)

And it is not difficult to see why.

Yes it looks like that when used in cirucit meshes. It has the extra voltage in it when you try to derive a "KVL like" equation from Maxwells equations. It has to be there otherwise there is a mistake in the process or Maxwells equations are wrong (And that's highly unlikely)


Let's get back to the infamous Lewin's circuit. Kirchhoff says that all voltages around a circuit add up to zero. So I'm going to do exactly what he says. I will "walk" around the circuit with my voltmeter. Since "bad probing" would give me the wrong results and I would probably be  laughed out of the room, I'm taking the proper precautions not to allow any stinking varying magnetic field to induce unwanted voltages on my probes. So, here we go.

(https://www.eevblog.com/forum/chat/does-kirchhoffs-law-hold-disagreeing-with-a-master/?action=dlattach;attach=619312;image)

So far so good. As an added precaution, I will measure the voltage across the wire, just to make sure it's what I expect.

(https://www.eevblog.com/forum/chat/does-kirchhoffs-law-hold-disagreeing-with-a-master/?action=dlattach;attach=619318;image)

And, bingo! Zero volts. No wonder. The wire is a dead short. Now it's time for the other resistor.

(https://www.eevblog.com/forum/chat/does-kirchhoffs-law-hold-disagreeing-with-a-master/?action=dlattach;attach=619324;image)

Notice that I'm maintaining the polarity of the meter coherent with the anticlockwise path that I chose. Now, let's check the voltage across the other piece of wire and add up the voltages.

(https://www.eevblog.com/forum/chat/does-kirchhoffs-law-hold-disagreeing-with-a-master/?action=dlattach;attach=619330;image)

Surprise! They do not add up to zero.

For Kirchhoff to hold there should be a V5 somewhere measuring exactly -1V. But I checked every corner of this circuit and didn't find any other voltage than the four I measured.

So, either Kirchhoff is a liar, or didn't see it coming. I prefer to believe in the second hypothesis.

But you are going to say, Aha! Gotcha! The loop is nothing more than the secondary of a transformer. But where exactly is that generator in the circuit? My measurements show that this generator is nowhere to be found.

So the logic conclusion is that voltages do not necessarily obey the observation made by Kirchhoff, petrified in his laws. There must be another phenomenon that, when present, brakes those laws.


Yes i completely agree you get those voltages, what else would you expect there?

The fact that you contain the field on the inside does not mean it has no effect on the probes going around it. It just makes it a special case where induced voltage in the probes is 0V for the formal definition. If the probes pass trough the middle then you get voltage and voltmeters show wrong results hence why probe path matters. Since this behavior does not mirror circuit mesh behavior directly (It does if you model the indutance tho) means KVL does not work. If you apply the Maxwell derived version of KVL it does work cause it has that Vi component in it.

If you do the same thing again with the conservative component of voltage and again place the voltmeter in such a case that it shows 0V across the probes you would get identical voltages across resistors and a voltage gradient across the wires and the whole loop adds up to zero as conservative fields always do. So if you plug those voltages into KVL it also adds up to zero.

Its just a matter of perspective. There is nothing special about the two cases. Both work just fine. And its not my problem if you only agree with one case.




From the point of view of Einstein's relativity, yes, Newton is for the birds. You cling to Newton, you won't be able to predict what Einstein did. Newton reveals an even worse relationship to Einstein, than Kirchhoff to Maxwell. The speed of light can't be infinite in practice, as you said. While we can have a zero magnetic field.

Quote
But the thing is that at any reasonable speeds we might encounter on earth the error in the result due to ignoring special relativity is pretty much in the parts per million or even smaller. So we use it anyway because it gives results that are still within margin of error, yet its much easier and faster to work with. In fact most physics equations we see in highschool only work in this fictional universe with infinite, speed of light, no atmosphere or drag and spherical cows. Yet a lot of these cut down formulas are still close enough to the real deal to be perfectly usable. Circuit meshes are the same sort of thing, not quite real but real enough for what they are supposed to do.

Such approximations work fine because we live under a relatively low constant gravitational field. We would only notice something wrong at astronomical scale. In fact we did, already in the 19th century. And that's why we have relativity today.

However the electromagnetic force is 10³⁶ times stronger than gravity. Noticeable deviations from the approximations such as those that we do to deduce Kirchhoff can be noticeable at a scale of millimeters. So they must be taken care of with much more attention.


Well you can get problems due to this approximation on earth just as well. Particles on earth that approach the speed of light start preciving time slower, tho this is not directly observable outside of a lab experiment. Also anything that moves a very small fraction of light speed but has a very accurate sense of time can observe the effects. Such as putting atomic clocks on a plane and flying it around, or to get an even bigger effect putting them into orbit around the earth such as GPS does. The part about gravity is a different effect. Infact the two effects are fighting each other in the case of GPS satellites. The fact that satellites are moving quickly is making them run slower while the fact that they experience less gravity up there is making them run faster, but the effects are not the same in magnitude so they don't cancel out and we do need to tweak the clocks on the satellites ever so slightly to fix it.

But those are rare cases where it matters, for everything else you can use the quick and simple formulas that work on spherical cows in a vacuum. Gets you the result quicker while being just as accurate once margins of error are considered. Its a matter of understanding the limitations of your abstraction.

If you want to include the effects of time dilation due to special relativity when calculating how long its going to take to to walk across town on foot be my guest. But don't expect everyone else to do it too.

Keep in mind Maxwells equations are an just an abstraction of Quantum electrodynamics. But not that it matters, they work for what they are meant to do. Just like circuit theory being a abstraction of Maxwells equations.


You're absolutely right. I can't handle the "abstraction". I must admit. So I humbly ask you a favor. Please, show me how to solve the circuit below using exclusively Kirchhoff.

(https://www.eevblog.com/forum/chat/does-kirchhoffs-law-hold-disagreeing-with-a-master/?action=dlattach;attach=619336;image)

Thank you in advance.

Can't do it using just Kirchhoff. But even if there was no magnetic field you couldn't solve it using only Kirchhoffs circuit laws. That's because the circuit contains resistors and KVL has no way to deal with that, it can only deal with voltages. So it requires the assistance of circuit mesh analysis tools to join it to a resistor model by using Ohms law to calculate the voltage and then present that to KVL.

In the same way Faradays law alone can't be used to solve this circuit. It will tell you the magnitude of the non conservative E field around this circuit and that is very helpful since we have a magnetic field but that's where its assistance ends. You have the total voltage but you have no idea where it is.

Id solve your particular circuit with numbers but the size of the loop area is not provided so a numeric result is not possible.

But a typical path to solve this circuit would be as follows:
1) Use Faradays law to get the induced voltage
2) Use Thevenins theorem to reduce the circuit to a single voltage source and resistor (Involves the use of series lumping to get there)
3) Use Ohms law to find the current flowing in this reduced cirucit
4) Use Kirchhoff current law to deduct this same current must flow trough all components (If there was a junction node there would be more work here) to find the current flowing on each resistor
5) Use Ohms law to turn the current on both resistors to the voltages on resistors.

There now all voltages and currents in the circuit are known. Notice that this involved 4 different formulas/procedures to get there. Any single one of them alone could not fully solve the circuit, thats because they don't spit out the voltages and currents for every point, or that they need extra input information that gets provided by a different formula.

Title: Re: Does Kirchhoff's Law Hold? Disagreeing with a Master
Post by: bsfeechannel on January 13, 2019, 03:17:46 am
Its just a matter of perspective.

Yes. The perspective of those who study Maxwell for real and understand the physical phenomenon with which they're dealing, and those who don't.

Quote
Keep in mind Maxwells equations are an just an abstraction of Quantum electrodynamics.

Poor Feynman. Won the Nobel Prize for nothing.

Quote
Can't do it using just Kirchhoff.

That's a pity. I was already picturing you being invited by the King of Sweden for a banquet.

Quote
But even if there was no magnetic field you couldn't solve it using only Kirchhoffs circuit laws.

You said you could calculate Lewin's circuit using Kirchhoff with the exclusion of Maxwell.

Quote
Id solve your particular circuit with numbers but the size of the loop area is not provided so a numeric result is not possible.

Didn't you say that Kirchhoff is an abstraction of Maxwell? Doesn't Kirchhoff hide the ugly underlying details of Maxwell? Why do you need them now? Isn't it because Kirchhoff is in fact a special case of Maxwell?

This is just yet another contradiction that shows that such claims are nothing but false.

Title: Re: Does Kirchhoff's Law Hold? Disagreeing with a Master
Post by: Berni on January 13, 2019, 09:30:11 am
Yes. The perspective of those who study Maxwell for real and understand the physical phenomenon with which they're dealing, and those who don't.

I understand both of the of the perspectives just fine, so there is a problem behind knowing too much? That makes no sense.

Poor Feynman. Won the Nobel Prize for nothing.

Feynman won the nobel prize exactly on the topic of quantum electrodynamics. His diagrams are also widely used to describe interactions in this theory. I can't know how he feels about his nobel prize but this stuff is still at the cutting edge of physics research.

Just because quantum electrodynamics are even more fundamental than Maxwells equations doesn't mean that they are suddenly useless. If you are looking for the underlying workings of the universe they are indeed not the place to look, but if you just want to deal with electromagnetic fields then they are great (Since that's exactly what those equations are supposed to do). If you just want a voltage on a resistor in a typical cirucit then all you need is circuit analysis theory. Use the appropriate theory for the task at hand and everything is fine.

You said you could calculate Lewin's circuit using Kirchhoff with the exclusion of Maxwell.

I said that you can properly model Dr. Lewins circuit as a circuit mesh to have KVL work just fine on it. Notice that in my procedure for solving the circuit i never used KVL.

Can you show me where i said that we don't need Maxwells equations or Faraday law? (Cause i don't remember it)

Didn't you say that Kirchhoff is an abstraction of Maxwell? Doesn't Kirchhoff hide the ugly underlying details of Maxwell? Why do you need them now? Isn't it because Kirchhoff is in fact a special case of Maxwell?

This is just yet another contradiction that shows that such claims are nothing but false.

Call it an abstraction or special case or whatever you want. Its all because of the way circuit mesh analysis works and KVL fits into that. Maxwell deals with it using fields, cirucit analysis deals with it using inductance, just a different way of going about it that gives the same result.

Okay can you then calculate the voltage for me without making up a random value for the area?


Title: Re: Does Kirchhoff's Law Hold? Disagreeing with a Master
Post by: bsfeechannel on January 14, 2019, 07:56:06 pm
Call it an abstraction or special case or whatever you want.

You urgently need to know the huge difference between the two.
Title: Re: Does Kirchhoff's Law Hold? Disagreeing with a Master
Post by: Sredni on January 16, 2019, 11:02:03 am

Whatever you say man.  I just want to point out what "Engineering Electromagnetics" by Hyat and Buck, Seventh Edition, says in page 94:

"Equation (21) (Curl integral of E.dl=0) is therefore just a more general form of Kirchhoff's circuital law for voltages, more general in that we can apply it to any region where an electric field exists and we are not restricted to a conventional circuit composed of wires, resistances, and batteries.  Equation (21) must be amended before we can apply it to time-varying fields.  We shall take care of this in Chapter 10, and in Chapter 13 we will them be able to establish the general form of Kirchhoff's voltage law for circuits in which currents and voltages vary with time."

Yep, it's talking about 'new' or 'extended' or 'amended' KVL that is used with lumped circuits.
But it's really Faraday and Lenz carrying around Kirchhoff's corpse.

https://i.ibb.co/bWNhLK8/screenshot-9.png

But this breaks as well when you try to apply a lumped circuit rule to a non-lumped circuit such as the Romer-Lewin ring.
Title: Re: Does Kirchhoff's Law Hold? Disagreeing with a Master
Post by: Sredni on January 16, 2019, 11:19:32 am
What probe wires???
There are no probes wires in the computation of the path integrals I've shown above.
The two different values we get, for the two possibile path along the circuit, are the result of induction. But that's how the system is. If you remove the induced part of the field, you are analyzing a different (unrealistic) system.

It's as if you subtracted the field generated by the point charge near a piece of copper to come to the conclusion that there is a nonzero field inside the metal (and then came up with tiny generators inside the metal) that produce the observed surface charge.

The ones that connect his oscilloscope to points A and B, since the BNC connector on a scope does not conveniently have the exact contact spacing to touch the two points of interest.

Im not saying that the two voltages result is due to the probe wires. I am trying to say that the path that the probe wires take is important.


You saw the field, you saw the computation of the integral along the two branches. It has nothing to do with the probes. It's the circuit itself that is path-dependent. When you are outside of the loop the voltmeters show this dependency as well, but they are not necessary to know this is happening to the circuit and has nothing to do with the measurement.
And in fact, as long as you do not cross the flux region (meaning also you cannot go 'on the other side' with your voltmeter) the probes path is not affecting the 'exterior' measurement.

Quote
If you take probing into account you can run the wires in any path you want and get the same result. Still two voltages across A and B.

This is inconsistent. Either you get the same result, or you get two voltages.

Can you please solve the circuit using your method?
You had to stop because the area was not given, so make it up. 10 cm^2 or any values that gives you a nice round value for the emf, such as 1V, 5V, 10V, you make it up BUT please make sure not to lose sight of the phase relationship between the flux, the emf and the 'effective' voltage. Because I believe, if you do, there are bad news in sight for Mabilde and co.
Title: Re: Does Kirchhoff's Law Hold? Disagreeing with a Master
Post by: Berni on January 16, 2019, 06:41:54 pm
Alright then. Lets assume the area is 10cm2. I just copy pasted my procedure and filled in numbers
 
1) Use Faradays law to get the induced voltage
Integ(E)=-Integ(∂B/∂t*dA)
Integ(E)=-Integ(∂sin(100*t)/∂t*0.001)
Integ(E)= -100 * cos(100*t) * 0.001
Vi = -0.1 * cos(100*t)
So this gives us 0.1 Vrms that is leading by 90 degrees

2) Use Thevenins theorem to reduce the circuit to a single voltage source and resistor (Involves the use of series lumping to get there)
U=Vi=0.1 * -cos(100*t)
R=R1+R2=100+900= 1000 Ohm

3) Use Ohms law to find the current flowing in this reduced cirucit
I=U/R=0.1 * -cos(100*t) / 1000
I= 100 * -cos(100*t) µA

4) Use Kirchhoff current law to deduct this same current must flow trough all components (If there was a junction node there would be more work here) to find the current flowing on each resistor
IR1 = IR2 = I =  100 * -cos(100*t) µA

5) Use Ohms law to turn the current on both resistors to the voltages on resistors.
U=I*R
UR1=IR1*R1 = 100E-6 * -cos(100*t) * 100 =  10 * -cos(100*t) mV
UR1=IR1*R1 = 100E-6 * -cos(100*t) * 900 =  90 * -cos(100*t) mV

There now voltages and current across all components are visible. The conditionality for all values is in the clockwise direction around the circuit diagram. This is pretty much what Dr. Lewin did on his whiteboard except it includes phase. What is so special about this?
Title: Re: Does Kirchhoff's Law Hold? Disagreeing with a Master
Post by: Sredni on January 16, 2019, 07:32:53 pm
I am sorry I wasn't clear, what I meant was to solve it to give the 'true' or 'effective' or 'coulombian' or 'unique' voltage. The one that in the case of an emf of 1V gives 0.4V between the midpoints. If you use ohm's law to the arc composed of piece of wire + resistor + piece of wire for the two half-circles, you end up with two different values for the voltage between the same points (something that Kirchhoffian would not tolerate).

Can you give the voltage at points between resistors to show, like Mabilde did, that you get a gradually changing voltage from one resistor to the other?

I am showing you my cards, here: my point is that, if that's the voltage associated with the conservative part of the total electric field, it will give contributes that are actually opposing the emf.
Title: Re: Does Kirchhoff's Law Hold? Disagreeing with a Master
Post by: Berni on January 16, 2019, 09:25:02 pm
The effective voltage across the resistors is the same so above calculations still hold. The circuit in question had no midpoint node marked in it so everything between the resistors was assumed to be an ideal wire.

If you want to add two points in the middle this makes it the same as the inner part of Dr. Lewins experiment and i already put that together 2 months ago:
https://www.eevblog.com/forum/chat/does-kirchhoffs-law-hold-disagreeing-with-a-master/msg1945312/#msg1945312 (https://www.eevblog.com/forum/chat/does-kirchhoffs-law-hold-disagreeing-with-a-master/msg1945312/#msg1945312)
Remember to interact a circuit mesh with magnetic fields the circuit has to have inductors otherwise all loop areas are 0 cm2

Okay it was done with SPICE, but its no problem to do it manually too.
We need to find the voltage on each lumped inductor. All four inductors form the entire loop so they are considered fractional turns around the given coupled field and the coupling factor is 1 so all inductance is in the form of coupled inductance. Considering that points A and B are in the middle gives each inductor an equal 1/4 share of the total turn ratio. This means that they basically catch 1/4 of the total field trough it. So we simply solve Faradays Law with a field 1/4 the strength.

But we already calculated it for the whole loop before so we can simply reuse the result and divide it by 4 (Since that's what we get if we put 1/4 the field in) and since the wires all also go in  the same direction around this gives them the same sign too however the effective voltage is the opposite sign to the EMF voltage due to charge separation (And in general voltage sources in circuits act like this because they essentially have a negative 'voltage drop'):
UL2=UL3=UL4=UL5= -(-0.1 * cos(100*t)) / 4 = 0.025 * cos(100*t) V

The circuit is again fully solved because we know the current and voltage on every component.

So now then lets test the result a bit by calculating the voltage across A and B in two ways.
1) Going trough L5 R1 L2:
UAB = UL5+UR1+UL2 = 0.025 * cos(100*t) +(-0.010 * cos(100*t)) + 0.025 * cos(100*t) = 0.040 * cos(100*t) V
2) Going trough L4 R2 L3:
UAB = -UL4-UR2-UL3 = -0.025 * cos(100*t) -(-0.090 * cos(100*t)) + 0.025 * cos(100*t) = 0.040 * cos(100*t) V

There you go, same voltage no matter what path you take

To top it off lets also show that KVL works, so all voltages should add up to zero:
Uloop= UR1+UL2+UL3+UR2+UL4+UL5 =
-0.010 * cos(100*t) + 0.025 * cos(100*t) + 0.025 * cos(100*t) + (-0.090 * cos(100*t) + 0.025 * cos(100*t) + 0.025 * cos(100*t) = 0 V
So KVL works as we got zero

You can also add the voltages of the other 4 inductors in the outer circuit and get the voltages at the voltmeter terminals. Or the points A and B can be moved to any other location by recalculating the inductor turns ratio to match.

Oh and i should  also note that this calculation is valid for the case where the applied field "1T * sin(100*t)" is constantly maintained by whatever is generating it (Such as a superconducting coil fed by a ideal voltage source), if not then the current flowing in the circuit would reduce the strength of the field inside the loop area and this would need to be accounted for with fairly complex math. Tho in this case with a field strength of 1T and 100uA of current the difference in the result would be very small.
Title: Re: Does Kirchhoff's Law Hold? Disagreeing with a Master
Post by: rfeecs on January 16, 2019, 10:10:37 pm
Alright then. Lets assume the area is 10cm2. I just copy pasted my procedure and filled in numbers
 
1) Use Faradays law to get the induced voltage
...
2) Use Thevenins theorem to reduce the circuit to a single voltage source and resistor (Involves the use of series lumping to get there)
...
3) Use Ohms law to find the current flowing in this reduced cirucit
...
4) Use Kirchhoff current law to deduct this same current must flow trough all components (If there was a junction node there would be more work here) to find the current flowing on each resistor
...
5) Use Ohms law to turn the current on both resistors to the voltages on resistors.
...
There now voltages and current across all components are visible. The conditionality for all values is in the clockwise direction around the circuit diagram. This is pretty much what Dr. Lewin did on his whiteboard except it includes phase. What is so special about this?

You could simplify to:
1)  Use Faraday's law to find the EMF in the loop.
2)  Use ohms law to find the total current in the loop (I = Total EMF / total resistance).
3)  Use ohms law to calculate the voltage drop across each resistor (V = IR).

Notice that you MUST use Faraday's law to solve this, and you do not use Kirchhoff's loop rule at all to solve it.

Doesn't that say something?
Title: Re: Does Kirchhoff's Law Hold? Disagreeing with a Master
Post by: jesuscf on January 17, 2019, 02:54:07 pm

Whatever you say man.  I just want to point out what "Engineering Electromagnetics" by Hyat and Buck, Seventh Edition, says in page 94:

"Equation (21) (Curl integral of E.dl=0) is therefore just a more general form of Kirchhoff's circuital law for voltages, more general in that we can apply it to any region where an electric field exists and we are not restricted to a conventional circuit composed of wires, resistances, and batteries.  Equation (21) must be amended before we can apply it to time-varying fields.  We shall take care of this in Chapter 10, and in Chapter 13 we will them be able to establish the general form of Kirchhoff's voltage law for circuits in which currents and voltages vary with time."

Yep, it's talking about 'new' or 'extended' or 'amended' KVL that is used with lumped circuits.
But it's really Faraday and Lenz carrying around Kirchhoff's corpse.

https://i.ibb.co/bWNhLK8/screenshot-9.png

But this breaks as well when you try to apply a lumped circuit rule to a non-lumped circuit such as the Romer-Lewin ring.

Aren't we talking here about Kirchhoff's circuital law for voltages?  Are you aware of the generalization of Kirchhoff's circuital laws to systems other than linear electric systems?  I can recommend you the book "Physical Networks" by Richard Sanford which explains how to apply KVL and KCL to other "circuits" with potential/flow properties.  The book deals not only with electrical systems, but rotational, translational, and fluids-flow (as water in tanks) systems as well as combinations of all of them via "transformers".  Although not covered in the book, the same rules (as in KCL and KVL) apply to thermal and magnetic circuits as well.

The problem you are having with the so called Romer-Lewin ring is that you are decoupling a circuit that can not be decoupled.  In the so called Romer-Lewin ring the inductor generating the time varying field is part of the circuit and must be included in the solution via the standard equations of transformers.  Then KVL "magically" works as shown by Electroboom.  I don't know man, maybe Richard Feynman was right when he said in one of his lectures on physics (Lecture 25: Linear Systems And Review, at about minute 25:40):

"The difference between a physicist and an electrical engineer is not the difference in anything he knows, except one fact... not the mathematical knowledge or anything else except just one extra fact the physicist knows and that is: electrical systems are not the only linear systems in the world.  All you have the same equations the same problems exactly in electrical engineering and you have in all the rest of physics.  And all the difference between a physicist and electrical engineer is that the physicist knows the same equations apply to another circumstances and he gets in [obviously] and the electrical engineer looks puzzled, and that so is simply why they hired a physicist... I know that but that wasn't an electrical circuit..."

 
Title: Re: Does Kirchhoff's Law Hold? Disagreeing with a Master
Post by: Berni on January 17, 2019, 05:22:38 pm
You could simplify to:
1)  Use Faraday's law to find the EMF in the loop.
2)  Use ohms law to find the total current in the loop (I = Total EMF / total resistance).
3)  Use ohms law to calculate the voltage drop across each resistor (V = IR).

Notice that you MUST use Faraday's law to solve this, and you do not use Kirchhoff's loop rule at all to solve it.

Doesn't that say something?

Yes in this case you could simplify out the two steps because the circuit is so simple you can see what the total loop resistance is and there is only one path to consider as the circuit does not branch anywhere. But when you describe a procedure you can't just pluck a number from your head because you see it, you have to explain how you got the number. I mention in the procedure that there is not much to do in this step, but in a more complex circuit there could be a significantly more math involved to evaluate all the paths.

And no you can't use only Kirchhoffs circuit laws to solve this, just like you CAN'T use ONLY Faradays law to solve it, you do need it for the first step but you will not be able to find the voltage on the resistors without involving Ohms law. That way my point, circuit analysis makes use of a whole collection of formulas to solve circuits and each formula has its use case.

Its all just a matter of using the right tool for the job. Because the circuit has a magnetic field defined in it you need Faradays law because its a tool for turning changing fields into voltage and once we have voltage we can forget about the field and proceed with circuit analysis as usual.

Aren't we talking here about Kirchhoff's circuital law for voltages?  Are you aware of the generalization of Kirchhoff's circuital laws to systems other than linear electric systems?  I can recommend you the book "Physical Networks" by Richard Sanford which explains how to apply KVL and KCL to other "circuits" with potential/flow properties.  The book deals not only with electrical systems, but rotational, translational, and fluids-flow (as water in tanks) systems as well as combinations of all of them via "transformers".  Although not covered in the book, the same rules (as in KCL and KVL) apply to thermal and magnetic circuits as well.

The problem you are having with the so called Romer-Lewin ring is that you are decoupling a circuit that can not be decoupled.  In the so called Romer-Lewin ring the inductor generating the time varying field is part of the circuit and must be included in the solution via the standard equations of transformers.  Then KVL "magically" works as shown by Electroboom.  I don't know man, maybe Richard Feynman was right when he said in one of his lectures on physics (Lecture 25: Linear Systems And Review, at about minute 25:40):

"The difference between a physicist and an electrical engineer is not the difference in anything he knows, except one fact... not the mathematical knowledge or anything else except just one extra fact the physicist knows and that is: electrical systems are not the only linear systems in the world.  All you have the same equations the same problems exactly in electrical engineering and you have in all the rest of physics.  And all the difference between a physicist and electrical engineer is that the physicist knows the same equations apply to another circumstances and he gets in [obviously] and the electrical engineer looks puzzled, and that so is simply why they hired a physicist... I know that but that wasn't an electrical circuit..."

Yes i did bring that up about 15 pages ago:
https://www.eevblog.com/forum/chat/does-kirchhoffs-law-hold-disagreeing-with-a-master/msg2004740/#msg2004740 (https://www.eevblog.com/forum/chat/does-kirchhoffs-law-hold-disagreeing-with-a-master/msg2004740/#msg2004740)

The form of KVL we popularly know is meant for use in circuit meshes and it always works there. If a cirucit mesh doesn't behave the same as your circuit that's because you made a mistake in modeling the real circuit as a mesh. But because all other physical systems also show similar behavior as electricity means you can use cirucits to model those too and apply KVL to heat flow, water flow, mechanical motion etc.

Tho to be honest i never seen physicists use circuit schematics to describe anything other than electrical things. It seams to me its more of an electrical engineering thing where engineers have no clue about some random physical system, so they do "when you have hammer every problem looks like a nail" and mold the problem to look like a circuit since that's the one thing they do know how to deal with easily.

At least i never saw this "circuit meshes are not just for electrons" concept even mentioned any of my physics classes, while i have used the trick extensively in a lot of engineering classes to make sense of magnetic circuits, water flow, heat flow etc.. and to actually calculate stuff with it.
Title: Re: Does Kirchhoff's Law Hold? Disagreeing with a Master
Post by: Sredni on January 17, 2019, 06:51:55 pm
Aren't we talking here about Kirchhoff's circuital law for voltages?  Are you aware of the generalization of Kirchhoff's circuital laws to systems other than linear electric systems?

Oh, Jesus...
How come I have this hunch you did not read all the previous posts?
Are you mistaking me for a kirchhoffian?

Quote
I can recommend you the book "Physical Networks" by Richard Sanford which explains how to apply KVL and KCL to other "circuits" with potential/flow properties.  The book deals not only with electrical systems, but rotational, translational, and fluids-flow (as water in tanks) systems as well as combinations of all of them via "transformers".  Although not covered in the book, the same rules (as in KCL and KVL) apply to thermal and magnetic circuits as well.

Good, we discovered analogies. They can be very useful. You might want to go back a couple of tens of pages and read my gravitational analogy with nonconservative drag.
What we are discussing here is the fact that when the electric field is not conservative, you can no longer have a potential function, so that voltage can no longer associated with the difference in the values of said function in two points, but depends on the path as well.
Meaning... when voltage is path dependent you can have two (actually infinitely many but... more than one) different value of voltage between two points. This is no biggie to a physicist and to many engineers. It's just simple math. And if you compute the field inside the ring you can see at a glance that it has nothing to do with probing. Now, I hope you agree that Kirchhoff needs single-valued voltages to live. So Kirchhoff dies inside the Romer-Lewin ring.
And yet there are people (which I call kirchhoffians) insisting that you can still use KVL inside that ring.
(If you want to see why you can use 'extended KVL' with lumped circuit, go read Ramo Whinnery Vanduzer, I am even tired to point out the chapter).

Quote
The problem you are having with the so called Romer-Lewin ring is that you are decoupling a circuit that can not be decoupled.  In the so called Romer-Lewin ring the inductor generating the time varying field is part of the circuit

So, you know. Then why...

Quote
and must be included in the solution via the standard equations of transformers.  Then KVL "magically" works as shown by Electroboom.

The problem is that if you do that you are lumping the unlumpable.
The circuit with one transformer secondary lumping all the emf in one point is a different circuit. So is the circuit with the emf lumped in four points, or eight, or a thousand.

And if you only consider the conservative part of the total electric field to get a field where you can apply KVL (the McDonald maneuver) you are considering only part of your system, specifically the part with the field that actually opposes the electromotive field generated by the primary coil (or moving magnet). Scalar potential can be very useful, but we must be aware of what it is: only half of the story.

To sum it up, if you believe that KVL can be applied to a non-lumped element circuit, you are mistaken. Neither Feynman nor Hayt ever said that. And while I've never had the pleasure to read Sanford, I am pretty confident he did not either. My bet is that when the system is rotational he uses some analog of Faraday's law and the analog of KVL dies an analog horrible death in a different branch of physics.

Title: Re: Does Kirchhoff's Law Hold? Disagreeing with a Master
Post by: bsfeechannel on January 17, 2019, 10:41:43 pm
The problem you are having with the so called Romer-Lewin ring is that you are decoupling a circuit that can not be decoupled.  In the so called Romer-Lewin ring the inductor generating the time varying field is part of the circuit and must be included in the solution via the standard equations of transformers. 

What I like about those people who do not study Maxwell and think they understand something about electromagnetism is to get acquainted with the pseudo scientific vocabulary:

"You are decoupling a circuit that cannot be decoupled." HAHAHAHAHA!

Quote
Then KVL "magically" works as shown by Electroboom.

The only magical power we have seen Mehdi show, up to now, is the power of attracting those who are lazy enough to understand electromagnetism and need a pseudo-theory to justify their ignorance.

Quote
I don't know man, maybe Richard Feynman was right when he said in one of his lectures on physics (Lecture 25: Linear Systems And Review, at about minute 25:40):

Everybody likes to quote Feynman, but no one has the guts to study the subject-matter of his lectures.
Title: Re: Does Kirchhoff's Law Hold? Disagreeing with a Master
Post by: jesuscf on January 18, 2019, 04:02:10 am
"You are decoupling a circuit that cannot be decoupled." HAHAHAHAHA!

I wonder... are you an electrical engineer?  If your answer is yes, then you can quickly tell me why an electrical engineer will not use the method Dr. Lewin used to solve his problem #24.

About a year ago, after I watched his series of videos about problem #24, I reached the conclusion that Dr. Lewin's teachings regarding the solution of electric circuits must be carefully considered before accepting them.

Quote
The only magical power we have seen Mehdi show, up to now, is the power of attracting those who are lazy enough to understand electromagnetism and need a pseudo-theory to justify their ignorance.

Ad hominem... that will prove you right.  KVL is a pseudo-theory?  Do you know KVL can be derived from Maxwell Equations?

Quote
Everybody likes to quote Feynman, but no one has the guts to study the subject-matter of his lectures.

So electrical engineers don't study linear circuits now!  Wow, that must be a new curriculum!
Title: Re: Does Kirchhoff's Law Hold? Disagreeing with a Master
Post by: jesuscf on January 18, 2019, 04:18:33 am

The problem is that if you do that you are lumping the unlumpable.


By applying the same logic an inductor is unlumpable...  therefore we can not use KVL in circuit that includes an inductor!?
Title: Re: Does Kirchhoff's Law Hold? Disagreeing with a Master
Post by: jesuscf on January 18, 2019, 04:33:53 am
Tho to be honest i never seen physicists use circuit schematics to describe anything other than electrical things. It seams to me its more of an electrical engineering thing where engineers have no clue about some random physical system, so they do "when you have hammer every problem looks like a nail" and mold the problem to look like a circuit since that's the one thing they do know how to deal with easily.

At least i never saw this "circuit meshes are not just for electrons" concept even mentioned any of my physics classes, while i have used the trick extensively in a lot of engineering classes to make sense of magnetic circuits, water flow, heat flow etc.. and to actually calculate stuff with it.

I believe it is even worst than that.  In my experience any linear electric circuit with more than 3 undefined nodes is enough to mess up most physicists.  And if you really want to have fun watching them trying to solve a circuit, add a non-linear element, say for example, a diode.
Title: Re: Does Kirchhoff's Law Hold? Disagreeing with a Master
Post by: bsfeechannel on January 18, 2019, 06:37:33 am
Ad hominem... that will prove you right.

You're right. I should have proved that his claims are bullshit before I accused him of brainwashing millions with the promise that they don't need to study Maxwell provided they sustain a false belief that Kirchhoff always holds.

Sorry about that.

Quote
KVL is a pseudo-theory?

Well, there is this rumor that for circuits under varying magnetic fields KVL sucks.

Quote
Do you know KVL can be derived from Maxwell Equations?

Some people say that KVL has nothing to do with Maxwell, but I never gave much credit to that.

Quote
So electrical engineers don't study linear circuits now!  Wow, that must be a new curriculum!

I don't know about electrical engineers, but those who don't study Maxwell don't understand electromagnetism.

Quote
By applying the same logic an inductor is unlumpable...  therefore we can not use KVL in circuit that includes an inductor!?

And you're the proof of that. Since you haven't read the Feynman's lectures recommended by Mehdi himself, you simply don't understand why an inductor is a lumped component and why Lewin's circuit is not solvable by KVL.
Title: Re: Does Kirchhoff's Law Hold? Disagreeing with a Master
Post by: jesuscf on January 18, 2019, 03:05:44 pm
I don't know about electrical engineers, but those who don't study Maxwell don't understand electromagnetism.

So, you are not an electrical engineer.  Are you familiar with the Dunning–Kruger effect?

Quote
And you're the proof of that. Since you haven't read the Feynman's lectures recommended by Mehdi himself, you simply don't understand why an inductor is a lumped component and why Lewin's circuit is not solvable by KVL.

Here, once again in case you missed  from "Engineering Electromagnetics" by Hyat and Buck, Seventh Edition:

"Equation (21) (Curl integral of E.dl=0) is therefore just a more general form of Kirchhoff's circuital law for voltages, more general in that we can apply it to any region where an electric field exists and we are not restricted to a conventional circuit composed of wires, resistances, and batteries.  Equation (21) must be amended before we can apply it to time-varying fields.  We shall take care of this in Chapter 10 [Maxwell's Equations], and in Chapter 13 [Plane Wave Reflection and Dispersion] we will them be able to establish the general form of Kirchhoff's voltage law for circuits in which currents and voltages vary with time."

That paragraph says it all, better than anybody yet... So, are you aware of the common practice of lumping about every kind of coupled electric circuit?  Probably not, because you are not an electrical engineer, but Mehdi is an electrical engineer and that is what he did to demonstrate that Lewin's conclusion is incorrect.  Maybe you should really understand what the Dunning–Kruger effect is.

Finally, you don't read Feynman lectures on Physics, you listen to them.
Title: Re: Does Kirchhoff's Law Hold? Disagreeing with a Master
Post by: Berni on January 18, 2019, 04:38:02 pm
Can you explain why a the section of wire in Dr. Lewins experiment can't be inductor lump modeled as a 1/4 fractional turn around a transformer? In what way does it act differently than a transformer?
Title: Re: Does Kirchhoff's Law Hold? Disagreeing with a Master
Post by: bsfeechannel on January 18, 2019, 08:52:57 pm
Read carefully Feyman's lecture Vol. 2,  Chapter 22 (the same you've already linked before), sections 22-1, 22-2 and 22-3 at least. If you do not manage to see the answer there, we'll be happy to help.
Title: Re: Does Kirchhoff's Law Hold? Disagreeing with a Master
Post by: Sredni on January 18, 2019, 11:06:41 pm
The problem is that if you do that you are lumping the unlumpable.
By applying the same logic an inductor is unlumpable...  therefore we can not use KVL in circuit that includes an inductor!?

See? I was right! You did not read previous posts. But it also appear you did not read the books you mention.
And when you read them, it seems to me, you read something that is not there. For example, in the passage you twice quoted from Hayt:

"Equation (21) (Curl integral of E.dl=0) is therefore just a more general form of Kirchhoff's circuital law for voltages, more general in that we can apply it to any region where an electric field exists and we are not restricted to a conventional circuit composed of wires, resistances, and batteries.  Equation (21) must be amended before we can apply it to time-varying fields.  We shall take care of this in Chapter 10 [Maxwell's Equations], and in Chapter 13 [Plane Wave Reflection and Dispersion] we will them be able to establish the general form of Kirchhoff's voltage law for circuits in which currents and voltages vary with time."

It appears you believe that "amended before we can apply to time-varying fields" means "it will work even when the circuit path encloses a time-varying magnetic field". Well, it does not. The amended KVL works when the time-varying magnetic field is neatly tucked inside a lumped component and can not be tampered with, either by crossing it, or by running "net" circles around it (I lost count of how many times I've explained this in more detail in my previous post, so if you need clarification start reading back posts).
This impossibility to tamper with the magnetic field region can be summarized with seeing the offending component as zero-dimensional, or point-like. Lumped is a word that comes to mind.

When you consider the inductor as a lumped component, you use Faraday to deduce the voltage at its terminals, and then pretend that voltage is like a 'potential difference' in the circuit it is part of. Now, the circuit should not enclose any varying magnetic field inside its contour, if I want 'amended KVL' to work there.

(https://i.ibb.co/hVCCqHC/screenshot-5.png)

But even though 'amended KVL' works in the circuit, KVL still does not work inside the inductor's path.

In the same way it does not work inside Romer-Lewin's ring.


EDIT: added "even though"
Title: Re: Does Kirchhoff's Law Hold? Disagreeing with a Master
Post by: Berni on January 18, 2019, 11:49:27 pm
Read carefully Feyman's lecture Vol. 2,  Chapter 22 (the same you've already linked before), sections 22-1, 22-2 and 22-3 at least. If you do not manage to see the answer there, we'll be happy to help.

But did you also read the beginning of section 22-8 too?
Title: Re: Does Kirchhoff's Law Hold? Disagreeing with a Master
Post by: bsfeechannel on January 19, 2019, 12:50:05 am
Read carefully Feyman's lecture Vol. 2,  Chapter 22 (the same you've already linked before), sections 22-1, 22-2 and 22-3 at least. If you do not manage to see the answer there, we'll be happy to help.

But did you also read the beginning of section 22-8 too?

Cool. Since I said at least 22-1, 22-2, 22-3, I'm pleased that you read the whole chapter and I hope that you've finally found why a section of wire in Dr. Lewins experiment can't be inductor lump modeled as a 1/4 fractional turn around a transformer and in what way it acts differently than a transformer.
Title: Re: Does Kirchhoff's Law Hold? Disagreeing with a Master
Post by: Berni on January 19, 2019, 09:59:07 am
Cool. Since I said at least 22-1, 22-2, 22-3, I'm pleased that you read the whole chapter and I hope that you've finally found why a section of wire in Dr. Lewins experiment can't be inductor lump modeled as a 1/4 fractional turn around a transformer and in what way it acts differently than a transformer.

I have quickly went trough it last year already because it is a very good explanation.

But what point in the lecture exactly does it say that this can't be done? Yes it does explain that when considering a lumped inductor all magnetic fields have to be contained inside it. This is another way of saying that the inductors field should not affect anything else around it. Note that the inductor still has wires coming out that are in two different physical locations in order to give you two terminals, this means it can't be a fully closed loop inside the shielded "lumping" box. We still need extra wire outside the box to close it and connect it to for example a voltmeter. The diference between the wires inside the box and outside the box is just that the ones outside have no magnetic field around then and so no EMF. This is the exact same thing as getting rid of the shielding box but placing the wires to the voltmeter in such a path that they generate no EMF. So any piece of wire taking any path inside the shielded box can be considered an inductor (Doesn't have to be coiled up around a former or a core). And if we are careful not to interact the rest of the circuit then the shielding box can be removed and it behaves the same. So putting all of this together any length of superconducting wire can be considered a lumped inductor (tho care must be taken if its not shielded). Are any of my claims here false?

But Dr. Lewins experimental circuit is not laid out in such a way that other parts of the circuit would avoid the field. All sections of wire are enveloping the magnetic field and are so affected by it in the form of EMF. But if you look at a transformer it also has this feature common. There are multiple sections of wire enveloping a common magnetic field in the core. It would be really annoying if we couldn't apply circuit analysis to any circuit containing a transformer, so the inductor model was 'upgraded' to allow it to be friends with other inductors in the same field, this is explained in the section i mentioned:
http://www.feynmanlectures.caltech.edu/II_22.html#Ch22-S8 (http://www.feynmanlectures.caltech.edu/II_22.html#Ch22-S8)

This concept of mutual inductance allows transformers to be modeled with multiple inductors almost as easily as a single inductor. Notice that the mutual inductance value is separate from self inductance, this allows the proportion of the two to be adjusted to obtain any intensity of coupling you want. This is effectively saying how much of the flux the two coupled inductors are sharing (Its also what determines leakage inductance in transformers). Fractional turns in transformers are also possible because a length of wire might not necessarily enclose 100% of the flux in the core.

Putting it all together now. So since any length of wire can be considered an inductor and because these inductors around a common 'core'  can be considered a transformer then this circuit could be considered a transformer with 4 secondary coils connected into a configuration with 2 resistors. If not, can you explain why?
Title: Re: Does Kirchhoff's Law Hold? Disagreeing with a Master
Post by: Sredni on January 19, 2019, 03:41:17 pm
Cool. Since I said at least 22-1, 22-2, 22-3, I'm pleased that you read the whole chapter and I hope that you've finally found why a section of wire in Dr. Lewins experiment can't be inductor lump modeled as a 1/4 fractional turn around a transformer and in what way it acts differently than a transformer.

I have quickly went trough it last year already because it is a very good explanation.

But what point in the lecture exactly does it say that this can't be done? Yes it does explain that when considering a lumped inductor all magnetic fields have to be contained inside it.

Right there, buddy, right there.
That's pretty much it.

EDIT: This post has been shortened and cleansed to avoid upsetting other children.
Whatever was written here can be found in any good EM book.
Title: Re: Does Kirchhoff's Law Hold? Disagreeing with a Master
Post by: jesuscf on January 19, 2019, 03:54:56 pm
Cool. Since I said at least 22-1, 22-2, 22-3, I'm pleased that you read the whole chapter and I hope that you've finally found why a section of wire in Dr. Lewins experiment can't be inductor lump modeled as a 1/4 fractional turn around a transformer and in what way it acts differently than a transformer.

I have quickly went trough it last year already because it is a very good explanation.

But what point in the lecture exactly does it say that this can't be done? Yes it does explain that when considering a lumped inductor all magnetic fields have to be contained inside it. This is another way of saying that the inductors field should not affect anything else around it. Note that the inductor still has wires coming out that are in two different physical locations in order to give you two terminals, this means it can't be a fully closed loop inside the shielded "lumping" box. We still need extra wire outside the box to close it and connect it to for example a voltmeter. The diference between the wires inside the box and outside the box is just that the ones outside have no magnetic field around then and so no EMF. This is the exact same thing as getting rid of the shielding box but placing the wires to the voltmeter in such a path that they generate no EMF. So any piece of wire taking any path inside the shielded box can be considered an inductor (Doesn't have to be coiled up around a former or a core). And if we are careful not to interact the rest of the circuit then the shielding box can be removed and it behaves the same. So putting all of this together any length of superconducting wire can be considered a lumped inductor (tho care must be taken if its not shielded). Are any of my claims here false?

But Dr. Lewins experimental circuit is not laid out in such a way that other parts of the circuit would avoid the field. All sections of wire are enveloping the magnetic field and are so affected by it in the form of EMF. But if you look at a transformer it also has this feature common. There are multiple sections of wire enveloping a common magnetic field in the core. It would be really annoying if we couldn't apply circuit analysis to any circuit containing a transformer, so the inductor model was 'upgraded' to allow it to be friends with other inductors in the same field, this is explained in the section i mentioned:
http://www.feynmanlectures.caltech.edu/II_22.html#Ch22-S8 (http://www.feynmanlectures.caltech.edu/II_22.html#Ch22-S8)

This concept of mutual inductance allows transformers to be modeled with multiple inductors almost as easily as a single inductor. Notice that the mutual inductance value is separate from self inductance, this allows the proportion of the two to be adjusted to obtain any intensity of coupling you want. This is effectively saying how much of the flux the two coupled inductors are sharing (Its also what determines leakage inductance in transformers). Fractional turns in transformers are also possible because a length of wire might not necessarily enclose 100% of the flux in the core.

Putting it all together now. So since any length of wire can be considered an inductor and because these inductors around a common 'core'  can be considered a transformer then this circuit could be considered a transformer with 4 secondary coils connected into a configuration with 2 resistors. If not, can you explain why?

Berni you are 100% right and you summarize it nicely.  Any electrical/electronics engineer will agree with you.  That is exactly what Mehdi demonstrated in his video as well as what your nice LTSpice simulation shows in the first page of this thread.  Dr. Lewin and others only see a "simple" circuit with two "ideal" resistors and two "ideal" wires which results in an analytical "solution" of one equation with one unknown where KVL apparently doesn't hold; but the circuit turns out to be more complex than they expected. A more realistic representation of the circuit has at least 9 coupled inductors, four resistors, and a voltage source.. with an analytical solution that consists of around 10 linear differential equations and 10 unknowns!  Too much for anybody that has not been trained on the solution of these problems.
Title: Re: Does Kirchhoff's Law Hold? Disagreeing with a Master
Post by: bsfeechannel on January 20, 2019, 03:11:24 am
Any electrical/electronics engineer will agree with you.

Any electrical/electronics engineer without a clue about electromagnetism will agree with you.

TIFIFY.
Title: Re: Does Kirchhoff's Law Hold? Disagreeing with a Master
Post by: bsfeechannel on January 20, 2019, 03:18:27 am
I have quickly went trough it last year already because it is a very good explanation.

I told you to read it carefully, not quickly.

Quote
But what point in the lecture exactly does it say that this can't be done?

Glad you asked.

Quote
This is the exact same thing as getting rid of the shielding box but placing the wires to the voltmeter in such a path that they generate no EMF.

[snip]

Are any of my claims here false?

Yes. There are. What generates the EMF is the varying magnetic field. Not the wires.

What the wires do is to nullify any electric field along their path. Just that.

Quote
Putting it all together now. So since any length of wire can be considered an inductor and because these inductors around a common 'core'  can be considered a transformer then this circuit could be considered a transformer with 4 secondary coils connected into a configuration with 2 resistors. If not, can you explain why?

With pleasure. Feynman explains mutual induction after he explains that an inductor is a lumped component.

An inductor can only be considered a lumped component if, and I quote, we assume that there is a negligible magnetic field in the external region near the terminals a and b.  

There are no negligible magnetic fields near the "terminals" of your "inductors" in Lewin's circuit.

So, sorry, you can't lump model Lewin's circuit. It is impossible.

Any more questions?
Title: Re: Does Kirchhoff's Law Hold? Disagreeing with a Master
Post by: jesuscf on January 20, 2019, 04:16:59 am
Any electrical/electronics engineer will agree with you.

Any electrical/electronics engineer without a clue about electromagnetism will agree with you.

TIFIFY.

Whatever you say man, if it makes you feel better about yourself.  Have a happy life.
Title: Re: Does Kirchhoff's Law Hold? Disagreeing with a Master
Post by: Berni on January 20, 2019, 09:26:28 am

I told you to read it carefully, not quickly.


I have read the relevant parts of it in more detail, was just saying why i noticed the section at the end about mutual inductance, as i remember it being there from back then.

A good part of the lecture also goes into circuit analysis and capacitors so i just quickly glossed over those parts rather that going into the details.


Yes. There are. What generates the EMF is the varying magnetic field. Not the wires.

What the wires do is to nullify any electric field along their path. Just that.

Yep that's how you get induction happen in wires. But what does this effect have to do with containing the field inside the component?

With pleasure. Feynman explains mutual induction after he explains that an inductor is a lumped component.

An inductor can only be considered a lumped component if, and I quote, we assume that there is a negligible magnetic field in the external region near the terminals a and b.  

There are no negligible magnetic fields near the "terminals" of your "inductors" in Lewin's circuit.

So, sorry, you can't lump model Lewin's circuit. It is impossible.

Any more questions?

So now what if you replace each section of wire with 1000 turns around the loop then putting it into a shielded box including the solenoid coil while leaving the resistors and voltmeters outside. This is now a transformer lump model because all fields happen inside (the shielding box is ideal so no field makes it outside) and everything inside the box is nothing else but coupled inductors. Don't you agree that such a circuit would behave identically to Dr. Lewins experiment? (Apart from all the voltages being 4000 times higher)
Title: Re: Does Kirchhoff's Law Hold? Disagreeing with a Master
Post by: Vtile on January 20, 2019, 09:52:27 pm
 :popcorn:
Title: Re: Does Kirchhoff's Law Hold? Disagreeing with a Master
Post by: Berni on January 21, 2019, 06:39:23 am
That's how the wires react to the induced field.

Exactly, your point is?

The induced E field depends on the magnetic B field (rate of change) inside the component.

Yes and it also depends on the area cross section trough the field. If the conductor is aligned with the field lines or the fields motion you also get zero EMF even tho the delta B is non zero.


Oh, for the love of...
Try to draw your new 'equivalent' circuit. Then you should see how equivalent it is to the Romer-Lewin ring.

The equivalent circuit is already here:
https://www.eevblog.com/forum/chat/does-kirchhoffs-law-hold-disagreeing-with-a-master/msg1945312/#msg1945312 (https://www.eevblog.com/forum/chat/does-kirchhoffs-law-hold-disagreeing-with-a-master/msg1945312/#msg1945312)

We just have to fix the inductor values to match the new turns ratios.
L = La*N^2

La = L / N^2 = 1µ / (1/4)^2 = 16 µH

Lnew= La * N^2 = 16µ * 1000^2 = 16H

So just replace L2 to L7 with 16H and you have the new equivalent circuit. The inductances seam rather high because i chose fairly high inductance in the original equivalent circuit. This just means that the ring would be fairly large in diameter to give it a large loop area.

Or did you mean make an equivalent circuit for Romers setup? Where a single multichannel oscilloscope can be used due to ground being common.
Title: Re: Does Kirchhoff's Law Hold? Disagreeing with a Master
Post by: Sredni on January 21, 2019, 11:11:54 am
EDIT:

This post has been shortened and cleansed to avoid upsetting other children.
Whatever was written here can be found in one or more of the following books (in no particular order, and without mentioning the usual suspects Feynman, Purcell, Griffiths, Ohanian, Jackson):

Panofsky, Phillips
Classical Electricity and Magnetism 2nd ed

John Kraus
Electromagnetism 2nd to 4th ed

Ramo, Whinnery, VanDuzer
Fields and Waves in Communication Electronics 2nd or 3rd ed

Bleaney
Electricity and Magnetism 3rd ed

Nayfeh, Brussel
Electricity and Magnetism

Kip
Fundamentals of Electricity and Magnetism 2nd ed

Lorrain, Courson
Electromagnetic Fields and Waves 2nd ed

"Books" are static paper based documents that can be found in libraries. They are like smartphones, but (usually) bigger, with lots and lots of extremely thin flexible e-ink screens and a very long battery life. Libraries are...
Oh, never mind. Keep on pushing that square peg into that round hole. With a big enough hammer, it will fit.
Title: Re: Does Kirchhoff's Law Hold? Disagreeing with a Master
Post by: bsfeechannel on January 21, 2019, 06:16:02 pm
Try to draw the circuit, WITH THE VARYING B FIELD REGION, and see if you can manage to do what you proposed: "replace each section of wire with 1000 turns around the loop then putting it into a shielded box including the solenoid coil while leaving the resistors and voltmeters outside".
My bet is that you will end up with circuit (a).

You beat me to it. As always your explanations are precise and rigorous. What Berni tried to do is that, since he believes that the wires are generating voltage, he instructed spice to consider an inductor in their stead. Of course, if you do that, you'll not measure what Lewin did. So he had to "compensate" for that discrepancy on the lead wires and replaced them by coupled inductors.

(https://www.eevblog.com/forum/chat/does-kirchhoffs-law-hold-disagreeing-with-a-master/?action=dlattach;attach=630631;image)

However, this is bullshit. The loop wires are not inductors, because you have a ginormous magnetic field at their terminals and the lead wires are not coupled with the solenoid because they form another loop where there is no varying magnetic field inside.

(https://www.eevblog.com/forum/chat/does-kirchhoffs-law-hold-disagreeing-with-a-master/?action=dlattach;attach=630190;image)

This comes from the fact that he can't understand when Feynman says The whole contribution to the line integral of E comes from the path outside the inductance from terminal b to terminal a. The line integral in the wires is simply zero.
 
He invented a voltage that doesn't exist and had to come up with a compensation that's not necessary.

The conclusion of this thread is that we lamentably have a whole bunch of half assed engineers that do not study, much less understand Maxwell,  and are so mentally crippled that they cannot even do a simple circuit analysis without using Spice as a crutch. They don't even know what they're doing with that tool.

I weep for the future.

Quote
PS
BTW, how do I embed images properly in this forum?

1) Upload your picture as an attachment.
2) Post you message.
3) View the message and copy the link to the picture.
4) Press Modify
5) Place your cursor to the point of your message where you want your picture.
6) Paste the link enclosed by [ img ][ /img ] (without the spaces).
7) Press save
8 ) See the world burn
Title: Re: Does Kirchhoff's Law Hold? Disagreeing with a Master
Post by: Berni on January 21, 2019, 08:46:24 pm
Ah alright that's what is bothering you, alright fine il do some drawing too.

By the way bsfeechannel two of your batteries are drawn backwards (Botton inner two)



So since we are so bothered with how the magnetic field affects things other than wires in the circuit lets fix that by putting the major wires into a shielded box. This box has infinitesimally small holes to conveniently get out wires out of it and is made out of a superconducting material or a material with an infinite permeability. This makes it impossible for the field inside to escape, but because magnetic monopolies are not possible in our universe means that the upwards flux must close itself somewhere. A superconducting box would get eddy currents induced on the inside that produces this opposite downwards flux in the walls, or in the case of an infinite permeability box the field lines would just follow the path of least resistance along the inner surface of the box to flow back down.

(https://www.eevblog.com/forum/chat/does-kirchhoffs-law-hold-disagreeing-with-a-master/?action=dlattach;attach=630313;image)

For convenience i also added some voltmeters around the scene. I think all of you will agree with the readings they are showing. Notice that the rightmost voltmeter is showing zero because the sum magnetic field outside the box is zero (If it was showing anything else than we don't have an ideal shielding box).

So since the field is contained and the box contains nothing else but coupled inductors we can turn it into a ideal transformer and all the voltmeters still show the same readings (Follow each path and add it up if you don't believe me)

(https://www.eevblog.com/forum/chat/does-kirchhoffs-law-hold-disagreeing-with-a-master/?action=dlattach;attach=630319;image)

This shows that an ideal transformer can behave like the above circuit when it is in a box, something that shouldn't be surprising.

So now lets make it actually look a bit like a transformer by giving each wire actual wingdings that go all the way around. For clarity i only went around once (Any grayed out all but two coils to make it easier to see) but you could go around any number of times. Given that the magnetic field stays the same this produces 5 times the voltage, but the circuit otherwise behaves identically.

(https://www.eevblog.com/forum/chat/does-kirchhoffs-law-hold-disagreeing-with-a-master/?action=dlattach;attach=630325;image)

Okay but we don't actually have this magical ideal shielding box. So lets look at what would happen if the box was removed?
Well unsurprisingly the most inner voltmeters that used to show 0.1V and 0.9V would show the same value. The path trough them does not enclose any extra flux so there is no reason for them to show something different. This now completes the chain, the circuit behaves like a transformer even if we don't contain the fields.

Where things do get messed up is all other voltmeters. The magnetic field is now affecting all there probe wires and as a result affecting the voltmeter readings. If we wish to continue doing circuit analysis then the model has to be updated to give those wires correct coupled inductance too, after that the voltmeter readings from circuit analysis will once again match the real thing.

Any objections to this explanation?

And i still don't see what part of Maxwell i supposedly don't understand. All of this makes perfect sense to me from the point of view of Maxwell or from the point of view of mesh  circuit analysis. Where do the explanations above violate Maxwell?

Sorry if it sounds rude but i get the feeling that you two understand Maxwell perfectly well but have some issues understanding circuit analysis and circuit modeling.

EDIT: Oh and where did i put in a magical voltage out of nowhere?
Title: Re: Does Kirchhoff's Law Hold? Disagreeing with a Master
Post by: Sredni on January 21, 2019, 10:00:06 pm
So since we are so bothered with how the magnetic field affects things other than wires ---

<sigh>

You do realize that in the Romer-Lewin ring, the whole ring, resistors included, sits in a region of space where there is NO MAGNETIC FIELD (ideal infinitely long primary coil)?
Why bother putting in a 'magnetic shield'?

Do you want to reconsider your post?

Quote
i get the feeling that you two understand Maxwell perfectly well but have some issues understanding circuit analysis and circuit modeling.

Before starting circuit analysis you need to correctly model your circuit.
And you are not modeling correctly.
Title: Re: Does Kirchhoff's Law Hold? Disagreeing with a Master
Post by: bsfeechannel on January 22, 2019, 04:01:28 am
Ah alright that's what is bothering you, alright fine il do some drawing too.

What's bothering me is that people don't understand electromagnetism and don't want to learn it. And keep coming up with all kinds of pseudo scientific excuses just to try to make their pseudo scientific theory work. This is an insult to serious engineering and is below the level of dignity of this forum.

We're not here to propagate pseudo-science.

Quote
By the way bsfeechannel two of your batteries are drawn backwards (Botton inner two)

I've corrected the picture, thank you, but it won't make any difference. That doesn't in any way model Lewin's circuit because those voltages are not there. It's just an invention to try to cheat Maxwell and make it look as if Kirchhoff always holds.

Quote
So since we are so bothered with how the magnetic field affects things other than wires in the circuit lets fix that by putting the major wires into a shielded box.

Varying magnetic fields do not affect wires. This is something you don't understand. The electrical field inside a wire is ZERO. So wires don't care about varying magnetic fields.

It is the path outside the wire that produces a voltage.

I'll repeat: IT IS THE PATH  O U T S I D E  THE WIRE THAT PRODUCES A VOLTAGE under a varying magnetic field.

And if there is not a varying magnetic field in an area defined by a wire and some path outside that wire, the voltage is ZERO.

Quote
This box has infinitesimally small holes to conveniently get out wires out of it and is made out of a superconducting material or a material with an infinite permeability. This makes it impossible for the field inside to escape, but because magnetic monopolies are not possible in our universe means that the upwards flux must close itself somewhere. A superconducting box would get eddy currents induced on the inside that produces this opposite downwards flux in the walls, or in the case of an infinite permeability box the field lines would just follow the path of least resistance along the inner surface of the box to flow back down.

(https://www.eevblog.com/forum/chat/does-kirchhoffs-law-hold-disagreeing-with-a-master/?action=dlattach;attach=630313;image)

For convenience i also added some voltmeters around the scene. I think all of you will agree with the readings they are showing. Notice that the rightmost voltmeter is showing zero because the sum magnetic field outside the box is zero (If it was showing anything else than we don't have an ideal shielding box).

So since the field is contained and the box contains nothing else but coupled inductors we can turn it into a ideal transformer and all the voltmeters still show the same readings (Follow each path and add it up if you don't believe me)

(https://www.eevblog.com/forum/chat/does-kirchhoffs-law-hold-disagreeing-with-a-master/?action=dlattach;attach=630319;image)

This shows that an ideal transformer can behave like the above circuit when it is in a box, something that shouldn't be surprising.

In this case your modelling is absolutely correct because THIS is a lumped circuit. All the magnetic field is confined outside the area of the circuit. Provided that your voltmeters do not invade the area of the "box", you can place them anywhere.

Quote
Okay but we don't actually have this magical ideal shielding box. So lets look at what would happen if the box was removed?

Well unsurprisingly the most inner voltmeters that used to show 0.1V and 0.9V would show the same value. The path trough them does not enclose any extra flux so there is no reason for them to show something different. This now completes the chain, the circuit behaves like a transformer even if we don't contain the fields.

Where things do get messed up is all other voltmeters. The magnetic field is now affecting all there probe wires and as a result affecting the voltmeter readings. If we wish to continue doing circuit analysis then the model has to be updated to give those wires correct coupled inductance too, after that the voltmeter readings from circuit analysis will once again match the real thing.

Any objections to this explanation?

A lot. This is where you, Mehdi and co., fail miserably. When you remove the box you destroy your transformer. The lines of the magnetic field will return elsewhere. You do not have a lumped component anymore, much less a coupled inductor. But you insist that there must be some kind of error, because the transformer must still exist somehow. After all, what is a transformer? Is it not made of a piece of wire? Well the wire is there, so there must be a transformer too.

Wrong. The wires couldn't care less about the varying magnetic field.

Quote
And i still don't see what part of Maxwell i supposedly don't understand. All of this makes perfect sense to me from the point of view of Maxwell or from the point of view of mesh  circuit analysis.

It makes perfect sense to you because your Maxwell is not the real Maxwell. With your Maxwell, the voltages of a circuit must always add up to zero.

With the real Maxwell, voltages may or may not add up to zero, depending on a series of conditions.

Quote
Where do the explanations above violate Maxwell?

Here and I quote:

This now completes the chain, the circuit behaves like a transformer even if we don't contain the fields.

Where things do get messed up is all other voltmeters. The magnetic field is now affecting all there probe wires and as a result affecting the voltmeter readings. If we wish to continue doing circuit analysis then the model has to be updated to give those wires correct coupled inductance too, after that the voltmeter readings from circuit analysis will once again match the real thing.


This is bullshit. The magnetic field does not affect the probes nor the voltmeter readings. No coupling occurs. No correction is needed.

This is not Maxwell.

The real thing is what the voltmeters are reading.

You're trying to find facts to support your a-priori conclusions. This how religion works, not science.

Quote
Sorry if it sounds rude but i get the feeling that you two understand Maxwell perfectly well but have some issues understanding circuit analysis and circuit modeling.

Circuit analysis is just a special case of Maxwell. If you say that we understand Maxwell, you're saying that we understand circuit analysis perfectly well.

And you have a serious problem with circuit modeling, because you see voltages where they aren't.
Title: Re: Does Kirchhoff's Law Hold? Disagreeing with a Master
Post by: ogden on January 22, 2019, 04:04:37 am
Ah alright that's what is bothering you, alright fine il do some drawing too.

I wonder when you realize that those guys do not have word "agree" in their vocabulary :D They are ready to disprove their own words - if it is you who is speaking ;)
Title: Re: Does Kirchhoff's Law Hold? Disagreeing with a Master
Post by: bsfeechannel on January 22, 2019, 04:47:45 am
Ah alright that's what is bothering you, alright fine il do some drawing too.

I wonder when you realize that those guys do not have word "agree" in their vocabulary :D They are ready to disprove their own words - if it is you who is speaking ;)

I can't see where we are disproving ourselves. We are not claiming anything. We're consistently showing that the claim that Kirchhoff always hold is nothing but quackery.
Title: Re: Does Kirchhoff's Law Hold? Disagreeing with a Master
Post by: bsfeechannel on January 22, 2019, 04:53:39 am
By the way. Here is the real thing without the lumping box, in case someone still has some doubts.

(https://www.eevblog.com/forum/chat/does-kirchhoffs-law-hold-disagreeing-with-a-master/?action=dlattach;attach=630868;image)
Title: Re: Does Kirchhoff's Law Hold? Disagreeing with a Master
Post by: Berni on January 22, 2019, 06:13:34 am
]
By the way. Here is the real thing without the lumping box, in case someone still has some doubts.

(https://www.eevblog.com/forum/chat/does-kirchhoffs-law-hold-disagreeing-with-a-master/?action=dlattach;attach=630868;image)

Yep that's what it would show without the box.

Notice that the inner voltmeters still show 0.1V and 0.9V? Care to explain why they didn't change regardless of the shielding box being there while others did change?




I can't see where we are disproving ourselves. We are not claiming anything. We're consistently showing that the claim that Kirchhoff always hold is nothing but quackery.

It always holds in circuit meshes, not elsewhere. Typical word twisting as usual.




I wonder when you realize that those guys do not have word "agree" in their vocabulary :D They are ready to disprove their own words - if it is you who is speaking ;)

Yeah this has turned into a Maxwell versus Kirchhoff pissing contest 15 pages ago. But so far i have yet to see a good explanation why the two can't be both used provided you know how to use them rather than just slapping formulas on things without knowing what they actually do.
Title: Re: Does Kirchhoff's Law Hold? Disagreeing with a Master
Post by: ogden on January 22, 2019, 11:59:07 am
We're consistently showing that the claim that Kirchhoff always hold is nothing but quackery.

You have serious issues. Nobody claims that Kirchhoff always hold.
Title: Re: Does Kirchhoff's Law Hold? Disagreeing with a Master
Post by: bsfeechannel on January 22, 2019, 02:01:42 pm
By the way. Here is the real thing without the lumping box, in case someone still has some doubts.

(https://www.eevblog.com/forum/chat/does-kirchhoffs-law-hold-disagreeing-with-a-master/?action=dlattach;attach=630868;image)

Yep that's what it would show without the box.

Notice that the inner voltmeters still show 0.1V and 0.9V? Care to explain why they didn't change regardless of the shielding box being there while others did change?

Of course. Because the line integral along the path that includes them and the wires is exactly the same. In other words, the varying magnetic field that the meters and the wires are encircling is exactly the same. The magnetic field outside the closed path doesn't affect the EMF.

This is what Faraday discovered and Maxwell described mathematically. As simple as that. That's the way nature works. There's nothing we can do to change that. You have to accept it. Not because I'm tell you, but because every time you try to repeat this experiment, it will always work that way.

There is, of course, an explanation for the underlying phenomenon of induction, but it is not the topic of this thread.

Quote
I can't see where we are disproving ourselves. We are not claiming anything. We're consistently showing that the claim that Kirchhoff always hold is nothing but quackery.

It always holds in circuit meshes, not elsewhere. Typical word twisting as usual.

NOOOOOOOOOO. Kirchhoff doesn't always hold even for circuit meshes. The inductor itself is a proof of that.

If Kirchhoff always held you couldn't even have inductors, as the voltage inside an inductor, i.e. along the path of the wire, is zero and outside it is different from zero. How can that be?

(https://www.eevblog.com/forum/chat/does-kirchhoffs-law-hold-disagreeing-with-a-master/?action=dlattach;attach=631189;image)

You don't understand it because you didn't read Feynman carefully as I recommended you to. This explanation is there.

It's because Kirchhoff fails that we have inductors, generators, transformers, antennas, etc.

Thanks to your favorite deity, or the lack thereof, that Kirchhoff fails. The failure of Kirchhoff is the best thing that could happen to humankind. Every time Kirchhoff fails, the world smiles. (I think I'll create a t-shirt with those words.)


Quote
I wonder when you realize that those guys do not have word "agree" in their vocabulary :D They are ready to disprove their own words - if it is you who is speaking ;)

Quote
Yeah this has turned into a Maxwell versus Kirchhoff pissing contest 15 pages ago. But so far i have yet to see a good explanation why the two can't be both used provided you know how to use them rather than just slapping formulas on things without knowing what they actually do.

This is not a pissing contest between Maxwell and Kirchhoff. As Kirchhoff is a special case of Maxwell, the only thing we are trying to show you is exactly that.
Title: Re: Does Kirchhoff's Law Hold? Disagreeing with a Master
Post by: bsfeechannel on January 22, 2019, 02:08:17 pm
We're consistently showing that the claim that Kirchhoff always hold is nothing but quackery.

You have serious issues. Nobody claims that Kirchhoff always hold.

Ogden said that Mehdi is nobody. Duly noted.
Title: Re: Does Kirchhoff's Law Hold? Disagreeing with a Master
Post by: ogden on January 22, 2019, 03:32:38 pm
We're consistently showing that the claim that Kirchhoff always hold is nothing but quackery.

You have serious issues. Nobody claims that Kirchhoff always hold.

Ogden said that Mehdi is nobody. Duly noted.

I did not say anything about Mehdi. Your perception is very strange to say it politely.
You shall provide proof of your claim - show where Mehdi say "Kirchhoff always hold".
Title: Re: Does Kirchhoff's Law Hold? Disagreeing with a Master
Post by: bsfeechannel on January 22, 2019, 03:37:06 pm
Look up the word sarcasm in the dictionary.
Title: Re: Does Kirchhoff's Law Hold? Disagreeing with a Master
Post by: ogden on January 22, 2019, 03:40:37 pm
Look up the word sarcasm in the dictionary.

There's no need to telegraph every new word you just learned.
Title: Re: Does Kirchhoff's Law Hold? Disagreeing with a Master
Post by: Berni on January 22, 2019, 04:01:47 pm
Of course. Because the line integral along the path that includes them and the wires is exactly the same. In other words, the varying magnetic field that the meters and the wires are encircling is exactly the same. The magnetic field outside the closed path doesn't affect the EMF.

This is what Faraday discovered and Maxwell described mathematically. As simple as that. That's the way nature works. There's nothing we can do to change that. You have to accept it. Not because I'm tell you, but because every time you try to repeat this experiment, it will always work that way.

There is, of course, an explanation for the underlying phenomenon of induction, but it is not the topic of this thread.


Yes, so when you say they are exactly the same also means that you are saying this circuit exactly acts exactly the same as a ideal transformer. If not, can you show in what way does it behave differently?

Quote
I can't see where we are disproving ourselves. We are not claiming anything. We're consistently showing that the claim that Kirchhoff always hold is nothing but quackery.

It always holds in circuit meshes, not elsewhere. Typical word twisting as usual.

NOOOOOOOOOO. Kirchhoff doesn't always hold even for circuit meshes. The inductor itself is a proof of that.

If Kirchhoff always held you couldn't even have inductors, as the voltage inside an inductor, i.e. along the path of the wire, is zero and outside it is different from zero. How can that be?

(https://www.eevblog.com/forum/chat/does-kirchhoffs-law-hold-disagreeing-with-a-master/?action=dlattach;attach=631189;image)

You don't understand it because you didn't read Feynman carefully as I recommended you to. This explanation is there.

It's because Kirchhoff fails that we have inductors, generators, transformers, antennas, etc.

Thanks to your favorite deity, or the lack thereof, that Kirchhoff fails. The failure of Kirchhoff is the best thing that could happen to humankind. Every time Kirchhoff fails, the world smiles. (I think I'll create a t-shirt with those words.)


Have you ever did AC circuit analysis my hand? If you did then i would have assumed you would have less trouble understanding what an inductor is. Sometimes circuit modeling even uses inductors where there are no magnetic effects involved  (One such example is the common model of a quartz crystal). An inductor is simply U=L*di . If you want to have always zero voltage over it just give it 0H of inductance, but i don't think that's a particularly useful use case for an inductor model.

What exactly are you trying to prove with that diagram? We all know you can't just directly use Kirchhoffs circuit laws inside real world magnetic fields. Did anyone say you can?


This is not a pissing contest between Maxwell and Kirchhoff. As Kirchhoff is a special case of Maxwell, the only thing we are trying to show you is exactly that.

Well yeah its a special case where circuit meshes (Where KVL is meant to be used) without realistically modeled wires happen to behave the same as a real world circuit.

Both Maxwell and Kirchhoff work just fine when used correctly. So why is it a problem that there are two ways to go about calculating electrical circuit behavior?



Title: Re: Does Kirchhoff's Law Hold? Disagreeing with a Master
Post by: bsfeechannel on January 22, 2019, 05:06:30 pm
Yes, so when you say they are exactly the same also means that you are saying this circuit exactly acts exactly the same as a ideal transformer.

No I'm not saying that.

Quote
If not, can you show in what way does it behave differently?

It behaves differently because you made the lines of the varying magnetic field return elsewhere. Now V1 and V2 are equal to V3 and V4, respectively. This means that the sum of the voltages around the inner loop is 1V, which rightfully violates Kirchhoff. So this is not an ideal transformer anymore. This is just a regular circuit subject to induction like simply all real circuits.

Quote
Have you ever did AC circuit analysis my hand? If you did then i would have assumed you would have less trouble understanding what an inductor is.

I do not have any trouble with inductors. I designed and built an isolation transformer and documented it on the Internet. Do you remember?

Quote
Sometimes circuit modeling even uses inductors where there are no magnetic effects involved  (One such example is the common model of a quartz crystal). An inductor is simply U=L*di . If you want to have always zero voltage over it just give it 0H of inductance, but i don't think that's a particularly useful use case for an inductor model.

I bet you read Feynman again and didn't understand what he says when he defines an inductor.

Quote
What exactly are you trying to prove with that diagram? We all know you can't just directly use Kirchhoffs circuit laws inside real world magnetic fields. Did anyone say you can?

The voltages are not measured inside the field. An inductor is the simplest circuit mesh possible. It's just a piece of wire connected to whatever. The voltage across the piece of wire is always zero. The voltage across whatever is different from zero. If you add them up you get something different from zero.

Read Feynman once more and if you still don't understand, maybe we can help.

Quote
Well yeah its a special case where circuit meshes (Where KVL is meant to be used) without realistically modeled wires happen to behave the same as a real world circuit.

No. Stop this pseudo-scientific talk. KVL is a special case of Faraday when there's no varying magnetic field inside the circuit. Repeat until enlightened.

Quote
Both Maxwell and Kirchhoff work just fine when used correctly. So why is it a problem that there are two ways to go about calculating electrical circuit behavior?

There's ONLY ONE theory to explain electricity and magnetism: Maxwell, and Kirchhoff is just a special case of it. This is a tried and proven truth.
Title: Re: Does Kirchhoff's Law Hold? Disagreeing with a Master
Post by: Berni on January 22, 2019, 06:05:31 pm
Quote
If not, can you show in what way does it behave differently?

It behaves differently because you made the lines of the varying magnetic field return elsewhere. Now V1 and V2 are equal to V3 and V4, respectively. This means that the sum of the voltages around the inner loop is 1V, which rightfully violates Kirchhoff. So this is not an ideal transformer anymore. This is just a regular circuit subject to induction like simply all real circuits.


So then if the voltages are different why do the voltmeters show the same values regardless of the circuit being represented with Maxwell or with a ideal transformer?

The last 3 voltmeters ware left out of the transformer model on purpose to show the difference. This is the reason why things don't work when Maxwell and ideal lumped circuit analysis is recklessly mixed together. If you stick to just one for the entire circuit then things work fine.

Quote
Have you ever did AC circuit analysis my hand? If you did then i would have assumed you would have less trouble understanding what an inductor is.

I do not have any trouble with inductors. I designed and built an isolation transformer and documented it on the Internet. Do you remember?


Then you also shouldn't have trouble understanding how to apply a transformer model to a real circuit.




Quote
What exactly are you trying to prove with that diagram? We all know you can't just directly use Kirchhoffs circuit laws inside real world magnetic fields. Did anyone say you can?

The voltages are not measured inside the field. An inductor is the simplest circuit mesh possible. It's just a piece of wire connected to whatever. The voltage across the piece of wire is always zero. The voltage across whatever is different from zero. If you add them up you get something different from zero.

Read Feynman once more and if you still don't understand, maybe we can help.

That's why i put the shielding box there, none of the voltages there are measured inside the field. What is the reason that the inside of the box could not be considered a transformer?

What part of Feynmans lecture did i understand wrong?

Quote
Well yeah its a special case where circuit meshes (Where KVL is meant to be used) without realistically modeled wires happen to behave the same as a real world circuit.

No. Stop this pseudo-scientific talk. KVL is a special case of Faraday when there's no varying magnetic field inside the circuit. Repeat until enlightened.

No thanks, that's called religion instead of engineering.

I believe something when its backed up by experimental data or a sensible explanation, not because someone told me to believe something without giving a reason to back it up. So far i still have not seen a explanation in what way circuit analysis is wrong about Dr. Lewins circuit (Even tho it reliably produces results that match the experimental results).

So far experimental results and explanations found on the topic show me that both are valid ways of evaluating the circuit and both produce identical results.

Quote
Both Maxwell and Kirchhoff work just fine when used correctly. So why is it a problem that there are two ways to go about calculating electrical circuit behavior?

There's ONLY ONE theory to explain electricity and magnetism: Maxwell, and Kirchhoff is just a special case of it. This is a tried and proven truth.

Exactly and Maxwell is explaining that one and is doing a great job. Kirhhoffs circuit laws instead explain how circuit meshes work, hence why they are not directly interchangeable.

Does this also mean we have to chose between Maxwell equations and Quantum electrodynamics to declare one of them being for the birds because they explain a similar area of science?
Title: Re: Does Kirchhoff's Law Hold? Disagreeing with a Master
Post by: bsfeechannel on January 22, 2019, 08:01:55 pm
What part of Feynmans lecture did i understand wrong?

Alright. Since that's a broad question, let's investigate. Please, tell me what is the voltage (it does need to be a number) between points A and B in the picture below? We have a varying magnetic field.

(https://www.eevblog.com/forum/chat/does-kirchhoffs-law-hold-disagreeing-with-a-master/?action=dlattach;attach=631513;image)
Title: Re: Does Kirchhoff's Law Hold? Disagreeing with a Master
Post by: Berni on January 22, 2019, 08:09:06 pm
Its undefined since you haven't specified a path.
Title: Re: Does Kirchhoff's Law Hold? Disagreeing with a Master
Post by: bsfeechannel on January 22, 2019, 09:52:01 pm
Perfect. Now let's suppose that I define two paths: #1 from A to B and #2 from B to A. Let's suppose, then, that the voltages measured between these two points following these two different paths are different. If we start form point A via path #1 and return to it via path #2, and if we add up these two voltages, will we have zero volts?

(https://www.eevblog.com/forum/chat/does-kirchhoffs-law-hold-disagreeing-with-a-master/?action=dlattach;attach=631588;image)
Title: Re: Does Kirchhoff's Law Hold? Disagreeing with a Master
Post by: Sredni on January 22, 2019, 10:03:28 pm
Ah alright that's what is bothering you, alright fine il do some drawing too.

I wonder when you realize that those guys do not have word "agree" in their vocabulary :D

You should be grateful for that.
Had I agreed with you a month ago, you would still not know how a transformer works.
Title: Re: Does Kirchhoff's Law Hold? Disagreeing with a Master
Post by: Berni on January 23, 2019, 06:15:42 am
Perfect. Now let's suppose that I define two paths: #1 from A to B and #2 from B to A. Let's suppose, then, that the voltages measured between these two points following these two different paths are different. If we start form point A via path #1 and return to it via path #2, and if we add up these two voltages, will we have zero volts?

(https://www.eevblog.com/forum/chat/does-kirchhoffs-law-hold-disagreeing-with-a-master/?action=dlattach;attach=631588;image)

Nope
Title: Re: Does Kirchhoff's Law Hold? Disagreeing with a Master
Post by: bsfeechannel on January 23, 2019, 01:08:48 pm
Thank you. So far so good. But we can already notice that there's something strange here. Because we are accustomed to talk about voltages between two different points. Normally we tend to think that the voltage from a point to itself should be zero. Just because we chose different paths to go and come back we have a voltage. Since this is weird, let's call this voltage EMF (Electromotive Force).

I don't know the length of those paths, nor the area they bound, but since I know you know how to calculate this EMF, given the magnetic field B and the area, let's skip that part and suppose that this EMF is 1V.

Now I'd like to introduce you to the wire. Wire is an interesting material. The electric field inside it is always zero. No matter what. It doesn't care about magnetic fields, varying or static. It also doesn't care about electric fields outside itself. Its "jurisdiction" is limited to its premises. There electric fields are banned. Let's have this properties very well understood in our minds for the next question.

If we connect points A and B with a piece of this wire, what will be the voltage between A and B?

(https://www.eevblog.com/forum/chat/does-kirchhoffs-law-hold-disagreeing-with-a-master/?action=dlattach;attach=632095;image)
Title: Re: Does Kirchhoff's Law Hold? Disagreeing with a Master
Post by: Berni on January 23, 2019, 04:27:25 pm
Following the textbook definition of voltage
Path1: 0V
Path2: 866.6 mV

Following the circuit analysis definition of voltage:
Path1: 133.4mV
Path2: 133.4mV
Title: Re: Does Kirchhoff's Law Hold? Disagreeing with a Master
Post by: bsfeechannel on January 23, 2019, 05:16:38 pm
Can  you please show how you reached those numbers?
Title: Re: Does Kirchhoff's Law Hold? Disagreeing with a Master
Post by: Berni on January 23, 2019, 06:01:17 pm
(https://www.eevblog.com/forum/chat/does-kirchhoffs-law-hold-disagreeing-with-a-master/?action=dlattach;attach=632236;image)

Following the textbook definition of voltage
Path1: 0V (Because as you said there is a wire there that nulls out the sum of E fields)
Path2: No wire to null the field so the voltage is purely the EMF around that path, however the EMF was specified for the whole loop so Path1 has to be subtracted out
Total Area of loop: 35.295 cm2
Total EMF voltage around the loop: 1V
Loop area occupied by Path1: 4.71 cm2
Loop area ratio: 4.71 / 35.295 =  0.1334
Loop2 EMF: 1V * (1-0.1334) = 866.6 mV



Following the circuit analysis definition of voltage:
Path1: Amount of charge separation caused by the EMF in the wire
Loop1 EMF: 1V * 0.1334 = 133.4mV
Path2: Same two points so same voltage: 133.4 mV


If your results don't agree please also explain how you reached your numbers.

EDIT:
Sorry forgot about the electrostatic field created by the wire affecting the other path. See: https://www.eevblog.com/forum/chat/does-kirchhoffs-law-hold-disagreeing-with-a-master/msg2144605/#msg2144605 (https://www.eevblog.com/forum/chat/does-kirchhoffs-law-hold-disagreeing-with-a-master/msg2144605/#msg2144605)
Title: Re: Does Kirchhoff's Law Hold? Disagreeing with a Master
Post by: Sredni on January 23, 2019, 08:32:50 pm

Following the circuit analysis definition of voltage:
Path1: Amount of charge separation caused by the EMF in the wire
Loop1 EMF: 1V * 0.1334 = 133.4mV
Path2: Same two points so same voltage: 133.4 mV


If your results don't agree please also explain how you reached your numbers.

I do not want to spoil bsfeechannel's fun, so I will only pose a question.
To be clear, the unique, single-valued, 'circuit-analysis-defined' voltage across the physically tangible piece of wire has a value that depends on the area (or ratio thereof) that such wire defines with an arbitrary imaginary path?

I mean, if the imaginary 'path #2' had a vertical side  comprised between the symbols "2" and "B" inside the square, you would have found a different value for the unique circuit-analysis voltage? And another one, if it went way out on the left of the paper?

Edit: grammar and clarified where w and B are.
Title: Re: Does Kirchhoff's Law Hold? Disagreeing with a Master
Post by: bsfeechannel on January 23, 2019, 08:41:42 pm

I do not want to spoil bsfeechannel's fun, so I will only pose a question.

No wuckers.
Title: Re: Does Kirchhoff's Law Hold? Disagreeing with a Master
Post by: bsfeechannel on January 23, 2019, 08:52:31 pm
Following the textbook definition of voltage
Path1: 0V (Because as you said there is a wire there that nulls out the sum of E fields)
Path2: No wire to null the field so the voltage is purely the EMF around that path, however the EMF was specified for the whole loop so Path1 has to be subtracted out
Total Area of loop: 35.295 cm2
Total EMF voltage around the loop: 1V
Loop area occupied by Path1: 4.71 cm2
Loop area ratio: 4.71 / 35.295 =  0.1334
Loop2 EMF: 1V * (1-0.1334) = 866.6 mV



Following the circuit analysis definition of voltage:
Path1: Amount of charge separation caused by the EMF in the wire
Loop1 EMF: 1V * 0.1334 = 133.4mV
Path2: Same two points so same voltage: 133.4 mV


If your results don't agree please also explain how you reached your numbers.

I think I owe you an apology. My drawing was not sufficiently clear. The magnetic field B should be spread uniformly all over the page. But that's OK, because the next step would be to make the magnetic field spread uniformly over the area like in the picture below. I.e., it is zero outside the loop formed by paths #1 and #2, and it is also zero for the blank portion of the same loop. B's intensity will be adjusted so that, together with that area (that you may consider square if you want), the EMF is still 1V.

I also forgot to say that the wire does not produce charge separation. It just "magically" nullifies any attempt at producing an electric field inside it. However you can keep on calculating your "circuit analysis definition of voltage" as if it were, if you want.

As for the voltage according to your "text definition of voltage", path #1 is OK, even with my unclear drawing. Path #2 I think would be a different value, but since my drawing is screwed, it is OK that at least you considered it different from zero.

So now, we're gonna use this ideal piece of wire to cover path #2 and we will leave path #1 free. We still need to know the voltage between points A and B. Let's see if our calculations will lead to the same value, shall we?

(https://www.eevblog.com/forum/chat/does-kirchhoffs-law-hold-disagreeing-with-a-master/?action=dlattach;attach=632479;image)
Title: Re: Does Kirchhoff's Law Hold? Disagreeing with a Master
Post by: Berni on January 23, 2019, 09:04:58 pm

Following the circuit analysis definition of voltage:
Path1: Amount of charge separation caused by the EMF in the wire
Loop1 EMF: 1V * 0.1334 = 133.4mV
Path2: Same two points so same voltage: 133.4 mV


If your results don't agree please also explain how you reached your numbers.

I do not want to spoil bsfeechannel's fun, so I will only pose a question.
To be clear, the unique, single-valued, 'circuit-analysis-defined' voltage across the physically tangible piece of wire has a value that depends on the area (or ratio thereof) that such wire define with an arbitrary imaginary path?

I mean, if the imaginary 'path #2' had a vertical side  comprise between 2 and B, you would have found a different value for the unique circuit-analysis voltage? And another one, if it went way out on the left of the paper?

The dependence on Path2 comes from the fact that bsfeechannel defined the strength of the field being as strong to generate 1V of EMF for the entire loop. So given this being a uniform field means that a larger surface area always means more of the field is enclosed in the loop, this means the field has to be weaker to still produce 1V. Hence why making the loop area around Path2 larger causes the voltage on Path1 appear smaller because the field is still in the same spot but its weaker. It also changes if you move the field origin around, but does not change once a loop is a fully closed path.
Title: Re: Does Kirchhoff's Law Hold? Disagreeing with a Master
Post by: Berni on January 23, 2019, 09:08:55 pm
I think I owe you an apology. My drawing was not sufficiently clear. The magnetic field B should be spread uniformly all over the page. But that's OK, because the next step would be to make the magnetic field spread uniformly over the area like in the picture below. I.e., it is zero outside the loop formed by paths #1 and #2, and it is also zero for the blank portion of the same loop. B's intensity will be adjusted so that, together with that area (that you may consider square if you want), the EMF is still 1V.

I also forgot to say that the wire does not produce charge separation. It just "magically" nullifies any attempt at producing an electric field inside it. However you can keep on calculating your "circuit analysis definition of voltage" as if it were, if you want.

As for the voltage according to your "text definition of voltage", path #1 is OK, even with my unclear drawing. Path #2 I think would be a different value, but since my drawing is screwed, it is OK that at least you considered it different from zero.

So now, we're gonna use this ideal piece of wire to cover path #2 and we will leave path #1 free. We still need to know the voltage between points A and B. Let's see if our calculations will lead to the same value, shall we?

(https://www.eevblog.com/forum/chat/does-kirchhoffs-law-hold-disagreeing-with-a-master/?action=dlattach;attach=632479;image)

Just missed your reply while writing my post.
Yes i assumed the field B to be uniform across the page since there was no confinement of the field mentioned.

Now that the wire is moved Path2 is 0V since the wire is nulling the sum E field along that path.
Title: Re: Does Kirchhoff's Law Hold? Disagreeing with a Master
Post by: bsfeechannel on January 23, 2019, 09:57:46 pm
And what about path #1?
Title: Re: Does Kirchhoff's Law Hold? Disagreeing with a Master
Post by: Sredni on January 23, 2019, 11:52:59 pm

The dependence on Path2 comes from the fact that bsfeechannel defined the strength of the field being as strong to generate 1V of EMF for the entire loop. So given this being a uniform field means that a larger surface area always means more of the field is enclosed in the loop, this means the field has to be weaker to still produce 1V. Hence why making the loop area around Path2 larger causes the voltage on Path1 appear smaller because the field is still in the same spot but its weaker.

This post has been shortened and cleansed to avoid upsetting other children.
Whatever was written here can be found in one or more of the following books (in no particular order, and without mentioning the usual suspects Feynman, Purcell, Griffiths, Ohanian, Jackson):

Panofsky, Phillips
Classical Electricity and Magnetism 2nd ed

John Kraus
Electromagnetism 2nd to 4th ed

Ramo, Whinnery, VanDuzer
Fields and Waves in Communication Electronics 2nd or 3rd ed

Bleaney
Electricity and Magnetism 3rd ed

Nayfeh, Brussel
Electricity and Magnetism

Kip
Fundamentals of Electricity and Magnetism 2nd ed

Lorrain, Courson
Electromagnetic Fields and Waves 2nd ed

Title: Re: Does Kirchhoff's Law Hold? Disagreeing with a Master
Post by: Berni on January 24, 2019, 06:13:26 am
And what about path #1?

Sorry when i headed off for the night i remembered that i forgot to consider the electrostatic effect of the wire on the other path. So what i calculated in the first example is the voltage across Path2 when the wire along Path1 is not there.

Going back to the first example with numbers the 866.6 mV is just the magnetic EMF, but in the wire charge separation equivalent to 133.4 mV has occurred in order to null out the sum E field. This field is obviusly not confined to the the wire so it also creates a electrostatic field of 133.4 mV on the outside of the wire and this includes Path2 so the two have to add up 133.4 mV + 866.6 mV = 1V.

So in the more recent example yeah Path1 is 1V now.
Title: Re: Does Kirchhoff's Law Hold? Disagreeing with a Master
Post by: Berni on January 24, 2019, 06:57:18 am

RESPONSE


This makes sense, but, if the field is uniform - and you made no assumptions on what has generated it (remember the boundary conditions set by the shape of the primary coil?) how do you decide how to split the area? You assume the point with the B is the "origin" of the B  field, and this term sounds a bit strange to me.
Are you assuming a circular generating coil so that the induced E field is directed along concentric circles centered at that point?


All magnetic fields have an origin no matter how they are generated, this is something i realized along the way of this thread.

One way you could generate this uniform field is a coil going around the whole page outside of view, origin is the center of this coil. Alternatively you could have the whole page set inside of a airgap of a giant magnetic core. In that case whatever source somewhere on the core is generating the field (Be it a coil or a moving magnet) and the core is bringing the field trough the page, in this case the geometric center of the core poles above and below the circuit determine the field origin. Another way is to have a flexible core with a fixed permanent magnet somewhere in it and then you change the surface area of the cores poles above and below the circuit to concentrate the field down into a smaller area and so make it stronger as the shape of the core moves inwards, in this case the center point of this uniform scale transformation applied to the core is the magnetic origin.

No matter how it is created this is a point where the magnetic filed lines stay put while other lines crowd up or disperse as the field changed strength.

But in a lot of cases this origin can be ignored because the effects of changing where it is null out.


Oh, before I forget, I finally managed to scan my drawings about your 'magnetically shielded' circuit. Let me see if I can show you that your lumped circuit with four lumped coils is a different system from the unlumpable Romer-Lewin circuit.

Let's say we have a hypermu material that captures all magnetic field lines and lets nothing out. We put primary and secondary coil inside it in this way:

(https://www.eevblog.com/forum/chat/does-kirchhoffs-law-hold-disagreeing-with-a-master/?action=dlattach;attach=632611)
Fig. the hypermu shield


Nice perspective drawing there by the way.


Now, let's look it as a cross section showing the B field lines entering and exiting the page. Since the material is magical, no field lines escape and I can exaggerate the holes needed to put our probes in. Circuit A1 is closest to the physical system, while circuit A2 is the same, but with the arc exaggerated.

(https://www.eevblog.com/forum/chat/does-kirchhoffs-law-hold-disagreeing-with-a-master/?action=dlattach;attach=632617)
Fig circuits A1 and A2

Do you recognize circuit (a) of my previous post? Here KVL works on both inner and outer circuit paths.
(Edit: with 'external circuit path' I mean the external voltmeter you placed to show 0V, as long as you enclose zero net (varying) magnetic field it behaves according to KVL)
(Edit: it seems I've used letters consistent with my previous post, after all)

Now, do you realize that systems A1 and A2 are different from system B, the Romer-Lewin ring?

(https://www.eevblog.com/forum/chat/does-kirchhoffs-law-hold-disagreeing-with-a-master/?action=dlattach;attach=632623)
Fig circuit A

Here KVL DIES on both inner and outer circuit paths. (the field lines close at infinity).


EDIT: added the missing "zero" before net magnetic field. Time to go to bed.

Well the way i see it is that Fig3 is sort of a zoomed in Fig2.

The field still returns back the other way to close itself, except that it happens a lot farther away (infinitely far). So anything the happened inside the hollow part of the toroid is now happening across the whole page.

The equivalent for going in reverse to get Fig3 to be like Fig2 is to take everything on the page in Fig3 and stuffing it inside that hollow HyperMu™ toroid. So the Romer-Lewin cirucit inside can see no difference between Fig2 and Fig3 since its inside in both cases. But if we move anything on the outside of the ring in Fig2 inside the toroid it suddenly becomes a transformer secondary as well, hence why i have modeled the voltmeter probe wires as coupled inductors. The only way the probes could avoid being affected by the field is having them run outside of the toroid in your so called "KVL safe area". Wires outside the toroid still have inductance, but it never shows any voltage across it (No intersecting field to generate it and no current flow trough the voltmeter to generate its own magnetic field) so we can just pretend the inductors are not there as it doesn't affect the circuit in any way.

Since as you pointed out this "KVL safe area" does not exist in Fig3 means that we must consider all wires passing around the field to be magnetically coupled to it.

Title: Re: Does Kirchhoff's Law Hold? Disagreeing with a Master
Post by: bsfeechannel on January 24, 2019, 11:12:17 am
And what about path #1?
[snip]
So in the more recent example yeah Path1 is 1V now.

Thank you. We are almost there. Please bare with me. So far, your answers for the last example are checking OK against Feynman's lectures. But we need to check a few more concepts.

Now can you explain why exactly 1V for path #1?
Title: Re: Does Kirchhoff's Law Hold? Disagreeing with a Master
Post by: Berni on January 24, 2019, 11:29:56 am
Because the total EMF around the loop is 1V.

So the voltage contribution from the EMF field down Path1 is 133.4 mV while the charge separation in the ends of the wire on Path2 cause an additional  866.6 mV of electrostatic field. So then 133.4 mV + 866.6 mV = 1V

Or another way is to just says that Faradays law says its 1V because that's how much you need to get to 1V for the integral around the loop. But that explanation is a bit underwhelming because it doesn't really show where the voltage comes from.
Title: Re: Does Kirchhoff's Law Hold? Disagreeing with a Master
Post by: bsfeechannel on January 24, 2019, 12:37:36 pm
Alright. The important thing here is to notice that the EMF is 1V with or without the wire. The wire seems to be irrelevant for the generation of that EMF, albeit it causes a distortion of the electric field along paths #1 and #2 (reducing #2 to zero and enhancing #1 in compensation).

Now comes the crucial question. What is generating this 1V EMF? In other words, what is the fundamental cause behind it?
Title: Re: Does Kirchhoff's Law Hold? Disagreeing with a Master
Post by: Berni on January 24, 2019, 04:53:26 pm
That depends how low do you define as being fundamental, so i'm going to go lower and lower in steps to hopefuly cover the level you are after.

Faraday law:
EMF Voltages comes from a loop area enclosing a changing magnetic field

Maxwell:
EMF Voltage comes from an the electric fields relation to the vector curl of the magnetic field.

Special relativity:
EMF Voltage comes from an apparent electric field generated by warping of spacetime due to the relative motion of charged particles.

Quantum electrodynamics:
EMF Voltage is the result of a moving observers interaction with a electromagnetic field trough the exchange of particles. (Please don't ask too many questions about this one because i don't really understand much about it)
Title: Re: Does Kirchhoff's Law Hold? Disagreeing with a Master
Post by: Sredni on January 24, 2019, 09:51:00 pm
Quote from: Sredni
Now, do you realize that systems A1 and A2 are different from system B, the Romer-Lewin ring?
Well the way i see it is that Fig3 is sort of a zoomed in Fig2.
I've corrected my captions, apologies for that. I believe you mean circuit (B) is different equivalent to circuit (A2)

It is not.

This post has been shortened and cleansed to avoid upsetting other children.
Whatever was written here can be found in one or more of the following books (in no particular order, and without mentioning the usual suspects Feynman, Purcell, Griffiths, Ohanian, Jackson):

Kip
Fundamentals of Electricity and Magnetism 2nd ed

Lorrain, Courson
Electromagnetic Fields and Waves 2nd ed

John Kraus
Electromagnetism 2nd to 4th ed

Ramo, Whinnery, VanDuzer
Fields and Waves in Communication Electronics 2nd or 3rd ed

Bleaney
Electricity and Magnetism 3rd ed

Nayfeh, Brussel
Electricity and Magnetism

Panofsky, Phillips
Classical Electricity and Magnetism 2nd ed
Title: Re: Does Kirchhoff's Law Hold? Disagreeing with a Master
Post by: bsfeechannel on January 25, 2019, 03:16:49 am
That depends how low do you define as being fundamental, so i'm going to go lower and lower in steps to hopefuly cover the level you are after.

Faraday law:
EMF Voltages comes from a loop area enclosing a changing magnetic field

Maxwell:
EMF Voltage comes from an the electric fields relation to the vector curl of the magnetic field.

Special relativity:
EMF Voltage comes from an apparent electric field generated by warping of spacetime due to the relative motion of charged particles.

Quantum electrodynamics:
EMF Voltage is the result of a moving observers interaction with a electromagnetic field trough the exchange of particles. (Please don't ask too many questions about this one because i don't really understand much about it)

What about somewhere between Faraday and Maxwell?

For these two the ultimate cause of the EMF is the varying magnetic field. But OK, this means we can proceed.

Now let me introduce you to the resistor. Differently from our ideal wire, resistors do allow the existence of an electric field inside them. When connected to a circuit, they will let a current flow that is proportional to the line integral of the electric field along their paths divided by their resistance.

Now let's connect this resistor to our wire . A current will obviously flow from A to B. This will generate a magnetic field with a direction that will tend to oppose the magnetic field that generated the E.M.F. However, let's suppose that the resistance is sufficiently high, the current is too low, and consequently this opposing magnetic field is so weak that we can consider it negligible.

Let's suppose that R = 1k ohms (producing a 1mA current).

Using my drawing below, i.e. respecting the geometry of the paths, how would you please model this circuit using your circuit analysis technique?

(https://www.eevblog.com/forum/chat/does-kirchhoffs-law-hold-disagreeing-with-a-master/?action=dlattach;attach=633748;image)
Title: Re: Does Kirchhoff's Law Hold? Disagreeing with a Master
Post by: Doctorandus_P on January 25, 2019, 04:01:35 am
I could only watch a part of the (always respected and liked) big brow guy.

When you draw a schematic, it is only a model, a representation of the outside world.
If there is some external magnetic field which induces a current in your schematic, you have to put a representation of that induced current it in your model.
So then it becomes a schematic with 2 resistors and a current source.
Problem solved.

Lewin is making assumptions from an incomplete / incorrect model by ignoring the current source while drawing his model for the Kirchof "law".
There really is no need to go any deeper into this.
Title: Re: Does Kirchhoff's Law Hold? Disagreeing with a Master
Post by: LukeW on January 25, 2019, 07:56:45 am
Lewin draws a schematic diagram that will be familiar to any freshman engineer - a battery, a couple of resistors. Let's analyze it using Ohm's and Kirchoff's laws.

It's tempting to immediately put an end to this "paradox" by specifying a more rigorous assumption for Kirchoff analysis - that Kirchoff's voltage law can't be applied to a loop with a magnetic flux applied through it. And KCL can't be applied to systems that emit or capture external electrons into that node.

Asking a question about the effect of an external magnetic flux on the circuit really makes this an electromagnetic compatibility question - and you're not going to understand EMC problems with lumped models and ideal wires.

Kirchoff's and Ohm's laws are not Maxwell's equations. They are deliberately simplified, and very useful tools within a certain context - but they're not really the right tools for an electrodynamics problem. They're not designed to do an electrodynamics job.

It's easy to see, naively, that applying magnetic flux may "break" Kirchoff's voltage law, in its traditional first-year-undergrad expression.

Similarly, we can imagine scenarios that "break" Kirchoff's current law - a thermionic tube, a cathode-ray tube, a Faraday cup, a radiation detector like an ion chamber. Any system that involves the emission or capture of electrons from outside, or to outside, the simplified system boundary will appear to "break" Kirchoff's current conservation at that circuit node.

But the problem here, the real paradox, is not the use of Kirchoff's laws.

Kirchoff's voltage law is, basically, a statement of conservation of energy (simplified within a certain context.) Kirchoff's current law is basically a statement of conservation of charge. These are really solid, foundational principles of physics - it's hard to imagine genuine violation of energy or charge conservation.

Kirchoff's laws work. Something is wrong with the circuit model.

The "paradox" here is not the use of lumped circuit elements, either.
We can make a reasonable model of the circuit with lumped elements.

The problem here is that we draw the schematic symbol for a battery, and the schematic symbol for a couple of resistors, and then we draw these lines between them. What are those lines on the blackboard, the lines between the battery and the resistor?
This is Lewin's great "paradox" in a nutshell.
Nobody teaching basic electronics ever talks about the lines, and we need to talk about the lines.

These lines are "ideal wire".

Ideal wire has no resistance, no capacitance to the groundplane, no mass, no cost, infinite tensile strength, infinite flexibility, infinite resistance to corrosion or insulation degradation, no resistive heating, no skin effect, no crosstalk, no limit to its current-carrying capacity before the insulation melts off, no voltage drop etc. It's really easy to solder, strip and terminate.

It's a spherical frictionless cow in a vacuum.

We take a coil of Ideal Wire, put it between the poles of a magnet, with a commutator, and spin it around. What voltage is observed?

No EMF generated? Nothing?

Frustrated, you check the Ideal Wire datasheet again.
Inductance: 0 nH/m.

Hmmm.

Ideal Wire can't couple to a magnetic (or electric) field.

In the freshman physics class, we don't really tease out the inductance of the wire loop as an important quantity that can be measured - it's not used the same way engineers use it.

But the treatment of electromagnetic systems with Maxwell's equations and integrals of the B vector dot product with the area vector and all that sort of thing can be used to shake out the fact that inductance is an intrinsic physical property of the wire loop, and it is not zero.

You can include the effect of magnetic flux coupling into the loop by drawing in an inductor as a lumped circuit element and identifying the appropriate voltage across it. You can keep it as a lumped element connected by ideal wires.

There's a lesson here for the students.
All the interesting things in EE, all the complicated things of practical importance - electromagnetic compatibility, signal integrity in high-speed digital systems, antennas, transmission lines, shielding, RF design, EMI, crosstalk, power transmission - all have at their core an understanding of Real Wires.

Cable assembly, manufacturing, labor, economics, testing at scale, reliability in demanding environments such as automotive or aerospace - Real Wires (and their connectors and terminations, which are a key part of real wires) are crucial here too.

When students are frustrated with breadboards, or when student projects don't work in the lab, it's never because their stock of resistors or transistors or opamps are faulty. It's almost always because of wires.
Title: Re: Does Kirchhoff's Law Hold? Disagreeing with a Master
Post by: bsfeechannel on January 25, 2019, 11:13:16 am
Lewin is making assumptions from an incomplete / incorrect model by ignoring the current source while drawing his model for the Kirchof "law".
There really is no need to go any deeper into this.

Those who say Lewin is wrong have a tendency to not want to go deeper in the subject. If Lewin was really wrong, they would not demonstrate this consistent fear.
Title: Re: Does Kirchhoff's Law Hold? Disagreeing with a Master
Post by: Sredni on January 25, 2019, 12:00:49 pm
We take a coil of Ideal Wire, put it between the poles of a magnet, with a commutator, and spin it around. What voltage is observed?
No EMF generated? Nothing?
Frustrated, you check the Ideal Wire datasheet again.
Inductance: 0 nH/m.
Hmmm.
Ideal Wire can't couple to a magnetic (or electric) field.

What. Are. You. Talking. About.

<sigh>

Edit: added context.
Title: Re: Does Kirchhoff's Law Hold? Disagreeing with a Master
Post by: Berni on January 25, 2019, 06:24:17 pm
What about somewhere between Faraday and Maxwell?

For these two the ultimate cause of the EMF is the varying magnetic field. But OK, this means we can proceed.

Now let me introduce you to the resistor. Differently from our ideal wire, resistors do allow the existence of an electric field inside them. When connected to a circuit, they will let a current flow that is proportional to the line integral of the electric field along their paths divided by their resistance.

Now let's connect this resistor to our wire . A current will obviously flow from A to B. This will generate a magnetic field with a direction that will tend to oppose the magnetic field that generated the E.M.F. However, let's suppose that the resistance is sufficiently high, the current is too low, and consequently this opposing magnetic field is so weak that we can consider it negligible.

Let's suppose that R = 1k ohms (producing a 1mA current).

Using my drawing below, i.e. respecting the geometry of the paths, how would you please model this circuit using your circuit analysis technique?

(https://www.eevblog.com/forum/chat/does-kirchhoffs-law-hold-disagreeing-with-a-master/?action=dlattach;attach=633748;image)

The circuit is simple enough that very little circuit analysis is actually needed on it.

We know the voltage around it is 1V, We know the total loop resistance is 1 KOhm so trough I=U/R=1V / 1KOhm = 1mA.
At this point the voltages and currents across components are known so the circuit is solved.

Since you are looking for the textbook definition the voltage between points AB is both 0V and 1V.

If you are after the charge density at the points then it involves a bit more work as the resistive bar must be modeled as a resistor in series with an inductor and then it does start to matter where in space the points are. But since you are not interested in this unscientific made up voltage il save myself some work and leave it there.






Right. The whole point is that when the lines close at infinity or, for what matters outside your lab, you have no way to access the circuit without having a net varying B field enclosed by the circuit's path. And this is what makes KVL fail.

Exactly because i don't have access to a wire that goes outside of it i simply model the wire as part of the cirucit. If i know the exact voltage across the wire means i can use math to figure out what voltage is on the other end when i know the voltage on this end(Where its shown by the voltmeter)

Instead of avoiding the field, its simply compensated for


Look better: in one case you cannot get rid of the varying B field inside the circuit path. The reason is that the 'returning lines' are way out of your reach. So you cannot 'hide' the whole field inside the components, hence you cannot have lumped components.
If you cannot see this, I don't know how to help you.



As for the rest, man, are you sure you do not have a reset button somewhere? I really mean no disrespect (I believe we've been able to argue without resorting to insults and keeping it quite polite) but... the way you try to compute 'partial emfs' based on areas makes me think of cargo cult science.

Using the area ratios works as long as the symmetry of the system allows it such as in the case of primary and secondary circular coils on the same axis. When they are off center, or have a different shapes, things are not so easy. So, while it is true that you get different results according to where the circuit is placed inside the field (let's focus on the field generated by a circular primary coil), you need to take the shape of the induced E field into account if you want to find the path integral on a portion of the closed curve (or the distribution of surface charge on the conductor).
(EDIT: come to think of it again, it seems that all is required is that the primary coil be circular as that seems the only way to get a uniformly distributed magnetic field inside. Someone has proof of that?)

And regarding the 'origin of the field' as the point where field lines stay put when you change the intensity... It looks intriguing, but does it have any physical motivation? To me is just a graphical representation, nothing more. You seem to think that when the field diminishes in strength the 'number of points' where you can find the B field diminished as well...


Edit: corrected duplicate "positioning" with "shapes"
Edit: grammar
EDIT: different --> equivalent. Man, I really need to sleep better. That fucking neighbor's dog, I can hear him with all windows closed. Maybe a 400W ultrasound whistle...


It works in 3D space just as well, except that the origin is not a point anymore but instead a line.

So far the way i worked with partial wire segments seamed to align with experimental results. The goal of this is circuit modeling to consistently construct a mesh model that behaves like the circuit in question. This model then allows to to reliably apply many circuit analysis tools(Including KVL) that don't always work when just slapped directly on a real circuit. Its not only KVL that sometimes breaks when directly slapped onto a real circuit, many other tools can misbehave just as badly if not worse. Hence why i never claimed that KVL always works. Circuit meshes are very bad at explaining what fundamentally happens to the electrons, they are good at explaining behavior of circuits in a high level way. This helps in making sense of large circuits containing 100s of components. Much like C++ is nice at explaining large complex programs, but glances over details that one has to deal in Assembler.

If you think the methods i have used to circuit model Dr. Lewins experimental circuit are flawed you are welcome to suggest a circuit that would create a incorrectly behaving circuit mesh when modeled with the same methods. Provided its easy to experimentally recreate i can built it later and compare the test results to the calculations.

I'm not trying to say that Mawell or Faraday, or the definition of voltage is wrong. I was just trying to show a way of applying KVL in a way that works without adding any magical voltages to make it work for just this special case, the same modeling methods can be applied to other circuits just as well.

I suppose if you want a physical representation of field lines in 3D you could imagine each field line being a long stringy bar magnet. All of those long bar magnets have the same magnetic flux and they contain all of this flux inside of them. So if you ware to recreate the field in a loop of wire using these stringy bar magnets as the current is ramping up you would find more and more of these magnets appearing at the edge of the loop and moving towards the center. If we now look at the 2D cross section of the area inside the loop it would look like a area of uniformly distributed dots that is "compressing together" around the center point, putting more and more of them into view as they get closer together. This is sort of analogues to a magnetic field moving inwards so a conductor going around this center point feels the same effect as if it was moving trough a magnetic field. This lets you think about it analogous to the explanation of a comutated DC motor/generator, we also have an equation for calculating the size of this voltage its Uemf=v*B*l .Similarly the force generated by a current carrying wire in a magnetic field can be calculated, putting both together gives you a connection between mechanical power and electrical power in the wire. None of this requires a defined loop area, just a section of wire. Look it up yourself instead of just reading my words.

Oh and by the way a ultrasonic blaster just makes them bark more. The trick is to blast it for a few seconds right after they bark so that they learn what awaits them the next time they do so.
Title: Re: Does Kirchhoff's Law Hold? Disagreeing with a Master
Post by: rfeecs on January 25, 2019, 11:08:36 pm
I suppose if you want a physical representation of field lines in 3D you could imagine each field line being a long stringy bar magnet. All of those long bar magnets have the same magnetic flux and they contain all of this flux inside of them. So if you ware to recreate the field in a loop of wire using these stringy bar magnets as the current is ramping up you would find more and more of these magnets appearing at the edge of the loop and moving towards the center. If we now look at the 2D cross section of the area inside the loop it would look like a area of uniformly distributed dots that is "compressing together" around the center point, putting more and more of them into view as they get closer together. This is sort of analogues to a magnetic field moving inwards so a conductor going around this center point feels the same effect as if it was moving trough a magnetic field. This lets you think about it analogous to the explanation of a comutated DC motor/generator, we also have an equation for calculating the size of this voltage its Uemf=v*B*l .Similarly the force generated by a current carrying wire in a magnetic field can be calculated, putting both together gives you a connection between mechanical power and electrical power in the wire. None of this requires a defined loop area, just a section of wire. Look it up yourself instead of just reading my words.

This is where thinking about "field lines" causes a problem.  In this case of a changing magnetic flux, nothing is moving.  The geometry of the magnetic field does not move.  By that I mean that at every point, the direction of the magnetic field vector doesn't change.  Only the amplitude changes.

The magnetic field is not creating a force to move the charges in the wire.  That force would be perpendicular to the wire, so it would have no effect on the current.

Faraday's law says that a magnetic field that is changing with time creates an electric field that rotates around the magnetic flux.  It is the electric field, not the magnetic field that moves the charge and creates the current.

This has nothing to do with special relativity.  It is just Faraday's law.
Title: Re: Does Kirchhoff's Law Hold? Disagreeing with a Master
Post by: rfeecs on January 25, 2019, 11:27:13 pm
(EDIT: come to think of it again, it seems that all is required is that the primary coil be circular as that seems the only way to get a uniformly distributed magnetic field inside. Someone has proof of that?)

If the solenoid is infinite or very long such that the field lines are parallel and the field outside can be neglected, then the field inside will be uniform.  This can be proven by Ampere's law.  Choose a rectangular path parallel and perpendicular to the field and surrounding some turns of the solenoid.  Vary the position of the side of the path that is inside the solenoid.  Then since the current surrounded by the path is always the same, the internal field must also always be the same, no matter what position.  The solenoid doesn't have to be circular.  It should work for any shape as long as the other conditions are met.
Title: Re: Does Kirchhoff's Law Hold? Disagreeing with a Master
Post by: Sredni on January 26, 2019, 02:45:19 am
(EDIT: come to think of it again, it seems that all is required is that the primary coil be circular as that seems the only way to get a uniformly distributed magnetic field inside. Someone has proof of that?)

If the solenoid is infinite or very long such that the field lines are parallel and the field outside can be neglected, then the field inside will be uniform.  This can be proven by Ampere's law.  Choose a rectangular path parallel and perpendicular to the field and surrounding some turns of the solenoid.  Vary the position of the side of the path that is inside the solenoid.  Then since the current surrounded by the path is always the same, the internal field must also always be the same, no matter what position.

Yep, that's what I was looking for. Of course it was in Ramo Whinnery VanDuzer as well.

Quote
The solenoid doesn't have to be circular.  It should work for any shape as long as the other conditions are met.

So it seems. And yet there has to be some difference in the induced electric field that is generated when the field varies, but I get into nasty integrals as soon as I try to change shape of the coil.
The reason I'm trying to find the induced E field expression for shapes other than a circle is that I believe that the area partitioning Berni proposes can cease to be useful when there is not a 'center' for the induced E field lines and even if there is one (because even if the field follows the profile of the coil close to the coil, it will smooth out farther away), the lines from such a center to the extremes of the segment we want to compute the emf contribute are not perpendicular to the field.

The circular E-field is particular in that it is perpendicular to the radial segments joining the center to the extremes of the open curve we want to find the emf contribute.
But I need to sleep on this. I'm just adding a draft of what I mean here

(https://www.eevblog.com/forum/chat/does-kirchhoffs-law-hold-disagreeing-with-a-master/?action=dlattach;attach=634651)

In the first part the values 12, 4, 4 12 are the line integrals computed along the square of side 4 immersed in the circular induced field whose magnitude grows with the distance from the center C. These were actually computed values since I had the expression for the field. So the emf of the whole square is 36 units. Sure enough, the ratio of the areas (A1/Atot) is the same as the ratio of the line integrals (on AB / on full perimeter).

In the second part I arbitrarily added two values to the radial paths, without changing the other values. This is not legit, but to push my hypothesis: if and I mean IF there is a net nonzero contribute to the emf on this radial paths, the ratio of the areas might not mirror the ratio of the path integrals.
I need to find the analytical expression of the E field for a non-circular generating coil to test it. It might as well be that, when the field is uniform, the net contribute of the path integrals along the two radial paths be zero.

Time to sleep.

Title: Re: Does Kirchhoff's Law Hold? Disagreeing with a Master
Post by: Berni on January 26, 2019, 06:13:25 pm

This is where thinking about "field lines" causes a problem.  In this case of a changing magnetic flux, nothing is moving.  The geometry of the magnetic field does not move.  By that I mean that at every point, the direction of the magnetic field vector doesn't change.  Only the amplitude changes.

The magnetic field is not creating a force to move the charges in the wire.  That force would be perpendicular to the wire, so it would have no effect on the current.

Faraday's law says that a magnetic field that is changing with time creates an electric field that rotates around the magnetic flux.  It is the electric field, not the magnetic field that moves the charge and creates the current.

This has nothing to do with special relativity.  It is just Faraday's law.

I was not trying to say that the field actually moves that way, but it just looks like its moving in such a way from the perspective of the wire in it.

As for the force on the wire that only happens when a current is flowing trough it where the force comes from the usual two magnets interacting with each other. My point was that there are formula for working with sections of wire rather than just loops. Tho usually they will still be in some way connected to Faradays law as in the wire is enclosing a different amount of field.
Title: Re: Does Kirchhoff's Law Hold? Disagreeing with a Master
Post by: Berni on January 26, 2019, 07:29:52 pm
The question of is the field inside a square coil still uniform got me thinking quite a bit too and i couldn't come up with a reasonable answer.

The math to do it does indeed get quite complicated so i decided to cheat my way around it a bit by creating a C# app that calculates it. It sidesteps the integrals by applying the vector math to every point and then just adding everything together. It mostly makes use of vector curl combined with the formula for finding the field around a wire that is based on Ampere's law. This then gets slapped onto every point in the array of vectors to make the magic happen. However this is NOT a proper EM simulation by far. It just assumes that the input is steadily ramping so it doesn't need to have timesteps and it works somewhere between 2D and 3D. It works on a 2D array of 3D vectors (Mostly because my bad unomptimised code already takes a few seconds to calculate this)

The Cyan arrows are the E field forced forced upon the world. This is essentially the path of a wire connected to a power source.
The Red/Green shading indicates the magnetic flux in the Z direction (This is generated by the Cyan E field)
The White arrows are the E field that is generated by the magnetic flux (This is actually the part that takes by far the longest to compute in my app)

(https://www.eevblog.com/forum/chat/does-kirchhoffs-law-hold-disagreeing-with-a-master/?action=dlattach;attach=635248;image)

Title: Re: Does Kirchhoff's Law Hold? Disagreeing with a Master
Post by: Berni on January 26, 2019, 07:55:27 pm
So here are the actual interesting results with complete closed loops.

As expected all of the field suddenly happens on the inside while the circulating E field spreads out to infinity. But a bit more surprisingly the inside of rectangular shapes indeed appear completely uniform, even with a long aspect ratio rectangle. The point where the EMF circulates around also ends up in the middle of the shape.

I have also included a square with a open side to show that this is indeed computed and not just a hand made diagram. Some similarities can be seen to the wire screenshots from above.

(https://www.eevblog.com/forum/chat/does-kirchhoffs-law-hold-disagreeing-with-a-master/?action=dlattach;attach=635254;image)



For curiosity i have also built a shielded box in it. As you can see it is perfectly functional too as there is no field visible outside while the field inside behaves as normal. Tho the way the box is created in the simulation is a bit of a cheat. There is no support for varying magnetic permeability (And it probably needs to be 3D to make sense) so instead the box is actually another coil with a current set to create the same size of flux in order cancel out the inner coil. This could be analogus to a superconducting box where this current would appear in the form of eddy currents.

(https://www.eevblog.com/forum/chat/does-kirchhoffs-law-hold-disagreeing-with-a-master/?action=dlattach;attach=635260;image)
Title: Re: Does Kirchhoff's Law Hold? Disagreeing with a Master
Post by: rfeecs on January 26, 2019, 10:48:00 pm
The Cyan arrows are the E field forced forced upon the world. This is essentially the path of a wire connected to a power source.

That's a little confusing.  I guess it would be equivalent to a displacement current, or a real current?  I'm not sure how the wire segments or open ended solenoid could exist in the real world, though.  I'm assuming "somewhere between 2D and 3D" means this is showing a cross section of an infinite/long solenoid.

The results look reasonable.  Very amazing you can create this so quickly with C#, graphics and all.  Great.
Title: Re: Does Kirchhoff's Law Hold? Disagreeing with a Master
Post by: Sredni on January 27, 2019, 12:03:58 am
@Berni

Can you create a big trapezoidal (the more asymmetric the better) primary coil and then compute the line integrals on the sides of a square coil closed path placed off-center (where 'center' is where the E field appears to go around), like the square I posted?
In my case, the side was 4 units, and the center C was placed at (3,1) (when the square center was at (2,2)).

Alternatively, instead of color-coding amplitudes, draw arrows with lenght proportional to the amplitude (but this might lead to too long and too short arrows...). If you can create a big enough picture, with enough resolution and linear scale for the arrows, we can compute the path integrals by adding the arrows' projections.
Title: Re: Does Kirchhoff's Law Hold? Disagreeing with a Master
Post by: Berni on January 27, 2019, 12:17:08 am
Yeah there is no such thing as "current" in the E field, its just representing the voltage placed on the coil. If you look closely you can see the white arrows always point opposite of the cyan arrows, this is the magnetic field trying to resist a current being put trough it by generating EMF in the opposite direction.

The part about it being sort of 2.5D is that it acts much like the diagrams we have been drawing in this thread where everything happens in a flat plane. For a physical analogy you can imagine this flat plane sandwiched between two infinite permeability plates. So the fields that come up "out of the page" simply get eaten up by the plate and returned back around infinitely far. This is the reason why you don't see the returning field come back in around a wire loop, it doesn't have enough room to bend back around the outside like it does with a real like solenoid in 3D space. To see that happen in here i likely need to have a 3D array of vectors, but that would likely turn the 3 second computation time into a 15 to 60 minutes. Real EM simulators likely have a whole bunch of math tricks and optimizations to make it calculate faster but my app is just quickly thrown together (I dont even use multiple CPU cores).

This thing did still take a good few hours to program, tho most of the time was spent sorting out the math to work correctly. Dealing with all these 3D vectors, angles, rotations etc... can easily go wrong when two things are the wrong way around or a - mistakenly becomes a +,


@Berni

Can you create a trapezoidal (the more asymmetric the better) primary coil and then compute the line integrals on the sides of a square coil placed off-center (where 'center' is where the E field appears to go around), like the square I posted?

Alternatively, instead of color-coding amplitudes, draw arrows with lenght proportional to the amplitude (but this might lead to too long and too short arrows...).


I have been experimenting with weird T or C shaped coils to try to break it and it was still surprisingly uniform. Il make some more screenshots tomorow

I do have a feature for scaling the arrow length, but during testing i found that making stronger arrows be brighter was easier to see (Short arrows loose ability to show direction as they become a point, or long arrows get excessively long). I don't have a feature for taking an integral around a path yet, but i could see that being very useful in simulating a secondary coil.

Tho a bit of a problem is that the EMF generated by the magnetic field appears to be 6 times larger than the E field that original caused the magnetic field. From my understanding the two should be almost the same. My guess is that i perhaps forgot a 2*PI somewhere or that my cheat of using 2.5D rather than full on 3D is the cause of some extra troubles. But i can fix this by simply overlaying another secondary coil over the primary and measure the ratio instead. Apart from that scaling problem all the rest does appear to act as it should according to Maxwell.
Title: Re: Does Kirchhoff's Law Hold? Disagreeing with a Master
Post by: Sredni on January 27, 2019, 12:27:03 am
If by uniform you mean the magnetic field, it has to be. Rfeecs has posted the demonstration it has to be - as long as there is nothing outside the coil.

What I am interested in is the shape of the curly E field.
Title: Re: Does Kirchhoff's Law Hold? Disagreeing with a Master
Post by: Berni on January 27, 2019, 09:01:37 am
Here are some more shapes.

To visualize the strength of the E field better i just put the other field into the heatmap. Its always green because vector magnitude can never go negative.

Some interesting things do happen. The trapezoid shape appears to have some returning field on the outside of the loop, not sure why but my guess is that not having proper 3D that is at fault here.

The dark spot in the middle of the E field seems to sit at the magnetic fields origin point as expected. But it does become a bit more complicated once this origin is located outside of the loop as it is in the C shape. You can still see the dark spot in there, but there are additional dark spots on the edges of the shape. Also a slight non uniformity in the field can be spotted on the inside corners of the C shape, but that's probably just a math hiccup due to too small step size.

(https://www.eevblog.com/forum/chat/does-kirchhoffs-law-hold-disagreeing-with-a-master/?action=dlattach;attach=635713;image)

(https://www.eevblog.com/forum/chat/does-kirchhoffs-law-hold-disagreeing-with-a-master/?action=dlattach;attach=635719;image)

(https://www.eevblog.com/forum/chat/does-kirchhoffs-law-hold-disagreeing-with-a-master/?action=dlattach;attach=635725;image)

EDIT: Fixed spelling
Title: Re: Does Kirchhoff's Law Hold? Disagreeing with a Master
Post by: Berni on January 27, 2019, 09:02:13 am
Attachments didn't all fit in one post
Title: Re: Does Kirchhoff's Law Hold? Disagreeing with a Master
Post by: Sredni on January 27, 2019, 01:48:41 pm
Here there was a picture showing why I wanted an asymmetric coil to generate the uniformly distributed magnetic field.

But the principles behind that can be found in any good book about electromagnetism.
Title: Re: Does Kirchhoff's Law Hold? Disagreeing with a Master
Post by: Berni on January 27, 2019, 07:01:46 pm
I have added path integrals to my code and have tried to compute the fields origin point from the B field by using a weighted average but that doesn't seam to work so far.

Il try instead to automate the calculation of such a partitioned rectangle and see what that gives. But i will admit your last drawing does look a bit worrying for my method.

Oh and thanks i have updated my previous post with the typo.
Title: Re: Does Kirchhoff's Law Hold? Disagreeing with a Master
Post by: bsfeechannel on January 28, 2019, 04:26:47 pm
The circuit is simple enough that very little circuit analysis is actually needed on it.

We know the voltage around it is 1V, We know the total loop resistance is 1 KOhm so trough I=U/R=1V / 1KOhm = 1mA.
At this point the voltages and currents across components are known so the circuit is solved.

Thank you for your replies, they are really helping in spotting the problem and will be useful to answer your question about what part of Feynman's lectures is being misunderstood.

Quote
Since you are looking for the textbook definition the voltage between points AB is both 0V and 1V.

Well, clearly Kirchhoff doesn't hold for the most elementary of the circuits when varying magnetic fields are present.

What you might have tried to do (at least I have) was to replace the wire and the field by a battery or some other lumped generator like in the picture below.

(https://www.eevblog.com/forum/chat/does-kirchhoffs-law-hold-disagreeing-with-a-master/?action=dlattach;attach=636847;image)

However, when you do that, obviously you will have to account for the electric field that this component will introduce along path #2. But path#2, we've already seen, has no electric field. So this is not a circuit of exclusively lumped components and no circuit analysis from the point of view of Kirchhoff can be employed.

But all is not lost. The answer to the next question may seem kind of obvious, but I am trying to prevent any hasty conclusions. I had to spend some time meditating about it myself.

I elongated path #2 so that we have a larger area without any varying magnetic field to the right like in the picture below. My question is, please, what is the voltage between points A' and B' via path #3?

(https://www.eevblog.com/forum/chat/does-kirchhoffs-law-hold-disagreeing-with-a-master/?action=dlattach;attach=636853;image)
Title: Re: Does Kirchhoff's Law Hold? Disagreeing with a Master
Post by: Berni on January 28, 2019, 07:29:19 pm
Did get around to implementing the thing now and played with it some and indeed that is the case.

(https://www.eevblog.com/forum/chat/does-kirchhoffs-law-hold-disagreeing-with-a-master/?action=dlattach;attach=636967;image)

It appears for this method to work reliably it does require the B field to one of the flowing:
-Infinite uniform field everywhere
-Confined to a radially symmetric shape
-Confined to a much smaller shape than the path (So that you observe the smoother far field)

I don't quickly see a way to make it work reliably in other cases. I get the feeling that EM simulation is actually required in these more complex cases. But if nothing else this integral feature did help me verify more of Maxwells behavior in my thrown together simulation, moving and resizing the yellow loop around does act exactly as Faradays law says it should. So i am reasonably confident that the E field it spits out is correct.

So yeah my sectioning method does appear to require special case conditions to work. In extreme cases of very long and skinny rectangles (Like 20:1 aspect ratio) it can get the short side wrong by 50%, but the total sum around is always correct (Since the origin gets canceled out). So i do have to admit i was a little bit wrong here, but hey now i can just stick those special cases into my shitty "EM Simulator" (If it even deserves to be called that) and have it calculate the lumpy distribution of EMF for me.
Title: Re: Does Kirchhoff's Law Hold? Disagreeing with a Master
Post by: Berni on January 28, 2019, 07:56:31 pm

Well, clearly Kirchhoff doesn't hold for the most elementary of the circuits when varying magnetic fields are present.

What you might have tried to do (at least I have) was to replace the wire and the field by a battery or some other lumped generator like in the picture below.

(https://www.eevblog.com/forum/chat/does-kirchhoffs-law-hold-disagreeing-with-a-master/?action=dlattach;attach=636847;image)

However, when you do that, obviously you will have to account for the electric field that this component will introduce along path #2. But path#2, we've already seen, has no electric field. So this is not a circuit of exclusively lumped components and no circuit analysis from the point of view of Kirchhoff can be employed.

Yes the battery is how the "Upgraded KVL" as you call it would deal with the EMF. You get a circuit that acts pretty much the same in the textbook voltage definition. Same current flows trough the circuit. Going from A to B down the bar gets you 1V and going from A to B around the loop you get 1V too (Remember voltage sources appear negative due to the opposite E field inside). Since we turned the field into a lumped battery means the circuit is no longer exposed to a field and so the sum around the loop should be 0V and if you go around you indeed get 0V.

If you do it in the form of inductors or magical batteries it doesn't matter. As long as the circuit mesh is somehow informed about the effects of the B field.



But all is not lost. The answer to the next question may seem kind of obvious, but I am trying to prevent any hasty conclusions. I had to spend some time meditating about it myself.

I elongated path #2 so that we have a larger area without any varying magnetic field to the right like in the picture below. My question is, please, what is the voltage between points A' and B' via path #3?

(https://www.eevblog.com/forum/chat/does-kirchhoffs-law-hold-disagreeing-with-a-master/?action=dlattach;attach=636853;image)

The voltage along that path is also 1V because i assume there is no wire there.
Title: Re: Does Kirchhoff's Law Hold? Disagreeing with a Master
Post by: bsfeechannel on January 28, 2019, 10:45:54 pm
The voltage along that path is also 1V because i assume there is no wire there.

That's what I thought. For your convenience I decided to tabulate below all the voltages in this circuit.

NodeNodePathVoltage (V)Reason
A'A#20Ideal wire
AB#11EMF across resistor
BB'#20Ideal wire
B'A'#3-1EMF across the air
A'B'#31EMF across the air
B'A'#20Ideal wire

Now let's add a 1V battery between nodes A' and B' like in the picture below. Will the voltages in the table above remain the same or will they change?

(https://www.eevblog.com/forum/chat/does-kirchhoffs-law-hold-disagreeing-with-a-master/?action=dlattach;attach=637069;image)
Title: Re: Does Kirchhoff's Law Hold? Disagreeing with a Master
Post by: Berni on January 29, 2019, 06:05:55 am
Still the same voltages.
Title: Re: Does Kirchhoff's Law Hold? Disagreeing with a Master
Post by: bsfeechannel on January 29, 2019, 11:12:41 pm
Precisely.

If you look at the table again, now considering the battery in the circuit, you are going to notice that nodes A', A, B, and B' form a mesh through path #1, two stretches of path#2 and path #3, where all the voltages add up to zero. A' and B' form a mesh through path #3 and the portion of path #2 that encloses the varying magnetic field, where the voltages do not add up to zero.

NodeNodePathVoltage (V)Reason
A'A#20Ideal wire
AB#11EMF across resistor
BB'#20Ideal wire
B'A'#3-1EMF across the air || battery
Total0No varying magnetic field in the mesh
A'B'#31EMF across the air || battery
B'A'#20Ideal wire
Total1Presence of varying magnetic field in the mesh

This would be enough to hint that Kirchhoff only holds when varying magnetic fields are not present in a mesh. But let's not be diverted by that "coincidence", because our interest is elsewhere.

 Let's turn our attention to the fact that our first attempt at employing circuit analysis failed because, when we introduced a battery in path #2, (although nothing changed for the resistor) we changed the electric field along that path. Now that we have introduced the battery in a path with a voltage that is exactly equal to the EMF of that path, nothing has changed. If we now disconnect the mesh on the left -- the one with the varying magnetic field -- the mesh on the right would see no difference in terms of voltage.

(https://www.eevblog.com/forum/chat/does-kirchhoffs-law-hold-disagreeing-with-a-master/?action=dlattach;attach=637891;image)

NodeNodePathVoltage (V)Reason
A'A#20Ideal wire
AB#11EMF across resistor
BB'#20Ideal wire
B'A'#3-1Battery
Total0No varying magnetic field in the mesh

And if we analyze the resulting closed path formed  by the portion of path#2 to the left and a straight line from A'' and B'' (not pictured), the voltages still remain as before.

NodeNodePathVoltage (V)Reason
A''B''#41EMF across the air
B''A''#20Ideal wire
Total1Presence of varying magnetic field in the closed path

So it seems that this last resulting circuit better models the original circuit, because, although it doesn't represent what is actually going on along path #2 -- in fact it hides it -- at least it doesn't lie about that. It doesn't introduce a field where a field should not be.

(https://www.eevblog.com/forum/chat/does-kirchhoffs-law-hold-disagreeing-with-a-master/?action=dlattach;attach=637897;image)
(https://www.eevblog.com/forum/chat/does-kirchhoffs-law-hold-disagreeing-with-a-master/?action=dlattach;attach=637903;image)
Title: Re: Does Kirchhoff's Law Hold? Disagreeing with a Master
Post by: bsfeechannel on January 30, 2019, 06:09:58 am
Now comes hopefully our last question. Given the circuit below, with our familiar ideal wire, varying magnetic field and ideal resistor, supposing that the field occupies the closed loop entirely, without any area that be free from it, and supposing we would like to replace the 1V EMF with a battery that will be located exactly where it occurs in the circuit, where would you place this battery? In other words, what path would you choose to connect this battery?

(https://www.eevblog.com/forum/chat/does-kirchhoffs-law-hold-disagreeing-with-a-master/?action=dlattach;attach=638308;image)
Title: Re: Does Kirchhoff's Law Hold? Disagreeing with a Master
Post by: Berni on January 30, 2019, 05:10:59 pm
Here it is then.

(https://www.eevblog.com/forum/chat/does-kirchhoffs-law-hold-disagreeing-with-a-master/?action=dlattach;attach=638590;image)

Since the field ends at the resistive bar that's where i put the battery.

But given that the behavior of the circuit doesn't change if its placed anywhere else its not that critical. The placement locations of components in circuit mesh schematics has no effect, its just that usually we try to match the real life placement for easier understanding of what components are what in the diagram.
Title: Re: Does Kirchhoff's Law Hold? Disagreeing with a Master
Post by: bsfeechannel on January 30, 2019, 08:22:33 pm
Thank you for all your replies.

We can see pretty much that this is theoretically and physically impossible, because a battery and a resistor can't occupy the same space at the same time.

So, not only this circuit more than violates--it rapes--Kirchhoff big time, as we have seen, but also cannot have an equivalent version with lumped components.

Now I owe you an answer to your question, what part of Feynman's lectures, namely Chapter 22 is being misunderstood? The short answer is all of it. It is so because people are disregarding basic assumptions that Feynman adamantly stresses in his text.

An answer a little less short is given by Prof. Belcher in his "MIT-quality report" where he elegantly showed where exactly Mehdi, and for that matter all those who still believe that KVL can have the slightest chance to hold under a varying magnetic field, goofed it up. However, after the report, Mehdi continued to espouse his previous ideas, which means that he didn't in fact learn anything. Perhaps, noticing this, even before Mehdi made his second video, Prof. Belcher said in his report:
Quote
I am grateful to Mr. Sadaghdar for a number of discussions about Faraday’s Law and KVL, which have improved my understanding of both.
I.e., my understanding improved.

Belcher concludes:
Quote
Many introductory texts on electromagnetism are not precise about what exactly they mean by the voltage drop across the inductor, and many students come to incorrect conclusions about what this actually means. The most common misconception is that the  - LdI/dt voltage read by the voltmeter just above represents a −∫abE⋅dl through the inductor. But if the inductor wires are perfectly conducting, this integral is zero because there is no electric field in the wires.

So, replacing perfectly conducting wires with batteries, or generators, is a noob mistake. It's a trap for young players. This means that the circuit below is not modelling Lewin's circuit.

(https://www.eevblog.com/forum/chat/does-kirchhoffs-law-hold-disagreeing-with-a-master/?action=dlattach;attach=638728;image)

In fact, Lewin's circuit is not lumpable, because we do not have anywhere inside the loop where we don't have varying magnetic fields, where we could replace the EMF with a battery and get away with it. The voltages that you can measure at the terminals of the resistors of the internal loop are the result of electric fields that are being generated along the very same path where the resistors are.

The failure to understand this basic principle of electromagnetism leads to all kinds of wrong conclusions.
Title: Re: Does Kirchhoff's Law Hold? Disagreeing with a Master
Post by: Berni on January 30, 2019, 09:03:46 pm
Thank you for all your replies.

We can see pretty much that this is theoretically and physically impossible, because a battery and a resistor can't occupy the same space at the same time.

So, not only this circuit more than violates--it rapes--Kirchhoff big time, as we have seen, but also cannot have an equivalent version with lumped components.

Now I owe you an answer to your question, what part of Feynman's lectures, namely Chapter 22 is being misunderstood? The short answer is all of it. It is so because people are disregarding basic assumptions that Feynman adamantly stresses in his text.

An answer a little less short is given by Prof. Belcher in his "MIT-quality report" where he elegantly showed where exactly Mehdi, and for that matter all those who still believe that KVL can have the slightest chance to hold under a varying magnetic field, goofed it up. However, after the report, Mehdi continued to espouse his previous ideas, which means that he didn't in fact learn anything. Perhaps, noticing this, even before Mehdi made his second video, Prof. Belcher said in his report:
Quote
I am grateful to Mr. Sadaghdar for a number of discussions about Faraday’s Law and KVL, which have improved my understanding of both.
I.e., my understanding improved.

I agree.

Again i never said in this whole thread that KVL always holds in real circuits. What made you think i did?

Well theoretically you can have two components occupy one space just fine, just like you can have ideal wires with zero inductance, or resistors with negative resistance values. In fact a real coil is an inductor, resistor and capacitor in one physical piece of material, just that the inductance part tends to be vastly larger than the other two. But everything theoretical is not necessarily realizable physically.

Belcher concludes:
Quote
Many introductory texts on electromagnetism are not precise about what exactly they mean by the voltage drop across the inductor, and many students come to incorrect conclusions about what this actually means. The most common misconception is that the  - LdI/dt voltage read by the voltmeter just above represents a −∫abE⋅dl through the inductor. But if the inductor wires are perfectly conducting, this integral is zero because there is no electric field in the wires.

So, replacing perfectly conducting wires with batteries, or generators, is a noob mistake. It's a trap for young players. This means that the circuit below is not modelling Lewin's circuit.

(https://www.eevblog.com/forum/chat/does-kirchhoffs-law-hold-disagreeing-with-a-master/?action=dlattach;attach=638728;image)

In fact, Lewin's circuit is not lumpable, because we do not have anywhere inside the loop where we don't have varying magnetic fields, where we could replace the EMF with a battery and get away with it. The voltages that you can measure at the terminals of the resistors of the internal loop are the result of electric fields that are being generated along the very same path where the resistors are.

The failure to understand this basic principle of electromagnetism leads to all kinds of wrong conclusions.

Well in terms of being unlumpable i still don't see how my lumped transformer model is wrong here:
https://www.eevblog.com/forum/chat/does-kirchhoffs-law-hold-disagreeing-with-a-master/msg2138140/#msg2138140 (https://www.eevblog.com/forum/chat/does-kirchhoffs-law-hold-disagreeing-with-a-master/msg2138140/#msg2138140)

It does fill a lot of criteria:
- It contains all fields inside the lumped transformer
- All other elements are not enclosed in a magnetic field so circuit analysis methods should work on them.
- It behaves exactly like the real experimental circuit, so in terms of functionality it is an accurate mesh model
- All parts of the mesh model belong to some physical part of the real circuit, no magical added components just to make it work.

This does NOT magically prove anything about KVL working in real circuits with magnetic fields in them. My claim is that this particular circuit can be reasonably lumped and mesh modeled to sufficient accuracy that circuit analysis predicts its behavior within a reasonable margin of error.

No need to tell me that a mesh model is not exactly the same as a real circuit. I know its not! No mesh model is, because we can't create these ideal circuit components in real life, but when modeling is done right it acts just like the real thing despite being simplified.
Title: Re: Does Kirchhoff's Law Hold? Disagreeing with a Master
Post by: Sredni on January 30, 2019, 10:17:44 pm
- All other elements are not enclosed in a magnetic field so circuit analysis methods should work on them.

When applied to the Romer-Lewin circuit, this is wrong and false.

The Romer-Lewin circuit encloses a net nonzero variable magnetic field.
Your lumped circuit is a different circuit for the very reason that by forcing the field to be confined inside the four arcs, you create a field-free zone inside the circuit connecting them.

As I said before, until you realize that, there is nothing I can do to help.

EDIT: heres a little help

(https://www.eevblog.com/forum/chat/does-kirchhoffs-law-hold-disagreeing-with-a-master/?action=dlattach;attach=650970)
Title: Re: Does Kirchhoff's Law Hold? Disagreeing with a Master
Post by: bsfeechannel on January 30, 2019, 11:07:10 pm
Well in terms of being unlumpable i still don't see how my lumped transformer model is wrong here:
https://www.eevblog.com/forum/chat/does-kirchhoffs-law-hold-disagreeing-with-a-master/msg2138140/#msg2138140 (https://www.eevblog.com/forum/chat/does-kirchhoffs-law-hold-disagreeing-with-a-master/msg2138140/#msg2138140)
[snip]
No need to tell me that a mesh model is not exactly the same as a real circuit. I know its not! No mesh model is, because we can't create these ideal circuit components in real life, but when modeling is done right it acts just like the real thing despite being simplified.

Let me help you.

Your model says you will find 250mV across all wires. Theory says it is zero. You measure, it is zero. Nature agrees with theory. What do you think is wrong? Nature? Theory? Perhaps both? Or your model?
Title: Re: Does Kirchhoff's Law Hold? Disagreeing with a Master
Post by: Berni on January 31, 2019, 06:10:35 am
I said in my last post that i know its not the exact same circuit.

But the real life circuit that the mesh model represents is very similar and behaves exactly the same as Dr. Lewins circuit.

And yes my model says that you will measure 250mV across the wires, but only if you use probes traveling outside the field to connect your voltmeter! If the voltmeter is hooked up with wires that go trough the same field inside the transformer you get 0V. So how is that 0V different?

Title: Re: Does Kirchhoff's Law Hold? Disagreeing with a Master
Post by: bsfeechannel on January 31, 2019, 10:14:03 am
I said in my last post that i know its not the exact same circuit.

So end of story. Theory, practice, and now your brain are saying that your circuit is not a model of Lewin's circuit. Listen to them.

Title: Re: Does Kirchhoff's Law Hold? Disagreeing with a Master
Post by: Berni on January 31, 2019, 05:39:55 pm
So then all cirucit mesh models are flawed because they don't exactly replicate every detail of a cirucit such as parasitic capacitance between all conductive materials, parasitic inductance in every piece of wire and models the magnetic field produced even by picoamps of current passing trough wires? And so we should throw away circuit analysis as a whole and stop using it because it never represents the exact circuit?
Title: Re: Does Kirchhoff's Law Hold? Disagreeing with a Master
Post by: Berni on February 01, 2019, 07:00:11 am
But where do you draw the line.

Real transformers also leak fields, especially if run close to saturation in a badly designed E core. So anything around them is now un-lumpable?

Or maybe the design has one of these non shielded inductors: https://si.farnell.com/coilcraft/pcv-2-564-08l/inductor-560uh-7a-10-power/dp/2457700 (https://si.farnell.com/coilcraft/pcv-2-564-08l/inductor-560uh-7a-10-power/dp/2457700)
These things spew bunch of the returning field out around them, so anything around that part on your board is now un-lumpable?

Or just running two parallel traces on a circuit board creates coupling between them, this is actually how RF couplers are made most of the time. So at what length do the two traces go from being lumpable to un-lumpable?

Or do we need to go to something that really makes strong use of magnetic coupling like for example a resonant loop antenna? Like this: http://webclass.org/k5ijb/antennas/Small-magnetic-loops-FAQ.htm (http://webclass.org/k5ijb/antennas/Small-magnetic-loops-FAQ.htm)


Title: Re: Does Kirchhoff's Law Hold? Disagreeing with a Master
Post by: bsfeechannel on February 01, 2019, 01:12:45 pm
But where do you draw the line.

That's your most significant question up to now.

Feynman's chapter 22 is all about drawing that line. Literally. He even names it Γ (gamma).
Title: Re: Does Kirchhoff's Law Hold? Disagreeing with a Master
Post by: Sredni on February 01, 2019, 02:05:52 pm
But where do you draw the line.

Where most---

EDIT
This post has been shortened and cleansed to avoid upsetting other children.
Whatever was written here can be found in one or more of the following books (in no particular order, and without mentioning the usual suspects Feynman, Purcell, Griffiths, Ohanian, Jackson):

Panofsky, Phillips
Classical Electricity and Magnetism 2nd ed

John Kraus
Electromagnetism 2nd to 4th ed

Ramo, Whinnery, VanDuzer
Fields and Waves in Communication Electronics 2nd or 3rd ed

Bleaney
Electricity and Magnetism 3rd ed

Nayfeh, Brussel
Electricity and Magnetism

Kip
Fundamentals of Electricity and Magnetism 2nd ed

Lorrain, Courson
Electromagnetic Fields and Waves 2nd ed
Title: Re: Does Kirchhoff's Law Hold? Disagreeing with a Master
Post by: ogden on February 01, 2019, 05:55:19 pm
Your model says you will find 250mV across all wires. Theory says it is zero.

Which exactly theory says it is zero?

Are you saying that for EMF voltage to appear - full winding is necessary? In your understanding there is no 1/4 * EMF voltage on the 1/4 winding (tap)?
Title: Re: Does Kirchhoff's Law Hold? Disagreeing with a Master
Post by: bsfeechannel on February 01, 2019, 06:55:40 pm
Your model says you will find 250mV across all wires. Theory says it is zero.

Which exactly theory says it is zero?

Since 19th century electromagnetism is explained by Maxwell's equations.

Quote
Are you saying that for EMF voltage to appear - full winding is necessary? In your understanding there is no 1/4 * EMF voltage on the 1/4 winding (tap)?

Where have you been lately? Have you followed our discussion since Jan 23rd?
Title: Re: Does Kirchhoff's Law Hold? Disagreeing with a Master
Post by: ogden on February 01, 2019, 07:59:00 pm
Are you saying that for EMF voltage to appear - full winding is necessary? In your understanding there is no 1/4 * EMF voltage on the 1/4 winding (tap)?

Where have you been lately? Have you followed our discussion since Jan 23rd?

If I missed your answer to exact same question I just asked - feel free to copy/paste it (again) here.
Title: Re: Does Kirchhoff's Law Hold? Disagreeing with a Master
Post by: bsfeechannel on February 01, 2019, 08:06:23 pm
Read Feynman's book chapter 22, "verses" 1 and 2. The answer to your question is there. I can't do better.
Title: Re: Does Kirchhoff's Law Hold? Disagreeing with a Master
Post by: ogden on February 01, 2019, 08:14:25 pm
I can't do better.

I did not ask to do better. Answer using your own words.
Title: Re: Does Kirchhoff's Law Hold? Disagreeing with a Master
Post by: bsfeechannel on February 01, 2019, 09:08:32 pm
I don't know how to answer your question without further investigation. Let's suppose that we have a varying magnetic field so that going from point A and returning to it again via the path indicated by the dashed line, we find an EMF = 1V like in the picture below.

Now consider that we introduce a piece of wire along the same path so that we have 3/4 of a turn, 1/2 a turn and 1/4 of a turn. What would be the voltages VAB, VAC and VDC following their respective dashed lines indicated in the picture?

(https://www.eevblog.com/forum/chat/does-kirchhoffs-law-hold-disagreeing-with-a-master/?action=dlattach;attach=640678;image)
Title: Re: Does Kirchhoff's Law Hold? Disagreeing with a Master
Post by: Berni on February 01, 2019, 11:03:59 pm
Im talking about the field around it due to us all agreeing that circuit analysis (And with that also KVL) works when there is no field.

What 1V jump is there? Its simply voltages measured across the wires inside the magnetic field by taking a different path with the voltmeter. The voltages are there in Dr. Lewins experimental circuit if you probe it just the right way.

The lumped model simply has a different way of expressing the effects of magnetic fields, that's it. The whole circuit still acts identical and that's what matters. Cirucit mesh models are supposed to model the high level behavior of circuits, not model physical electrons moving trough wires and the fields they make around them. As long as the circuit behaves the same its considered a accurate model.
Title: Re: Does Kirchhoff's Law Hold? Disagreeing with a Master
Post by: bsfeechannel on February 02, 2019, 12:23:35 am
What 1V jump is there? Its simply voltages measured across the wires inside the magnetic field by taking a different path with the voltmeter. The voltages are there in Dr. Lewins experimental circuit if you probe it just the right way.

And that's what he did. There is 0.9V across one resistor, and 0.1 across the other. Those are the only voltages present in the circuit. If you're measuring anything else, it's because you are choosing a different path than that of the circuit.

Simple as that.

Quote
The lumped model simply has a different way of expressing the effects of magnetic fields, that's it.

This is impossible since the lumped model is derived from Maxwell. That's explained by Feynman in his chapter 22.

Quote
The whole circuit still acts identical and that's what matters. Cirucit mesh models are supposed to model the high level behavior of circuits, not model physical electrons moving trough wires and the fields they make around them.

When Maxwell published his equations, they didn't know about the existence of electrons. Maxwell is about fields and geometry. Every time your circuit is affected by fields or geometry, you'll have to use them.

Quote
As long as the circuit behaves the same its considered a accurate model.

Unfortunately your "model" doesn't behave the same as Lewin's circuit nor is accurate. In fact it is aberrant. And, if you pardon me, asinine. It proposes the existence of 250mV across a wire that has a resistance of about zero ohms carrying a current of 1mA.

250mV = 0Ω · 1mA !!!!!!!!!!!!!

Besides, your model contradicts Faraday's law that states that any circuit under varying magnetic will have its voltages adding up to a value different from zero.

To help you avoid those gross errors, I prepared a quick guide to lumpiness. I hope that it will be useful for you.

(https://www.eevblog.com/forum/chat/does-kirchhoffs-law-hold-disagreeing-with-a-master/?action=dlattach;attach=640816;image)
Title: Re: Does Kirchhoff's Law Hold? Disagreeing with a Master
Post by: ogden on February 02, 2019, 12:59:15 am
I don't know how to answer your question without further investigation. Let's suppose that we have a varying magnetic field so that going from point A and returning to it again via the path indicated by the dashed line, we find an EMF = 1V like in the picture below.

Now consider that we introduce a piece of wire along the same path so that we have 3/4 of a turn, 1/2 a turn and 1/4 of a turn. What would be the voltages VAB

I was talking about 1/4 (part) of the winding/turn. No need to introduce anything. It was simple question. - Transformer with single winding/turn with 1/4-turn tap. What's voltage on it if full winding gives 1V?

Title: Re: Does Kirchhoff's Law Hold? Disagreeing with a Master
Post by: ogden on February 02, 2019, 01:02:24 am
Unfortunately your "model" doesn't behave the same as Lewin's circuit nor is accurate. In fact it is aberrant. And, if you pardon me, asinine. It proposes the existence of 250mV across a wire that has a resistance of about zero ohms carrying a current of 1mA.

250mV = 0Ω · 1mA !!!!!!!!!!!!!

You say you mastered Maxwell's equations?  :-DD :-DD :-DD :-DD :-DD :-DD
Title: Re: Does Kirchhoff's Law Hold? Disagreeing with a Master
Post by: bsfeechannel on February 02, 2019, 01:25:54 am
I was talking about 1/4 (part) of the winding/turn. No need to introduce anything. It was simple question. - Transformer with single winding/turn with 1/4-turn tap. What's voltage on it if full winding gives 1V?

Can you please provide a schematic of how you get a 1/4 tap from a transformer with a single-turn winding?
Title: Re: Does Kirchhoff's Law Hold? Disagreeing with a Master
Post by: bsfeechannel on February 02, 2019, 01:32:08 am
Unfortunately your "model" doesn't behave the same as Lewin's circuit nor is accurate. In fact it is aberrant. And, if you pardon me, asinine. It proposes the existence of 250mV across a wire that has a resistance of about zero ohms carrying a current of 1mA.

250mV = 0Ω · 1mA !!!!!!!!!!!!!

You say you mastered Maxwell's equations?  :-DD :-DD :-DD :-DD :-DD :-DD

Isn't that hilarious? A static wire with a next to zero ohm internal resistance sporting 250mVDC and 1mA!!!

It doesn't need to be a Maxwell expert to realize how moronic that conclusion is. I agree with you.


 
Title: Re: Does Kirchhoff's Law Hold? Disagreeing with a Master
Post by: ogden on February 02, 2019, 01:32:26 am
I was talking about 1/4 (part) of the winding/turn. No need to introduce anything. It was simple question. - Transformer with single winding/turn with 1/4-turn tap. What's voltage on it if full winding gives 1V?

Can you, please provide a schematic of how you get a 1/4 tap from a transformer with a single-turn winding?

This one is good enough. There are loads of 1/4 windings, with voltmeters attached:

(https://www.eevblog.com/forum/chat/does-kirchhoffs-law-hold-disagreeing-with-a-master/?action=dlattach;attach=630190;image)
Title: Re: Does Kirchhoff's Law Hold? Disagreeing with a Master
Post by: ogden on February 02, 2019, 01:35:43 am
Unfortunately your "model" doesn't behave the same as Lewin's circuit nor is accurate. In fact it is aberrant. And, if you pardon me, asinine. It proposes the existence of 250mV across a wire that has a resistance of about zero ohms carrying a current of 1mA.

250mV = 0Ω · 1mA !!!!!!!!!!!!!

You say you mastered Maxwell's equations?  :-DD :-DD :-DD :-DD :-DD :-DD

Isn't that hilarious? A static wire with a next to zero ohm internal resistance sporting 250mVDC and 1mA!!!

It doesn't need to be a Maxwell expert to realize how moronic that conclusion is. I agree with you.

Right. So you agree that you do not know what EMF actually is.
Title: Re: Does Kirchhoff's Law Hold? Disagreeing with a Master
Post by: bsfeechannel on February 02, 2019, 01:38:41 am
This one is good enough. There are loads of 1/4 windings, with voltmeters attached:

(https://www.eevblog.com/forum/chat/does-kirchhoffs-law-hold-disagreeing-with-a-master/?action=dlattach;attach=630190;image)

Where is the transformer?
Title: Re: Does Kirchhoff's Law Hold? Disagreeing with a Master
Post by: bsfeechannel on February 02, 2019, 01:39:47 am
Right. So you agree that you do not know what EMF actually is.

What is the EMF?
Title: Re: Does Kirchhoff's Law Hold? Disagreeing with a Master
Post by: ogden on February 02, 2019, 01:59:27 am
What is the EMF?

https://en.wikipedia.org/wiki/Electromotive_force (https://en.wikipedia.org/wiki/Electromotive_force)

Where is the transformer?

Dr.Lewin's experiment is transformer. Didn't you notice?  :palm:
Title: Re: Does Kirchhoff's Law Hold? Disagreeing with a Master
Post by: bsfeechannel on February 02, 2019, 02:16:58 am
https://en.wikipedia.org/wiki/Electromotive_force (https://en.wikipedia.org/wiki/Electromotive_force)

Thank you.

Quote
Dr.Lewin's experiment is transformer. Didn't you notice?  :palm:

So any circuit under a varying magnetic field is a transformer?
Title: Re: Does Kirchhoff's Law Hold? Disagreeing with a Master
Post by: ogden on February 02, 2019, 02:20:49 am
Isn't that hilarious? A static wire with a next to zero ohm internal resistance sporting 250mVDC and 1mA!!!

It doesn't need to be a Maxwell expert to realize how moronic that conclusion is. I agree with you.

Take any Li-Ion battery and put it into your reasoning ;)
Title: Re: Does Kirchhoff's Law Hold? Disagreeing with a Master
Post by: ogden on February 02, 2019, 02:25:13 am
Quote
Dr.Lewin's experiment is transformer. Didn't you notice?  :palm:

So any circuit under a varying magnetic field is a transformer?

I did not say any circuit. I said that Dr.Lewin's experiment is transformer. You are clearly avoiding my quite straight and simple question:

Transformer with single winding/turn with 1/4-turn tap. What's voltage on it if full winding gives 1V?
Title: Re: Does Kirchhoff's Law Hold? Disagreeing with a Master
Post by: bsfeechannel on February 02, 2019, 02:30:00 am
Isn't that hilarious? A static wire with a next to zero ohm internal resistance sporting 250mVDC and 1mA!!!

It doesn't need to be a Maxwell expert to realize how moronic that conclusion is. I agree with you.

Take any Li-Ion battery and put it into your reasoning ;)

Got it. I'll replace the batteries of my cell phone with a piece of wire. Why didn't i think of that before?
Title: Re: Does Kirchhoff's Law Hold? Disagreeing with a Master
Post by: bsfeechannel on February 02, 2019, 02:35:19 am
I did not say any circuit. I said that Dr.Lewin's experiment is transformer.

Well, it is a circuit, innit? It has two resistors and wires connecting them so that current can flow.

But since you said it is a transformer, what are the criteria to consider a circuit under a varying magnetic field a transformer?

Quote
You are clearly avoiding my quite straight and simple question:

Transformer with single winding/turn with 1/4-turn tap. What's voltage on it if full winding gives 1V?

Maybe you're asking the wrong questions.
Title: Re: Does Kirchhoff's Law Hold? Disagreeing with a Master
Post by: ogden on February 02, 2019, 02:37:02 am
Isn't that hilarious? A static wire with a next to zero ohm internal resistance sporting 250mVDC and 1mA!!!

It doesn't need to be a Maxwell expert to realize how moronic that conclusion is. I agree with you.

Take any Li-Ion battery and put it into your reasoning ;)

Got it. I'll replace the batteries of my cell phone by a piece of wire. Why didn't i think of that before?

In case you did not know - batteries have low internal resistance. That was my point.
Title: Re: Does Kirchhoff's Law Hold? Disagreeing with a Master
Post by: bsfeechannel on February 02, 2019, 02:40:18 am
Batteries have electric fields inside them.
Title: Re: Does Kirchhoff's Law Hold? Disagreeing with a Master
Post by: ogden on February 02, 2019, 02:42:21 am
But since you said it is a transformer, what are the criteria to consider a circuit under a varying magnetic field a transformer?

Since you do not know what is transformer - why do you even participate in this discussion?

Quote
Maybe you're asking the wrong questions.

You just pretend that you do not understand what I am asking.
Title: Re: Does Kirchhoff's Law Hold? Disagreeing with a Master
Post by: ogden on February 02, 2019, 02:46:18 am
Batteries have electric fields inside them.

So what? How does it changes Ohm's law you mentioned?

Isn't that hilarious? A static wire with a next to zero ohm internal resistance sporting 250mVDC and 1mA!!!
Title: Re: Does Kirchhoff's Law Hold? Disagreeing with a Master
Post by: bsfeechannel on February 02, 2019, 03:05:31 am
Since you do not know what is transformer - why do you even participate in this discussion?

I don't know. Perhaps because you could tell me what a transformer is.

Quote
You just pretend that you do not understand what I am asking.

I'm trying to be polite.
Title: Re: Does Kirchhoff's Law Hold? Disagreeing with a Master
Post by: bsfeechannel on February 02, 2019, 03:08:51 am
So what? How does it changes Ohm's law you mentioned?
Wires are not batteries.
Title: Re: Does Kirchhoff's Law Hold? Disagreeing with a Master
Post by: Sredni on February 02, 2019, 03:48:35 am
Note: I should be sleeping but...

Im talking about the field around it due to us all agreeing that circuit analysis (And with that also KVL) works when there is no field.

EDIT
This post has been shortened and cleansed to avoid upsetting other children.
Whatever was written here can be found in one or more of the following books (in no particular order, and without mentioning the usual suspects Feynman, Purcell, Griffiths, Ohanian, Jackson):

Nayfeh, Brussel
Electricity and Magnetism

Kip
Fundamentals of Electricity and Magnetism 2nd ed

Lorrain, Courson
Electromagnetic Fields and Waves 2nd ed

Panofsky, Phillips
Classical Electricity and Magnetism 2nd ed

John Kraus
Electromagnetism 2nd to 4th ed

Ramo, Whinnery, VanDuzer
Fields and Waves in Communication Electronics 2nd or 3rd ed

Bleaney
Electricity and Magnetism 3rd ed


Title: Re: Does Kirchhoff's Law Hold? Disagreeing with a Master
Post by: Sredni on February 02, 2019, 04:00:47 am
:-DD :-DD :-DD :-DD :-DD :-DD

It's Groundhog Day.
Again!

<sigh>

EDIT

This post has been shortened and cleansed to avoid upsetting other children.
Whatever was written here can be found in one or more of the following books (in no particular order, and without mentioning the usual suspects Feynman, Purcell, Griffiths, Ohanian, Jackson):

Panofsky, Phillips
Classical Electricity and Magnetism 2nd ed

John Kraus
Electromagnetism 2nd to 4th ed

Ramo, Whinnery, VanDuzer
Fields and Waves in Communication Electronics 2nd or 3rd ed

Bleaney
Electricity and Magnetism 3rd ed

Nayfeh, Brussel
Electricity and Magnetism

Kip
Fundamentals of Electricity and Magnetism 2nd ed

Lorrain, Courson
Electromagnetic Fields and Waves 2nd ed

"Books" are static paper based documents that can be found in libraries. They are like smartphones, but (usually) bigger, with lots and lots of extremely thin flexible e-ink screens and a very long battery life. Libraries are...
Oh, never mind. Keep on pushing that square peg into that round hole. With a big enough hammer, it will fit.
Title: Re: Does Kirchhoff's Law Hold? Disagreeing with a Master
Post by: bsfeechannel on February 02, 2019, 05:35:08 am
:-DD :-DD :-DD :-DD :-DD :-DD

It's Groundhog Day.
Again!


Now I understand when Lewin said that these guys are flat-earthers. People advocating pseudo-scientific claims in electronics is the end of our profession.
Title: Re: Does Kirchhoff's Law Hold? Disagreeing with a Master
Post by: rfeecs on February 02, 2019, 06:06:19 am
OK, probably way off topic, but I guess I'm losing my mind.  I watched this video.  What the hell?  Could something on the internet not be right?  Or am I confused?  How could this guy get the fields so wrong?  He doesn't even agree with Wikipedia.  So I can't believe everything on the internet? :
(https://upload.wikimedia.org/wikipedia/commons/thumb/8/82/Poynting_vectors_of_DC_circuit.svg/660px-Poynting_vectors_of_DC_circuit.svg.png)
Yikes.  "We have a strong electric field inside the wire":

https://youtu.be/C7tQJ42nGno (https://youtu.be/C7tQJ42nGno)

Comments are overwelmingly positive "mind blown"!
Title: Re: Does Kirchhoff's Law Hold? Disagreeing with a Master
Post by: Berni on February 02, 2019, 10:32:33 am
Yes there are infinitely many paths trough air to close a 1/4 turn around a transformer. But for circuit analysis its most convenient to define that path as being a path with zero EMF, this is so that the component is properly lumped so that all of the field effects happen inside the component. This brings the EMF over a section of wire to a single well defined number.

And if you are after the circuit analysis perspective of voltage at every point in Dr. Lewins circuit here it is (Tho R1=R2 in here because it makes the diagram clearer):
(https://www.eevblog.com/forum/chat/does-kirchhoffs-law-hold-disagreeing-with-a-master/?action=dlattach;attach=611443;image)
(Taken from https://www.eevblog.com/forum/chat/does-kirchhoffs-law-hold-disagreeing-with-a-master/msg2085523/#msg2085523 (https://www.eevblog.com/forum/chat/does-kirchhoffs-law-hold-disagreeing-with-a-master/msg2085523/#msg2085523) )

Notice that there are no sudden 1V jumps anywhere.

Circuit analysis abstracts magnetic effects down to just UL=L*di/dt on inductors. And this is what you would get if you had thousands of series inductors forming the way around the loop, you get a tiny voltage on each one that slowly and gradually adds up as you go around.

Close each one of those inductors with a wire that travels 90 degrees to the induced electric field at all times and you get the voltages in this diagram. And thats how you get 250mV across a 1/4 turn. You can measure it in Dr. Lewins circuit with a voltmeter. Go ahed and try it if you don't believe me.


OK, probably way off topic, but I guess I'm losing my mind.  I watched this video.  What the hell?  Could something on the internet not be right?  Or am I confused?  How could this guy get the fields so wrong?  He doesn't even agree with Wikipedia.  So I can't believe everything on the internet? :
(https://upload.wikimedia.org/wikipedia/commons/thumb/8/82/Poynting_vectors_of_DC_circuit.svg/660px-Poynting_vectors_of_DC_circuit.svg.png)
Yikes.  "We have a strong electric field inside the wire":

Comments are overwelmingly positive "mind blown"!

Yeah maybe he doesn't get all of it right, but the Poynting vector thing is pretty interesting as i haven't seen it before.
Title: Re: Does Kirchhoff's Law Hold? Disagreeing with a Master
Post by: bsfeechannel on February 02, 2019, 01:30:42 pm
OK, probably way off topic, but I guess I'm losing my mind.  I watched this video.  What the hell?  Could something on the internet not be right?  Or am I confused?  How could this guy get the fields so wrong?  He doesn't even agree with Wikipedia.  So I can't believe everything on the internet? :
(https://upload.wikimedia.org/wikipedia/commons/thumb/8/82/Poynting_vectors_of_DC_circuit.svg/660px-Poynting_vectors_of_DC_circuit.svg.png)
Yikes.  "We have a strong electric field inside the wire":

https://youtu.be/C7tQJ42nGno (https://youtu.be/C7tQJ42nGno)

Comments are overwelmingly positive "mind blown"!

It's on topic.

That's the problem with those "science" and "engineering" channels. If you want to make a career out of Youtube, what matters is views. Views, views, views. Youtube recommends you to publish at least one video a week. It is impossible to produce a science or engineering "report" with all the rigor they deserve every week unless you have a professional team of consultants, experts, proofreaders, etc. And even the channels that do have those professionals and lots of funding produce gross errors sometimes.

I only watch the EEVBlog videos because Dave is honest. In the video EEVBlog #22 (6:39) he said:

Quote
I got a rather interesting comment once, in fact I've had more than once in various forms, but the comment was basically: how do we know you're RIGHT? How  can we take your word for it? On, you know, all these things and all these topics? And, well, you know, it's a really good question, and the answer is you SHOULDN'T. You should never take anyone's word for it. Don't take anything i say on these blogs as gospel. Uh, you know, I've been in the industry for 20 years so, you know, I like to think I do know what I'm talking about mostly. But, you know, don't take my word for it. All my blogs, and all the things I talk about on here are designed to be food for thought. You're supposed to use your own engineering judgment and, you know, and go out and verify things. If you're, you know, if you're really interested in something, don't complain that i didn't explain it right or and you know i might have got it a bit wrong or something like that. Go out and investigate for yourself. That's what it's all about: food for thought.

This pretty much uncovers what video blogging is about. However, freed from rigor, the appeal to sensationalism is too tempting for some guys like Mehdi and this guy of the video above. It may stir up debates, but without the proper disclaimers, it helps to propagate all kinds of inconsequential stupid conceptions like we've seen lately.

Which is absolutely detrimental to our métier.
Title: Re: Does Kirchhoff's Law Hold? Disagreeing with a Master
Post by: Sredni on February 02, 2019, 03:19:06 pm
OK, probably way off topic, but I guess I'm losing my mind.  I watched this video.  What the hell?  Could something on the internet not be right?  Or am I confused?  How could this guy get the fields so wrong?  He doesn't even agree with Wikipedia.  So I can't believe everything on the internet? :

Yikes.  "We have a strong electric field inside the wire"

Oh, it's more on topic than it could seem.
[snip]

one thing I want to say now: I am grateful for Mehdi's ignorance, because it gave me the opportunity to deepen my understanding of how circuits work at a field level)

Quote
Comments are overwelmingly positive "mind blown"!

This is one of the dangerous aspects of the social networks. The majority of uneducated and superficial people self-reinforcing their incorrect beliefs. The voting system (like the one about comments) is basically a way to spread ignorance.
(Man, I am starting to think like Anakin in "Attack of the Clones").
Title: Re: Does Kirchhoff's Law Hold? Disagreeing with a Master
Post by: rfeecs on February 02, 2019, 11:18:58 pm
I don't know where the "We've got a strong electric field inside (the wires)" came from.  But the rest of the picture seems to come from the one reference that he lists for the video, which is the Feynman lectures:
http://www.feynmanlectures.caltech.edu/II_27.html (http://www.feynmanlectures.caltech.edu/II_27.html)
It's pretty funny Feynman constantly calls this a "crazy" theory, "obviously nuts", etc.

The example Feynman gives is for a piece of resistance wire, where he says the E field is parallel to the surface of the wire.  But I would assume that in the video the wire is a good conductor and the E field would be almost perpendicular to the surface of the wire.

The video, including the animation showing energy flow, gives the impression that energy is radiated out of the battery and into the wire and the light bulb with most of the energy flowing into the wire from the space around it.  A few people in the comments are wondering how electrical circuits can work if the wires are absorbing electromagnetic energy from all around.

Here are a few references that seem to have better simple explanations:
http://sites.huji.ac.il/science/stc/staff_h/Igal/Research%20Articles/Pointing-AJP.pdf (http://sites.huji.ac.il/science/stc/staff_h/Igal/Research%20Articles/Pointing-AJP.pdf)
http://www.furryelephant.com/content/electricity/visualizing-electric-current/surface-charges-poynting-vector/ (http://www.furryelephant.com/content/electricity/visualizing-electric-current/surface-charges-poynting-vector/)
http://www.abc.net.au/science/articles/2014/02/05/3937083.htm (http://www.abc.net.au/science/articles/2014/02/05/3937083.htm)

Title: Re: Does Kirchhoff's Law Hold? Disagreeing with a Master
Post by: Sredni on February 02, 2019, 11:54:30 pm
Pages 26 and 27 of the reference I've mentioned before refer to that passage in Feynman's book.

EDIT:
[snip]

All I want to add that that dude has chosen to stick to his "the E field is STRONGER in the cables" and is also convinced that immediately out of the conductor the E field is parallel to it.
It's "not a mistake". It's an "educational choice".
Title: Re: Does Kirchhoff's Law Hold? Disagreeing with a Master
Post by: ogden on February 04, 2019, 06:52:03 pm
So, you keep asking what is the voltage across a quarter turn, but you have to tell us where is the varying magnetic field region and along which path among the infinitely many you want to compute that voltage. You might end up with a quarter of a volt but also with much less.

I said "transformer" which means magnetic fields are contained, they do not influence voltmeter leads. I say 1/4 turn will give 1/4 V EMF. Do you agree?
Title: Re: Does Kirchhoff's Law Hold? Disagreeing with a Master
Post by: Sredni on February 04, 2019, 07:07:15 pm
So, you keep asking what is the voltage across a quarter turn, but you have to tell us where is the varying magnetic field region and along which path among the infinitely many you want to compute that voltage. You might end up with a quarter of a volt but also with much less.

I said "transformer" which means magnetic fields are contained, they do not influence voltmeter leads. I say 1/4 turn will give 1/4 V EMF. Do you agree?

As I said before, you have to specify where the B field region is and where your path is. Post a picture of your quarter of a transformer with a shaded region showing where the B field varies and we'll see if we agree or not. You might discover that while you can have the field contained with a quarter of an arc, you no longer can have it when you consider that arc as part of a circular 'transformer'. Unless you consider a different system, like the one I showed Berni a few pages back.

But please, post the picture, so that we can be sure what we are talking about.

Edit: plurals, it appears I place those at random.
Title: Re: Does Kirchhoff's Law Hold? Disagreeing with a Master
Post by: ogden on February 04, 2019, 07:59:56 pm
So, you keep asking what is the voltage across a quarter turn, but you have to tell us where is the varying magnetic field region and along which path among the infinitely many you want to compute that voltage. You might end up with a quarter of a volt but also with much less.

I said "transformer" which means magnetic fields are contained, they do not influence voltmeter leads. I say 1/4 turn will give 1/4 V EMF. Do you agree?

As I said before, you have to specify where the B field region is and where your path is. Post a picture of your quarter of a transformer with a shaded region showing where the B field varies and we'll see if we agree or not. You might discover that while you can have the field contained with a quarter of an arc, you no longer can have it when you consider that arc as part of a circular 'transformer'. Unless you consider a different system, like the one I showed Berni a few pages back.

But please, post the picture, so that we can be sure what we are talking about.

Edit: plurals, it appears I place those at random.

So you don't know what magnetically shielded transformer is, I have to show picture?  :-// Do you know what voltmeter is? Or wire?

Shall I show picture of voltmeter and wire as well?
Title: Re: Does Kirchhoff's Law Hold? Disagreeing with a Master
Post by: Sredni on February 04, 2019, 08:23:37 pm
So you don't know what magnetically shielded transformer is, I have to show picture?

Yes, please. You can draw, can't you?
Maybe, when you draw it you realize where the problem is.
Don't forget that a lumped component is essentially zero dimensional so its terminals are very close together.
Title: Re: Does Kirchhoff's Law Hold? Disagreeing with a Master
Post by: ogden on February 04, 2019, 08:47:07 pm
So you don't know what magnetically shielded transformer is, I have to show picture?  :-// Do you know what voltmeter is? Or wire?

Shall I show picture of voltmeter and wire as well?

Great news! It appears you can find virtually anything on internet, so you got picture. In following circuit "no magnetic field here" is clearly noted for wires of voltmeter "V2". I say it shall show 1/4V. What do you say?

(https://www.eevblog.com/forum/chat/does-kirchhoffs-law-hold-disagreeing-with-a-master/?action=dlattach;attach=644045;image)
Title: Re: Does Kirchhoff's Law Hold? Disagreeing with a Master
Post by: Sredni on February 04, 2019, 08:59:20 pm
Just draw your quarter circle isolated transformer, please.

Or, as Rihanna would put it: "Shut up and draw!"
Title: Re: Does Kirchhoff's Law Hold? Disagreeing with a Master
Post by: ogden on February 04, 2019, 09:07:57 pm
Just draw your quarter circle isolated transformer, please.

Or, as Rihanna would put it: "Shut up and draw!"

Can't you see picture in my previous post? If you do not understand "magnetic field of transformer do not influence voltmeter leads" and ask for drawing, then you have problem, kid.
Title: Re: Does Kirchhoff's Law Hold? Disagreeing with a Master
Post by: Sredni on February 04, 2019, 09:17:47 pm
I can see the picture in your previous post and that is not what I asked you to draw. I asked you to draw your "1/4 turn magnetically isolated transformer" highlighting the region of space where the dB/dt happens, so that we can reason on that.

Can't you do it? There is more than a way to do it and you are undecided? Pick one. We'll go from that.
Title: Re: Does Kirchhoff's Law Hold? Disagreeing with a Master
Post by: ogden on February 04, 2019, 09:31:29 pm
I can see the picture in your previous post and that is not what I asked you to draw. I asked you to draw your "1/4 turn magnetically isolated transformer" highlighting the region of space where the dB/dt happens, so that we can reason on that.

Can't you do it? There is more than a way to do it and you are undecided? Pick one. We'll go from that.

I do not need to. Picture I did show is good enough. Inside two circuit loops there's magnetic field and it is specifically shown where's no magnetic filed, indicated by note "no magnetic field here" (leads of voltmeter "V2") - meaning leads of voltmeter "V2" are not influenced by magnetic field. Do you have problem to understand that circuit or what?

[edit] I repeat question - what will be voltage shown by voltmeter V2?
Title: Re: Does Kirchhoff's Law Hold? Disagreeing with a Master
Post by: Sredni on February 04, 2019, 09:53:26 pm
Maybe you did not read the last 32 pages, but the whole point of this thread is that there are people like you who believe the Romer-Lewin ring is lumpable, and people like me who believe it isn't. So, no, I do not see a magnetically isolated transformer there (for one, where do the magnetic field lines return?, the way it is drawn they return at infinity), much less a quarter turn magnetically isolated transformer. So please, put words aside and produce a picture of your quarter turn magnetically isolated (and as such lumped) transformer.

EDIT: added boldface to highlight the part that should make it clear that the field is NOT contained.
Title: Re: Does Kirchhoff's Law Hold? Disagreeing with a Master
Post by: ogden on February 04, 2019, 10:23:10 pm
Maybe you did not read the last 32 pages but the whole point of this thread is that there are people like you who believe the Romer-Lewin ring is lumpable, and people like me who believe it isn't.

I did not say my transformer is like "Romer-Lewin ring". In case I did - please quote me. I did say "magnetic fields are contained". Seems, you have problem to comprehend what it means.

I said "transformer" which means magnetic fields are contained, they do not influence voltmeter leads. I say 1/4 turn will give 1/4 V EMF. Do you agree?

For one who understands Maxwell's equations, It shall be easy to answer w/o requesting picture, oil painting or youtube video.

[edit] Transformer core which does contain magnetic field:

https://www.eevblog.com/forum/chat/does-kirchhoffs-law-hold-disagreeing-with-a-master/msg1972940/#msg1972940 (https://www.eevblog.com/forum/chat/does-kirchhoffs-law-hold-disagreeing-with-a-master/msg1972940/#msg1972940)
Title: Re: Does Kirchhoff's Law Hold? Disagreeing with a Master
Post by: ogden on February 04, 2019, 10:41:00 pm
where do the magnetic field lines return?, the way it is drawn they return at infinity

It is said - they are contained  |O
Title: Re: Does Kirchhoff's Law Hold? Disagreeing with a Master
Post by: ogden on February 04, 2019, 10:46:55 pm
EDIT: added boldface to highlight the part that should make it clear that the field is NOT contained.

Most ridiculous debate imaginable. I said that magnetic field is contained, your argument is "should make it clear that the field is NOT contained".

 :-DD  :-DD  :-DD  :-DD  :-DD  :-DD  :-DD  :-DD

[edit] BTW You did not answer the question about indication of "voltmeter V2"
Title: Re: Does Kirchhoff's Law Hold? Disagreeing with a Master
Post by: bsfeechannel on February 04, 2019, 11:20:33 pm
[edit] I repeat question - what will be voltage shown by voltmeter V2?

You can easily detect that someone's claim is based on pseudo-scientific assumptions when it leads to contradictions. You say the voltage that the voltmeter will show is 250mV, but Mabilde and Mehdi, two other kirchhoffools like you, measured 0V (Mehdi's fist video @6:42 and Mabilde @21:55). Of course, they came with stupid explanations as to why the voltage they measured didn't match their expectations, while Faraday's law was predicting exactly what they measured.

Kirchhoffools' claims are so flawed that they can't even agree with each other. Pathetic.


Title: Re: Does Kirchhoff's Law Hold? Disagreeing with a Master
Post by: ogden on February 04, 2019, 11:50:10 pm
You say the voltage that the voltmeter will show is 250mV, but Mabilde and Mehdi, two other kirchhoffools like you, measured 0V (Mehdi's fist video @6:42 and Mabilde @21:55). Of course, they came with stupid explanations as to why the voltage they measured didn't match their expectations, while Faraday's law was predicting exactly what they measured.

Kirchhoffools' claims are so flawed that they can't even agree with each other. Pathetic.

Don't be mad that I am using your mistake as an argument :D  Mehi's video @6:42 do not measure 0V but as he say 1/10 of the voltage measured before - because as he says "current of the loop is the same, resistance is 1/10".

[edit] In case you did not notice, we don't talk about Kirchoff's rules at all, yet you manage to mention them, insulting way. THAT's pathetic
Title: Re: Does Kirchhoff's Law Hold? Disagreeing with a Master
Post by: Sredni on February 05, 2019, 12:45:08 am
Most ridiculous debate imaginable. I said that magnetic field is contained, your argument is "should make it clear that the field is NOT contained".

The fact that you say the field is contained does not mean that it is contained.
You have to contain all the lines of the magnetic field. Like in a toroidal transformer, or an M or EI transformer. In this case the field is generated by an infinitely long solenoid, and the field lines are not contained. As a matter of fact they close at infinity and the whole universe is in the middle.

The fact that you do not see it does not make it less true.

Quote
[edit] BTW You did not answer the question about indication of "voltmeter V2"

In the Romer-Lewin circuit? (BTW, maybe you meant v6) It's the value shown by bsfeechannel: 0V if the conductor is perfect, otherwise it's the drop in the copper for the current flowing in the circuit. A few tens microvolts, maybe, depending on the geometry and dimensions of the conductor. But we can say it's zero, if there is no leakage of magnetic flux (something that is hard to realize in practice with small primary coils).

edit hundreds to tens, anyway we can easily compute it
Title: Re: Does Kirchhoff's Law Hold? Disagreeing with a Master
Post by: bsfeechannel on February 05, 2019, 12:47:15 am
Don't be mad that I am using your mistake as an argument :D
I'm not mad, why would I be?

Theory and experiment confirm what's in the drawing.

I only pity you for being so hardheaded.

Quote
  Mehi's video @6:42 do not measure 0V

Oh yes he does. I'm referring to the original video, the one Dave posted.

Quote
[edit] In case you did not notice, we don't talk about Kirchoff's rules at all, yet you manage to mention them, insulting way. THAT's pathetic

I always forget that you own the thread. My sincere apologies.
Title: Re: Does Kirchhoff's Law Hold? Disagreeing with a Master
Post by: ogden on February 05, 2019, 12:56:50 am
The fact that you say the field is contained does not mean that it is contained.
You have to contain all the lines of the magnetic field. Like in a toroidal transformer, or an M or EI transformer.

When you say that wires have zero resistance, I do not ague that. The same here - if I say that I talk about ideal transformer, you shall not try to argue that transformer is not ideal. It's stupid to make preconditions as an argument.
Title: Re: Does Kirchhoff's Law Hold? Disagreeing with a Master
Post by: Sredni on February 05, 2019, 01:01:59 am
No, you don't understand. It's not that there is a little leakage: the whole lot of field lines are missing! In the infinitely long solenoid you miss the 'return' lines. So it's impossible to contain the field.
You can force the field lines into a high magnetic permeability material, and I also produced a drawing,

(https://www.eevblog.com/forum/chat/does-kirchhoffs-law-hold-disagreeing-with-a-master/?action=dlattach;attach=632611)

but that system is a DIFFERENT system from the infinitely long solenoid.

I hope it is clear now.

EDIT: pluralSSSSSSSSSSS. What is it with me and all those SSSSSSSSS???
Title: Re: Does Kirchhoff's Law Hold? Disagreeing with a Master
Post by: ogden on February 05, 2019, 01:03:29 am
Quote
  Mehi's video @6:42 do not measure 0V

Oh yes he does. I'm referring to the original video, the one Dave posted.

Well, you are welcome to show where Mehdi measure 0V. With timestamped screenshot & YT link.
Title: Re: Does Kirchhoff's Law Hold? Disagreeing with a Master
Post by: ogden on February 05, 2019, 01:16:21 am
No, you don't understand. It's not that there is a little leakage: the whole lot of field lines are missing! In the infinitely long solenoid you miss the 'return' lines. So it's impossible to contain the field.
You can force the field lines into a high magnetic permeability material, and I also produced a drawing, but that systems is a DIFFERENT systems from the infinitely long solenoid.

I hope it is clear now.

LOL. This is hilarious. Precondition is transformer with contained fields, meaning ideal transformer., Question is: what is voltage of 1/4 turn if 1/1 turn gives 1V. Your answer is "No, you don't understand. It's not that there is a little leakage"  :palm:

 :-DD
Title: Re: Does Kirchhoff's Law Hold? Disagreeing with a Master
Post by: Sredni on February 05, 2019, 01:19:05 am
I produced a drawing.
Do the same: draw your quarter turn magnetically isolated transformer and I will answer your question.

Title: Re: Does Kirchhoff's Law Hold? Disagreeing with a Master
Post by: ogden on February 05, 2019, 01:24:14 am
I produced a drawing.
Do the same: draw your quarter turn magnetically isolated transformer and I will answer your question.

Oh, you reduce our discussion to drawing. Why don't you ask me to play some music on flute so we can continue discussion?

[edit] You do not understand what is "ideal transformer", right?

Reminder that rules are easy and your request for drawing is sorry excuse:

I do not need to. Picture I did show is good enough. Inside two circuit loops there's magnetic field and it is specifically shown where's no magnetic filed, indicated by note "no magnetic field here" (leads of voltmeter "V2") - meaning leads of voltmeter "V2" are not influenced by magnetic field. Do you have problem to understand that circuit or what?

[edit] I repeat question - what will be voltage shown by voltmeter V2?
Title: Re: Does Kirchhoff's Law Hold? Disagreeing with a Master
Post by: bsfeechannel on February 05, 2019, 02:18:13 am
Quote
  Mehi's video @6:42 do not measure 0V

Oh yes he does. I'm referring to the original video, the one Dave posted.

Well, you are welcome to show where Mehdi measure 0V. With timestamped screenshot & YT link.

What is funny is that Mehdi and Mabilde give completely different explanations for the zero volts they're measuring. Mehdi thinks that the two halves of the loop have voltages that cancel each other, while Mabilde thinks that it is the probes that are canceling what he supposed to be measuring. A little earlier he says that the EMF becomes "invisible" to the voltmeter and he called the 0V that he's measuring the "MASKED EMF", typical pseudo-scientific terminology and talk.

And now you are denying what they measured by saying that the voltmeter should indicate 250mV.

You should demand them an explanation, not us.
Title: Re: Does Kirchhoff's Law Hold? Disagreeing with a Master
Post by: Berni on February 05, 2019, 06:06:47 am
Well does anyone also notice that the solenoid used in Dr. Lewins experiment is significantly shorter than infinity. It is smaller than the height of a HP digitizing scope.

So then is his experiment a scam because he is not recreating the same circuit as on the blackboard or does it simply just not matter?
Title: Re: Does Kirchhoff's Law Hold? Disagreeing with a Master
Post by: Sredni on February 05, 2019, 06:19:50 am
Well does anyone also notice that the solenoid used in Dr. Lewins experiment is significantly shorter than infinity. It is smaller than the height of a HP digitizing scope.

So then is his experiment a scam because he is not recreating the same circuit as on the blackboard or does it simply just not matter?

HE IS NOT CLAIMING THAT THE CIRCUIT CAN BE LUMPED.

Quite the contrary.

In his case the only problem with the finite size of the solenoid is that there will be flux on the outside as well. And that can give rise to spurious readings from the external voltmeters.
Title: Re: Does Kirchhoff's Law Hold? Disagreeing with a Master
Post by: Berni on February 05, 2019, 06:32:49 am
Never said he claimed anything about lumping, you are welcome to find me a quote for that.

Exactly as you say, his magnetic flux around the circuit is not the same as what he drew on the blackboard. And this was exactly your argument against using a lumped transformer. So in what way is the situation different for Dr. Lewins experimental setup? His experiment also doesn't doesn't exactly match the circuit drawing, yet acts exactly like the math says that drawing should act.

What does the field around his experimental circuit look like if you put the volmeters a significant distance away? (Such as 100 times the solenoid length)
Title: Re: Does Kirchhoff's Law Hold? Disagreeing with a Master
Post by: Simon on February 05, 2019, 07:29:39 am
calm down children please.....
Title: Re: Does Kirchhoff's Law Hold? Disagreeing with a Master
Post by: bsfeechannel on February 05, 2019, 04:26:48 pm
Well does anyone also notice that the solenoid used in Dr. Lewins experiment is significantly shorter than infinity. It is smaller than the height of a HP digitizing scope.

So then is his experiment a scam because he is not recreating the same circuit as on the blackboard or does it simply just not matter?

He showed earlier in the same lecture how a solenoid is a fairly good approximation of what is on the board. He showed how the size and shape of the loop around the solenoid don't matter. Everyone who tried to replicate Lewin's experiment got the same result: there are no voltages in the wires, the resistors show two different voltages and they add up to a value different from zero.

Lewin explained the result using the consistent, simple and elegant Faraday's law, which contradicts Kirchhoff's law when you have varying magnetic fields in the circuit. Others tried various conflicting, convoluted, confusing and contradictory pseudo-scientific explanations without success.
Title: Re: Does Kirchhoff's Law Hold? Disagreeing with a Master
Post by: Sredni on February 05, 2019, 04:33:17 pm
Never said he claimed anything about lumping, you are welcome to find me a quote for that.

Exactly as you say, his magnetic flux around the circuit is not the same as what he drew on the blackboard. And this was exactly your argument against using a lumped transformer. So in what way is the situation different for Dr. Lewins experimental setup? His experiment also doesn't doesn't exactly match the circuit drawing, yet acts exactly like the math says that drawing should act.

EDIT: [snipped the technical part]

Quote from: Simon
calm down children please.....

Oh, shut up daddy, or I'll tell mom what you did with the nanny.
Title: Re: Does Kirchhoff's Law Hold? Disagreeing with a Master
Post by: Berni on February 05, 2019, 04:55:41 pm
You can get rid of that unwanted flux because you are trying to model the ideal case of an infinitely long solenoid where that leakage flux is exactly zero. Or, if you prefer, you can get rid of the effects of that unwanted flux because it has no effect on the two resistor loop.

Yes, so its not the same thing but we treat it as being the same thing because it behaves identically. Just like a idealized transformer model behaves the same even tho the real life equivalent shows slightly different fields.

All parts of the circuit that get affected by the field are trapped inside the lumped transformer, so by definition it is correctly lumped. The only parts that are outside are the voltmeter and resistors. But as you said they have a size of zero and such can't have EMF generated inside of them so it does not matter that they are outside the transformer. No matter how strong of a field you apply to them they will act as if there is no field.

In the same way Dr. Lewins experimental circuit doesn't have field lines terminating at infinity, but instead right outside the coil and a lot even terminate inside the loop.

So the fact that im ignoring the fields effect on voltmeters and resistors because they can't be effected is just the same thing as Dr. Lewins experiment ignoring the extra returning field around the whole circuit. Both are different than the original drawing on the blackboard, but in both cases the minor difference has no effect on the circuit. Both are just the field closing back on itself in a different way, if not please explain why.
Title: Re: Does Kirchhoff's Law Hold? Disagreeing with a Master
Post by: Sredni on February 05, 2019, 05:11:17 pm
I don't know how to say it in a different way.

Picture, then
(https://www.eevblog.com/forum/chat/does-kirchhoffs-law-hold-disagreeing-with-a-master/?action=dlattach;attach=650976)
Title: Re: Does Kirchhoff's Law Hold? Disagreeing with a Master
Post by: Berni on February 05, 2019, 05:17:48 pm
Then explain why the results still match the experiment if its the wrong way to do it?
Title: Re: Does Kirchhoff's Law Hold? Disagreeing with a Master
Post by: bsfeechannel on February 05, 2019, 05:39:47 pm
Then explain why the results still match the experiment if its the wrong way to do it?

They don't. Only the voltages across the resistors are the same. The voltages in the wires are not. In Lewin's circuit the voltages are zero. Yours have 250mV. Huge difference.
Title: Re: Does Kirchhoff's Law Hold? Disagreeing with a Master
Post by: Berni on February 05, 2019, 06:18:49 pm
Then explain why the results still match the experiment if its the wrong way to do it?

They don't. Only the voltages across the resistors are the same. The voltages in the wires are not. In Lewin's circuit the voltages are zero. Yours have 250mV. Huge difference.

The voltages across the voltmeters also match up. So this means 4/4 components in the circuit show identical results when circuit analyzed as a lumped circuit.

To measure the 250mV across the wire section you need to connect to the section with ideal wires that ignore magnetic fields. If you do that in real life (by choosing a path with 0V EMF because we don't have these mythical ideal wires) you get the voltmeter also showing 250mV. If the wires does interact with the field then this is a different circuit and when modeled in(as extra inductors on that transformer) now can produce 0V for a loop that encloses no field. So the mesh model is behaving like the real life circuit in both cases. How does this differ from experimental results?
Title: Re: Does Kirchhoff's Law Hold? Disagreeing with a Master
Post by: bsfeechannel on February 05, 2019, 08:17:14 pm
If you do that in real life (by choosing a path with 0V EMF because we don't have these mythical ideal wires) you get the voltmeter also showing 250mV.

No. You don't. Mabilde and Mehdi measured precisely 0V. And they used real life wires. Then they spent the rest of their videos (and in case of Mehdi the whole second video too) trying to cook up an excuse to justify why they couldn't measure whatever voltages different from zero.
Title: Re: Does Kirchhoff's Law Hold? Disagreeing with a Master
Post by: EEVblog on February 05, 2019, 11:00:25 pm
Well does anyone also notice that the solenoid used in Dr. Lewins experiment is significantly shorter than infinity. It is smaller than the height of a HP digitizing scope.
So then is his experiment a scam because he is not recreating the same circuit as on the blackboard or does it simply just not matter?

It doesn't matter to him. He's trying to make a fundamental physics point and found a way, any way, regardless if it's a good or accurate practical analogy or not, to do it. He's never really addressed this, and probably think he doesn't have to because his theory is not wrong.
Title: Re: Does Kirchhoff's Law Hold? Disagreeing with a Master
Post by: SiliconWizard on February 05, 2019, 11:49:37 pm
Well does anyone also notice that the solenoid used in Dr. Lewins experiment is significantly shorter than infinity. It is smaller than the height of a HP digitizing scope.
So then is his experiment a scam because he is not recreating the same circuit as on the blackboard or does it simply just not matter?

It doesn't matter to him. He's trying to make a fundamental physics point and found a way, any way, regardless if it's a good or accurate practical analogy or not, to do it. He's never really addressed this, and probably think he doesn't have to because his theory is not wrong.

I think you nailed it. Getting stuck to his (in)famous experiment and trying to criticize, explain, praise or debunk it, as many of us have done one way or another, turns out completely pointless.
You're right, it's about fundamental physics, and whereas I still think the experiment itself is flawed, and has led some of us to misinterpret his point at first, he probably couldn't care less.

I still think he's caused enough confusion to many - you just need to look at this endless thread - that his approach is pedagogically flawed. As I noted much earlier, his written courses are actually much clearer than the drama he tends to make with his oral lectures - at least IMO. But I know you have to keep your students attentive. Or at least "entertained"...

The good point is that this has raised a series of interesting questioning. And after all, if this was his intention, that's well done.
Title: Re: Does Kirchhoff's Law Hold? Disagreeing with a Master
Post by: bsfeechannel on February 06, 2019, 12:09:29 am
Well does anyone also notice that the solenoid used in Dr. Lewins experiment is significantly shorter than infinity. It is smaller than the height of a HP digitizing scope.
So then is his experiment a scam because he is not recreating the same circuit as on the blackboard or does it simply just not matter?

It doesn't matter to him. He's trying to make a fundamental physics point and found a way, any way, regardless if it's a good or accurate practical analogy or not, to do it. He's never really addressed this, and probably think he doesn't have to because his theory is not wrong.

I beg to differ. If you pay attention to his lecture from the beginning, you'll see that he not only explains what approximations he is considering, but also he demonstrates them from a practical point of view.

So, Lewin didn't find just a careless way to make his point. He was careful, accurate and practical, precisely because he had faced objections before.
Title: Re: Does Kirchhoff's Law Hold? Disagreeing with a Master
Post by: bsfeechannel on February 06, 2019, 12:23:35 am
I think you nailed it. Getting stuck to his (in)famous experiment and trying to criticize, explain, praise or debunk it, as many of us have done one way or another, turns out completely pointless.
You're right, it's about fundamental physics, and whereas I still think the experiment itself is flawed, and has led some of us to misinterpret his point at first, he probably couldn't care less.

I still think he's caused enough confusion to many - you just need to look at this endless thread - that his approach is pedagogically flawed. As I noted much earlier, his written courses are actually much clearer than the drama he tends to make with his oral lectures - at least IMO. But I know you have to keep your students attentive. Or at least "entertained"...

The good point is that this has raised a series of interesting questioning. And after all, if this was his intention, that's well done.

At first, that's what I thought. Lewin was right but messed things up when trying to explain it. However after, what?, three or four months discussing about the subject, reading and re-reading papers, books, analyzing the videos, etc., and even performing experiments in my lab, I came to conclusion that the one who nailed it was exactly Lewin.

He touched on highly sensitive taboo, or myth, that is the validity of Kirchhoff's laws. Those who bash him are exactly those who consider RF, or anything Maxwell related, black magic.

I, and others, on this thread managed to realize not only how removed from understanding the basic tenet of electronics many involved with it are, but how recalcitrant they are to even try to. And this is alarming.
Title: Re: Does Kirchhoff's Law Hold? Disagreeing with a Master
Post by: Berni on February 06, 2019, 06:10:07 am
Well does anyone also notice that the solenoid used in Dr. Lewins experiment is significantly shorter than infinity. It is smaller than the height of a HP digitizing scope.
So then is his experiment a scam because he is not recreating the same circuit as on the blackboard or does it simply just not matter?

It doesn't matter to him. He's trying to make a fundamental physics point and found a way, any way, regardless if it's a good or accurate practical analogy or not, to do it. He's never really addressed this, and probably think he doesn't have to because his theory is not wrong.

I was not trying to say Dr. Lewin is wrong with his experiment. My point was that his experiment is a good enough approximation of what is drawn the blackboard just like a lumped transformer is a good enough approximation of it too. The blackboard math, the experiment and a lumped transformer model all behave identically even tho the fine details of how the fields work inside are slightly different between all 3 of those.

The only spot i disagree with Dr. Lewin is the use of KVL on a circuit that has not been modeled to include the magnetic properties of wires. Once they are modeled everything works fine and gives identical results without any sort of paradox.
Title: Re: Does Kirchhoff's Law Hold? Disagreeing with a Master
Post by: ogden on February 06, 2019, 09:55:05 am
He touched on highly sensitive taboo, or myth, that is the validity of Kirchhoff's laws. Those who bash him are exactly those who consider RF, or anything Maxwell related, black magic.

You are the one who do not consider RF black magic and see voltmeter leads as transmission lines (https://www.eevblog.com/forum/chat/does-kirchhoffs-law-hold-disagreeing-with-a-master/msg2084404/#msg2084404), Dr.Lewin's experiment as loop antenna - for frequencies (< 300Hz) present in experiment. Right.
Title: Re: Does Kirchhoff's Law Hold? Disagreeing with a Master
Post by: bsfeechannel on February 06, 2019, 12:58:22 pm
The phenomenon is exactly the same.
Title: Re: Does Kirchhoff's Law Hold? Disagreeing with a Master
Post by: ogden on February 06, 2019, 01:59:54 pm
The phenomenon is exactly the same.

You say that circuit of Dr.Lewin's experiment at RF frequencies like 3GHz acts same way as 300Hz?
Title: Re: Does Kirchhoff's Law Hold? Disagreeing with a Master
Post by: bsfeechannel on February 06, 2019, 02:34:49 pm
The only spot i disagree with Dr. Lewin is the use of KVL on a circuit that has not been modeled to include the magnetic properties of wires.

The wires do not have "magnetic properties". The only way for a wire to "react" to magnetic field is if it is moving in relation to a frame of reference.

This is stated clearly in Feynman's chapter 22:

Quote
So what is always true is that the sum of the electric field E and the cross product of the velocity of the conductor and the magnetic field B—which is the total force on a unit charge—must have the value zero inside the conductor:

F/unit charge = E+v×B = 0 (in a perfect conductor), (22.12)

where v represents the velocity of the conductor. Our earlier statement that there is no electric field inside a perfect conductor is all right if the velocity v of the conductor is zero;

So since our conductor is static, the electric field inside it is "immune" to the magnetic field.

When current flows, however, it will generate a magnetic field that will oppose the magnetizing field of the solenoid. But Lewin deliberately and carefully chose resistors with values sufficiently high so that the current will be small and generate a negligible opposing field.

If my calculations are not wrong, just to give you an idea, you need 100mT @150Hz for a 0.01m² (10x10cm) loop to generate a 1V EMF. 1mA flowing through the loop will generate less 70µT near the surface of a 0.4mm² (21AWG) wire (J ~ 3A/m²), and, since its intensity is inversely proportional to the distance of the center of the wire, it will be much less elsewhere.

So we can pretty much consider our conductors "unaware" of the magnetic field.

But they are real conductors, and what are in fact real conductors? Just resistors with a very low resistance. And resistors do allow the existence of an electric field inside them.

Let's suppose that in Lewin's experiment, each piece of conducting wire had a 0.1Ω resistance. So now the EMF will be incident all along the circuit, since there will be no ideal wire to zero the electric field along any stretch of the path of the circuit.

(https://www.eevblog.com/forum/chat/does-kirchhoffs-law-hold-disagreeing-with-a-master/?action=dlattach;attach=645443;image)

So, the total series resistance will be 1000.4Ω. Subjected to a 1V EMF, it will generate a 999.60015µA current. Multiplied by 0.1Ω that will be precisely 99.960015µV. It is not zero, but it is closer to zero than to 250mV.

Quote
Once they are modeled everything works fine and gives identical results without any sort of paradox.

Once you understand that Faraday's law invalidates Kirchhoff's law for varying magnetic fields, the paradox goes away. Believe me.

EDIT: Replaced "the charges inside it are immune" with "the electric field inside it is "immune""
Title: Re: Does Kirchhoff's Law Hold? Disagreeing with a Master
Post by: bsfeechannel on February 06, 2019, 02:40:01 pm
The phenomenon is exactly the same.

You say that circuit of Dr.Lewin's experiment at RF frequencies like 3GHz acts same way as 300Hz?

Let me see. To explain Lewin's circuit, we need Maxwell's equations. To explain how a loop antenna works, we need Maxwell's equations. I'm starting to see a coincidence there.
Title: Re: Does Kirchhoff's Law Hold? Disagreeing with a Master
Post by: ogden on February 06, 2019, 04:45:11 pm
You say that circuit of Dr.Lewin's experiment at RF frequencies like 3GHz acts same way as 300Hz?

Let me see. To explain Lewin's circuit, we need Maxwell's equations. To explain how a loop antenna works, we need Maxwell's equations. I'm starting to see a coincidence there.

Your BS answer actually proves my point :D
Title: Re: Does Kirchhoff's Law Hold? Disagreeing with a Master
Post by: bsfeechannel on February 06, 2019, 05:03:22 pm
Your BS answer actually proves my point :D

Does that make you happy?
Title: Re: Does Kirchhoff's Law Hold? Disagreeing with a Master
Post by: Berni on February 06, 2019, 05:42:29 pm
The only spot i disagree with Dr. Lewin is the use of KVL on a circuit that has not been modeled to include the magnetic properties of wires.
The wires do not have "magnetic properties". The only way for a wire to "react" to magnetic field is if it is moving in relation to a frame of reference.

This is stated clearly in Feynman's chapter 22:
Yes we can argue about what a wire is, ask a chemist and he will see a large number of copper atoms arranged in to a rod shape with some on the outside being bound to oxygen. From the point of view of circuit analysis the wire is a magnetic component because the current inside it interacts with the magnetic field, much like a capacitor is an electrostatic component (Even if its just parasitic capacitance between two wires rather than an actual component with parallel plates inside)





To prove my point that you can measure 0V or 250mV depending on what you want to see i have put together some examples:

Here is the case of an ideal transformer mesh model where we see those dreadful 250mV across one of the wire sections inside it. Below are multiple ways of constructing the physical cirucit to match that. Notice that the field is taken care of in 3 different ways, yet the result is the same. As long as the magnetic effects are contained to its wingdings it doesn't matter what the field does. Ideal cirucit analysis wires are not capable of interacting with fields, wherever it needs to interact it needs a circuit component to represent the mathematical transfer function for the field its interacting with(Inductors for magnetic fields, Capacitors for electrostatic fields).

(https://www.eevblog.com/forum/chat/does-kirchhoffs-law-hold-disagreeing-with-a-master/?action=dlattach;attach=645539;image)


Now lets see how you can get the result of 0V across the wire section. Now that the fields effects are not contained inside the transformer means that it is affecting the wires traveling around it, this makes those wires a transformer secondary too. When you model that in you now have a mesh model that behaves correctly even when the field is not contained. We didn't ignore the field, we just 'expanded' the transformer to cover it.

(https://www.eevblog.com/forum/chat/does-kirchhoffs-law-hold-disagreeing-with-a-master/?action=dlattach;attach=645545;image)

So there you go, that's how you measure 250mV in there, if you don't believe me go on and try building one of these circuits to see for yourself. Circuit analysis stays perfectly consistent when you model things correctly.
Title: Re: Does Kirchhoff's Law Hold? Disagreeing with a Master
Post by: bsfeechannel on February 06, 2019, 09:11:42 pm
From the point of view of circuit analysis the wire is a magnetic component because the current inside it interacts with the magnetic field, much like a capacitor is an electrostatic component (Even if its just parasitic capacitance between two wires rather than an actual component with parallel plates inside)

From the point of view of circuit analysis a wire is a wire. It has zero ohm resistance and does not interact with any magnetic field, because there can't be any mangnetic fields in a lumped circuit. Period.

Quote
To prove my point that you can measure 0V or 250mV depending on what you want to see i have put together some examples:

You want to see 250mV. The thing is that they are not there. None of those circuits are equivalent to Lewin's circuit, so let's forget them.

The only approximations are 2b and 2c even though they also contain errors.

Quote
So there you go, that's how you measure 250mV in there, if you don't believe me go on and try building one of these circuits to see for yourself.

I can imagine a bunch of other circuits that will give me 250mV. Only that they are not Lewin's circuit.

Quote
Circuit analysis stays perfectly consistent when you model things correctly.

If you arrange a circuit and decree in your head that it is a model of another, it may appear consistent to you, but don't expect others, especially those with a minimum understanding of the physical phenomenon, to believe you.
Title: Re: Does Kirchhoff's Law Hold? Disagreeing with a Master
Post by: Berni on February 07, 2019, 06:16:51 am
Okay i do admit that figure 1b is diferent then Dr. Lewins circuit because it has a shielding box that actually intersects the circuit, but the point of that one was to show that the box does NOT alter the behavior of the circuit as the voltmeters still show the exact same values.

But all of other circuits are the same circuit on the blackboard or represent his experimental setup. Only difference being of the 2 extra voltmeters added around it and since voltmeters have infinite impedance they don't affect the operation of the circuit in any way, they just show voltages.

Its all about connecting the voltmeter in the right way. If its 0V that you want to see just look at circuits in figures 2a 2b 2c and see it is indeed 0V. Happy now?

There is no paradox with this circuit as long as its correctly modeled.
Title: Re: Does Kirchhoff's Law Hold? Disagreeing with a Master
Post by: rfeecs on February 08, 2019, 12:50:42 am
With regard to the issue of "He's not probing it right":

A reasonable way to probe this, given that you want the voltage difference of two separated points and realizing that there are stray fields around, might be to use coaxial probes, and do a differential measurement between the two points.

For example, use typical scope probes and pull the clips off, leaving just the probe points with coax shielding almost all the way to the tips.  Use channel 1 and channel 2 for the two probes and do a CH1-CH2 measurement:

(https://www.eevblog.com/forum/chat/does-kirchhoffs-law-hold-disagreeing-with-a-master/?action=dlattach;attach=646799;image)

So everything is in one plane, 2 dimensional.

Now what do you measure?  Does it matter if the scope is on the left side or the right side?

What if you measure a quarter turn of the wire?  Do you get 0V, or something else?
Title: Re: Does Kirchhoff's Law Hold? Disagreeing with a Master
Post by: bsfeechannel on February 08, 2019, 02:51:14 am
If its 0V that you want to see just look at circuits in figures 2a 2b 2c and see it is indeed 0V. Happy now?

I don't want to see 0V. Nature shows it is 0V.

Mehdi and Mabilde reconstructed Lewin's circuit. Mabilde got even to the point of recreating Lewin's solenoid.

They thought like you: there must be a voltage in the wires. Lewin goofed it up. He bad probed the whole thing. He doesn't know how to lump model his circuit.

When they measured exactly zero volts they got puzzled, and invented each their own completely different convoluted pseudo-scientific theory to explain what the Maxwell's equations predict with simplicity and elegance: there are no voltages in the wires, nor in the probes, Kirchhoff doesn't hold for varying magnetic fields in a circuit, Faraday's law is what accounts for the voltages across the resistors, nothing else.

Quote
There is no paradox with this circuit as long as its correctly modeled.

The only thing your circuit is modeling is the lack of understanding of electromagnetism.

Your version contradicts Mehdi's version which contradicts Mabilde's version.  Which shows that the assumption that Kirchhoff always holds for varying magnetic fields is self-contradictory, i.e. is a paradox. Which is what Lewin brilliantly showed.

Since it is impossible to lump model Lewin's circuit nor any other circuit with varying magnetic fields in them, the paradox ceases to exist when you realize that the only way to solve them is to apply Faraday's law and give Kirchhoff and his doggone "law" to what it deserves: the birds.
Title: Re: Does Kirchhoff's Law Hold? Disagreeing with a Master
Post by: seagreh on February 08, 2019, 06:43:27 am
Does Kirchhoff's Law Hold?

Is it about Mehdi and/or Mabilde against Prof. Lewin and/or Robert H. Romer or is it about Gustav Robert Kirchhoff against Michael Faraday and/or James Clerk Maxwell ?

1845 Gustav Robert Kirchhoff - still being a student at this time - wrote an article in a scientific physics journal “Annals of Physics”
https://gallica.bnf.fr/ark:/12148/bpt6k151490/f509.item
About electric current passing through a plane, in particular through a circular one; from the student Kirchhoff.

And at https://gallica.bnf.fr/ark:/12148/bpt6k151490/f525.item last 3 paragraphs at this page you find his two rules.
As well as next page https://gallica.bnf.fr/ark:/12148/bpt6k151490/f526.item first paragraph.
Rule 2) being, what everybody calls the KVL today.

As it is in German, a very good translation you find in ‘Elementary Treatise on Electricity’ from James Clerk Maxwell (Oxford 1881) - page 127.
https://archive.org/details/elementarytreati00maxwrich/page/126

…Kirchhoff has stated the conditions of a linear system in the following manner, in which the consideration of the potential is avoided.
(1)   (Condition of ‘continuity.’) At any point of the system the sum of all the current which flow towards that point is zero.
(2)   In any complete circuit formed by the conductors the sum of electromotive forces  taken round the circuit is equal to the sum of products of the currents in each conductor multiplied by the resistance of that conductor….

Kirchhoff did not say

Σ Vk = 0
 
But he basically said, the sum of voltage drops (he expressed as product of resistance and current) equals to the sum of EMFs.
He did not say if emf needs to be based on:

•   electrochemical effects (galvanic cells)
•   electromagnetic induction (motional emf or transformer emf)
•   solar cell or photodiode
•   fuel cell based
•   Peltier effect
•   Seebeck effect
•   Hall effect
•   or thermopiles responding on radioactive radiation, laser radiation or pressure

Hence, applying Kirchhoff’s second rule would lead to:

I * R1 + I * R2 = Σ EMFs
1 mA * 100 Ω + 1 mA * 900 Ω = Σ EMFs
1 V = Σ EMFs

and not to   ‚1V = 0‘

Although the Σ EMFs within a mesh can be as well zero, in case the mash contains no emf, only power consuming elements. But then the sum of voltage drops is zero as well.
How, to know if you measure an emf or a voltage drop? Somehow funny some people argue, ‘you cannot measure an emf’! Partly true, as you measure a small current and conclude to a voltage drop (via a known high resistor) - hence you measure a voltage drop on your internal resistor. But, still you can measure the voltage (even of an emf)! You can measure the voltage of your car battery, the voltage of a transformer, a generator, a solar cell…

To be sure if acertain element is an emf or a voltage drop you need only to watch the current flow direction. For emf current flow is from minus to plus!

Although, Kirchhoff is not wrong and thus his rule holds, he won’t tell us where the emf is hidden exactly !
And he won't help you too much with induction in you probe wires.
Now we need to consult Mr. Faraday.
Title: Re: Does Kirchhoff's Law Hold? Disagreeing with a Master
Post by: Berni on February 08, 2019, 09:22:35 am
(https://www.eevblog.com/forum/chat/does-kirchhoffs-law-hold-disagreeing-with-a-master/?action=dlattach;attach=646799;image)

So everything is in one plane, 2 dimensional.

Now what do you measure?  Does it matter if the scope is on the left side or the right side?

What if you measure a quarter turn of the wire?  Do you get 0V, or something else?

Typical coax cable doesn't really give any shielding to magnetic fields so the scope would show different values depending on what side it is on. If the shielding material on the coax cable is made of a infinite permeability material then you should get the same value no matter where you put the scope. Same difference as Figures 1 and 2 in my post, its a different magnetic path.




I don't want to see 0V. Nature shows it is 0V.

Mehdi and Mabilde reconstructed Lewin's circuit. Mabilde got even to the point of recreating Lewin's solenoid.

They thought like you: there must be a voltage in the wires. Lewin goofed it up. He bad probed the whole thing. He doesn't know how to lump model his circuit.

When they measured exactly zero volts they got puzzled, and invented each their own completely different convoluted pseudo-scientific theory to explain what the Maxwell's equations predict with simplicity and elegance: there are no voltages in the wires, nor in the probes, Kirchhoff doesn't hold for varying magnetic fields in a circuit, Faraday's law is what accounts for the voltages across the resistors, nothing else.

I KNOW the voltage appears as a net field at the ends of the wire.

If you know enough about circuit meshes you also know that you can't have any voltage jumps within the same net, hence why a component is introduced into the mesh to represent the voltage jump, this component is what represents the potential across the ends of the wire. There is Faradays law itself inside that component.

How is it surprising that it shows 0V ? I don't remember any of them finding it particularly surprising, all they did is gave some extra explanation apart from just religiously saying "Its because Faradays law" to help people understand it from different perspectives.

Quote
There is no paradox with this circuit as long as its correctly modeled.

The only thing your circuit is modeling is the lack of understanding of electromagnetism.

Your version contradicts Mehdi's version which contradicts Mabilde's version.  Which shows that the assumption that Kirchhoff always holds for varying magnetic fields is self-contradictory, i.e. is a paradox. Which is what Lewin brilliantly showed.

Since it is impossible to lump model Lewin's circuit nor any other circuit with varying magnetic fields in them, the paradox ceases to exist when you realize that the only way to solve them is to apply Faraday's law and give Kirchhoff and his doggone "law" to what it deserves: the birds.

Can you please show me a quote where i say that Kirchhoff always holds for varying magnetic fields? (Cause i really don't remember saying that, but i have not deleted any of my posts so you should have no problem finding where i said it)

Mesh modeling is supposed to abstract away electromagnetism because there are no fields present in circuit meshes, its one of the tasks that modeling has!

What lack of understanding do i have of electromagnetism? If you say i'm lacking it then you also surely have something to quote me for where i said something wrong about it.

All i did was show that Dr. Lewins circuit can be lump modeled just fine and that the resulting circuit mesh behaves identically while causing no paradoxes. If you are going to keep accusing me of saying some made up things i never said in the whole tread, then we might as well end the discussion here.
Title: Re: Does Kirchhoff's Law Hold? Disagreeing with a Master
Post by: bsfeechannel on February 08, 2019, 04:44:52 pm
If you know enough about circuit meshes you also know that you can't have any voltage jumps within the same net, hence why a component is introduced into the mesh to represent the voltage jump, this component is what represents the potential across the ends of the wire. There is Faradays law itself inside that component.

You should get rid of this dogma.

The real and profound meaning of Faraday's law is exactly that there can be "voltage jumps" in a circuit mesh.

A voltage can appear in your mesh without any generator associated with it. Maxwell showed that electric and magnetic fields are a property of space, not a property of circuits. In our previous discussions we showed you that there can be voltages in an empty space under a varying magnetic field without any circuit. This is the proof that there can be voltages without any generators associated with them.

The source of your confusion lies in the fact that to apply circuit analysis we have to take care of any varying magnetic field first and see if it is confined to a specific region of the circuit. If it is, we declare that a forbidden zone and treat the effects of that field at the interface (the terminals) of that part of the circuit. If we already know the relation between voltages and currents at the interface, good. If we don't, we have to calculate it using Maxwell. Then the rest of the circuit can be solved using Kirchhoff, because there won't be no varying fields left. All the "voltage jumps" will be known and treated properly.

In short, we change the path of the circuit to avoid those fields. That's the meaning of this picture in Feynman's lectures.

(https://www.eevblog.com/forum/chat/does-kirchhoffs-law-hold-disagreeing-with-a-master/?action=dlattach;attach=647237;image)

No one pays the due attention to the dashed line that he calls gamma (Γ). See how it avoids the path of the unlumped circuit. So the schematics that you see on paper or on your screen are following the line gamma. Sometimes this line coincide with some wire (DC circuits, for instance), most of the times it doesn't.

If it is not possible to consider the effects of the varying magnetic field confined to a specific region of your circuit, or to specific regions (plural), in other words, if it is physically impossible to draw that dashed line away from any fields, you're doomed. Your circuit analysis is cactus.

And that's precisely what happens to Lewin's circuit.

(https://www.eevblog.com/forum/chat/does-kirchhoffs-law-hold-disagreeing-with-a-master/?action=dlattach;attach=647243;image)

So you're confounding the technique with the physical principle, which is something that I've already pointed out before.

Quote
All i did was show that Dr. Lewins circuit can be lump modeled just fine and that the resulting circuit mesh behaves identically while causing no paradoxes.

Unfortunately you didn't. Your, if we can say, "model" reveals gross errors in the understanding of electromagnetism AND circuit modeling and analysis, too. Sorry.
Title: Re: Does Kirchhoff's Law Hold? Disagreeing with a Master
Post by: Berni on February 08, 2019, 05:10:11 pm
Yes so you take the dotted Γ line in a path that encloses no net field, everything else is solved using Maxwell and everything is fine.

If this is the wrong way to apply circuit analysis can you then explain why all the voltmeters still show correct values regardless if the type of analysis applied is Faradays loop equation or just circuit analysis as a transformer? If the voltmeters are supposed to show something else feel free to point it out.
(Regarding to this post: https://www.eevblog.com/forum/chat/does-kirchhoffs-law-hold-disagreeing-with-a-master/msg2182160/#msg2182160 (https://www.eevblog.com/forum/chat/does-kirchhoffs-law-hold-disagreeing-with-a-master/msg2182160/#msg2182160) )

If its wrong you should be able to make it spit out wrong results too (at least using some special case circuit if not in general).
Title: Re: Does Kirchhoff's Law Hold? Disagreeing with a Master
Post by: rfeecs on February 08, 2019, 05:48:49 pm
(https://www.eevblog.com/forum/chat/does-kirchhoffs-law-hold-disagreeing-with-a-master/?action=dlattach;attach=646799;image)

So everything is in one plane, 2 dimensional.

Now what do you measure?  Does it matter if the scope is on the left side or the right side?

What if you measure a quarter turn of the wire?  Do you get 0V, or something else?

Typical coax cable doesn't really give any shielding to magnetic fields so the scope would show different values depending on what side it is on. If the shielding material on the coax cable is made of a infinite permeability material then you should get the same value no matter where you put the scope. Same difference as Figures 1 and 2 in my post, its a different magnetic path.


There are no magnetic fields where the coax is.  You yourself simulated the field pattern for a solenoid.  There is only an electric field.  The coax should have some effect with an electric field present.
Title: Re: Does Kirchhoff's Law Hold? Disagreeing with a Master
Post by: Berni on February 08, 2019, 06:14:27 pm
(https://www.eevblog.com/forum/chat/does-kirchhoffs-law-hold-disagreeing-with-a-master/?action=dlattach;attach=646799;image)

So everything is in one plane, 2 dimensional.

Now what do you measure?  Does it matter if the scope is on the left side or the right side?

What if you measure a quarter turn of the wire?  Do you get 0V, or something else?

Typical coax cable doesn't really give any shielding to magnetic fields so the scope would show different values depending on what side it is on. If the shielding material on the coax cable is made of a infinite permeability material then you should get the same value no matter where you put the scope. Same difference as Figures 1 and 2 in my post, its a different magnetic path.


There are no magnetic fields where the coax is.  You yourself simulated the field pattern for a solenoid.  There is only an electric field.  The coax should have some effect with an electric field present.

Yes the conductive shield around a coax does make it immune to electrostatic fields as the charges in the shield will redistribute to perfectly oppose it. However the non conservative field caused by the vicinity of a magnetic field is not the same thing. The charges need to be able to flow along the length of the cables shield in order to counter the effects and the shield would need to be superconductive to create a opposing field that is just as strong. Since the coax abruptly ends this is not possible.

The reason Romer uses coax cable is to make sure none of the effect comes from capacitive crosstalk since parts of the circuit are undergoing rapid voltage changes during the experiment. If coax cable was capable of hiding its inner conductor from magnetic effects then Romer would have seen no voltage.

However coax cables are good at maintaining a low leakage inductance between the inner conductor and the shield, so applying a voltage on one end between the conductor and shield will make it to the other end of the cable unaffected (between the conductor and shield) even if the cable passes trough strong magnetic fields. In other words the conductor and shield appear to occupy the same physical space so there is no loop area and so Faradays law says the voltage is zero no matter how strong of a field there is.

EDIT: Sorry i incorrectly remembered the coax part. Romer didn't use any coax, just said that the wires have to run close to each other.
Title: Re: Does Kirchhoff's Law Hold? Disagreeing with a Master
Post by: rfeecs on February 08, 2019, 07:26:45 pm
Yes the conductive shield around a coax does make it immune to electrostatic fields as the charges in the shield will redistribute to perfectly oppose it. However the non conservative field caused by the vicinity of a magnetic field is not the same thing.
This is a quasi-electrostatic situation, at least the way Romer set up the experiment.  The electric field is constant during the measurement period.

Once again you are saying that charge can tell the difference between "different" electric fields?

I don't think so.  But I agree that the charge on the outer surface of the coax shield will redistribute itself to create an opposing field resulting in no tangential field along the outer surface of the coax.  The same thing happens on the inner surface.  So the center conductor sees no E field in the direction along it's path.

Consequently the charge on the center conductor does not rearrange itself in response to the outside field.  It is shielded by the outer conductor.

Quote
The reason Romer uses coax cable...
I don't see any mention of coax cable in Romer's paper.
Title: Re: Does Kirchhoff's Law Hold? Disagreeing with a Master
Post by: Berni on February 08, 2019, 08:36:41 pm
Sorry i remembered Romers paper a bit incorrectly. He just says about wires having to be run close to each other, not actually using coax.

The difference between a purely electrostatic field and one induced by a magnetic field is the non conservative part. This gives the field ability to push electrons around closed loops of conductor while a electrostatic field can only redistribute them but not sustain a current apart from the very brief transient as they redistribute.

Since the inner conductor forms a continuous loop trough the voltmeter, it means that the the field can push the electrons around it and create a current that the voltmeter detects as voltage across its internal resistance. The shield however does not form a continuous loop and as such can't experience any current trough it.

If you are to connect the ends of the two shields together with a wire you would get currents flowing inside of it and there would be effects on the voltmeters readings. How exactly they would be affected depends on how exactly the ends are connected. If the conductivity of the shield is sufficiently high then you can probably get the result of 0.1V or 0.9V or anything in between depending on what path you run that shield connecting wire. But this is going a bit off topic, its not really relevant. My point is that the coax shield in this configuration has no effect.
Title: Re: Does Kirchhoff's Law Hold? Disagreeing with a Master
Post by: bsfeechannel on February 08, 2019, 10:03:52 pm
Yes so you take the dotted Γ line in a path that encloses no net field, everything else is solved using Maxwell and everything is fine.

If this is the wrong way to apply circuit analysis can you then explain why all the voltmeters still show correct values regardless if the type of analysis applied is Faradays loop equation or just circuit analysis as a transformer? If the voltmeters are supposed to show something else feel free to point it out.
(Regarding to this post: https://www.eevblog.com/forum/chat/does-kirchhoffs-law-hold-disagreeing-with-a-master/msg2182160/#msg2182160 (https://www.eevblog.com/forum/chat/does-kirchhoffs-law-hold-disagreeing-with-a-master/msg2182160/#msg2182160) )

If its wrong you should be able to make it spit out wrong results too (at least using some special case circuit if not in general).

It is wrong because you added "compensation" to your probes to account for a non existing voltage in the wires.

You know, this is the problem with using Spice as a learning tool. Spice is a stupid software with no critical thinking. If your premises are wrong it will accept them as truth and will sheepishly confirm whatever wrong conclusions you have drawn.

Ditch this devilish program at once and learn electromagnetism as it should: studying the classics, like Feynman's lectures and such.
Title: Re: Does Kirchhoff's Law Hold? Disagreeing with a Master
Post by: Berni on February 08, 2019, 10:25:04 pm
It is wrong because you added "compensation" to your probes to account for a non existing voltage in the wires.

You know, this is the problem with using Spice as a learning tool. Spice is a stupid software with no critical thinking. If your premises are wrong it will accept them as truth and will sheepishly confirm whatever wrong conclusions you have drawn.

Ditch this devilish program at once and learn electromagnetism as it should: studying the classics, like Feynman's lectures and such.

Where is the compensation? I'm just treating the probe wires the same way as the circuit wires since they follow the same path. Should wires that connect voltmeters be treated in some special way?

And what is wrong with SPICE? Its just a set of tools for analyzing circuit meshes because doing it manually by pen and paper becomes very time consuming for large circuits. It doesn't invent any new way of modeling circuits, just automates the existing circuit analysis math that people had to do by hand before computers came around Can you recommend a better alternative for predicting the behavior of a circuit that includes active nonlinear components? A good circuit to show as a example on might be the Royer oscillator as it involves magnetic components.
Title: Re: Does Kirchhoff's Law Hold? Disagreeing with a Master
Post by: bsfeechannel on February 08, 2019, 11:37:00 pm
Where is the compensation? I'm just treating the probe wires the same way as the circuit wires since they follow the same path. Should wires that connect voltmeters be treated in some special way?

The "transformers" you introduced there. They don't exist. Transformers are lumped components. They must have terminals where there be no varying magnetic fields. You already know how to lump part of a mesh under varying magnetic fields. If you look at Lewin's circuit again you'll see no way to draw the gamma line.

What your circuit is modeling is something else, not Lewin's circuit.

(https://www.eevblog.com/forum/chat/does-kirchhoffs-law-hold-disagreeing-with-a-master/?action=dlattach;attach=647435;image)

Quote
And what is wrong with SPICE?

Spice is a simulation software, not a learning tool. As I said, it has no mechanism to check whether your assumptions are sound or not. And that's your problem. Your assumptions are wrong. Hence you are drawing the wrong conclusions, and since Spice is just confirming the conclusions you draw from wrong assumptions, you think your reasoning is right.

Spice is already inadequate as a tool to learn about circuits. It is an insanity to try to use it to understand electromagnetism.
Title: Re: Does Kirchhoff's Law Hold? Disagreeing with a Master
Post by: seagreh on February 09, 2019, 01:42:12 am
Following your theory…

The inner circle in Lewin’s circuit underlies varying magnetic flux and all other nasty things as well.
Kirchhoff doesn’t hold, because it won’t add up to zero! The sky is FALLING. We solve this with Maxwell’s equations.

The outside ‘circle’ (consisting of the probe leads) are NOT underlying a varying magnetic field. is this what you are stressing ?
Hence, now Kirchhoff holds in this domain, because it adds up to zero! The sky is BLUE!

0.9V + 0.1V = 0 V

Title: Re: Does Kirchhoff's Law Hold? Disagreeing with a Master
Post by: Berni on February 09, 2019, 12:01:03 pm
Yes that is another way to draw the physical representation of my lumped circuit, it looks slightly different but behaves identical. Much like Dr. Lewins physical experiment, the fields in there are not the same (Field closes around on his table and not at the edge of the universe), but the end result is identical so does it matter?

I was never suggesting SPICE is a good tool for learning about electromagnetism. Its just a automated implementation of manual pen and paper mesh circuit analysis that is thought in every electronics engineering school on the planet. Mesh circuits have no fields or physical dimensions, just nodes, volts and amps. It doesn't ignore fields in a physical circuit, just hides them away into a lumped component. This abstraction makes it great instead for learning about the circuits behavior under given conditions, especially when circuits get large and contain 100s of components.


So then on the topic of the correct way to analyze a circuit.

(https://www.eevblog.com/forum/chat/does-kirchhoffs-law-hold-disagreeing-with-a-master/?action=dlattach;attach=647828;image)

This is a slightly more complex circuit than Dr. Lewins experiment, increasing the component count to 3 parts. However one of the parts now exhibits non-linear behavior due to being a semiconductor. But it is still simpler than 99% of circuits you can find on the internet, so determining its behavior should not be difficult right?

How would you analyze this circuit in the correct way?

What happens to to the current in the inner and outer loops upon connecting the battery power source?
Title: Re: Does Kirchhoff's Law Hold? Disagreeing with a Master
Post by: rfeecs on February 10, 2019, 12:36:59 am
The difference between a purely electrostatic field and one induced by a magnetic field is the non conservative part. This gives the field ability to push electrons around closed loops of conductor while a electrostatic field can only redistribute them but not sustain a current apart from the very brief transient as they redistribute.

Since the inner conductor forms a continuous loop trough the voltmeter, it means that the the field can push the electrons around it and create a current that the voltmeter detects as voltage across its internal resistance. The shield however does not form a continuous loop and as such can't experience any current trough it.

I don't see why you would need current to shield a static electric field.

Quote
...My point is that the coax shield in this configuration has no effect.

I'm not convinced.  I expect the coax will shield the center conductor from the induced electric field.  So you would measure the same voltage whether the scope was on the left or the right side.
Title: Re: Does Kirchhoff's Law Hold? Disagreeing with a Master
Post by: seagreh on February 10, 2019, 08:28:29 pm
Does bsfeechannel’s thesis hold?

(https://www.eevblog.com/forum/chat/does-kirchhoffs-law-hold-disagreeing-with-a-master/?action=dlattach;attach=648777;image)

Assumption about the magnetic field:

So far I am not asking where the real seat of EMF actually is  :)

Title: Re: Does Kirchhoff's Law Hold? Disagreeing with a Master
Post by: Berni on February 11, 2019, 05:19:46 pm
I don't see why you would need current to shield a static electric field.
Quote
...My point is that the coax shield in this configuration has no effect.

I'm not convinced.  I expect the coax will shield the center conductor from the induced electric field.  So you would measure the same voltage whether the scope was on the left or the right side.

Conductors only null the field inside of them, not outside. Hence why current flow is needed in order to generate a opposing magnetic field to cancel out the original one outside the conductor. The induced non conservative electric field appears everywhere around the magnetic field, no need for the magnetic field to actually intersect the conductor.

To find out you can do a quick experiment to join two channels of a scope with a coax cable, wrap the cable around the solenoid and put power into it. When the shield is continuous from one port of the scope to the other you will get next to no voltage, but if you break the shield by cutting it (Or placing a adapter in the cable that disconnects it but leaves center connected) you should suddenly now see plenty of voltage when power is connected to the solenoid.

Does bsfeechannel’s thesis hold?

(https://www.eevblog.com/forum/chat/does-kirchhoffs-law-hold-disagreeing-with-a-master/?action=dlattach;attach=648777;image)

I don't quite get it how the inner circuit would influence the two meters on the outside at all. Its not connected at all and the resistances in the inner circuit are too high to cause a significant counter field to be generated. You could just get both voltmeters showing half the EMF.
Title: Re: Does Kirchhoff's Law Hold? Disagreeing with a Master
Post by: Sredni on February 13, 2019, 07:58:48 am
This post has been cleansed to avoid upsetting other children.
Whatever was written here can be found in one or more of the following books (in no particular order, and without mentioning the usual suspects Feynman, Purcell, Griffiths, Ohanian, Jackson):

Panofsky, Phillips
Classical Electricity and Magnetism 2nd ed

John Kraus
Electromagnetism 2nd to 4th ed

Ramo, Whinnery, VanDuzer
Fields and Waves in Communication Electronics 2nd or 3rd ed

Bleaney
Electricity and Magnetism 3rd ed

Nayfeh, Brussel
Electricity and Magnetism

Kip
Fundamentals of Electricity and Magnetism 2nd ed

Lorrain, Courson
Electromagnetic Fields and Waves 2nd ed


As a gift to the other children, here are some color pictures that illustrate the problem you need to solve to lump the Romer-Lewin circuit

(https://www.eevblog.com/forum/chat/does-kirchhoffs-law-hold-disagreeing-with-a-master/?action=dlattach;attach=650970)
What a Kirchhoffian does not get

The youngest among you could aspire to win the Fields Medal.
BTW, a topologist might not agree but...

(https://www.eevblog.com/forum/chat/does-kirchhoffs-law-hold-disagreeing-with-a-master/?action=dlattach;attach=650982)
What a Flatearther does not get

Yep, the maroon challenge above is about the same level of difficulty a flatearther has to face.


"Books" are static paper based documents that can be found in libraries. They are like smartphones, but (usually) bigger, with lots and lots of extremely thin flexible e-ink screens and a very long battery life. Libraries are...
Oh, never mind. Keep on pushing that square peg into that round hole. With a big enough hammer, it will fit.
Title: Re: Does Kirchhoff's Law Hold? Disagreeing with a Master
Post by: Sredni on February 13, 2019, 08:00:27 am
calm down children please.....

As for the mod who banned me (which I presume it's you given the timing), let me teach you a lesson in what a moderator should do.
A moderator should protect the discussion, not their fragile ego. Can you point out what granted me a seven day ban, except for the fact that I responded to a silly remark in jest?
If you want respect, show some fucking respect in the first place. I am probably old enough to be your father, so don't you fucking call people you don't know children, especially when they are using their own free time to bring a little bit of knowledge in this forum. If you use that condescending tone, the least you can expect is for people to answer in tone. If you didn't like the tone, you should have not used it in the first place.

If you wanted to make a formal request you should have done so and should have specified what you expect people to do: saying their opponent is right so as to not piss them off? Use flowery language? Avoiding repetitions even if they forget what was shown a few pages back? Deleting the parts that show they're wrong to avoid upsetting them? (Hey, that's an idea! Consider it done.)

I do not see any contribution from you in this discussion (including what should be a moderator's job). Actually, thanks to your incompetence and easily punctured ego, you ended up taking away from it.
You took away, to begin with, the answer I had written and got lost because I was banned (for... pulling your thin-skinned majesty's leg) when I was still editing it.
And you took away any other answer from me (*).

Learn to do your 'job', or leave it to someone who has the skills and attitude to do it.

It takes but one incompetent bloke...


(*) As a side note, this is the second time my time gets wasted here, and I too have rules. 
I guess I'll have to thank you for having put me in leech mode: less effort on my part. I'll enjoy flatland from above, where the inhabitants cannot see, nor hear me.


P.S.
I removed only the technical parts from my previous posts. I left all that remained.
Title: Re: Does Kirchhoff's Law Hold? Disagreeing with a Master
Post by: Simon on February 13, 2019, 11:19:26 am
The topic was getting heated and was reported, I in turn was trying to jest in calming things down without upsetting anyone. If you don't like a moderation comment then don't answer in jest as the situation has already been determined to be fragile. With no country showing on your profile i can't even put your response down to language barrier.

I don't care for your respect, all I ask is that the forum is respected, my job is to look after the forum, think of me as you will. Granted I may not always get it right, you can always contact me and I'll be only too happy to listen and explain and happily be convinced to remove a ban.
Title: Re: Does Kirchhoff's Law Hold? Disagreeing with a Master
Post by: Siwastaja on February 13, 2019, 11:24:56 am
The topic was getting heated and was reported, I in turn was trying to jest in calming things down without upsetting anyone.

Umm, you know, when things get heated, a certain number of people always abuse the "report to moderator" button. It's their way of dealing with the emotions.

You don't need to react to each and every report. You are supposed to see for yourself. You are not obliged to ban people (or react whatsoever) just because you are seeing reports. You are supposed to ban people if they really disregard the actual rules severely enough. You are supposed to be the objective "third party". This shouldn't be a voting system. IMHO.

I have seen your decision making is clearly being affected by the reports, and in my opinion this is an actual problem.
Title: Re: Does Kirchhoff's Law Hold? Disagreeing with a Master
Post by: ogden on February 13, 2019, 11:26:58 am
I am probably old enough to be your father, so don't you fucking call people you don't know children

Seems, you have issues. Don't act like kid if you don't like to be labelled as such. By deleting your posts you did not act like a man at all. Attitude you show now is disgusting.
Title: Re: Does Kirchhoff's Law Hold? Disagreeing with a Master
Post by: Simon on February 13, 2019, 11:29:50 am
Indeed i do not react to every report and sometimes even email the reporter back and tell them to "suck it up". I looked at the current page of the report and the one before and noted that things were getting a little "heated" with no particular person to blame although one had been by virtue of the report and i was not prepared to blame them so I tried to diffuse the situation.
Title: Re: Does Kirchhoff's Law Hold? Disagreeing with a Master
Post by: Simon on February 13, 2019, 11:31:03 am
I am probably old enough to be your father, so don't you fucking call people you don't know children

Seems, you have issues. Don't act like kid if you don't like to be labelled as such. By deleting your posts you did not act like a man at all. Attitude you show now is disgusting.


I have had to "deal" with many a member who is older than me, well old enough to be my father. Age makes no difference on here, we are all the same but in intellect!
Title: Re: Does Kirchhoff's Law Hold? Disagreeing with a Master
Post by: ogden on February 13, 2019, 11:34:05 am
Umm, you know, when things get heated, a certain number of people always abuse the "report to moderator" button. It's their way of dealing with the emotions.

While ago I did hurt feelings of Sredni. Somebody called moderator who just said "calm down kids children". I am not 100% sure, but I think one who pressed button was Sredni. It just unexpectedly backfired after he insulted moderator.

Upon return from ban he literally destroyed most of his posts in this thread like little children do when upset. Seems like Simon was right :D
Title: Re: Does Kirchhoff's Law Hold? Disagreeing with a Master
Post by: Simon on February 13, 2019, 11:37:30 am
i can't remember who made the report and I don't care.
Title: Re: Does Kirchhoff's Law Hold? Disagreeing with a Master
Post by: bsfeechannel on February 13, 2019, 09:04:25 pm
As for the mod who banned me (which I presume it's you given the timing), let me teach you a lesson in what a moderator should do. A moderator should protect the discussion, not their fragile ego. Can you point out what granted me a seven day ban, except for the fact that I responded to a silly remark in jest?

I can understand that you are pissed off because of all of the hours spent in front of the computer, trying to convey the most scientifically precise and consistent replies. I'm shocked to know that you were banned temporarily.

However we need to express our appreciation for Simon's work. I've been a moderator before. It is one of the most ungrateful tasks, let me tell you.

Quote
P.S.
I removed only the technical parts from my previous posts. I left all that remained.

Please reconsider your decision. Your posts are one of the reasons that I reckon this one of the best threads of the forum. Think of all other people who will read this thread and get educated on the subject we've been discussing.
Title: Re: Does Kirchhoff's Law Hold? Disagreeing with a Master
Post by: EEVblog on February 14, 2019, 12:03:06 am
I think you nailed it. Getting stuck to his (in)famous experiment and trying to criticize, explain, praise or debunk it, as many of us have done one way or another, turns out completely pointless.
You're right, it's about fundamental physics, and whereas I still think the experiment itself is flawed, and has led some of us to misinterpret his point at first, he probably couldn't care less.

I still think he's caused enough confusion to many - you just need to look at this endless thread - that his approach is pedagogically flawed. As I noted much earlier, his written courses are actually much clearer than the drama he tends to make with his oral lectures - at least IMO. But I know you have to keep your students attentive. Or at least "entertained"...
The good point is that this has raised a series of interesting questioning. And after all, if this was his intention, that's well done.
At first, that's what I thought. Lewin was right but messed things up when trying to explain it. However after, what?, three or four months discussing about the subject, reading and re-reading papers, books, analyzing the videos, etc., and even performing experiments in my lab, I came to conclusion that the one who nailed it was exactly Lewin.
He touched on highly sensitive taboo, or myth, that is the validity of Kirchhoff's laws. Those who bash him are exactly those who consider RF, or anything Maxwell related, black magic.
I, and others, on this thread managed to realize not only how removed from understanding the basic tenet of electronics many involved with it are, but how recalcitrant they are to even try to. And this is alarming.

Yes, engineers are inherently recalcitrant, they usually need to be because electronics engineering is more of an applied practical science than a theoretical science. Most practical practicing engineers rarely dive into the deeply theoretical world, they just use practical tools like Kirchhoff's to get the job done.
What would Bob Pease do...
Title: Re: Does Kirchhoff's Law Hold? Disagreeing with a Master
Post by: EEVblog on February 14, 2019, 12:06:21 am
P.S.
I removed only the technical parts from my previous posts. I left all that remained.

What does that mean exactly?
Did you remove all technical content form all your previous posts?
What is your intention for future posts?

EDIT: I see you have gone through and edited a lot of your previous posts to the same thing. This is borderline spam.

Quote
This post has been shortened and cleansed to avoid upsetting other children.
Whatever was written here can be found in one or more of the following books (in no particular order, and without mentioning the usual suspects Feynman, Purcell, Griffiths, Ohanian, Jackson):


Kip
Fundamentals of Electricity and Magnetism 2nd ed

Lorrain, Courson
Electromagnetic Fields and Waves 2nd ed

John Kraus
Electromagnetism 2nd to 4th ed

Ramo, Whinnery, VanDuzer
Fields and Waves in Communication Electronics 2nd or 3rd ed

Panofsky, Phillips
Classical Electricity and Magnetism 2nd ed

Bleaney
Electricity and Magnetism 3rd ed

Nayfeh, Brussel
Electricity and Magnetism

Title: Re: Does Kirchhoff's Law Hold? Disagreeing with a Master
Post by: bsfeechannel on February 14, 2019, 04:07:08 am
Yes, engineers are inherently recalcitrant, they usually need to be because electronics engineering is more of an applied practical science than a theoretical science. Most practical practicing engineers rarely dive into the deeply theoretical world, they just use practical tools like Kirchhoff's to get the job done.

Many on this forum and I are engineers who are not recalcitrant to learn when it is shown that a certain concept is wrong. Sticking to pseudo-scientific claims has nothing to do with being practical.

We proved that Mehdi's claims are bullshit and there is no justification in any exercise of any practice that can reverse that fact.

If not knowing theory was a sign of good or practical engineering, you should be discouraged to learn Kirchhoff's laws themselves. They are no less "theoretical" than Maxwell's equations. In fact, Kirchhoff's laws ARE Maxwell's equations for special cases.

This equation: N = Vpkp/(2πBsatA) is a solution of Faraday's equation and that's what we use to calculate the dimensions and number of turns of the primary of any transformer with EI, CI, toroidal, etc, ferromagnetic cores. It is a practical equation. You must have transformers by the hundreds in your lab and your house. It is a circuital law where KVL is invalid. I.e., if KVL held in this case, you would never have a transformer.

So, ignoring Maxwell's equations has nothing to do with being practical. It is just an incapacitating feature. Deciding whether it is worthwhile learning them is a matter of choice.

Quote
What would Bob Pease do...

I pretty much think that he would not confuse practicality with ignorance. He was the Czar of Bandgaps. I would kill to have 10% of his knowledge.
Title: Re: Does Kirchhoff's Law Hold? Disagreeing with a Master
Post by: EEVblog on February 14, 2019, 05:04:04 am
So, ignoring Maxwell's equations has nothing to do with being practical. It is just an incapacitating feature. Deciding whether it is worthwhile learning them is a matter of choice.

I think you are taking this a bit too seriously. No one (at least not me) is talking about being deliberately ignorant, or that being ignorant of the deep theory is a good thing.
Just pointing out that most practical engineers need not concern themselves with Maxwells equations (esp in a case like this) and this whole debate is all but pointless to any practical engineer. They'll just happily continue to use Kirchhoff's to make practical stuff that works, and just go "meh" to Lewin's academic argument (in this case). And there's nothing wrong with doing that, horses for courses.

I have not followed this whole thread, so I don't know about your arguments that Electroboom's claims are 100% bullshit, but I suspect that this isn't such a black and white case.
I don't think anyone doubts that Lewin is ultimately right (he is), but AFAIK he failed to address any of Electroboom's practical points.
From what I have seen, it's Lewin with his fingers in his ears repeating "KVL doesn't hold" 100 times, vs Electroboom trying to methodically evaluate the problem from a practical demonstration standpoint. From that I know who I have more respect for at the very least.
Title: Re: Does Kirchhoff's Law Hold? Disagreeing with a Master
Post by: Berni on February 14, 2019, 06:26:02 am
The half the whole tread is mostly just about being overly picky about the details and naming of things in physics. The thread did provide some interesting thought experiments along the way, but untimely things never got any closer to an agreement.

Circuit analysis (And that includes KVL) was never meant to be used to explain the underlying physics, but instead doing the opposite, made to abstract away any non vital parts of physics to let you focus on the operation of a circuit. If you apply circuit analysis the wrong way to your circuit then you get wrong results. Garbage in garbage out simple as that.

So if KVL is for the birds then the entirety of circuit analysis theory is for the bids as well. I'm sure any proper electronics engineer will disagree because circuit analysis has served them well ever since learning it in school.

So please circuit analysis for explaining circuits and not physics.

I also don't understand what Sredni was trying to accomplish by deleting content from his posts. Basically vandalizing his own work to make it harder for someone else to follow the tread.
Title: Re: Does Kirchhoff's Law Hold? Disagreeing with a Master
Post by: bsfeechannel on February 14, 2019, 06:40:35 am
Just pointing out that most practical engineers need not concern themselves with Maxwells equations (esp in a case like this) and this whole debate is all but pointless to any practical engineer. They'll just happily continue to use Kirchhoff's to make practical stuff that works, and just go "meh" to Lewin's academic argument (in this case). And there's nothing wrong with doing that, horses for courses.

I am a practical engineer, and this is discussion is not pointless at all. Lewin's experiment is only a "meh" experiment for the inattentive viewer (as I was one day).

Quote
I have not followed this whole thread, so I don't know about your arguments that Electroboom's claims are 100% bullshit, but I suspect that this isn't such a black and white case.

I regret to inform you that Mehdi's claim that Kirchhoff always holds is 100% bullshit. This thread is long, but is awesome. I spent a lot of time that I didn't and I don't have with it, but it was worth every second.

Quote
I don't think anyone doubts that Lewin is ultimately right (he is), but AFAIK he failed to address any of Electroboom's practical points.

Our investigation on the subject showed that Lewin addressed his practical points. But Mehdi denied them one by one.

Quote
From what I have seen, it's Lewin with his fingers in his ears repeating "KVL doesn't hold" 100 times, vs Electroboom trying to methodically evaluate the problem from a practical demonstration standpoint. From that I know who I have more respect for at the very least.

Mehdi made you believe that. But we are not buying it. We're not so gullible. This is just one example. His video is full of facepalm moments.

Richard Feynman (http://www.feynmanlectures.caltech.edu/II_22.html), recommended by Mehdi, said:

Quote
The difference of these two potentials is what we call the voltage difference, or simply the voltage V, so we have

V = −∫baE⋅ds = −∮E⋅ds.

In his second video, "Kirchhoff's Voltage Law vs Faraday's Voltage Law" @ 8:07, he says:

Quote
But then you might say that the voltage is integral of E dl [i.e −∮E⋅ds] . Well, that's not true. Voltage is any energy per unit charge. Not just energy from electric sources. Hmm. Is that it? Does Dr. Lewin believe that voltage is only defined by electric forces?

You clearly see that Mehdi not only denies the definition of voltage given by Richard Feynman and repeated by Lewin, but also doesn't understand what the integral means. And he has the petulance of throwing his misunderstanding upon Lewin's shoulders.

After that he draws every kind of stupid conclusion with his experiments because of course he refuses to understand the phenomenon.

However, if his refractory attitude to understanding (a.k.a stupidity) is not enough, he ends his video saying that he read Feynman's reference above (@13:16) and @13:58 he says
Quote
So Dr. Belcher also concluded that Dr. Feynman himself and I have the same definition for voltage [...]

Now you can see that he's an outright liar.

People like that do not deserve our respect and should not be addressed as engineers.

And you can see how dimwitted Mehdi's audience is for not noticing this.

EDIT: Wrong reference to Mehdi's video corrected.
Title: Re: Does Kirchhoff's Law Hold? Disagreeing with a Master
Post by: beanflying on February 14, 2019, 06:52:16 am

....

You continue to think being an arrogant intellectual bully who time and again resorted to petty name calling and demeaning anyone not totally agreeing with you is a credible Scientific or Engineering method. On any level and as I mentioned a long while ago would not be tolerated in most workplaces or institutions of learning.

In this thread is some great reading but so much useless non Science and Engineering and OTT Ego driven non debate.

Play the science and engineering not play the man!
Title: Re: Does Kirchhoff's Law Hold? Disagreeing with a Master
Post by: bsfeechannel on February 14, 2019, 07:29:38 am
The half the whole tread is mostly just about being overly picky about the details and naming of things in physics. The thread did provide some interesting thought experiments along the way, but untimely things never got any closer to an agreement.

This thread was never about coming to an agreement.

Does Kirchhof hold or not? Under varying magnetic fields, definitely not. Period. Never held. Never will.

You can disagree until the cows come home. The truth is established and that's what counts.

Engineers do not design their projects upon opinions, but upon established truths.

Quote
Circuit analysis (And that includes KVL) was never meant to be used to explain the underlying physics, but instead doing the opposite, made to abstract away any non vital parts of physics to let you focus on the operation of a circuit. If you apply circuit analysis the wrong way to your circuit then you get wrong results. Garbage in garbage out simple as that.

Kirchhoff's laws were meant to explain the underlying physics. Read Kirchhoff's original paper (https://gallica.bnf.fr/ark:/12148/bpt6k151490/f509.item) posted by seagreh. If you can't read German, cross the border and kindly ask an Austrian inhabitant to read it for you.

Quote
So if KVL is for the birds then the entirety of circuit analysis theory is for the bids as well. I'm sure any proper electronics engineer will disagree because circuit analysis has served them well ever since learning it in school.

Kirchhoff is for the birds means that Kirchhoff is not a fundamental law. Just that. Be prepared to see it being violated repeatedly. It just means that circuit analysis has its limits. It cannot be used for every kind of circuit. And you have to know when it is not applicable.

I bet SPICE didn't tell you that.

Quote
So please circuit analysis for explaining circuits and not physics.

Circuit analysis is physics.

Quote
I also don't understand what Sredni was trying to accomplish by deleting content from his posts. Basically vandalizing his own work to make it harder for someone else to follow the tread.

I guess he found that this forum was not worthy of his posts. But I guess he has to recognize that talking back to a moderator, in his duty as a moderator, is never a good idea. Even more because Sredni was doing nothing wrong. However his attitude induced to Simon think that he was the cause of the problem and banned him.

In my opinion, Sredni didn't deserve to be banned, but Simon didn't deserve Sredni's rant either.
Title: Re: Does Kirchhoff's Law Hold? Disagreeing with a Master
Post by: EEVblog on February 14, 2019, 07:36:44 am
I also don't understand what Sredni was trying to accomplish by deleting content from his posts. Basically vandalizing his own work to make it harder for someone else to follow the tread.

Yep, I don't get it either, all that hard work gone, but we've seen this before on the forum and it usually doesn't end well unfortunately. I hope that doesn't end up being the case here.
Title: Re: Does Kirchhoff's Law Hold? Disagreeing with a Master
Post by: bsfeechannel on February 14, 2019, 09:00:56 am

....

You continue to think being an arrogant intellectual bully who time and again resorted to petty name calling and demeaning anyone not totally agreeing with you is a credible Scientific or Engineering method. On any level and as I mentioned a long while ago would not be tolerated in most workplaces or institutions of learning.

In this thread is some great reading but so much useless non Science and Engineering and OTT Ego driven non debate.

Play the science and engineering not play the man!

Wait a minute! Dave can say that he didn't follow the thread and that he doesn't respect Lewin based on mere impressions. I discussed the theme exhaustively, concluded, not decided, that Lewin deserves our respect and Mehdi doesn't and I am an arrogant intellectual bully?

Is that because I cared to study, read, read again, check, double check, watch Mehdi's videos more times than any of his own subscribes,  watch Lewins videos the same number of times, just to ascertain the truth? Just because I don't want to forward misconceptions?

Am I a bad guy?
Title: Re: Does Kirchhoff's Law Hold? Disagreeing with a Master
Post by: Siwastaja on February 14, 2019, 09:09:46 am
Is that because I cared to study, read, read again, check, double check, watch Mehdi's videos more times than any of his own subscribes,  watch Lewins videos the same number of times, just to ascertain the truth? Just because I don't want to forward misconceptions?

No, not at all. At least I hugely respect your work on studying, checking, and explaining; I believe others respect it as well.

All the namecalling (like the flat-earther shit), roasting inbetween is what people don't like. It's called "bullying". But I think you have been getting better during the thread. Try to keep on the subject, try to be less condescending, and you'll do fine. You are not superior. If you feel like others are really dumb, you are often not seeing something.

Social constructs can be difficult sometimes.
Title: Re: Does Kirchhoff's Law Hold? Disagreeing with a Master
Post by: Simon on February 14, 2019, 09:20:25 am
If you feel like others are really dumb, you are often not seeing something.

Social constructs can be difficult sometimes.


Depends, 2 minutes ago I interrupted a conversation between my manager and a colleague to point out that they are trying to talk about how to put holes in metal work for wiring before we have even agreed the wiring (that i will have to design). Am I superior? yes I am! The world is full of dumb people that call themselves engineers because they do drawings. Doing drawings and designing are two entirely different things.
Title: Re: Does Kirchhoff's Law Hold? Disagreeing with a Master
Post by: beanflying on February 14, 2019, 09:25:41 am
Wait a minute! Dave can say that he didn't follow the thread and that he doesn't respect Lewin based on mere impressions. I discussed the theme exhaustively, concluded, not decided, that Lewin deserves our respect and Mehdi doesn't and I am an arrogant intellectual bully?

Is that because I cared to study, read, read again, check, double check, watch Mehdi's videos more times than any of his own subscribes,  watch Lewins videos the same number of times, just to ascertain the truth? Just because I don't want to forward misconceptions?

Am I a bad guy?

For someone with intelligence you are WRONG to believe aggression and name calling adds to any debate, stick to the Science and Engineering.

Even Lewin managed to apologize for an earlier video retort to Mehdi and if this socially flawed human can manage that there may be hope for you too.
Title: Re: Does Kirchhoff's Law Hold? Disagreeing with a Master
Post by: EEVblog on February 14, 2019, 09:32:38 am

....

You continue to think being an arrogant intellectual bully who time and again resorted to petty name calling and demeaning anyone not totally agreeing with you is a credible Scientific or Engineering method. On any level and as I mentioned a long while ago would not be tolerated in most workplaces or institutions of learning.

In this thread is some great reading but so much useless non Science and Engineering and OTT Ego driven non debate.

Play the science and engineering not play the man!

Wait a minute! Dave can say that he didn't follow the thread and that he doesn't respect Lewin based on mere impressions. I discussed the theme exhaustively, concluded, not decided, that Lewin deserves our respect and Mehdi doesn't and I am an arrogant intellectual bully?

I didn't attack people, an entire channel's audience, and members on this forum.

Quote
Is that because I cared to study, read, read again, check, double check, watch Mehdi's videos more times than any of his own subscribes,  watch Lewins videos the same number of times, just to ascertain the truth?

Bingo.
You make it sound like all this is so plainly obvious, that it's all obviously settled, and "how dimwitted Mehdi's audience is for not noticing this", that Mehdi " do not deserve our respect and should not be addressed as engineers".
Yet here you are saying that it took you all this effort to figure it all out watching the video countless times, reading, re-reading, studying, double checking etc  ::)
And now we are simply take your word for it that this it's all settled and it's obvious Mehdi is completely wrong on every point etc, so much so that we shouldn't respect him as an engineer?
Sorry, but I'm going to play along with that.

This is not a trivial argument that even the most experienced and well educated engineers understand, in fact the argument has been going on decades.
Mehdi had every right to question Lewin's experiment, because without excruciatingly detailed investigation it seems like a dodgy setup to demonstrate his point.
And from what I can see there still seems to be debate on this forum about the exact applicability and implementation of the demonstration.
And from what I saw in Lewin's videos he didn't really even attempt to break down and explain clearly and simply why Mehdi is wrong on all his points. If he did then you and a couple of others wouldn't have  had to have spent, what, maybe a hundred or two man-hours trying to break this down and debate and explain it?
Title: Re: Does Kirchhoff's Law Hold? Disagreeing with a Master
Post by: ogden on February 14, 2019, 09:40:13 am
Depends, 2 minutes ago I interrupted a conversation between my manager and a colleague to point out that they are trying to talk about how to put holes in metal work for wiring before we have even agreed the wiring (that i will have to design). Am I superior? yes I am! The world is full of dumb people that call themselves engineers because they do drawings. Doing drawings and designing are two entirely different things.

It is a matter of perspective. *You* may think that knowledge of hole drilling/cutting/punching/whatever is needed *only* in case when *you* design wiring that requires holes. What if one of them wanted to show off his knowledge (to you or other guy). What if one of them thought - I know how it shall be done, let's tell it now disregarding he will need it or not. In both cases I would find discussion about holes reasonable and not dumb.
Title: Re: Does Kirchhoff's Law Hold? Disagreeing with a Master
Post by: Simon on February 14, 2019, 09:43:51 am
no it was a silly discussion about do we need to power 2 devices on one wire or separately. The electrical architecture has not yet been designed yet they are trying to finalise the metalwork :palm:
Title: Re: Does Kirchhoff's Law Hold? Disagreeing with a Master
Post by: Berni on February 14, 2019, 10:30:47 am
Kirchhoff's laws were meant to explain the underlying physics. Read Kirchhoff's original paper (https://gallica.bnf.fr/ark:/12148/bpt6k151490/f509.item) posted by seagreh. If you can't read German, cross the border and kindly ask an Austrian inhabitant to read it for you.

Quote
So if KVL is for the birds then the entirety of circuit analysis theory is for the bids as well. I'm sure any proper electronics engineer will disagree because circuit analysis has served them well ever since learning it in school.

Kirchhoff is for the birds means that Kirchhoff is not a fundamental law. Just that. Be prepared to see it being violated repeatedly. It just means that circuit analysis has its limits. It cannot be used for every kind of circuit. And you have to know when it is not applicable.

I bet SPICE didn't tell you that.

Quote
So please circuit analysis for explaining circuits and not physics.

Circuit analysis is physics.

Yes at the time it was the best explanation for this wonderful newfangled electricity thing so it was adopted as a theory of electricity. Just like we thought that light is continuous and that neutrons, protons and electrons are the most fundamental buildings blocks, today we know that's wrong, but 100 years ago we had no idea. Even today we can't know for sure if certain things we assume as fact are actually true.

Circuit theory is just a practical application of Physics. Much like Physics is a practical application of Math.



So you still haven't answered me what is the correct way to analyze the circuit in this post:
https://www.eevblog.com/forum/chat/does-kirchhoffs-law-hold-disagreeing-with-a-master/msg2189216/#msg2189216 (https://www.eevblog.com/forum/chat/does-kirchhoffs-law-hold-disagreeing-with-a-master/msg2189216/#msg2189216)

If we can't use circuit analysis, then what do we use to determine what it does?
Title: Re: Does Kirchhoff's Law Hold? Disagreeing with a Master
Post by: bsfeechannel on February 14, 2019, 07:43:07 pm
I didn't attack people, an entire channel's audience, and members on this forum.

What member of this forum am I attacking?

Anyway, the first time I heard the term "audiophool" was from you. I thought that those were tagged with that moniker because, as the name suggests, they aren't considered exactly smart people. Then I came across the "idiot assembler (sic) programmers" (EEVBlog#45). Perhaps that was just a joke that I missed, but then you compared the faith of religious people with the beliefs of the audiophools (EEVBlog#833 @38:05), which I totally agree. For the record I'm not a religious audiophile assembly programmer, but those examples encouraged me to give names to people who cannot, or do not want to, accept that their claims are bullshit, especially in light of scientific evidence. It is clear now that this move only proved that I'm wrong. I should have foreseen it. It was my fault. Sorry about that.

Quote
You make it sound like all this is so plainly obvious, that it's all obviously settled, and "how dimwitted Mehdi's audience is for not noticing this", that Mehdi " do not deserve our respect and should not be addressed as engineers".
Yet here you are saying that it took you all this effort to figure it all out watching the video countless times, reading, re-reading, studying, double checking etc  ::)

I make it sound obvious because I understood Lewin's answer as soon as he posted his first video in response to Mehdi's first video (the one you posted). However, when Mehdi posted his second video, implying that Lewin has wrong "beliefs" about electromagnetism because they were divergent from Mehdi's own wrong conclusions; asserting categorically that Kirchhoff holds for varying magnetic fields; and claiming that Feynman and Belcher agree with him, among countless other false claims, we had to go deeper into the subject and debunk point for point. Not to say about Mabilde's videos and other false claims from members of this forum. This only strengthened our understanding of Lewin's answer.

You obviously don't need to do all that research we did to solve Lewin's exercise.

Quote
And now we are simply take your word for it that this it's all settled and it's obvious Mehdi is completely wrong on every point etc, so much so that we shouldn't respect him as an engineer?

Sorry, but I'm going to play along with that.

You should never take my word for it. Go out and investigate for yourself. I learned those words with a famous video blogger which I really respect (exactly because of those words). Please feel free to check the video references, the references in Feynman's lectures. Draw your own conclusions. I'm just pointing things out (together with expressing my opinion, albeit not exactly popular, I can see it now).

Quote
This is not a trivial argument that even the most experienced and well educated engineers understand, in fact the argument has been going on decades.

This discussion was settled long ago. Lewin's experiment is in fact right. The "discussion" keeps going on because people insist that KVL holds for any circuit. But this is an illusion provided by the fact that Kirchhoff's laws, or better saying the modern version of it, can be applied to an awful lot of circuits under certain special conditions that we take for granted.

When this circumstances fail, Kirchhoff (KVL, KCL or both) is not valid anymore. You have to resort to Maxwell's equations. Lewin's is one of those circuits. Transformers, inductors, generators, motors, transmission lines, antennas, etc. can only be explained by the same approach.

Lewin's experiment is all about identifying exactly when and where Kirchhoff's laws fail. We have but to thank him eternally for that.

Quote
Mehdi had every right to question Lewin's experiment, because without excruciatingly detailed investigation it seems like a dodgy setup to demonstrate his point.

Mehdi has the right to question. But his second video is not about questioning. It is about decreeing his (lack of) understanding as a proof of his "theorem". He even ends his video writing the words Q.E.D.

We did what you recommended and didn't take his word for it. We went out and investigated for ourselves and the result was not pretty I'm afraid.

Quote
And from what I can see there still seems to be debate on this forum about the exact applicability and implementation of the demonstration.
And from what I saw in Lewin's videos he didn't really even attempt to break down and explain why Mehdi is wrong.

I think Lewin did. It is clear to me. But if you believe Lewin didn't, rest assured we did. It is exactly this thread. Please stay around. We perhaps have some more debunking to do, which will make it all very clear to you, I guess.

Although I must confess, I'm running out of time to contribute lately. I only decided to stop by because of the shocking news of Sredni's banning. At a certain point I thought of doing a video entitled "Debunking the debunkers: why Mehdi and Mabilde are wrong and Lewin is right." or something like that. But first I need to find time.
Title: Re: Does Kirchhoff's Law Hold? Disagreeing with a Master
Post by: bsfeechannel on February 14, 2019, 09:19:57 pm
Kirchhoff did not say

Σ Vk = 0
 
But he basically said, the sum of voltage drops (he expressed as product of resistance and current) equals to the sum of EMFs.

Absolutely correct.

Kirchhoff says:

Quote
[...]nennen wir [...] die elektromotorische Kraft, die ihren Sitz in der Berührungsstelle dieses und des folgenden Drahtes hat, Ki [...]

Let's call the electromotive force, that has its place between this [wire i] and the following wire, Ki.

Quote
[...] wenn die Drähte 1, 2, ...ν eine geschlossene Figur bilden,[...]

If the wires 1, 2, ... ν form a closed figure,

Quote
[...]so können wir diese Gleichung schreiben:

I11 + I22 + ... + Iνν = K1 + K2 ... + Kν

then we can write this equation:

I11 + I22 + ... + Iνν = K1 + K2 ... + Kν

Quote
[...]wo, ω1, ω2, ... die Widerstände der Drähte, I1, I2 ... die Intensitäten der Ströme bezeichnen[...]

where, ω1, ω2, ... designate the resistance of the wires, I1, I2 ... designate the current intensities.

There you have it. Straight from the horse's mouth.

Now let's take the open secondary of a transformer, for example, comprised of a single turn. This experiment is very easy to reproduce with a linear transformer (I did that using a MOT).

Since the loop is open the current in the wire is zero. The EMF is 1V (my MOT can give me 600mV for a single turn). Applying Kirchhoff's law using exactly the terminology of his seminal paper we have:

(https://www.eevblog.com/forum/chat/does-kirchhoffs-law-hold-disagreeing-with-a-master/?action=dlattach;attach=652383;image)

an absurd.
Title: Re: Does Kirchhoff's Law Hold? Disagreeing with a Master
Post by: ogden on February 14, 2019, 09:51:05 pm
Now let's take the open secondary of a transformer, for example, comprised of a single turn. This experiment is very easy to reproduce with a linear transformer (I did that using a MOT).

Since the loop is open the current in the wire is zero. The EMF is 1V (my MOT can give me 600mV for a single turn). Applying Kirchhoff's law using exactly the terminology of his seminal paper we have:

an absurd.

Absurd indeed. -  To apply Kirchoff's Circuit Law (Kirchoff's loop rule) to EMF source alone, w/o actual circuit or loop.  :palm:
Title: Re: Does Kirchhoff's Law Hold? Disagreeing with a Master
Post by: bsfeechannel on February 14, 2019, 10:50:28 pm

Absurd indeed. -  To apply Kirchoff's Circuit Law (Kirchoff's loop rule) to EMF source alone, w/o actual circuit or loop.  :palm:

Alright. Let's close the circuit.

Since there is no EMF between the point of contacts of the two wires (Kirchhoff is adamant about that) K1 + K2 ... + Kν = 0V. Then we have:

(https://www.eevblog.com/forum/chat/does-kirchhoffs-law-hold-disagreeing-with-a-master/?action=dlattach;attach=652413;image)

another absurd.

Title: Re: Does Kirchhoff's Law Hold? Disagreeing with a Master
Post by: ogden on February 14, 2019, 11:12:26 pm
another absurd.

You can't even apply Maxwell's equation correctly, use I*R instead. - That's absurd.
[edit] After all this time you did not learn that sum of all fields means sum of all types of the fields - conservative and non-conservative.
[edit] Also you do not seem to discern two voltages - voltage through the inductor and voltage across the inductor. One (I*R) is zero if R=0 but another is EMF, -L(di/dt) .
[edit] Correct equation is I1*R1+I2*R2+(-L(di/dt))=0.
Title: Re: Does Kirchhoff's Law Hold? Disagreeing with a Master
Post by: rfeecs on February 15, 2019, 01:01:12 am
The difference between a purely electrostatic field and one induced by a magnetic field is the non conservative part. This gives the field ability to push electrons around closed loops of conductor while a electrostatic field can only redistribute them but not sustain a current apart from the very brief transient as they redistribute.

Since the inner conductor forms a continuous loop trough the voltmeter, it means that the the field can push the electrons around it and create a current that the voltmeter detects as voltage across its internal resistance. The shield however does not form a continuous loop and as such can't experience any current trough it.

I don't see why you would need current to shield a static electric field.

Quote
...My point is that the coax shield in this configuration has no effect.

I'm not convinced.  I expect the coax will shield the center conductor from the induced electric field.  So you would measure the same voltage whether the scope was on the left or the right side.

OK, I'm convinced.  I tried it.  The coax doesn't do a damn thing.

You are right.

My new explanation, pretty much the same as yours, is the induced field component that causes the current is parallel to the wire.  Putting a coaxial shield parallel to the field does nothing.

So even though there is no magnetic field in the region of the test leads, only the induced electric field, you can't shield them with only a conductive shield.
Title: Re: Does Kirchhoff's Law Hold? Disagreeing with a Master
Post by: bsfeechannel on February 15, 2019, 02:18:00 am
[edit] Correct equation is I1*R1+I2*R2+(-L(di/dt))=0.

Since i is constant in my example di/dt is zero  (or next to it if you consider a very low current at a low frequency).

So  I1*R1  +  I2*R2 =0, 1V = 0V !!.  The absurd remains the same.

But today is your lucky day. I'll show you how Kirchhoff fails once more. There are infinite ways to show that Kirchhoff doesn't hold. I could do this the whole day. It's a shame that I am not paid for that. Perhaps I'll open a Patreon account and ask people to help me debunk this pseudo scientific claim.

Anyway, in the circuit below I connect a battery, but if you don't like it, you can connect a generator, that produces the same voltage of the EMF. Of course the current is going to be zero. See what happens.

(https://www.eevblog.com/forum/chat/does-kirchhoffs-law-hold-disagreeing-with-a-master/?action=dlattach;attach=652575;image)

Faraday 3 x Kirchhoff 0


Title: Re: Does Kirchhoff's Law Hold? Disagreeing with a Master
Post by: Berni on February 15, 2019, 06:19:19 am
OK, I'm convinced.  I tried it.  The coax doesn't do a damn thing.

You are right.

My new explanation, pretty much the same as yours, is the induced field component that causes the current is parallel to the wire.  Putting a coaxial shield parallel to the field does nothing.

So even though there is no magnetic field in the region of the test leads, only the induced electric field, you can't shield them with only a conductive shield.

Awesome to see the experiment done for real. :-+

To be fair if i was asked the same question at the very beginning of the tread i would have likely expected the coax to shield it too, that's what the job of the shield on a coax is after all. But magnetic fields can be sneaky sometimes.




[edit] Correct equation is I1*R1+I2*R2+(-L(di/dt))=0.

Since i is constant in my example di/dt is zero  (or next to it if you consider a very low current at a low frequency).

So  I1*R1  +  I2*R2 =0, 1V = 0V !!.  The absurd remains the same.

But today is your lucky day. I'll show you how Kirchhoff fails once more. There are infinite ways to show that Kirchhoff doesn't hold. I could do this the whole day. It's a shame that I am not paid for that. Perhaps I'll open a Patreon account and ask people to help me debunk this pseudo scientific claim.

Anyway, in the circuit below I connect a battery, but if you don't like it, you can connect a generator, that produces the same voltage of the EMF. Of course the current is going to be zero. See what happens.

(https://www.eevblog.com/forum/chat/does-kirchhoffs-law-hold-disagreeing-with-a-master/?action=dlattach;attach=652575;image)

Faraday 3 x Kirchhoff 0

But you can still get 0V or 1V depending on what path you take trough the circuit in the case of Maxwell. So you can just as well turn it around and show the opposite by always choosing the voltage that does agree to KVL. So is then KVL in a quantum superposition of being dead and alive at the same time?

We all know you can't use KVL in varying magnetic fields, so why do you keep trying to prove it? My claim is that it always works in LUMPED circuit meshes.

So if you want KVL to work then lump model the whole thing and it will work fine, mixing lumped and non lumped models wont work.
Title: Re: Does Kirchhoff's Law Hold? Disagreeing with a Master
Post by: ogden on February 15, 2019, 08:44:07 am
[edit] Correct equation is I1*R1+I2*R2+(-L(di/dt))=0.

Since i is constant in my example di/dt is zero  (or next to it if you consider a very low current at a low frequency).

So  I1*R1  +  I2*R2 =0, 1V = 0V !!.  The absurd remains the same.

You do not seem to comprehend momentary current as well.  :palm:

di is zero *only* at very beginning of the pulse. After some time elapsed meaning dt is not zero anymore, di is not zero as well.

Before you talk about Maxwell equations, make sure you understand elementary physics.

[edit] -L(di/dt) is Maxwell's equation. We agreed that result (EMF) is 1V long ago. So what you think about di is irrelevant, you simply put -1V in the equation: 0.1V + 0.9V - 1V = 0. KVL holds.
Title: Re: Does Kirchhoff's Law Hold? Disagreeing with a Master
Post by: bsfeechannel on February 15, 2019, 02:54:29 pm
So is then KVL in a quantum superposition of being dead and alive at the same time?

If we choose the same path, we'll get the same voltage. Maxwell/Kirchhoff is still classical physics. Ironically in their time the seeds for quantum mechanics were already planted by no one less than Kirchhoff himself with his study of the black-body radiation. Max Planck was one of his students and the rest is history.

Quote
We all know you can't use KVL in varying magnetic fields, so why do you keep trying to prove it?

I did that to reply to a post that claimed that Kirchhoff didn't say that Σ Vk = 0, but that the sum of voltage drops, expressed as the product of resistances and currents, equals to the sum of EMFs.

I proved that, even if you use Kirchhoff's terminology, instead of our modern, KVL doesn't hold for varying magnetic fields. Kirchhoff's version of his own law implies certain assumptions and conventions (explicitly stated in his thesis) that render the same result.

But then, ogden intervened, which is always amusing and didactic I must say.
Title: Re: Does Kirchhoff's Law Hold? Disagreeing with a Master
Post by: bsfeechannel on February 15, 2019, 05:14:15 pm
you simply put -1V in the equation: 0.1V + 0.9V - 1V = 0.

You put -1V there and explicitly violate Kirchhoff' law.

Quote
Before you talk about Maxwell equations, make sure you understand elementary physics.

Learning German and reading Kirchhoff's original paper also helps.
Title: Re: Does Kirchhoff's Law Hold? Disagreeing with a Master
Post by: ogden on February 15, 2019, 06:35:27 pm
you simply put -1V in the equation: 0.1V + 0.9V - 1V = 0.

You put -1V there and explicitly violates Kirchhoff' law.

You mistakenly think so because as you already demonstrated, you do not comprehend what di/dt and "current at the time of observation" means. I am not sure that you fully understand conservative and non-conservative fields as well. Any clue that they differ and you have to take in account all fields? Your original formula "I1*R1+I2*R2=0" did not include EMF, so I corrected it: I1*R1+I2*R2+(-L(di/dt))=0.
Title: Re: Does Kirchhoff's Law Hold? Disagreeing with a Master
Post by: bsfeechannel on February 16, 2019, 07:41:55 pm
You mistakenly think so because as you already demonstrated, you do not comprehend what di/dt and "current at the time of observation" means.

Well, I may not understand a lot of things, but you surely don't know the difference between an inductor and a generator. Please, read this (http://www.feynmanlectures.caltech.edu/II_22.html).

Quote
I am not sure that you fully understand conservative and non-conservative fields as well.

You could say that if there were a conservative magnetic field there.

Quote
Any clue that they differ and you have to take in account all fields? Your original formula "I1*R1+I2*R2=0" did not include EMF, so I corrected it: I1*R1+I2*R2+(-L(di/dt))=0.

Although it would be nice to take credit for Kirchhoff's work and say that the formula is mine, we'd better not only attribute it to Kirchhoff but also not try to surreptitiously and clandestinely sneak "corrections" into it.
Title: Re: Does Kirchhoff's Law Hold? Disagreeing with a Master
Post by: bsfeechannel on February 17, 2019, 12:42:53 am
A little rant to feed our thoughts.

No, not at all. At least I hugely respect your work on studying, checking, and explaining; I believe others respect it as well.

All the namecalling (like the flat-earther shit), roasting inbetween is what people don't like. It's called "bullying". But I think you have been getting better during the thread. Try to keep on the subject, try to be less condescending, and you'll do fine. You are not superior. If you feel like others are really dumb, you are often not seeing something.

Social constructs can be difficult sometimes.

Dave has an interesting recent video about how to become an engineer. He pointed out, and people confirmed in the discussions, that what is considered an engineer varies from country to country due to culture, legislation, economical environment, etc.

To me, an engineer professing pseudo-scientific claims (which is different from questioning), or advocating deficient knowledge as an advantage, especially at the expense of the reputation of another professional is morally appalling.

Besides, I am an old fart. While we already had SPICE around, back in my day we were taught to use our brains.

Simulation software in the hands of an engineer who does not master the fundamentals is the recipe for disaster, my experience showed.

So, the "flat-earther shit" is the result of being repeatedly exposed to what I consider an aggression to my standards and is not an expression of condescension, but of exasperation, which is not a good feeling.


Even Lewin managed to apologize for an earlier video retort to Mehdi and if this socially flawed human can manage that there may be hope for you too.

This is a total inversion of values. The savvy have to apologize to the ignorant for being savvy. Almost makes you discourage to seek knowledge.

Dr. House has a politically incorrect quote: "Yesterday's sluts are today's empowered women. Today's sluts are celebrities."

The ignorant of today are celebrities.
Title: Re: Does Kirchhoff's Law Hold? Disagreeing with a Master
Post by: unitedatoms on February 17, 2019, 02:19:43 am
Great video by Electroboom. Lewin is mixing models in wrong way.

In my humble opinion, The circuit laws apply only to pure sizeless graphs of lumped sizeless components connected with sizeless lossless equipotential wires conducting with infinite speed. Whole modeled circuit has total volume of zero cubic microns and completely electroneutral and disconnected from rest of universe. No fields of any kind are present.
Title: Re: Does Kirchhoff's Law Hold? Disagreeing with a Master
Post by: beanflying on February 17, 2019, 02:51:20 am
A little rant to feed our thoughts.
....

A load of self indulgent WAFFLE to justify your ongoing poor behavior toward others. Lewin rightly apologized for the personal attack on Mehdi as regards his background and training and rightly not for what he sees as errors of the technicalities as is appropriate. He certainly never ranted te word 'Liar' as you have done and went about explaining what he thought was incorrect.

You clearly and incorrectly draw no distinction between those two things. On multiple ocasions at a minimum you have told members to 'get an education' which is simple Baiting, Bullying or being simply demeaning to them and not to their position on the subject under discussion.

Are you a 'Bad Person' - others can draw their own conclusions.
Title: Re: Does Kirchhoff's Law Hold? Disagreeing with a Master
Post by: jesuscf on February 17, 2019, 05:20:04 am
Great video by Electroboom. Lewin is mixing models in wrong way.

In my humble opinion, The circuit laws apply only to pure sizeless graphs of lumped sizeless components connected with sizeless lossless equipotential wires conducting with infinite speed. Whole modeled circuit has total volume of zero cubic microns and completely electroneutral and disconnected from rest of universe. No fields of any kind are present.

What about transmission lines?
Title: Re: Does Kirchhoff's Law Hold? Disagreeing with a Master
Post by: unitedatoms on February 17, 2019, 05:39:13 am
What about transmission lines?

The transmission lines are OK to consider sizeless too. They are not special case. Absolutism of model requires everything to be a net component, but does not limit complexity. So lines are infinitely fragmented chains of LCRG parts with characteristic values expressed in Ohms, Farads, Henries and Siemenses like the rest of simple components.

One may notice how slow Spice becomes when it has line in the model.
Title: Re: Does Kirchhoff's Law Hold? Disagreeing with a Master
Post by: EEVblog on February 17, 2019, 05:57:32 am
To me, an engineer professing pseudo-scientific claims (which is different from questioning), or advocating deficient knowledge as an advantage, especially at the expense of the reputation of another professional is morally appalling.

Who the hell is advocating for "deficient knowledge" as being an advantage?

This is a total inversion of values. The savvy have to apologize to the ignorant for being savvy. Almost makes you discourage to seek knowledge.

You don't have to apologise for anything, no one has a gun to your head demanding that. But people are free to think of you and the way you express yourself in any way they like, including having a not so good opinion of you.
Title: Re: Does Kirchhoff's Law Hold? Disagreeing with a Master
Post by: EEVblog on February 17, 2019, 06:08:42 am
You should never take my word for it. Go out and investigate for yourself. I learned those words with a famous video blogger which I really respect (exactly because of those words). Please feel free to check the video references, the references in Feynman's lectures. Draw your own conclusions. I'm just pointing things out (together with expressing my opinion, albeit not exactly popular, I can see it now).

You're damn right it's not popular.
When you say that Mehdi "should not be addressed as (an) engineer", you aren't going to win many friends on here.
Not to mention calling Mehdi's audience "dimwitted"
Title: Re: Does Kirchhoff's Law Hold? Disagreeing with a Master
Post by: Berni on February 17, 2019, 11:49:38 am
What about transmission lines?

Yep a transmission line is what is used to model that travel time in lumped circuits. The actual implementation of a transmission line probably varies a bit, but for simulators you usually specify a characteristic impedance and propagation delay.


And yeah SPICE does not do any of the thinking for you. Its the users responsibility for giving it a accurate circuit model so you still need to know what you are doing. So why would spice be bad? Its just a programatical implementation of existing circuit analysis theory.
Title: Re: Does Kirchhoff's Law Hold? Disagreeing with a Master
Post by: jesuscf on February 17, 2019, 04:24:33 pm
What about transmission lines?

Yep a transmission line is what is used to model that travel time in lumped circuits. The actual implementation of a transmission line probably varies a bit, but for simulators you usually specify a characteristic impedance and propagation delay.


And yeah SPICE does not do any of the thinking for you. Its the users responsibility for giving it a accurate circuit model so you still need to know what you are doing. So why would spice be bad? Its just a programatical implementation of existing circuit analysis theory.

Thanks Berni, I am well aware on how transmission lines are modeled and simulated.  I was being a bit sarcastic when I posted my comment, but 'unitedatoms' was right on with his reply.   By the way, the SPICE model for transmission lines is rather simple and doesn't include coupling, like the kind of coupling present in three phase power lines (or at least it didn't a few years back).   But according to Dr. Lewin and his supporters the laws of Circuit Theory will not apply to an arrangement of wires like that.  Gladly, electrical engineers, despite their 'ignorance of Maxwell equations and laws of physics in general', had managed to model power transmission lines with great detail and use them in circuit simulators like EMTP (Electromagnetic's Transient Program).

 
Title: Re: Does Kirchhoff's Law Hold? Disagreeing with a Master
Post by: unitedatoms on February 17, 2019, 04:44:17 pm
circuit simulators like EMTP (Electromagnetic's Transient Program).

What they do with magnetism in EMTP? Is EMTP sircuit based or space based ? (Circuit vs space like in Spice vs Autocad 3D). I heard of mechanical modeling approach Finite Elements Modeling, is it somehow similar to spice in mechanics.

What if someone modelled free space and metal surfaces as net lump components, then you can Spice model RF shapes of any kind. Dont know if is possible.
Title: Re: Does Kirchhoff's Law Hold? Disagreeing with a Master
Post by: Berni on February 17, 2019, 07:09:19 pm
Well lump modeling is used quite a bit in RF too.

The EM simulations of distributed element filters on PCBs or 3D filter structures can get very slow. So often the results of those simulations are converted into a lumped equivalent circuit that can then be used to run simulations quickly in efficiently with tools like SPICE while still behaving accurately.

Alternatively given that you already have a physical component and want to include it into a simulator then you typically hook it up to a network analyzer and measure all of its S parameters over the frequency range of interest. This data can then be used in a simulation model or calculated into a equivalent circuit of the component.

Tho these sorts of equivalent circuits can become quite a bit mess of resistors inductors and capacitors and very few nodes in that cirucit have any actual meaning to the real life cirucit. Most voltages and currents in these big complex lump models are just partial math results of something, its only the voltage across the terminals of the whole black box is the one that actually matches a real voltage in the real cirucit.

For example here is the lumped model for a Ethernet transformer made by TDK:
(https://www.eevblog.com/forum/chat/does-kirchhoffs-law-hold-disagreeing-with-a-master/?action=dlattach;attach=655296;image)

This model was likely constructed from a large number of network analyzer measurements of the component, but will give very accurate behavior in a simulation to give you accurate imperfect coupling of the coils and accurate crosstalk between terminals. All while being simulated in under a second inside SPICE as opposed to being simulated for hours in a EM simulator.

Its all about engineers being efficient with there time because they have to deal with things called deadlines. Not because they wouldn't care about our old friend Maxwell, they do care, but will take a shortcut if the shortcut gives results that are just as good.

Title: Re: Does Kirchhoff's Law Hold? Disagreeing with a Master
Post by: ogden on February 17, 2019, 11:13:50 pm
You mistakenly think so because as you already demonstrated, you do not comprehend what di/dt and "current at the time of observation" means.

Well, I may not understand a lot of things, but you surely don't know the difference between an inductor and a generator. Please, read this (http://www.feynmanlectures.caltech.edu/II_22.html).

Oh, you think that transformer (inductor with two windings) is generator?  :palm:

Quote
Although it would be nice to take credit for Kirchhoff's work and say that the formula is mine, we'd better not only attribute it to Kirchhoff but also not try to surreptitiously and clandestinely sneak "corrections" into it.

Perpetum mobile formula of voltage drop on loads that does not include voltage source (EMF) is truly yours. Do not disgrace Kirchoff, please.

[edit] Image of "your formula" attached. You wrote I1*R1+I2*R2=0V
Title: Re: Does Kirchhoff's Law Hold? Disagreeing with a Master
Post by: bsfeechannel on February 18, 2019, 04:36:56 am
Who the hell is advocating for "deficient knowledge" as being an advantage?

Mehdi and everyone that advocates that knowledge of the Maxwell's equations can be replaced by the "much simpler" Kirchhoff's equations.

This thread has proved that if you don't understand the underlying physics of electromagnetism you'll be limited in your ability to design and analyze circuits.

Quote
You don't have to apologise for anything, no one has a gun to your head demanding that. But people are free to think of you and the way you express yourself in any way they like, including having a not so good opinion of you.

When you say that Mehdi "should not be addressed as (an) engineer", you aren't going to win many friends on here.
Not to mention calling Mehdi's audience "dimwitted"

Fair enough. I'm here because of your invitation to "anyone involved in electronics design" and because "This is where everyone hangs out and rants and chats about whatever electronics topics that don't fit into the other more specific categories on this forum."

So, here I am ranting like crazy about what I find to be wrong with the world when it comes to electronics. If I don't win friends or influence people (I have the book), at least I hope someone can learn something with it.
Title: Re: Does Kirchhoff's Law Hold? Disagreeing with a Master
Post by: unitedatoms on February 18, 2019, 04:59:50 am
@beefefees
You said you wrote the book ? What was the subject? I did not follow the drama, but it looks for me like the discussion about pedagogy. Not the scientific matters. Please do not be discouraged. The only problem I have observed was that there is good and bad way to deliver the knowledge. Quality of explanation matters. Students shall not be made confused.

I will try to fix this with my little essay:

Once upon a time there in a little hole in a ground there was an Ohm. Nearby in a village lived his fellow neighbors
Volt, Ampere, Siemens, Coulomb and Second.
Children we're named Henry and Farad.
Everyone was happy, and that time we all remember as the time of zero's Planks Epoch. That was a Kirghoffs Universe. The village was ruled by a couple of Zero and Infinity, and their single child was named One.
However one day the the One said I want a sibling, and if you will not give me a sibling named Imaginary One!
Thus was born the Imaginary One. And things turn Complex. Everyone was suspecting that real father was Second. And it was only a beginning.
Soon the epoch ended, because Second was feeling insecure and sneaked into other village and became friend to evil neighbours.
Nobody was expecting the change, but change happened. An army of evil intruders attacked the village. They turned everything
Into fragments, and ever since the the things were apart. Newcomers named Meter and Kilogramm were were the eveliest of all. They
created Space, and all 3 directions to confuse the innocent villagers. They even evicted the innocents and forced them live in Fields.
Fields were ruled by Space, the evil child of Meter and Kilogramm. And that was the start of second Plank's Epoch.

The poor villagers had no choice, but to tp follow new rules. They we gathering crops using Differentials and the produce was
Stocked in large towers named Integrals. Second was promoted to be in charge of accounting and bookkeeping.
She gathered produce and reported to Joule. Joule was old neutral neighbor, but he took an advantage of all the politics.

One day Joule became enamored with Second, and they got achild named Watt. Watt Wass grew up as unremarcable fellow.
All he was caring about was efficiency, typical MBA major kind. However he was decent enough to stay away from Weber, Tesla and alike.
What Weber and Tesla did, is they built one round fence around the village. The Wall. Weber and Tesla wanted to keep
The crop safe from elements. This wall caused so may conflicting feels in a village.
However Volt was most encouraged by wall, and he volunteered to patrol it. Second supported Volt in his intitatve
and she pushed the village comitee to add Ampere and Ohm to the patrolling schedule. Weber and Tesla silently celebrated.

The story to (be unlikely) continue... Sorry too much red wine this Sunday


Title: Re: Does Kirchhoff's Law Hold? Disagreeing with a Master
Post by: Berni on February 18, 2019, 06:11:47 am
Don't think i learned much from that, but i did find the story pretty amusing, it actually fits together rather well.
Title: Re: Does Kirchhoff's Law Hold? Disagreeing with a Master
Post by: bsfeechannel on February 18, 2019, 06:41:31 am
Well lump modeling is used quite a bit in RF too.

The EM simulations of distributed element filters on PCBs or 3D filter structures can get very slow. So often the results of those simulations are converted into a lumped equivalent circuit that can then be used to run simulations quickly in efficiently with tools like SPICE while still behaving accurately.

Of course we can lump model RF elements. Otherwise you couldn't replace an antenna with a signal generator to test your receiver in the lab. You can treat propagation through the air as a big attenuator if you have a direct line of sight and no obstacle in the Fresnel zone, etc., etc. etc People who design radios do that all the time in the lab.

What is necessary to understand is when this trick fails. And it fails because you're not aware that you are making implicit assumptions when you lump model a circuit. I said this I guess three months ago or more.

Quote
Tho these sorts of equivalent circuits can become quite a bit mess of resistors inductors and capacitors and very few nodes in that cirucit have any actual meaning to the real life cirucit.

They do have an actual meaning. They are approximations. There are in fact capacitances between the wires and between the windings. The wires are really resistive. The leakage inductance is really an inductance that is coupled to the no other winding. Part of the energy is really dissipated as heat due to eddy currents and hysteresis.

They are not palpable but can be measured, as your picture shows.

Quote
Most voltages and currents in these big complex lump models are just partial math results of something, its only the voltage across the terminals of the whole black box is the one that actually matches a real voltage in the real cirucit.

If I give you a black box with two terminals hanging out and you perform the following test, you'll be easily able to build yourself a gadget that reproduces the exact effect with lumped components.

(https://www.eevblog.com/forum/chat/does-kirchhoffs-law-hold-disagreeing-with-a-master/?action=dlattach;attach=655911;image)

But if I give you another black box where you cannot determine exactly what voltage and what current it delivers it will be impossible to build it with lumped components.

(https://www.eevblog.com/forum/chat/does-kirchhoffs-law-hold-disagreeing-with-a-master/?action=dlattach;attach=655917;image)

I can obviously restrict how to take the voltage of this crazy black box and then I can try to lump model it. But hey, look what I'm doing, I'm assuming something: that the voltage can only be taken in a certain way.

(https://www.eevblog.com/forum/chat/does-kirchhoffs-law-hold-disagreeing-with-a-master/?action=dlattach;attach=655923;image)

Look how the technique can be confused with the phenomenon. This limits the understanding of a lot of circuits. Including some very ubiquitous circuits that you have around you.

Quote
Its all about engineers being efficient with there time because they have to deal with things called deadlines.

It's all about engineers knowing what they're doing. If you don't know what you're doing will never be efficient and will never meet any deadlines.
Title: Re: Does Kirchhoff's Law Hold? Disagreeing with a Master
Post by: EEVblog on February 18, 2019, 06:51:21 am
@beefefees
You said you wrote the book ? What was the subject? I did not follow the drama, but it looks for me like the discussion about pedagogy. Not the scientific matters. Please do not be discouraged. The only problem I have observed was that there is good and bad way to deliver the knowledge. Quality of explanation matters. Students shall not be made confused.

Completely unavoidable, confusion is inevitable with any explanation. This is why dozens of text books explain the same thing in different ways, one way "clicks" for one person but doesn't click for another.
Title: Re: Does Kirchhoff's Law Hold? Disagreeing with a Master
Post by: EEVblog on February 18, 2019, 06:59:26 am
Fair enough. I'm here because of your invitation to "anyone involved in electronics design" and because "This is where everyone hangs out and rants and chats about whatever electronics topics that don't fit into the other more specific categories on this forum."
So, here I am ranting like crazy about what I find to be wrong with the world when it comes to electronics. If I don't win friends or influence people (I have the book), at least I hope someone can learn something with it.

People are learning, and technical input like yours is always massively appreciated.
But unfortunately your approach has been somewhat abrasive (even for an engineering forum, were lack of tact and copious amounts of abrasion are commonplace) and I'm not surprised it's rubbed some people the wrong way. That was just inevitable with your approach.
Title: Re: Does Kirchhoff's Law Hold? Disagreeing with a Master
Post by: jesuscf on February 18, 2019, 07:00:04 am
This thread has proved that if you don't understand the underlying physics of electromagnetism you'll be limited in your ability to design and analyze circuits.

And this sentence proves without doubt, that bsfeechannel is not an electrical/electronics engineer or anything remotely similar.  Here you have it, the Dunning–Kruger effect at work.  It reminds me of Dr. Lewin solving his problem 34 :palm:
Title: Re: Does Kirchhoff's Law Hold? Disagreeing with a Master
Post by: bsfeechannel on February 18, 2019, 07:04:05 am
Oh, you think that transformer (inductor with two windings) is generator?  :palm:

Where did you find a transformer in the circuit? There are only two wires connected to each other forming a closed figure. Exactly as Kirchhoff said it had to be.

Quote
Perpetum mobile formula of voltage drop on loads that does not include voltage source (EMF) is truly yours.

There is no voltage source in the circuit. Only two resistive wires.

Quote
Do not disgrace Kirchoff, please.

Absolutely. I'd never introduce something to Kirchhoff's formula that he has not authorized me to.
Title: Re: Does Kirchhoff's Law Hold? Disagreeing with a Master
Post by: Berni on February 18, 2019, 07:26:13 am
Quote
Its all about engineers being efficient with there time because they have to deal with things called deadlines.

It's all about engineers knowing what they're doing. If you don't know what you're doing will never be efficient and will never meet any deadlines.

Exactly. As i said the spice simulator does not do any of the thinking for you, it just munches trough the math for you. Hence why in the very first page of this long thread has my spice model that includes coupled inductors added. Observing the physical circuit i have determined that the effect of coupled inductance is a significant effect to the behavior, so i have put them in and ran the simulation to see how well it does. It behaved just like the real circuit so from that i assumed that i have captured all the most significant parasitics of the circuit. Its all about knowing your tools.

Just because there is an inductor in the spice model doesn't mean you have a physical coil of wire, or that all of this voltage appears at one point on a wire. It just tells the circuit model that this path trough the circuit experiences inductance, or in this case coupled inductance.

The black box you show up there is not really a black box as they should behave as a self contained unit that only interacts with its surrounding trough the terminals. This also means a black box works with circuit mesh analysis without making any spooky paradoxes.
Title: Re: Does Kirchhoff's Law Hold? Disagreeing with a Master
Post by: bsfeechannel on February 18, 2019, 07:37:31 am
People are learning, and technical input like yours is always massively appreciated.
But unfortunately your approach has been somewhat abrasive (even for an engineering forum, were lack of tact and copious amounts of abrasion are commonplace) and I'm not surprised it's rubbed some people the wrong way. That was just inevitable with your approach.

I'm really surprised to read that. I thought I was in my element here.

But look what I have to deal with.

This thread has proved that if you don't understand the underlying physics of electromagnetism you'll be limited in your ability to design and analyze circuits.

And this sentence proves without doubt, that bsfeechannel is not an electrical/electronics engineer or anything remotely similar.  Here you have it, the Dunning–Kruger effect at work.  It reminds me of Dr. Lewin solving his problem 34 :palm:


I don't think I have said something abrasive here. How do I reply to that?

This is not the first time this guy says this about me and he did it long before I said that Mehdi shouldn't be addressed as an engineer. He is not even molested by the moderation and he'll probably be even thanked by it.

So I'm really at a loss as to what are the acceptable limits for "abrasion" in this forum.  :-//

I like to contribute, but it is really frustrating to be mocked for that when guys like this are free to troll at will.
Title: Re: Does Kirchhoff's Law Hold? Disagreeing with a Master
Post by: EEVblog on February 18, 2019, 07:53:46 am
But look what I have to deal with.
This thread has proved that if you don't understand the underlying physics of electromagnetism you'll be limited in your ability to design and analyze circuits.
And this sentence proves without doubt, that bsfeechannel is not an electrical/electronics engineer or anything remotely similar.  Here you have it, the Dunning–Kruger effect at work.  It reminds me of Dr. Lewin solving his problem 34 :palm:

I don't think I have said something abrasive here. How do I reply to that?

Simple, you don't. You ignore it and keep the talk technical.

Quote
This is not the first time this guy says this about me and he did it long before I said that Mehdi shouldn't be addressed as an engineer. He is not even molested by the moderation and he'll probably be even thanked by it.

Moderators do not actually "moderate" conversation here, we work based on reports and then we might look at the person history.
We get countless "storms in a teacup" that could be easily avoided if the person simply doesn't respond to them.

Quote
So I'm really at a loss as to what are the acceptable limits for "abrasion" in this forum.  :-//

Start by not calling people all sorts of things and taking things too personally.
Hint, you can't be "abrasive" with just facts and technical points. But you can be with your tone and pejoratives.

IMO it's you taking this too personally that is the problem. We have seen this countless times on the forum and often it leads to rage quitting like this other poster in here seems to have done, and that's a huge shame.
Quote
I like to contribute, but it is really frustrating to be mocked for that when guys like this are free to troll at will.

You are free to ignore them, try it, it works wonders.
Title: Re: Does Kirchhoff's Law Hold? Disagreeing with a Master
Post by: jesuscf on February 18, 2019, 08:45:45 am
This thread has proved that if you don't understand the underlying physics of electromagnetism you'll be limited in your ability to design and analyze circuits.
And this sentence proves without doubt, that bsfeechannel is not an electrical/electronics engineer or anything remotely similar.  Here you have it, the Dunning–Kruger effect at work.  It reminds me of Dr. Lewin solving his problem 34 :palm:

I don't think I have said something abrasive here. How do I reply to that?

This is not the first time this guy says this about me and he did it long before I said that Mehdi shouldn't be addressed as an engineer. He is not even molested by the moderation and he'll probably be even thanked by it.

So I'm really at a loss as to what are the acceptable limits for "abrasion" in this forum.  :-//

I like to contribute, but it is really frustrating to be mocked for that when guys like this are free to troll at will.

Dear bsfeechannel I offer my most sincere apologies.  I'll very careful not to say something like this again.  One thing you can do is adding me to your "ignore list".


Title: Re: Does Kirchhoff's Law Hold? Disagreeing with a Master
Post by: beanflying on February 18, 2019, 09:06:56 am
...

I like to contribute, but it is really frustrating to be mocked for that when guys like this are free to troll at will.

So we should all just be quiet and let you go on attacking anyone you feel like on a 'personal' level? I tried to raise this politely back is December and I got Waffle back from you about 'protecting lives' to mask your attitude toward others and not to their opinions.

You 'claim' the capacity to learn but have failed miserably to manage that and comprehend the problem that others have with your so called debate and discussion methodology.


You better behave

Here's a dollar, kid. Go get yourself a better education.

or this


His video is a crime against humanity.

Who is being arrogant, after all? Mabilde and all those who recalcitrantly refuse to learn, or Lewin who dedicated an entire life to teaching?

This stubborn attitude is what is getting under our skin.

So don't fool yourself. You're not doing science a favor. If you really love science do as we all do: humbly learn.

And I won't bother repeating you @BSChannel rant of a few days ago where you used the word Liar on multiple times toward Mehdi.

Perhaps it is you who may just be getting under our skin just a little?


So, never try anymore to hide your lack of knowledge behind excuses like that. Convince yourself and others of the need to be ready to learn something new every day.

Title: Re: Does Kirchhoff's Law Hold? Disagreeing with a Master
Post by: ogden on February 18, 2019, 09:30:18 am
Oh, you think that transformer (inductor with two windings) is generator?  :palm:

Where did you find a transformer in the circuit? There are only two wires connected to each other forming a closed figure. Exactly as Kirchhoff said it had to be.

You said it is transformer:

Now let's take the open secondary of a transformer, for example, comprised of a single turn.

Quote
Quote
Perpetum mobile formula of voltage drop on loads that does not include voltage source (EMF) is truly yours.

There is no voltage source in the circuit. Only two resistive wires.

You are making your own reality on the go:

The EMF is 1V (my MOT can give me 600mV for a single turn).
Title: Re: Does Kirchhoff's Law Hold? Disagreeing with a Master
Post by: bsfeechannel on February 18, 2019, 10:28:58 am
You said it is transformer:

Now let's take the open secondary of a transformer, for example, comprised of a single turn.

I said the secondary of a transformer. The secondary of a transformer is not a transformer. But if that confuses you, instead of thinking that the varying field is being generated by the primary winding of a transformer, think of it being generated by a moving permanent magnet. It doesn't make any difference as stated in Feynman's lecture.

Quote
You are making your own reality on the go:
The EMF is 1V (my MOT can give me 600mV for a single turn).

Nope. This is a different circuit. It only has one wire and one EMF. The other one has two wires and no EMF where Kirchhoff said it should be.
Title: Re: Does Kirchhoff's Law Hold? Disagreeing with a Master
Post by: Berni on February 18, 2019, 10:53:31 am
Nope. This is a different circuit. It only has one wire and one EMF. The other one has two wires and no EMF where Kirchhoff said it should be.

Why would KVL say there is no EMF?
See this equation: http://www.feynmanlectures.caltech.edu/II_22.html#mjx-eqn-EqII2234 (http://www.feynmanlectures.caltech.edu/II_22.html#mjx-eqn-EqII2234)
Title: Re: Does Kirchhoff's Law Hold? Disagreeing with a Master
Post by: bsfeechannel on February 18, 2019, 04:51:01 pm
Nope. This is a different circuit. It only has one wire and one EMF. The other one has two wires and no EMF where Kirchhoff said it should be.

Why would KVL say there is no EMF?
See this equation: http://www.feynmanlectures.caltech.edu/II_22.html#mjx-eqn-EqII2234 (http://www.feynmanlectures.caltech.edu/II_22.html#mjx-eqn-EqII2234)

Those equations imply some important assumptions. The wires are ideal and there's no varying magnetic fields between the terminals of the mutually coupled inductors. The terminals, i.e. the interface between Kirchhoff and Maxwell, are very important.  The word terminal occurs 81 times in this chapter. Try reading every paragraph where it is included.

To help you a bit with that, let me summarize it all with a new drawing.

(https://www.eevblog.com/forum/chat/does-kirchhoffs-law-hold-disagreeing-with-a-master/?action=dlattach;attach=656205;image)
Title: Re: Does Kirchhoff's Law Hold? Disagreeing with a Master
Post by: Berni on February 18, 2019, 05:14:47 pm
Yep since KVL works in lumped circuits and attributes the voltage to an inductor while with Maxwell its just sums it in as a whole. Just two different ways of going about the same thing that come to the same result when used correctly.


So if you take yourself as being so knowledgeable in how to analyze circuits, have you decided yet what is the correct way to analyze the behavior of this circuit?
https://www.eevblog.com/forum/chat/does-kirchhoffs-law-hold-disagreeing-with-a-master/msg2189216/#msg2189216 (https://www.eevblog.com/forum/chat/does-kirchhoffs-law-hold-disagreeing-with-a-master/msg2189216/#msg2189216)
(https://www.eevblog.com/forum/chat/does-kirchhoffs-law-hold-disagreeing-with-a-master/?action=dlattach;attach=647828;image)

I can give a trip that gain degradation on the transistor plays no role in the operation so a ideal transistor with a fixed Hfe can be used, but the parasitic capacitance in the transistor are important to the operation, all of them are stated in the datasheet for that transistor (Tho a ballpark figure for most small signal transistors is close enugh)
Title: Re: Does Kirchhoff's Law Hold? Disagreeing with a Master
Post by: rfeecs on February 18, 2019, 06:47:19 pm
Yep since KVL works in lumped circuits and attributes the voltage to an inductor while with Maxwell its just sums it in as a whole. Just two different ways of going about the same thing that come to the same result when used correctly.


So if you take yourself as being so knowledgeable in how to analyze circuits, have you decided yet what is the correct way to analyze the behavior of this circuit?
https://www.eevblog.com/forum/chat/does-kirchhoffs-law-hold-disagreeing-with-a-master/msg2189216/#msg2189216 (https://www.eevblog.com/forum/chat/does-kirchhoffs-law-hold-disagreeing-with-a-master/msg2189216/#msg2189216)
(https://www.eevblog.com/forum/chat/does-kirchhoffs-law-hold-disagreeing-with-a-master/?action=dlattach;attach=647828;image)

I can give a trip that gain degradation on the transistor plays no role in the operation so a ideal transistor with a fixed Hfe can be used, but the parasitic capacitance in the transistor are important to the operation, all of them are stated in the datasheet for that transistor (Tho a ballpark figure for most small signal transistors is close enugh)

I'm guessing those loops are supposed to form a transformer and the circuit is an oscillator?  So we replace the loops with a lumped transformer?  I'm trying to read your mind, maybe not successfully.

The other way to read the schematic is a wire is a zero length connection in a netlist, so that circuit doesn't do much.

How about not such an easy one:

(https://www.eevblog.com/forum/chat/does-kirchhoffs-law-hold-disagreeing-with-a-master/?action=dlattach;attach=656277;image)

The boxes are microstrip lines (see Dave's videos of spectrum analyzer teardowns).  Does Kirchhoff's law apply?

The traditional way to design such a filter would be to select a transfer function like Chebyshev or something that meets the requirements.  Then synthesize a lumped filter.  Then convert the lumped filter to microstrip "equivalent".  Then make it and try to tune it to get it to work.

Instead, nowadays, you run a computer program and it spits out the design.  Then you probably would do EM simulation to verify your design and layout in the enclosure.  Then you make it and try to tune it to get it to work.

So as you have said, you can make a lumped model where KVL applies.  But the real filter fundamentally isn't lumped and KVL doesn't apply to it.  So you can separate the physical three dimensional filter where you would use Maxwell's equations (in the form of a computer simulation if you value your sanity); and the abstract lumped model where you use KVL and KCL and still use a computer simulation for the same reason.

There are other structures (antennas may be one example) where it is very difficult to come up with a lumped model or there is simply no reason to try.  There you use EM (Maxwell's equations) all the way.  The end result is a 2-port or N-port black box where Kirchhoff can be applied at the ports.
Title: Re: Does Kirchhoff's Law Hold? Disagreeing with a Master
Post by: ogden on February 18, 2019, 10:19:55 pm
The boxes are microstrip lines (see Dave's videos of spectrum analyzer teardowns).  Does Kirchhoff's law apply?

Obviously not. You do not apply Kirchoff's law to circuits where current in the conductor is not uniform. It is already discussed here in this thread. Dimensions of Dr.Lewin's experiment "secondary loop" is so small that any transmission line effect can be ignored for frequencies of experiment (300 Hz).
Title: Re: Does Kirchhoff's Law Hold? Disagreeing with a Master
Post by: jesuscf on February 19, 2019, 12:44:22 am
How about not such an easy one:

(https://www.eevblog.com/forum/chat/does-kirchhoffs-law-hold-disagreeing-with-a-master/?action=dlattach;attach=656277;image)

The boxes are microstrip lines (see Dave's videos of spectrum analyzer teardowns).  Does Kirchhoff's law apply?

What you are saying is equivalent to say: here is a silicon diode,  does Kirchhoff's law apply?  Or if you prefer: here is a transmission line, does Kirchhoff's law apply?  Don't you have to put the element in a circuit first?  Once you have a complete circuit, with every element of the circuit properly modeled, the laws of Circuits Theory apply.  That includes KVL.  In the case of this micro strip band pass filter, can you replace it with its transfer function?  If so, there are many ways of modeling the filter and include such model in the circuit.  In this case we'll have to use Maxwell's equations to derive the model (I guess, I have never done it), but that is different from using the model in a circuit.
Title: Re: Does Kirchhoff's Law Hold? Disagreeing with a Master
Post by: rfeecs on February 19, 2019, 07:10:55 pm
What you are saying is equivalent to say: here is a silicon diode,  does Kirchhoff's law apply?  Or if you prefer: here is a transmission line, does Kirchhoff's law apply?  Don't you have to put the element in a circuit first?

Well it is a bandpass filter, so the complete circuit would be a generator at the input and a load at the output, both with 50 ohm impedance in this case.

Even without that, I would still call this a circuit.  Just like a lumped bandpass filter is called a circuit.  This microstrip filter has paths from one element to the next and from the elements to ground, forming loops, just like the lumped version.

The Lewin circuit is so simple, with just wire loops and lumped elements.  So it could be easy to say it is just a transformer plus some lumped resistors and voltmeters.

I was trying to show a circuit that obviously wasn't that simple.  I guess I went too far to the point where it doesn't even look like a circuit at all.
Title: Re: Does Kirchhoff's Law Hold? Disagreeing with a Master
Post by: ogden on February 19, 2019, 08:10:45 pm
I was trying to show a circuit that obviously wasn't that simple.

Kirchoff's circuit Law do not apply to AC circuits that does not have DC circuit properties.  Look at the input of that microstrip filter - for DC current it is dead short. The same about output.

Wikipedia explains it well enough (https://en.wikipedia.org/wiki/Kirchhoff%27s_circuit_laws):

Quote
Both of Kirchhoff's laws can be understood as corollaries of Maxwell's equations in the low-frequency limit. They are accurate for DC circuits, and for AC circuits at frequencies where the wavelengths of electromagnetic radiation are very large compared to the circuits.

Other way to say it:

You do not apply Kirchoff's law to circuits where current in the conductor is not uniform.
Title: Re: Does Kirchhoff's Law Hold? Disagreeing with a Master
Post by: jesuscf on February 20, 2019, 01:25:46 am
I was trying to show a circuit that obviously wasn't that simple.

Kirchoff's circuit Law do not apply to AC circuits that does not have DC circuit properties.  Look at the input of that microstrip filter - for DC current it is dead short. The same about output.

Wikipedia explains it well enough (https://en.wikipedia.org/wiki/Kirchhoff%27s_circuit_laws):

Quote
Both of Kirchhoff's laws can be understood as corollaries of Maxwell's equations in the low-frequency limit. They are accurate for DC circuits, and for AC circuits at frequencies where the wavelengths of electromagnetic radiation are very large compared to the circuits.

Other way to say it:

You do not apply Kirchoff's law to circuits where current in the conductor is not uniform.

Kirchoff's circuit laws apply to anything that is modeled correctly for the range of frequencies you want to design/analyze.  The keyword in the Wikipedia quote above is 'accurate'.  For example, if you want to place a resistor in an AC circuit working at frequencies over 100MHz, it  will not behave just like a resistor.   Take a look at this EDN article:

https://www.edn.com/design/components-and-packaging/4423492/Resistors-aren-t-resistors (https://www.edn.com/design/components-and-packaging/4423492/Resistors-aren-t-resistors)

where they show a model of a 1k ohm carbon resistor used at such frequencies:

(https://m.eet.com/media/1201081/resistors_arent_resistor_fig2.jpg)

The same applies to circuits whose physical dimensions are similar to the wavelength of the applied AC signals.  In these cases even wires must be modeled accordingly, usually as transmission lines.  It is a compromise: how accurate do you want your results vs. how much you are willing to work.

As for the statement "You do not apply Kirchoff's law to circuits where current in the conductor is not uniform": what if you define a Gaussian surface surrounding the conductor with non-uniform current (or any part of a circuit by that matter) and compute the integral of all the currents crossing the surface?  You'll find that the integral is zero; same as Kirchoff's current law!
 
Title: Re: Does Kirchhoff's Law Hold? Disagreeing with a Master
Post by: ogden on February 20, 2019, 02:56:45 am
As for the statement "You do not apply Kirchoff's law to circuits where current in the conductor is not uniform": what if you define a Gaussian surface surrounding the conductor with non-uniform current (or any part of a circuit by that matter) and compute the integral of all the currents crossing the surface?  You'll find that the integral is zero; same as Kirchoff's current law!

I did not mean skin effect but transmission line effect - current uniformity trough the length of the circuit/loop. Sorry about wording.

Example: Speed of electric impulse in the cable (velocity factor) is ~65...85% speed of light. We just pick 195000 km/sec figure. So we have battery, lightbulb and 195km cable (twisted pair) to connect both. When we connect battery to the cable, current trough battery is flowing immediately but lightbulb is not lit yet, current will start to flow trough it (only) after 1ms. In this case Kirchoff's circuit law does not hold apply, sorry.
Title: Re: Does Kirchhoff's Law Hold? Disagreeing with a Master
Post by: bsfeechannel on February 20, 2019, 03:43:39 am
In this case Kirchoff's circuit law does not hold apply, sorry.

Kirchhoff's law doesn't hold. Lump modelling doesn't apply.
Title: Re: Does Kirchhoff's Law Hold? Disagreeing with a Master
Post by: rfeecs on February 20, 2019, 04:05:31 am
To use network analysis with transmission lines, the transmission lines are treated as components, not wires.  So, for example, a transmission line is treated as a two port.  At each port the voltage and current is uniquely defined.  All the retardation effects are contained within the two port.
[Edit] - Wikipedia entry for Two-port network:  https://en.wikipedia.org/wiki/Two-port_network (https://en.wikipedia.org/wiki/Two-port_network)

This has already been discussed in this thread.

It is similar to the "fix-ups" used for inductors and capacitors.

If you want to say Kirchhoff's laws can't be used in circuits with transmission lines, then you must say they can't be used in circuits with inductors, transformers and capacitors.  Or that they can only be used at DC.

The reality is that RF engineers simulate circuits with all those components.  Those simulators rely on KVL and KCL, and they work.  There is no argument here beyond semantics.


Title: Re: Does Kirchhoff's Law Hold? Disagreeing with a Master
Post by: jesuscf on February 20, 2019, 04:21:26 am
As for the statement "You do not apply Kirchoff's law to circuits where current in the conductor is not uniform": what if you define a Gaussian surface surrounding the conductor with non-uniform current (or any part of a circuit by that matter) and compute the integral of all the currents crossing the surface?  You'll find that the integral is zero; same as Kirchoff's current law!

I did not mean skin effect but transmission line effect - current uniformity trough the length of the circuit/loop. Sorry about wording.

This is what I meant:

(https://www.eevblog.com/forum/chat/does-kirchhoffs-law-hold-disagreeing-with-a-master/?action=dlattach;attach=657252;image)

Example: Speed of electric impulse in the cable (velocity factor) is ~65...85% speed of light. We just pick 195000 km/sec figure. So we have battery, lightbulb and 195km cable (twisted pair) to connect both. When we connect battery to the cable, current trough battery is flowing immediately but lightbulb is not lit yet, current will start to flow trough it (only) after 1ms. In this case Kirchoff's circuit law does not hold apply, sorry.

Yes Kirchoff's circuit laws do work:

(https://www.eevblog.com/forum/chat/does-kirchhoffs-law-hold-disagreeing-with-a-master/?action=dlattach;attach=657258;image)






Title: Re: Does Kirchhoff's Law Hold? Disagreeing with a Master
Post by: bsfeechannel on February 20, 2019, 06:10:32 am
The reality is that RF engineers simulate circuits with all those components.  Those simulators rely on KVL and KCL, and they work.  There is no argument here beyond semantics.

We need to use this thread to end this confusion once and for all. And the problem is not related to semantics.

Kirchhoff implies lump-modelling. Inverting this logic is false. Let's see this better exposed with a truth table.

Kirchhoff holdsLump-modelling possibleImplicationComment
TrueTrueTrueEvery circuit for which Kirchhoff holds can be automatically lump-modeled.
TrueFalseFalseIf you're saying that you can't lump-model a circuit where Kirchhoff holds you're contradicting the line above. So, false.
FalseFalseTrueIf you find a circuit that is impossible to lump-model then Kirchhoff doesn't hold (Lewin's circuit).
FalseTrueTrueKirchhoff doesn't hold, but you can still lump model  the circuit under certain conditions. Feynman picture 22-9 (http://www.feynmanlectures.caltech.edu/img/FLP_II/f22-09/f22-09_tc_big.svgz)

Inside a transformer, an inductor, a generator, an antenna or a transmission line, Kirchhoff doesn't hold. But you can still lump model them.

People see that those components can be lump-modeled and think that Kirchhoff holds. No, it doesn't. It will be impossible to explain how those things work using Kirchhoff.

But they do worse. Because they see that the behavior of the components that can only be explained by Maxwell's equations can be lumped, they think that every circuit can be lump-modeled.

This intuition is false.

Lump-modelling is a technique for circuit analysis. Kirchhoff's laws are the description of a physical phenomenon. Let's not confuse them.
Title: Re: Does Kirchhoff's Law Hold? Disagreeing with a Master
Post by: Berni on February 20, 2019, 06:14:49 am
Long wires are again one of the gotcha's of circuit modeling.

Just like ignoring inductance in Dr. Lewins experiment, ignoring transmission times long enough to affect behavior causes KVL to break down if the wire is just drawn as a line in the equivalent schematic.

As we have found multiple times on this thread Kirchoffs circuit laws are not fundamental laws of the universe. But a lot of circuits are a special case where they can be directly turned into a schematic and they work fine with circuit analysis. Such as connecting a resistor across a battery with short cables. There still are inductive and capacitive parasitics, transmission line effects etc... that we omitted in the schematic, but due to the nature of the particular circuit they don't affect how the circuit works so it doesn't matter that they got ignored.

So for the 195km cable you can model it as a before mentioned 4 port device to lump the entire cable. Then Kirchhoffs circuit laws work again as they always work in fully lumped circuits. The transmission line inside that box can be modeled in various ways, but the chain of inductors and capacitors is a perfectly valid way of doing it. That way of modeling a transmission line is actually quite well suited for explaining them as you can actually see the waves traveling down the chain as they do over the cable and each node on the LC chain does correspond in voltage at a point along the cable.

So yeah Kirchhoffs circuit laws indeed don't apply to long cables in real life, but they can be lump modeled to make it work.
Title: Re: Does Kirchhoff's Law Hold? Disagreeing with a Master
Post by: ogden on February 20, 2019, 08:09:48 am
So for the 195km cable you can model it as a before mentioned 4 port device to lump the entire cable. Then Kirchhoffs circuit laws work again as they always work in fully lumped circuits.

Sure you are right - *IF* you model cable as a string of many lumped elements. My oversimplified example contained three elements: 1) battery 2) cable 3) light bulb. That was whole point of example - to show case where you can't apply KVL and what's even more important - *why*. No need to cheat to get around pre-requirement for KVL I was actually illustrating! During described 1ms you can't apply KVL but later when current is uniform through the loop, KVL appies.

Hopefully you did not get sick with "putting unsaid words into debate opponents mouth disease", diagnosed for some here in this thread? ;)

[edit] Removed incorrect statement about engineering, thanx to @jesuscf
Title: Re: Does Kirchhoff's Law Hold? Disagreeing with a Master
Post by: ogden on February 20, 2019, 08:40:42 am
If you want to say Kirchhoff's laws can't be used in circuits with transmission lines

Thing is I did not say that. Cool down your imagination please ;) Also I did not say that KVL applies only to DC circuits. After all Lewin's experiment is AC circuit, just transmission line effects can be ignored for circuit and frequencies used.

[edit] Actually I can say that KVL cannot be applied to transmission line having transmission line effects (current is nonuniform through it) unless it is properly "lumped down".

Quote
then you must say they can't be used in circuits with inductors, transformers and capacitors.

I can't imagine how KVL can be applied to (across) transformer or microstrip filter shown here. Could you explain it please? - I am curious.

Title: Re: Does Kirchhoff's Law Hold? Disagreeing with a Master
Post by: jesuscf on February 20, 2019, 08:56:47 am
Sure you are right - *IF* you model cable as a string of many lumped elements. Engineering is exact science and circuit of my oversimplified example contained three elements: 1) battery 2) cable 3) light bulb. That was whole point of example - to show case where you can't apply KVL and what's even more important - *why*. No need to cheat to get around pre-requirement for KVL I was illustrating. During described 1ms you can't apply KVL but later when current is uniform through the loop, KVL appies.

First of all, I am sorry to break these news to you but engineering is neither a science nor exact!  Second, a very long 'cable' is far from an ideal conductor.  Similarly, a battery is far from an ideal source.  Finally, a light bulb is far from an ideal resistor.  Assuming you have idealized 'real' components and then proclaiming that KVL can not be applied is a straw man fallacy.

Circuit theory requires the correct modeling of all the physical devices used to describe the circuit.  If the circuit solution doesn't behave as the measurements in the circuit, it is not the fault of circuit theory, it is either the fault of the models used or the measurements themselves.
Title: Re: Does Kirchhoff's Law Hold? Disagreeing with a Master
Post by: ogden on February 20, 2019, 09:53:56 am
Second, a very long 'cable' is far from an ideal conductor.  Similarly, a battery is far from an ideal source.  Finally, a light bulb is far from an ideal resistor.
Assuming you have idealized 'real' components and then proclaiming that KVL can not be applied is a straw man fallacy.

Components are ideal until it is explicitly said that they are not. As you are not on the same page and fail to comprehend whole idea of dumbed down example so generic public can understand described transmission line effects, I have nothing to further discuss with you.
Title: Re: Does Kirchhoff's Law Hold? Disagreeing with a Master
Post by: Berni on February 20, 2019, 10:18:20 am
Sure you are right - *IF* you model cable as a string of many lumped elements. Engineering is exact science and circuit of my oversimplified example contained three elements: 1) battery 2) cable 3) light bulb. That was whole point of example - to show case where you can't apply KVL and what's even more important - *why*. No need to cheat to get around pre-requirement for KVL I was actually illustrating! During described 1ms you can't apply KVL but later when current is uniform through the loop, KVL appies.

Hopefully you did not get sick with "putting unsaid words into debate opponents mouth disease", diagnosed for some here in this thread? ;)

Did i claim that a user said something? If so then that was not the intention sorry.

And yep it is a good example that shows that KVL can't simply be directly slapped onto a real circuit by approximating everything as being ideal, including wires.



Yep since KVL works in lumped circuits and attributes the voltage to an inductor while with Maxwell its just sums it in as a whole. Just two different ways of going about the same thing that come to the same result when used correctly.


So if you take yourself as being so knowledgeable in how to analyze circuits, have you decided yet what is the correct way to analyze the behavior of this circuit?
https://www.eevblog.com/forum/chat/does-kirchhoffs-law-hold-disagreeing-with-a-master/msg2189216/#msg2189216 (https://www.eevblog.com/forum/chat/does-kirchhoffs-law-hold-disagreeing-with-a-master/msg2189216/#msg2189216)
(https://www.eevblog.com/forum/chat/does-kirchhoffs-law-hold-disagreeing-with-a-master/?action=dlattach;attach=647828;image)

I can give a trip that gain degradation on the transistor plays no role in the operation so a ideal transistor with a fixed Hfe can be used, but the parasitic capacitance in the transistor are important to the operation, all of them are stated in the datasheet for that transistor (Tho a ballpark figure for most small signal transistors is close enugh)

I'm guessing those loops are supposed to form a transformer and the circuit is an oscillator?  So we replace the loops with a lumped transformer?  I'm trying to read your mind, maybe not successfully.

The other way to read the schematic is a wire is a zero length connection in a netlist, so that circuit doesn't do much.

Yes the circuit has dimension that's why the size of the loops is marked down.

It is indeed an oscillator. Functions much like the classical blocking oscillator, but i modified the circuit to help keep the transistors operating point at a spot where it has lots of gain (sort of self regulates its collector bias current). This helps it keep oscillating with such a low inductance leaky "transformer" over a decent supply voltage range.

I have built the circuit for real on the bench and built it in LtSpice. Worked fine in both cases.

I'm just looking forward to how bsfeechannel explains how one should analyze this circuit correctly since all of it is inside the so called "KVL dies here" zone. So if classical circuit analysis theory supposedly can't be used here, then there must be some other way to calculate its behavior.
Title: Re: Does Kirchhoff's Law Hold? Disagreeing with a Master
Post by: ogden on February 20, 2019, 01:37:42 pm
Hopefully you did not get sick with "putting unsaid words into debate opponents mouth disease", diagnosed for some here in this thread? ;)

Did i claim that a user said something? If so then that was not the intention sorry.

Indeed sorry - that many including you, ignored or missed purpose of my "195km transmission line" example: to illustrate how transmission line works and how it can be that current is not uniform trough the length of the circuit/loop. Obviosuly to apply KVL, we have to do something with that line  - either lower working frequency or use it's lumped elements model. Kind of whole thread is about "lumping to apply KVL" :)

Those who become agitated with feelings "how dare you explain such a simple things to *me*" can rest assured - this is forum, not chat. Here are plenty of readers who just read w/o posting. Also in case I use Layman's terms to explain something it does not necessarily mean that I think in Layman's terms.

Sorry about this offtopic.
Title: Re: Does Kirchhoff's Law Hold? Disagreeing with a Master
Post by: bsfeechannel on February 20, 2019, 02:43:28 pm
I have built the circuit for real on the bench and built it in LtSpice. Worked fine in both cases.

I'm just looking forward to how bsfeechannel explains how one should analyze this circuit correctly since all of it is inside the so called "KVL dies here" zone. So if classical circuit analysis theory supposedly can't be used here, then there must be some other way to calculate its behavior.

If I tell your LTSpice that what you have between the base and the collector of your transistor is a dead short, it will refuse to simulate your oscillator.

Please refer to my previous message (https://www.eevblog.com/forum/chat/does-kirchhoffs-law-hold-disagreeing-with-a-master/msg2212704/#msg2212704) where I show the origin of the confusion between Kirchhoff's law and circuit analysis. They are not the same. I've been pointing that out along the whole thread.
Title: Re: Does Kirchhoff's Law Hold? Disagreeing with a Master
Post by: jesuscf on February 20, 2019, 04:19:50 pm
Second, a very long 'cable' is far from an ideal conductor.  Similarly, a battery is far from an ideal source.  Finally, a light bulb is far from an ideal resistor.
Assuming you have idealized 'real' components and then proclaiming that KVL can not be applied is a straw man fallacy.

Components are ideal until it is explicitly said that they are not. As you are not on the same page and fail to comprehend whole idea of dumbed down example so generic public can understand described transmission line effects, I have nothing to further discuss with you.

As soon as you say a 'cable' is 195 km long it stops being ideal.  Forget about modeling it using the telegrapher's equation, just the fact that the resistance per meter will accumulate to a value significantly larger that the load after such length, renders your example as useless.

Note: The fact that you have nothing to discuss with me will not stop me from pointing out your fallacies.  After all, this is public forum.
Title: Re: Does Kirchhoff's Law Hold? Disagreeing with a Master
Post by: jesuscf on February 20, 2019, 05:25:57 pm
(https://www.eevblog.com/forum/chat/does-kirchhoffs-law-hold-disagreeing-with-a-master/?action=dlattach;attach=647828;image)

Berni, may I ask what is the resonant frequency of your circuit?   Also, what would happen if you put a piece of metal close to the loops?  I am asking because your circuit could make a fine metal detector...
Title: Re: Does Kirchhoff's Law Hold? Disagreeing with a Master
Post by: Berni on February 20, 2019, 05:41:17 pm
Berni, may I ask what is the resonant frequency of your circuit?   Also, what would happen if you put a piece of metal close to the loops?  I am asking because your circuit could make a fine metal detector...

I think the real circuit ran at about 17MHz while in spice somewhere around 21MHz. Tho i was using just a regular passive scope probe so the capacitance of it has likely affected the frequency too. As one might expect putting a lump of ferrite in there does drag down the frequency slightly. Haven't tested if non ferrous metal around the coil affects it, i would guess not since that doesn't change the inductance, just introduces extra loss in the form of eddy currents, large enough aluminium plate could probably stop the oscillator if it sucked away enough power from the coil.

Could probably also be used as a weak CW radio transmitter, but the waveform is not a sinewave and is not even symmetrical so there would be plenty of even and odd harmonics, but the power is really low as the whole circuit only uses 1 or 2 mA from the power supply.
Title: Re: Does Kirchhoff's Law Hold? Disagreeing with a Master
Post by: jesuscf on February 20, 2019, 06:11:41 pm
Berni, may I ask what is the resonant frequency of your circuit?   Also, what would happen if you put a piece of metal close to the loops?  I am asking because your circuit could make a fine metal detector...

I think the real circuit ran at about 17MHz while in spice somewhere around 21MHz. Tho i was using just a regular passive scope probe so the capacitance of it has likely affected the frequency too. As one might expect putting a lump of ferrite in there does drag down the frequency slightly. Haven't tested if non ferrous metal around the coil affects it, i would guess not since that doesn't change the inductance, just introduces extra loss in the form of eddy currents, large enough aluminium plate could probably stop the oscillator if it sucked away enough power from the coil.

Could probably also be used as a weak CW radio transmitter, but the waveform is not a sinewave and is not even symmetrical so there would be plenty of even and odd harmonics, but the power is really low as the whole circuit only uses 1 or 2 mA from the power supply.

My understanding is that non ferrous metals should decrease the inductance, therefore increasing the resonant frequency.  Anyhow, nice simple circuit with useful practical applications...  Did you use magnet wire?  What gauge?  AA battery?
Title: Re: Does Kirchhoff's Law Hold? Disagreeing with a Master
Post by: ogden on February 20, 2019, 06:46:09 pm
As soon as you say a 'cable' is 195 km long it stops being ideal.  Forget about modeling it using the telegrapher's equation, just the fact that the resistance per meter will accumulate to a value significantly larger that the load after such length, renders your example as useless.

It does not need to be ideal. I just picked long enough line to bring timing into intervals most will not be afraid of. If I would talk about > 2GHz frequencies and according wavelengths, many would not understand how transmission lines work, what is velocity factor. "Telegraphers equation" indicates that you do not understand transmission line physics as well.

Quote
Second, a very long 'cable' is far from an ideal conductor.

So what? Fact that cable has some significant resistance somehow makes my example of transmission line inoperable or what? Don't blame me if you can't wrap something into your linear logic.
Title: Re: Does Kirchhoff's Law Hold? Disagreeing with a Master
Post by: Berni on February 20, 2019, 07:09:38 pm
My understanding is that non ferrous metals should decrease the inductance, therefore increasing the resonant frequency.  Anyhow, nice simple circuit with useful practical applications...  Did you use magnet wire?  What gauge?  AA battery?

I still had the circuit so i hooked it up and played some more with it.

To power it i use a capacitor with twisted wires coming up to it from my lab PSU so that i can easily adjust the voltage while the capacitor ensures a low impedance at high frequency. For the transistor i used (random 2SC2705 i had a bag of in TO92) the circuit works from 0.6V to 3.5V so i picked 1.5V as somewhere in the middle and conveniently can be a alkaline cell. For the wire i just used some wire from a CAT5 cable. You can use a twisted pair to get both coils in one loop.

In terms of being a metal detector its not so good as shown since its very sensitive to stray capacitance. It does make a great theremin tho since the frequency wanders as you wave your hand around the loop. The 1K resistor makes the whole thing high impedance.

But you can fix that if you add a 1uF capacitor in parallel with the resistor and battery. This gives the transistor a low impedance supply voltage that it regulates on its own. Using this the operating voltage range becomes 0.8V to 60V (Probably even higher if the 1K resistor handles the power) tho it does seam to shift into a different oscillation mode above 5V. With that the circuit is no longer sensitive to stray capacitance as much. Placing a ferrite core inside again lowers the frequency but placing a aluminum heatsink inside raises the frequency so it is indeed a metal detector. Tho the frequency can also shift with changing the supply voltage or the transistors temperature.
Title: Re: Does Kirchhoff's Law Hold? Disagreeing with a Master
Post by: jesuscf on February 20, 2019, 08:46:57 pm
It does not need to be ideal. I just picked long enough line to bring timing into intervals most will not be afraid of. If I would talk about > 2GHz frequencies and according wavelengths, many would not understand how transmission lines work, what is velocity factor. "Telegraphers equation" indicates that you do not understand transmission line physics as well.

Really?  I would recommend you some enlightening reading from chapter 10 of "Engineering Electromagnetics" by William H. Hayt and John A. Buck where the figure below comes from:

(https://www.eevblog.com/forum/chat/does-kirchhoffs-law-hold-disagreeing-with-a-master/?action=dlattach;attach=657813;image)
Title: Re: Does Kirchhoff's Law Hold? Disagreeing with a Master
Post by: ogden on February 20, 2019, 09:08:29 pm
It does not need to be ideal. I just picked long enough line to bring timing into intervals most will not be afraid of. If I would talk about > 2GHz frequencies and according wavelengths, many would not understand how transmission lines work, what is velocity factor. "Telegraphers equation" indicates that you do not understand transmission line physics as well.

Really?  I would recommend you some enlightening reading from chapter 10 of "Engineering Electromagnetics" by William H. Hayt and John A. Buck where the figure below comes from:

What exactly you imply by pointing to this common model of transmission line? - That you understand it contrary to my trolling? :D If you understand - good for you.

Even if 195 km twisted pair line is built using copper conductor, it does not make my example useless as you say. After all battery voltage nor light bulb current were specified. By arguing about nonessential properties of my oversimplified example you are just wasting electronic ink of internet and to be honest - making yourself fool.
Title: Re: Does Kirchhoff's Law Hold? Disagreeing with a Master
Post by: jesuscf on February 21, 2019, 01:33:34 am
What exactly you imply by pointing to this common model of transmission line? - That you understand it contrary to my trolling? :D If you understand - good for you.

Even if 195 km twisted pair line is built using copper conductor, it does not make my example useless as you say. After all battery voltage nor light bulb current were specified. By arguing about nonessential properties of my oversimplified example you are just wasting electronic ink of internet and to be honest - making yourself fool.

Wow, ad hominem fallacy!

Non-essential properties you say!  Go tell that to the electrical engineers of your local power utility and watch their reaction.  If reading the book I recommended is out of your reach, try watching a PBS documentary from the series American Experience about the first trans Atlantic cable.  I think YouTube has it.

Title: Re: Does Kirchhoff's Law Hold? Disagreeing with a Master
Post by: ogden on February 21, 2019, 05:40:49 am
Non-essential properties you say!  Go tell that to the electrical engineers of your local power utility and watch their reaction.  If reading the book I recommended is out of your reach, try watching a PBS documentary from the series American Experience about the first trans Atlantic cable.  I think YouTube has it.

Instead of offering me read that book maybe you can read it yourself? Please show me how transmission line DC resistance property relates to what I was talking about in my example? You may start with velocity factor formula and check where you put DC resistance in. Anywhere?

(https://wikimedia.org/api/rest_v1/media/math/render/svg/8554ab053195511103518ae463e6c7b52ad5136a)

[edit] It does not add to the discussion about wavelengths in the cable if you are trolling about how big attenuation will be in 195km cable or how hugely expensive it will be. It does not matter in the context!
Title: Re: Does Kirchhoff's Law Hold? Disagreeing with a Master
Post by: jesuscf on February 21, 2019, 07:56:53 am
Non-essential properties you say!  Go tell that to the electrical engineers of your local power utility and watch their reaction.  If reading the book I recommended is out of your reach, try watching a PBS documentary from the series American Experience about the first trans Atlantic cable.  I think YouTube has it.

Instead of offering me read that book maybe you can read it yourself? Please show me how transmission line DC resistance property relates to what I was talking about in my example? You may start with velocity factor formula and check where you put DC resistance in. Anywhere?

(https://wikimedia.org/api/rest_v1/media/math/render/svg/8554ab053195511103518ae463e6c7b52ad5136a)

[edit] It does not add to the discussion about wavelengths in the cable if you are trolling about how big attenuation will be in 195km cable or how hugely expensive it will be. It does not matter in the context!

Your example is fundamentally flawed because you failed to properly model the elements of the circuit before attempting to analyzed it.  Hence you incorrectly concluded that Kirchhoff's laws didn't work.  I was just trying to show you how to properly model the elements of the circuit you proposed.  That includes the attenuation of the transmission line.  When you model all the elements of your circuit correctly then Kirchoff's laws do work!

It is the same with Dr. Lewin's demonstration.  He conducted an experiment.  The results of the experiment allegedly didn't match the laws of circuit theory.  Hence he concluded that the laws of circuit theory didn't work under the conditions of the experiment.  Along comes Mehdi (electroboom) and points out that important mistakes or omissions were made with the measurements and modeling of the circuit in Dr. Lewin's experiment.  Then Mehdi both models the circuit correctly and performs sensible measurements that prove that indeed the laws of circuit theory work.
Title: Re: Does Kirchhoff's Law Hold? Disagreeing with a Master
Post by: ogden on February 21, 2019, 10:34:27 am
Your example is fundamentally flawed because you failed to properly model the elements of the circuit before attempting to analyzed it.

Your logic is flawed or you are trying to shift goalposts, again.

I used model described in example on purpose - to simply demonstrate case where KVL does not apply and explain what I did mean by "non-uniform current". I did explain it already, pay close attention to fact that proper modeling of transmission line as precondition for KVL is mentioned as well:

Indeed sorry - that many including you, ignored or missed purpose of my "195km transmission line" example: to illustrate how transmission line works and how it can be that current is not uniform trough the length of the circuit/loop. Obviosuly to apply KVL, we have to do something with that line  - either lower working frequency or use it's lumped elements model.

In case this explanation still does not work for you - fine. I am ready to simply agree on disagreement.

Quote
When you model all the elements of your circuit correctly then Kirchoff's laws do work!

Yes I know. I confirmed that directly and indirectly here in this forum countless times. I am on your side - in case picking sides is important for you
Title: Re: Does Kirchhoff's Law Hold? Disagreeing with a Master
Post by: bsfeechannel on February 21, 2019, 03:03:32 pm
It is interesting to note that those who say that modeling your circuit "correctly" Kirchhoff always holds can never agree with each other. And have to take sides.

Those of us who care to study electromagnetism always agree with each other: I agree with Sredni, who agrees with Feynman, who agrees with Maxwell who agrees with Faraday.

There's never any animosity or name calling between us. Quite the opposite: we always thank each other for helping us have an even deeper understanding of physics because we know that this will make us better and more efficient design engineers and will save us the embarrassment of advocating pseudo scientific claims.

Where people refuse to accept the truth there can never be rational thinking.
 
Title: Re: Does Kirchhoff's Law Hold? Disagreeing with a Master
Post by: Sredni on March 05, 2019, 07:44:40 am
post 2 of 3
(posted out of order for reasons to be explained in post 1 of 3)


On Kirchhoffians

Quote from: EEVBlog
If he did then you and a couple of others wouldn't have  had to have spent, what, maybe a hundred or two man-hours trying to break this down and debate and explain it? 

No, the concept is pretty simple (even though its implications can be counterintuitive, much like with the postulates of special relativity). What is difficult is trying to explain to people who do not have the prerequisites for understanding it (and/or have no intention to remedy that situation).

Faraday (from emf) and meshes
When you place a voltmeter in the Romer-Lewin circuit the mesh who is linking the flux has the bigger or smaller resistor in it, depending on where you place it. That's why you see different voltage depending on where you put the voltmeter.

    But no, meshes are not good, if you use the seesaw line to represent resistor, they get confused and think everything should be lumped, so where is the primary of the transformer? Surely it will spring out when they apply Faraday from scratch using the integrals.

Faraday (from flux) and integrals
There are integrals, so  you need areas, and areas have boundaries, there is an inside and an outside. Depending on where the varying magnetic field happens to be (inside or outside?) you get a different result.

    But no, areas are not good, they feel they can compute everything pointwise, on open paths and with undefined areas. So they want to go full Field.

Field theory
Maxwell's equations are not algebraic vector equations: they are partial differential equations. You need boundary conditions to go along with them. One can try to explain how rotor and divergence are related to circulation and flux and that you still need elementary areas and relative closed boundaries. That the total electric field along an open segment of your circuit is not uniquely determined by the segment itself, it depends on the rest of the circuit (or lack thereof)  because the circuit path as a whole is setting the boundary conditions.

    But no, they still think they can split the solution on separate segments irregardless of what happens on the rest of the closed path, basically trying to define the inside and outside of an open curve. And even when you show that it is not the case because it would lead to wrong results, their conclusion is that their method is not wrong, it's just a 'bit off'. As if it were a numerical approximation error.
When you insist they produce a drawing of their closed-open curve, the discussion "becomes a bit heated" and you get banned.

And it never ends. They will always find some new twist.
Recently they mentioned transmission lines. They do not care when a system can be modeled by a transmission line, they just proceed by analogy: there's something that looks like a cable, and a model that has many inductances (and something else they do not care about) that represents its behavior. So, since the Romer-Lewin ring has some copper cable in it, it must certainly be that. Do you think they ever bothered to ask themselves what behavior is being modeled  in a transmission line?

Same goes for the equivalent circuit of a transformer that was brought up lately: it should model the behavior as seen from the terminals, but they don't care. They see that a transformer - which is basically two pieces of wire wound around something - has an equivalent circuit (as seen from the terminals) with many lumped components and think "That must be it!". They incorrectly infer that the lumped model must be what shows the voltages and currents inside the piece of wire!

This is plug and chug mentality, mixed with cargo cult science.
The root of the problem is the same: there are people who do not know when a certain model (a model other people, who actually studied the matter, have developed, refined and made so simple even a ten year old can apply it) can be used. Even worse: they have been taught what the limits of the model are, but they either forgot or do not care.

They twist reality to suit it to their warped, Aristotelian, thinking.
And, no, apparently you cannot even tell them to get an education because that is bullying.
You have to apologize, especially if the ignorant person that refuses to learn and persist in their error is a nice guy. So Lewin is the terrible human being who finally realized his godawful error: telling a youtuber to go study basic physics. Because Mehdi is wrong, and Lewin is right (you said so yourself: "I don't think anyone doubts that Lewin is ultimately right (he is)"), and yet it is Lewin who has to apologize.
Form over substance.

This is an ad from a magazine of thirty or more years ago.

(https://i.ibb.co/gjmgKgq/screenshot-3.png)

Now things have changed. The moralists and the PC police are here.
Nobody should be allowed to point out how wrong certain people are, not even a (former) MIT professor. Because that's rude. It's "wrong and never works". After all, everybody is special and cannot possibly be wrong and even when they are wrong it's not important, especially if they are nice.
Nobody should be allowed to tell them to go read a book on basic electromagnetism, to get educated. "Wrong" is a bad word, "ignorant" is an insult, "educate yourself" is probably a misdemeanor, if not a felony.
This is where we're at:

https://www.youtube.com/watch?v=Zh3Yz3PiXZw (https://www.youtube.com/watch?v=Zh3Yz3PiXZw)
Alternative Math (youtube video Zh3Yz3PiXZw)

And this leaves us stuck with a lot of ignorant nice special people who are just 'alternatively right'.
We are not supposed to tell them to stop wasting everybody's time by spreading ignorance and go study basic physics. We are supposed to school them from kindergarten up, step by step, repeating the same things over and over every time they reset to factory settings. [note 1]

And the kindergarten stuff, sadly, is not sarchasm. When I posted this picture...

(https://i.ibb.co/y6Y36bC/kirchhoff.png)
What a Kirchhoffian does not get

...I thought I had reduced the problem to its miminum terms: a disk is not an annulus, a simply connected set is not a multiple connected set. But I was wrong, that is too complex and confusing: what Kirchhoffians fail to grasp is the concept itself of "inside" and "outside". So I should have posted this picture instead

(https://i.ibb.co/CH8zFD8/screenshot.png)
source: "Inside_and_Outside_Comparison_for_Kids" on YouTube

Now, to strengthen the concept and avoiding confusion due to the fact that above 'inside' is associated with two elements, while outside only has one (of the same category), here's another example.

(https://i.ibb.co/pwfNDpB/screenshot-2.png)
source: "Inside_and_Outside_Comparison_for_Kids" on YouTube

Inside.
Outside.
They are different.

The key is that if the (net) varying magnetic field happens to be inside your circuit path is one thing, but if it happens to be outside, it's a different thing. You can see that with meshes, with Faraday in integral form, with Faraday in differential form, and everything checks beautifully. But this simple concept is simply ignored.
30 pages are not enough. Not even 30 thousand might suffice. But not for the reason you think:

Quote from: EEVBlog
This is not a trivial argument that even the most experienced and well educated engineers understand, in fact the argument has been going on decades.
Mehdi had every right to question Lewin's experiment, because without excruciatingly detailed investigation it seems like a dodgy setup to demonstrate his point.

Sorry Dave, but this is not because the concept of path-dependent voltage is some esoteric theoretical fundamental shit like QED, virtual quarks in an alternate universe or the subliminated state of unobtanium. This is 'fundamental' only in the sense of 'basic', 'elementary', 'first year' physics. Once you understand that voltage is path dependent when there is a varying magnetic field inside your circuit, you do not need probing to know that there have to be different voltages across the two resistors.

Provided someone taught you the difference between "inside" and "outside" and you managed to retain the concept.[note 1, again]

If anything, this concept is so basic that, being taught in first or second year classes, many people simply forgot about it (relapse into illiteracy is a real thing, I experience it everyday with the English language) and when someone point out it is so basic that they should grab a first year physics book to refresh basic notions, they refuse to do it, because they think they know better and you are just insulting them (or because... see the conclusion of post 3 of 3).

[note 1] The retaining ability alone looks more and more like a superpower.
Look at how the -L di/dt term creeped up in a recent post. There were pages and pages of discussion regarding how the concept of inductance is defined for lumped components that are part of a circuit inside which there are no varying magnetic fields with the integral of the total electric field broken down into pieces to show why it has to be that way. But it is as if they never existed (so, what's the point of leaving my posts here? They just waste server space, and energy for their retrieval). All the Kirchhoffian sees is: there is something that resembles a coil in a magnetic field, hence that's an inductance L, let's plug the formula for that somewhere in the equations and proceed from there. To hell all the prerequisites and conditions that are required to condense a complex behavior into a simple quantity L.
If you get a big enough hammer, that square peg will eventually fit into that round hole. After all, if you have used successfully for pushing all those round pegs before...




Title: Re: Does Kirchhoff's Law Hold? Disagreeing with a Master
Post by: Sredni on March 05, 2019, 07:56:55 am
post 3 of 3

On Mehdi's video in particular

Quote from: EEVBlog
I don't think anyone doubts that Lewin is ultimately right (he is), but AFAIK he failed to address any of Electroboom's practical points.
From what I have seen, it's Lewin with his fingers in his ears repeating "KVL doesn't hold" 100 times, vs Electroboom trying to methodically evaluate the problem from a practical demonstration standpoint. From that I know who I have more respect for at the very least. 

A lot of people asked why hasn't Lewin responded to Medhi's objections. But he did, well before Mehdi's video of three months ago (look at the date: one year ago)

(https://i.ibb.co/NtGbRkn/screenshot-4.png)
comments by lewin from youtube video LzT_YZ0xCFY

Lewin also mentioned this fact when he was recently asked why he did not answer to the latest video, saying that he had already addressed Mehdi's misconceptions.
And if you go through the various Lewin's videos on this matter,  in the comment sections you will find a single comment repeated over and over (what you would certainly call 'borderline spam'), it's the one above that is dated '3 months ago'.

(https://i.ibb.co/M9mMQwX/screenshot-5.png)
multiple comment by lewin, post mehdi's video

Guess what: he's not repeating it because he's having a bad senescence episode. He's repeating it because, basically, that's all there is to it.

Quote
Lewin: In the case of an induced EMF the potentials in a circuit are no longer determined, they depend on the path

Should he go on indefinitely to answer such basic stuff? I can understand why he stopped doing it.

Quote from: EEVBlog
You make it sound like all this is so plainly obvious, that it's all obviously settled, and "how dimwitted Mehdi's audience is for not noticing this",

Look at the most recent video by Mehdi, "The conclusion"

He tries to put the all thing down to the OLD KVL vs NEW KVL problem. But even there he fails to see the crucial point.

Mehdi@8:08 "KVL says that the sum of all voltages in a loop must be zero".

So, there you have it. He's saying this looking at the Romer-Lewin ring.
He is assuming you can apply KVL  to a varying magnetic field region, or that you can have single valued voltages inside the loop so that their sum has a meaning without further specification. This is the problem.
You can have such single valued voltages only if there are no varying magnetic fields enclosed by the loop.
This is the part that Kirchhoffians do not understand. They seem to believe that when people tell them that they should not have dB/dt within the circuit's premises, it is implied there has to be no dB/dt at all, anywhere. Sorry guys, but if you had read some serious book on EM you would know that NEW KVL works with varying magnetic fields inside the components BUT still require that there be NO VARYING magnetic field INSIDE the CIRCUIT PATH.

Like other Kirchhoffians, Mehdi is struggling with the concepts of "inside" and "outside".

To pretend that Lewin is mistakenly using the old (pre-Faraday) version of KVL, where there are no varying magnetic fields at all anywhere, is intellectually dishonest. And since a large chunk of Mehdi's audience appears - from the comments - to have bought that line of defense, I wouldn't be so harsh in dismissing the qualifier of "dim-witted" attributed to them by bsfeechannel.

Also, Mehdi should pay attention to what E is. Is it the coulombian field, or is it the total field, sum of the coloumbian and induced parts? It appears to me that Belcher tried to tell him, but he seems not to have understood it.

I could go on with little things like these (that might happen to be just slips of the tongue):

Mehdi@10:22 "each meter reads the voltage across the resistor on their side"

    not quite, the meters read a voltage that is very close to the voltage of the resistor on their side. But that's nitpicking.

Mehdi@11:33 "if the field is going through the page in that way, the induced current is in this way"

    nope, it's not how the field is directed that counts, it's how it is varying. You can have opposite currents with the field always entering the page, depending on whether the field is increasing or decreasing. Not nitipicking, this is an important distinction, but it's a forgivable slip in a long video.

Mehdi@11:40 "we can split this integral..."

    does Mehdi realize that in addition to the endpoints a and b he has to specify the path, as well? It might seem so, but I am not sure at all because of what he says later on.

Or I can get to the real beef that comes at minute 13 or something - when Mehdi mentions Feynman. He says he understood how the voltage across the inductor can be nonzero even if the path integral across the conductor it is made of is basically zero and shows the circuit with the gamma line (fig. 22-9)

Mehdi@13:51 "and doing so the book shows that the sum of all voltages in the loop is zero and KVL holds"

From the smile of satisfaction (and the triumphant music) I infer that he believes this is proving him right. And he goes on saying

Mehdi@13:58 "So professor Belcher also concluded that doctor Feynman himself and I have the same definition of voltage and that KVL holds in all cases"

All of this is a load of bullshit. Why? Because in Feynman's book it is clearly stated that THERE HAS TO BE NO VARYING MAGNETIC FIELD INSIDE THE GAMMA LINE.

http://www.feynmanlectures.caltech.edu/II_22.html (http://www.feynmanlectures.caltech.edu/II_22.html)
(https://i.ibb.co/DVbp881/screenshot-6.png)

A condition that is not satisfied by the Romer-Lewin ring.

Inside. Outside. Different things.

KVL does not hold in all cases (yes, Medhi actually said that KVL holds in all cases) and neither Feynman nor Belcher have ever written that it does.
And yet Mehdi put that load of bullshit into their mouths and his audience bought it. Hook, line and sink.

Oh, and then he goes on for minutes showing off how KVL is so useful, completely omitting the fact that its usefulness is subordinated to its applicability. Dude, learn to use your tools, but most importantly learn when you can use them.
It looks like he's throwing smoke in the eyes of his audience as if to say: look how useful this thing is! Can you believe Lewin wanted to do away with it?

Quote
Professor X is saying I cannot walk on water. But here I am, walking on grass. See? I can even jump and run. Walking is very useful, a lot of runners walk and run every day. Professor Y and Z agree with me.
QED.

This is intellectual dishonesty.
If you do not see this and believe that I have just demonstrated professor X is wrong, well, you are dimwitted.
(Yes, I can see beanflying bleeding from the eyes and bursting into flames)





<pause while someone grabs a fire extinguisher and put off the fire>






And I know it never ends. You have to go till the end of the video, otherwise the dimwitted audience will take that omission as running away from the real explanation that surely will been given from that point onwards. So...

At 14:58 Mehdi even reads Feynman

Mehdi@14:58 "A separation is made between what happens inside and what happens outside"

Inside.
Outside.
So they really are different things.
Does Mehdi know how important is to distinguish between them?

Mehdi@15:02 "This is important to know because..."

It seems so! But then:

"...even if the field is not contained and leaks outside, we still have an inductor and [??defined??] voltages are everywhere and KVL holds"

Oh, no, it seems he doesn't. But wait:

Mehdi@15:10 "But this creates huge complexities in our calculation because now the leaking field affects all the components and wires in the circuit that we have to take into account."

Oh, he seems to get it, but wait again...

Mehdi@15:20 "... which is almost impossible because now there are so many new variables introduced that we might easily overlook".

Actually it's more that you no longer have a single-valued voltage in your circuit (as Lewin tried to tell him) and you need to resort to fields and compute path integral along specified paths. (Feynman says it's almost imposibile when you have hundreds of components, but it can still be done rather easily with the Romer Lewin ring - as a matter of fact, a little bit of vector calculus will show why the voltage along the circuit happens to be only at the resistors)
Did he get it? Let's see:

Mehdi@15:26 "Our simple loop and probes was the easiest example of uncontained field. The field closes around the entire circuit and affects everything. Every piece of wire and even the resistor itself  become and inductor and the secondary of a transformer".

So, it's definitive. He does not get it.
He does not get it that in order to model the components as inductors and secondaries of transformers you need to satisfy the prerequisites for the application of lumped circuit theory. That is, you need for your CIRCUIT PATH to have NO VARYING MAGNETIC FIELDS INSIDE OF IT.

Inside.
Outside.
Different things.

Q.E.D.
(sounds pompous, doesn't it?)


P.S.
what remains regards avoiding the leakage unwanted flux on the outside of the circuit (he seems to still be thinking that the multiple valued voltage can be explained with bad probing) and him still falling into the trap of believing there is a voltage developed along the conductor

Mehdi@17:50 "every piece of wire is an inductor. If we have a voltage V across the main loop we would also have half a V induced on equal length of prove wire exposed to the same magnetic field"

fig a 17:54

"A more proper scientist like Romer would say..."

Yes he would say something different because the voltage across an open wire depends on the path along which it is evaluated. And if you go along the wire as Mehdi shows with the movement of his marker (and lack of specification of any other path), it is essentially zero.
To say that there is v/2 on the half arc is the same as saying that there is a build-up in voltage along the coil of a transformer, Mabilde-like. And that is wrong.
But no, the concept of wrong is not allowed.

"it is not wrong, but it's a bit misleading"

Of course, wrong is a bad word. How could anything be wrong? Just like the energy flow guy from Science Asylum ("Energy doesn't flow the way you think", youtbue video C7tQJ42nGno) who is propagating the concept that the electric field inside the copper wires is much stronger than immediately outside, where according to him it is parallel to the wires. In the pinned comment he says "it's not a mistake, it's an educational choice".
Despite the shitload of papers some viewers have posted to prove how wrong he is.

Someone here mentioned the Dunning-Kruger effect. Yes, I guess it plays a role in all these matters. I only wonder where is the threshold: one hundred thousand? five hundred thousand? one million subscriptions?

As I said before, scientific populism is going to be a big problem a few yeas ahead.
Flatearthers, moonhoaxers, antivax and climate change deniers are just the tip of the iceberg.


Title: Re: Does Kirchhoff's Law Hold? Disagreeing with a Master
Post by: Sredni on March 05, 2019, 08:01:32 am
Post 1 of 3

On the Moderation


Quote from: bsfeechannel
However we need to express our appreciation for Simon's work. I've been a moderator before. It is one of the most ungrateful tasks, let me tell you. 

I'm sorry but there's no room for appreciation for sloppy work and for the incongruent justification he gave for my ban. I could go into details, but it is clear from what he did and wrote that feedback is something to print on T-shirts and that's it. So, why waste time? He probably does not even know how his messages are rendered by browsers outside the one he uses (or that when one is banned there is no way to send PMs).

But I want to add something to explain why I deleted the technical part of my last posts and, since the thread is now dead, to address what I see as the real problem in the Lewin-Mehdi discussion. I won't get into the physics, it's not worth the time spent.

Quote from: Berni
I also don't understand what Sredni was trying to accomplish by deleting content from his posts. Basically vandalizing his own work to make it harder for someone else to follow the tread. 

Quote from: EEVBlog
Yep, I don't get it either, all that hard work gone, but we've seen this before on the forum and it usually doesn't end well unfortunately. I hope that doesn't end up being the case here. 

What do you expect? Me going Turbo and try to kill princess Vanellope?
Nah, I simply decided not to contribute to this blog anymore than it is necessary to get my potential future questions answered. You had someone who could provide content (and believe me, I had a lot more material to share), now you have a leech.

I removed only the technical parts from my previous posts and left all that remained because I still had the curiosity to know what earned me a ban (apart from Simon to be a precog, as postulated by ogden), but it's clear that that curiosity is going to remain unanswered. Since I did not see anything in particular, I deduced that what irked the guy who reported 'the topic was getting heated' was the technical part. So I removed it. Yeah, a waste of work and time, much in the same way the time and work I had put in my last post (on when it is desirable to contain the whole field) was wasted due to the ban.
 
If the people who manage the site don't care, why should I?


Technically this was post 1 of 3, but I posted it as last because, you know, when it is not clear why you guys ban people, one never knows when their 'hard work' can go wasted.


Title: Re: Does Kirchhoff's Law Hold? Disagreeing with a Master
Post by: Claudiovpm on March 05, 2019, 08:33:29 pm
 :)
Title: Re: Does Kirchhoff's Law Hold? Disagreeing with a Master
Post by: HuronKing on March 06, 2019, 12:16:28 am
I've benefited from your contributions Sredni. I've been silently following the conversation here (I just registered so I could comment) but I actively participated in the Youtube conversations on both EB's and Lewin's channels. I was the person who reminded Lewin about the Romer paper which he mentioned in one of his response videos (he credited me by my YT moniker).

As a matter of fact I've used this very discussion as a teaching example in my lab classes on electric motors and power electronics. I've made it a point to remind the graduating seniors of electrical engineering that Maxwell's Equations are the Laws of Classical Electrodynamics and have used the Romer-Lewin Ring paper and demonstration as a pointer to that fact.

It's a good thing our technological progress is governed by hard mathematics and experiment, otherwise we would still believe that objects of different masses fall at the same rate, because believing otherwise violates our 'intuition.'

By the way, it's easy to demonstrate that fact even without a vacuum chamber. Put a tiny sheet of paper on top of a textbook and drop them both.

Anyways, Lewin was/is correct and has been all along.  :-+
Title: Re: Does Kirchhoff's Law Hold? Disagreeing with a Master
Post by: bsfeechannel on March 09, 2019, 07:34:36 pm
I've benefited from your contributions Sredni. I've been silently following the conversation here (I just registered so I could comment) but I actively participated in the Youtube conversations on both EB's and Lewin's channels. I was the person who reminded Lewin about the Romer paper which he mentioned in one of his response videos (he credited me by my YT moniker).

Welcome to the forum. We need more people like you and Sredni around here to help us disentangle those fundamental concepts.

Quote
As a matter of fact I've used this very discussion as a teaching example in my lab classes on electric motors and power electronics. I've made it a point to remind the graduating seniors of electrical engineering that Maxwell's Equations are the Laws of Classical Electrodynamics and have used the Romer-Lewin Ring paper and demonstration as a pointer to that fact.

That's why we keep the discussion going on, because of the silent majority that really benefits from it. Our "opponents" help us reveal what exactly is the difficult point in the understanding.

Quote
It's a good thing our technological progress is governed by hard mathematics and experiment, otherwise we would still believe that objects of different masses fall at the same rate, because believing otherwise violates our 'intuition.'

By the way, it's easy to demonstrate that fact even without a vacuum chamber. Put a tiny sheet of paper on top of a textbook and drop them both.

There has been good and encouraging threads on this forum about the importance of math and fundamental physics for engineers. Coming from a practical electronics background, I felt kinda overwhelmed by the amount of math and theory we had at college, but I learned to appreciate it along the years. It really makes a difference knowing why, not only how.

Knowing how gets the job done. Knowing why makes you the boss.

Quote
Anyways, Lewin was/is correct and has been all along.  :-+

One thing that intrigued me was why Mehdi had this obsession about Lewin. Why would he and Mabilde try to discredit him? They say they "respect" him, but at the same time try to convey the idea that he essentially has no clue about circuits.
 
That obviously makes no sense at all. He was teaching Maxwell's equations, which contain the most fundamental circuital laws: Faraday's and Ampere's laws. So Lewin understands everything about circuits.

If you want to really understand about circuits, look no further: study electromagnetism.

Another thing that intrigued me was why Mehdi was not honest about his own perplexity. And finally, why no one in his audience managed to point out the contradictions in his rhetoric.

Then two things came to my mind. First it was this message (https://www.eevblog.com/forum/blog/eevblog-1116-ts100-vs-ts80/msg1761842/#msg1761842), where Dave said:

Quote
The TS100 has a massive installed base of fanboys that have been pestering me for a year now to basically validate their purchase for them.

The other thing was this video titled "Why can't you go faster than light?" (https://www.eevblog.com/forum/chat/why-c-(the-speed-of-light)-must-be-limited/msg2210118/#msg2210118) published in 2017 by Fermilab. There Dr. Don Lincoln, the host, addresses the following problem:

Quote
While each of us have developed an intuition about how the world works, it's very important to remember that this intuition only applies to a very limited set of conditions. For instance, there's absolutely no reason to expect that matter will act the same in the center of the sun as it does here on Earth on a bright and sunny day. However, that last statement is hard for some people to accept and, judging by my email inbox, the extreme realm that causes people the most difficulty is what happens when things are going super-fast.

I like this shot of Dr. Lincoln's mock-up inbox taken @00:54 while he gives his talk.

(https://www.eevblog.com/forum/chat/does-kirchhoffs-law-hold-disagreeing-with-a-master/?action=dlattach;attach=672417;image)

Pay attention to the name of the fake senders and the subject of their messages.

I connected the dots and concluded that the motivation behind Mabilde's and Mehdi's videos was to get Lewin to validate their intuition. An intuition that only applies to a very limited set of conditions. Lewin refused that role and took the heat. Dr. Licoln did the same with his video, albeit more successfully.

However, it's clear that this is not an isolated phenomenon: people who do not care to master the anti-intuitive character of nature feel themselves entitled to try to validate their intuition and discredit those who refuse it.

Finally, I was intrigued about how recalcitrant people are to accept the scientific truth and ditch their pseudo-scientific claims, even after we repeatedly showed that they are utterly false. Then I came across this dialog between Dave and Shahriar in this interview (https://theamphour.com/430-shahriar-discusses-5g/) on The Amp Hour Podcast starting from 45:28, which sounds like a symphony, so timely it happened in relation to the current state of affairs:

Quote
Shahriar: We are at an age where information is free, right? This is pretty much the first time in human history, where you have access to the to the entire collective human knowledge in the palm of your hand. This is an astonishing outcome of our ingenuity, but at the same time what you also have in the palm of your hand is an unlimited set of misinformation, so what is the skill you need? Right? Thirty years ago you needed the skill to know things. Now you need the skill to know how to put things apart. That's the skill you need. So, but, are we teaching that? [...] But, if you don't teach people how to sort through information, in an information age, then things get out of control, because then you have no idea how this is going to turn out. And then we see effects of that in the world.

Dave: And people are susceptible to uh more powerful people who wanna take advantage of that fact, that people don't know how to sort out.

Shahriar: I mean it becomes so easy, right? And the issue with this is that any piece of information put on line especially in social media, social media is just, I mean, we use social media all the time, so, you know, we're bashing exactly the thing we're using, but at the same time it's such a disaster, because every piece of information that shows up, especially a person's ignorance, is just as valuable as a scientist's input, because there's no way to discern them, if you don't know how to discern them.

Dave: But it's not just that, because you can put true, you know, incontrovertible data in front of someone and they still won't believe it.

Shahriar: That's a whole other issue is that people... In order for you to change your mind about something that you fundamentally hold dear is that you have to make an emotional sacrifice because you have to abandon something you sure relied on and that can be difficult and that can be hard and more importantly you have connected with other human beings who share your point of view and now you're all of a sudden alone if you let go of that idea. [...] [P]eople build communities around information which may be false ultimately.[...]

I can't help comparing Shahriar to Mehdi. Shahriar is everything Mehdi isn't: knowledgeable, insightful, respectful of the work of scientists, open to abandon any false intuition in favor of true scientific knowledge. He doesn't need to play a clown to call the attention upon himself: he works at the Bell Labs, darn it, where every design engineer would like to be. His channel is non-for-profit because he wants to give back to the community as much as he can and because "the most wonderful thing you can do to another human being aside from loving them is teaching them" (1:14:04) . Shahriar suddenly became my own personal Jesus Christ.

So, that explains Mehdi's lying about his intuition being ratified by science. He and his audience have an enormous difficulty in accepting that their intuition is bullshit because that has an emotional cost. This is serious business especially due to Mehdi's popularity.

Shahriar and Dave are right: it doesn't help just showing the true and incontrovertible data, we need to help people discern bullshit from truth.
Title: Re: Does Kirchhoff's Law Hold? Disagreeing with a Master
Post by: HuronKing on March 10, 2019, 07:31:17 pm
Lewin actually predicted this reaction in his original lecture video with the story about the professors of physics and engineering accusing him of cheating on the demonstration years before. All he said was, "that tells you something about them... their brains couldn't handle non-conservative fields."

I will say lots of EB's audience tried to point out the contradictions to him. I was one of them. I wasn't one of his Patreon subscribers so I guess my voice might've held less weight in his private echo-chamber. This is what made me lose lots of respect for EB (and has made me unsubscribe from him). Behind his smile and faux-humble demeanor of "just asking questions" was a smugness of trying to "show Lewin his errors." Go find any of his comments to people on Reddit or those that are publicly viewable in his Patreon page about this.

He wasn't interested in learning from the professor or trying to seek a higher understanding of the phenomena. EB certainly did nothing to actually correct Lewin's supposed errors (see my remarks further below).
For example, he would've waited to complete his dialogue with Dr. Belcher before releasing a video on his channel accusing Lewin of the "bad probing" nonsense. Lewin was quite generous in this regard to direct EB to someone still actively teaching at MIT in order to answer his questions.  ::)

Or, EB would've pushed the scope of his research farther than skimming a Wikipedia article on voltage and KVL... you know, like cracking a book on Applied EM (Ulaby, Hayt, Jackson, Purcell which is available for free online, etc). Someone claiming a Masters of EE running a channel teaching people about electricity ought to have at least one lying around and some basic skill at vector calculus to understand Stokes' Theorem (also known as The Srendi Inside-Outside Theorem, lol). He would've seen the path-dependency requirement of voltage in non-conservative vector fields if Lewin's lecture wasn't thorough enough.

Or EB could've done a 2-second cursory Google search to find Romer's original paper detailing the experiment, as I did. Which would've led him to Feynman's Lectures and Feynman's EXPLICIT statements that the Lumped Model assumption only holds when time-varying magnetic fields aren't shooting through your circuit path - which isn't the case in the Romer-Lewin Ring.
I found this link below after 5 seconds of Googling:
https://courses.cit.cornell.edu/ece303/Lectures/lecture11.pdf

Like, how much preparatory research did EB actually do before essentially accusing Lewin of cheating on the experiment to millions of subscribers? I know people who seek better rigor in their research when presenting a paper to 30 other engineers at a small IEEE Conference, let alone a channel with millions of subscribers counting on well-researched information.  :-\

I was flabbergasted at the end of EB's video in which he proposes a scheme for measuring the One Ring Voltage to Rule Them All... but never performs the measurement despite having shown he has set up the apparatus to do it! It's telling to me that his original video offers no proposal of what this One Ring Voltage to Rule Them All should be. No mathematical prediction, nothing. Just an insistence on the intransigence of KVL even in time-varying magnetic fields. For someone trying to correct Lewin's errors, he didn't actually offer an alternative explanation of the phenomena being observed.
So, okay, KVL Always Hold and there is a One Ring Voltage to Rule Them All. Then show us what it is, Sauron!  ;)  ;D

There is a lot to learn from this case-study beyond just the issues with understanding non-intuitive things and hiding bad pop-pseudoscience with flashy graphics and funny editing. EB also demonstrated how NOT to research something.  :'(

So, I am grateful for your contributions Sredni and bsfeechannel. You guys have shown some very useful examples and perspectives on this phenomena (both Faraday's Law and the pseudo scientific understandings of it) I hadn't considered before. Thanks!
Title: Re: Does Kirchhoff's Law Hold? Disagreeing with a Master
Post by: jesuscf on April 07, 2019, 12:23:51 am
For those of you that keep defending Dr. Lewin's 'superior' understanding of circuit theory, please take a look at its series of videos about problem #24: 'Circuit with 5 resistors': 

https://www.youtube.com/watch?v=W9-yxYIWkqg (https://www.youtube.com/watch?v=W9-yxYIWkqg)
https://www.youtube.com/watch?v=7Y7FZ_AtrEs (https://www.youtube.com/watch?v=7Y7FZ_AtrEs)
https://www.youtube.com/watch?v=Vg_NKnmk5lc (https://www.youtube.com/watch?v=Vg_NKnmk5lc)
https://www.youtube.com/watch?v=zObhlCDjbXQ (https://www.youtube.com/watch?v=zObhlCDjbXQ)
https://www.youtube.com/watch?v=Ho97_KmZVvg (https://www.youtube.com/watch?v=Ho97_KmZVvg)

Now, tell me what is wrong with 'his' approach at solving such a simple circuit.
Title: Re: Does Kirchhoff's Law Hold? Disagreeing with a Master
Post by: EEVblog on April 07, 2019, 12:57:29 am
On the Moderation
Quote from: bsfeechannel
However we need to express our appreciation for Simon's work. I've been a moderator before. It is one of the most ungrateful tasks, let me tell you. 

I'm sorry but there's no room for appreciation for sloppy work and for the incongruent justification he gave for my ban. I could go into details, but it is clear from what he did and wrote that feedback is something to print on T-shirts and that's it. So, why waste time? He probably does not even know how his messages are rendered by browsers outside the one he uses (or that when one is banned there is no way to send PMs).

But I want to add something to explain why I deleted the technical part of my last posts and, since the thread is now dead, to address what I see as the real problem in the Lewin-Mehdi discussion. I won't get into the physics, it's not worth the time spent.

Quote from: Berni
I also don't understand what Sredni was trying to accomplish by deleting content from his posts. Basically vandalizing his own work to make it harder for someone else to follow the tread. 

Quote from: EEVBlog
Yep, I don't get it either, all that hard work gone, but we've seen this before on the forum and it usually doesn't end well unfortunately. I hope that doesn't end up being the case here. 

What do you expect? Me going Turbo and try to kill princess Vanellope?


We have seen people start off deleting their posts and then going on to do real harm to this forum as some form of vendetta.
I'm not saying you will, just saying we have seen this too many times in the past, it happens.

Quote
Nah, I simply decided not to contribute to this blog anymore than it is necessary to get my potential future questions answered. You had someone who could provide content (and believe me, I had a lot more material to share), now you have a leech.

That's a shame. But you are the one who deleted all your posts, not us.

Quote
I removed only the technical parts from my previous posts and left all that remained because I still had the curiosity to know what earned me a ban (apart from Simon to be a precog, as postulated by ogden), but it's clear that that curiosity is going to remain unanswered. Since I did not see anything in particular, I deduced that what irked the guy who reported 'the topic was getting heated' was the technical part. So I removed it. Yeah, a waste of work and time, much in the same way the time and work I had put in my last post (on when it is desirable to contain the whole field) was wasted due to the ban.
 
If the people who manage the site don't care, why should I?

We do care that you removed your content, that's a real shame. But it's you who made that decision, I hope you are not accusing us of somehow forcing you to do that?

Quote
Technically this was post 1 of 3, but I posted it as last because, you know, when it is not clear why you guys ban people, one never knows when their 'hard work' can go wasted.

When people are banned their posts are NOT deleted, so no "hard work" is lost, the content still remains for others to enjoy, learn from and discuss. You and you alone decided to delete all your posts and your hard work.
Title: Re: Does Kirchhoff's Law Hold? Disagreeing with a Master
Post by: rfeecs on April 08, 2019, 12:57:31 am
For those of you that keep defending Dr. Lewin's 'superior' understanding of circuit theory, please take a look at its series of videos about problem #24: 'Circuit with 5 resistors': 

...

Now, tell me what is wrong with 'his' approach at solving such a simple circuit.

He presented a correct solution in the first 5 minutes of the second video.  He used mesh analysis with three loops.  (Which of course involves KVL).

Then he mistakenly thought he had a very clever way of doing it with two loops.  He got the right answer but had wrong assumptions and drew some wrong conclusions.  So all the rest of the videos are his admitting he made a mistake and explaining what he did wrong.

So he made a mistake, and then corrected it.  Not a great reason for bashing him.
Title: Re: Does Kirchhoff's Law Hold? Disagreeing with a Master
Post by: jesuscf on April 08, 2019, 02:50:10 am
For those of you that keep defending Dr. Lewin's 'superior' understanding of circuit theory, please take a look at its series of videos about problem #24: 'Circuit with 5 resistors': 

...

Now, tell me what is wrong with 'his' approach at solving such a simple circuit.

He presented a correct solution in the first 5 minutes of the second video.  He used mesh analysis with three loops.  (Which of course involves KVL).

Then he mistakenly thought he had a very clever way of doing it with two loops.  He got the right answer but had wrong assumptions and drew some wrong conclusions.  So all the rest of the videos are his admitting he made a mistake and explaining what he did wrong.

So he made a mistake, and then corrected it.  Not a great reason for bashing him.

Nope.  The issue is more fundamental than that.
Title: Re: Does Kirchhoff's Law Hold? Disagreeing with a Master
Post by: Berni on April 08, 2019, 05:10:40 am
Yeah this is exactly what Kirchhoffs circuit laws are meant to be used for. Follow them correctly and they always work in ideal circuit meshes.

Tho me personally i would solve the circuit by using Thevenins theorem to create voltage dividers out of the pairs of resistors across the battery. It gets to the result by only plugging a few numbers into ohms law rather than a hefty system of equations.

Tho you will find Thenenins theorem being "for the birds" too if you suddenly start mixing it with Maxwell carelessly.
Title: Re: Does Kirchhoff's Law Hold? Disagreeing with a Master
Post by: rfeecs on April 08, 2019, 04:04:05 pm
Nope.  The issue is more fundamental than that.

Care to explain?
Title: Re: Does Kirchhoff's Law Hold? Disagreeing with a Master
Post by: jesuscf on April 08, 2019, 07:13:35 pm
Nope.  The issue is more fundamental than that.

Care to explain?

Sure thing; thanks for asking!

Dr. Lewin is using mesh analysis and making a big fuzz about 'his' solution that results in only two equations and two unknowns.  This compared to the 'brute force' method that yields three equations and three unknowns.  What Dr. Lewin doesn't seem to know is that if he uses nodal analysis he'll end up with the minimum number possible of equations and unknowns.  Always.  The difference in efficiency and simplicity is so significant, that mesh analysis should be only used in the simplest of circuits; those with only one or two meshes.  Unfortunately it seems to me that mesh analysis is the only method taught in physics courses.

For example, consider the problem Dr. Lewin is proposing, and solve it using nodal analysis:

(https://www.eevblog.com/forum/chat/does-kirchhoffs-law-hold-disagreeing-with-a-master/?action=dlattach;attach=699876)

Where I added anything in red.  The problem is to compute 'I' and 'i'.  I start by defining the unknown node voltages V1 and V2.  Also, I picked a reference node, in this case the negative terminal of the voltage source.  Now, apply KCL at nodes with voltages V1 and V2:

\$
\left\{ {\begin{array}{*{20}c}
   {i_a  - i - i_b  = 0}  \\
   {i_c  + i - i_d  = 0}  \\
\end{array}} \right.
\$

The currents can be computed from the node voltages:

\$
\left\{ {\begin{array}{*{20}c}
   {\frac{{V - V_1 }}{R} - \frac{{V_1  - V_2 }}{R} - \frac{{V_1 }}{{2R}} = 0}  \\
   {\frac{{V - V_2 }}{{2R}} + \frac{{V_1  - V_2 }}{R} - \frac{{V_2 }}{R} = 0}  \\
\end{array}} \right.
\$

Simplifying:

\$
\left\{ {\begin{array}{*{20}c}
   {5V_1  - 2V_2  - 2V = 0}  \\
   {2V_1  - 5V_2  + V = 0}  \\
\end{array}} \right.
\$

From which we find both V1 and V2:

\$
\begin{array}{l}
 V_1  = \frac{4}{7}V \\
 V_2  = \frac{3}{7}V \\
 \end{array}
\$

With the node voltages it is a piece of cake to compute the required currents:

\$
i = \frac{{V_1  - V_2 }}{R} = \left( {\frac{4}{7} - \frac{3}{7}} \right)\frac{V}{R} = \frac{1}{7}\frac{V}{R}
\$

\$
I = \frac{{V_2 }}{R} + \frac{{V_1 }}{{2R}} = \left( {\frac{3}{7} + \frac{4}{{2 \times 7}}} \right)\frac{V}{R} = \frac{5}{7}\frac{V}{R}
\$

It is also possible to quickly obtain the nodal equations by inspection, but I leave up to here.
Title: Re: Does Kirchhoff's Law Hold? Disagreeing with a Master
Post by: rfeecs on April 08, 2019, 10:38:16 pm
Nope.  The issue is more fundamental than that.

Care to explain?

Sure thing; thanks for asking!

Dr. Lewin is using mesh analysis and making a big fuzz about 'his' solution that results in only two equations and two unknowns.  This compared to the 'brute force' method that yields three equations and three unknowns.  What Dr. Lewin doesn't seem to know is that if he uses nodal analysis he'll end up with the minimum number possible of equations and unknowns.  Always.  The difference in efficiency and simplicity is so significant, that mesh analysis should be only used in the simplest of circuits; those with only one or two meshes.  Unfortunately it seems to me that mesh analysis is the only method taught in physics courses.


I agree your nodal solution is a "super" solution with only two equations and two unknowns.  Dr. Lewin definitely screwed that one up.

There are other ways to solve it as well, like Berni mentioned Thevenin equivalents.  There is also Delta to Y conversion.  I'm sure there are others that might be taught in circuit analysis classes but not in first year physics classes.

I'm not sure nodal analysis always results in the fewest equations and unknowns.  For example, this circuit I think has 9 meshes, but 11 nodes, not counting the three corner nodes:

(https://www.eevblog.com/forum/chat/does-kirchhoffs-law-hold-disagreeing-with-a-master/?action=dlattach;attach=700020;image)
Title: Re: Does Kirchhoff's Law Hold? Disagreeing with a Master
Post by: jesuscf on April 09, 2019, 12:43:37 am

I'm not sure nodal analysis always results in the fewest equations and unknowns.  For example, this circuit I think has 9 meshes, but 11 nodes, not counting the three corner nodes:


Excellent point!  I should had been more precise by indicating that the unknowns are actual branch currents and/or node voltages.  With the circuit shown and 9 meshes we can calculate 9 mesh currents, but many of those 9 mesh currents are not actual branch currents.  Therefore extra equations are needed if we want to compute them.  Actually, we can probably get away with fewer meshes/loops, but still need extra equations to compute the branch currents.  Here lies the problem with mesh analysis: what meshes should we pick?  With nodal analysis there is no guessing.  With 11 nodes we get 11 equations and 11 unknowns; even better, the unknowns are the nodal voltages.
Title: Re: Does Kirchhoff's Law Hold? Disagreeing with a Master
Post by: EEVblog on April 09, 2019, 12:55:09 am
Unfortunately it seems to me that mesh analysis is the only method taught in physics courses.

EEVuniversity teaches all three  ;D

https://www.youtube.com/watch?v=8f-2yXiYmRI (https://www.youtube.com/watch?v=8f-2yXiYmRI)
Title: Re: Does Kirchhoff's Law Hold? Disagreeing with a Master
Post by: Berni on April 09, 2019, 05:24:40 am
Yeah there are indeed many ways to tackle solving this.

Just that the mesh analysis with loops requires the least knowledge to apply. The same exact thing is done for every section of the circuit and and all of the currents pop out as a result.Tho as the circuit gets bigger and more complex the system of equations can start getting unreasonably large for calculating by hand (I always kept a graphic calculator around for solving these automagicaly).
Title: Re: Does Kirchhoff's Law Hold? Disagreeing with a Master
Post by: jesuscf on April 09, 2019, 06:24:44 am
I was planing on getting the equations for the sudoku circuit both using nodal analysis and mesh analysis but after the nodal analysis I am out off gas!  I see why people use computers to solve these things...

(https://www.eevblog.com/forum/chat/does-kirchhoffs-law-hold-disagreeing-with-a-master/?action=dlattach;attach=700245)

\$
\left\{ {\begin{array}{*{20}c}
   {\frac{{V - V_1 }}{{R_{11} }} - \frac{{V_1  - V_3 }}{{R_8 }} - \frac{{V_1  - V_2 }}{{R_4  + R_1 }} = 0}  \\
   {\frac{{V_1  - V_2 }}{{R_4  + R_1 }} - \frac{{V_2  - V_3 }}{{R_5 }} - \frac{{V_2  - V_6 }}{{R_2 }} = 0}  \\
   {\frac{{V_2  - V_3 }}{{R_5 }} + \frac{{V_1  - V_3 }}{{R_8 }} - \frac{{V_3  - V_7 }}{{R_9 }} - \frac{{V_3  - V_4 }}{{R_{12} }} = 0}  \\
   {\frac{{V - V_4 }}{{R_{15} }} + \frac{{V_3  - V_4 }}{{R_{12} }} - \frac{{V_4  - V_8 }}{{R_{16} }} - \frac{{V_4  - V_5 }}{{R_{18} }} = 0}  \\
   {\frac{{V_4  - V_5 }}{{R_{18} }} - \frac{{V_5 }}{{R_{21} }} - \frac{{V_5  - V_9 }}{{R_{22} }} = 0}  \\
   {\frac{{V_2  - V_6 }}{{R_2 }} - \frac{{V_6  - V_7 }}{{R_6 }} - \frac{{V_6  - V_{10} }}{{R_3  + R_7 }} = 0}  \\
   {\frac{{V_6  - V_7 }}{{R_6 }} + \frac{{V_3  - V_7 }}{{R_9 }} - \frac{{V_7  - V_{10} }}{{R_{10} }} - \frac{{V_7  - V_8 }}{{R_{13} }} = 0}  \\
   {\frac{{V_7  - V_8 }}{{R_{13} }} + \frac{{V_4  - V_8 }}{{R_{16} }} - \frac{{V_8  - V_{11} }}{{R_{17} }} - \frac{{V_8  - V_9 }}{{R_{19} }} = 0}  \\
   {\frac{{V_8  - V_9 }}{{R_{19} }} + \frac{{V_5  - V_9 }}{{R_{22} }} - \frac{{V_9  - V_{11} }}{{R_{23}  + R_{20} }} = 0}  \\
   {\frac{{V_6  - V_{10} }}{{R_3  + R_7 }} + \frac{{V_7  - V_{10} }}{{R_{10} }} - \frac{{V_{10}  - V_{11} }}{{R_{14} }} = 0}  \\
   {\frac{{V_{10}  - V_{11} }}{{R_{14} }} + \frac{{V_8  - V_{11} }}{{R_{17} }} + \frac{{V_9  - V_{11} }}{{R_{20}  + R_{23} }} -  = 0}  \\
\end{array}} \right.
\$


Title: Re: Does Kirchhoff's Law Hold? Disagreeing with a Master
Post by: jesuscf on April 09, 2019, 04:50:12 pm
Yeah there are indeed many ways to tackle solving this.

Just that the mesh analysis with loops requires the least knowledge to apply. The same exact thing is done for every section of the circuit and and all of the currents pop out as a result.Tho as the circuit gets bigger and more complex the system of equations can start getting unreasonably large for calculating by hand (I always kept a graphic calculator around for solving these automagicaly).

You have a very good point there. Although nodal analysis is convenient in the sense that the 'brute force' solution is often the optimal solution from the point of view of number of equations and unknowns, it requires some knowledge/expertise to deal with certain circuit configurations.  For example, a floating voltage source may add an extra equation/unknown, where the unknown is the current through the source.  Something similar applies for voltage controlled voltage sources, current controlled current sources, current controlled voltage sources, operational amplifiers, and transformers.
Title: Re: Does Kirchhoff's Law Hold? Disagreeing with a Master
Post by: Jony130 on April 13, 2019, 02:18:49 pm
Have you seen this video about KVL/Faraday's law and non-conservative field?

https://youtu.be/pUsdiIl1Kyg
Title: Re: Does Kirchhoff's Law Hold? Disagreeing with a Master
Post by: bsfeechannel on April 14, 2019, 02:55:41 am
Have you seen this video about KVL/Faraday's law and non-conservative field?

https://youtu.be/pUsdiIl1Kyg

Unfortunately this guy's "intuition" is taking him to the wrong conclusions.

Although he started with the right assumptions, he was giving various hints that sooner or later along the video he would utter some serious bullshit. And he does that exactly @9:36 when he writes elp - iR = 0.

And he concludes, very pseudo-scientifically that "This is a mix, actually, of Kirchhoff and Faraday law, because you don't see this [elp] as a discrete component here in the circuit but it is hovering around and inducing this voltage in the loop".

Note how the people who like to assert that Kirchhoff law holds for non-conservative fields give each one a different explanation for the incongruence between their claims and the results that the phenomenon is proving right in front of their noses.

He said  what he said that because he knows that the only law capable of explaining the phenomenon of induction is Faraday's law. And on top of that, there's no component on the circuit to justify Kirchhoff's EMF. But he can't accept that Kirchhoff doesn't hold for non-conservative fields.

So let's correct his mistakes.

1) elp - iR = 0 violates Kirchhoff's law, because Kirchhoff is adamant about stating that the EMF (elp) has to happen along the path of the circuit at a different place where the voltage drops occur. The EMF cannot happen inside the wire, because the wire (if considered ideal) won't allow the existence of an electric field inside it (its resistance is zero). So it happens inside the resistor, which does allow the existence of an electric field inside it.

Following Kirchhoff's instructions on how to verify his law, we have:

i*0 + i*R = elp != 0. 

¡Hasta la vista, Kirchhoff!

Now let's jump to @21:30

2) The voltage on an impedance is ONE no matter how you measure it.

Impedances are measured where there's no varying magnetic flux, so the way you measure it counts.

3) In non conservative circuits both Kirchhoff and Faraday Laws come into effect and need to be included in the analysis.

Nope. In this case Kirchhoff fails and Faraday holds. Kirchhoff is just a special case of Faraday's law when the varying magnetic flux is zero.

4) When calculating and/or measuring voltages in a non conservative circuits, care should be taken not to include non relevant magnetic flux changes.

All magnetic flux changes that happen in the area bounded by the circuit count. All the magnetic flux changes that happen outside the area bounded by the circuit don't count. Faraday's law. As simple as that.

The four equations that explain the behavior of any electromagnetic phenomenon are Maxwell's (which include Faraday's law). They have been proven time and again for more than a 100 years. So there's no place for "intuition" here, i.e., "alternative" explanations.

Everyone who tries to get around them ends up being cursed.
Title: Re: Does Kirchhoff's Law Hold? Disagreeing with a Master
Post by: ogden on April 14, 2019, 08:13:35 am
1) elp - iR = 0 violates Kirchhoff's law, because Kirchhoff is adamant about stating that the EMF (elp) has to happen along the path of the circuit at a different place where the voltage drops occur.

Here we go again    :palm:

All the wire of Lewin's circuit is EMF source, thus "wiring" is zero length point where EMF-generating wire is connected to resistor. Resistors are infinismall as well - because their dimensions are *not* specified. You agree that there is actual voltage drop on the resistor (IR) that is equal to EMF voltage created in the loop (elp), yet you say that elp - iR = 0 is bullshit. Come one... Before you decide to participate in the discussion about electromagnetic induction, make sure to learn fundamental laws of physics such as law of conservation of energy.
Title: Re: Does Kirchhoff's Law Hold? Disagreeing with a Master
Post by: bsfeechannel on April 14, 2019, 09:50:34 am
Here we go again    :palm:

Yep. While there will be people spreading pseudo-scientific claims, it is our duty to dispel them. This is not a forum of flat-earthers.

Quote
All the wire of Lewin's circuit is EMF source, thus "wiring" is zero length point where EMF-generating wire is connected to resistor. Resistors are infinismall as well - because their dimensions are *not* specified.

The wires in Lewin's circuit do have a length and so do the resistors. There's nothing "infinitesimal" there.

What amazes me about the people who claim that Kirchhoff holds for varying magnetic fields, and that Lewin's circuit is wrong, is the creativity of the wording of their explanations. Every day is a different explanation. Each one has their own. None of them makes sense, none of them agrees with each other, but they are entertaining, nevertheless.

Keep them coming.

Quote
You agree that there is actual voltage drop on the resistor (IR) that is equal to EMF voltage created in the loop (elp), yet you say that elp - iR = 0 is bullshit.

Yes. Surely. The EMF is NOT equal to the voltage drop. The EMF IS the voltage drop. There is just one voltage there. That's why the sum of the voltages in the circuit is different from zero and Kirchhoff doesn't hold.

Now it's up to you to discover how the conservation of energy holds for such a circuit (hint: Maxwell's equations).

Quote
Come one... Before you decide to participate in the discussion about electromagnetic induction, make sure to learn fundamental laws of physics such as law of conservation of energy.

I said before, I always expect you to teach me fundamental physics. It'll be fun.
Title: Re: Does Kirchhoff's Law Hold? Disagreeing with a Master
Post by: ogden on April 14, 2019, 11:41:50 am
The wires in Lewin's circuit do have a length and so do the resistors. There's nothing "infinitesimal" /there.

Are you serious or just trolling? I repeat: all the wire of the contraption is secondary winding, there's no "interconnection wire" that would not generate EMF. So we assume that it's (connection wire) length is zero. It is there only in the circuit *schematics*, not in real world. Dr.Lewin ignores EMF voltage generated *in* the resistors, so he ignores their dimensions, so they are assumed as zero. Is it so hard to comprehend?
 
Quote
Now it's up to you to discover how the conservation of energy holds for such a circuit (hint: Maxwell's equations).

Could you explain please? For simplicity there is single 1Ohm resistor, EMF is 1V, 1A is flowing for 1 sec. Please consider equation in form: power_generated - power_dissipated_in_the_resistor = 0.
Title: Re: Does Kirchhoff's Law Hold? Disagreeing with a Master
Post by: bsfeechannel on April 16, 2019, 04:18:54 am
Are you serious or just trolling?

I might be brutally honest, reveal shocking truths and bust your myths. But you know that I never troll.

Trolls want to see the world burn. They try to destroy the quality of the discussions and hence the value of the forum. I do exactly the opposite. I voluntarily spend my private time to raise the bar of the debate. Trolls don't do that.

Learn the difference, please.

Quote
I repeat: all the wire of the contraption is secondary winding, there's no "interconnection wire" that would not generate EMF.

No static wire in relation to a frame of reference can generate EMF, but I'll interpret your statement as the wire of Lewin's experiment encloses an area with a varying magnetic field and that we don't have terminals in the vicinity of which we don't have this field.

This is precisely the reason why it is impossible to lump-model this circuit.

Quote
So we assume that it's (connection wire) length is zero. It is there only in the circuit *schematics*, not in real world.

The wire represented in the schematic is real wire. He even shows that @44:32 of his lecture 16:

I have a circuit that's exactly what you have here [points at the circuit drawn on the blackboard]: nine hundred ohms and a conducting copper wire here and a hundred ohms here, and here is the solenoid [...] [points at the components of the "contraption" on the table].[/i]

Quote
Dr.Lewin ignores EMF voltage generated *in* the resistors, so he ignores their dimensions, so they are assumed as zero.

Cyriel Mabilde trying to "debunk" Lewin says the following in his video "Walter Lewin: electromagnetic induction=not for the birds(lecture 16)" @3:55:

[Lewin] draws an easy conclusion that an EMF cannot exist in the wires, but only, and only, in the resistors. I quote his own words: "The induced EMF is exclusively in the resistors"[...]

So Lewin does not ignore "the EMF generated *in* the resistors". Quite the contrary.

Quote
Is it so hard to comprehend?

Sometimes it is difficult to decipher the pseudo-scientific talk because it is always contradictory in nature.

Quote
Could you explain please? For simplicity there is single 1Ohm resistor, EMF is 1V, 1A is flowing for 1 sec. Please consider equation in form: power_generated - power_dissipated_in_the_resistor = 0.

Before we can address this problem, let's suppose that we live on a planet where g = 10 m/s². We lift an object of 1kg from the ground up to 1m. The potential energy transferred to this object will be U = mgh = 1kg*10m/s²*1m = 10 joules. If we drop down this object, when it hits the ground the potential energy will be converted to kinetic energy, which will be exactly 10 joules. Energy is conserved.

Now let's suppose that we live on a planet where g can vary. We lift the same object to a height of 1m with g = 10m/s² as before. But as soon as we drop the object, g suddenly becomes 20m/s². When the object hits the ground the kinetic energy will be 20 joules.

Where did this excess energy come from?
Title: Re: Does Kirchhoff's Law Hold? Disagreeing with a Master
Post by: Berni on April 16, 2019, 05:31:40 am
2) The voltage on an impedance is ONE no matter how you measure it.

Impedances are measured where there's no varying magnetic flux, so the way you measure it counts.

That's a new one in this thread that i don't agree in.

Why would the impedance of a passive component change in the presence of a magnetic field? The component is still passing current in the same dependence to voltage as before it was placed in a field, since this very dependence is called impedance means the impedance has not changed. If you measured it being different in a changing field then your measured quantities are likely affected by the field (Since impedance is measured indirectly from other quantities) rather than the impedance being affected.

That is unless you make your resistor out of a material that shows significant magnetoresistive effects. But as far as i know pure copper or carbon based resistors don't have any significant magnetoresistive properties. Even then they would react to DC fields too, not only AC fields.
Title: Re: Does Kirchhoff's Law Hold? Disagreeing with a Master
Post by: ogden on April 16, 2019, 05:36:07 am
Quote
Could you explain please? For simplicity there is single 1Ohm resistor, EMF is 1V, 1A is flowing for 1 sec. Please consider equation in form: power_generated - power_dissipated_in_the_resistor = 0.

Before we can address this problem, let's suppose that we live on a planet

Yes, we live on a planet. It is a fact. Are you going to explain conservation of energy using Maxwell's equations or not? Your usual goalpost shifting tactics is not funny anymore, you shall stop it.
Title: Re: Does Kirchhoff's Law Hold? Disagreeing with a Master
Post by: bsfeechannel on April 16, 2019, 05:03:49 pm

    2) The voltage on an impedance is ONE no matter how you measure it.

    Impedances are measured where there's no varying magnetic flux, so the way you measure it counts.

That's a new one in this thread that i don't agree in.

Au contraire. It is explicitly and utterly expressed in the chapter 22 of Feynman's lectures. And we've called the attention to this fact repeatedly along the thread.

Quote
Why would the impedance of a passive component change in the presence of a magnetic field? The component is still passing current in the same dependence to voltage as before it was placed in a field, since this very dependence is called impedance means the impedance has not changed. If you measured it being different in a changing field then your measured quantities are likely affected by the field (Since impedance is measured indirectly from other quantities) rather than the impedance being affected.

An impedance is Z = V/I. If you're under a varying magnetic field, the voltage will be dependent on the path. So the way you measure this voltage will affect the value you get for your impedance. In extreme cases, as Feynman points out, the varying magnetic field overcomes all the internal reactions and the component becomes a generator, which is not an impedance anymore.

Under a varying magnetic field an impedance can not only change depending on the path, but also cease to be.

Which means that, if you want a "stable" impedance, you have to measure it away from any varying magnetic fields, at its terminals, defined by Feynman as the region where you have no varying magnetic fields.

Quote
That is unless you make your resistor out of a material that shows significant magnetoresistive effects. But as far as i know pure copper or carbon based resistors don't have any significant magnetoresistive properties. Even then they would react to DC fields too, not only AC fields.

A lot of things you took for granted changes when you understand electromagnetism. It opens your mind to many new perspectives. That's the beauty of this phenomenon.
Title: Re: Does Kirchhoff's Law Hold? Disagreeing with a Master
Post by: bsfeechannel on April 16, 2019, 05:24:22 pm
Quote
Could you explain please? For simplicity there is single 1Ohm resistor, EMF is 1V, 1A is flowing for 1 sec. Please consider equation in form: power_generated - power_dissipated_in_the_resistor = 0.

Before we can address this problem, let's suppose that we live on a planet

Yes, we live on a planet. It is a fact. Are you going to explain conservation of energy using Maxwell's equations or not? Your usual goalpost shifting tactics is not funny anymore, you shall stop it.

¡No, señor! You teach me fundamental physics, and I teach you Maxwell. That's the deal. Answer my question and half of the explanation you want about Maxwell will be answered, requiring only to adapt the formalism.

Besides I'm really busy. So I'm outsourcing my reasoning to your brain and the experiment I proposed can be easily performed right here on earth. If you don't know where the extra energy comes from, just say it, there's no shame about that.
Title: Re: Does Kirchhoff's Law Hold? Disagreeing with a Master
Post by: ogden on April 16, 2019, 06:30:54 pm
Besides I'm really busy. So I'm outsourcing my reasoning to your brain and the experiment I proposed can be easily performed right here on earth. If you don't know where the extra energy comes from, just say it, there's no shame about that.

¡No, señor! You bragged to let me discover how the conservation of energy holds for such a circuit using Maxwell's equations, yet can't produce anything. Instead came-up with new "experiment" that is not even related to discussion. Please do not shift goalposts and explain yourself. Other option: you can admit that you did not know what you were talking about.  There's no shame about that.
Title: Re: Does Kirchhoff's Law Hold? Disagreeing with a Master
Post by: Berni on April 16, 2019, 07:17:32 pm
Quote
Why would the impedance of a passive component change in the presence of a magnetic field? The component is still passing current in the same dependence to voltage as before it was placed in a field, since this very dependence is called impedance means the impedance has not changed. If you measured it being different in a changing field then your measured quantities are likely affected by the field (Since impedance is measured indirectly from other quantities) rather than the impedance being affected.

An impedance is Z = V/I. If you're under a varying magnetic field, the voltage will be dependent on the path. So the way you measure this voltage will affect the value you get for your impedance. In extreme cases, as Feynman points out, the varying magnetic field overcomes all the internal reactions and the component becomes a generator, which is not an impedance anymore.

Under a varying magnetic field an impedance can not only change depending on the path, but also cease to be.

Which means that, if you want a "stable" impedance, you have to measure it away from any varying magnetic fields, at its terminals, defined by Feynman as the region where you have no varying magnetic fields.

In that case the component you are describing is no longer just a passive component.

So then could you please explain how would you measure the impedance of a real battery? (Hence it has internal resistance that can't be directly mesured) Lets also put the battery in a place where there are no magnetic fields present, so in that case we should get only one impedance value as a result, right?
Title: Re: Does Kirchhoff's Law Hold? Disagreeing with a Master
Post by: bsfeechannel on April 17, 2019, 04:30:30 am
In that case the component you are describing is no longer just a passive component.

What is the surprise?

Quote
So then could you please explain how would you measure the impedance of a real battery? (Hence it has internal resistance that can't be directly mesured) Lets also put the battery in a place where there are no magnetic fields present, so in that case we should get only one impedance value as a result, right?

Impedance is a constant coefficient of proportionality between V and I. It is in general complex number and a function of frequency. Can we talk about a constant coefficient of proportionality between voltage and current of a battery?
Title: Re: Does Kirchhoff's Law Hold? Disagreeing with a Master
Post by: Berni on April 17, 2019, 05:19:38 am
Impedance is a constant coefficient of proportionality between V and I. It is in general complex number and a function of frequency. Can we talk about a constant coefficient of proportionality between voltage and current of a battery?

Yep i agree. And to keep things from getting complicated lets suppose this battery is built up inside as 1.5V voltage source with a 1 Ohm series resistance. Such a battery would have no reactive impedance component at all frequencies (At least ones low enough to not get affected by the physical size of the battery) and has no chemistry related variation of voltage.

So then what is the correct procedure for measuring the impedance of this battery from its terminals? You can also give acual numbers in each step since all of the batteries properties are defined with values in the paragraph above.
Title: Re: Does Kirchhoff's Law Hold? Disagreeing with a Master
Post by: ogden on April 17, 2019, 05:52:50 am
Answer my question and half of the explanation you want about Maxwell will be answered, requiring only to adapt the formalism.

So your question was:

Before we can address this problem, let's suppose that we live on a planet where g = 10 m/s². We lift an object of 1kg from the ground up to 1m. The potential energy transferred to this object will be U = mgh = 1kg*10m/s²*1m = 10 joules. If we drop down this object, when it hits the ground the potential energy will be converted to kinetic energy, which will be exactly 10 joules. Energy is conserved.

Now let's suppose that we live on a planet where g can vary. We lift the same object to a height of 1m with g = 10m/s² as before. But as soon as we drop the object, g suddenly becomes 20m/s². When the object hits the ground the kinetic energy will be 20 joules.

Where did this excess energy come from?

Energy was added to the system by one who brought extra mass to the planet (https://en.wikipedia.org/wiki/Newton%27s_law_of_universal_gravitation). Now please stop playing challenge games and be so kind - explain law of conservation of energy using Maxwell's equations. I am looking for equation in form: power_generated - power_dissipated_in_the_resistor = 0.
Title: Re: Does Kirchhoff's Law Hold? Disagreeing with a Master
Post by: bsfeechannel on April 17, 2019, 07:01:20 am
Energy was added to the system by one who brought extra mass to the planet.

Thank you.

Quote
Now please stop playing challenge games and be so kind - explain law of conservation of energy using Maxwell's equations. I am looking for equation in form: power_generated - power_dissipated_in_the_resistor = 0.

Alright. Google for Poynting's theorem. Or go directly to chapter 27 of volume II of Feynman's lectures. Good read.
Title: Re: Does Kirchhoff's Law Hold? Disagreeing with a Master
Post by: bsfeechannel on April 17, 2019, 07:13:01 am
So then what is the correct procedure for measuring the impedance of this battery from its terminals? You can also give acual numbers in each step since all of the batteries properties are defined with values in the paragraph above.

Is a battery a component of an AC circuit? Do you understand in what context we can talk about impedances?  If you are in doubt I recommend you read the title of chapter 22 of Feynman's lectures, Volume II.
Title: Re: Does Kirchhoff's Law Hold? Disagreeing with a Master
Post by: ogden on April 17, 2019, 07:27:18 am
Quote
Now please stop playing challenge games and be so kind - explain law of conservation of energy using Maxwell's equations. I am looking for equation in form: power_generated - power_dissipated_in_the_resistor = 0.

Alright. Google for Poynting's theorem. Or go directly to chapter 27 of volume II of Feynman's lectures. Good read.

Poynting's theorem does not explain anything about resistor. Please come back to earth from your arrogance heights and explain w/o sending to google. Others reading this... khm... debate could be interested in **your** wizdom as well.

[edit] Ok..

As energy generated by wire loop is equal to energy dissipated in the resistor, we conclude that at any instant moment of time power delivered by wire loop is equal to the power dissipated by resistor. Now we will express what I just stated using equation: L(di/dt)*I = R*I^2. Do you agree?

Then we  divide it all by I:

L(di/dt) = R*I

It results in what I was looking for,

L(di/dt) - R*I = 0
Title: Re: Does Kirchhoff's Law Hold? Disagreeing with a Master
Post by: Berni on April 17, 2019, 07:40:47 am
Is a battery a component of an AC circuit? Do you understand in what context we can talk about impedances?  If you are in doubt I recommend you read the title of chapter 22 of Feynman's lectures, Volume II.

Yes batteries are perfectly valid in AC circuits because AC current is capable of flowing trough them. It just doesn't have any frequency dependence. Just like a resistor.

I never found any mention in that particular Feynman lecture about impedance not being applicable to components that act the same in AC and DC.

So do we get a method for measuring impedance of a battery? Or is such a thing impossible to do?
Title: Re: Does Kirchhoff's Law Hold? Disagreeing with a Master
Post by: ogden on April 17, 2019, 09:09:20 am
So do we get a method for measuring impedance of a battery? Or is such a thing impossible to do?

It is possible or not - depends on convenience of your debate opponent.

He takes sentences and words out of context, twist meaning as he pleases to suit his agenda of combative debate. Example: I say "Dr.Lewin does not account for EMF voltage generated *in* the resistors", his argument is that Dr.Lewin said: "EMF cannot exist in the wires, but only, and only, in the resistors". Dr.Lewin did not say that resistors generate EMF, he said that EMF generated by the loop of wire can be observed on the terminals of resistor. Absolutely different meaning, but hey - most readers of this thread will not even notice, right?  :-DD
Title: Re: Does Kirchhoff's Law Hold? Disagreeing with a Master
Post by: Zucca on April 17, 2019, 09:51:35 am
bsfeechannel and ogden you should meet and drink a beer together.
I guess after some time you will agree, probably only after just the first beer.

In forums it's easy to escalate for no real reasons.
Title: Re: Does Kirchhoff's Law Hold? Disagreeing with a Master
Post by: ogden on April 17, 2019, 10:04:31 am
bsfeechannel and ogden you should meet and drink a beer together.
I guess after some time you will agree, probably only after just the first beer.

So we all (including you) meet in Hannover, Germany, during next electronics trade show? ;)
Title: Re: Does Kirchhoff's Law Hold? Disagreeing with a Master
Post by: bsfeechannel on April 17, 2019, 06:28:24 pm
Yes batteries are perfectly valid in AC circuits because AC current is capable of flowing trough them. It just doesn't have any frequency dependence. Just like a resistor.

I never found any mention in that particular Feynman lecture about impedance not being applicable to components that act the same in AC and DC.

So do we get a method for measuring impedance of a battery? Or is such a thing impossible to do?

Before we do that, I need you to do me a favor.

Please find a battery whose voltage at its terminals varies like this

V(t)=Veiωt

I.e., a battery that has a sinusoidal output. Yes because that's the condition that Feynman imposed to study AC circuits, that the voltages would be sinusoidal. I'm sure you read chapter 22 carefully and paid attention to this detail.

Then I need you to establish the coefficient of proportionality between the current and the voltage of this strange battery at its terminals. You understand that a coefficient of proportionality implies that when the amplitude of the current at the terminals of the battery is zero, the amplitude of the voltage is also zero.

Doesn't need to be numeric. You can come with just an equation or something like that.

After we know what to measure, we can discuss a method to do it. What do you say?
Title: Re: Does Kirchhoff's Law Hold? Disagreeing with a Master
Post by: Berni on April 17, 2019, 07:38:08 pm
Here is the whole model with all values:
(https://www.eevblog.com/forum/chat/does-kirchhoffs-law-hold-disagreeing-with-a-master/?action=dlattach;attach=708420;image)

If you want it in that form you can take the battery voltage as being V(t)=1.5 + 0*ei*0*t. If such a thing is impossible then feel free to chose different numbers at your liking, can get rid of the DC component too if you think it matters. If a load needs to be placed across the output that is also fine as long as you specify what the load is.

What to measure is this: Whatever quantity you claim is impedance across the output terminals as a numeric value.

In the case the impedance at DC does not have a value, please explain the reason for it.
Title: Re: Does Kirchhoff's Law Hold? Disagreeing with a Master
Post by: bsfeechannel on April 17, 2019, 10:56:29 pm
Poynting's theorem does not explain anything about resistor. Please come back to earth from your arrogance heights and explain w/o sending to google. Others reading this... khm... debate could be interested in **your** wizdom as well.

That's why you can't have nice things. If I show you the equations you don't understand. If I try to explain it to you without resorting to the fast lane of math you tell me to stop it.

Quote
[edit] Ok..

As energy generated by wire loop is equal to energy dissipated in the resistor, we conclude that at any instant moment of time power delivered by wire loop is equal to the power dissipated by resistor. Now we will express what I just stated using equation: L(di/dt)*I = R*I^2. Do you agree?

Then we  divide it all by I:

L(di/dt) = R*I

It results in what I was looking for,

L(di/dt) - R*I = 0

Assuming that your reasoning were correct, since the current in the resistor is the same in the wire, di/dt = R*i/L. If i= sin(2πft), then we will have 2πfcos(2πft) = R*sin(2πft)/L => cos(2πft) = (R/2πfL) * sin(2πft). Sine and cosine are orthogonal functions, which means that you can't multiply one by a constant and have the other. So we have a problem here.

And the problem arises from the fact that we simply do not have an inductance there. The adage that any piece of wire is an inductor is a myth. A piece a wire can be an inductor under CERTAIN circumstances with which this piece wire does not comply.

So, the energy that's powering the resistor must be coming from somewhere else.

If you let me go on with my "goalpost-shifting experiment", you will understand why this piece of wire is not an inductor and how conservation of energy can be maintained even when KVL fails.
Title: Re: Does Kirchhoff's Law Hold? Disagreeing with a Master
Post by: bsfeechannel on April 17, 2019, 11:19:46 pm
So do we get a method for measuring impedance of a battery? Or is such a thing impossible to do?

It is possible or not - depends on convenience of your debate opponent.

He takes sentences and words out of context, twist meaning as he pleases to suit his agenda of combative debate. Example: I say "Dr.Lewin does not account for EMF voltage generated *in* the resistors", his argument is that Dr.Lewin said: "EMF cannot exist in the wires, but only, and only, in the resistors". Dr.Lewin did not say that resistors generate EMF, he said that EMF generated by the loop of wire can be observed on the terminals of resistor. Absolutely different meaning, but hey - most readers of this thread will not even notice, right?  :-DD

It's impossible to reason on pseudo-scientific assumptions because they lead you inevitably to contradictions. So, before we can answer anything we need to check if we are not talking nonsense.
Title: Re: Does Kirchhoff's Law Hold? Disagreeing with a Master
Post by: bsfeechannel on April 18, 2019, 01:46:17 am
Here is the whole model with all values:
(https://www.eevblog.com/forum/chat/does-kirchhoffs-law-hold-disagreeing-with-a-master/?action=dlattach;attach=708420;image)

If you want it in that form you can take the battery voltage as being V(t)=1.5 + 0*ei*0*t. If such a thing is impossible then feel free to chose different numbers at your liking, can get rid of the DC component too if you think it matters. If a load needs to be placed across the output that is also fine as long as you specify what the load is.

What to measure is this: Whatever quantity you claim is impedance across the output terminals as a numeric value.

In the case the impedance at DC does not have a value, please explain the reason for it.

The answer is: it is not possible to measure the impedance across the terminals because this is not an impedance. You see, the voltage V and the current I won't establish a constant proportion, which is a requisite for considering anything an impedance.

Connect your ohmmeter to this battery and see what it indicates.

You can however try to determine an equivalent circuit as I showed in the Reply #886, on 18 February (https://www.eevblog.com/forum/chat/does-kirchhoffs-law-hold-disagreeing-with-a-master/msg2208588/#msg2208588).

(https://www.eevblog.com/forum/chat/does-kirchhoffs-law-hold-disagreeing-with-a-master/?action=dlattach;attach=655911;image)
Title: Re: Does Kirchhoff's Law Hold? Disagreeing with a Master
Post by: Berni on April 18, 2019, 04:27:28 am
The answer is: it is not possible to measure the impedance across the terminals because this is not an impedance. You see, the voltage V and the current I won't establish a constant proportion, which is a requisite for considering anything an impedance.

Connect your ohmmeter to this battery and see what it indicates.

You can however try to determine an equivalent circuit as I showed in the Reply #886, on 18 February (https://www.eevblog.com/forum/chat/does-kirchhoffs-law-hold-disagreeing-with-a-master/msg2208588/#msg2208588).

(https://www.eevblog.com/forum/chat/does-kirchhoffs-law-hold-disagreeing-with-a-master/?action=dlattach;attach=655911;image)

Makes sense.

How about if the voltage of the battery is indeed a sine wave with no bias, defined as:
V(t)=1.5*ei*6,28*t

Is there an impedance then?


And the problem arises from the fact that we simply do not have an inductance there. The adage that any piece of wire is an inductor is a myth. A piece a wire can be an inductor under CERTAIN circumstances with which this piece wire does not comply.

Btw i am interested in what exactly are these certain circumstances when a piece of wire suddenly gets the properties of inductance and when it does not.
Title: Re: Does Kirchhoff's Law Hold? Disagreeing with a Master
Post by: ogden on April 18, 2019, 01:51:28 pm
If I show you the equations you don't understand.

You clearly have issues. Anyway thank you, I will try harder :D

Quote
The adage that any piece of wire is an inductor is a myth.

Problem with your reasoning is that I was not explicit about material of wire or magnetic core or absence of it. Ok, fine. Forget about inductor. - You apparently have problems to fathom not only non-conservative versus conservative fields, but true power versus reactive power as well. Let's assume that variable magnetic field just happens and it induces 1V EMF in the loop, EMF equals (eip). Now what is *correct* (mine was incorrect as you say) equation that shows law of conservation of energy between wire loop and resistor? Just try me and produce equation even if in doubt that anybody except you will understand it.

It's impossible to reason on pseudo-scientific assumptions because they lead you inevitably to contradictions. So, before we can answer anything we need to check if we are not talking nonsense.

Discussion about some property of experiment or part of it is not nonsense, especially if property is so small that it does not affect accuracy of measurements. Dr.Lewin ignores resistance of wire - it is scientific and accepted by you. When I say that EMF generated in the (by the) resistors are ignored by Dr.Lewin - just because dimensions of resistors are ignored as well, you say it is pseudoscience and nonsense  |O Take a moment and look from different angles on what I just said.
Title: Re: Does Kirchhoff's Law Hold? Disagreeing with a Master
Post by: jesuscf on April 18, 2019, 06:37:23 pm
Allow me to falsify Dr. Lewin's experiment.  I'll start with two pieces of wire and two ordinary 5% 1/4W resistors one order of magnitude apart, say 10k\$\Omega\$ and 100k\$\Omega\$.  The resistance of the two pieces of wire are about 0.25\$\Omega\$, so I think we can ignore their resistances for this experiment.  I also attached alligator clips to the pieces of wire to allow for easy and fast connections.

(https://www.eevblog.com/forum/chat/does-kirchhoffs-law-hold-disagreeing-with-a-master/?action=dlattach;attach=709227;image)

Without  the presence of any electro-magnetic fields that are not natural, such as the earth magnetic field, (I didn't put an inductor in the middle of the ring) I proceeded to measure, and this it what I got:

(https://www.eevblog.com/forum/chat/does-kirchhoffs-law-hold-disagreeing-with-a-master/?action=dlattach;attach=709233;image)

What?  The voltage across the 10k\$\Omega\$ resistor is -0.111mV while the voltage across the 100k\$\Omega\$ resistor is 1.058mV.  Very similar to what Dr. Lewin's got in his experiment.  Then, Kirchhoff's voltage law doesn't work under this mysterious conditions?!  I am not so sure about that, just to be certain allow me to measure the voltage across the two pieces of wire:

(https://www.eevblog.com/forum/chat/does-kirchhoffs-law-hold-disagreeing-with-a-master/?action=dlattach;attach=709239;image)

As you can see, there is a voltage across the bottom wire! (Second multi-meter from the top).  If you add the voltages of the four multi-meters the total should be zero.  The problem here is that these multi-meters are out of calibration and also have been through horrible things in the last 10 years, so I didn't expect to have 'perfect' results.  Nevertheless the result is illustrative enough.  The point here is that somehow, there is a voltage induced in the bottom wire.  In this case, the voltage is not induced using an external magnetic field.  What I didn't tell you at the beginning, is that one of the wires is made of the alloy chromel while the other wire is made of the alloy alumel which I removed from a k-type thermocouple.  To produce a voltage, I heated up one of wires with a soldering iron as shown below.

(https://www.eevblog.com/forum/chat/does-kirchhoffs-law-hold-disagreeing-with-a-master/?action=dlattach;attach=709245;image)
Title: Re: Does Kirchhoff's Law Hold? Disagreeing with a Master
Post by: bsfeechannel on April 18, 2019, 10:37:08 pm
Makes sense.

Of course it does.

Quote
How about if the voltage of the battery is indeed a sine wave with no bias, defined as:
V(t)=1.5*ei*6,28*t

Is there an impedance then?

Nope. I regret to tell you. The voltage will be defined by the "battery" and the current by whatever load you connect to its terminals, so there won't be a fixed coefficient of proportionality between the amplitude of the voltage and the amplitude of the current.

You can try to calculate or measure it.

Quote
Btw i am interested in what exactly are these certain circumstances when a piece of wire suddenly gets the properties of inductance and when it does not.

Glad you asked. Take a linear transformer. The secondary of a transformer will be a piece of wire wound around some core, be it air or whatever. But to simplify our reasoning, let's suppose that the core is vacuum.

Well, a piece of wire wound around a core? You say. It is an inductor, isn't it? Of course it is. Take whatever LCR meter you have and connect it to the terminals of this secondary and it'll confirm that.

You can even calculate and measure its impedance  for whatever frequency you want. |Z| = |V|/|I| = 2πfL.

Now connect the primary to a voltage source. A voltage will appear at the terminals of the secondary, won't it? But wait a minute, nothing is connected to the secondary, so the current is zero. If I take this voltage and divide it by the current I will have |Z| = |V|/0, which is undefined.

Don't pull your hair out yet. Suppose that I now connect a resistive load. The resistor will force the current to be in phase with the voltage. The same current that is traversing the wire of the secondary. But in an inductor shouldn't the current be lagging the voltage by 90°?

Yet you have a coiled piece of wire where you have an alternate current in phase with the voltage at its terminals. So much for our inductor!

So, how can it be? What changed from the condition where the primary was disconnected to when it was connected? Certainly it was not the wire, because we haven't touched it. Something else must be producing this change.

What is it?
Title: Re: Does Kirchhoff's Law Hold? Disagreeing with a Master
Post by: bsfeechannel on April 19, 2019, 12:02:47 am
You clearly have issues.

I have many issues, but electromagnetism is not one of them (at least not in the level we are discussing).

Quote
Anyway thank you, I will try harder :D

The world is grateful.

Quote
Problem with your reasoning is that I was not explicit about material of wire or magnetic core or absence of it. Ok, fine. Forget about inductor. - You apparently have problems to fathom not only non-conservative versus conservative fields, but true power versus reactive power as well. Let's assume that variable magnetic field just happens and it induces 1V EMF in the loop, EMF equals (eip). Now what is *correct* (mine was incorrect as you say) equation that shows law of conservation of energy between wire loop and resistor? Just try me and produce equation even if in doubt that anybody except you will understand it.

Your problem is that you think that electricity and magnetism are properties of circuits. You are a circuit head, a circuity guy. You are forever condemned to think electricity and magnetism in terms of circuits and that's crippling your brain. You treat non conservative fields as if they were conservative, and that's why you think that the energy must have come from some component in the circuit, the wire, for instance.

Ditch that bullshit right away.

Here, lemme help you. In the "goalpost-shifting" problem that I proposed to you, the 1kg object that had 10J of potential energy suddenly acquired 10J more. You said that the energy was provided by someone who doubled the mass of the planet. That might have taken a lot of energy. However only 10J was added to the object. We haven't touched it, i. e., we haven't moved it to a higher position or pushed it down. What exactly gave the extra energy to the object? It was the? Anyone? Anyone? (Hint: it starts with an F).

Quote
Dr.Lewin ignores resistance of wire - it is scientific and accepted by you.

That's OK. The resistance in the wire is negligible. Who cares?

Quote
When I say that EMF generated in the (by the) resistors are ignored by Dr.Lewin - just because dimensions of resistors are ignored as well, you say it is pseudoscience and nonsense  |O

The EMF cannot be generated BY the resistors. However it is generated IN the resistors. The dimensions of the resistors are absolutely irrelevant. What counts is the path of the circuit. That's Maxwell's equations. End of story.

Quote
Take a moment and look from different angles on what I just said.

The angle of science is just one. The angles of pseudo-science are many. Each one is free to cook up their own contradictory explanations. I can't keep up with all of them.
Title: Re: Does Kirchhoff's Law Hold? Disagreeing with a Master
Post by: HuronKing on April 19, 2019, 01:06:47 am
I would add something... but you're doing fine bsfeechannel.
Title: Re: Does Kirchhoff's Law Hold? Disagreeing with a Master
Post by: bsfeechannel on April 19, 2019, 01:54:22 am
Thank you HuronKing. By all means feel free to add whatever you deem relevant. I want to learn too.
Title: Re: Does Kirchhoff's Law Hold? Disagreeing with a Master
Post by: HuronKing on April 19, 2019, 03:14:11 am
The only thing I might want to add is to make even more explicit the nature of non-conservative forces. This isn't some weird voodoo-freaky electromagnetism thing. In Classical Electrodynamics, the analogy that led Maxwell to his discoveries were hydrodynamic vortices. Maxwell's initial picture was inaccurate and Heaviside placed them in the vector form we know and love, but it's useful to go back to what techniques are used to solve explicitly non-conservative problems. Like, friction forces:

https://opentextbc.ca/physicstestbook2/chapter/nonconservative-forces/

Curious note, none of these simple example problems move all the values over to one side and set the whole thing equal to zero. That is, it's not 5+3-8 = 0.

Two things I think are worth noting in these alternative explanations to Faraday's Law (beautiful as the law is):
1) The mathematical contradiction inherent in assuming a value of L just randomly exists as you demonstrated aptly.
2) The assumption that if a mass has more kinetic energy when it hits the ground than it's gravitational potential energy MUST mean someone doubled the mass of the planet.  :o

I admit that last one made me chuckle.
Title: Re: Does Kirchhoff's Law Hold? Disagreeing with a Master
Post by: seagreh on April 19, 2019, 03:41:41 am
bsfeechannel,

Since it starts with an F, it is Michael Faraday!
He had his inductive charger with him and pumped a 10 Joules into that‘s things battery! He hasn‘t touched the thing, he hasn‘t moved it neither.
10 seconds charging only. Smart guy.
Kirchhoff - not being aware about inductive chargers - would have tried to bring Mars and Venus over to our planet Earth, to double its mass.

As You can keep up with all angles - that of science and those of pseudo-science...

You say, the EMF is generated IN the resistor ! Hmmm. The resistor being the seat of EMF? You are sure? Otherwise you state correctly, what counts is the path.
Actually, the path itself is already enough for the electric field, won’t even need a wire. But not good enough for the EMF, needing a ‚wire’ as well!

Imagine a super-conductive ring with a single tiny 10k resistor inserted.
You say the EMF sits in the resistor. Now, removing the resistor, would remove the EMF as well ? Or does the EMF now jump into the air gap. Can vacuum as well be a seat for EMF?
Now, let’s close the super conductive ring - short cutting the resistor. Have we killed the EMF now? Assume the ring sits in an environment where a magnetic flux of 1 Wb changes to 0 Wb within 1 second! All the flux of 1 Wb streams through our ring (don‘t care how much energy it takes to do that). Still 0V EMF, since no resistor ? Or could this violate the definition of 1 Wb?

Let‘s assume the EMF sitting in the resistor. Wouldn‘t it then compensate with the voltage drop within the resistor - the resistor being a closed system with energy sourcing part and energy dissipating part in itself? Wouldn‘t this result in 0V at the resistor terminals ? Actually fitting the 0V along your superconductive ring ?

Are you still 100% sure, this is not a pseudo scientific angle of view?
Many people have a hard times to understand the difference between an EMF and a voltage drop. The voltage drop represents the energy dissipating part, the EMF the energy sourcing part. The EMF can be easily identified by transporting charges from negative terminal to positive terminal, resulting in current flow in opposite direction of voltage drop....
I hear you saying - come on, don‘t try to teach me basics...
But on the other hand you say the EMF is NOT equal to the voltage drop. The EMF IS the voltage drop you say! Hmmm.
As I said before, many pople have hard times to understand the difference between EMF and voltage drop!

And maybe Kirchhoff could help you here :-)
As you read his original paper, you must have seen, Kirchhoff‘s intention was not to summarize whole complete world and ‚zerorize‘ it! He simply summed up voltage drops on one side of the equation and summed up EMFs on the other side of the equation. There was no zero in his equation. Now, EMFs on the right side could be coulomb EMFs and non-coulomb EMFs (e.g. Faraday‘s - dPhi/dt).

Hence Prof. Sam Ben-Yaakov is right -

EMF (being -dPhi/dt) = iR (being the line integral of E dl).

Left side of the equation being the energy sourcing part (EMF) being equal to energy consuming part (right side).
Title: Re: Does Kirchhoff's Law Hold? Disagreeing with a Master
Post by: jesuscf on April 19, 2019, 06:34:01 am
This is from chapter 13 of "Engineering Electromagnetics" by Hyat, third edition.

THE LAWS OF CIRCUIT THEORY

To indicate how Maxwell’s equations, the potential definitions, and the concepts of resistance, capacitance, and inductance combine to produce the common expressions of circuit analysis, consider the configuration shown in Fig. 13.1 Between the two points 0 and 1 an external electric field is applied.  These terminals are very close together, and a sinusoidal electric field may be assumed.  Perhaps we could visualize a microscopic transistor oscillator, a rotating machine the size of a pinhead (complete with prime mover), or even a cooperative flea moving a mouthful of charge alternately toward point 1 and point 0.  Whatever the nature of the source, it continues to produce an electric field between these two terminals that is independent of any currents that may consequently flow.  Between points 2 and 3 there is a region of lossy material of cross-sectional area \$S_R\$, length \$d_R\$, and conductivity \$\sigma\$.  At points 4 and 5 are located two capacitor plates of area \$S_c\$, with separation \$d_c\$ and dielectric of permittivity \$\epsilon\$.  These several points are connected as shown by a filamentary perfect conductor of negligible cross section.  Between points 6 and 8 the filament is wound into a helix of N turns having a very fine pitch.

(https://www.eevblog.com/forum/chat/does-kirchhoffs-law-hold-disagreeing-with-a-master/?action=dlattach;attach=709632;image)

It is probably obvious that we are now going to develop the familiar circuit equation

(1) \$V_{10}  = IR + L_{ind}\frac{{dI}}{{dt}} + \frac{1}{C}\int\limits_{ - \infty }^t {Idt}\$

from Maxwell’s equations.   As we do so, it is interesting to watch how each of these terms arises and what assumptions we have to make in the process.  Our beginning point is the integral from of Faraday’s law,

(2) \$\oint {E \cdot dL = }  - \frac{\partial }{{\partial t}}\int\limits_S {B \cdot dS}\$

We shall see that the right side of this equation provides us with only one term in (1), that involving the inductance.  The other three terms all arise from the closed line integral.

Let us consider the surface integral on the right side of (2).  Since the configuration of the circuit does not change with time, the partial derivative may be replaced with the ordinary derivative.  Also, the filament between points 6 and 8, an N-turn helix, serves to produce a much larger magnetic field within the helix than in any other region along the filament.  If we assume that a total magnetic flux \$\Phi \$ links all N turns, the surface integral becomes  \$ - \frac{{d\Phi }}{{dt}}\$, or  \$ - L_{ind}\left( {\frac{{dI}}{{dt}}} \right)\$ by the definition of inductance, and

\$\oint {E \cdot dL =  - L_{ind}\frac{{dI}}{{dt}}} \$

where I is the filamentary current in each turn of the helix.

The closed line integral is taken along the filament, directly between the capacitor plates and points 0 and 1, as indicated by the dashed line.  The contribution from the perfectly conducting filament is zero, because tangential E must be zero there; this includes the helix, surprising as that may be.  We therefore have

\$\oint {E \cdot dL = \int\limits_0^1 {} }  + \int\limits_2^3 {}  + \int\limits_4^5 {} \$

The first integral on the right is the negative of the voltage between points 1 and 0,

\$\int\limits_0^1 {E \cdot dL =  - V_{10} }\$

This integral is a function only of the external source and does not depend on the configuration shown in figure 13.1.  The path is directly between the adjacent terminals, and since we are more used to considering an external source as a voltage than as an electric field intensity, we usually call \$V_{10} \$ an applied voltage.

The second integral is taken across the lossy material, and we apply Ohm’s law in point form and the definition of total resistance R,

\$\int\limits_2^3 {E \cdot dL = } \int\limits_2^3 {\frac{J}{\sigma } \cdot dL = } \int\limits_2^3 {\frac{{JdL}}{\sigma } = } \frac{{Jd_R }}{\sigma } = \frac{{Id_R }}{{\sigma S_R }} = IR\$

The same total current I is assumed, and this justified only when two conditions are met.  There can be no displacement currents flowing from one point of the filament to another (such as from point 3 to 8 ), because we require the continuity of conduction plus displacement current density to be satisfied by conduction current alone.  In other language, we are assuming that stray capacitances are neglected.  Also the dimension of the filamentary path must be small compared to a wavelength.  This will be applied in the final section of this chapter, but our study of wave motion should indicate the complete reversal which may occur in afield over a distance of one-half wavelength.  Here we wish to avoid radiation, but later in the chapter it will provide our main item of interest.

The third integral is evaluated across the region between the capacitor plates where conduction current is zero but displacement current is equal to the current I, as we assumed earlier.  Here we may represent the integral by

\$\int\limits_4^5 {E \cdot dL = } \int\limits_4^5 {\frac{D}{\varepsilon } \cdot dL = } \frac{{Dd_C }}{\varepsilon } = \frac{{Qd_C }}{{\varepsilon S_C }} = \frac{Q}{C}\$

or

\$\int\limits_4^5 {E \cdot dL = } \frac{1}{C}\int\limits_{ - \infty }^t {Idt} \$

where we assume that there is no charge on the capacitor at \$t =  - \infty \$.

Combining these results, we have

\$ - V_{10}  + IR + \frac{1}{C}\int\limits_{ - \infty }^t {Idt}  =  - L_{ind}\frac{{dI}}{{dt}}\$

or

\$V_{10}  = IR + L_{ind}\frac{{dI}}{{dt}} + \frac{1}{C}\int\limits_{ - \infty }^t {Idt}\$

which is the familiar equation for an RLC series circuit that we hoped we would obtain.

Title: Re: Does Kirchhoff's Law Hold? Disagreeing with a Master
Post by: Berni on April 19, 2019, 06:41:54 am
Nope. I regret to tell you. The voltage will be defined by the "battery" and the current by whatever load you connect to its terminals, so there won't be a fixed coefficient of proportionality between the amplitude of the voltage and the amplitude of the current.

You can try to calculate or measure it.

This was exactly my point. It doesn't make sense to talk about a voltage source as purely being an impedance.

When analyzing the circuit it makes more sense to think of it as an equivalent circuit. This separates the voltage source from the impedance part, allowing both to remain the same no matter the load.

Or is the use of equivalent models somehow forbidden in circuit analysis?

Quote
Btw i am interested in what exactly are these certain circumstances when a piece of wire suddenly gets the properties of inductance and when it does not.

Glad you asked. Take a linear transformer. The secondary of a transformer will be a piece of wire wound around some core, be it air or whatever. But to simplify our reasoning, let's suppose that the core is vacuum.

Well, a piece of wire wound around a core? You say. It is an inductor, isn't it? Of course it is. Take whatever LCR meter you have and connect it to the terminals of this secondary and it'll confirm that.

You can even calculate and measure its impedance  for whatever frequency you want. |Z| = |V|/|I| = 2πfL.

Now connect the primary to a voltage source. A voltage will appear at the terminals of the secondary, won't it? But wait a minute, nothing is connected to the secondary, so the current is zero. If I take this voltage and divide it by the current I will have |Z| = |V|/0, which is undefined.

Don't pull your hair out yet. Suppose that I now connect a resistive load. The resistor will force the current to be in phase with the voltage. The same current that is traversing the wire of the secondary. But in an inductor shouldn't the current be lagging the voltage by 90°?

Yet you have a coiled piece of wire where you have an alternate current in phase with the voltage at its terminals. So much for our inductor!

So, how can it be? What changed from the condition where the primary was disconnected to when it was connected? Certainly it was not the wire, because we haven't touched it. Something else must be producing this change.

What is it?

Oh we are already at transformers from inductance, good to hear. Yep we all agree that a coil of wire is indeed an inductor.

What you describe is called mutual inductance in circuit analysis. As soon as inductors start sharing magnetic fields they also start sharing there inductance too. The total inductance of the coil stays the same so as mutual inductance increases the self in distance decreases. In transformers this self inductance is actually called leakage inductance. Here is a quick summary of how this works: https://physics.stackexchange.com/questions/119638/choosing-sign-for-kvl-mutual-inductance

So now how does the transformer secondary not have 90 degree phase shift if its an inductor? Because the voltage is all coming from mutual part of inductance and this inductance only cares about the current in the primary. The current trough the load resistor actually affects the voltage of the primary side. Once all of these currents and voltages are summed up neither the primary or secondary have 90 degree phase shifts anymore. In an ideal 1:1 transformer with no leakage the currents and voltages are actually all perfectly in phase (If the load is a resistor like in your example) or 180 out of phase depending on what way around you connect the coil.

All my hair is still in its place as there is no need to pull it out over a concept that works just fine, or is the use of mutual inductance somehow forbidden?

So then back to the original question. Under what circumstances is a wire not an inductor and when it is? Or does only self inductance count? If not then at what coupling factor (k) does it stop counting as an inductor? 0.5? 0.1? 1E-6?

EDIT: Oh and btw and the ideal 1:1 transformer above also has infinite inductance. For a real transformer with a finite inductance the primary inductance still takes a current according to its impedance, this is the magnetizing current for a transformer and its what limits the lowest usable frequency for the transformer. This current is indeed lagging 90 degrees, but once the load current joins it then the total sum current has a phase shift less than 90.(Tho the currents don't literally sum due to where the leakage inductance is)
Title: Re: Does Kirchhoff's Law Hold? Disagreeing with a Master
Post by: jesuscf on April 19, 2019, 04:03:29 pm
So then back to the original question. Under what circumstances is a wire not an inductor and when it is? Or does only self inductance count? If not then at what coupling factor (k) does it stop counting as an inductor? 0.5? 0.1? 1E-6?

This wire inductance calculator may be useful for the person this question was directed:

https://www.eeweb.com/tools/wire-inductance (https://www.eeweb.com/tools/wire-inductance)

Title: Re: Does Kirchhoff's Law Hold? Disagreeing with a Master
Post by: bsfeechannel on April 19, 2019, 04:14:05 pm
Since it starts with an F, it is Michael Faraday!

That's a good one. But nope. It's not Faraday.

Quote
You say, the EMF is generated IN the resistor ! Hmmm. The resistor being the seat of EMF? You are sure?

Absolutely sure.

Quote
Imagine a super-conductive ring with a single tiny 10k resistor inserted.
You say the EMF sits in the resistor. Now, removing the resistor, would remove the EMF as well ? Or does the EMF now jump into the air gap. Can vacuum as well be a seat for EMF?

The resistor happens to be where the EMF is. If you remove the resistor, the EMF stays in the same place. You don't need a superconductor to prove this. Just connect a voltmeter to the secondary of a transformer capable of providing a sufficiently large current. Then connect a 10k resistor. The voltage won't change.

Quote
Let‘s assume the EMF sitting in the resistor. Wouldn‘t it then compensate with the voltage drop within the resistor - the resistor being a closed system with energy sourcing part and energy dissipating part in itself? Wouldn‘t this result in 0V at the resistor terminals ? Actually fitting the 0V along your superconductive ring ?

Sensational! I was waiting for someone to come up with exactly that "explanation". The EMF and the voltage drop cancelling each other inside a resistor. You have a current induced through the resistor and its voltage is zero.  It is pseudo-science in all its splendor. You can't blame them for their creativity. I love it.

The EMF is path-dependent. So following the path inside the conductor it is zero. Following the path along the resistor it is non zero. Faraday's law. There's no way around it.

Quote
As I said before, many pople have hard times to understand the difference between EMF and voltage drop!

Electromagnetism, the fundamentals of our trade, is not intuitive. You have to use reason to understand it. Tell these people to study.

Quote
And maybe Kirchhoff could help you here :-)
As you read his original paper, you must have seen, Kirchhoff‘s intention was not to summarize whole complete world and ‚zerorize‘ it! He simply summed up voltage drops on one side of the equation and summed up EMFs on the other side of the equation. There was no zero in his equation. Now, EMFs on the right side could be coulomb EMFs and non-coulomb EMFs (e.g. Faraday‘s - dPhi/dt).

Hence Prof. Sam Ben-Yaakov is right -

EMF (being -dPhi/dt) = iR (being the line integral of E dl).

Left side of the equation being the energy sourcing part (EMF) being equal to energy consuming part (right side).

The problem has nothing to do with where you place the terms of the equation. This is just a convention. The problem lies in the fact that Kirchhoff demands that EMFs and voltage drops must be found in separate places IN the circuit. Using the convention Kirchhoff did, the sum of all voltage drops must be equal to the sum of all EMFs. Or, using our modern convention, all the voltages FOUND IN THE CIRCUIT, i.e. directly measured, (EMFs and drops) must add up to zero.

Prof. Sam Ben-Yaakov says, to the add more confusion to the mess:

"This is a mix, actually, of Kirchhoff and Faraday law, because you don't see this [elp] as a discrete component here in the circuit but it is hovering around and inducing this voltage in the loop".

If Kirchhoff held in this case, then elp should be found in the circuit as a separate entity. But it obviously isn't. And he even declares that ("you don't see", "it is hovering around").

So he contradicts Kirchhoff, who demands the voltage to be explicit.

Walter Lewin is a vehement critic of this kind of people espousing those ideas for obvious reasons: it not only causes all kinds of misconceptions but is also completely pseudo-scientific. In short it is WRONG.

Conventionally today the voltages (emfs or voltage drops) found in the circuit are placed to the left of the equation. To the right is the account of the effect of induction. If there's no induction, all the voltages add up to zero and KVL holds. If there is induction, the voltages will add up to the calculation indicated by the right side, and Kirchhoff doesn't hold anymore.

The calculation to the right side is not a voltage that you'll find in the circuit (as Prof. Ben-Yaakov confirmed), which means that you cannot conventionally place it to the left side as if you'd find that with your voltmeter. This is simply stupid. (And it is even "stupider" to look at the zero left to the right of the equation after this mathematical bodge and declare because of this that Kirchhoff holds.)

In fact, none of the (duly debunked) "debunkers" of Lewin managed to measure this voltage in any of their circuits.

Νot even this unfortunate professor.
Title: Re: Does Kirchhoff's Law Hold? Disagreeing with a Master
Post by: bsfeechannel on April 19, 2019, 04:16:40 pm
bsfeechannel and ogden you should meet and drink a beer together.
I guess after some time you will agree, probably only after just the first beer.

In forums it's easy to escalate for no real reasons.

ogden needs to agree with nature, not with me.

I would have no problem drinking beer with anyone in the forum. I have no hard feelings against ogden or anyone else. Since Shahriar said that the next thing you could do to a person besides loving them is teaching them, I think that by this time I must be madly in love with ogden (Berni comes second).
Title: Re: Does Kirchhoff's Law Hold? Disagreeing with a Master
Post by: jesuscf on April 19, 2019, 04:37:36 pm
Conventionally today the voltages (emfs or voltage drops) found in the circuit are placed to the left of the equation. To the right is the account of the effect of induction. If there's no induction, all the voltages add up to zero and KVL holds. If there is induction, the voltages will add up to the calculation indicated by the right side, and Kirchhoff doesn't hold anymore.

The calculation to the right side is not a voltage that you'll find in the circuit (as Prof. Ben-Yaakov confirmed), which means that you cannot conventionally place it to the left side as if you'd find that with your voltmeter. This is simply stupid. (And it is even "stupider" to look at the zero left to the right of the equation after this mathematical bodge and declare because of this that Kirchhoff holds.)

In fact, none of the (duly debunked) "debunkers" of Lewin managed to measure this voltage in any of their circuits.

Νot even this unfortunate professor.

I just want to preserve this for posterity in case the original poster decides to erase it once he realizes how massively incorrect it is.
Title: Re: Does Kirchhoff's Law Hold? Disagreeing with a Master
Post by: ogden on April 19, 2019, 07:04:12 pm
In fact, none of the (duly debunked) "debunkers" of Lewin managed to measure this voltage in any of their circuits.

Every AC transformer in operation by itself is proof that you are mistaken.

The resistor happens to be where the EMF is. If you remove the resistor, the EMF stays in the same place. You don't need a superconductor to prove this. Just connect a voltmeter to the secondary of a transformer capable of providing a sufficiently large current. Then connect a 10k resistor. The voltage won't change.

Seems, you accidentally disproved yourself - because in case you just described, KVL holds.

2) The assumption that if a mass has more kinetic energy when it hits the ground than it's gravitational potential energy MUST mean someone doubled the mass of the planet.  :o
I admit that last one made me chuckle.

Your self-embarrassment could be avoided if you would read original post carefully meaning all the provided info, not just last words:

Now let's suppose that we live on a planet where g can vary. We lift the same object to a height of 1m with g = 10m/s² as before. But as soon as we drop the object, g suddenly becomes 20m/s². When the object hits the ground the kinetic energy will be 20 joules.
Title: Re: Does Kirchhoff's Law Hold? Disagreeing with a Master
Post by: Berni on April 19, 2019, 07:28:00 pm
I do sometimes end up at these electronics trade shows in Germany. Always up for a chat if the timing is right, the accompanying beer is even free if you find the right booth handing it out.

Tho lately I'm finding it a bit hard to even tell what everyone is even arguing about in this thread. It feels like people are contradicting each other simply for the sake of contradicting them. Tho surprisingly my explanation of mutual inductance hasn't been contradicted yet, but its probably just a matter of time until someone disagrees with at least part of it.

Still fun thread to read tho.
Title: Re: Does Kirchhoff's Law Hold? Disagreeing with a Master
Post by: seagreh on April 19, 2019, 10:00:12 pm
You say, the EMF is generated IN the resistor ! Hmmm. The resistor being the seat of EMF? You are sure?

Absolutely sure.

Quote
Imagine a super-conductive ring with a single tiny 10k inserted.
You say the EMF sits in the resistor. Now, removing the resistor, would remove the EMF as well ? Or does the EMF now jump into the air gap. Can vacuum as well be a seat for EMF?

The resistor happens to be where the EMF is. If you remove the resistor, the EMF stays in the same place. You don't need a superconductor to prove this. Just connect a voltmeter to the secondary of a transformer capable of providing a sufficiently large current. Then connect a 10k resistor. The voltage won't change.

I did your experiment. At the begin of the experiment the EMF must have been in the air gap between both secondary terminals. However, as soon as I connected the voltmeter, I got the feeling the EMF jumped into my voltmeter (as it is 10MOhm resistive. As you say EMF is path dependent). With the 10k resistor I was expecting it to sit now in the Voltmeter AND in the resistor at the same time - according to your thesis.

And I always thought, the EMF is defined as the tangential force per unit charge in the wire integrated over length, once around the complete circuit/loop! But not at all, it was hiding in the resistor and my voltmeter!

But you confused me a bit later, when talking about the [elp]. The [elp] being the EMF (I guess the Israeli Prof meant [Epsilon loop] with this abbreviation). You tell me it’s invisible and can’t be measured? Although you told me before it’s sitting in the resistor and how to measure it?

On the other hand you say today’s convention is to put voltage drops AND EMFs on the left side. But insist at the same time to keep the -dFlux/dt on the right side? -dFlux/dt being the EMF.
Looks like you don’t consider -dFlux/dt as EMF, but rather the EMF as part of the line integral E ds?
Title: Re: Does Kirchhoff's Law Hold? Disagreeing with a Master
Post by: Sredni on April 20, 2019, 12:52:42 am

Quote
Nah, I simply decided not to contribute to this blog anymore than it is necessary to get my potential future questions answered. You had someone who could provide content (and believe me, I had a lot more material to share), now you have a leech.

That's a shame. But you are the one who deleted all your posts, not us.

Not all of them.

Quote from: EEVblog
We do care that you removed your content, that's a real shame. But it's you who made that decision, I hope you are not accusing us of somehow forcing you to do that?

When people are banned their posts are NOT deleted, so no "hard work" is lost, the content still remains for others to enjoy, learn from and discuss. You and you alone decided to delete all your posts and your hard work.

Again, not all of them. The reasons I deleted my posts was twofold: one I explained in my previous post, and the other was to give you a taste of your medicine. When you ban someone when he's writing a post, that someone finds out to have been banned only when he hits the "Post" button.
Result: the post he has just written is lost, wasted, deleted.
Now, if the ban was rightful, I could tolerate that. But I did not see any explanation for my ban, except for the one I assumed. That is, puncturing the ego of someone who DID NOT QUALIFY AS A MODERATOR (to me that was some bloke cracking a joke, and I treated him as such), and made no specific requests as to what we should have done. Stop posting? Saying the counterpart was right? Take a bow?
Do you even know how the posts in this forum are rendered in an Android browser? There is no "moderator" tag.
So, learn to do your 'job' before wasting other people's time.

A site like this exists because of the content people contribute. If you show you do not care about the effort they put into it, why should they care themselves?

This is feedback, in case you are wondering.

All that said, let me add that I have no hard feelings toward the site. As a matter of fact, apart from this 'incident', I believe it's one of the most relaxing places where to talk about electronics. But the moderator screwed up, and you lost a contributor.

I am still leeching, tho.


Title: Re: Does Kirchhoff's Law Hold? Disagreeing with a Master
Post by: ogden on April 20, 2019, 04:43:43 am
But I did not see any explanation for my ban, except for the one I assumed.

You got your ban because you insulted people, including moderator. How about such explanation?

Quote
That is, puncturing the ego of someone who DID NOT QUALIFY AS A MODERATOR (to me that was some bloke cracking a joke, and I treated him as such), and made no specific requests as to what we should have done. Stop posting? Saying the counterpart was right? Take a bow?

Seems you got gigantic hole in your ego as well and it is still there.  :popcorn:
Title: Re: Does Kirchhoff's Law Hold? Disagreeing with a Master
Post by: bsfeechannel on April 20, 2019, 04:00:51 pm
I did your experiment. At the begin of the experiment the EMF must have been in the air gap between both secondary terminals.

Yep.

Quote
However, as soon as I connected the voltmeter, I got the feeling the EMF jumped into my voltmeter

Yep.
 
Quote
(as it is 10MOhm resistive.  As you say EMF is path dependent).

Under a varying magnetic field it is. Where the magnetic field doesn't vary, i.e. outside the transformer, it is not.

Quote
With the 10k resistor I was expecting it to sit now in the Voltmeter AND in the resistor at the same time - according to your thesis.

I do not have a thesis. Maxwell does. You need to get along with him more often.

Quote
And I always thought, the EMF is defined as the tangential force per unit charge in the wire integrated over length, once around the complete circuit/loop! But not at all, it was hiding in the resistor and my voltmeter!

You clearly thought wrong, I regret. EMFs will never "hide" inside (static) wires (considered as ideal, that is).

Quote
But you confused me a bit later, when talking about the [elp]. The [elp] being the EMF (I guess the Israeli Prof meant [Epsilon loop] with this abbreviation). You tell me it’s invisible and can’t be measured? Although you told me before it’s sitting in the resistor and how to measure it?

It's understandable that you are confused. Electromag is unintuitive. It requires diligent study. You didn't pay attention when I said that the EMF can't be measured as a SEPARATE ENTITY in this circuit.

Once you measure the voltage on the 10Mohm of your meter or the 10kohm load, you won't find this voltage anywhere else in the circuit. All that you'll find is a wire with (ideally) zero volts. This violates Kirchhoff's law which demands that a second voltage be found around the path of the wire to balance the one you found across the resistor.

Quote
On the other hand you say today’s convention is to put voltage drops AND EMFs on the left side. But insist at the same time to keep the -dFlux/dt on the right side? -dFlux/dt being the EMF.
Looks like you don’t consider -dFlux/dt as EMF, but rather the EMF as part of the line integral E ds?

Again you left out the phrase FOUND IN THE CIRCUIT. To the right are the EMFs FOUND IN THE CIRCUIT, i.e. directly measured. -dΦ/dt is an EMF that cannot be found in the circuit as a separate entity, so it must be on the right side of the equation.

But you are on the right track. You are at least questioning, instead of trying to disprove Faraday's law with pseudo-scientific assumptions, which is a good thing.

You'll understand electromagnetism sooner than those who thanked you.
Title: Re: Does Kirchhoff's Law Hold? Disagreeing with a Master
Post by: ogden on April 20, 2019, 04:15:01 pm
Once you measure the voltage on the 10Mohm of your meter or the 10kohm load, you won't find this voltage anywhere else in the circuit. All that you'll find is a wire with (ideally) zero volts.

Wow. How transformers with multi-tap secondary windings work then?

Quote
You'll understand electromagnetism sooner than those who thanked you.

 :-DD

Right. Be prepared to become enlightened. Those who thanked you are lost souls - nonbelievers. They do not believe that Dr.Lewin is always right even when he is not.
Title: Re: Does Kirchhoff's Law Hold? Disagreeing with a Master
Post by: bsfeechannel on April 20, 2019, 04:35:33 pm
All that said, let me add that I have no hard feelings toward the site. As a matter of fact, apart from this 'incident', I believe it's one of the most relaxing places where to talk about electronics.

Yay! Sredni is back!

Quote
But the moderator screwed up, and you lost a contributor.

Sredni, man, can't we leave this moderation imbroglio behind? After all, your access has been reinstated, you were allowed to rant on what happened, the thread is still open for us to discuss the subject and no one is bothering us.

Quote
I am still leeching, tho.

You're not punishing Dave, you are punishing us. Dave and Simon have more important things to worry about than this "electromagnetism rubbish". So leave them be.

If you have something to contribute to the discussion, please do. We are in desperate need of people who can help us destroy bullshit thinking in engineering.
Title: Re: Does Kirchhoff's Law Hold? Disagreeing with a Master
Post by: bsfeechannel on April 22, 2019, 05:27:43 am
This was exactly my point. It doesn't make sense to talk about a voltage source as purely being an impedance.

A voltage source is NOT an impedance, much less "purely". Read Feynman lectures volume II chapter 22.

Quote
When analyzing the circuit it makes more sense to think of it as an equivalent circuit. This separates the voltage source from the impedance part, allowing both to remain the same no matter the load.

Or is the use of equivalent models somehow forbidden in circuit analysis?

What determining the equivalent circuit of a battery has to do with measuring an impedance in an AC circuit under a varying magnetic field?

Quote
What you describe is called mutual inductance in circuit analysis. As soon as inductors start sharing magnetic fields they also start sharing there inductance too.

So what causes the change in their behavior is the _ _ _ _ _.

Quote
Here is a quick summary of how this works: https://physics.stackexchange.com/questions/119638/choosing-sign-for-kvl-mutual-inductance

So now how does the transformer secondary not have 90 degree phase shift if its an inductor? Because the voltage is all coming from mutual part of inductance and this inductance only cares about the current in the primary.

As you confirm, not only the secondary doesn't behave like an inductor (there's no 90 degree phase shift), but also it is not even an impedance: because the voltage on the secondary is a function of something external to itself, which is in fact the _ _ _ _ _ generated by the current in the primary.

Quote
The current trough the load resistor actually affects the voltage of the primary side.

How can this be, once the primary is connected to a voltage source?

Quote
Once all of these currents and voltages are summed up neither the primary or secondary have 90 degree phase shifts anymore. In an ideal 1:1 transformer with no leakage the currents and voltages are actually all perfectly in phase (If the load is a  resistor like in your example) or 180 out of phase depending on what way around you connect the coil.

So now not even the primary can be identified as an inductor anymore.

Quote
All my hair is still in its place as there is no need to pull it out over a concept that works just fine, or is the use of mutual inductance somehow forbidden?

Forbidden? Why? It perfectly demonstrates what I said before: if you don't understand the underlying physics of electromagnetism you'll be limited in your ability to design and analyze circuits.

Quote
So then back to the original question. Under what circumstances is a wire not an inductor and when it is?

If you read your own words again you'll see that you've already answered your question.
Title: Re: Does Kirchhoff's Law Hold? Disagreeing with a Master
Post by: bsfeechannel on April 22, 2019, 06:16:20 am
:-DD

Right. Be prepared to become enlightened.
Since you rejected my baby-stepped method for guiding you to enlightenment, I'll be forced to reveal the shocking truth to you. The conservation of energy holds for when Kirchhoff fails because the energy the resistor is dissipating comes from the fuh...., the fuh-fieh..., the fie... No, I can't. I can't put you through this emotional sacrifice. I have scruples.

Title: Re: Does Kirchhoff's Law Hold? Disagreeing with a Master
Post by: ogden on April 22, 2019, 06:29:06 am
Right. Be prepared to become enlightened.
Since you rejected my baby-stepped method for guiding you to enlightenment, I'll be forced to reveal the shocking truth to you. The conservation of energy holds for when Kirchhoff fails because the energy the resistor is dissipating comes from the fuh...., the fuh-fieh..., the fie... No, I can't. I can't put you through this emotional sacrifice. I have scruples.

Again you show that you can't even read  :palm:

I did not ask to tell where energy comes from. I ask you to show law of conservation of electrical energy using Maxwell's equations. Apparently you are afraid of what comes next after you do it.
Title: Re: Does Kirchhoff's Law Hold? Disagreeing with a Master
Post by: bsfeechannel on April 22, 2019, 07:42:17 am

Again you show that you can't even read  :palm:

I did not ask to tell where energy comes from. I ask you to show law of conservation of electrical energy using Maxwell's equations. Apparently you are afraid of what comes next after you do it.

Yes. I'm afraid you won't understand. Since you don't want to know where the energy comes from, it'll be impossible for you to understand it using Maxwell's equations, or whatever.
Title: Re: Does Kirchhoff's Law Hold? Disagreeing with a Master
Post by: ogden on April 22, 2019, 09:04:24 am
Since you don't want to know where the energy comes from, it'll be impossible for you to understand it using Maxwell's equations, or whatever.

Do not confuse public forum with private chat, tell for those who you think will understand.

To cut the crap: KVL is based on conservation of the charge which is based on law of conservation of energy. As soon as you write energy_generated_inthe_loop = energy_dissipated_inthe_resistor equation, KVL can be derived from it. Seems, you know it well and seems I know what abbreviation "BSFEE" means.
Title: Re: Does Kirchhoff's Law Hold? Disagreeing with a Master
Post by: Berni on April 22, 2019, 11:44:46 am
This was exactly my point. It doesn't make sense to talk about a voltage source as purely being an impedance.

A voltage source is NOT an impedance, much less "purely". Read Feynman lectures volume II chapter 22.
...
What determining the equivalent circuit of a battery has to do with measuring an impedance in an AC circuit under a varying magnetic field?
...
So what causes the change in their behavior is the _ _ _ _ _.
Yes exactly my point that a voltage source is not an impedance.

In that chapter 22 that you mention it clearly shows that induced voltage acts like a voltage source. So including the induced voltage when calculating impedance of a component is just as nonsense as directly calculating the impedance of a voltage source using the voltage across its terminals.

So putting it all together:
2 terminal components can't have multiple impedance at the same moment in time.
2 terminal components can't have multiple different currents running trough it at the same moment in time.
Since impedance relates current to voltage this also means the before mentioned impedance can only have one well defined voltage across it. But this also defines the path of the voltage as being trough this impedance, hence why it doesn't go against Maxwell. The induced voltage didn't disappear, but the component it self doesn't care about it. Its only the circuit around it that sees the voltage (Since the loop needs to be closed) as this voltage adds on to whatever voltage is presented on the terminals. Much like the equivalent model of a voltage source looks on its terminals.

is there any reason why you replaced the world field with underscores? Its not exactly a secret that magnetic fields are what makes inductors work.

...
As you confirm, not only the secondary doesn't behave like an inductor (there's no 90 degree phase shift), but also it is not even an impedance: because the voltage on the secondary is a function of something external to itself, which is in fact the _ _ _ _ _ generated by the current in the primary.
...
How can this be, once the primary is connected to a voltage source?
...
So now not even the primary can be identified as an inductor anymore.
...
Forbidden? Why? It perfectly demonstrates what I said before: if you don't understand the underlying physics of electromagnetism you'll be limited in your ability to design and analyze circuits.
...
If you read your own words again you'll see that you've already answered your question.

Exactly it doesn't behave like an inductor anymore, but instead behaves like a coupled inductor.

But it doesn't mean its inductance simply disappeared. The same magnetizing current is still needed to hold up the field, but because the two inductors are sharing the same flux means they can also share the magnetizing current. Since the resistor on the secondary can't provide a source of reactive current means that all the magnetizing current comes from the voltage source on the primary. The resistive load does however cause a in phase current that makes a opposing field in the secondary that affects the field in the primary, where it induces voltage according to Faradays law. Since there is a voltage source forcing a well defined voltage across the primary, this instead draws extra current to correct this field. This extra  in phase current is what drags the total current back away from lagging 90 degrees. As for the secondary, it has no need for out of phase current, because all of its magnetizing current is provided by the primary.

So the inductance didn't go anywhere. The effect of inductance is simply buried under the other currents.

So yeah, sorry i still don't see the exact circumstance when a piece of wire stops being an inductor. Or does becoming a coupled inductor not count as being inductive? If so please explain why.

At what point is my inductance explanation inconsistent with physics?
Title: Re: Does Kirchhoff's Law Hold? Disagreeing with a Master
Post by: bsfeechannel on April 23, 2019, 09:01:23 am
Today at 08:42:17 am » Insert Quote
You are ignoring this user.

It seems that you can't ignore me. Thank you. No wonder. I am managing to destroy mercilessly and systematically the myths you always held dear. But cheer up, dude. You'll be empowered by your new disbelief.

Quote
Do not confuse public forum with private chat, tell for those who you think will understand.

I've already done it (https://www.eevblog.com/forum/chat/does-kirchhoffs-law-hold-disagreeing-with-a-master/msg2348454/#msg2348454). But you rejected it unceremoniously.

Quote
To cut the crap: KVL is based on conservation of the charge which is based on law of conservation of energy.

Charge conservation and energy conservation are two separate principles. Both can be derived for electromagnetism from Maxwell's equations.

Let's start with charge conservation, which is easier for a circuit-head guy like you to understand. You can find it demonstrated here (https://en.wikipedia.org/wiki/Maxwell%27s_equations#Charge_conservation).

In the picture below you can see how KCL fails, but charge conservation holds.

(https://www.eevblog.com/forum/chat/does-kirchhoffs-law-hold-disagreeing-with-a-master/?action=dlattach;attach=714729)

The sum of currents going in and coming out of the volume represented by the closed dashed line to the left is different from zero, violating KCL. But conservation of charge holds because the excess charge is being stored in the body inside the volume. On the volume to the right KCL holds, because no charge is being stored in the volume, so all the currents must add up to zero.

Quote
As soon as you write energy_generated_inthe_loop = energy_dissipated_inthe_resistor equation, KVL can be derived from it.

Energy conservation in electromagnetism is a little bit more complicated (http://www.feynmanlectures.caltech.edu/II_27.html), and can be derived, as in the case of conservation of charge, from the modified (by Maxwell) Ampère's law, however you need to understand that the energy that's powering the resistor is not coming from a component in the circuit. While you don't accept this you'll be looking in vain around the wire for an EMF that will balance the voltage drop, and smashing your head against the wall.

And as in the case of charge conservation, we can also have KVL failing while energy conservation holds.

The difficulty Mehdi expressed in his first video comes exactly from not understanding this. For him, KVL can never fail, because that would represent a violation of energy conservation.

Nothing could be further than the truth.

EDIT: corrected the charge conservation equation in the attached picture (had forgot the minus sign).
Title: Re: Does Kirchhoff's Law Hold? Disagreeing with a Master
Post by: ogden on April 23, 2019, 10:15:50 am
Energy conservation in electromagnetism is a little bit more complicated (http://www.feynmanlectures.caltech.edu/II_27.html), and can be derived, as in the case of conservation of charge, from the modified (by Maxwell) Ampère's law, however you need to understand that the energy that's powering the resistor is not coming from a component in the circuit.

Energy is delivered by the component of the circuit - wire loop. Most sources of electrical energy are energy conversion devices - so what? Instead of talking about law of electrical energy conservation, you play games around meaning of the word "generator (https://www.merriam-webster.com/dictionary/generator)". I can rewrite equation that needs to be filled-in with your wizdom if you are so picky about words I use: energy_delivered_bythe_wire_loop = energy_dissipated_inthe_resistor.
Title: Re: Does Kirchhoff's Law Hold? Disagreeing with a Master
Post by: ogden on April 23, 2019, 01:13:02 pm
Let's start with charge conservation, which is easier for a circuit-head guy like you to understand. You can find it demonstrated here (https://en.wikipedia.org/wiki/Maxwell%27s_equations#Charge_conservation).

In the picture below you can see how KCL fails, but charge conservation holds.

(https://www.eevblog.com/forum/chat/does-kirchhoffs-law-hold-disagreeing-with-a-master/?action=dlattach;attach=713982;image)

The sum of currents going in and coming out of the volume represented by the closed dashed line to the left is different from zero, violating KCL. But conservation of charge holds because the excess charge is being stored in the body inside the volume.

It's just next fallacy of yours. Those two round objects on the left together form charged capacitor and wire between them - load where energy that is stored in the capacitor, dissipates. KCL holds.
Title: Re: Does Kirchhoff's Law Hold? Disagreeing with a Master
Post by: Berni on April 23, 2019, 03:24:52 pm
I dont see how charge is conserved inside the dotted circle. Curent is charge divided by time anyway.

But yeah i can see what the point is, an example of KVL not holding. We KNOW that KVL is not a law of physics. It holds in most cases but not all, it only always holds in lumped circuit meshes. Hence why it cant be directly slapped onto any physical circuit, much like the rest of circuit analysis math. Go ahed and try to find a quote of me claiming it always holds in physical circuits in any of this thread
Title: Re: Does Kirchhoff's Law Hold? Disagreeing with a Master
Post by: ogden on April 23, 2019, 03:46:50 pm
I dont see how charge is conserved inside the dotted circle. Curent is charge divided by time anyway.

He most likely did mean that charge is distributed evenly between two round objects. Yes, it is so - if there is separate point of reference (ground?) to measure voltages/charges of both round objects. In such case we have two capacitor paradox (https://en.wikipedia.org/wiki/Two_capacitor_paradox) which again agrees with KVL.
Title: Re: Does Kirchhoff's Law Hold? Disagreeing with a Master
Post by: bsfeechannel on April 24, 2019, 04:05:21 am
I dont see how charge is conserved inside the dotted circle.

Charge conservation is just a matter of bookkeeping (as a matter of fact all conservation principles are). The dollar you spend has to be discounted from your account balance. That sort of thing.

∇·J = - ∂ρ/∂t means that the charges you see going away for good have to be discounted from the charges you hoard inside your closed surface (represented by the dashed line).

Or a little more rigorously, the total amount of charge you lose per unit area, per unit time, to the exterior of a close volume (∇·J) is equal to the rate of change over time of a decrease (hence the minus sign) on the amount of charge per unit volume (- ∂ρ/∂t).

Quote
Curent is charge divided by time anyway.

Precisely. Current is the rate of charges crossing an area per unit time.

So choose a closed region of space. Count the amount of charge you have in that volume. Say 10 coulombs. If you measure a current of 1 A, i.e. 1 coulomb per second, going out of that volume (and you see no other current), in one second you'll have 9 coulombs, because 1 coulomb will be gone.

If you have this current, plus another with a 2 amp intensity going in the volume, you will end up with 11 coulombs after one second.

You can have a situation in which the amount coulombs stored in the volume must be kept constant. In that case all that comes in per unit time has to go out. In other words, all currents must add up to zero. And that's KCL.

Quote
But yeah i can see what the point is, an example of KVL not holding. We KNOW that KVL is not a law of physics.

Kirchhoff studied carefully the behavior of currents and voltages in circuits and published the results of his findings in the "Annals of Physics". Then he derived his theorems, as he called them, from those empirical data. His discovery was a major breakthrough.

So KVL and KCL ARE laws of physics. But a law of physics has not to work under whatever condition. As important as it is to understand KVL and KCL it is to know when they hold and when they fail.

Quote
It holds in most cases but not all,

I would say that KVL and KCL do NOT hold most of the times. Where can you find a place on earth where you don't have varying electromagnetic fields? The thing is that we PRETEND that KVL and KCL hold by approximation. We stash fields inside capacitors and inductors, we create ground planes, employ shielded conductors, we decouple lines, inductors, capacitances, all to avoid having to deal with fields. And when we have to deal with them, we employ rules of thumb and equivalent approximate models.

And what we cannot tame and make to conform to KVL/KCL we call "parasitics".

This creates the illusion that KVL and KCL hold "in most cases". But it's only an illusion. Not that we will abandon this illusion all of a sudden. We need to know what it means, and not to try to linger to it if it clearly shows that we will be limited in our ability to interpret the phenomena around us.
Title: Re: Does Kirchhoff's Law Hold? Disagreeing with a Master
Post by: Berni on April 24, 2019, 05:53:49 am
Quote
But yeah i can see what the point is, an example of KVL not holding. We KNOW that KVL is not a law of physics.

Kirchhoff studied carefully the behavior of currents and voltages in circuits and published the results of his findings in the "Annals of Physics". Then he derived his theorems, as he called them, from those empirical data. His discovery was a major breakthrough.

So KVL and KCL ARE laws of physics. But a law of physics has not to work under whatever condition. As important as it is to understand KVL and KCL it is to know when they hold and when they fail.

Quote
It holds in most cases but not all,

I would say that KVL and KCL do NOT hold most of the times. Where can you find a place on earth where you don't have varying electromagnetic fields? The thing is that we PRETEND that KVL and KCL hold by approximation. We stash fields inside capacitors and inductors, we create ground planes, employ shielded conductors, we decouple lines, inductors, capacitances, all to avoid having to deal with fields. And when we have to deal with them, we employ rules of thumb and equivalent approximate models.

And what we cannot tame and make to conform to KVL/KCL we call "parasitics".

This creates the illusion that KVL and KCL hold "in most cases". But it's only an illusion. Not that we will abandon this illusion all of a sudden. We need to know what it means, and not to try to linger to it if it clearly shows that we will be limited in our ability to interpret the phenomena around us.

Yes for a time Kirchhoffs circuit laws ware the best explanation of electricity we had, but this was back in the mid 1800s. At that time we had no idea what an atom is made out of, we didn't even know electrons existed, radio waves have yet to be used to transmit information, mathematicians ware still discovering methods for working with complex numbers and some parts of calculus ware still being figured out.

Just like our understanding of what matter is made up of went from alchemy to Mendeleevs periodic table to atoms, to explaining atoms are made of some stuff in the middle, to figuring out its actually protons and neutrons in there, saying that are fundamental particles, but then later on managing to split them apart and finding quarks inside of them.

In the same way Maxwell did the next big step by explaining how electricity works in better more accurate detail that Kirchhoff could. At that point Kirchhoffs laws ware no longer considered laws of physics, its even in the name, they are called "Kirchoffs Cirucit Laws" since its circuits they apply to, not the theory of electricity itself, Maxwell has that now. But it didn't stop there, science kept on going and upon closer inspection found out that actually Maxwells explanation is still not quite the whole story, so the theory of Quantum electrodynamics was put together and it now carries the crown for the best theory of electricity (To this date that is, who knows for how long, there might be a even more fundamental one we haven't figured out yet).

So why do we still use Maxwells equations if they are wrong according to Quantum electrodynamics?  Its much like the reason why we still use Kirchhoffs circuit laws despite being wrong according to Maxwell. On a macroscopic level and under certain conditions these laws still work just fine, while being much more convenient to work with when you want to actually calculate them with actual numbers. The 3 sets of laws are basically just different abstraction layers for electricity. Just chose the desired abstraction level and be aware of its known limitations.

Tho to be honest there is very very little reason to go any deeper than Maxwells level of abstraction for engineering use.
Title: Re: Does Kirchhoff's Law Hold? Disagreeing with a Master
Post by: ogden on April 24, 2019, 05:59:34 am
As important as it is to understand KVL and KCL it is to know when they hold and when they fail.

Sure. Even more important is to understand when you do not apply KVL and KCL (Kirchoff's Circuit Law). You can't draw construct which is not even circuit - that has no return path for current, try to convince that current is flowing into nowhere, then happily conclude that KCL does not hold. It's not even laughable.

Quote
I would say that KVL and KCL do NOT hold most of the times. Where can you find a place on earth where you don't have varying electromagnetic fields? The thing is that we PRETEND that KVL and KCL hold by approximation. We stash fields inside capacitors and inductors, we create ground planes, employ shielded conductors, we decouple lines, inductors, capacitances, all to avoid having to deal with fields. And when we have to deal with them, we employ rules of thumb and equivalent approximate models.

And what we cannot tame and make to conform to KVL/KCL we call "parasitics".

Wow. With such rhetoric one can invalidate nearly every law of physics.

P.S. Nice video regarding electromagnetics (https://youtu.be/sENgdSF8ppA?t=1)
Title: Re: Does Kirchhoff's Law Hold? Disagreeing with a Master
Post by: SparkyFX on April 24, 2019, 09:56:01 am
As I said before, scientific populism is going to be a big problem a few yeas ahead.
Opinion based decisions are surely a problem (and i am guilty of making them myself), but anyway, science teaches to check your results, so "scientific populism" is an oxymoron - "pseudoscience" would be the word. Science also teaches that the observation of a process might interfere with the result (and therefore needs taken care of).

One usually does not need to understand astrophysics or biology (or have an opinion about them) to pick a fruit from a tree and eat it -> results count, the fruit exists, the process therefore works. It starts to matter once you want to know why and how the fruit grew, manipulate the process or want to make predictions about it.

I think it is a bit on the troll side to find two different perspectives on the same subject and try to push a wedge between the two, although both are able to produce the same result -> describe the same process. I reckon it is a good conversation starter and helped students to understand the differences between abstractions, simplifications (wiring diagrams do usually not contain dimensions, equivalent circuits) and physical reality, but if you start thinking in "camps" or "sides", insult others, the cause is somehow lost.

Coming from a practical approach you measure voltages, not fields - and field sensors will usually output an equivalent voltage. So the proof seems complicated without KVL. Trying to measure fields using the force exerted on an object takes the object (and eddy currents in it) into the equations, making it as complicated to calculate as a wiring diagram with all equivalent components (parasitic inductance, capacitance) in place. So both suffer from the same problem, even if one is proven with "all fields are considered" and the other is disproven using "observation influenced the result".
Title: Re: Does Kirchhoff's Law Hold? Disagreeing with a Master
Post by: bsfeechannel on April 24, 2019, 09:43:10 pm
It's just next fallacy of yours. Those two round objects on the left together form charged capacitor and wire between them - load where energy that is stored in the capacitor, dissipates. KCL holds.

Let's examine KCL in the words of Kirchhoff himself (https://gallica.bnf.fr/ark:/12148/bpt6k151490/f525.item):

Quote
Wird ein System von Dräten, die auf eine ganz beliebige Weise mit einander verbunden sind, von galvanischen Strömen durchflossen, so ist:

1) wenn die Drähte 1, 2, ...µ in einem Punkte zusammenstoßen,

I1 + I2 + ...+Iµ = 0,

wo, I1, I2, ... die Intensitäten der Ströme bezeichnen, die jene Drähte durchfließen, alle nach dem Berührungspunkte zu als positiv gerechnet;

Translation

Let a system of wires, which are connnected with each other in an entirely arbitrary way, be traversed by galvanic currents [i.e. DC], then:

1) if the wires 1, 2, ...µ meet at one point,

                                I1 + I2 + ...+Iµ = 0,

where, I1, I2, ... designate the intensities of the currents, which flow through each wire, all calculated as positive in the direction of the point of contact;

We have just one wire, with just one current. This problem is analogous to the KVL problem: we have just one voltage, and no other to balance it that can be directly measured in the circuit.
Title: Re: Does Kirchhoff's Law Hold? Disagreeing with a Master
Post by: bsfeechannel on April 24, 2019, 09:51:39 pm
So why do we still use Maxwells equations if they are wrong according to Quantum electrodynamics?  Its much like the reason why we still use Kirchhoffs circuit laws despite being wrong according to Maxwell.

Kirchhoff's laws are not wrong according to Maxwell. Maxwell's equations do not only confirm that KVL and KCL are right, but also state in WHAT CONDITION they are right.

The source of the errors is elsewhere.

Quote
On a macroscopic level and under certain conditions these laws still work just fine, while being much more convenient to work with when you want to actually calculate them with actual numbers. The 3 sets of laws are basically just different abstraction layers for electricity. Just chose the desired abstraction level and be aware of its known limitations.

A cat is a special case of mammal, and the description of a mammal is an abstraction of a cat (and every other mammal, as a matter of fact).

QED is an abstraction of Maxwell's equations, as Maxwell's equations are abstractions of KVL/KCL.

KVL/KCL are a special case of Maxwell's equations, as Maxwell's equations are special cases of QED.

KVL/KCL are NOT abstractions of Maxwell's equations by the simple fact that they don't work for all the cases for which Maxwell's equations do.

This is another illusion. Because we ram KVL/KCL down the throats of our circuits and close our eyes to the errors due to this approximation (and we manage to get away with it in many cases) we have the impression that KVL/KCL are just simplified (hence abstracted) cases of Maxwell's equations. But this mindset will bite you in the butt sooner or later. You need to be always aware of the limitations of KCL/KVL and, if the error due to approximating Maxwell to Kirchhoff is gross enough, resort to what will give you the most accurate prediction.

Quote
Tho to be honest there is very very little reason to go any deeper than Maxwells level of abstraction for engineering use.

I think the inventors of the transistor would not agree with you, but anyway...
Title: Re: Does Kirchhoff's Law Hold? Disagreeing with a Master
Post by: bsfeechannel on April 24, 2019, 10:02:51 pm
I dont see how charge is conserved inside the dotted circle. Curent is charge divided by time anyway.

He most likely did mean that charge is distributed evenly between two round objects. Yes, it is so - if there is separate point of reference (ground?) to measure voltages/charges of both round objects. In such case we have two capacitor paradox (https://en.wikipedia.org/wiki/Two_capacitor_paradox) which again agrees with KVL.

Nope. The two capacitor paradox is about energy conservation not charge conservation. In the paradox, you have two wires. Here we have just one. In the paradox the wires are ideal. Here the wire can have resistance, inductance, and nonzero length.
Title: Re: Does Kirchhoff's Law Hold? Disagreeing with a Master
Post by: ogden on April 24, 2019, 10:31:16 pm
We have just one wire, with just one current.

In case you insist that there is just one wire and just one current meaning no path for return current - then I say that you don't even have circuit, thus Kirchoff's Circuit Law do not apply. There is huge difference between "do not apply" and "do not hold".

Quote
This problem is analogous to the KVL problem: we have just one voltage, and no other to balance it that can be directly measured in the circuit.

You just mentioned voltage. Very nice. Charge was mentioned before - so those two round objects together indeed is capacitor, C=q/V. Here's your circuit - capacitor with load in form of wire. I already said so you just do not listen:

It's just next fallacy of yours. Those two round objects on the left together form charged capacitor and wire between them - load where energy that is stored in the capacitor, dissipates. KCL holds.
Title: Re: Does Kirchhoff's Law Hold? Disagreeing with a Master
Post by: bsfeechannel on April 25, 2019, 12:55:49 am
In case you insist that there is just one wire and just one current meaning no path for return current - then I say that you don't even have circuit, thus Kirchoff's Circuit Law do not apply. There is huge difference between "do not apply" and "do not hold".

Nope. Kirchhoff says clearly that the wires can be connected in an entirely arbitrary way. So there is no requirement for them to form a circuit or to provide a path for a return current.

Quote
so those two round objects together indeed is capacitor, C=q/V.

Not inside the dashed line. There is only one wire providing charges to (or removing from) that area.
Title: Re: Does Kirchhoff's Law Hold? Disagreeing with a Master
Post by: seagreh on April 25, 2019, 02:01:25 am

Quote
And I always thought, the EMF is defined as the tangential force per unit charge in the wire integrated over length, once around the complete circuit/loop! But not at all, it was hiding in the resistor and my voltmeter!

You clearly thought wrong, I regret. EMFs will never "hide" inside (static) wires (considered as ideal, that is).
Sorry, my wording was wrong, I should have said ‘I always believed, the EMF...’.!
Because the clearly wrong ‘thought’ is from Feynman Volume II chapter 16 Induced Currents.
As you say, electromag is so unintuitive, so much bullshit around.... you can’t even trust books anymore.
Title: Re: Does Kirchhoff's Law Hold? Disagreeing with a Master
Post by: seagreh on April 25, 2019, 02:23:58 am

Let's examine KCL in the words of Kirchhoff himself (https://gallica.bnf.fr/ark:/12148/bpt6k151490/f525.item):

Quote
Wird ein System von Dräten, die auf eine ganz beliebige Weise mit einander verbunden sind, von galvanischen Strömen durchflossen, so ist:

1) wenn die Drähte 1, 2, ...µ in einem Punkte zusammenstoßen,

I1 + I2 + ...+Iµ = 0,

wo, I1, I2, ... die Intensitäten der Ströme bezeichnen, die jene Drähte durchfließen, alle nach dem Berührungspunkte zu als positiv gerechnet;

Translation

Let a system of wires, which are connnected with each other in an entirely arbitrary way, be traversed by galvanic currents [i.e. DC], then:

1) if the wires 1, 2, ...µ meet at one point,

                                I1 + I2 + ...+Iµ = 0,

where, I1, I2, ... designate the intensities of the currents, which flow through each wire, all calculated as positive in the direction of the point of contact;

We have just one wire, with just one current. This problem is analogous to the KVL problem: we have just one voltage, and no other to balance it that can be directly measured in the circuit.
This phenomena is well known in literature as the ‘One-Wire-Single Current-Sophism”.
For sure, can only be solved with Maxwell equations   :clap:
Maybe you missed the point, the ‘connection point’ (Berührungspunkt)?
Hence, a single wire hitting the connection point is a dead end, hence I1 = 0 .
Title: Re: Does Kirchhoff's Law Hold? Disagreeing with a Master
Post by: bsfeechannel on April 25, 2019, 03:29:25 am

Quote
And I always thought, the EMF is defined as the tangential force per unit charge in the wire integrated over length, once around the complete circuit/loop! But not at all, it was hiding in the resistor and my voltmeter!

You clearly thought wrong, I regret. EMFs will never "hide" inside (static) wires (considered as ideal, that is).
Sorry, my wording was wrong, I should have said ‘I always believed, the EMF...’.!
Because the clearly wrong ‘thought’ is from Feynman Volume II chapter 16 Induced Currents.
As you say, electromag is so unintuitive, so much bullshit around.... you can’t even trust books anymore.

Thank you for bringing that up. This is the source of much confusion.

In chapter 16, Feynman is talking about REAL wires, that admit the presence of electric fields inside them. Lewin does the same in his famous Lecture 16 and also generates the same kind of confusion. In the middle of the lecture he talks about EMFs in the path of the wires.

Real wires behave just like a resistor. In fact they ARE resistors.

At the end of his lecture, Lewin uses real wires, but his setup allows the approximation to ideal wires because the resistance of the wires are much less than the resistors he uses (100 and 900 ohms). Since he doesn't make that explicitly clear, there are people up to this day looking for the missing EMF in the wires.

In chapter 22, Feynman treat the wires as ideal wires, that he calls "perfect conductor". The electric field inside those wires are ideally (duh!) zero. And that's the kind of wires we've been using in this discussion, unless noted differently.

So, to make it clear, I'll repeat: EMFs will never "hide" inside (static) wires (considered as ideal, that is).
Title: Re: Does Kirchhoff's Law Hold? Disagreeing with a Master
Post by: bsfeechannel on April 25, 2019, 03:45:56 am
This phenomena is well known in literature as the ‘One-Wire-Single Current-Sophism”.

Sophism or not there is only one wire there. That's all that counts for KCL.

Quote
For sure, can only be solved with Maxwell equations   :clap:

And I did it in from of your eyes: ∇·J = - ∂ρ/∂t

Quote
Maybe you missed the point, the ‘connection point’ (Berührungspunkt)?
Hence, a single wire hitting the connection point is a dead end, hence I1 = 0 .

Nope. If I is zero, then charge conservation is violated. The volume inside the dashed line is having a variation in the amount of charge, and there's no current crossing the border to account for this variation. It is like a bank account having its balance increasing or decreasing without money being deposited or withdrawn.
Title: Re: Does Kirchhoff's Law Hold? Disagreeing with a Master
Post by: bsfeechannel on April 25, 2019, 05:02:19 am
Yes exactly my point that a voltage source is not an impedance.

In that chapter 22 that you mention it clearly shows that induced voltage acts like a voltage source. So including the induced voltage when calculating impedance of a component is just as nonsense as directly calculating the impedance of a voltage source using the voltage across its terminals.

If the voltage source is NOT an impedance there's no sense in talking about direct or indirect calculation of its "impedance" :o or the inclusion or not of said "induced voltage". It is not an impedance, period.

Quote
2 terminal components can't have multiple impedance at the same moment in time.

For that to happen you have to comply to at least the following conditions: you can't have external varying fields; the field generated by the component has to be confined and away from its terminals.

Quote
is there any reason why you replaced the world field with underscores?

Because I wanted to check if you really understand that the answers to explain how circuits work are not always found in the path of circuits themselves.

Quote
Its not exactly a secret that magnetic fields are what makes inductors work.

Well, fields are what makes everything work in electronics, even when you are not thinking about them.

Quote
Exactly it doesn't behave like an inductor anymore, but instead behaves like a coupled inductor.

Coupled inductors cannot be considered 2 terminal components anymore.

Quote
But it doesn't mean its inductance simply disappeared. The same magnetizing current is still needed to hold up the field, but because the two inductors are sharing the same flux means they can also share the magnetizing current. Since the resistor on the secondary can't provide a source of reactive current means that all the magnetizing current comes from the voltage source on the primary. The resistive load does however cause a in phase current that makes a opposing field in the secondary that affects the field in the primary, where it induces voltage according to Faradays law. Since there is a voltage source forcing a well defined voltage across the primary, this instead draws extra current to correct this field. This extra  in phase current is what drags the total current back away from lagging 90 degrees. As for the secondary, it has no need for out of phase current, because all of its magnetizing current is provided by the primary.

So the inductance didn't go anywhere. The effect of inductance is simply buried under the other currents.

The secondary is a generator, not an inductor. Although what is going on between the wires is the phenomenon of induction, for the circuit connected to the terminals of those wires they are not. Always remember that lumped components are only seen at their terminals. So if a piece of wire looks like a generator and smells like a generator at its terminals it is because it is a generator. The primary acts as an impedance that can change depending on the load connected to the secondary.

Quote
So yeah, sorry i still don't see the exact circumstance when a piece of wire stops being an inductor. Or does becoming a coupled inductor not count as being inductive? If so please explain why.

I think it is clear to you now that a piece of wire can be many things, an inductor, a generator, or whatever impedance depending on whether or not it's under a varying magnetic field.

If you consider it always an inductor, this will lead you to try to write things like L(di/dt) - R*I = 0, when you should have written R*I = -d/dt (∬ΣB·dS).
Title: Re: Does Kirchhoff's Law Hold? Disagreeing with a Master
Post by: Berni on April 25, 2019, 06:02:46 am
So why do we still use Maxwells equations if they are wrong according to Quantum electrodynamics?  Its much like the reason why we still use Kirchhoffs circuit laws despite being wrong according to Maxwell.

Kirchhoff's laws are not wrong according to Maxwell. Maxwell's equations do not only confirm that KVL and KCL are right, but also state in WHAT CONDITION they are right.

The source of the errors is elsewhere.


Yes the source of the error is people like Dr. Lewin applying it directly to a circuit without consideration if its applicable in those CONDITIONS. Fix those conditions by properly modeling the thing as a circuit mesh, if not then forget about Kirchhoffs circuit laws and stick to Maxwell.


Quote
On a macroscopic level and under certain conditions these laws still work just fine, while being much more convenient to work with when you want to actually calculate them with actual numbers. The 3 sets of laws are basically just different abstraction layers for electricity. Just chose the desired abstraction level and be aware of its known limitations.

A cat is a special case of mammal, and the description of a mammal is an abstraction of a cat (and every other mammal, as a matter of fact).

QED is an abstraction of Maxwell's equations, as Maxwell's equations are abstractions of KVL/KCL.

KVL/KCL are a special case of Maxwell's equations, as Maxwell's equations are special cases of QED.

KVL/KCL are NOT abstractions of Maxwell's equations by the simple fact that they don't work for all the cases for which Maxwell's equations do.

This is another illusion. Because we ram KVL/KCL down the throats of our circuits and close our eyes to the errors due to this approximation (and we manage to get away with it in many cases) we have the impression that KVL/KCL are just simplified (hence abstracted) cases of Maxwell's equations. But this mindset will bite you in the butt sooner or later. You need to be always aware of the limitations of KCL/KVL and, if the error due to approximating Maxwell to Kirchhoff is gross enough, resort to what will give you the most accurate prediction.


Okay yes i was using more the software engineering definition of abstraction layers rather than the mathematics definition of abstraction. With the mathematics definition it is indeed the other way around.

And i fully agree, all of the levels of explaining electricity work fine as long as you use them within those limitations. Hence why i never really had any problems applying KVL to Dr. Lewins experimental circuit from his lecture. Any paradox between KVL and Maxwell with that circuit is simply down to using them wrong.

Quote
Tho to be honest there is very very little reason to go any deeper than Maxwells level of abstraction for engineering use.
I think the inventors of the transistor would not agree with you, but anyway...

I'm pretty sure QED had nothing to do with the development of the first transistors.

There are patents for field effect transistors from before QED was a thing, tho nobody tried to make them at that point. Then the first working BJT device popped up by accident from a bunch of people trying to build a better solid state RF mixer.

The usual transistors can still be explained rather well with Maxwells fields pushing charged particles around. Mostly just a case of controlling where the charge carriers are rather than making use of any quantum mechanical effect. Tho the worlds finest lithography processes used for digital chips are now getting to making transistors small enough for quantum effects to start becoming a problem.

I'm pretty sure very few forum members here work in a semiconductor fab, let alone one that works with such fine feature capabilities or work on building quantum computers. Hence why very very few engineers would have a good reason to dig deeper than Maxwell, heck for 95% of cases even Kirchhoff is close enough(As long as you know about the other 5%).

Title: Re: Does Kirchhoff's Law Hold? Disagreeing with a Master
Post by: ogden on April 25, 2019, 07:32:54 am
In case you insist that there is just one wire and just one current meaning no path for return current - then I say that you don't even have circuit, thus Kirchoff's Circuit Law do not apply. There is huge difference between "do not apply" and "do not hold".

Nope. Kirchhoff says clearly that the wires can be connected in an entirely arbitrary way. So there is no requirement for them to form a circuit or to provide a path for a return current.

Exactly. Kirchhoff says "wires", thus more than single wire you desperately insist on. When you have single wire - you do not have conditions to apply KCL. That's why I looked for return current which I BTW found in the "spatially distributed capacitor", yet you managed to shift goalposts and change your mantra into talk about dotted area, not whole circuit. It is like showing one body of mass to happily conclude that Newton's law of universal gravitation do not work. It's not even unscientific. It's utterly stupid.
Title: Re: Does Kirchhoff's Law Hold? Disagreeing with a Master
Post by: bsfeechannel on April 25, 2019, 03:26:27 pm
Yes the source of the error is people like Dr. Lewin applying it directly to a circuit without consideration if its applicable in those CONDITIONS.

Nope. Lewin was absolutely impeccable in the "application" of the laws. He used Kirchhoff to show that it works when you do not have varying magnetic fields.

He substituted the varying magnetic field for the battery and then showed how wrong it is to apply Kirchhoff to a circuit with varying magnetic fields. You will measure different voltages, because they are now path-dependent.

And left the problem for his students to solve. The solution is Maxwell and only Maxwell.

Quote
Fix those conditions by properly modeling the thing as a circuit mesh,

We've already proved in this thread that Lewin's circuit can't be modeled by lumped components.

Quote
if not then forget about Kirchhoffs circuit laws and stick to Maxwell.

The problem with your reasoning is that you think that if things are connected like a circuit, then Kirchhoff MUST hold. You think that if you find a single circuit where it can't be employed to explain its behavior then the whole theory is cactus.

This is BULLSHIT.

There are circuits for which Kirchhoff holds and others for which it doesn't. By now you should have known the difference. And the difference is that if the circuit is drenched with a varying magnetic field, KVL is out. If not, then you can merrily use your KVL and your precious LTSPICE to model it.

Lewin's circuit, if you have not paid the due attention to it yet, is just a sophisticated version of Faraday's original demonstration of induction. Lewin is just reenacting the same experiment with two fancy oscilloscopes and two resistors of relative high value. But the experiment is essentially the same.

Saying Lewin is wrong is the same as denying Faraday's law.

Quote
Okay yes i was using more the software engineering definition of abstraction layers rather than the mathematics definition of abstraction. With the mathematics definition it is indeed the other way around.

Abstraction in software is commonly thought of from the point of view of the user, but it conforms with the general definition of abstraction in which the "abstracted" software is in fact more inclusive than the special case.

Quote
And i fully agree, all of the levels of explaining electricity work fine as long as you use them within those limitations. Hence why i never really had any problems applying KVL to Dr. Lewins experimental circuit from his lecture. Any paradox between KVL and Maxwell with that circuit is simply down to using them wrong.

We've already showed that your "modelling" just don't apply to Lewin's circuit. It introduces gross errors. If I connect the lead of MY voltmeter (not yours with 250mV in series with the probes) from one resistor to the other where a wire should be in your "model" I will measure 0.5V, whereas in Lewin's circuit this voltage is zero, and everyone measured exactly zero volts.

There can't be any paradox between KVL and Maxwell. Lewin changed the condition for the validity of KVL and invited his students to explain the new behavior of that circuit. Those that think that Kirchhoff applies to just about any circuit are trying to find each one a different explanation. While those who understand Faraday's law could explain what is going on immediately.

Quote
I'm pretty sure QED had nothing to do with the development of the first transistors.
Quote
Then the first working BJT device popped up by ]accident from a bunch of people trying to build a better solid state RF mixer.

Wow! Shockley, Bardeen and Bratain won the Nobel Prize by accident! Lucky bastards!

Quote
The usual transistors can still be explained rather well with Maxwells fields pushing charged particles around.

Isn't this what essentially QED is: electrodynamics applied to subatomic particles like the electron?

Quote
Mostly just a case of controlling where the charge carriers are rather than making use of any quantum mechanical effect.

Strange. I had the impression that electrons received and emitted photons when they change their quantic levels of energy in the electronic band structure.

Quote
I'm pretty sure very few forum members here work in a semiconductor fab, let alone one that works with such fine feature capabilities or work on building quantum computers. Hence why very very few engineers would have a good reason to dig deeper than Maxwell, heck for 95% of cases even Kirchhoff is close enough(As long as you know about the other 5%).

I also had the impression that every electronics engineer around the world learns at least some rudimentary concepts of QED as part of their regular course which is required to understand solid state electronics. But I may be wrong.
Title: Re: Does Kirchhoff's Law Hold? Disagreeing with a Master
Post by: bsfeechannel on April 25, 2019, 03:53:51 pm

Exactly. Kirchhoff says "wires", thus more than single wire you desperately insist on. When you have single wire - you do not have conditions to apply KCL.

Alright. I created an ogden version of KCL fail for you. I hope it now can make its way into your everything-must be-a-circuit head.

(https://www.eevblog.com/forum/chat/does-kirchhoffs-law-hold-disagreeing-with-a-master/?action=dlattach;attach=716439;image)

Quote
That's why I looked for return current which I BTW found in the "spatially distributed capacitor",

Cool! Can you connect an ammeter and measure this current flowing through a... wire?
Title: Re: Does Kirchhoff's Law Hold? Disagreeing with a Master
Post by: ogden on April 25, 2019, 04:56:19 pm
Alright. I created an ogden version of KCL fail for you.

Brilliant. Exactly what I was looking for. Could you sign it and put high resolution file on google drive? - So I can show what stupid ingenuity looks like.
Title: Re: Does Kirchhoff's Law Hold? Disagreeing with a Master
Post by: Berni on April 25, 2019, 05:48:22 pm
Quote
if not then forget about Kirchhoffs circuit laws and stick to Maxwell.
The problem with your reasoning is that you think that if things are connected like a circuit, then Kirchhoff MUST hold. You think that if you find a single circuit where it can't be employed to explain its behavior then the whole theory is cactus.
This is BULLSHIT.
There are circuits for which Kirchhoff holds and others for which it doesn't. By now you should have known the difference. And the difference is that if the circuit is drenched with a varying magnetic field, KVL is out. If not, then you can merrily use your KVL and your precious LTSPICE to model it.
Lewin's circuit, if you have not paid the due attention to it yet, is just a sophisticated version of Faraday's original demonstration of induction. Lewin is just reenacting the same experiment with two fancy oscilloscopes and two resistors of relative high value. But the experiment is essentially the same.
Saying Lewin is wrong is the same as denying Faraday's law.

No i said that KVL holds in every circuit mesh model. Not every physical circuit, this is an important distinction so don't carelessly throw them in the same bag.

I'm not saying Lewin is wrong. Kirchoffs circuit laws indeed don't work in such a use case. All i did was show a method of using KVL in a way that does work in that circuit, useful in cases where you would want to apply other circuit analysis tools.


Quote
And i fully agree, all of the levels of explaining electricity work fine as long as you use them within those limitations. Hence why i never really had any problems applying KVL to Dr. Lewins experimental circuit from his lecture. Any paradox between KVL and Maxwell with that circuit is simply down to using them wrong.
We've already showed that your "modelling" just don't apply to Lewin's circuit. It introduces gross errors. If I connect the lead of MY voltmeter (not yours with 250mV in series with the probes) from one resistor to the other where a wire should be in your "model" I will measure 0.5V, whereas in Lewin's circuit this voltage is zero, and everyone measured exactly zero volts.
There can't be any paradox between KVL and Maxwell. Lewin changed the condition for the validity of KVL and invited his students to explain the new behavior of that circuit. Those that think that Kirchhoff applies to just about any circuit are trying to find each one a different explanation. While those who understand Faraday's law could explain what is going on immediately.

I did say there is no paradox in my own post so why are you arguing for the same thing.

Oh and if you do have a voltmeter that can integrate the work needed to move an electron along any chosen path id love to see a write up on its operation. Would give you a pretty good chance at a Nobel prize even.

If you are so good at it how about showing me how to correctly analyze this simple circuit:
https://www.eevblog.com/forum/chat/does-kirchhoffs-law-hold-disagreeing-with-a-master/msg2189216/#msg2189216 (https://www.eevblog.com/forum/chat/does-kirchhoffs-law-hold-disagreeing-with-a-master/msg2189216/#msg2189216)
(https://www.eevblog.com/forum/chat/does-kirchhoffs-law-hold-disagreeing-with-a-master/?action=dlattach;attach=647828;image)

Feel free to use any method you want, as long as it shows what the circuit will do once given power.


...
Wow! Shockley, Bardeen and Bratain won the Nobel Prize by accident! Lucky bastards!
...
Isn't this what essentially QED is: electrodynamics applied to subatomic particles like the electron?
...
Strange. I had the impression that electrons received and emitted photons when they change their quantic levels of energy in the electronic band structure.
...
I also had the impression that every electronics engineer around the world learns at least some rudimentary concepts of QED as part of their regular course which is required to understand solid state electronics. But I may be wrong.

The inventors of the transistor certainly did amazing work in the field of semiconductors, but just saying that making a transistor was not what they ware trying to do when they made one. Its not the only major discovery that had a little bit of luck in it.

I was certainly ever shown any QED in lectures about semiconductors, tho to be honest those ware pretty dull lectures so i mostly just memorized enough stuff to pass the test, rather than show much interest. Its all mostly just electrostatic fields with a bit of electron physics thrown in. Nice to know about, but not terribly useful to know in deep detail. Much like teaching software engineers some assembler, not really practical for the majority of cases but good to know the basics.

Those electrons do certainly interact trough photons according to QED, but the overall behavior is more sensible to explain with Maxwell since it still works just fine in there.
Title: Re: Does Kirchhoff's Law Hold? Disagreeing with a Master
Post by: ogden on June 19, 2020, 07:57:35 pm
Latest news

(https://www.eevblog.com/forum/chat/does-kirchhoffs-law-hold-disagreeing-with-a-master/?action=dlattach;attach=1004902;image)